You are on page 1of 586

886607 800091 5

One novel feature of this course reader is that, as all the


content is licensed under creative commons copyright,
it will continuously evolve. The current edition is the
1st, published February 17, 2013.
Students wishing to contribute rewritten Case in Point
boxes may send them to matthew.holian@sjsu.edu.
Cases that demonstrate economic models and that
provide new statistics may be included in future editions
of this reader.
This is a course reader assembled by Professor Matthew J. Holian. It contains
sections on Microeconomics and Macroeconomics, from Libby Rittenberg and
Timothy Tregarthens Principles of Economics, and Statistics, from Barbara
Illowsky and Susan Deans Collaborative Statistics. Both of these were
published under creative commons copyright licenses.
By carefully reading the selections, and by diligently working through the
exercises, a motivated student can use this reader to develop a firm
understanding of economic principles, and to gain an understanding of how
economists use statistical techniques to test hypotheses and generate new
knowledge.
The reader is divided into three sections. Section I contains chapters from
economics and statistics; Chapter 2 from Principles of Economics introduces
two key models: a theoretical model illustrating comparative advantage, and
an empirical model estimated using ordinary least squares (OLS) regression.
Thus Section I contains examples of the types of modelstheoretical and
empiricalthat economists use. Following this introduction to economics, we
cover chapters 1-2 and 12 of Collaborative Statistics, in order to give students
the minimum statistical skills they will need to apply their knowledge.
Section II of reader covers core microeconomic concepts and theory,
including demand and supply, elasticity, welfare measures, the neoclassical
theory of the firm, and the following market structures: perfect competition,
monopoly and oligopoly.
Section III of the reader covers core macroeconomic concepts and theory,
including interest rates, measures of economic activity, financial institutions,
the aggregate demand / aggregate supply model, money, growth, and bond
and currency markets.
Assembled by Prof. Matthew J. Holian
An Introduction to Economics with
Statistics: A Course Reader
A
s
s
e
m
b
l e
d
b
y
P
r
o
f
.
M
a
t
t
h
e
w
J
.
H
o
l i a
n
A
n
I
n
t
r
o
d
u
c
t
i o
n
t
o
E
c
o
n
o
m
i c
s
w
i t
h
S
t
a
t
i s
t
i c
s
:
A
C
o
u
r
s
e
R
e
a
d
e
r
An Introduction to Economics with Statistics
Course Reader
February 17, 2013

This is a course reader assembled by Professor Matthew J. Holian. It contains sections on
Microeconomics and Macroeconomics, from Rittenberg and Tregarthens Principles of Economics,
and Statistics, from Illowsky and Deans Collaborative Statistics.
By carefully reading these sections, and by diligently working through the exercises given below, a
motivated student can develop a firm understanding of economic principles, and also gain a good sense
of how economists use statistical techniques to test hypotheses and generate new knowledge.
The course starts off in Section I with the first two chapters of Principles of Economics. Chapter 2
introduces two key models: a theoretical model illustrating comparative advantage, and an empirical
model estimated using ordinary least squares (OLS) regression. Thus already in Chapter 2 we are given
examples of the types of modelstheoretical and empiricalthat economists use. Following this
introduction to economics, we cover chapters 1-2 and 12 of Collaborative Statistics, in order to give
students the minimum statistical skills they will need to write an economics research paper.
Section II of the course covers core microeconomic concepts and theory, including demand and supply,
elasticity, welfare measures, the neoclassical theory of the firm, and the following market structures:
perfect competition, monopoly and oligopoly.
1

Section III of the course covers core macroeconomic concepts and theory, including interest rates,
measures of economic activity, financial institutions, the aggregate demand / aggregate supply model,
money, growth, and bond and currency markets.
2

To apply their knowledge after completing the readings and exercises, a student might write a short
paper. Ideally the student would use the Internet to find a data source related to a topic covered in this
reader. Then the student would carry out a short study using one or more economic models and the
data they found to help answer a specific question. Such a study might be similar to the Case in Point
boxes below, but as stressed in this course, would clearly use models and new statistics.
References:
Rittenberg, Libby & Timothy Tregarthen, 2009. Principles of Economics. 1st edition. Flat World
Knowledge (www.flatworldknowledge.com). These chapters were published under a Creative Commons
BY-NC-SA 3.0 license.
Illowsky, Barbara & Susan Dean, 2012. Collaborative Statistics. http://cnx.org/content/col10522/latest/.
The Creative Commons attribution license (http://creativecommons.org/licenses/by/2.0/) applies.

1
To economize on space and time, this reader does not include a chapter on the theory of consumer choice. An
interested reader could consult Chapter 7 of Rittenberg and Tregarthens Principles of Economics, available at
http://www.sjsu.edu/faculty/matthew.holian/RT/Master.html.
2
This reader does not emphasize comparing different macroeconomic theories; an interested reader could consult
Chapter 32 of Rittenberg and Tregarthen (2009).



Table of Contents (Brief)

Part I: Introduction to Economics and Statistics (Rittenberg and Tregarthen & Illowsky and Dean)
Chapter 1: Economics: The Study of Choice (Rittenberg and Tregarthen)
Chapter 2: Confronting Scarcity: Choices in Production (Rittenberg and Tregarthen)
Chapter 1: Sampling and Data (Illowsky and Dean)
Chapter 2: Descriptive Statistics (Illowsky and Dean)
Chapter 12: Linear Regression and Correlation (Illowsky and Dean)

Part II: Microeconomics (From Rittenberg and Tregarthen)
Chapter 3: Demand and Supply
Chapter 4: Applications of Demand and Supply
Chapter 5: Elasticity: A Measure of Response
Chapter 6: Markets, Maximizers, and Efficiency
Chapter 8: Production and Cost
Chapter 9: Competitive Markets for Goods and Services
Chapter 10: Monopoly
Chapter 11: The World of Imperfect Competition (Oligopoly)

Part III: Macroeconomics (From Rittenberg and Tregarthen)
Chapter 13: Interest Rates and the Markets for Capital and Natural Resources
Chapter 20: Macroeconomics: The Big Picture
Chapter 21: Measuring Total Output and Income
Chapter 22: Aggregate Demand and Aggregate Supply
Chapter 23: Economic Growth
Chapter 24: The Nature and Creation of Money
Chapter 25: Financial Markets and the Economy (Bond and Currency Markets)








Table of Contents (Detailed)

Part I: Introduction to Economics and Statistics


Chapter 1: Economics: The Study of Choice (Rittenberg and Tregarthen)

Section 1: Defining Economics
Section 2: The Field of Economics
Section 3: The Economists Tool Kit
Section 4: Review and Practice

Chapter 2: Confronting Scarcity: Choices in Production (Rittenberg and Tregarthen)

Section 1: Factors of Production
Section 2: The Production Possibilities Curve
Section 3: Applications of the Production Possibilities Model
Section 4: Review and Practice

Chapter 1: Sampling and Data (Illowsky and Dean)

Sections 1.1-1.5: Sampling, Statistics, Probability and Data
Sections 1.6-1.9: Sampling, Variation, Answers and Rounding Off
Sections 1.10-1.14: Summary, Practice, Homework, Lab 1 and Lab 2
Solutions

Chapter 2: Descriptive Statistics (Illowsky and Dean)

Sections 2.1-2.5: Describing Data, Displaying Data (Stemplots, Line & Bar Graphs, Histograms, Box Plot)
Sections 2.6 -2.7: Measures of the Location and Center of the Data
Sections 2.8-2.9: Skewness, Mean, Median, Mode and Measures of the Spread of the Data
Sections 2.10-2.14: Summary of Formulas, Practice 1, Practice 2, Homework, Lab
Solutions

Chapter 12: Linear Regression and Correlation (Illowsky and Dean)

Sections 12.1 -12.3: Linear Regression and Correlation, Linear Equations, Slope and Y-Intercept
Sections 12.4-12.6: Scatter Plots, Regression Equation, Correlation Coefficient, r
2

Sections 12.7-12.9: Testing the Significance of the Correlation Coefficient , Outliers
Sections 12.10: Critical Values of the Sample Correlation Coefficient 95% Table
Sections 12.11-12.14: Summary, Practice, Lab 1, Lab 2, Lab 3
Solutions






Part II: Microeconomics

Chapter 3: Demand and Supply

Section 1: Demand
Section 2: Supply
Section 3: Demand, Supply, and Equilibrium
Section 4: Review and Practice

Chapter 4: Applications of Demand and Supply

Section 1: Putting Demand and Supply to Work
Section 2: Government Intervention in Market Prices: Price Floors and Price Ceilings
Section 3: The Market for Health-Care Services
Section 4: Review and Practice

Chapter 5: Elasticity: A Measure of Response

Section 1: The Price Elasticity of Demand
Section 2: Responsiveness of Demand to Other Factors
Section 3: Price Elasticity of Supply
Section 4: Review and Practice

Chapter 6: Markets, Maximizers, and Efficiency

Section 1: The Logic of Maximizing Behavior
Section 2: Maximizing in the Marketplace
Section 3: Market Failure
Section 4: Review And Practice

Chapter 8: Production and Cost

Section 1: Production Choices and Costs: The Short Run
Section 2: Production Choices and Costs: The Long Run
Section 3: Review and Practice

Chapter 9: Competitive Markets for Goods and Services

Section 1: Perfect Competition: A Model
Section 2: Output Determination in the Short Run
Section 3: Perfect Competition in the Long Run
Section 4: Review and Practice

Chapter 10: Monopoly

Section 1: The Nature of Monopoly
Section 2: The Monopoly Model
Section 3: Assessing Monopoly
Section 4: Review and Practice

Chapter 11: The World of Imperfect Competition

Section 2: Oligopoly: Competition Among the Few
Section 3: Extensions of Imperfect Competition: Advertising and Price Discrimination
Section 4: Review and Practice

Part III: Macroeconomics

Chapter 13: Interest Rates and the Markets for Capital and Natural Resources

Section 1: Time and Interest Rates
Section 2: Interest Rates and Capital
Section 4: Review and Practice

Chapter 20: Macroeconomics: The Big Picture

Section 1: Growth of Real GDP and Business Cycles
Section 2: Price-Level Changes
Section 3: Unemployment
Section 4: Review and Practice

Chapter 21: Measuring Total Output and Income

Section 1: Measuring Total Output
Section 2: Measuring Total Income
Section 3: GDP and Economic Well-Being
Section 4: Review and Practice

Chapter 22: Aggregate Demand and Aggregate Supply

Section 1: Aggregate Demand
Section 2: Aggregate Demand and Aggregate Supply: the Long Run and the Short Run
Section 3: Recessionary and Inflationary Gaps and Long-run Macroeconomic Equilibrium
Section 4: Review and Practice

Chapter 23: Economic Growth

Section 1: The Significance of Economic Growth
Section 2: Growth and the Long-Run Aggregate Supply Curve
Section 3: Determinants of Economic Growth
Section 4: Review and Practice

Chapter 24: The Nature and Creation of Money

Section 1: What Is Money?
Section 2: The Banking System and Money Creation
Section 3: The Federal Reserve System
Section 4: Review and Practice

Chapter 25: Financial Markets and the Economy

Section 1: The Bond and Foreign Exchange Markets
Section 2: Demand, Supply, and Equilibrium in the Money Market
Section 3: Review and Practice
| A | | | 1
Economics: The Study of
Choice
S7AR7 UP: CONOMICS IN 7H NWS
2008 seeneJ tc be t|e ye.. c| eccrcn|c re.s. |.cn t|e .c.st |r.rc|.| c.|s|s s|rce t|e C.e.t |ep.ess|cr tc t|e
pcss|b|||ty c| . g|cb.| .ecess|cr, tc gy..t|rg g.sc||re .rJ |ccJ p.|ces, .rJ tc p|urg|rg |cus|rg p.|ces, eccrcn|c
,uest|crs .e.e t|e p.|n..y |.ctc.s |r t|e p.es|Jert|.| c.np.|gr c| 2008 .rJ Jcn|r.teJ t|e re.s gere..||y.
\|.t c.uses t|e p.|ces c| scne gccJ tc .|se .|||e t|e p.|ces c| scne ct|e. gccJs |.||` |.|ce Jete.n|r.t|cr |s
cre c| t|e t||rgs t|.t .e .||| stuJy |r t||s bcck. \e .||| .|sc ccrs|Je. |.ctc.s t|.t |e.J .r eccrcny tc |.|| |rtc . .e
cess|cr.rJ t|e .ttenpts tc ||n|t |t.
\|||e t|e |rvest|g.t|cr c| t|ese p.cb|ens su.e|y |.||s .|t||r t|e p.cv|rce c| eccrcn|cs, eccrcn|cs erccn
p.sses . |.. b.c.Je. ..rge c| |ssues. ||t|n.te|y, eccrcn|cs |s t|e stuJy c| c|c|ce. bec.use c|c|ces ..rge cve. eve.y
|n.g|r.b|e .spect c| |un.r expe.|erce, sc Jces eccrcn|cs. |ccrcn|sts |.ve |rvest|g.teJ t|e r.tu.e c| |.n||y |||e,
t|e ..ts, eJuc.t|cr, c.|ne, spc.ts, cb c.e.t|crt|e ||st |s v|.tu.||y erJ|ess bec.use sc nuc| c| cu. ||ves |rvc|ves
n.k|rg c|c|ces.
|c. Jc |rJ|v|Ju.|s n.ke c|c|ces. \cu|J ycu ||ke bette. g..Jes` Vc.e t|ne tc .e|.x` Vc.e t|ne ..tc||rg
ncv|es` Cett|rg bette. g..Jes p.cb.b|y .e,u|.es nc.e t|ne stuJy|rg, .rJ pe.|.ps |ess .e|.x.t|cr .rJ erte.t.|r
nert. |ct cr|y nust .e n.ke c|c|ces .s |rJ|v|Ju.|s, .e nust n.ke c|c|ces .s . scc|ety. |c .e ..rt . c|e.re. er
v|.crnert` |.ste. eccrcn|c g.c.t|` bct| n.y be Jes|..b|e, but e|c.ts tc c|e.r up t|e erv|.crnert n.y ccr||ct
.|t| |.ste. eccrcn|c g.c.t|. Scc|ety nust n.ke c|c|ces.
|ccrcn|cs |s Je|reJ |ess by t|e subects eccrcn|sts |rvest|g.te t|.r by t|e ..y |r .||c| eccrcn|sts |rvest
|g.te t|en. |ccrcn|sts |.ve . ..y c| |cck|rg .t t|e .c.|J t|.t J||e.s |.cn t|e ..y sc|c|..s |r ct|e. J|sc|p||res
|cck .t t|e .c.|J. |t |s t|e ecoo|c .o, o| |||||, t||s c|.pte. |rt.cJuces t|.t ..y c| t||rk|rg.
economics
A scc|.| sc|erce t|.t
ex.n|res |c. pecp|e
c|ccse .ncrg t|e
.|te.r.t|ves .v.||.b|e tc t|en.
scarcity
|e ccrJ|t|cr c| |.v|rg tc
c|ccse .ncrg .|te.r.t|ves.
scarce good
A gccJ |c. .||c| t|e c|c|ce
c| cre .|te.r.t|ve .e,u|.es
t|.t .rct|e. be g|ver up.
free good
A gccJ |c. .||c| t|e c|c|ce
c| cre use Jces rct .e,u|.e
t|.t .rct|e. be g|ver up.
1. DEFINING ECONOMICS
L A R N I N G O 8 1 C 7 I V S
1. Dene economics.
2. xpIain the concepts of scarcity and opportunity cost and how they reIate to the denition of
economics.
3. Understand the three fundamentaI economic questions: What shouId be produced1 How
shouId goods and services be produced1 Ior whom shouId goods and services be produced1
Economics is a social science that examines how people choose among the alteinatives available to
them. It is social because it involves people and theii behavioi. It is a science because it uses, as much as
possible, a scientifc appioach in its investigation of choices.
1.1 Scaicity, Choice, and Cost
All choices mean that one alteinative is selected ovei anothei. Selecting among alteinatives involves
thiee ideas cential to economics: scaicity, choice, and oppoitunity cost.
Scarcity
Oui iesouices aie limited. At any one time, we have only so much land, so many factoiies, so much oil,
so many people. But oui wants, oui desiies foi the things that we can pioduce with those iesouices, aie
unlimited. We would always like moie and bettei housing, moie and bettei education-moie and bet-
tei of piactically eveiything.
If oui iesouices weie also unlimited, we could say yes to each of oui wants-and theie would be
no economics. Because oui iesouices aie limited, we cannot say yes to eveiything. To say yes to one
thing iequiies that we say no to anothei. Whethei we like it oi not, we must make choices.
Oui unlimited wants aie continually colliding with the limits of oui iesouices, foicing us to pick
some activities and to ieject otheis. Scarcity is the condition of having to choose among alteinatives. A
scarce good is one foi which the choice of one alteinative iequiies that anothei be given up.
Considei a paicel of land. The paicel piesents us with seveial alteinative uses. We could build a
house on it. We could put a gas station on it. We could cieate a small paik on it. We could leave the
land undeveloped in oidei to be able to make a decision latei as to how it should be used.
Suppose we have decided the land should be used foi housing. Should it be a laige and expensive
house oi seveial modest ones: Suppose it is to be a laige and expensive house. Who should live in the
house: If the Lees live in it, the Nguyens cannot. Theie aie alteinative uses of the land both in the sense
of the type of use and also in the sense of who gets to use it. The fact that land is scaice means that soci-
ety must make choices conceining its use.
Viitually eveiything is scaice. Considei the aii we bieathe, which is available in huge quantity at
no chaige to us. Could it possibly be scaice:
The test of whethei aii is scaice is whethei it has alteinative uses. What uses can we make of the
aii: We bieathe it. We pollute it when we diive oui cais, heat oui houses, oi opeiate oui factoiies. In
efect, one use of the aii is as a gaibage dump. We ceitainly need the aii to bieathe. But just as ceitainly,
we choose to dump gaibage in it. Those two uses aie cleaily alteinatives to each othei. The moie
gaibage we dump in the aii, the less desiiable-and healthy-it will be to bieathe. If we decide we want
to bieathe cleanei aii, we must limit the activities that geneiate pollution. Aii is a scaice good because
it has alteinative uses.
Not all goods, howevei, confiont us with such choices. A free good is one foi which the choice of
one use does not iequiie that we give up anothei. One example of a fiee good is giavity. The fact that
giavity is holding you to the eaith does not mean that youi neighboi is foiced to diift up into space!
One peison's use of giavity is not an alteinative to anothei peison's use.
Theie aie not many fiee goods. Outei space, foi example, was a fiee good when the only use we
made of it was to gaze at it. But now, oui use of space has ieached the point wheie one use can be an al-
teinative to anothei. Conficts have alieady aiisen ovei the allocation of oibital slots foi communica-
tions satellites. Thus, even paits of outei space aie scaice. Space will suiely become moie scaice as we
fnd new ways to use it. Scaicity chaiacteiizes viitually eveiything. Consequently, the scope of econom-
ics is wide indeed.
8 PRINCIPLS OI CONOMICS
opportunity cost
|e v.|ue c| t|e best
.|te.r.t|ve |c.gcre |r n.k|rg
.ry c|c|ce.
Scarcity and the IundamentaI conomic Questions
The choices we confiont as a iesult of scaicity iaise thiee sets of issues. Eveiy economy must answei
the following questions:
1. What should be produced: Using the economy's scaice iesouices to pioduce one thing iequiies
giving up anothei. Pioducing bettei education, foi example, may iequiie cutting back on othei
seivices, such as health caie. A decision to pieseive a wildeiness aiea iequiies giving up othei
uses of the land. Eveiy society must decide what it will pioduce with its scaice iesouices.
2. How should goods and services be produced: Theie aie all soits of choices to be made in
deteimining how goods and seivices should be pioduced. Should a fim employ a few skilled oi a
lot of unskilled woikeis: Should it pioduce in its own countiy oi should it use foieign plants:
Should manufactuiing fims use new oi iecycled iaw mateiials to make theii pioducts:
3. For whom should goods and services be produced: If a good oi seivice is pioduced, a decision
must be made about who will get it. A decision to have one peison oi gioup ieceive a good oi
seivice usually means it will not be available to someone else. Foi example, iepiesentatives of the
pooiest nations on eaith often complain that eneigy consumption pei peison in the United States
is 17 times gieatei than eneigy consumption pei peison in the woild's 62 pooiest countiies.
Ciitics aigue that the woild's eneigy should be moie evenly allocated. Should it: That is a foi
whom" question.
Eveiy economy must deteimine what should be pioduced, how it should be pioduced, and foi whom it
should be pioduced. We shall ietuin to these questions again and again.
Opportunity Cost
It is within the context of scaicity that economists defne what is peihaps the most impoitant concept
in all of economics, the concept of oppoitunity cost. Opportunity cost is the value of the best altein-
ative foigone in making any choice.
The oppoitunity cost to you of ieading the iemaindei of this chaptei will be the value of the best
othei use to which you could have put youi time. If you choose to spend $20 on a potted plant, you
have simultaneously chosen to give up the benefts of spending the $20 on pizzas oi a papeiback book
oi a night at the movies. If the book is the most valuable of those alteinatives, then the oppoitunity cost
of the plant is the value of the enjoyment you otheiwise expected to ieceive fiom the book.
The concept of oppoitunity cost must not be confused with the puichase piice of an item. Con-
sidei the cost of a college oi univeisity education. That includes the value of the best alteinative use of
money spent foi tuition, fees, and books. But the most impoitant cost of a college education is the value
of the foigone alteinative uses of time spent studying and attending class instead of using the time in
some othei endeavoi. Students saciifce that time in hopes of even gieatei eainings in the futuie oi be-
cause they place a value on the oppoitunity to leain. Oi considei the cost of going to the doctoi. Pait of
that cost is the value of the best alteinative use of the money iequiied to see the doctoi. But, the cost
also includes the value of the best alteinative use of the time iequiied to see the doctoi. The essential
thing to see in the concept of oppoitunity cost is found in the name of the concept. Oppoitunity cost is
the value of the best oppoitunity foigone in a paiticulai choice. It is not simply the amount spent on
that choice.
The concepts of scaicity, choice, and oppoitunity cost aie at the heait of economics. A good is
scaice if the choice of one alteinative iequiies that anothei be given up. The existence of alteinative
uses foices us to make choices. The oppoitunity cost of any choice is the value of the best alteinative
foigone in making it.
k Y 7 A k A W A Y S
< |ccrcn|cs |s . scc|.| sc|erce t|.t ex.n|res |c. pecp|e c|ccse .ncrg t|e .|te.r.t|ves .v.||.b|e tc t|en.
< Sc..c|ty |np||es t|.t .e nust g|ve up cre .|te.r.t|ve |r se|ect|rg .rct|e.. A gccJ t|.t |s rct sc..ce |s .
|.ee gccJ.
< |e t|.ee |urJ.nert.| eccrcn|c ,uest|crs ..e. \|.t s|cu|J be p.cJuceJ` |c. s|cu|J gccJs .rJ
se.v|ces be p.cJuceJ` |c. .|cn s|cu|J gccJs .rJ se.v|ces be p.cJuceJ`
< |ve.y c|c|ce |.s .r cppc.tur|ty ccst .rJ cppc.tur|ty ccsts .|ect t|e c|c|ces pecp|e n.ke. |e
cppc.tur|ty ccst c| .ry c|c|ce |s t|e v.|ue c| t|e best .|te.r.t|ve t|.t |.J tc be |c.gcre |r n.k|rg t|.t
c|c|ce.
CHAP7R 1 CONOMICS: 7H S7UDY OI CHOIC 9
7 R Y I 7 !
|Jert||y t|e e|enerts c| sc..c|ty, c|c|ce, .rJ cppc.tur|ty ccst |r e.c| c| t|e |c||c.|rg.
1. |e |rv|.crnert.| |.ctect|cr Agercy |s ccrs|Je.|rg .r c.Je. t|.t . 500.c.e ..e. cr t|e cutsk|.ts c| .
|..ge c|ty be p.ese.veJ |r |ts r.tu..| st.te, bec.use t|e ..e. |s |cne tc . .cJert t|.t |s ccrs|Je.eJ .r
erJ.rge.eJ spec|es. |eve|cpe.s |.J p|.rreJ tc bu||J . |cus|rg Jeve|cpnert cr t|e |.rJ.
2. |e n.r.ge. c| .r .utcncb||e .ssenb|y p|.rt |s ccrs|Je.|rg .|et|e. tc p.cJuce c..s c. spc.t ut|||ty
ve||c|es (S|\s) rext ncrt|. Assune t|.t t|e ,u.rt|t|es c| |.bc. .rJ ct|e. n.te.|.|s .e,u|.eJ .cu|J be t|e
s.ne |c. e|t|e. type c| p.cJuct|cr.
3. A ycurg n.r .|c .ert tc .c.k .s . ru.ses .|Je .|te. g..Ju.t|rg |.cn ||g| sc|cc| |e.ves ||s cb tc gc tc
cc||ege, .|e.e |e .||| cbt.|r t..|r|rg .s . .eg|ste.eJ ru.se.
Case in Point: 7he Rising Cost of nergy
2010 jupiterimages Corporation
O|| |s .r ex|.ust|b|e .escu.ce. |e c|| .e bu.r tcJ.y .||| rct be .v.||.b|e |c. use |r t|e |utu.e. |..t c| t|e cp
pc.tur|ty ccst c| cu. ccrsunpt|cr c| gccJs suc| .s g.sc||re t|.t ..e p.cJuceJ |.cn c|| |rc|uJes t|e v.|ue
pecp|e |r t|e |utu.e n|g|t |.ve p|.ceJ cr c|| .e use tcJ.y.
|t .ppe..s t|.t t|e ccst c| cu. use c| c|| n.y be .|s|rg. \e |.ve beer us|rg ||g|t c.uJe, t|e c|| |curJ |r t|e
g.curJ |r Jepcs|ts t|.t c.r be .e.J||y t.ppeJ. As ||g|t c.uJe beccnes nc.e sc..ce, t|e .c.|J n.y reeJ tc
tu.r tc scc.||eJ |e.vy c.uJe, t|e c.uJe c|| t|.t |s |curJ |r t|e s.rJy sc|| c| p|.ces suc| .s .r.J. .rJ
\ere.ue|.. |.t c|| ex|sts |r suc| .burJ.rce t|.t |t p.cpe|s \ere.ue|. tc t|e tcp c| t|e .c.|J ||st c| .v.||.b|e
c||. S.uJ| A..b|. ncves tc t|e seccrJ pcs|t|cr, .r.J. |s t||.J.
|e J||cu|ty .|t| t|e c|| n|xeJ |r t|e s.rJ |s t|.t ext..ct|rg |t |s |.. nc.e ccst|y t|.r ||g|t c.uJe, bct| |r
te.ns c| t|e experJ|tu.es .e,u|.eJ .rJ |r te.ns c| t|e erv|.crnert.| J.n.ge t|.t n|r|rg |t c.e.tes. |c.t|e.r
A|be.t., |r .r.J., bc.sts . ||c.|J.s|.eJ ..e. .|cse s.rJy sc||s ..e .|c| |r c.uJe c||. Scne c| t|.t c|| |s 1,200
|eet urJe.g.curJ. |xt..ct|rg |t .e,u|.es punp|rg ste.n |rtc t|e c||y s.rJ .rJ t|er punp|rg up t|e .esu|t.rt
c||y sy.up. |.t sy.up |s t|er p|.ceJ |rtc |uge, |rJust.|.|s|.eJ ..s||rg n.c||res t|.t sep...te c.uJe c||. \|.t
|s |e|t cve. |s tcx|c .rJ .||| be p|.ceJ |r |uge |.kes t|.t ..e be|rg c.e.teJ by J|gg|rg p|ts |r t|e g.curJ 200
|eet Jeep. |e c|| p.cJuceJ |.cn t|ese s.rJs |.s beccne |npc.t.rtA|be.t. |s t|e |..gest |c.e|gr supp||e.
c| c|| tc t|e |r|teJ St.tes.
10 PRINCIPLS OI CONOMICS
S.rJs t|.t ..e c|cse. tc t|e su.|.ce ..e .encveJ by bu||Jc.e.s .rJ g|.rt c..res, t|e |c.est cve. |t |s c|e..eJ
...y. |e c||y s.rJ |s t|er |.u|eJ c| |r t.cstc.y Junp t.ucks .||c|, .|er |||eJ, .e|g| nc.e t|.r . bce|rg
4. ct.| SA, . |.erc| ccnp.ry, |s |e.J|rg t|e ..ce tc Jeve|cp .r.J.s c||. 'e.r |ucCu|.|cu, t|e p.es|Jert c|
ct.| SAs .r.J|.r cpe..t|crs, s.ys t|.t t|e ext..c.J|r..||y ccst|y p.ccess c| ext..ct|rg |e.vy c.uJe |s
scnet||rg t|e .c.|J |s gc|rg tc |.ve tc get useJ tc. |e ||g|t c.uJe urJ|sccve.eJ tcJ.y |s gett|rg sc..ce.
.rJ sc..ce., |e tc|J |e /o|| .|ee| .ooo|. \e |.ve tc .ccept t|e .e.||ty c| gecsc|erce, .||c| |s t|.t t|e rext
gere..t|cr c| c|| .escu.ces .||| be |e.v|e..
A|.e.Jy, ct.| SA |.s c|e..cut t|cus.rJs c| .c.es c| |c.est |.rJ |r c.Je. tc g.|r .ccess tc t|e c||y s.rJ be|c..
|e p.ccess c| ext..ct|rg |e.vy c.uJe c|| ccsts t|e ccnp.ry 25 . b...e|ccnp..eJ tc t|e 6 pe. b...e| ccst
c| ext..ct|rg .rJ .e|r|rg ||g|t c.uJe. |xt..ct|rg |e.vy c.uJe gere..tes t|.ee t|nes .s nuc| g.eer|cuse g.s
pe. b...e| .s Jces ||g|t c.uJe. by 2015, |c.t VcVu...y, t|e sn.|| (pcpu|.t|cr 61,000) tc.r t|.t |.s beccne
t|e |e.J,u..te.s c| |c.t|e.r A|be.t.s c.uJe c|| bccn, .||| en|t nc.e g.eer|cuse g.s t|.r t|e ert|.e ccurt.y
c| |ern..k (pcpu|.t|cr 5.4 n||||cr). .r.J. .||| exceeJ |ts g.eer|cuse g.s ,uct. set by t|e |yctc Ac
cc.Js.r |rte.r.t|cr.| t.e.ty .|neJ .t ||n|t|rg g|cb.| ...n|rg|..ge|y .s . .esu|t c| Jeve|cp|rg |ts |e.vy
c.uJe Jepcs|ts.
|c cre ever ccrs|Je.eJ t|e ext..ct|cr c| |e.vy c.uJe .|er ||g|t c.uJe ..s c|e.p. |r t|e |.te 1990s, c|| ccst
ust 12 pe. b...e|, .rJ Jepcs|ts c| |e.vy c.uJe suc| .s t|cse |r .r.J. .tt..cteJ ||tt|e .ttert|cr. by n|J2006,
c|| sc|J |c. nc.e t|.r 0 pe. b...e|, .rJ .r.J.s |e.vy c.uJe ..s suJJer|y . |ct ccnncJ|ty. |t ncveJ
|.cn be|rg ust .r |rte.est|rg expe.|nert |r rc.t|e.r .r.J. tc .e.||y t||s |s t|e |utu.e scu.ce c| c|| supp|y,
C.eg St.|rg|.n c| t|e .r.J|.r Asscc|.t|cr c| |et.c|eun |.cJuce.s tc|J A| '..ee...
A|be.t.s ere.gy n|r|ste., C.eg Ve|c||r, Je|erJs t|e p.cv|rces Jec|s|cr tc p.cceeJ .|t| t|e exp|c|t.t|cr c| |ts
c||y s.rJ. |e.e |s . ccst tc |t, but t|e bere|ts ..e subst.rt|.||y g.e.te., |e |rs|sts.
|ct eve.ycre .g.ees. Cec.ge |c|t..s, . nenbe. c| t|e V|k|se. .ee t.|be, ||ves Jc.rst.e.n |.cn t|e c|| s.rJs
Jeve|cpnert. +cu see . |ct c| t|e |.rJ Jug up, . |ct c| t|e bc.e.| |c.est st.uck Jc.r .rJ |t's upsett|rg, |t |||s
ne .|t| ..ge, |e s.ys. ||.r. C|bscr c| t|e |..k|.rJ |rst|tute, .r erv|.crnert.| .Jvcc.cy g.cup, s.ys t|.t ycu
c.r see t|e erv|.crnert.| J.n.ge gere..teJ by t|e ext..ct|cr c| c|| s.rJs ..curJ |c.t VcVu...y |.cn t|e
nccr. \|.t .e ..e gc|rg tc be |.v|rg |s Jest.uct|cr c| ve.y, ve.y v.|u.b|e eccsystens, .rJ pe.n.rert pc|
|ut|cr, s|e s.ys.
.ooce ^|e|o |eo., || |oJe.' ^| .oeeo ||||. /oc| .. 200S (ee e|||o|,oeeoe|:. oJ |oe|| Oo|J. ^ ||ce .oe. || O|o| o
.|oJe ||o Oo|J.' |e /o|| .|ee| .ooo| ||e. /oc| 2.. 2006. ^
A N S W R S 7 O 7 R Y I 7 ! P R O 8 L M S
1. |e 500.c.e ..e. |s sc..ce bec.use |t |.s .|te.r.t|ve uses. p.ese.v.t|cr |r |ts r.tu..| st.te c. . s|te |c.
|cnes. A c|c|ce nust be n.Je bet.eer t|ese uses. |e cppc.tur|ty ccst c| p.ese.v|rg t|e |.rJ |r |ts
r.tu..| st.te |s t|e |c.gcre v.|ue c| t|e |.rJ .s . |cus|rg Jeve|cpnert. |e cppc.tur|ty ccst c| us|rg t|e
|.rJ .s . |cus|rg Jeve|cpnert |s t|e |c.gcre v.|ue c| p.ese.v|rg t|e |.rJ.
2. |e sc..ce .escu.ces ..e t|e p|.rt .rJ t|e |.bc. .t t|e p|.rt. |e n.r.ge. nust c|ccse bet.eer
p.cJuc|rg c..s .rJ p.cJuc|rg S|\s. |e cppc.tur|ty ccst c| p.cJuc|rg c..s |s t|e p.c|t t|.t ccu|J be
e..reJ |.cn p.cJuc|rg S|\s, t|e cppc.tur|ty ccst c| p.cJuc|rg S|\s |s t|e p.c|t t|.t ccu|J be e..reJ
|.cn p.cJuc|rg c..s.
3. |e n.r c.r Jevcte ||s t|ne tc ||s cu..ert c..ee. c. tc .r eJuc.t|cr, ||s t|ne |s . sc..ce .escu.ce. |e
nust c|ccse bet.eer t|ese .|te.r.t|ves. |e cppc.tur|ty ccst c| ccrt|ru|rg .s . ru.ses .|Je |s t|e
|c.gcre bere|t |e expects |.cn t..|r|rg .s . .eg|ste.eJ ru.se, t|e cppc.tur|ty ccst c| gc|rg tc cc||ege |s
t|e |c.gcre |rccne |e ccu|J |.ve e..reJ .c.k|rg |u||t|ne .s . ru.ses .|Je.
CHAP7R 1 CONOMICS: 7H S7UDY OI CHOIC 11
2. THE FIELD OF ECONOMICS
L A R N I N G O 8 1 C 7 I V S
1. xpIain the distinguishing characteristics of the economic way of thinking.
2. Distinguish between microeconomics and macroeconomics.
We have examined the basic concepts of scaicity, choice, and oppoitunity cost in economics. In this
section, we will look at economics as a feld of study. We begin with the chaiacteiistics that distinguish
economics fiom othei social sciences.
2.1 The Economic Way of Thinking
Economists study choices that scaicity iequiies us to make. This fact is not what distinguishes econom-
ics fiom othei social sciences; all social scientists aie inteiested in choices. An anthiopologist might
study the choices of ancient peoples; a political scientist might study the choices of legislatuies; a psy-
chologist might study how people choose a mate; a sociologist might study the factois that have led to a
iise in single-paient households. Economists study such questions as well. What is it about the study of
choices by economists that makes economics difeient fiom these othei social sciences:
Thiee featuies distinguish the economic appioach to choice fiom the appioaches taken in othei
social sciences:
1. Economists give special emphasis to the iole of oppoitunity costs in theii analysis of choices.
2. Economists assume that individuals make choices that seek to maximize the value of some
objective, and that they defne theii objectives in teims of theii own self-inteiest.
3. Individuals maximize by deciding whethei to do a little moie oi a little less of something.
Economists aigue that individuals pay attention to the consequences of small changes in the
levels of the activities they puisue.
The emphasis economists place on oppoitunity cost, the idea that people make choices that maximize
the value of objectives that seive theii self-inteiest, and a focus on the efects of small changes aie ideas
of gieat powei. They constitute the coie of economic thinking. The next thiee sections examine these
ideas in gieatei detail.
Opportunity Costs Are Important
If doing one thing iequiies giving up anothei, then the expected benefts of the alteinatives we face will
afect the ones we choose. Economists aigue that an undeistanding of oppoitunity cost is ciucial to the
examination of choices.
As the set of available alteinatives changes, we expect that the choices individuals make will
change. A iainy day could change the oppoitunity cost of ieading a good book; we might expect moie
ieading to get done in bad than in good weathei. A high income can make it veiy costly to take a day
of; we might expect highly paid individuals to woik moie houis than those who aie not paid as well. If
individuals aie maximizing theii level of satisfaction and fims aie maximizing piofts, then a change in
the set of alteinatives they face may afect theii choices in a piedictable way.
The emphasis on oppoitunity costs is an emphasis on the examination of alteinatives. One beneft
of the economic way of thinking is that it pushes us to think about the value of alteinatives in each
pioblem involving choice.
IndividuaIs Maximize in Pursuing SeIf-Interest
What motivates people as they make choices: Peihaps moie than anything else, it is the economist's
answei to this question that distinguishes economics fiom othei felds.
Economists assume that individuals make choices that they expect will cieate the maximum value
of some objective, given the constiaints they face. Fuitheimoie, economists assume that people's ob-
jectives will be those that seive theii own self-inteiest.
Economists assume, foi example, that the owneis of business fims seek to maximize pioft. Given
the assumed goal of pioft maximization, economists can piedict how fims in an industiy will iespond
to changes in the maikets in which they opeiate. As laboi costs in the United States iise, foi example,
economists aie not suipiised to see fims moving some of theii manufactuiing opeiations oveiseas.
12 PRINCIPLS OI CONOMICS
margin
|e cu..ert |eve| c| .r
.ct|v|ty.
choice at the margin
A Jec|s|cr tc Jc . ||tt|e nc.e
c. . ||tt|e |ess c| scnet||rg.
microeconomics
|e b..rc| c| eccrcn|cs
t|.t |ccuses cr t|e c|c|ces
n.Je by ccrsune.s .rJ
|.ns .rJ t|e |np.cts t|cse
c|c|ces |.ve cr |rJ|v|Ju.|
n..kets.
macroeconomics
|e b..rc| c| eccrcn|cs
t|.t |ccuses cr t|e |np.ct c|
c|c|ces cr t|e tct.|, c.
.gg.eg.te, |eve| c| eccrcn|c
.ct|v|ty.
Similaily, economists assume that maximizing behavioi is at woik when they examine the behavi-
oi of consumeis. In studying consumeis, economists assume that individual consumeis make choices
aimed at maximizing theii level of satisfaction. In the next chaptei, we will look at the iesults of the
shift fiom skiing to snowboaiding; that is a shift that iefects the puisuit of self-inteiest by consumeis
and by manufactuieis.
In assuming that people puisue theii self-inteiest, economists aie not assuming people aie selfsh.
People cleaily gain satisfaction by helping otheis, as suggested by the laige chaiitable contiibutions
people make. Puisuing one's own self-inteiest means puisuing the things that give one satisfaction. It
need not imply gieed oi selfshness.
Choices Are Made at the Margin
Economists aigue that most choices aie made at the maigin." The margin is the cuiient level of an
activity. Think of it as the edge fiom which a choice is to be made. A choice at the margin is a de-
cision to do a little moie oi a little less of something.
Assessing choices at the maigin can lead to extiemely useful insights. Considei, foi example, the
pioblem of cuitailing watei consumption when the amount of watei available falls shoit of the amount
people now use. Economists aigue that one way to induce people to conseive watei is to iaise its piice.
A common iesponse to this iecommendation is that a highei piice would have no efect on watei con-
sumption, because watei is a necessity. Many people asseit that piices do not afect watei consumption
because people need" watei.
But choices in watei consumption, like viitually all choices, aie made at the maigin. Individuals do
not make choices about whethei they should oi should not consume watei. Rathei, they decide wheth-
ei to consume a little moie oi a little less watei. Household watei consumption in the United States
totals about 103 gallons pei peison pei day. Think of that staiting point as the edge fiom which a
choice at the maigin in watei consumption is made. Could a highei piice cause you to use less watei
biushing youi teeth, take shoitei showeis, oi watei youi lawn less: Could a highei piice cause people
to ieduce theii use, say, to 104 gallons pei peison pei day: To 103: When we examine the choice to
consume watei at the maigin, the notion that a highei piice would ieduce consumption seems much
moie plausible. Piices afect oui consumption of watei because choices in watei consumption, like oth-
ei choices, aie made at the maigin.
The elements of oppoitunity cost, maximization, and choices at the maigin can be found in each
of two bioad aieas of economic analysis: micioeconomics and macioeconomics. Youi economics
couise, foi example, may be designated as a micio" oi as a macio" couise. We will look at these two
aieas of economic thought in the next section.
2.2 Micioeconomics and Macioeconomics
The feld of economics is typically divided into two bioad iealms: micioeconomics and macioeconom-
ics. It is impoitant to see the distinctions between these bioad aieas of study.
Microeconomics is the bianch of economics that focuses on the choices made by individual
decision-making units in the economy-typically consumeis and fims-and the impacts those choices
have on individual maikets. Macroeconomics is the bianch of economics that focuses on the impact
of choices on the total, oi aggiegate, level of economic activity.
Why do tickets to the best conceits cost so much: How does the thieat of global waiming afect
ieal estate piices in coastal aieas: Why do women end up doing most of the housewoik: Why do seni-
oi citizens get discounts on public tiansit systems: These questions aie geneially iegaided as micioeco-
nomic because they focus on individual units oi maikets in the economy.
Is the total level of economic activity iising oi falling: Is the iate of infation incieasing oi decieas-
ing: What is happening to the unemployment iate: These aie questions that deal with aggiegates, oi
totals, in the economy; they aie pioblems of macioeconomics. The question about the level of econom-
ic activity, foi example, iefeis to the total value of all goods and seivices pioduced in the economy.
Infation is a measuie of the iate of change in the aveiage piice level foi the entiie economy; it is a mac-
ioeconomic pioblem. The total levels of employment and unemployment in the economy iepiesent the
aggiegate of all laboi maikets; unemployment is also a topic of macioeconomics.
Both micioeconomics and macioeconomics give attention to individual maikets. But in micioeco-
nomics that attention is an end in itself; in macioeconomics it is aimed at explaining the movement of
majoi economic aggiegates-the level of total output, the level of employment, and the piice level.
We have now examined the chaiacteiistics that defne the economic way of thinking and the two
bianches of this way of thinking: micioeconomics and macioeconomics. In the next section, we will
have a look at what one can do with tiaining in economics.
CHAP7R 1 CONOMICS: 7H S7UDY OI CHOIC 13
2.3 Putting Economics to Woik
Economics is one way of looking at the woild. Because the economic way of thinking has pioven quite
useful, tiaining in economics can be put to woik in a wide iange of felds. One, of couise, is in woik as
an economist. Undeigiaduate woik in economics can be applied to othei caieeis as well.
Careers in conomics
Economists woik in thiee types of oiganizations. About 38 of economists woik foi goveinment agen-
cies.
[1]
The iemaindei woik foi business fims oi in colleges and univeisities.
Economists woiking foi business fims and goveinment agencies sometimes foiecast economic
activity to assist theii employeis in planning. They also apply economic analysis to the activities of the
fims oi agencies foi which they woik oi consult. Economists employed at colleges and univeisities
teach and conduct ieseaich.
Peiuse the website of youi college oi univeisity's economics depaitment. Chances aie the depait-
ment will discuss the wide vaiiety of occupations that theii economics majois entei. Unlike engineei-
ing and accounting majois, economics and othei social science majois tend to be distiibuted ovei a
bioad iange of occupations.
AppIying conomics to Other IieIds
Suppose that you aie consideiing something othei than a caieei in economics. Would choosing to
study economics help you:
The evidence suggests it may. Suppose, foi example, that you aie consideiing law school. The
study of law iequiies keen analytical skills; studying economics shaipens such skills. Economists have
tiaditionally aigued that undeigiaduate woik in economics seives as excellent piepaiation foi law
school. Economist Michael Nieswiadomy of the Univeisity of Noith Texas collected data on Law
School Admittance Test (LSAT) scoies foi undeigiaduate majois listed by 2,200 oi moie students tak-
ing the test in 2003. Table 1.1 gives the scoies, as well as the ianking foi each of these majois, in 2003
and in two pievious yeais in which the iankings weie compiled. In iankings foi all thiee yeais, eco-
nomics majois iecoided the highest scoies.
7A8L 1. 1 LSA7 Scores and Undergraduate Ma|ors
|e.e ..e t|e .ve..ge |SA scc.es .rJ ..rk|rgs |c. t|e 12 urJe.g..Ju.te n.c.s .|t| nc.e t|.r 2200 stuJerts
t.k|rg t|e test tc erte. |.. sc|cc| |r t|e 20032004 .c.Jen|c ye...
Ma|or eId LSA7 average 2003-2004 2003-2004 Rank 1994-1995 Rank 1991-1992 Rank
|ccrcn|cs 156.6 1 1 1
|rg|ree.|rg 155.4 2 4 2
||stc.y 155.0 3 2 3
|rg||s| 154.3 4 3 4
||r.rce 152.6 5 6 5
|c||t|c.| sc|erce 152.1 6 9 9
|syc|c|cgy 152.1 8
Acccurt|rg 151.1 8 8 6
cnnur|c.t|crs 150.5 9 10 10
Scc|c|cgy 150.2 10 12 13
bus. AJn|r|st..t|cr 149.6 11 13 12
.|n|r.| 'ust|ce 144. 12 14 14
Source Michael ^ieswiadomy, LSAT Scores of Economics Majors 2003-2004 Class Update, journal of Economic Education, 37(2) (Spring 200)
244-247 and Michael ^ieswiadomy, LSAT Scores of Economics Majors journal of Economic Education, 29(4) (Fall 1998) 377-379.
Did the stiong peifoimance by economics, engineeiing, and histoiy majois mean that tiaining in those
felds shaipens analytical skills tested in the LSAT, oi that students with good analytical skills aie moie
likely to majoi in them: Both factois weie piobably at woik. Economics cleaily attiacts students with
good analytical skills-and studying economics helps develop those skills.
Economics majois shine in othei aieas as well. Accoiding to the Buieau of Laboi Statistics Occupa-
tional Outlook Handbook, a stiong backgiound in economic theoiy, mathematics, and statistics
piovides the basis foi competing foi the best job oppoitunities, paiticulaily ieseaich assistant positions,
in a bioad iange of felds. Many giaduates with bacheloi's degiees will fnd good jobs in industiy and
14 PRINCIPLS OI CONOMICS
business as management oi sales tiainees oi as administiative assistants. Because economists aie con-
ceined with undeistanding and inteipieting fnancial matteis, among othei subjects, they will also be
attiacted to and qualifed foi jobs as fnancial manageis, fnancial analysts, undeiwiiteis, actuaiies, se-
cuiities and fnancial seivices sales woikeis, ciedit analysts, loan and budget omceis, and uiban and ie-
gional planneis.
Table 1.2 shows aveiage yeaily salaiy ofeis foi bacheloi degiee candidates foi May 2006 and the
outlook foi ielated occupations to 2014.
7A8L 1. 2 Average YearIy SaIary Offers, May 2006 and OccupationaI OutIook 2004-2014, SeIected
Ma|ors/Occupations
Undergraduate ma|or Average $ Oer
May, 2006
Pro|ected % Change in 7otaI mpIoyment
in Occupation 2004-2014
cnpute. |rg|ree.|rg 54,200 10.1
||ect.|c.|/||ect.cr|c |rg|ree.|rg 54,053 11.8
cnpute. Sc|erce 50,892 25.6
Acccurt|rg 46,188 22.4
|ccrcn|cs .rJ ||r.rce 45,058 12.4
V.r.genert |r|c.n.t|cr Systens 44,55 25.9
|cg|st|cs .rJ V.te.|.|s V.r.genert 43,426 13.2
bus|ress AJn|r|st..t|cr 40,96 1.0
|rv|.crnert.| Sc|erces (|rc|uJ|rg |c.est.y
.rJ ccrse.v.t|cr sc|erce)
39,50 6.3
Ot|e. bus|ress V.c.s (e.g., V..ket|rg) 3,446 20.8
|un.r |escu.ces (|rc|. |.bc. |e|.t|crs) 36,256 15.9
Cec|cgy .rJ Cec|cg|c.| Sc|erces 35,034 8.3
Scc|c|cgy 33,52 4.
|c||t|c.| Sc|erce/Ccve.rnert 33,151 .3
||be..| A.ts Sc|erces (gere..| stuJ|es) 32,62 r.
|ub||c |e|.t|crs 32,623 21.
Spec|.| |Juc.t|cr 31,81 23.3
||enert..y |Juc.t|cr 31,8 18.2
|c.e|gr |.rgu.ges 31,364 r.
|ette.s (|rc|. |rg||s|) 31,204 20.4
Ot|e. Scc|.| Sc|erces (|rc|uJ|rg .|n|r.|
'ust|ce .rJ ||stc.y)
30,88 12.3
|syc|c|cgy 30,308 9.9
|.ee|enert..y |Juc.t|cr 2,550 22.4
Scc|.| \c.k 25,865 19.6
\|su.| .rJ |e.|c.n|rg A.ts 21,26 15.2
Sources ^ational Association of Colleges and Employers, Salary Survey, Spring 200 http//naceweb.org, Bureau of Labor Statistics, 200-2007 edition
of the Occupational Outlook Handbook, Occupational Employment, Training, and Earnings Educational Level Report (May, 200) URL
http//data.bls.gov/oep/noeted/empoptd.jsp (note na = not reported, that is, no specifc occupation was reported in BLS report, Other business majors,
Other social sciences, Social work (including Sociology), and Environmental Sciences are weighted averages of various disciplines, calculated by
authors.)
One's choice of a majoi, oi minoi, is not likely to be based solely on consideiations of potential eain-
ings oi the piospect of landing a spot in law school. You will also considei youi inteiests and abilities in
making a decision about whethei to puisue fuithei study in economics. And, of couise, you will con-
sidei the expected benefts of alteinative couises of study. What is your oppoitunity cost of puisuing
study of economics: Does studying moie economics seive youi inteiests and will doing so maximize
youi satisfaction level: These consideiations may be on youi mind as you begin to study economics at
the college level and obviously students will make many difeient choices. But, should you decide to
puisue a majoi oi minoi in economics, you should know that a backgiound in this feld is likely to
seive you well in a wide iange of caieeis.
CHAP7R 1 CONOMICS: 7H S7UDY OI CHOIC 15
k Y 7 A k A W A Y S
< |ccrcn|sts |ccus cr t|e cppc.tur|ty ccsts c| c|c|ces, t|ey .ssune t|.t |rJ|v|Ju.|s n.ke c|c|ces |r . ..y
t|.t n.x|n|.es t|e v.|ue c| .r cbect|ve Je|reJ |r te.ns c| t|e|. c.r se|||rte.est, .rJ t|ey .ssune t|.t
|rJ|v|Ju.|s n.ke t|cse c|c|ces .t t|e n..g|r.
< |ccrcn|cs |s J|v|JeJ |rtc t.c b.c.J ..e.s. n|c.ceccrcn|cs .rJ n.c.ceccrcn|cs.
< A .|Je ..rge c| c..ee. cppc.tur|t|es |s cper tc eccrcn|cs n.c.s. |np|.|c.| ev|Jerce suggests t|.t
stuJerts .|c erte. t|e cb n..ket .|t| . n.c. |r eccrcn|cs terJ tc e..r nc.e t|.r Jc stuJerts |r ncst
ct|e. n.c.s. |u.t|e., eccrcn|cs n.c.s Jc p..t|cu|..|y .e|| cr t|e |SA.
7 R Y I 7 !
|e |ep..tnert c| Ag.|cu|tu.e est|n.teJ t|.t t|e experJ|tu.es . n|JJ|e|rccne, |usb.rJ.||e |.n||y c|
t|.ee .cu|J |rcu. tc ..|se cre .JJ|t|cr.| c|||J |.cn b|.t| |r 2005 tc .ge 1 .cu|J be 250,530. |r .|.t ..y
Jces t||s est|n.te |||ust..te t|e eccrcn|c ..y c| t||rk|rg` \cu|J t|e |ep..tnerts est|n.te be .r ex.np|e
c| n|c.ceccrcn|c c. c| n.c.ceccrcn|c .r.|ys|s` \|y`
Case in Point: 7he IinanciaI Payoff to Studying conomics
2010 jupiterimages Corporation
c||ege eccrcn|cs p.c|essc.s |.ve |crg ..gueJ t|.t stuJy|rg eccrcn|cs |s gccJ p.ep...t|cr |c. . v..|ety c|
c..ee.s. A .ecert stuJy suggests t|ey ..e .|g|t .rJ t|.t stuJy|rg eccrcn|cs |s ever ||ke|y tc n.ke stuJerts
nc.e p.cspe.cus. StuJerts .|c n.c. |r eccrcn|cs but J|J rct pu.sue g..Ju.te .c.k ..e ||ke|y tc e..r nc.e
t|.r stuJerts |r v|.tu.||y eve.y ct|e. cc||ege n.c.. StuJerts .|c n.c. |r eccrcn|cs .rJ t|er gc cr tc |..
sc|cc| c. .r VbA p.cg..n ..e ||ke|y tc e..r nc.e t|.r stuJerts .|c .pp.c.c| t|cse ..e.s c| stuJy |.v|rg
n.c.eJ |r ncst ct|e. ..e.s.
|ccrcn|sts |.r A. b|.ck, Set| S.rJe.s, .rJ |c.e|| .y|c. useJ t|e 1993 |.t|cr.| Su.vey c| c||ege C..Ju.tes,
.||c| |rc|uJeJ nc.e t|.r 86,000 cc||egeeJuc.teJ .c.ke.s bet.eer t|e .ges c| 25 .rJ 55 t|.t .skeJ .|.t
|e|J t|ey |.J n.c.eJ |r. |ey t|er ccrt.c||eJ |c. v..|.b|es suc| .s gerJe., ..ce, .rJ et|r|c|ty. |ey |curJ
t|.t stuJerts .|c |.J rct Jcre g..Ju.te .c.k .rJ |.J n.c.eJ |r eccrcn|cs e..reJ nc.e t|.r stuJerts |r
.ry ct|e. n.c. except erg|ree.|rg. Spec||c.||y, eccrcn|cs n.c.s e..reJ .bcut 13 nc.e t|.r ct|e. scc|.|
sc|erces n.c.s, 11 nc.e t|.r bus|ress .Jn|r|st..t|cr n.c.s, .rJ .bcut t|e s.ne .s r.tu..| sc|erce .rJ
.cccurt|rg n.c.s. |e eccrcn|cs n.c.s |r t|e|. su.vey, ||ke t|cse .|c n.c.eJ |r ct|e. scc|.| sc|erces .rJ
bus|ress .Jn|r|st..t|cr .rJ ur||ke t|cse .|c n.c.eJ |r erg|ree.|rg c. .cccurt|rg, .e.e sp.e.J cut cve. .
.|Je ..rge c| cccup.t|crs but .|t| n.ry |r n.r.genert pcs|t|crs.
16 PRINCIPLS OI CONOMICS
variabIe
Scnet||rg .|cse v.|ue c.r
c|.rge.
constant
Scnet||rg .|cse v.|ue Jces
rct c|.rge.
b.seJ cr t|e su.vey t|ey useJ, cve. 40 c| eccrcn|cs n.c.s .ert cr tc e..r g..Ju.te Jeg.ees, n.ry |r |..
.rJ bus|ress. |ccrcn|cs n.c.s ..rkeJ |.st |r te.ns c| ..ges, .s ccnp..eJ tc ct|e. |.. sc|cc| g..Ju.tes
.|t| t|e 12 ncst ccnncr p.e|.. n.c.s (|rc|uJ|rg suc| n.c.s .s bus|ress .Jn|r|st..t|cr, |r.rce, |rg||s|,
||stc.y, psyc|c|cgy, .rJ pc||t|c.| sc|erce). VbA g..Ju.tes .|c |.J n.c.eJ |r eccrcn|cs e..reJ nc.e t|.r
t|cse .|c |.J n.c.eJ |r .ry ct|e. |e|J except c|en|c.| erg|ree.|rg. Spec||c.||y, urJe.g..Ju.te eccrcn
|cs n.c.s .|t| VbAs e..reJ .bcut 15 nc.e t|.r t|cse .|c |.J n.c.eJ |r ct|e. J|sc|p||res .ep.eserteJ |r
t|e su.vey, |rc|uJ|rg bus|ress.e|.teJ n.c.s.
|t |s .en..k.b|e t|.t .|| c| t|e bus|ress.e|.teJ n.c.s gere..teJ s.|..|es nuc| |c.e. t|.r t|cse e..reJ by
eccrcn|cs n.c.s .|t| .r VbA. Ore ccu|J ..gue t|.t t||s .e|ects se||se|ect|cr, t|.t stuJerts .|c n.c. |r
eccrcn|cs ..e s|np|y b.|g|te.. but, stuJerts .|c n.c. |r p|ys|cs |.ve ||g| SA scc.es, yet t|ey, tcc, e..reJ
..ges t|.t .e.e .bcut 20 |c.e. t|.r VbA stuJerts .|c |.J n.c.eJ |r eccrcn|cs. ||s |rJ|rg |erJs
scne c.eJerce tc t|e rct|cr t|.t t|e n..ketp|.ce .e...Js t..|r|rg |r t|e eccrcn|c ..y c| t||rk|rg.
.ooce |o ^ ||oc|. .e|| .oJe. oJ |o.e|| o,|o. |e |coo|c |e.oJ |o .|oJ,| |coo|c.' |coo|c |o|,. 4(3:. .o|, 2003. 3653..
A N S W R 7 O 7 R Y I 7 ! P R O 8 L M
|e |r|c.n.t|cr g|ver suggests cre e|enert c| t|e eccrcn|c ..y c| t||rk|rg. .ssess|rg t|e c|c|ce .t t|e
n..g|r. |e est|n.te .e|ects t|e ccst c| cre nc.e c|||J |c. . |.n||y t|.t .|.e.Jy |.s cre. |t |s rct c|e.. |.cn
t|e |r|c.n.t|cr g|ver |c. c|cse t|e est|n.te c| ccst ccnes tc t|e eccrcn|c ccrcept c| cppc.tur|ty ccst.
|e |ep..tnert c| Ag.|cu|tu.es est|n.te |rc|uJeJ suc| ccsts .s |cus|rg, |ccJ, t..rspc.t.t|cr, c|ct||rg,
|e.|t| c..e, c|||J c..e, .rJ eJuc.t|cr. Ar eccrcn|st .cu|J .JJ t|e v.|ue c| t|e best .|te.r.t|ve use c| t|e .J
J|t|cr.| t|ne t|.t .||| be .e,u|.eJ |c. t|e c|||J. || t|e ccup|e |s |cck|rg |.. .|e.J, |t n.y ..rt tc ccrs|Je. t|e
cppc.tur|ty ccst c| serJ|rg . c|||J tc cc||ege. ArJ, || |t |s |cck|rg .e, |.. .|e.J, |t n.y ..rt tc ccrs|Je. t|e
|.ct t|.t re..|y |.|| c| .|| p..erts cve. t|e .ge c| 50 suppc.t .t |e.st cre c|||J cve. t|e .ge c| 21. ||s |s .
p.cb|en |r n|c.ceccrcn|c .r.|ys|s, bec.use |t |ccuses cr t|e c|c|ces c| |rJ|v|Ju.| |cuse|c|Js.
3. THE ECONOMISTS' TOOL KIT
L A R N I N G O 8 1 C 7 I V S
1. xpIain how economists test hypotheses, deveIop economic theories, and use modeIs in their
anaIyses.
2. xpIain how the aII-other-things unchanged (ceteris paribus) probIem and the faIIacy of faIse
cause aect the testing of economic hypotheses and how economists try to overcome these
probIems.
3. Distinguish between normative and positive statements.
Economics difeis fiom othei social sciences because of its emphasis on oppoitunity cost, the assump-
tion of maximization in teims of one's own self-inteiest, and the analysis of choices at the maigin. But
ceitainly much of the basic methodology of economics and many of its dimculties aie common to
eveiy social science-indeed, to eveiy science. This section exploies the application of the scientifc
method to economics.
Reseaicheis often examine ielationships between vaiiables. A variable is something whose value
can change. By contiast, a constant is something whose value does not change. The speed at which a
cai is tiaveling is an example of a vaiiable. The numbei of minutes in an houi is an example of a
constant.
CHAP7R 1 CONOMICS: 7H S7UDY OI CHOIC 17
scientic method
A systen.t|c set c|
p.cceJu.es t|.cug| .||c|
krc.|eJge |s c.e.teJ.
hypothesis
Ar .sse.t|cr c| . .e|.t|crs||p
bet.eer t.c c. nc.e
v..|.b|es t|.t ccu|J be
p.cver tc be |.|se.
theory
A |ypct|es|s t|.t |.s rct
beer .eecteJ .|te.
.|Jesp.e.J test|rg .rJ t|.t
.|rs gere..| .ccept.rce.
Iaw
A t|ec.y t|.t |.s beer
subecteJ tc ever nc.e
test|rg .rJ t|.t |.s .cr
v|.tu.||y ur|ve.s.| .ccept.rce.
modeI
A set c| s|np|||y|rg
.ssunpt|crs .bcut scne
.spect c| t|e .e.| .c.|J.
ceteris paribus
A |.t|r p|..se t|.t ne.rs,
.|| ct|e. t||rgs urc|.rgeJ.
Reseaich is geneially conducted within a fiamewoik called the scientinc method, a systematic
set of pioceduies thiough which knowledge is cieated. In the scientifc method, hypotheses aie sugges-
ted and then tested. A hypothesis is an asseition of a ielationship between two oi moie vaiiables that
could be pioven to be false. A statement is not a hypothesis if no conceivable test could show it to be
false. The statement Plants like sunshine" is not a hypothesis; theie is no way to test whethei plants
like sunshine oi not, so it is impossible to piove the statement false. The statement Incieased solai ia-
diation incieases the iate of plant giowth" is a hypothesis; expeiiments could be done to show the iela-
tionship between solai iadiation and plant giowth. If solai iadiation weie shown to be unielated to
plant giowth oi to ietaid plant giowth, then the hypothesis would be demonstiated to be false.
If a test ieveals that a paiticulai hypothesis is false, then the hypothesis is iejected oi modifed. In
the case of the hypothesis about solai iadiation and plant giowth, we would piobably fnd that moie
sunlight incieases plant giowth ovei some iange but that too much can actually ietaid plant giowth.
Such iesults would lead us to modify oui hypothesis about the ielationship between solai iadiation and
plant giowth.
If the tests of a hypothesis yield iesults consistent with it, then fuithei tests aie conducted. A hypo-
thesis that has not been iejected aftei widespiead testing and that wins geneial acceptance is commonly
called a theory. A theoiy that has been subjected to even moie testing and that has won viitually uni-
veisal acceptance becomes a law. We will examine two economic laws in the next two chapteis.
Even a hypothesis that has achieved the status of a law cannot be pioven tiue. Theie is always a
possibility that someone may fnd a case that invalidates the hypothesis. That possibility means that
nothing in economics, oi in any othei social science, oi in any science, can evei be proven tiue. We can
have gieat confdence in a paiticulai pioposition, but it is always a mistake to asseit that it is pioven."
3.1 Models in Economics
All scientifc thought involves simplifcations of ieality. The ieal woild is fai too complex foi the hu-
man mind-oi the most poweiful computei-to considei. Scientists use models instead. A model is a
set of simplifying assumptions about some aspect of the ieal woild. Models aie always based on as-
sumed conditions that aie simplei than those of the ieal woild, assumptions that aie necessaiily false.
A model of the ieal woild cannot be the ieal woild.
We will encountei oui fist economic model in Chaptei 33. Foi that model, we will assume that an
economy can pioduce only two goods. Then we will exploie the model of demand and supply. One of
the assumptions we will make theie is that all the goods pioduced by fims in a paiticulai maiket aie
identical. Of couise, ieal economies and ieal maikets aie not that simple. Reality is nevei as simple as a
model; one point of a model is to simplify the woild to impiove oui undeistanding of it.
Economists often use giaphs to iepiesent economic models. The appendix to this chaptei piovides
a quick, iefieshei couise, if you think you need one, on undeistanding, building, and using giaphs.
Models in economics also help us to geneiate hypotheses about the ieal woild. In the next section,
we will examine some of the pioblems we encountei in testing those hypotheses.
3.2 Testing Hypotheses in Economics
Heie is a hypothesis suggested by the model of demand and supply: an inciease in the piice of gasoline
will ieduce the quantity of gasoline consumeis demand. How might we test such a hypothesis:
Economists tiy to test hypotheses such as this one by obseiving actual behavioi and using empiiic-
al (that is, ieal-woild) data. The aveiage ietail piice of gasoline in the United States iose fiom an avei-
age of $2.12 pei gallon on May 22, 2003 to $2.88 pei gallon on May 22, 2006. The numbei of gallons of
gasoline consumed by U.S. motoiists iose 0.3 duiing that peiiod.
The small inciease in the quantity of gasoline consumed by motoiists as its piice iose is inconsist-
ent with the hypothesis that an incieased piice will lead to an ieduction in the quantity demanded.
Does that mean that we should dismiss the oiiginal hypothesis: On the contiaiy, we must be cautious
in assessing this evidence. Seveial pioblems exist in inteipieting any set of economic data. One piob-
lem is that seveial things may be changing at once; anothei is that the initial event may be unielated to
the event that follows. The next two sections examine these pioblems in detail.
7he AII-Other-7hings-Unchanged ProbIem
The hypothesis that an inciease in the piice of gasoline pioduces a ieduction in the quantity demanded
by consumeis caiiies with it the assumption that theie aie no othei changes that might also afect con-
sumei demand. A bettei statement of the hypothesis would be: An inciease in the piice of gasoline will
ieduce the quantity consumeis demand, ceteiis paiibus. Ceteris paribus is a Latin phiase that means
all othei things unchanged."
18 PRINCIPLS OI CONOMICS
dependent variabIe
A v..|.b|e t|.t .espcrJs tc
c|.rge.
independent variabIe
A v..|.b|e t|.t |rJuces .
c|.rge.
faIIacy of faIse cause
|e |rcc..ect .ssunpt|cr
t|.t cre evert c.uses
.rct|e. bec.use t|e t.c
everts terJ tc cccu.
tcget|e..
positive statement
A st.tenert c| |.ct c. .
|ypct|es|s.
normative statement
A st.tenert t|.t n.kes .
v.|ue uJgnert.
But things changed between May 2003 and May 2006. Economic activity and incomes iose both in
the United States and in many othei countiies, paiticulaily China, and people with highei incomes aie
likely to buy moie gasoline. Employment iose as well, and people with jobs use moie gasoline as they
diive to woik. Population in the United States giew duiing the peiiod. In shoit, many things happened
duiing the peiiod, all of which tended to inciease the quantity of gasoline people puichased.
Oui obseivation of the gasoline maiket between May 2003 and May 2006 did not ofei a conclus-
ive test of the hypothesis that an inciease in the piice of gasoline would lead to a ieduction in the
quantity demanded by consumeis. Othei things changed and afected gasoline consumption. Such
pioblems aie likely to afect any analysis of economic events. We cannot ask the woild to stand still
while we conduct expeiiments in economic phenomena. Economists employ a vaiiety of statistical
methods to allow them to isolate the impact of single events such as piice changes, but they can nevei
be ceitain that they have accuiately isolated the impact of a single event in a woild in which viitually
eveiything is changing all the time.
In laboiatoiy sciences such as chemistiy and biology, it is ielatively easy to conduct expeiiments in
which only selected things change and all othei factois aie held constant. The economists' laboiatoiy is
the ieal woild; thus, economists do not geneially have the luxuiy of conducting contiolled
expeiiments.
7he IaIIacy of IaIse Cause
Hypotheses in economics typically specify a ielationship in which a change in one vaiiable causes an-
othei to change. We call the vaiiable that iesponds to the change the dependent variable; the vaiiable
that induces a change is called the independent variable. Sometimes the fact that two vaiiables move
togethei can suggest the false conclusion that one of the vaiiables has acted as an independent vaiiable
that has caused the change we obseive in the dependent vaiiable.
Considei the following hypothesis: People weaiing shoits cause waim weathei. Ceitainly, we ob-
seive that moie people weai shoits when the weathei is waim. Piesumably, though, it is the waim
weathei that causes people to weai shoits iathei than the weaiing of shoits that causes waim weathei;
it would be incoiiect to infei fiom this that people cause waim weathei by weaiing shoits.
Reaching the incoiiect conclusion that one event causes anothei because the two events tend to
occui togethei is called the fallacy of false cause. The accompanying essay on baldness and heait dis-
ease suggests an example of this fallacy.
Because of the dangei of the fallacy of false cause, economists use special statistical tests that aie
designed to deteimine whethei changes in one thing actually do cause changes obseived in anothei.
Given the inability to peifoim contiolled expeiiments, howevei, these tests do not always ofei convin-
cing evidence that peisuades all economists that one thing does, in fact, cause changes in anothei.
In the case of gasoline piices and consumption between May 2003 and May 2006, theie is good
theoietical ieason to believe the piice inciease should lead to a ieduction in the quantity consumeis de-
mand. And economists have tested the hypothesis about piice and the quantity demanded quite ex-
tensively. They have developed elaboiate statistical tests aimed at iuling out pioblems of the fallacy of
false cause. While we cannot piove that an inciease in piice will, ceteiis paiibus, lead to a ieduction in
the quantity consumeis demand, we can have consideiable confdence in the pioposition.
Normative and Positive Statements
Two kinds of asseitions in economics can be subjected to testing. We have alieady examined one, the
hypothesis. Anothei testable asseition is a statement of fact, such as It is iaining outside" oi
Miciosoft is the laigest pioducei of opeiating systems foi peisonal computeis in the woild." Like hy-
potheses, such asseitions can be demonstiated to be false. Unlike hypotheses, they can also be shown to
be coiiect. A statement of fact oi a hypothesis is a positive statement.
Although people often disagiee about positive statements, such disagieements can ultimately be
iesolved thiough investigation. Theie is anothei categoiy of asseitions, howevei, foi which investiga-
tion can nevei iesolve difeiences. A normative statement is one that makes a value judgment. Such
a judgment is the opinion of the speakei; no one can piove" that the statement is oi is not coiiect.
Heie aie some examples of noimative statements in economics: We ought to do moie to help the
pooi." People in the United States should save moie." Coipoiate piofts aie too high." The state-
ments aie based on the values of the peison who makes them. They cannot be pioven false.
Because people have difeient values, noimative statements often piovoke disagieement. An eco-
nomist whose values lead him oi hei to conclude that we should piovide moie help foi the pooi will
disagiee with one whose values lead to a conclusion that we should not. Because no test exists foi these
values, these two economists will continue to disagiee, unless one peisuades the othei to adopt a difei-
ent set of values. Many of the disagieements among economists aie based on such difeiences in values
and theiefoie aie unlikely to be iesolved.
CHAP7R 1 CONOMICS: 7H S7UDY OI CHOIC 19
k Y 7 A k A W A Y S
< |ccrcn|sts t.y tc enp|cy t|e sc|ert||c net|cJ |r t|e|. .ese..c|.
< Sc|ert|sts c.rrct p.cve . |ypct|es|s tc be t.ue, t|ey c.r cr|y |.|| tc p.cve |t |.|se.
< |ccrcn|sts, ||ke ct|e. scc|.| sc|ert|sts .rJ sc|ert|sts, use ncJe|s tc .ss|st t|en |r t|e|. .r.|yses.
< .c p.cb|ens |r|e.ert |r tests c| |ypct|eses |r eccrcn|cs ..e t|e .||ct|e.t||rgsurc|.rgeJ p.cb|en
.rJ t|e |.||.cy c| |.|se c.use.
< |cs|t|ve st.tenerts ..e |.ctu.| .rJ c.r be testeJ. |c.n.t|ve st.tenerts ..e v.|ue uJgnerts t|.t c.rrct
be testeJ. V.ry c| t|e J|s.g.eenerts .ncrg eccrcn|sts sten |.cn J||e.erces |r v.|ues.
7 R Y I 7 !
|cck .g.|r .t t|e J.t. |r .b|e 1.1. |c. ccrs|Je. t|e |ypct|es|s. V.c.|rg |r eccrcn|cs .||| .esu|t |r . ||g|e.
|SA scc.e. A.e t|e J.t. g|ver ccrs|stert .|t| t||s |ypct|es|s` |c t|e J.t. p.cve t|.t t||s |ypct|es|s |s cc.
.ect` \|.t |.||.cy n|g|t be |rvc|veJ |r .ccept|rg t|e |ypct|es|s`
Case in Point: Does 8aIdness Cause Heart Disease1
2010 jupiterimages Corporation
A .ebs|te c.||eJ enb....ss|rgp.cb|ens.ccn .ece|veJ t|e |c||c.|rg en.||.
|eo | /ooe|.
| ee |o e o| o|J ^ccoJ| |o ,oo .e||e. ||| eo | oe |||e|, |o |o.e o
|eo| o||oc| || | |o|e o Jo |o e.e| |o| |o. .||| || eJoce , || o| o |eo| o||oc|.'
20 PRINCIPLS OI CONOMICS
\|.t J|J |.. V..g..et .rs.e.` Vcst |npc.t.rt|y, s|e J|J rct .eccnnerJ t|.t t|e ,uest|cre. t.ke J.ugs
tc t.e.t ||s b.|Jress, bec.use Jcctc.s Jc rct t||rk t|.t t|e b.|Jress c.uses t|e |e..t J|se.se. A nc.e ||ke|y
exp|.r.t|cr |c. t|e .sscc|.t|cr bet.eer b.|Jress .rJ |e..t J|se.se |s t|.t bct| ccrJ|t|crs ..e .|ecteJ by .r
urJe.|y|rg |.ctc.. \|||e rct|rg t|.t nc.e .ese..c| reeJs tc be Jcre, cre |ypct|es|s t|.t |.. V..g..et c|e.s
|s t|.t ||g|e. testcste.cre |eve|s n|g|t be t.|gge.|rg bct| t|e |.|. |css .rJ t|e |e..t J|se.se. |e gccJ re.s
|c. pecp|e .|t| e..|y b.|J|rg (.||c| |s .e.||y .|e.e t|e .sscc|.t|cr .|t| |rc.e.seJ .|sk c| |e..t J|se.se |.s
beer cbse.veJ) |s t|.t t|ey |.ve . s|gr.| t|.t n|g|t |e.J t|en tc be c|eckeJ e..|y cr |c. |e..t J|se.se.
.ooce |||...eoo|o|ecoo|eo|eoe230.0||
A N S W R 7 O 7 R Y I 7 ! P R O 8 L M
|e J.t. ..e ccrs|stert .|t| t|e |ypct|es|s, but |t |s reve. pcss|b|e tc p.cve t|.t . |ypct|es|s |s cc..ect. Ac
cept|rg t|e |ypct|es|s ccu|J |rvc|ve t|e |.||.cy c| |.|se c.use, stuJerts .|c n.c. |r eccrcn|cs n.y .|.e.Jy
|.ve t|e .r.|yt|c.| sk|||s reeJeJ tc Jc .e|| cr t|e ex.n.
4. REVIEW AND PRACTICE
Summary
|c|ces ..e |c.ceJ cr us by sc..c|ty, eccrcn|sts stuJy t|e c|c|ces t|.t pecp|e n.ke. Sc..ce gccJs ..e t|cse
|c. .||c| t|e c|c|ce c| cre .|te.r.t|ve .e,u|.es g|v|rg up .rct|e.. |e cppc.tur|ty ccst c| .ry c|c|ce |s t|e
v.|ue c| t|e best .|te.r.t|ve |c.gcre |r n.k|rg t|.t c|c|ce.
Scne key c|c|ces .ssesseJ by eccrcn|sts |rc|uJe .|.t tc p.cJuce, |c. tc p.cJuce |t, .rJ |c. .|cn |t
s|cu|J be p.cJuceJ. |ccrcn|cs |s J|st|rgu|s|eJ |.cn ct|e. .c.Jen|c J|sc|p||res t|.t .|sc stuJy c|c|ces by .r
enp|.s|s cr t|e cert..| |npc.t.rce c| cppc.tur|ty ccsts |r ev.|u.t|rg c|c|ces, t|e .ssunpt|cr c| n.x|n|.|rg
be|.v|c. t|.t se.ves t|e |rte.ests c| |rJ|v|Ju.| Jec|s|cr n.ke.s, .rJ . |ccus cr ev.|u.t|rg c|c|ces .t t|e
n..g|r.
|ccrcn|c .r.|yses n.y be .|neJ .t exp|.|r|rg |rJ|v|Ju.| c|c|ce c. c|c|ces |r .r |rJ|v|Ju.| n..ket, suc| |r
vest|g.t|crs ..e |..ge|y t|e |ccus c| n|c.ceccrcn|cs. |e .r.|ys|s c| t|e |np.ct c| t|cse |rJ|v|Ju.| c|c|ces cr
suc| .gg.eg.tes .s tct.| cutput, t|e |eve| c| enp|cynert, .rJ t|e p.|ce |eve| |s t|e ccrce.r c|
n.c.ceccrcn|cs.
\c.k|rg .|t||r t|e |..ne.c.k c| t|e sc|ert||c net|cJ, eccrcn|sts |c.nu|.te |ypct|eses .rJ t|er test t|en.
|ese tests c.r cr|y .e|ute . |ypct|es|s, |ypct|eses |r sc|erce c.rrct be p.cveJ. A |ypct|es|s t|.t |.s beer
.|Je|y testeJ c|ter ccnes tc be .eg..JeJ .s . t|ec.y, cre t|.t |.s .cr v|.tu.||y ur|ve.s.| .ccept.rce |s . |...
bec.use c| t|e ccnp|ex|ty c| t|e .e.| .c.|J, eccrcn|sts .e|y cr ncJe|s t|.t .est cr . se.|es c| s|np|||y|rg .s
sunpt|crs. |e ncJe|s ..e useJ tc gere..te |ypct|eses .bcut t|e eccrcny t|.t c.r be testeJ us|rg .e.|
.c.|J J.t..
St.tenerts c| |.ct .rJ |ypct|eses ..e pcs|t|ve st.tenerts. |c.n.t|ve st.tenerts, ur||ke pcs|t|ve st.tenerts,
c.rrct be testeJ .rJ p.cv|Je . scu.ce |c. pctert|.| J|s.g.eenert.
CHAP7R 1 CONOMICS: 7H S7UDY OI CHOIC 21
P R O 8 L M S
1. \|y Jces t|e |.ct t|.t scnet||rg |s sc..ce .e,u|.e t|.t .e n.ke c|c|ces`
2. |ces t|e |.ct t|.t scnet||rg |s .burJ.rt ne.r |t |s rct sc..ce |r t|e eccrcn|c serse` \|y c. .|y rct`
3. |r scne ccurt.|es, suc| .s ub. .rJ |c.t| |c.e., t|e gcve.rnert n.kes ncst c| t|e Jec|s|crs .bcut
.|.t .||| be p.cJuceJ, |c. |t .||| be p.cJuceJ, .rJ |c. .|cn. |ces t|e |.ct t|.t t|ese c|c|ces ..e n.Je
by t|e gcve.rnert e||n|r.te sc..c|ty |r t|ese ccurt.|es` \|y c. .|y rct`
4. |xp|.|r .|.t |s ne.rt by t|e cppc.tur|ty ccst c| . c|c|ce.
5. \|.t |s t|e .pp.cx|n.te Jc||.. ccst c| t|e tu|t|cr .rJ ct|e. |ees .sscc|.teJ .|t| t|e eccrcn|cs ccu.se
ycu ..e t.k|rg` |ces t||s Jc||.. ccst |u||y .e|ect t|e cppc.tur|ty ccst tc ycu c| t.k|rg t|e ccu.se`
6. |r t|e .se |r |c|rt ess.y |e ||s|rg cst c| |re.gy, .|.t .cu|J be scne c| t|e t||rgs t|.t .cu|J be
|rc|uJeJ |r .r est|n.te c| t|e cppc.tur|ty ccst c| p.ese.v|rg p..t c| rc.t|e.r A|be.t. .r.J. by
p.c||b|t|rg |e.vy c.uJe c|| ext..ct|cr` |c ycu t||rk t|.t t|e |rc.e.seJ ext..ct|cr .ep.eserts t|e best use
c| t|e |.rJ` \|y c. .|y rct`
. |rJ|c.te .|et|e. e.c| c| t|e |c||c.|rg |s . tcp|c c| n|c.ceccrcn|cs c. n.c.ceccrcn|cs.
.. |e |np.ct c| ||g|e. c|| p.|ces cr t|e p.cJuct|cr c| stee|
b. |e |rc.e.seJ Jen.rJ |r t|e |.st 15 ye..s |c. exct|c J|et..y supp|enerts
c. |e su.ge |r .gg.eg.te eccrcn|c .ct|v|ty t|.t ||t nuc| c| As|. |.te |r t|e e..|y 2000s
J. |e s|..p |rc.e.ses |r |.S. enp|cynert .rJ tct.| cutput t|.t cccu..eJ bet.eer 2003 .rJ 200
e. |e |np.ct c| p.ese.v.t|cr c| .||Je.ress ..e.s cr t|e |cgg|rg |rJust.y .rJ cr t|e p.|ce c|
|unbe.
8. |ete.n|re .|et|e. e.c| c| t|e |c||c.|rg ..|ses . .|.t, |c., c. |c. .|cn |ssue. A.e t|e st.tenerts
rc.n.t|ve c. pcs|t|ve`
.. A .e,u|.enert t|.t .|un|run useJ |r c..s be n.Je |.cn .ecyc|eJ n.te.|.|s .||| ..|se t|e p.|ce c|
.utcncb||es.
b. |e |eJe..| gcve.rnert Jces rct sperJ ercug| |c. c|||J.er.
c. Ar |rc.e.se |r pc||ce .escu.ces p.cv|JeJ tc t|e |rre. c|ty .||| |c.e. t|e c.|ne ..te.
J. Autcn.t|cr Jest.cys cbs.
e. ||c.ts tc |np.cve t|e erv|.crnert terJ tc .eJuce p.cJuct|cr .rJ enp|cynert.
|. '.p.rese |.ns s|cu|J be nc.e .||||rg tc ||.e .JJ|t|cr.| .c.ke.s .|er p.cJuct|cr .|ses .rJ tc
|.y c| .c.ke.s .|er p.cJuct|cr |.||s.
g. Access tc |e.|t| c..e s|cu|J rct be ||n|teJ by |rccne.
9. +cu. t|ne |s . sc..ce .escu.ce. \|.t || t|e ,u.rt|ty c| t|ne .e.e |rc.e.seJ, s.y tc 48 |cu.s pe. J.y, .rJ
eve.ycre st||| ||veJ .s n.ry J.ys .s be|c.e. \cu|J t|ne st||| be sc..ce`
10. Vcst cc||ege stuJerts ..e urJe. .ge 25. C|ve t.c exp|.r.t|crs |c. t||scre b.seJ cr t|e bere|ts
pecp|e c| J||e.ert .ges ..e ||ke|y tc .ece|ve |.cn ||g|e. eJuc.t|cr .rJ cre b.seJ cr t|e cppc.tur|ty
ccsts c| . cc||ege eJuc.t|cr tc stuJerts c| J||e.ert .ges.
11. Scne nur|c|p.| ..te. ccnp.r|es c|..ge custcne.s . |.t |ee e.c| ncrt|, .eg..J|ess c| t|e .ncurt c|
..te. t|ey ccrsune. Ot|e.s nete. ..te. use .rJ c|..ge .ccc.J|rg tc t|e ,u.rt|ty c| ..te. custcne.s
use. cnp..e t|e ..y t|e t.c systens .|ect t|e ccst c| ..te. use .t t|e n..g|r.
12. |c. n|g|t ycu test e.c| c| t|e |c||c.|rg |ypct|eses` Suggest scne p.cb|ens t|.t n|g|t ..|se |r e.c|
test Jue tc t|e cete.|s p..|bus (.||ct|e.t||rgsurc|.rgeJ) p.cb|en .rJ t|e |.||.cy c| |.|se c.use.
.. |eJuc|rg t|e ,u.rt|ty c| |e.c|r .v.||.b|e .||| |rc.e.se tct.| sperJ|rg cr |e.c|r .rJ |rc.e.se t|e
c.|ne ..te.
b. ||g|e. |rccnes n.ke pecp|e |.pp|e..
c. ||g|e. |rccnes n.ke pecp|e ||ve |crge..
13. V.ry ncJe|s |r p|ys|cs .rJ |r c|en|st.y .ssune t|e ex|sterce c| . pe.|ect v.cuun (t|.t |s, . sp.ce
ert|.e|y enpty c| n.tte.). +et .e krc. t|.t . pe.|ect v.cuun c.rrct ex|st. A.e suc| ncJe|s v.||J` \|y
..e ncJe|s b.seJ cr .ssunpt|crs t|.t ..e essert|.||y |rcc..ect`
14. Suppcse ycu .e.e .skeJ tc test t|e p.cpcs|t|cr t|.t pub||s||rg stuJerts te.c|e. ev.|u.t|crs c.uses
g..Je |r|.t|cr. \|.t ev|Jerce n|g|t ycu ..rt tc ccrs|Je.` |c. .cu|J t|e |r.b|||ty tc c...y cut
ccrt.c||eJ expe.|nerts n.ke ycu. .r.|ys|s nc.e J||cu|t`
15. |e|e..|rg tc t|e .se |r |c|rt b.|Jress .rJ |e..t ||se.se, exp|.|r t|e pcss|b|e |.||.cy c| |.|se c.use |r
ccrc|uJ|rg t|.t b.|Jress n.kes . pe.scr nc.e ||ke|y tc |.ve |e..t J|se.se.
16. |r 2005 t|e |ccJ .rJ |.ug AJn|r|st..t|cr c.Je.eJ t|.t \|cxx .rJ ct|e. pcpu|.. J.ugs |c. t.e.t|rg t|e
p.|r c| ..t|.|t|s be .|t|J...r |.cn t|e n..ket. |e c.Je. .esu|teJ |.cn . |rJ|rg t|.t pecp|e t.k|rg t|e
J.ugs |.J .r |rc.e.seJ .|sk c| c..J|cv.scu|.. p.cb|ens. Scne .ese..c|e.s c.|t|c|.eJ t|e gcve.rnerts
.ct|cr, ..gu|rg t|.t ccrc|uJ|rg t|.t t|e J.ugs c.useJ t|e c..J|cv.scu|.. p.cb|ens .ep.eserteJ .r
ex.np|e c| t|e |.||.cy c| |.|se c.use. .r ycu t||rk c| .ry .e.scr .|y t||s n|g|t be t|e c.se`
22 PRINCIPLS OI CONOMICS
1.
ENDNOTES
bu.e.u c| |.bc. St.t|st|cs ccoo||oo| o||oo| .t |ttp.//....b|s.gcv/ccc/.
CHAP7R 1 CONOMICS: 7H S7UDY OI CHOIC 23
24 PRINCIPLS OI CONOMICS
production possibiIities
modeI
VcJe| t|.t s|c.s t|e gccJs
.rJ se.v|ces t|.t .r eccrcny
|s c.p.b|e c| p.cJuc|rg|ts
pcss|b|||t|esg|ver t|e
|.ctc.s c| p.cJuct|cr .rJ t|e
tec|rc|cgy |t |.s .v.||.b|e.
economic system
|e set c| .u|es t|.t Je|re
|c. .r eccrcnys .escu.ces
..e tc be c.reJ .rJ |c.
Jec|s|crs .bcut t|e|. use ..e
tc be n.Je.
| A | | | 2
Confronting Scarcity:
Choices in Production
S7AR7 UP: 7IGH7NING SCURI7Y A7 7H WORLD'S
AIRPOR7S
|c ycu ..rt s.|e. .|. t..ve| c. rct` \|||e t|.t ,uest|cr |s se|Jcn .skeJ sc b|urt|y, .ry pe.scr .|c t..ve|s by .|.
c.r te|| ycu t|.t cu. cc||ect|ve .rs.e. |.s beer yes, .rJ |t |.s beer .cccnp.r|eJ by |rc.e.ses |r secu.|ty .rJ |ts
.sscc|.teJ ccsts .t .|.pc.ts .|| cve. t|e .c.|J. \|y` |r s|c.t, 9/11. e..c.|sts |.ckeJ |cu. |.S. ccnne.c|.| .|.||re.s
cr Septenbe. 11, 2001, .rJ t|e t..g|c .esu|ts t|.t |c||c.eJ |eJ tc . s|..p t|g|ter|rg |r .|.pc.t secu.|ty.
|r .r e|c.t tc p.evert s|n||.. J|s.ste.s, .|.pc.t secu.|ty c|c|.|s sc.ut|r|.e |ugg.ge .rJ p.sserge.s nc.e c..e
|u||y t|.r eve. be|c.e. |r t|e ncrt|s |c||c.|rg 9/11, Je|.ys c| .s nuc| .s t|.ee |cu.s .e.e ccnncr .s .gerts
t.|eJ tc .ssu.e t|.t rc .e.pcrs c. bcnbs ccu|J be snugg|eJ crtc .rct|e. p|.re.
\|.t tc p.cJuce` |s . |urJ.nert.| eccrcn|c ,uest|cr. |ve.y eccrcny nust .rs.e. t||s ,uest|cr. S|cu|J |t
p.cJuce nc.e eJuc.t|cr, bette. |e.|t| c..e, |np.cveJ t..rspc.t.t|cr, . c|e.re. erv|.crnert` |e.e ..e ||n|ts tc
.|.t . r.t|cr c.r p.cJuce, Jec|J|rg tc p.cJuce nc.e c| cre t||rg |rev|t.b|y ne.rs p.cJuc|rg |ess c| scnet||rg
e|se. |rJ|v|Ju.|s |r nuc| c| t|e .c.|J, .|te. t|e t..geJy c| 9/11, c|e..|y .e.e .||||rg tc g|ve up t|ne, .rJ . |.|.
.ncurt c| |rJ|v|Ju.| p.|v.cy, |r .r e|c.t tc cbt.|r g.e.te. secu.|ty. |.t|crs .rJ |rJ|v|Ju.| c|t|es .|sc JevcteJ .JJ|
t|cr.| .escu.ces tc pc||ce .rJ ct|e. |c.ns c| p.ctect|cr |r .r e|c.t tc p.evert t..geJ|es suc| .s 9/11. |ecp|e .||
cve. t|e .c.|J c|cse tc p.cJuce |ess c| ct|e. gccJs |r c.Je. tc Jevcte nc.e .escu.ces tc t|e p.cJuct|cr c| g.e.t
e. secu.|ty. ArJ, .s c| e..|y 2009, t|e c|c|ce tc Jevcte nc.e .escu.ces tc secu.|ty |.J p.|J c|, t|e.e |.J beer rc
s|n||.. |.ck|rgs |r t|e |r|teJ St.tes.
|r t||s c|.pte. .e use cu. |.st ncJe|, t|e p.cJuct|cr pcss|b|||t|es ncJe|, tc ex.n|re t|e r.tu.e c| c|c|ces tc
p.cJuce nc.e c| scne gccJs .rJ |ess c| ct|e.s. As |ts r.ne suggests, t|e production possibiIities modeI
s|c.s t|e gccJs .rJ se.v|ces t|.t .r eccrcny |s c.p.b|e c| p.cJuc|rg|ts pcss|b|||t|esg|ver t|e |.ctc.s c| p.c
Juct|cr .rJ t|e tec|rc|cgy |t |.s .v.||.b|e. |e ncJe| spec||es .|.t |t ne.rs tc use .escu.ces |u||y .rJ e|c|ert|y
.rJ suggests scne |npc.t.rt |np||c.t|crs |c. |rte.r.t|cr.| t..Je. \e c.r .|sc use t|e ncJe| tc |||ust..te eccrcn|c
g.c.t|, . p.ccess t|.t exp.rJs t|e set c| p.cJuct|cr pcss|b|||t|es .v.||.b|e tc .r eccrcny.
\e t|er tu.r tc .r ex.n|r.t|cr c| t|e type c| eccrcn|c systen |r .||c| c|c|ces ..e n.Je. Ar economic
system|s t|e set c| .u|es t|.t Je|re |c. .r eccrcnys .escu.ces ..e tc be c.reJ .rJ |c. Jec|s|crs .bcut t|e|.
use ..e tc be n.Je. \e .||| see t|.t eccrcn|c systens J||e. |r te.ns c| |c. t|ey .rs.e. t|e |urJ.nert.| ecc
rcn|c ,uest|crs. V.ry c| t|e .c.|Js eccrcn|c systens, |rc|uJ|rg t|e systens t|.t p.ev.|| |r |c.t| Ane.|c.,
|u.cpe, .rJ nuc| c| As|. .rJ ert..| .rJ Scut| Ane.|c., .e|y cr |rJ|v|Ju.|s cpe..t|rg |r . n..ket eccrcny tc
n.ke t|cse c|c|ces. Ot|e. eccrcn|c systens, |rc|uJ|rg t|cse c| ub. .rJ |c.t| |c.e. tcJ.y .rJ ||stc.|c.||y
factors of production
|e .escu.ces .v.||.b|e tc t|e
eccrcny |c. t|e p.cJuct|cr
c| gccJs .rJ se.v|ces.
utiIity
|e v.|ue, c. s.t|s|.ct|cr, t|.t
pecp|e Je.|ve |.cn t|e
gccJs .rJ se.v|ces t|ey
ccrsune .rJ t|e .ct|v|t|es
t|ey pu.sue.
Iabor
|e |un.r e|c.t t|.t c.r be
.pp||eJ tc t|e p.cJuct|cr c|
gccJs .rJ se.v|ces.
capitaI
A |.ctc. c| p.cJuct|cr t|.t
|.s beer p.cJuceJ |c. use |r
t|e p.cJuct|cr c| ct|e.
gccJs .rJ se.v|ces.
naturaI resources
|e .escu.ces c| r.tu.e t|.t
c.r be useJ |c. t|e
p.cJuct|cr c| gccJs .rJ
se.v|ces.
human capitaI
|e sk|||s . .c.ke. |.s .s .
.esu|t c| eJuc.t|cr, t..|r|rg,
c. expe.|erce t|.t c.r be
useJ |r p.cJuct|cr.
t|cse c| t|e |c.ne. Scv|et |r|cr, Scv|et b|cc ccurt.|es, .rJ ||r., .e|yc. .e||eJcr gcve.rnert tc n.ke t|ese
c|c|ces. |||e.ert eccrcn|c systens .esu|t |r J||e.ert sets c| c|c|ces .rJ t|us J||e.ert cutccnes, t|e |.ct t|.t
n..ket eccrcn|es gere..||y cutpe.|c.n t|e ct|e.s .|er |t ccnes tc p.cv|J|rg nc.e c| t|e t||rgs t|.t pecp|e
..rt |e|ps tc exp|.|r t|e J..n.t|c s|||t |.cn gcve.rnertJcn|r.teJ tc...J n..ketJcn|r.teJ eccrcn|c sys
tens t|.t |.s cccu..eJ t|.cug|cut t|e .c.|J |r t|e p.st 25 ye..s. |e c|.pte. ccrc|uJes .|t| .r ex.n|r.t|cr c|
t|e .c|e c| gcve.rnert |r .r eccrcny t|.t .e||es c||e|y cr n..kets tc .||cc.te gccJs .rJ se.v|ces.
1. FACTORS OF PRODUCTION
L A R N I N G O 8 1 C 7 I V S
1. Dene the three factors of productionIabor, capitaI, and naturaI resources.
2. xpIain the roIe of technoIogy and entrepreneurs in the utiIization of the economy's factors of
production.
Choices conceining what goods and seivices to pioduce aie choices about an economy's use of its
factors of production, the iesouices available to it foi the pioduction of goods and seivices. The
value, oi satisfaction, that people deiive fiom the goods and seivices they consume and the activities
they puisue is called utility. Ultimately, then, an economy's factois of pioduction cieate utility; they
seive the inteiests of people.
The factois of pioduction in an economy aie its laboi, capital, and natuial iesouices. Iabor is the
human efoit that can be applied to the pioduction of goods and seivices. People who aie employed oi
would like to be aie consideied pait of the laboi available to the economy. Capital is a factoi of pio-
duction that has been pioduced foi use in the pioduction of othei goods and seivices. Omce buildings,
machineiy, and tools aie examples of capital. Natural resources aie the iesouices of natuie that can
be used foi the pioduction of goods and seivices.
In the next thiee sections, we will take a closei look at the factois of pioduction we use to pioduce
the goods and seivices we consume. The thiee basic building blocks of laboi, capital, and natuial ie-
souices may be used in difeient ways to pioduce difeient goods and seivices, but they still lie at the
coie of pioduction. We will then look at the ioles played by technology and entiepieneuis in putting
these factois of pioduction to woik. As economists began to giapple with the pioblems of scaicity,
choice, and oppoitunity cost two centuiies ago, they focused on these concepts, just as they aie likely to
do two centuiies hence.
1.1 Laboi
Laboi is human efoit that can be applied to pioduction. People who woik to iepaii tiies, pilot aii-
planes, teach childien, oi enfoice laws aie all pait of the economy's laboi. People who would like to
woik but have not found employment-who aie unemployed-aie also consideied pait of the laboi
available to the economy.
In some contexts, it is useful to distinguish two foims of laboi. The fist is the human equivalent of
a natuial iesouice. It is the natuial ability an untiained, uneducated peison biings to a paiticulai pio-
duction piocess. But most woikeis biing fai moie. The skills a woikei has as a iesult of education,
tiaining, oi expeiience that can be used in pioduction aie called human capital. Students who aie at-
tending a college oi univeisity aie acquiiing human capital. Woikeis who aie gaining skills thiough ex-
peiience oi thiough tiaining aie acquiiing human capital. Childien who aie leaining to iead aie ac-
quiiing human capital.
The amount of laboi available to an economy can be incieased in two ways. One is to inciease the
total quantity of laboi, eithei by incieasing the numbei of people available to woik oi by incieasing the
aveiage numbei of houis of woik pei week. The othei is to inciease the amount of human capital pos-
sessed by woikeis.
26 PRINCIPLS OI CONOMICS
nanciaI capitaI
|rc|uJes ncrey .rJ ct|e.
p.pe. .ssets (suc| .s stccks
.rJ bcrJs) t|.t .ep.esert
c|.|ns cr |utu.e p.ynerts.
1.2 Capital
Long ago, when the fist human beings walked the eaith, they pioduced food by picking leaves oi fiuit
of a plant oi by catching an animal and eating it. We know that veiy eaily on, howevei, they began
shaping stones into tools, appaiently foi use in butcheiing animals. Those tools weie the fist capital
because they weie pioduced foi use in pioducing othei goods-food and clothing.
Modein veisions of the fist stone tools include saws, meat cleaveis, hooks, and giindeis; all aie
used in butcheiing animals. Tools such as hammeis, sciewdiiveis, and wienches aie also capital.
Tianspoitation equipment, such as cais and tiucks, is capital. Facilities such as ioads, biidges, poits,
and aiipoits aie capital. Buildings, too, aie capital; they help us to pioduce goods and seivices.
Capital does not consist solely of physical objects. The scoie foi a new symphony is capital because
it will be used to pioduce conceits. Computei softwaie used by business fims oi goveinment agencies
to pioduce goods and seivices is capital. Capital may thus include physical goods and intellectual dis-
coveiies. Any iesouice is capital if it satisfes two ciiteiia:
1. The iesouice must have been pioduced.
2. The iesouice can be used to pioduce othei goods and seivices.
One thing that is not consideied capital is money. A fim cannot use money diiectly to pioduce othei
goods, so money does not satisfy the second ciiteiion foi capital. Fiims can, howevei, use money to ac-
quiie capital. Money is a foim of fnancial capital. Financial capital includes money and othei
papei" assets (such as stocks and bonds) that iepiesent claims on futuie payments. These fnancial as-
sets aie not capital, but they can be used diiectly oi indiiectly to puichase factois of pioduction oi
goods and seivices.
1.3 Natuial Resouices
Theie aie two essential chaiacteiistics of natuial iesouices. The fist is that they aie found in
natuie-that no human efoit has been used to make oi altei them. The second is that they can be used
foi the pioduction of goods and seivices. That iequiies knowledge; we must know how to use the
things we fnd in natuie befoie they become iesouices.
Considei oil. Oil in the giound is a natuial iesouice because it is found (not manufactuied) and
can be used to pioduce goods and seivices. Howevei, 230 yeais ago oil was a nuisance, not a natuial ie-
souice. Pennsylvania faimeis in the eighteenth centuiy who found oil oozing up thiough theii soil
weie dismayed, not delighted. No one knew what could be done with the oil. It was not until the mid-
nineteenth centuiy that a method was found foi iefning oil into keiosene that could be used to genei-
ate eneigy, tiansfoiming oil into a natuial iesouice. Oil is now used to make all soits of things, includ-
ing clothing, diugs, gasoline, and plastic. It became a natuial iesouice because people discoveied and
implemented a way to use it.
Defning something as a natuial iesouice only if it can be used to pioduce goods and seivices does
not mean that a tiee has value only foi its wood oi that a mountain has value only foi its mineials. If
people gain utility fiom the existence of a beautiful wildeiness aiea, then that wildeiness piovides a sei-
vice. The wildeiness is thus a natuial iesouice.
The natuial iesouices available to us can be expanded in thiee ways. One is the discoveiy of new
natuial iesouices, such as the discoveiy of a deposit of oie containing titanium. The second is the dis-
coveiy of new uses foi iesouices, as happened when new techniques allowed oil to be put to pioductive
use oi sand to be used in manufactuiing computei chips. The thiid is the discoveiy of new ways to ex-
tiact natuial iesouices in oidei to use them. New methods of discoveiing and mapping oil deposits
have incieased the woild's supply of this impoitant natuial iesouice.
CHAP7R 2 CONIRON7ING SCARCI7Y: CHOICS IN PRODUC7ION 27
technoIogy
|e krc.|eJge t|.t c.r be
.pp||eJ tc t|e p.cJuct|cr c|
gccJs .rJ se.v|ces.
entrepreneur
A pe.scr .|c, cpe..t|rg
.|t||r t|e ccrtext c| .
n..ket eccrcny, seeks tc
e..r p.c|ts by |rJ|rg re.
..ys tc c.g.r|.e |.ctc.s c|
p.cJuct|cr.
1.4 Technology and the Entiepieneui
Goods and seivices aie pioduced using the factois of pioduction available to the economy. Two things
play a ciucial iole in putting these factois of pioduction to woik. The fist is technology, the know-
ledge that can be applied to the pioduction of goods and seivices. The second is an individual who
plays a key iole in a maiket economy: the entiepieneui. An entrepreneur is a peison who, opeiating
within the context of a maiket economy, seeks to eain piofts by fnding new ways to oiganize factois
of pioduction. In non-maiket economies the iole of the entiepieneui is played by buieauciats and oth-
ei decision makeis who iespond to incentives othei than pioft to guide theii choices about iesouice
allocation decisions.
The inteiplay of entiepieneuis and technology afects all oui lives. Entiepieneuis put new techno-
logies to woik eveiy day, changing the way factois of pioduction aie used. Faimeis and factoiy woik-
eis, engineeis and electiicians, technicians and teacheis all woik difeiently than they did just a few
yeais ago, using new technologies intioduced by entiepieneuis. The music you enjoy, the books you
iead, the athletic equipment with which you play aie pioduced difeiently than they weie fve yeais
ago. The book you aie ieading was wiitten and manufactuied using technologies that did not exist ten
yeais ago. We can dispute whethei all the changes have made oui lives bettei. What we cannot dispute
is that they have made oui lives difeient.
k Y 7 A k A W A Y S
< |.ctc.s c| p.cJuct|cr ..e t|e .escu.ces t|e eccrcny |.s .v.||.b|e tc p.cJuce gccJs .rJ se.v|ces.
< |.bc. |s t|e |un.r e|c.t t|.t c.r be .pp||eJ tc t|e p.cJuct|cr c| gccJs .rJ se.v|ces. |.bc.s
ccrt.|but|cr tc .r eccrcnys cutput c| gccJs .rJ se.v|ces c.r be |rc.e.seJ e|t|e. by |rc.e.s|rg t|e
,u.rt|ty c| |.bc. c. by |rc.e.s|rg |un.r c.p|t.|.
< .p|t.| |s . |.ctc. c| p.cJuct|cr t|.t |.s beer p.cJuceJ |c. use |r t|e p.cJuct|cr c| ct|e. gccJs .rJ
se.v|ces.
< |.tu..| .escu.ces ..e t|cse t||rgs |curJ |r r.tu.e t|.t c.r be useJ |c. t|e p.cJuct|cr c| gccJs .rJ
se.v|ces.
< .c keys tc t|e ut|||..t|cr c| .r eccrcnys |.ctc.s c| p.cJuct|cr ..e tec|rc|cgy .rJ, |r t|e c.se c| .
n..ket eccrcn|c systen, t|e e|c.ts c| ert.ep.ereu.s.
7 R Y I 7 !
|xp|.|r .|et|e. e.c| c| t|e |c||c.|rg |s |.bc., c.p|t.|, c. . r.tu..| .escu.ce.
1. Ar urenp|cyeJ |.ctc.y .c.ke.
2. A cc||ege p.c|essc.
3. |e ||b...y bu||J|rg cr ycu. c.npus
4. +e||c.stcre |.t|cr.| |..k
5. Ar urt.ppeJ Jepcs|t c| r.tu..| g.s
6. |e \||te |cuse
. |e |cc.| pc.e. p|.rt
28 PRINCIPLS OI CONOMICS
Case in Point: 7echnoIogy Cuts Costs, 8oosts Productivity and Profits
2010 jupiterimages Corporation
ec|rc|cgy c.r seen .r .bst..ct |c.ce |r t|e eccrcny|npc.t.rt, but |rv|s|b|e.
|t |s rct |rv|s|b|e tc t|e 130 pecp|e .|c .c.k cr . S|e|| O|| cnp.ry c|| .|g c.||eJ V..s, |cc.teJ |r t|e Jeep
..te.s c| t|e Cu|| c| Vex|cc, .bcut 160 n||es scut|.est c| |ers.cc|., ||c.|J.. |e r.ne V..s .e|ects |ts ct|
e..c.|J .ppe...rce|t exterJs 300 |eet .bcve t|e ..te.s su.|.ce .rJ |.s stee| terJcrs t|.t .e.c| 3,000
|eet tc t|e |cc. c| t|e gu||. ||s |.c|||ty .cu|J rct ex|st || |t .e.e rct |c. t|e Jeve|cpnert c| bette. c|| J|sccv
e.y net|cJs t|.t |rc|uJe t|.eeJ|ners|cr.| se|sn|c n.pp|rg tec|r|,ues, s.te|||tes t|.t |cc.te c|| |.cn sp.ce,
.rJ J.|||s t|.t c.r n.ke tu.rs .s J.||||rg |c.ener stee. t|en by ncr|tc.|rg t|en cr ccnpute. sc.eers |.cn
t|e ccn|c.t c| V..s. \e Jcrt ||t .s n.ry J.y |c|es, ccnnerteJ S|e|| n.r.ge. V||es b...ett. As . .esu|t c|
t|ese re. tec|rc|cg|es, cve. t|e p.st t.c Jec.Jes, t|e ccst c| J|sccve.|rg . b...e| c| c|| J.cppeJ |.cn 20 tc
urJe. 5. ArJ t|e tec|rc|cg|es ccrt|rue tc |np.cve. |.eeJ|ners|cr.| su.veys ..e be|rg .ep|.ceJ .|t| |cu.
J|ners|cr.| cres t|.t .||c. gec|cg|sts tc see |c. t|e c|| |e|Js c|.rge cve. t|ne.
|e V..s p.cect ..s Jest.cyeJ by |u..|c.re |.t.|r. |r 2005. |cy.| |utc| S|e|| ccnp|eteJ .ep.|.s |r 2006.t
. ccst c| 200 n||||cr. but, t|e |.c|||ty |s .g.|r punp|rg 130,000 b...e|s c| c|| pe. J.y .rJ 150 n||||cr cub|c
|eet c| r.tu..| g.st|e ere.gy e,u|v.|ert c| .r .JJ|t|cr.| 26,000 b...e|s c| c||.
ec|rc|cgy |s Jc|rg nc.e t|.r |e|p|rg ere.gy ccnp.r|es t..ck c|| Jepcs|ts. |t |s c|.rg|rg t|e ..y sc|t J.|rks
.rJ ct|e. g.cce.y |tens ..e Je||ve.eJ tc .et.|| stc.es. |c. ex.np|e, .|er . |eps|c Je||ve.y J.|ve. ...|ves .t .
||ever, t|e J.|ve. keys |rtc . |.rJ|e|J ccnpute. t|e |rvertc.y c| sc|t J.|rks, c||ps, .rJ ct|e. |eps|c
p.cJucts. |e |r|c.n.t|cr |s t..rsn|tteJ tc . n.|r ccnpute. .t t|e ...e|cuse t|.t beg|rs p.ccess|rg t|e
rext c.Je. |c. t|.t stc.e. |e .esu|t |s t|.t t|e J.|ve. c.r v|s|t nc.e stc.es |r . J.y .rJ |eps|c c.r ccve. . g|v
er te..|tc.y .|t| |e.e. J.|ve.s .rJ t.ucks.
|e. tec|rc|cgy |s ever |e|p|rg tc p.cJuce nc.e n||k |.cn cc.s. |J |..ser, .|c c.rs . 1,200cc. J.|.y
|..n |r \|sccrs|r, reve. gets up be|c.e J..r tc n||k t|e cc.s, t|e ..y |e J|J .s . bcy. |.t|e., t|e cc.s ..e
|cckeJ up tc e|ect.cr|c n||ke.s. cnpute.s ne.su.e e.c| cc.s cutput, .rJ cc.s p.cJuc|rg ||tt|e n||k ..e
sert tc . |csp|t.| .|rg |c. t.e.tnert. \|t| t|e |e|p c| suc| tec|rc|cgy, .s .e|| .s bette. |eeJ, tcJ.ys J.|.y
cc.s p.cJuce 50 nc.e n||k t|.r J|J cc.s 20 ye..s .gc. |ver t|cug| t|e runbe. c| J.|.y cc.s |r t|e |r
|teJ St.tes |r t|e |.st 20 ye..s |.s |.||er 1, n||k cutput |.s |rc.e.seJ 25.
CHAP7R 2 CONIRON7ING SCARCI7Y: CHOICS IN PRODUC7ION 29
production possibiIities
curve
A g..p||c.| .ep.esert.t|cr c|
t|e .|te.r.t|ve ccnb|r.t|crs
c| gccJs .rJ se.v|ces .r
eccrcny c.r p.cJuce.
\|c bere|ts |.cn tec|rc|cg|c.| p.cg.ess` crsune.s g.|r |.cn |c.e. p.|ces .rJ bette. se.v|ce. \c.ke.s
g.|r. |e|. g.e.te. .b|||ty tc p.cJuce gccJs .rJ se.v|ces t..rs|.tes |rtc ||g|e. ..ges. ArJ |.ns g.|r. |c.e.
p.cJuct|cr ccsts ne.r ||g|e. p.c|ts. O| ccu.se, scne pecp|e |cse .s tec|rc|cgy .Jv.rces. Scne cbs ..e
e||n|r.teJ, .rJ scne |.ns |rJ t|e|. se.v|ces ..e rc |crge. reeJeJ. Ore c.r ..gue .bcut .|et|e. p..t|cu|..
tec|rc|cg|c.| c|.rges |.ve |np.cveJ cu. ||ves, but t|ey |.ve c|e..|y n.Je.rJ .||| ccrt|rue tc
n.ket|en |.. J||e.ert.
.ooce |o.|J |o|||oJ. ||||| | ||e Oo|| |||e o /o.' .| |e|eo |e (||o|Jo:. ^oo| 5. 200. ^. |ooo |oeoo|. |o|, |o
|.o|.e |o .o.|.e.' |.^ oJo,. ^oo| .. 2003. |. |e| .oe oJ |ooo |oe. .ec|o| |eo| ^ /|o /|o o| |oJoc||.||,.' |.^ oJo,. ^oo|
30. 200. |. oJ |||o|e |e|o. .|e|| .|oc|eJ. |oe ||e. .o|, 2.. 2006
A N S W R S 7 O 7 R Y I 7 ! P R O 8 L M S
1. Ar urenp|cyeJ |.ctc.y .c.ke. ccu|J be put tc .c.k, |e c. s|e ccurts .s |.bc..
2. A cc||ege p.c|essc. |s |.bc..
3. |e ||b...y bu||J|rg cr ycu. c.npus |s p..t c| c.p|t.|.
4. +e||c.stcre |.t|cr.| |..k. |cse ..e.s c| t|e p..k |e|t |r t|e|. r.tu..| st.te ..e . r.tu..| .escu.ce. |.c|||t|es
suc| .s v|s|tc.s certe.s, .c.Js, .rJ c.npg.curJs ..e c.p|t.|.
5. Ar urt.ppeJ Jepcs|t c| r.tu..| g.s |s . r.tu..| .escu.ce. Orce ext..cteJ .rJ put |r . stc..ge t.rk, r.tu..|
g.s |s c.p|t.|.
6. |e \||te |cuse |s c.p|t.|.
. |e |cc.| pc.e. p|.rt |s c.p|t.|.
2. THE PRODUCTION POSSIBILITIES CURVE
L A R N I N G O 8 1 C 7 I V S
1. xpIain the concept of the production possibiIities curve and understand the impIications of its
downward sIope and bowed-out shape.
2. Use the production possibiIities modeI to distinguish between fuII empIoyment and situations
of idIe factors of production and between emcient and inemcient production.
3. Understand speciaIization and its reIationship to the production possibiIities modeI and com-
parative advantage.
An economy's factois of pioduction aie scaice; they cannot pioduce an unlimited quantity of goods
and seivices. A production possibilities curve is a giaphical iepiesentation of the alteinative com-
binations of goods and seivices an economy can pioduce. It illustiates the pioduction possibilities
model. In diawing the pioduction possibilities cuive, we shall assume that the economy can pioduce
only two goods and that the quantities of factois of pioduction and the technology available to the eco-
nomy aie fxed.
2.1 Constiucting a Pioduction Possibilities Cuive
To constiuct a pioduction possibilities cuive, we will begin with the case of a hypothetical fim, Alpine
Spoits, Inc., a specialized spoits equipment manufactuiei. Chiistie Rydei began the business 13 yeais
ago with a single ski pioduction facility neai Killington ski iesoit in cential Veimont. Ski sales giew,
and she also saw demand foi snowboaids iising-paiticulaily aftei snowboaid competition events
weie included in the 2002 Wintei Olympics in Salt Lake City. She added a second plant in a neaiby
town. The second plant, while smallei than the fist, was designed to pioduce snowboaids as well as
skis. She also modifed the fist plant so that it could pioduce both snowboaids and skis. Two yeais
latei she added a thiid plant in anothei town. While even smallei than the second plant, the thiid was
piimaiily designed foi snowboaid pioduction but could also pioduce skis.
30 PRINCIPLS OI CONOMICS
We can think of each of Ms. Rydei's thiee plants as a miniatuie economy and analyze them using
the pioduction possibilities model. We assume that the factois of pioduction and technology available
to each of the plants opeiated by Alpine Spoits aie unchanged.
Suppose the fist plant, Plant 1, can pioduce 200 paiis of skis pei month when it pioduces only
skis. When devoted solely to snowboaids, it pioduces 100 snowboaids pei month. It can pioduce skis
and snowboaids simultaneously as well.
The table in Figuie 2.2 gives thiee combinations of skis and snowboaids that Plant 1 can pioduce
each month. Combination A involves devoting the plant entiiely to ski pioduction; combination C
means shifting all of the plant's iesouices to snowboaid pioduction; combination B involves the pio-
duction of both goods. These values aie plotted in a pioduction possibilities cuive foi Plant 1. The
cuive is a downwaid-sloping stiaight line, indicating that theie is a lineai, negative ielationship
between the pioduction of the two goods.
Neithei skis noi snowboaids is an independent oi a dependent vaiiable in the pioduction possibil-
ities model; we can assign eithei one to the veitical oi to the hoiizontal axis. Heie, we have placed the
numbei of paiis of skis pioduced pei month on the veitical axis and the numbei of snowboaids pio-
duced pei month on the hoiizontal axis.
The negative slope of the pioduction possibilities cuive iefects the scaicity of the plant's capital
and laboi. Pioducing moie snowboaids iequiies shifting iesouices out of ski pioduction and thus pio-
ducing fewei skis. Pioducing moie skis iequiies shifting iesouices out of snowboaid pioduction and
thus pioducing fewei snowboaids.
The slope of Plant 1's pioduction possibilities cuive measuies the iate at which Alpine Spoits must
give up ski pioduction to pioduce additional snowboaids. Because the pioduction possibilities cuive
foi Plant 1 is lineai, we can compute the slope between any two points on the cuive and get the same
iesult. Between points A and B, foi example, the slope equals 2 paiis of skis/snowboaid (equals 100
paiis of skis/30 snowboaids). (Many students aie helped when told to iead this iesult as 2 paiis of
skis per snowboaid.") We get the same value between points B and C, and between points A and C.
II GUR 2. 2 A Production PossibiIities Curve
|e t.b|e s|c.s t|e ccnb|r.t|crs c| p.|.s c| sk|s .rJ src.bc..Js t|.t ||.rt 1 |s c.p.b|e c| p.cJuc|rg e.c| ncrt|.
|ese ..e .|sc |||ust..teJ .|t| . p.cJuct|cr pcss|b|||t|es cu.ve. |ct|ce t|.t t||s cu.ve |s ||re...
To see this ielationship moie cleaily, examine Figuie 2.3. Suppose Plant 1 is pioducing 100 paiis of skis
and 30 snowboaids pei month at point B. Now considei what would happen if Ms. Rydei decided to
pioduce 1 moie snowboaid pei month. The segment of the cuive aiound point B is magnifed in Fig-
uie 2.3. The slope between points B and B is 2 paiis of skis/snowboaid. Pioducing 1 additional snow-
boaid at point B iequiies giving up 2 paiis of skis. We can think of this as the oppoitunity cost of pio-
ducing an additional snowboaid at Plant 1. This oppoitunity cost equals the absolute value of the slope
of the pioduction possibilities cuive.
CHAP7R 2 CONIRON7ING SCARCI7Y: CHOICS IN PRODUC7ION 31
II GUR 2. 3 7he SIope of a Production PossibiIities Curve
|e s|cpe c| t|e ||re.. p.cJuct|cr pcss|b|||t|es cu.ve |r ||gu.e 2.2 |s ccrst.rt, |t |s 2 p.|.s c| sk|s/src.bc..J. |r t|e
sect|cr c| t|e cu.ve s|c.r |e.e, t|e s|cpe c.r be c.|cu|.teJ bet.eer pc|rts b .rJ b. |xp.rJ|rg src.bc..J
p.cJuct|cr tc 51 src.bc..Js pe. ncrt| |.cn 50 src.bc..Js pe. ncrt| .e,u|.es . .eJuct|cr |r sk| p.cJuct|cr tc
98 p.|.s c| sk|s pe. ncrt| |.cn 100 p.|.s. |e s|cpe e,u.|s 2 p.|.s c| sk|s/src.bc..J (t|.t |s, |t nust g|ve up t.c
p.|.s c| sk|s tc |.ee up t|e .escu.ces recess..y tc p.cJuce cre .JJ|t|cr.| src.bc..J). c s|||t |.cn b tc b, A|p|re
Spc.ts nust g|ve up t.c nc.e p.|.s c| sk|s pe. src.bc..J. |e .bsc|ute v.|ue c| t|e s|cpe c| . p.cJuct|cr
pcss|b|||t|es cu.ve ne.su.es t|e cppc.tur|ty ccst c| .r .JJ|t|cr.| ur|t c| t|e gccJ cr t|e |c.|.crt.| .x|s ne.su.eJ
|r te.ns c| t|e ,u.rt|ty c| t|e gccJ cr t|e ve.t|c.| .x|s t|.t nust be |c.gcre.
The absolute value of the slope of any pioduction possibilities cuive equals the oppoitunity cost of an
additional unit of the good on the hoiizontal axis. It is the amount of the good on the veitical axis that
must be given up in oidei to fiee up the iesouices iequiied to pioduce one moie unit of the good on
the hoiizontal axis. We will make use of this impoitant fact as we continue oui investigation of the pio-
duction possibilities cuive.
Figuie 2.4 shows pioduction possibilities cuives foi each of the fim's thiee plants. Each of the
plants, if devoted entiiely to snowboaids, could pioduce 100 snowboaids. Plants 2 and 3, if devoted ex-
clusively to ski pioduction, can pioduce 100 and 30 paiis of skis pei month, iespectively. The exhibit
gives the slopes of the pioduction possibilities cuives foi each plant. The oppoitunity cost of an addi-
tional snowboaid at each plant equals the absolute values of these slopes (that is, the numbei of paiis of
skis that must be given up pei snowboaid).
32 PRINCIPLS OI CONOMICS
comparative advantage
|r p.cJuc|rg . gccJ c.
se.v|ce, t|e s|tu.t|cr t|.t
cccu.s || t|e cppc.tur|ty ccst
c| p.cJuc|rg t|.t gccJ c.
se.v|ce |s |c.e. |c. t|.t
eccrcny t|.r |c. .ry ct|e..
II GUR 2. 4 Production PossibiIities at 7hree PIants
|e s|cpes c| t|e p.cJuct|cr pcss|b|||t|es cu.ves |c. e.c| p|.rt J||e.. |e steepe. t|e cu.ve, t|e g.e.te. t|e
cppc.tur|ty ccst c| .r .JJ|t|cr.| src.bc..J. |e.e, t|e cppc.tur|ty ccst |s |c.est .t ||.rt 3 .rJ g.e.test .t ||.rt 1.
The exhibit gives the slopes of the pioduction possibilities cuives foi each of the fim's thiee plants.
The oppoitunity cost of an additional snowboaid at each plant equals the absolute values of these
slopes. Moie geneially, the absolute value of the slope of any pioduction possibilities cuive at any point
gives the oppoitunity cost of an additional unit of the good on the hoiizontal axis, measuied in teims
of the numbei of units of the good on the veitical axis that must be foigone.
The gieatei the absolute value of the slope of the pioduction possibilities cuive, the gieatei the op-
poitunity cost will be. The plant foi which the oppoitunity cost of an additional snowboaid is gieatest
is the plant with the steepest pioduction possibilities cuive; the plant foi which the oppoitunity cost is
lowest is the plant with the fattest pioduction possibilities cuive. The plant with the lowest oppoitun-
ity cost of pioducing snowboaids is Plant 3; its slope of 0.3 means that Ms. Rydei must give up half a
paii of skis in that plant to pioduce an additional snowboaid. In Plant 2, she must give up one paii of
skis to gain one moie snowboaid. We have alieady seen that an additional snowboaid iequiies giving
up two paiis of skis in Plant 1.
2.2 Compaiative Advantage and the Pioduction Possibilities Cuive
To constiuct a combined pioduction possibilities cuive foi all thiee plants, we can begin by asking how
many paiis of skis Alpine Spoits could pioduce if it weie pioducing only skis. To fnd this quantity, we
add up the values at the veitical inteicepts of each of the pioduction possibilities cuives in Figuie 2.4.
These inteicepts tell us the maximum numbei of paiis of skis each plant can pioduce. Plant 1 can pio-
duce 200 paiis of skis pei month, Plant 2 can pioduce 100 paiis of skis at pei month, and Plant 3 can
pioduce 30 paiis. Alpine Spoits can thus pioduce 330 paiis of skis pei month if it devotes its iesouices
exclusively to ski pioduction. In that case, it pioduces no snowboaids.
Now suppose the fim decides to pioduce 100 snowboaids. That will iequiie shifting one of its
plants out of ski pioduction. Which one will it choose to shift: The sensible thing foi it to do is to
choose the plant in which snowboaids have the lowest oppoitunity cost-Plant 3. It has an advantage
not because it can pioduce moie snowboaids than the othei plants (all the plants in this example aie
capable of pioducing up to 100 snowboaids pei month) but because it is the least pioductive plant foi
making skis. Pioducing a snowboaid in Plant 3 iequiies giving up just half a paii of skis.
Economists say that an economy has a comparative advantage in pioducing a good oi seivice if
the oppoitunity cost of pioducing that good oi seivice is lowei foi that economy than foi any othei.
Plant 3 has a compaiative advantage in snowboaid pioduction because it is the plant foi which the op-
poitunity cost of additional snowboaids is lowest. To put this in teims of the pioduction possibilities
cuive, Plant 3 has a compaiative advantage in snowboaid pioduction (the good on the hoiizontal axis)
because its pioduction possibilities cuive is the fattest of the thiee cuives.
CHAP7R 2 CONIRON7ING SCARCI7Y: CHOICS IN PRODUC7ION 33
Iaw of increasing
opportunity cost
As .r eccrcny ncves .|crg
|ts p.cJuct|cr pcss|b|||t|es
cu.ve |r t|e J|.ect|cr c|
p.cJuc|rg nc.e c| .
p..t|cu|.. gccJ, t|e
cppc.tur|ty ccst c| .JJ|t|cr.|
ur|ts c| t|.t gccJ .|||
|rc.e.se.
II GUR 2. 5 7he Combined Production PossibiIities Curve for AIpine Sports
|e cu.ve s|c.r ccnb|res t|e p.cJuct|cr pcss|b|||t|es cu.ves |c. e.c| p|.rt. At pc|rt A, A|p|re Spc.ts p.cJuces
350 p.|.s c| sk|s pe. ncrt| .rJ rc src.bc..Js. || t|e |.n .|s|es tc |rc.e.se src.bc..J p.cJuct|cr, |t .||| |.st use
||.rt 3, .||c| |.s . ccnp...t|ve .Jv.rt.ge |r src.bc..Js.
Plant 3's compaiative advantage in snowboaid pioduction makes a ciucial point about the natuie of
compaiative advantage. It need not imply that a paiticulai plant is especially good at an activity. In oui
example, all thiee plants aie equally good at snowboaid pioduction. Plant 3, though, is the least em-
cient of the thiee in ski pioduction. Alpine thus gives up fewei skis when it pioduces snowboaids in
Plant 3. Compaiative advantage thus can stem fiom a lack of emciency in the pioduction of an altein-
ative good iathei than a special piofciency in the pioduction of the fist good.
The combined pioduction possibilities cuive foi the fim's thiee plants is shown in Figuie 2.3. We
begin at point A, with all thiee plants pioducing only skis. Pioduction totals 330 paiis of skis pei
month and zeio snowboaids. If the fim weie to pioduce 100 snowboaids at Plant 3, ski pioduction
would fall by 30 paiis pei month (iecall that the oppoitunity cost pei snowboaid at Plant 3 is half a paii
of skis). That would biing ski pioduction to 300 paiis, at point B. If Alpine Spoits weie to pioduce still
moie snowboaids in a single month, it would shift pioduction to Plant 2, the facility with the next-low-
est oppoitunity cost. Pioducing 100 snowboaids at Plant 2 would leave Alpine Spoits pioducing 200
snowboaids and 200 paiis of skis pei month, at point C. If the fim weie to switch entiiely to snow-
boaid pioduction, Plant 1 would be the last to switch because the cost of each snowboaid theie is 2
paiis of skis. With all thiee plants pioducing only snowboaids, the fim is at point D on the combined
pioduction possibilities cuive, pioducing 300 snowboaids pei month and no skis.
Notice that this pioduction possibilities cuive, which is made up of lineai segments fiom each as-
sembly plant, has a bowed-out shape; the absolute value of its slope incieases as Alpine Spoits pioduces
moie and moie snowboaids. This is a iesult of tiansfeiiing iesouices fiom the pioduction of one good
to anothei accoiding to compaiative advantage. We shall examine the signifcance of the bowed-out
shape of the cuive in the next section.
2.3 The Law of Incieasing Oppoitunity Cost
We see in Figuie 2.3 that, beginning at point A and pioducing only skis, Alpine Spoits expeiiences
highei and highei oppoitunity costs as it pioduces moie snowboaids. The fact that the oppoitunity
cost of additional snowboaids incieases as the fim pioduces moie of them is a iefection of an impoit-
ant economic law. The law of increasing opportunity cost holds that as an economy moves along
its pioduction possibilities cuive in the diiection of pioducing moie of a paiticulai good, the oppoi-
tunity cost of additional units of that good will inciease.
We have seen the law of incieasing oppoitunity cost at woik tiaveling fiom point A towaid point
D on the pioduction possibilities cuive in Figuie 2.3. The oppoitunity cost of each of the fist 100
snowboaids equals half a paii of skis; each of the next 100 snowboaids has an oppoitunity cost of 1 paii
of skis, and each of the last 100 snowboaids has an oppoitunity cost of 2 paiis of skis. The law also ap-
plies as the fim shifts fiom snowboaids to skis. Suppose it begins at point D, pioducing 300 snow-
boaids pei month and no skis. It can shift to ski pioduction at a ielatively low cost at fist. The
34 PRINCIPLS OI CONOMICS
oppoitunity cost of the fist 200 paiis of skis is just 100 snowboaids at Plant 1, a movement fiom point
D to point C, oi 0.3 snowboaids pei paii of skis. We would say that Plant 1 has a compaiative advant-
age in ski pioduction. The next 100 paiis of skis would be pioduced at Plant 2, wheie snowboaid pio-
duction would fall by 100 snowboaids pei month. The oppoitunity cost of skis at Plant 2 is 1 snow-
boaid pei paii of skis. Plant 3 would be the last plant conveited to ski pioduction. Theie, 30 paiis of
skis could be pioduced pei month at a cost of 100 snowboaids, oi an oppoitunity cost of 2 snowboaids
pei paii of skis.
The bowed-out pioduction possibilities cuive foi Alpine Spoits illustiates the law of incieasing op-
poitunity cost. Scaicity implies that a pioduction possibilities cuive is downwaid sloping; the law of in-
cieasing oppoitunity cost implies that it will be bowed out, oi concave, in shape.
The bowed-out cuive of Figuie 2.3 becomes smoothei as we include moie pioduction facilities.
Suppose Alpine Spoits expands to 10 plants, each with a lineai pioduction possibilities cuive. Panel (a)
of Figuie 2.6 shows the combined cuive foi the expanded fim, constiucted as we did in Figuie 2.3.
This pioduction possibilities cuive includes 10 lineai segments and is almost a smooth cuive. As we in-
clude moie and moie pioduction units, the cuive will become smoothei and smoothei. In an actual
economy, with a tiemendous numbei of fims and woikeis, it is easy to see that the pioduction possib-
ilities cuive will be smooth. We will geneially diaw pioduction possibilities cuives foi the economy as
smooth, bowed-out cuives, like the one in Panel (b). This pioduction possibilities cuive shows an eco-
nomy that pioduces only skis and snowboaids. Notice the cuive still has a bowed-out shape; it still has
a negative slope. Notice also that this cuive has no numbeis. Economists often use models such as the
pioduction possibilities model with giaphs that show the geneial shapes of cuives but that do not in-
clude specifc numbeis.
II GUR 2. 6 Production PossibiIities for the conomy
As .e ccnb|re t|e p.cJuct|cr pcss|b|||t|es cu.ves |c. nc.e .rJ nc.e ur|ts, t|e cu.ve beccnes sncct|e.. |t .et.|rs
|ts reg.t|ve s|cpe .rJ bc.eJcut s|.pe. |r |.re| (.) .e |.ve . ccnb|reJ p.cJuct|cr pcss|b|||t|es cu.ve |c. A|p|re
Spc.ts, .ssun|rg t|.t |t rc. |.s 10 p|.rts p.cJuc|rg sk|s .rJ src.bc..Js. |ver t|cug| e.c| c| t|e p|.rts |.s .
||re.. cu.ve, ccnb|r|rg t|en .ccc.J|rg tc ccnp...t|ve .Jv.rt.ge, .s .e J|J .|t| 3 p|.rts |r ||gu.e 2.5, p.cJuces
.|.t .ppe..s tc be . sncct|, rcr||re.. cu.ve, ever t|cug| |t |s n.Je up c| ||re.. segnerts. |r J...|rg
p.cJuct|cr pcss|b|||t|es cu.ves |c. t|e eccrcny, .e s|.|| gere..||y .ssune t|ey ..e sncct| .rJ bc.eJ cut, .s |r
|.re| (b). ||s cu.ve Jep|cts .r ert|.e eccrcny t|.t p.cJuces cr|y sk|s .rJ src.bc..Js.
2.4 Movements Along the Pioduction Possibilities Cuive
We can use the pioduction possibilities model to examine choices in the pioduction of goods and sei-
vices. In applying the model, we assume that the economy can pioduce two goods, and we assume that
technology and the factois of pioduction available to the economy iemain unchanged. In this section,
we shall assume that the economy opeiates on its pioduction possibilities cuive so that an inciease in
the pioduction of one good in the model implies a ieduction in the pioduction of the othei.
We shall considei two goods and seivices: national secuiity and a categoiy we shall call all othei
goods and seivices." This second categoiy includes the entiie iange of goods and seivices the economy
can pioduce, aside fiom national defense and secuiity. Cleaily, the tiansfei of iesouices to the efoit to
enhance national secuiity ieduces the quantity of othei goods and seivices that can be pioduced. In the
CHAP7R 2 CONIRON7ING SCARCI7Y: CHOICS IN PRODUC7ION 35
II GUR 2. 7 Spending More for Security
|e.e, .r eccrcny t|.t c.r p.cJuce t.c
c.tegc.|es c| gccJs, secu.|ty .rJ .|| ct|e.
gccJs .rJ se.v|ces, beg|rs .t pc|rt A cr |ts
p.cJuct|cr pcss|b|||t|es cu.ve. |e eccrcny
p.cJuces .
A
ur|ts c| secu.|ty .rJ
A
ur|ts c| .||
ct|e. gccJs .rJ se.v|ces pe. pe.|cJ. A
ncvenert |.cn A tc b .e,u|.es s|||t|rg
.escu.ces cut c| t|e p.cJuct|cr c| .|| ct|e.
gccJs .rJ se.v|ces .rJ |rtc sperJ|rg cr
secu.|ty. |e |rc.e.se |r sperJ|rg cr secu.|ty,
tc .
A
ur|ts c| secu.|ty pe. pe.|cJ, |.s .r
cppc.tur|ty ccst c| .eJuceJ p.cJuct|cr c| .||
ct|e. gccJs .rJ se.v|ces. |.cJuct|cr c| .||
ct|e. gccJs .rJ se.v|ces |.||s by O
A

b
ur|ts
pe. pe.|cJ.
fuII empIoyment
S|tu.t|cr |r .||c| .|| t|e
|.ctc.s c| p.cJuct|cr t|.t ..e
.v.||.b|e |c. use urJe.
cu..ert n..ket ccrJ|t|crs ..e
be|rg ut|||.eJ.
wake of the 9/11 attacks in 2001, nations thioughout the woild incieased theii spending foi national
secuiity. This spending took a vaiiety of foims. One, of couise, was incieased defense spending. Local
and state goveinments also incieased spending in an efoit to pievent teiioiist attacks. Aiipoits aiound
the woild hiied additional agents to inspect luggage and passengeis.
The inciease in iesouices devoted to secuiity meant fewei othei goods and seivices" could be
pioduced. In teims of the pioduction possibilities cuive in Figuie 2.7, the choice to pioduce moie se-
cuiity and less of othei goods and seivices means a movement fiom A to B. Of couise, an economy
cannot ieally produce secuiity; it can only attempt to piovide it. The attempt to piovide it iequiies ie-
souices; it is in that sense that we shall speak of the economy as pioducing" secuiity.
At point A, the economy was pioducing S
A
units of secuiity on the veitical ax-
is-defense seivices and vaiious foims of police piotection-and O
A
units of othei
goods and seivices on the hoiizontal axis. The decision to devote moie iesouices to se-
cuiity and less to othei goods and seivices iepiesents the choice we discussed in the
chaptei intioduction. In this case we have categoiies of goods iathei than specifc
goods. Thus, the economy chose to inciease spending on secuiity in the efoit to defeat
teiioiism. Since we have assumed that the economy has a fxed quantity of available ie-
souices, the incieased use of iesouices foi secuiity and national defense necessaiily ie-
duces the numbei of iesouices available foi the pioduction of othei goods and seivices.
The law of incieasing oppoitunity cost tells us that, as the economy moves along
the pioduction possibilities cuive in the diiection of moie of one good, its oppoitunity
cost will inciease. We may conclude that, as the economy moved along this cuive in the
diiection of gieatei pioduction of secuiity, the oppoitunity cost of the additional secui-
ity began to inciease. That is because the iesouices tiansfeiied fiom the pioduction of
othei goods and seivices to the pioduction of secuiity had a gieatei and gieatei com-
paiative advantage in pioducing things othei than secuiity.
The pioduction possibilities model does not tell us wheie on the cuive a paiticulai
economy will opeiate. Instead, it lays out the possibilities facing the economy. Many
countiies, foi example, chose to move along theii iespective pioduction possibilities
cuives to pioduce moie secuiity and national defense and less of all othei goods in the
wake of 9/11. We will see in the chaptei on demand and supply how choices about
what to pioduce aie made in the maiketplace.
2.3 Pioducing on Veisus Pioducing Inside the Pioduction
Possibilities Cuive
An economy that is opeiating inside its pioduction possibilities cuive could, by moving
onto it, pioduce moie of all the goods and seivices that people value, such as food,
housing, education, medical caie, and music. Incieasing the availability of these goods
would impiove the standaid of living. Economists conclude that it is bettei to be on the
pioduction possibilities cuive than inside it.
Two things could leave an economy opeiating at a point inside its pioduction pos-
sibilities cuive. Fiist, the economy might fail to use fully the iesouices available to it.
Second, it might not allocate iesouices on the basis of compaiative advantage. In eithei
case, pioduction within the pioduction possibilities cuive implies the economy could
impiove its peifoimance.
IdIe Iactors of Production
Suppose an economy fails to put all its factois of pioduction to woik. Some woikeis aie without jobs,
some buildings aie without occupants, some felds aie without ciops. Because an economy's pioduc-
tion possibilities cuive assumes the full use of the factois of pioduction available to it, the failuie to use
some factois iesults in a level of pioduction that lies inside the pioduction possibilities cuive.
If all the factois of pioduction that aie available foi use undei cuiient maiket conditions aie being
utilized, the economy has achieved full employment. An economy cannot opeiate on its pioduction
possibilities cuive unless it has full employment.
36 PRINCIPLS OI CONOMICS
emcient production
\|er .r eccrcny |s
cpe..t|rg cr |ts p.cJuct|cr
pcss|b|||t|es cu.ve.
inemcient production
S|tu.t|cr |r .||c| t|e
eccrcny |s us|rg t|e s.ne
,u.rt|t|es c| |.ctc.s c|
p.cJuct|cr but |s cpe..t|rg
|rs|Je |ts p.cJuct|cr
pcss|b|||t|es cu.ve.
II GUR 2. 8 IdIe Iactors and Production
|e p.cJuct|cr pcss|b|||t|es cu.ve s|c.r suggests .r eccrcny t|.t c.r p.cJuce t.c gccJs, |ccJ .rJ c|ct||rg. As
. .esu|t c| . |.||u.e tc .c||eve |u|| enp|cynert, t|e eccrcny cpe..tes .t . pc|rt suc| .s b, p.cJuc|rg |
b
ur|ts c|
|ccJ .rJ
b
ur|ts c| c|ct||rg pe. pe.|cJ. |utt|rg |ts |.ctc.s c| p.cJuct|cr tc .c.k .||c.s . ncve tc t|e p.cJuct|cr
pcss|b|||t|es cu.ve, tc . pc|rt suc| .s A. |e p.cJuct|cr c| bct| gccJs .|ses.
Figuie 2.8 shows an economy that can pioduce food and clothing. If it chooses to pioduce at point A,
foi example, it can pioduce F
A
units of food and C
A
units of clothing. Now suppose that a laige fiac-
tion of the economy's woikeis lose theii jobs, so the economy no longei makes full use of one factoi of
pioduction: laboi. In this example, pioduction moves to point B, wheie the economy pioduces less
food (F
B
) and less clothing (C
B
) than at point A. We often think of the loss of jobs in teims of the
woikeis; they have lost a chance to woik and to eain income. But the pioduction possibilities model
points to anothei loss: goods and seivices the economy could have pioduced that aie not being
pioduced.
Inefficient Production
Now suppose Alpine Spoits is fully employing its factois of pioduction. Could it still opeiate inside its
pioduction possibilities cuive: Could an economy that is using all its factois of pioduction still pio-
duce less than it could: The answei is Yes," and the key lies in compaiative advantage. An economy
achieves a point on its pioduction possibilities cuive only if it allocates its factois of pioduction on the
basis of compaiative advantage. If it fails to do that, it will opeiate inside the cuive.
Suppose that, as befoie, Alpine Spoits has been pioducing only skis. With all thiee of its plants
pioducing skis, it can pioduce 330 paiis of skis pei month (and no snowboaids). The fim then staits
pioducing snowboaids. This time, howevei, imagine that Alpine Spoits switches plants fiom skis to
snowboaids in numeiical oidei: Plant 1 fist, Plant 2 second, and then Plant 3. Figuie 2.9 illustiates the
iesult. Instead of the bowed-out pioduction possibilities cuive ABCD, we get a bowed-in cuive,
ABCD. Suppose that Alpine Spoits is pioducing 100 snowboaids and 130 paiis of skis at point B. Had
the fim based its pioduction choices on compaiative advantage, it would have switched Plant 3 to
snowboaids and then Plant 2, so it could have opeiated at a point such as C. It would be pioducing
moie snowboaids and moie paiis of skis-and using the same quantities of factois of pioduction it was
using at B. Had the fim based its pioduction choices on compaiative advantage, it would have
switched Plant 3 to snowboaids and then Plant 2, so it would have opeiated at point C. It would be
pioducing moie snowboaids and moie paiis of skis-and using the same quantities of factois of pio-
duction it was using at B. When an economy is opeiating on its pioduction possibilities cuive, we say
that it is engaging in emcient production. If it is using the same quantities of factois of pioduction
but is opeiating inside its pioduction possibilities cuive, it is engaging in inemcient production.
Inemcient pioduction implies that the economy could be pioducing moie goods without using any ad-
ditional laboi, capital, oi natuial iesouices.
CHAP7R 2 CONIRON7ING SCARCI7Y: CHOICS IN PRODUC7ION 37
II GUR 2. 9 fficient Versus Inefficient
Production
\|er |.ctc.s c| p.cJuct|cr ..e .||cc.teJ cr .
b.s|s ct|e. t|.r ccnp...t|ve .Jv.rt.ge, t|e
.esu|t |s |re|c|ert p.cJuct|cr. Suppcse A|p|re
Spc.ts cpe..tes t|e t|.ee p|.rts .e ex.n|reJ
|r ||gu.e 2.4. Suppcse |u.t|e. t|.t .|| t|.ee
p|.rts ..e JevcteJ exc|us|ve|y tc sk|
p.cJuct|cr, t|e |.n cpe..tes .t A. |c.
suppcse t|.t, tc |rc.e.se src.bc..J
p.cJuct|cr, |t t..rs|e.s p|.rts |r rune.|c.|
c.Je.. ||.rt 1 |.st, t|er ||.rt 2, .rJ |r.||y
||.rt 3. |e .esu|t |s t|e bc.eJ|r cu.ve
Ab|. |.cJuct|cr cr t|e p.cJuct|cr
pcss|b|||t|es cu.ve Ab| .e,u|.es t|.t |.ctc.s c|
p.cJuct|cr be t..rs|e..eJ .ccc.J|rg tc
ccnp...t|ve .Jv.rt.ge.
speciaIization
S|tu.t|cr |r .||c| .r
eccrcny |s p.cJuc|rg t|e
gccJs .rJ se.v|ces |r .||c|
|t |.s . ccnp...t|ve
.Jv.rt.ge.
Points on the pioduction possibilities cuive thus satisfy two conditions: the eco-
nomy is making full use of its factois of pioduction, and it is making emcient use of its
factois of pioduction. If theie aie idle oi inemciently allocated factois of pioduction,
the economy will opeiate inside the pioduction possibilities cuive. Thus, the pioduc-
tion possibilities cuive not only shows what can be pioduced; it piovides insight into
how goods and seivices should be pioduced. It suggests that to obtain emciency in pio-
duction, factois of pioduction should be allocated on the basis of compaiative advant-
age. Fuithei, the economy must make full use of its factois of pioduction if it is to pio-
duce the goods and seivices it is capable of pioducing.
SpeciaIization
The pioduction possibilities model suggests that specialization will occui. Specializa-
tion implies that an economy is pioducing the goods and seivices in which it has a
compaiative advantage. If Alpine Spoits selects point C in Figuie 2.9, foi example, it
will assign Plant 1 exclusively to ski pioduction and Plants 2 and 3 exclusively to snow-
boaid pioduction.
Such specialization is typical in an economic system. Woikeis, foi example, spe-
cialize in paiticulai felds in which they have a compaiative advantage. People woik
and use the income they eain to buy-peihaps impoit-goods and seivices fiom
people who have a compaiative advantage in doing othei things. The iesult is a fai
gieatei quantity of goods and seivices than would be available without this
specialization.
Think about what life would be like without specialization. Imagine that you aie
suddenly completely cut of fiom the iest of the economy. You must pioduce
eveiything you consume; you obtain nothing fiom anyone else. Would you be able to
consume what you consume now: Cleaily not. It is haid to imagine that most of us
could even suivive in such a setting. The gains we achieve thiough specialization aie
enoimous.
Nations specialize as well. Much of the land in the United States has a compaiative
advantage in agiicultuial pioduction and is devoted to that activity. Hong Kong, with
its huge population and tiny endowment of land, allocates viitually none of its land to
agiicultuial use; that option would be too costly. Its land is devoted laigely to nonagii-
cultuial use.
k Y 7 A k A W A Y S
< A p.cJuct|cr pcss|b|||t|es cu.ve s|c.s t|e ccnb|r.t|crs c| t.c gccJs .r eccrcny |s c.p.b|e c|
p.cJuc|rg.
< |e Jc.r...J s|cpe c| t|e p.cJuct|cr pcss|b|||t|es cu.ve |s .r |np||c.t|cr c| sc..c|ty.
< |e bc.eJcut s|.pe c| t|e p.cJuct|cr pcss|b|||t|es cu.ve .esu|ts |.cn .||cc.t|rg .escu.ces b.seJ cr
ccnp...t|ve .Jv.rt.ge. Suc| .r .||cc.t|cr |np||es t|.t t|e |.. c| |rc.e.s|rg cppc.tur|ty ccst .||| |c|J.
< Ar eccrcny t|.t |.||s tc n.ke |u|| .rJ e|c|ert use c| |ts |.ctc.s c| p.cJuct|cr .||| cpe..te |rs|Je |ts
p.cJuct|cr pcss|b|||t|es cu.ve.
< Spec|.||..t|cr ne.rs t|.t .r eccrcny |s p.cJuc|rg t|e gccJs .rJ se.v|ces |r .||c| |t |.s . ccnp...t|ve
.Jv.rt.ge.
38 PRINCIPLS OI CONOMICS
7 R Y I 7 !
Suppcse . n.ru|.ctu.|rg |.n |s e,u|ppeJ tc p.cJuce ..J|cs c. c.|cu|.tc.s. |t |.s t.c p|.rts, ||.rt | .rJ
||.rt S, .t .||c| |t c.r p.cJuce t|ese gccJs. C|ver t|e |.bc. .rJ t|e c.p|t.| .v.||.b|e .t bct| p|.rts, |t c.r
p.cJuce t|e ccnb|r.t|crs c| t|e t.c gccJs .t t|e t.c p|.rts s|c.r.
Output per day, PIant R
Combination CaIcuIators Radios
A 100 0
b 50 25
0 50
Output per day, PIant S
Combination CaIcuIators Radios
| 50 0
| 25 50
| 0 100
|ut c.|cu|.tc.s cr t|e ve.t|c.| .x|s .rJ ..J|cs cr t|e |c.|.crt.| .x|s. |... t|e p.cJuct|cr pcss|b|||t|es cu.ve |c.
||.rt |. Or . sep...te g..p|, J... t|e p.cJuct|cr pcss|b|||t|es cu.ve |c. ||.rt S. \||c| p|.rt |.s . ccnp...t
|ve .Jv.rt.ge |r c.|cu|.tc.s` |r ..J|cs` |c. J... t|e ccnb|reJ cu.ves |c. t|e t.c p|.rts. Suppcse t|e |.n
Jec|Jes tc p.cJuce 100 ..J|cs. \|e.e .||| |t p.cJuce t|en` |c. n.ry c.|cu|.tc.s .||| |t be .b|e tc p.cJuce`
\|e.e .||| |t p.cJuce t|e c.|cu|.tc.s`
Case in Point: 7he Cost of the Great Depression
2010 jupiterimages Corporation
|e |.S. eccrcny |cckeJ ve.y |e.|t|y |r t|e beg|rr|rg c| 1929. |t |.J ercyeJ sever ye..s c| J..n.t|c
g.c.t| .rJ urp.eceJerteJ p.cspe.|ty. |ts .escu.ces .e.e |u||y enp|cyeJ, |t ..s cpe..t|rg ,u|te c|cse tc |ts
p.cJuct|cr pcss|b|||t|es cu.ve.
CHAP7R 2 CONIRON7ING SCARCI7Y: CHOICS IN PRODUC7ION 39
|r t|e sunne. c| 1929, |c.eve., t||rgs st..teJ gc|rg ..crg. |.cJuct|cr .rJ enp|cynert |e||. |ey ccrt|r
ueJ tc |.|| |c. seve..| ye..s. by 1933, nc.e t|.r 25 c| t|e r.t|crs .c.ke.s |.J |cst t|e|. cbs. |.cJuct|cr |.J
p|unneteJ by .|ncst 30. |e eccrcny |.J ncveJ .e|| .|t||r |ts p.cJuct|cr pcss|b|||t|es cu.ve.
Output beg.r tc g.c. .|te. 1933, but t|e eccrcny ccrt|rueJ tc |.ve v.st runbe.s c| |J|e .c.ke.s, |J|e
|.ctc.|es, .rJ |J|e |..ns. |ese .escu.ces .e.e rct put b.ck tc .c.k |u||y urt|| 1942, .|te. t|e |.S. ert.y |rtc
\c.|J \.. || Jen.rJeJ ncb|||..t|cr c| t|e eccrcnys |.ctc.s c| p.cJuct|cr.
bet.eer 1929 .rJ 1942, t|e eccrcny p.cJuceJ 25 |e.e. gccJs .rJ se.v|ces t|.r |t .cu|J |.ve || |ts .e
scu.ces |.J beer |u||y enp|cyeJ. |.t ..s . |css, ne.su.eJ |r tcJ.ys Jc||..s, c| .e|| cve. 3 t.||||cr. |r n.te.|
.| te.ns, t|e |c.gcre cutput .ep.eserteJ . g.e.te. ccst t|.r t|e |r|teJ St.tes .cu|J u|t|n.te|y sperJ |r
\c.|J \.. ||. |e C.e.t |ep.ess|cr ..s . ccst|y expe.|erce |rJeeJ.
A N S W R 7 O 7 R Y I 7 ! P R O 8 L M
|e p.cJuct|cr pcss|b|||t|es cu.ves |c. t|e t.c p|.rts ..e s|c.r, .|crg .|t| t|e ccnb|reJ cu.ve |c. bct|
p|.rts. ||.rt | |.s . ccnp...t|ve .Jv.rt.ge |r p.cJuc|rg c.|cu|.tc.s. ||.rt S |.s . ccnp...t|ve .Jv.rt.ge |r
p.cJuc|rg ..J|cs, sc, || t|e |.n gces |.cn p.cJuc|rg 150 c.|cu|.tc.s .rJ rc ..J|cs tc p.cJuc|rg 100 ..J|cs, |t
.||| p.cJuce t|en .t ||.rt S. |r t|e p.cJuct|cr pcss|b|||t|es cu.ve |c. bct| p|.rts, t|e |.n .cu|J be .t V, p.c
Juc|rg 100 c.|cu|.tc.s .t ||.rt |.
3. APPLICATIONS OF THE PRODUCTION POSSIBILITIES
MODEL
L A R N I N G O 8 1 C 7 I V S
1. Understand the argument for unrestricted internationaI trade in terms of economic speciaIiza-
tion and comparative advantage.
2. Dene economic growth in terms of the production possibiIities modeI and discuss factors that
make such growth possibIe.
3. xpIain the cIassication of economic systems, the roIe of government in dierent economic
systems, and the strengths and weaknesses of dierent systems.
The pioduction possibilities cuive gives us a model of an economy. The model piovides poweiful in-
sights about the ieal woild, insights that help us to answei some impoitant questions: How does tiade
between two countiies afect the quantities of goods available to people: What deteimines the iate at
which pioduction will inciease ovei time: What is the iole of economic fieedom in the economy: In
this section we exploie applications of the model to questions of inteinational tiade, economic giowth,
and the choice of an economic system.
40 PRINCIPLS OI CONOMICS
3.1 Compaiative Advantage and Inteinational Tiade
One of the most impoitant implications of the concepts of compaiative advantage and the pioduction
possibilities cuive ielates to inteinational tiade. We can think of difeient nations as being equivalent
to Chiistie Rydei's plants. Each will have a compaiative advantage in ceitain activities, and emcient
woild pioduction iequiies that each nation specialize in those activities in which it has a compaiative
advantage. A failuie to allocate iesouices in this way means that woild pioduction falls inside the pio-
duction possibilities cuive; moie of each good could be pioduced by ielying on compaiative advantage.
If nations specialize, then they must iely on each othei. They will sell the goods in which they spe-
cialize and puichase othei goods fiom othei nations. Suppose, foi example, that the woild consists of
two continents that can each pioduce two goods: South Ameiica and Euiope can pioduce food and
computeis. Suppose they can pioduce the two goods accoiding to the tables in Panels (a) and (b) of
Figuie 2.12. We have simplifed this example by assuming that each continent has a lineai pioduction
possibilities cuive; the cuives aie plotted below the tables in Panels (a) and (b). Each continent has a
sepaiate pioduction possibilities cuive; the two have been combined to illustiate a woild pioduction
possibilities cuive in Panel (c) of the exhibit.
II GUR 2. 12 Production PossibiIities Curves and 7rade
Suppcse t|e .c.|J ccrs|sts c| t.c ccrt|rerts. Scut| Ane.|c. .rJ |u.cpe. |ey c.r e.c| p.cJuce t.c gccJs. |ccJ .rJ ccnpute.s. |r t||s ex.np|e,
.e .ssune t|.t e.c| ccrt|rert |.s . ||re.. p.cJuct|cr pcss|b|||t|es cu.ve, .s s|c.r |r |.re|s (.) .rJ (b). Scut| Ane.|c. |.s . ccnp...t|ve
.Jv.rt.ge |r |ccJ p.cJuct|cr .rJ |u.cpe |.s . ccnp...t|ve .Jv.rt.ge |r ccnpute. p.cJuct|cr. \|t| |.ee t..Je, t|e .c.|J c.r cpe..te cr t|e
bc.eJcut cu.ve C||, s|c.r |r |.re| (c). || t|e ccrt|rerts .e|use tc t..Je, t|e .c.|J .||| cpe..te |rs|Je |ts p.cJuct|cr pcss|b|||t|es cu.ve. ||, |c.
ex.np|e, e.c| ccrt|rert .e.e tc p.cJuce .t t|e n|Jpc|rt c| |ts p.cJuct|cr pcss|b|||t|es cu.ve, t|e .c.|J .cu|J p.cJuce 300 ccnpute.s .rJ 300
ur|ts c| |ccJ pe. pe.|cJ .t pc|rt . || e.c| ccrt|rert .e.e tc spec|.||.e |r t|e gccJ |r .||c| |t |.s . ccnp...t|ve .Jv.rt.ge, .c.|J p.cJuct|cr
ccu|J ncve tc . pc|rt suc| .s |, .|t| nc.e c| bct| gccJs p.cJuceJ.
The woild pioduction possibilities cuive assumes that iesouices aie allocated between computei and
food pioduction based on compaiative advantage. Notice that, even with only two economies and the
assumption of lineai pioduction possibilities cuives foi each, the combined cuive still has a bowed-out
shape. At point H, foi example, South Ameiica specializes in food, while Euiope pioduces only com-
puteis. Woild pioduction equals 400 units of each good. In this situation, we would expect South
Ameiica to expoit food to Euiope while Euiope expoits computeis to South Ameiica.
But suppose the iegions iefuse to tiade; each insists on pioducing its own food and computeis.
Suppose fuithei that each chooses to pioduce at the midpoint of its own pioduction possibilities cuive.
South Ameiica pioduces 100 units of computeis and 200 units of food pei peiiod, while Euiope pio-
duces 200 units of computeis and 100 units of food pei peiiod. Woild pioduction thus totals 300 units
of each good pei peiiod; the woild opeiates at point Q in Figuie 2.12. If the two continents weie willing
to move fiom isolation to tiade, the woild could achieve an inciease in the pioduction of both goods.
Pioducing at point H iequiies no moie iesouices, no moie efoit than pioduction at Q. It does,
howevei, iequiie that the woild's iesouices be allocated on the basis of compaiative advantage.
CHAP7R 2 CONIRON7ING SCARCI7Y: CHOICS IN PRODUC7ION 41
economic growth
|e p.ccess t|.cug| .||c|
.r eccrcny .c||eves .r
cut...J s|||t |r |ts
p.cJuct|cr pcss|b|||t|es cu.ve.
The implications of oui model foi tiade aie poweiful indeed. Fiist, we see that tiade allows the
pioduction of moie of all goods and seivices. Restiictions on tiade thus ieduce pioduction of goods
and seivices. Second, we see a lesson often missed in discussions of tiade: a nation's tiade policy has
nothing to do with its level of employment of its factois of pioduction. In oui example, when South
Ameiica and Euiope do not engage in tiade and pioduce at the midpoints of each of theii iespective
pioduction possibilities cuives, they each have full employment. With tiade, the two nations still opei-
ate on theii iespective pioduction possibilities cuives: they each have full employment. Tiade ceitainly
iedistiibutes employment in the two continents. In South Ameiica, employment shifts fiom computei
pioduction to food pioduction. In Euiope, it shifts fiom food pioduction to computei pioduction.
Once the shift is made, though, theie is no efect on employment in eithei continent.
Of couise, this idealized example would have all of South Ameiica's computei expeits becoming
faimeis while all of Euiope's faimeis become computei geeks! That is a bit much to swallow, but it is
meiely the iesult of assuming lineai pioduction possibilities cuives and complete specialization. In the
ieal woild, pioduction possibilities cuives aie concave, and the ieallocation of iesouices iequiied by
tiade is not neaily as diamatic. Still, fiee tiade can iequiie shifts in iesouices fiom one activity to an-
othei. These shifts pioduce enoimous benefts, but they do not come without costs.
Neaily all economists agiee that laigely uniestiicted tiade between countiies is desiiable; iestiic-
tions on tiade geneially foice the woild to opeiate inside its pioduction possibilities cuive. In some
cases iestiictions on tiade could be desiiable, but in the main, fiee tiade piomotes gieatei pioduction
of goods and seivices foi the woild's people. The iole of inteinational tiade is exploied in gieatei detail
in subsequent chapteis of this book.
3.2 Economic Giowth
An inciease in the physical quantity oi in the quality of factois of pioduction available to an economy
oi a technological gain will allow the economy to pioduce moie goods and seivices; it will shift the eco-
nomy's pioduction possibilities cuive outwaid. The piocess thiough which an economy achieves an
outwaid shift in its pioduction possibilities cuive is called economic growth. An outwaid shift in a
pioduction possibilities cuive is illustiated in Figuie 2.13. In Panel (a), a point such as N is not attain-
able; it lies outside the pioduction possibilities cuive. Giowth shifts the cuive outwaid, as in Panel (b),
making pieviously unattainable levels of pioduction possible.
II GUR 2. 13 conomic Growth and the Production PossibiIities Curve
Ar eccrcny c.p.b|e c| p.cJuc|rg t.c gccJs, A .rJ b, |s |r|t|.||y cpe..t|rg .t pc|rt V cr p.cJuct|cr pcss|b|||t|es
cu.ve OV| |r |.re| (.). C|ver t||s p.cJuct|cr pcss|b|||t|es cu.ve, t|e eccrcny ccu|J rct p.cJuce . ccnb|r.t|cr
suc| .s s|c.r by pc|rt |, .||c| ||es cuts|Je t|e cu.ve. Ar |rc.e.se |r t|e |.ctc.s c| p.cJuct|cr .v.||.b|e tc t|e
eccrcny .cu|J s|||t t|e cu.ve cut...J tc S|, .||c.|rg t|e c|c|ce c| . pc|rt suc| .s |, .t .||c| nc.e c| bct|
gccJs .||| be p.cJuceJ.
7he Sources of conomic Growth
Economic giowth implies an outwaid shift in an economy's pioduction possibilities cuive. Recall that
when we diaw such a cuive, we assume that the quantity and quality of the economy's factois of
42 PRINCIPLS OI CONOMICS
pioduction and its technology aie unchanged. Changing these will shift the cuive. Anything that in-
cieases the quantity oi quality of the factois of pioduction available to the economy oi that impioves
the technology available to the economy contiibutes to economic giowth.
Considei, foi example, the diamatic gains in human capital that have occuiied in the United States
since the beginning of the past centuiy. In 1900, about 3.3 of U.S. woikeis had completed a high
school education. By 2006, that peicentage iose almost to 92. Fewei than 1 of the woikeis in 1900
had giaduated fiom college; as late as 1940 only 3.3 had giaduated fiom college. By 2006, neaily 32
had giaduated fiom college. In addition to being bettei educated, today's woikeis have ieceived moie
and bettei tiaining on the job. They biing fai moie economically useful knowledge and skills to theii
woik than did woikeis a centuiy ago.
Moieovei, the technological changes that have occuiied within the past 100 yeais have gieatly ie-
duced the time and efoit iequiied to pioduce most goods and seivices. Automated pioduction has be-
come commonplace. Innovations in tianspoitation (automobiles, tiucks, and aiiplanes) have made the
movement of goods and people cheapei and fastei. A dizzying aiiay of new mateiials is available foi
manufactuiing. And the development of modein infoimation technology-including computeis, soft-
waie, and communications equipment-that seemed to pioceed at bieathtaking pace especially duiing
the fnal yeais of the last centuiy and continuing to the piesent has tiansfoimed the way we live and
woik.
Look again at the technological changes of the last few yeais desciibed in the Case in Point on ad-
vances in technology. Those examples of technological piogiess thiough applications of computei
technology-fiom new ways of mapping oil deposits to new methods of milking cows-helped piopel
the United States and othei economies to diamatic gains in the ability to pioduce goods and seivices.
They have helped shift the countiies' pioduction possibilities cuive outwaid. They have helped fuel
economic giowth.
Table 2.1 summaiizes the factois that have contiibuted to U.S. economic giowth in the past half
centuiy. When looking at the peiiod of 1948-2002 as a whole we see that about 60 of economic
giowth stems fiom incieases in the quantities of capital and laboi and 40 fiom incieases in the qualit-
ies of the factois of pioduction and impiovements in technology. In the most iecent peiiod,
1993-2002, howevei, these peicentages aie essentially ieveised, with a little less than 30 explained by
incieases in quantities of the factois of pioduction and a whopping 70 explained by impiovements in
factoi quality and technology.
CHAP7R 2 CONIRON7ING SCARCI7Y: CHOICS IN PRODUC7ION 43
7A8L 2. 1 Sources of U.S. conomic Growth, 1948-2002
ct.| cutput Ju.|rg t|e pe.|cJ s|c.r |rc.e.seJ s|x|c|J. |e c|..t s|c.s t|e pe.cert.ge c| t||s |rc.e.se .cccurteJ
|c. by |rc.e.ses |r t|e ,u.rt|ty c| |.bc. .rJ c| c.p|t.| .rJ by |rc.e.ses |r t|e ,u.||ty c| |.bc. .rJ c| c.p|t.| .rJ
|np.cvenerts |r tec|rc|cgy. |r t|e 19952002 pe.|cJ, t|e |rcc.pc..t|cr c| |r|c.n.t|cr tec|rc|cgy |eJ tc
|np.cvenerts |r t|e ,u.||ty c| c.p|t.| .rJ tec|rc|cgy t|.t g.e.t|y ccrt.|buteJ tc g.c.t|.
Year Percentage contribution to growth Period growth rate
Years 1948-2002 3.46%
|rc.e.se |r ,u.rt|ty c| |.bc. 21
|rc.e.se |r ,u.rt|ty c| c.p|t.| 41
|rc.e.se |r ,u.||ty c| |.bc. 10
|rc.e.se |r ,u.||ty c| c.p|t.| 20
|np.cveJ tec|rc|cgy 25
Years 1948-1973 3.99%
|rc.e.se |r ,u.rt|ty c| |.bc. 15
|rc.e.se |r ,u.rt|ty c| c.p|t.| 44
|rc.e.se |r ,u.||ty c| |.bc. 11
|rc.e.se |r ,u.||ty c| c.p|t.| 5
|np.cveJ tec|rc|cgy 25
Years 1973-1989 2.97%
|rc.e.se |r ,u.rt|ty c| |.bc. 31
|rc.e.se |r ,u.rt|ty c| c.p|t.| 39
|rc.e.se |r ,u.||ty c| |.bc.
|rc.e.se |r ,u.||ty c| c.p|t.| 12
|np.cveJ tec|rc|cgy 10
Years 1989-1995 2.43%
|rc.e.se |r ,u.rt|ty c| |.bc. 26
|rc.e.se |r ,u.rt|ty c| c.p|t.| 33
|rc.e.se |r ,u.||ty c| |.bc. 15
|rc.e.se |r ,u.||ty c| c.p|t.| 1
|np.cveJ tec|rc|cgy 11
Years 1995-2002 3.59%
|rc.e.se |r ,u.rt|ty c| |.bc. 19
|rc.e.se |r ,u.rt|ty c| c.p|t.| 8
|rc.e.se |r ,u.||ty c| |.bc. 5
|rc.e.se |r ,u.||ty c| c.p|t.| 4
|np.cveJ tec|rc|cgy 20
Source Based on Dale V. jorgenson, Accounting for Growth in the Information Age, Handbook of Economic Growth, Phillipe Aghion and Steven
Durlauf, eds. Amsterdam ^orth Holland, 2005.
Anothei way of looking at these data foi the most iecent peiiod is to notice that the inciease in the iate
of economic giowth between the 1989 to 1993 peiiod and the 1993 to 2002 peiiod of moie than one
peicentage point pei yeai is laigely explained by bettei-quality capital and bettei technology. The study
by economist Dale Joigenson on which the data shown in Table 2.1 aie deiived notes that these two
main contiibutois to highei economic giowth can be laigely attiibuted to the development of infoima-
tion technology and its incoipoiation in the woikplace.
Waiting for Growth
One key to giowth is, in efect, the willingness to wait, to postpone cuiient consumption in oidei to en-
hance futuie pioductive capability. When Stone Age people fashioned the fist tools, they weie spend-
ing time building capital iathei than engaging in consumption. They delayed cuiient consumption to
enhance theii futuie consumption; the tools they made would make them moie pioductive in the
futuie.
44 PRINCIPLS OI CONOMICS
market capitaIist economy
|ccrcny |r .||c| .escu.ces
..e gere..||y c.reJ by
p.|v.te |rJ|v|Ju.|s .|c |.ve
t|e pc.e. tc n.ke Jec|s|crs
.bcut t|e|. use.
command sociaIist
economy
|ccrcny |r .||c|
gcve.rnert |s t|e p.|n..y
c.re. c| c.p|t.| .rJ r.tu..|
.escu.ces .rJ |.s b.c.J
pc.e. tc .||cc.te t|e use c|
|.ctc.s c| p.cJuct|cr.
mixed economies
|ccrcny t|.t ccnb|re
e|enerts c| n..ket c.p|t.||st
.rJ c| ccnn.rJ scc|.||st
eccrcn|c systens.
Resouices society could have used to pioduce consumei goods aie being used to pioduce new cap-
ital goods and new knowledge foi pioduction instead-all to enhance futuie pioduction. An even moie
impoitant souice of giowth in many nations has been incieased human capital. Incieases in human
capital often iequiie the postponement of consumption. If you aie a college student, you aie engaged in
piecisely this efoit. You aie devoting time to study that could have been spent woiking, eaining in-
come, and thus engaging in a highei level of consumption. If you aie like most students, you aie mak-
ing this choice to postpone consumption because you expect it will allow you to eain moie income, and
thus enjoy gieatei consumption, in the futuie.
Think of an economy as being able to pioduce two goods, capital and consumei goods (those
destined foi immediate use by consumeis). By focusing on the pioduction of consumei goods, the
people in the economy will be able to enjoy a highei standaid of living today. If they ieduce theii con-
sumption-and theii standaid of living-today to enhance theii ability to pioduce goods and seivices
in the futuie, they will be able to shift theii pioduction possibilities cuive outwaid. That may allow
them to pioduce even moie consumei goods. A decision foi gieatei giowth typically involves the sac-
iifce of piesent consumption.
3.3 Aienas foi Choice: A Compaiison of Economic Systems
Undei what ciicumstances will a nation achieve emciency in the use of its factois of pioduction: The
discussion above suggested that Chiistie Rydei would have an incentive to allocate hei plants emciently
because by doing so she could achieve gieatei output of skis and snowboaids than would be possible
fiom inemcient pioduction. But why would she want to pioduce moie of these two goods-oi of any
goods: Why would decision makeis thioughout the economy want to achieve such emciency:
Economists assume that piivately owned fims seek to maximize theii piofts. The diive to maxim-
ize piofts will lead fims such as Alpine Spoits to allocate iesouices emciently to gain as much pioduc-
tion as possible fiom theii factois of pioduction. But whethei fims will seek to maximize piofts de-
pends on the natuie of the economic system within which they opeiate.
CIassifying conomic Systems
Each of the woild's economies can be viewed as opeiating somewheie on a spectium between maiket
capitalism and command socialism. In a market capitalist economy, iesouices aie geneially owned
by piivate individuals who have the powei to make decisions about theii use. A maiket capitalist sys-
tem is often iefeiied to as a fiee enteipiise economic system. In a command socialist economy, the
goveinment is the piimaiy ownei of capital and natuial iesouices and has bioad powei to allocate the
use of factois of pioduction. Between these two categoiies lie mixed economies that combine ele-
ments of maiket capitalist and of command socialist economic systems.
No economy iepiesents a puie case of eithei maiket capitalism oi command socialism. To deteim-
ine wheie an economy lies between these two types of systems, we evaluate the extent of goveinment
owneiship of capital and natuial iesouices and the degiee to which goveinment is involved in de-
cisions about the use of factois of pioduction.
The diagiam below suggests the spectium of economic systems. Maiket capitalist economies lie to-
waid the left end of this spectium; command socialist economies appeai towaid the iight. Mixed eco-
nomies lie in between. The maiket capitalist end of the spectium includes countiies such as the United
States, the United Kingdom, and Chile. Hong Kong, though now pait of China, has a long histoiy as a
maiket capitalist economy and is geneially iegaided as opeiating at the maiket capitalist end of the
spectium. Countiies at the command socialist end of the spectium include Noith Koiea and Cuba.
II GUR 2. 14 conomic Systems
Some Euiopean economies, such as Fiance, Geimany, and Sweden, have a sumciently high degiee of
iegulation that we considei them as opeiating moie towaid the centei of the spectium. Russia and Ch-
ina, which long opeiated at the command socialist end of the spectium, can now be consideied mixed
economies. Most economies in Latin Ameiica once opeiated towaid the iight end of the spectium.
While theii goveinments did not exeicise the extensive owneiship of capital and natuial iesouices that
aie one chaiacteiistic of command socialist systems, theii goveinments did impose extensive
CHAP7R 2 CONIRON7ING SCARCI7Y: CHOICS IN PRODUC7ION 45
iegulations. Many of these nations aie in the piocess of caiiying out economic iefoims that will move
them fuithei in the diiection of maiket capitalism.
The global shift towaid maiket capitalist economic systems that occuiied in the 1980s and 1990s
was in laige pait the iesult of thiee impoitant featuies of such economies. Fiist, the emphasis on indi-
vidual owneiship and decision-making powei has geneially yielded gieatei individual fieedom than
has been available undei command socialist oi some moie heavily iegulated mixed economic systems
that lie towaid the command socialist end of the spectium. People seeking political, ieligious, and eco-
nomic fieedom have thus giavitated towaid maiket capitalism. Second, maiket economies aie moie
likely than othei systems to allocate iesouices on the basis of compaiative advantage. They thus tend to
geneiate highei levels of pioduction and income than do othei economic systems. Thiid, maiket
capitalist-type systems appeai to be the most conducive to entiepieneuiial activity.
Suppose Chiistie Rydei had the same thiee plants we consideied eailiei in this chaptei but was op-
eiating in a mixed economic system with extensive goveinment iegulation. In such a system, she might
be piohibited fiom tiansfeiiing iesouices fiom one use to anothei to achieve the gains possible fiom
compaiative advantage. If she weie opeiating undei a command socialist system, she would not be the
ownei of the plants and thus would be unlikely to pioft fiom theii emcient use. If that weie the case,
theie is no ieason to believe she would make any efoit to assuie the emcient use of the thiee plants.
Geneially speaking, it is economies towaid the maiket capitalist end of the spectium that ofei the
gieatest inducement to allocate iesouices on the basis of compaiative advantage. They tend to be moie
pioductive and to delivei highei mateiial standaids of living than do economies that opeiate at oi neai
the command socialist end of the spectium.
Maiket capitalist economies iely on economic fieedom. Indeed, one way we can assess the degiee
to which a countiy can be consideied maiket capitalist is by the degiee of economic fieedom it peimits.
Seveial oiganizations have attempted to compaie economic fieedom in vaiious countiies. One of the
most extensive compaiisons is a joint annual efoit by the Heiitage Foundation and The Vall Street
journal. The 2008 iating was based on policies in efect in 162 nations eaily that yeai. The iepoit ianks
these nations on the basis of such things as the degiee of iegulation of fims, tax levels, and iestiictions
on inteinational tiade. Hong Kong ianked as the fieest economy in the woild. Noith Koiea ieceived
the dubious distinction of being the least fiee.
46 PRINCIPLS OI CONOMICS
II GUR 2. 15 conomic Ireedom and Income
|e |c.|.crt.| .x|s s|c.s t|e Jeg.ee c| eccrcn|c |.eeJcn|.ee, ncst|y |.ee, ncst|y ur|.ee, .rJ
.ep.esseJ.ccc.J|rg tc t|e ne.su.es useJ by t|e |e.|t.ge |curJ.t|cr .rJ |e /o|| .|ee| .ooo|. |e g..p|
s|c.s t|e .e|.t|crs||p bet.eer eccrcn|c |.eeJcn .rJ pe. c.p|t. |rccne. curt.|es .|t| ||g|e. Jeg.ees c|
eccrcn|c |.eeJcn terJeJ tc |.ve ||g|e. pe. c.p|t. |rccnes.
Source Vorld Bank, Vorld Development Indicators Online, available by subscription at www.worldbank.org/data, Central Intelligence Agency, The
Vorld Factbook 2004, available at http//www.cia.gov/cia/publications/factbook/index.html for the following countries Bahamas, Burma, Cuba,
Cyprus, Equatorial Guinea, ^orth Korea, Libya, Qatar, Suriname, Taiwan, Zimbabwe, Marc A. Miles, Edwin j. Feulner, and Mary Anastasia
OGrady, 2005 Index of Economic Freedom (Vashington, D.C. The Heritage Foundation and Dow jones o Company, Inc., 2005), at
www.heritage.org/index.
It seems ieasonable to expect that the gieatei the degiee of economic fieedom a countiy peimits, the
gieatei the amount of income pei peison it will geneiate. This pioposition is illustiated in Figuie 2.13.
The gioup of countiies categoiized as fiee" geneiated the highest incomes in the Heiitage Founda-
tion/Vall Street journal study; those iated as iepiessed" had the lowest. The study also found that
countiies that ovei the last decade have done the most to impiove theii positions in the economic fiee-
dom iankings have also had the highest iates of giowth. We must be waiy of slipping into the fallacy of
false cause by concluding fiom this evidence that economic fieedom geneiates highei incomes. It could
be that highei incomes lead nations to opt foi gieatei economic fieedom. But in this case, it seems ieas-
onable to conclude that, in geneial, economic fieedom does lead to highei incomes.
3.4 Goveinment in a Maiket Economy
The pioduction possibilities model piovides a menu of choices among alteinative combinations of
goods and seivices. Given those choices, which combinations will be pioduced:
In a maiket economy, this question is answeied in laige pait thiough the inteiaction of individual
buyeis and selleis. As we have alieady seen, goveinment plays a iole as well. It may seek to encouiage
gieatei consumption of some goods and discouiage consumption of otheis. In the United States, foi
example, taxes imposed on cigaiettes discouiage smoking, while special tieatment of piopeity taxes
and moitgage inteiest in the fedeial income tax encouiages home owneiship. Goveinment may tiy to
stop the pioduction and consumption of some goods altogethei, as many goveinments do with diugs
such as heioin and cocaine. Goveinment may supplement the piivate consumption of some goods by
pioducing moie of them itself, as many U.S. cities do with golf couises and tennis couits. In othei
cases, theie may be no piivate maiket foi a good oi seivice at all. In the choice between secuiity and
defense veisus all othei goods and seivices outlined at the beginning of this chaptei, goveinment agen-
cies aie viitually the sole piovideis of secuiity and national defense.
All nations also iely on goveinment to piovide defense, enfoice laws, and iedistiibute income.
Even maiket economies iely on goveinment to iegulate the activities of piivate fims, to piotect the
CHAP7R 2 CONIRON7ING SCARCI7Y: CHOICS IN PRODUC7ION 47
enviionment, to piovide education, and to pioduce a wide iange of othei goods and seivices. Govein-
ment's iole may be limited in a maiket economy, but it iemains fundamentally impoitant.
k Y 7 A k A W A Y S
< |e |Je.s c| ccnp...t|ve .Jv.rt.ge .rJ spec|.||..t|cr suggest t|.t .est.|ct|crs cr |rte.r.t|cr.| t..Je ..e
||ke|y tc .eJuce p.cJuct|cr c| gccJs .rJ se.v|ces.
< |ccrcn|c g.c.t| |s t|e .esu|t c| |rc.e.s|rg t|e ,u.rt|ty c. ,u.||ty c| .r eccrcnys |.ctc.s c| p.cJuct|cr
.rJ c| .Jv.rces |r tec|rc|cgy.
< |c||c|es tc erccu..ge g.c.t| gere..||y |rvc|ve pcstpcr|rg ccrsunpt|cr tc |rc.e.se c.p|t.| .rJ |un.r
c.p|t.|.
< V..ket c.p|t.||st eccrcn|es |.ve gere..||y p.cveJ nc.e p.cJuct|ve t|.r n|xeJ c. ccnn.rJ scc|.||st
eccrcn|es.
< Ccve.rnert p|.ys . c.uc|.| .c|e |r .ry n..ket eccrcny.
7 R Y I 7 !
|... . p.cJuct|cr pcss|b|||t|es cu.ve |c. .r eccrcny t|.t c.r p.cJuce t.c gccJs, | p|.ye.s .rJ .ckets.
+cu Jc rct |.ve runbe.s |c. t||s creust J... . cu.ve .|t| t|e usu.| bc.eJcut s|.pe. |ut t|e ,u.rt|ty
c| | p|.ye.s pe. pe.|cJ cr t|e ve.t|c.| .x|s .rJ t|e ,u.rt|ty c| .ckets pe. pe.|cJ cr t|e |c.|.crt.| .x|s. |c.
n..k . pc|rt A cr t|e cu.ve ycu |.ve J...r, exterJ JctteJ ||res |.cn t||s pc|rt tc t|e |c.|.crt.| .rJ ve.t|c.|
.xes. V..k t|e |r|t|.| ,u.rt|t|es c| t|e t.c gccJs .s |
^
.rJ '
^
, .espect|ve|y. |xp|.|r .|y, |r t|e .bserce c|
eccrcn|c g.c.t|, .r |rc.e.se |r .cket p.cJuct|cr .e,u|.es . .eJuct|cr |r t|e p.cJuct|cr c| | p|.ye.s. |c.
s|c. |c. eccrcn|c g.c.t| ccu|J |e.J tc .r |rc.e.se |r t|e p.cJuct|cr c| bct| gccJs.
Case in Point: 7he uropean Union and the Production PossibiIities Curve
2010 jupiterimages Corporation
|c.neJ by t|e V..st.|c|t .e.ty c| 1993, |e |u.cpe.r |r|cr .ep.eserts cre c| t|e bc|Jest e|c.ts c| cu.
t|ne tc exp|c|t t|e t|ec.y c| ccnp...t|ve .Jv.rt.ge. |e .e.ty scug|t tc e||n|r.te .|| t..Je b...|e.s bet.eer
t|e |u.cpe.r |r|crs nenbe.s. |t est.b||s|eJ . |u.cpe.r |..||.nert .rJ . |u.cpe.r ert..| b.rk. |e b.rk
|rt.cJuceJ t|e eu.c |r 1999, . cu..ercy t|.t .ep|.ceJ r.t|cr.| cu..erc|es suc| .s t|e Ce.n.r Jeutsc|e n..k
48 PRINCIPLS OI CONOMICS
.rJ t|e |.erc| |..rc. At |.st, t|e eu.c ..s useJ cr|y |c. t..rs.ct|crs bet.eer b.rks. 320 n||||cr pecp|e |r 15
|| r.t|crs (Aust.|., be|g|un, yp.us, ||r|.rJ, |..rce, Ce.n.ry, C.eece, |.e|.rJ, |t.|y, |uxenbcu.g, V.|t., t|e
|et|e.|.rJs, |c.tug.|, S|cver|., .rJ Sp.|r) useJ t|e eu.c by 2008. \|||e t|e Jc||.. ccrt|rues tc be nc.e
.|Je|y useJ, t|e tct.| v.|ue c| eu.cs |r c|.cu|.t|cr exceeJs t|.t c| Jc||..s.
|e ncvenert tc...J |u.cpe.r |rteg..t|cr c.r be J.teJ b.ck nc.e t|.r |.|| . certu.y. |r 1950, ust |ve
ye..s .|te. . ... t|.t |.J Jev.st.teJ nuc| c| t|e .c.|J, |cbe.t Sc|un.r, t|e |.erc| V|r|ste. c| |c.e|gr
A|.|.s, p.cpcseJ . ur|cr bet.eer |..rce .rJ Ce.n.ry tc cccpe..te |r t|e p.cJuct|cr c| |.cr .rJ stee|. |r
t|e ccrtext c| t|e t|ne, Sc|un.rs p.cpcs.| ..s . ..J|c.| cre. \c.|J \.. || |.J begur .|t| Ce.n.rys .t
tenpt tc se|.e ccrt.c| c| |u.cpe.rJ u|t|n.te|y t|e .c.|J. '.p.r .rJ |t.|y c|reJ Ce.n.ry |r t||s e|c.t. Ce.
n.ry |.J c.ptu.eJ |..rce, |..rce |.J beer ||be..teJ |r 1944 by t|e A|||eJ |rv.s|cr |r |c.n.rJy. |e p.c
pcs.| |c. cccpe..t|cr bet.eer t.c ccurt.|es t|.t |.J beer t|e ncst b|tte. c| eren|es ..s . .evc|ut|cr..y
cre. Sc|un.rs speec|, Je||ve.eJ cr V.y 9, 1950, |s ce|eb..teJ t|.cug|cut |u.cpe .s |u.cpe |.y.
|r e|ect, t|e |u.cpe.r |r|cr |.s c.e.teJ .r ert|ty ve.y nuc| ||ke t|e |r|teJ St.tes. curt.|es .|t||r t|e
|u.cpe.r |r|cr .et.|r t|e|. c.r |.rgu.ges .rJ cu|tu..| J||e.erces, but t|ey |.ve ceJeJ . .en..k.b|e Je
g.ee c| scve.e|grty tc t|e |r|cr. Venbe.s c| t|e |u.cpe.r |r|cr c.r t..Je .s |.ee|y .|t| e.c| ct|e. .s c.r
st.tes .|t||r t|e |r|teJ St.tes. 'ust .s t|e |.S. crst|tut|cr p.c||b|ts st.tes |.cn .est.|ct|rg t..Je .|t| ct|e.
st.tes, t|e |u.cpe.r |r|cr |.s J|sn.rt|eJ .|| |c.ns c| .est.|ct|crs t|.t ccurt.|es .|t||r t|e |r|cr useJ tc |n
pcse cr cre .rct|e.. 'ust .s .est.|ct|crs cr spec|.||..t|cr .ncrg Vs. |yJe.s p|.rts |r A|p|re Spc.ts .cu|J
|.ve |c.ceJ |t tc cpe..te |rs|Je |ts p.cJuct|cr pcss|b|||t|es cu.ve, .est.|ct|crs t|.t |.J ex|steJ .ncrg nen
be.s c| t|e |u.cpe.r |r|cr crce put t|e nenbe.s c| t|e |r|cr |rs|Je t|e|. cc||ect|ve p.cJuct|cr pcss|b|||t|es
cu.ve.
|e expe.|nert .ppe..s tc |.ve beer . success. ..Je .ncrg nenbe. r.t|crs |.s exp.rJeJ s|..p|y. A stuJy
by ..ner ||.. Vc.., .r eccrcn|st .t t|e |r|ve.s|ty c| .st|||.|. V.rc|. |r Sp.|r, |curJ t|.t t|e bu|k c| t|e
exp.rJeJ t..Je .|t||r t|e |r|cr ..s t..Je .|t||r |rJust.|es .rJ t|.t |t ..s J.|ver by ccnp...t|ve .Jv.rt
.ge. |r p..t|cu|.., s|e |curJ t|.t ccurt.|es |r t|e rc.t|e.r p..t c| t|e |r|cr, suc| .s |..rce .rJ Ce.n.ry, ter
JeJ tc spec|.||.e |r .e|.t|ve|y ||g|v.|ueJ gccJsc|ce e,u|pnert .rJ e|ect.|c.| gccJs.|||e ccurt.|es |r
t|e scut|e.r p..t c| t|e |r|cr spec|.||.eJ |r .e|.t|ve|y |c.v.|ueJ gccJs suc| .s |ccJ .rJ text||e p.cJucts. |r
t..Je .|t||r t|e c|ct||rg |rJust.y, ccurt.|es suc| .s |t.|y terJ tc spec|.||.e |r t|e p.cJuct|cr c| ||g|e.v.|ueJ
c|ct||rg, .|||e |c.e.|rccne ccurt.|es suc| .s |c.tug.| spec|.||.e |r t|e p.cJuct|cr c| c|e.pe. c|ct||rg. |r
sp..k||rg .|res, |..rce spec|.||.es |r t|e ||g|e.,u.||ty erJ c| t|e spect.un, .|||e Sp.|r spec|.||.es |r t|e
|c.,u.||ty erJ. S|n||..|y, Ce.n.ry spec|.||.es |r t|e p.cJuct|cr c| ||g|e.,u.||ty c..s .|||e Sp.|r spec|.||.es
|r |c.e.,u.||ty ve||c|es. S|n||.. exc|.rges cccu. .c.css . .|Je ..rge c| gccJs .rJ se.v|ces.
||.. Vc.. |curJ t|.t ccnp...t|ve .Jv.rt.ge terJeJ tc cc..espcrJ tc |rccne |eve|s. curt.|es |r t|e rc.t|
e.r p..t c| t|e |u.cpe.r |r|cr terJ tc |.ve ||g| pe. c.p|t. |rccnes .rJ ||g| |eve|s c| |un.r c.p|t.| .rJ
tec|rc|cgyt|ese ccurt.|es g.|reJ by spec|.||.|rg |r t|e p.cJuct|cr c| ||g|v.|ueJ gccJs. curt.|es |r t|e
scut|e.r p..t c| t|e |r|cr .|sc g.|reJ by spec|.||..t|cr|r t|e p.cJuct|cr c| |c.v.|ueJ gccJs. ||s spec|.|
|..t|cr |.s |rc.e.seJ t|e .e||..e c| pecp|e t|.cug|cut t|e |r|cr.
.ooce oe ||o /oo. |e |o|e o| ooo||.e ^J.o|oe | oJe /|||| |Jo||e ^ |oe| |o|o ^ooc| |o ||e |ooeo ||o.'
/e||.||c|o||||c|e ^c||. 3S2 (2002:. 2936
CHAP7R 2 CONIRON7ING SCARCI7Y: CHOICS IN PRODUC7ION 49
A N S W R 7 O 7 R Y I 7 ! P R O 8 L M
+cu. |.st p.cJuct|cr pcss|b|||t|es cu.ve s|cu|J .esenb|e t|e cre |r |.re| (.). St..t|rg .t pc|rt A, .r |rc.e.se |r
.cket p.cJuct|cr .e,u|.es . ncve Jc.r .rJ tc t|e .|g|t .|crg t|e cu.ve, .s s|c.r by t|e ...c., .rJ t|us .
.eJuct|cr |r t|e p.cJuct|cr c| | p|.ye.s. A|te.r.t|ve|y, || t|e.e |s eccrcn|c g.c.t|, |t s|||ts t|e p.cJuct|cr
pcss|b|||t|es cu.ve cut...J, .s |r |.re| (b). ||s s|||t .||c.s .r |rc.e.se |r p.cJuct|cr c| bct| gccJs, .s sug
gesteJ by t|e ...c..
4. REVIEW AND PRACTICE
Summary
|ccrcn|cs Je.|s .|t| c|c|ces. |r t||s c|.pte. .e |.ve ex.n|reJ nc.e c..e|u||y t|e ..rge c| c|c|ces |r p.c
Juct|cr t|.t nust be n.Je |r .ry eccrcny. |r p..t|cu|.., .e |cckeJ .t c|c|ces |rvc|v|rg t|e .||cc.t|cr c| .r
eccrcnys |.ctc.s c| p.cJuct|cr. |.bc., c.p|t.|, .rJ r.tu..| .escu.ces.
|r .JJ|t|cr, |r .ry eccrcny, t|e |eve| c| tec|rc|cgy p|.ys . key .c|e |r Jete.n|r|rg |c. p.cJuct|ve t|e
|.ctc.s c| p.cJuct|cr .||| be. |r . n..ket eccrcny, ert.ep.ereu.s c.g.r|.e |.ctc.s c| p.cJuct|cr .rJ .ct tc |r
t.cJuce tec|rc|cg|c.| c|.rge.
|e p.cJuct|cr pcss|b|||t|es ncJe| |s . Jev|ce t|.t .ss|sts us |r t||rk|rg .bcut n.ry c| t|e c|c|ces .bcut .e
scu.ce .||cc.t|cr |r .r eccrcny. |e ncJe| .ssunes t|.t t|e eccrcny |.s |.ctc.s c| p.cJuct|cr t|.t ..e
|xeJ |r bct| ,u.rt|ty .rJ ,u.||ty. \|er |||ust..teJ g..p||c.||y, t|e p.cJuct|cr pcss|b|||t|es ncJe| typ|c.||y
||n|ts cu. .r.|ys|s tc t.c gccJs. C|ver t|e eccrcnys |.ctc.s c| p.cJuct|cr .rJ tec|rc|cgy, t|e eccrcny c.r
p.cJuce v..|cus ccnb|r.t|crs c| t|e t.c gccJs. || |t uses |ts |.ctc.s c| p.cJuct|cr e|c|ert|y .rJ |.s |u|| en
p|cynert, |t .||| be cpe..t|rg cr t|e p.cJuct|cr pcss|b|||t|es cu.ve.
.c c|...cte.|st|cs c| t|e p.cJuct|cr pcss|b|||t|es cu.ve ..e p..t|cu|..|y |npc.t.rt. ||.st, |t |s Jc.r...J s|cp
|rg. ||s .e|ects t|e sc..c|ty c| t|e |.ctc.s c| p.cJuct|cr .v.||.b|e tc t|e eccrcny, p.cJuc|rg nc.e c| cre
gccJ .e,u|.es g|v|rg up scne c| t|e ct|e.. SeccrJ, t|e cu.ve |s bc.eJ cut. Arct|e. ..y c| s.y|rg t||s |s tc
s.y t|.t t|e cu.ve gets steepe. .s .e ncve |.cn |e|t tc .|g|t, t|e .bsc|ute v.|ue c| |ts s|cpe |s |rc.e.s|rg. |.c
Juc|rg e.c| .JJ|t|cr.| ur|t c| t|e gccJ cr t|e |c.|.crt.| .x|s .e,u|.es . g.e.te. s.c.||ce c| t|e gccJ cr t|e
ve.t|c.| .x|s t|.r J|J t|e p.ev|cus ur|ts p.cJuceJ. ||s |.ct, c.||eJ t|e |.. c| |rc.e.s|rg cppc.tur|ty ccst, |s t|e
|rev|t.b|e .esu|t c| e|c|ert c|c|ces |r p.cJuct|crc|c|ces b.seJ cr ccnp...t|ve .Jv.rt.ge.
|e p.cJuct|cr pcss|b|||t|es ncJe| |.s |npc.t.rt |np||c.t|crs |c. |rte.r.t|cr.| t..Je. |t suggests t|.t |.ee
t..Je .||| .||c. ccurt.|es tc spec|.||.e |r t|e p.cJuct|cr c| gccJs .rJ se.v|ces |r .||c| t|ey |.ve . ccnp...t
|ve .Jv.rt.ge. ||s spec|.||..t|cr |rc.e.ses t|e p.cJuct|cr c| .|| gccJs .rJ se.v|ces.
|rc.e.s|rg t|e ,u.rt|ty c. ,u.||ty c| |.ctc.s c| p.cJuct|cr .rJ/c. |np.cv|rg tec|rc|cgy .||| s|||t t|e p.cJuc
t|cr pcss|b|||t|es cu.ve cut...J. ||s p.ccess |s c.||eJ eccrcn|c g.c.t|. |r t|e |.st 50 ye..s, eccrcn|c g.c.t|
|r t|e |r|teJ St.tes |.s .esu|teJ c||e|y |.cn |rc.e.ses |r |un.r c.p|t.| .rJ |.cn tec|rc|cg|c.| .Jv.rce.
50 PRINCIPLS OI CONOMICS
|c|ces ccrce.r|rg t|e use c| sc..ce .escu.ces t.ke p|.ce .|t||r t|e ccrtext c| . set c| |rst|tut|cr.| ....rge
nerts t|.t Je|re .r eccrcn|c systen. |e p.|rc|p.| J|st|rct|crs bet.eer systens ||e |r t|e Jeg.ee tc .||c|
c.re.s||p c| c.p|t.| .rJ r.tu..| .escu.ces .rJ Jec|s|cr n.k|rg .ut|c.|ty cve. sc..ce .escu.ces ..e |e|J by
gcve.rnert c. by p.|v.te |rJ|v|Ju.|s. |ccrcn|c systens |rc|uJe n..ket c.p|t.||st, n|xeJ, .rJ ccnn.rJ sc
c|.||st eccrcn|es. Ar |rc.e.s|rg bcJy c| ev|Jerce suggests t|.t n..ket c.p|t.||st eccrcn|es terJ tc be ncst
p.cJuct|ve, n.ry ccnn.rJ scc|.||st .rJ n|xeJ eccrcn|es ..e ncv|rg |r t|e J|.ect|cr c| n..ket c.p|t.||st
systens.
|e p.esunpt|cr |r |.vc. c| n..ketb.seJ systens Jces rct p.ec|uJe . .c|e |c. gcve.rnert. Ccve.rnert |s
recess..y tc p.cv|Je t|e systen c| |..s cr .||c| n..ket systens ..e |curJeJ. |t n.y .|sc be useJ tc p.cv|Je
ce.t.|r gccJs .rJ se.v|ces, tc |e|p |rJ|v|Ju.|s |r reeJ, .rJ tc .egu|.te t|e .ct|crs c| |rJ|v|Ju.|s .rJ |.ns.
CHAP7R 2 CONIRON7ING SCARCI7Y: CHOICS IN PRODUC7ION 51
C O N C P 7 P R O 8 L M S
1. |c. Jces . cc||ege eJuc.t|cr |rc.e.se cres |un.r c.p|t.|`
2. \|y Jces t|e Jc.r...Js|cp|rg p.cJuct|cr pcss|b|||t|es cu.ve |np|y t|.t |.ctc.s c| p.cJuct|cr ..e
sc..ce`
3. |r .|.t ..ys ..e t|e bc.eJcut s|.pe c| t|e p.cJuct|cr pcss|b|||t|es cu.ve .rJ t|e |.. c| |rc.e.s|rg
cppc.tur|ty ccst .e|.teJ`
4. \|.t |s t|e .e|.t|crs||p bet.eer t|e ccrcept c| ccnp...t|ve .Jv.rt.ge .rJ t|e |.. c| |rc.e.s|rg
cppc.tur|ty ccst`
5. Suppcse .r eccrcny c.r p.cJuce t.c gccJs, A .rJ b. |t |s rc. cpe..t|rg .t pc|rt | cr p.cJuct|cr
pcss|b|||t|es cu.ve |. Ar |np.cvenert |r t|e tec|rc|cgy .v.||.b|e tc p.cJuce gccJ A s|||ts t|e cu.ve tc
S, .rJ t|e eccrcny se|ects pc|rt |. |c. Jces t||s c|.rge .|ect t|e cppc.tur|ty ccst c| p.cJuc|rg .r
.JJ|t|cr.| ur|t c| gccJ b`
6. cu|J . r.t|crs p.cJuct|cr pcss|b|||t|es cu.ve eve. s|||t |r...J` |xp|.|r .|.t suc| . s|||t .cu|J ne.r,
.rJ J|scuss everts t|.t n|g|t c.use suc| . s|||t tc cccu..
. Suppcse b|ueeyeJ pecp|e .e.e b.rreJ |.cn .c.k|rg. |c. .cu|J t||s .|ect . r.t|crs p.cJuct|cr
pcss|b|||t|es cu.ve`
8. |v.|u.te t||s st.tenert. |e |.S. eccrcny ccu|J .c||eve g.e.te. g.c.t| by Jevct|rg |e.e. .escu.ces tc
ccrsunpt|cr .rJ nc.e tc |rvestnert, |t |c||c.s t|.t suc| . s|||t .cu|J be Jes|..b|e.
9. .c ccurt.|es, Spc.ts|.rJ .rJ |ccJ|.rJ, |.ve s|n||.. tct.| ,u.rt|t|es c| |.bc., c.p|t.|, .rJ r.tu..|
.escu.ces. bct| c.r p.cJuce t.c gccJs, |gs .rJ |cctb.||s. Spc.ts|.rJs .escu.ces ..e p..t|cu|..|y .e||
su|teJ tc t|e p.cJuct|cr c| |cctb.||s but ..e rct ve.y p.cJuct|ve |r p.cJuc|rg |gs. |ccJ|.rJs .escu.ces
..e ve.y p.cJuct|ve .|er useJ |c. |gs but ..e rct c.p.b|e c| p.cJuc|rg n.ry |cctb.||s. |r .||c| ccurt.y
|s t|e ccst c| .JJ|t|cr.| |cctb.||s gere..||y g.e.te.` |xp|.|r.
10. Suppcse . ccurt.y |s ccnn|tteJ tc us|rg |ts .escu.ces b.seJ cr t|e .eve.se c| ccnp...t|ve .Jv.rt.ge
Jcct.|re. |t |.st t..rs|e.s t|cse .escu.ces |c. .||c| t|e ccst |s g.e.test, rct |c.est. |esc.|be t||s ccurt.ys
p.cJuct|cr pcss|b|||t|es cu.ve.
11. |e |.S. crst|tut|cr b.rs st.tes |.cn .est.|ct|rg |npc.ts c| gccJs .rJ se.v|ces |.cn ct|e. st.tes.
Suppcse t||s .est.|ct|cr J|J rct ex|st .rJ t|.t st.tes .e.e .||c.eJ tc ||n|t |npc.ts c| gccJs .rJ se.v|ces
p.cJuceJ |r ct|e. st.tes. |c. Jc ycu t||rk t||s .cu|J .|ect |.S. cutput` |xp|.|r.
12. by 1993, r.t|crs |r t|e |u.cpe.r |r|cr (||) |.J e||n|r.teJ .|| b...|e.s tc t|e |c. c| gccJs, se.v|ces,
|.bc., .rJ c.p|t.| .c.css t|e|. bc.Je.s. |ver suc| t||rgs .s ccrsune. p.ctect|cr |..s .rJ t|e types c|
p|ugs .e,u|.eJ tc p|ug |r .pp||.rces |.ve beer st.rJ..J|.eJ tc ersu.e t|.t t|e.e .||| be rc b...|e.s tc
t..Je. |c. Jc ycu t||rk t||s e||n|r.t|cr c| t..Je b...|e.s .|ecteJ || cutput`
13. |c. J|J t|e tec|rc|cg|c.| c|.rges Jesc.|beJ |r t|e .se |r |c|rt ec|rc|cgy uts csts, bccsts
|.cJuct|v|ty .rJ |.c|ts .|ect t|e p.cJuct|cr pcss|b|||t|es cu.ve |c. t|e |r|teJ St.tes`
52 PRINCIPLS OI CONOMICS
N U M R I C A L P R O 8 L M S
1. |.t|.r c.r nc. |cu. |..rs |r . J.y c. p|.rt 20 t.ees |r . J.y.
.. |... |.t|.rs p.cJuct|cr pcss|b|||t|es cu.ve |c. nc.|rg |..rs .rJ p|.rt|rg t.ees. Assune t|e
p.cJuct|cr pcss|b|||t|es cu.ve |s ||re.. .rJ put t|e ,u.rt|ty c| |..rs nc.eJ pe. J.y cr t|e
|c.|.crt.| .x|s .rJ t|e ,u.rt|ty c| t.ees p|.rteJ pe. J.y cr t|e ve.t|c.| .x|s.
b. \|.t |s |.t|.rs cppc.tur|ty ccst c| p|.rt|rg t.ees`
c. \|.t |s |.t|.rs cppc.tur|ty ccst c| nc.|rg |..rs`
2. |.v|J c.r nc. |cu. |..rs |r . J.y c. p|.rt |cu. t.ees |r . J.y.
.. |... |.v|Js p.cJuct|cr pcss|b|||t|es cu.ve |c. nc.|rg |..rs .rJ p|.rt|rg t.ees. Ag.|r, .ssune
. ||re.. p.cJuct|cr pcss|b|||t|es cu.ve .rJ put t|e ,u.rt|ty c| |..rs nc.eJ pe. J.y cr t|e
|c.|.crt.| .x|s.
b. \|.t |s |.v|Js cppc.tur|ty ccst c| p|.rt|rg t.ees`
c. \|.t |s |.v|Js cppc.tur|ty ccst c| nc.|rg |..rs`
3. C|ver t|e p.cJuct|cr |r|c.n.t|cr |r p.cb|ens 1 .rJ 2 .bcve, .|c |.s t|e ccnp...t|ve .Jv.rt.ge |r
p|.rt|rg t.ees` Vc.|rg |..rs`
4. |e ex||b|ts be|c. Jesc.|be t|e p.cJuct|cr pcss|b|||t|es |c. Ce.n.ry .rJ u.key.
.. \|.t |s t|e s|cpe c| Ce.n.rys p.cJuct|cr pcss|b|||t|es cu.ve`
b. \|.t |s t|e s|cpe c| u.keys p.cJuct|cr pcss|b|||t|es cu.ve`
c. \|.t |s t|e cppc.tur|ty ccst c| p.cJuc|rg s||.ts |r Ce.n.ry`
J. \|.t |s t|e cppc.tur|ty ccst c| p.cJuc|rg s||.ts |r u.key`
e. \|.t |s t|e cppc.tur|ty ccst c| p.cJuc|rg cpt|c.| |rst.unerts |r Ce.n.ry`
|. \|.t |s t|e cppc.tur|ty ccst c| p.cJuc|rg cpt|c.| |rst.unerts |r u.key`
g. |r .||c| gccJ Jces Ce.n.ry |.ve . ccnp...t|ve .Jv.rt.ge`
|. |r .||c| gccJ Jces u.key |.ve . ccnp...t|ve .Jv.rt.ge`
5. |e r.t|cr c| |e|su.e|.rJ c.r p.cJuce t.c gccJs, b|cyc|es .rJ bc.||rg b.||s. |e .este.r .eg|cr c|
|e|su.e|.rJ c.r, || |t Jevctes .|| |ts .escu.ces tc b|cyc|e p.cJuct|cr, p.cJuce 100 b|cyc|es pe. ncrt|.
A|te.r.t|ve|y, |t ccu|J Jevcte .|| |ts .escu.ces tc bc.||rg b.||s .rJ p.cJuce 400 pe. ncrt|c. |t ccu|J
p.cJuce .ry ccnb|r.t|cr c| b|cyc|es .rJ bc.||rg b.||s |y|rg cr . st..|g|t ||re bet.eer t|ese t.c
ext.enes.
.. |... . p.cJuct|cr pcss|b|||t|es cu.ve |c. .este.r |e|su.e|.rJ (.|t| b|cyc|es cr t|e ve.t|c.| .x|s).
b. \|.t |t |s t|e cppc.tur|ty ccst c| p.cJuc|rg .r .JJ|t|cr.| bc.||rg b.|| ne.su.eJ |r te.ns c|
|c.gcre b|cyc|es |r .este.r |e|su.e|.rJ`
c. Suppcse t|.t e.ste.r |e|su.e|.rJ c.r, || |t Jevctes .|| |ts .escu.ces tc t|e p.cJuct|cr c| b|cyc|es,
p.cJuce 400. || |t Jevctes .|| |ts .escu.ces tc bc.||rg b.|| p.cJuct|cr, t|cug|, |t c.r p.cJuce cr|y
100. |... t|e p.cJuct|cr pcss|b|||t|es cu.ve |c. e.ste.r |e|su.e|.rJ (.g.|r, .ssune |t |s ||re.. .rJ
put b|cyc|es cr t|e ve.t|c.| .x|s).
CHAP7R 2 CONIRON7ING SCARCI7Y: CHOICS IN PRODUC7ION 53
J. \|.t |s t|e cppc.tur|ty ccst c| p.cJuc|rg .r .JJ|t|cr.| bc.||rg b.|| ne.su.eJ |r te.ns c|
|c.gcre b|cyc|es |r e.ste.r |e|su.e|.rJ`
e. |xp|.|r t|e J||e.erce |r cppc.tur|ty ccst bet.eer .este.r .rJ e.ste.r |e|su.e|.rJ. \||c|
.eg|cr |.s . ccnp...t|ve .Jv.rt.ge |r p.cJuc|rg bc.||rg b.||s` b|cyc|es`
|. |... t|e p.cJuct|cr pcss|b|||t|es cu.ve |c. |e|su.e|.rJ, cre t|.t ccnb|res t|e cu.ves |c. .este.r
.rJ e.ste.r |e|su.e|.rJ.
g. Suppcse |t |s Jete.n|reJ t|.t 400 b|cyc|es nust be p.cJuceJ. |c. n.ry bc.||rg b.||s c.r be
p.cJuceJ`
|. \|e.e .||| t|ese gccJs be p.cJuceJ`
6. |e t.b|e be|c. s|c.s t|e p.cJuct|cr pcss|b|||t|es sc|eJu|e |c. .r eccrcny.
Production AIternatives CapitaI goods per period Consumer goods per period
A 0 40
b 1 36
2 28
| 3 16
| 4 0
.. |utt|rg c.p|t.| gccJs pe. pe.|cJ cr t|e |c.|.crt.| .x|s .rJ ccrsune. gccJs pe. pe.|cJ cr t|e
ve.t|c.| .x|s, g..p| t|e p.cJuct|cr pcss|b|||t|es cu.ve |c. t|e eccrcny.
b. || t|e eccrcny |s p.cJuc|rg .t .|te.r.t|ve b, .|.t |s t|e cppc.tur|ty ccst tc |t c| p.cJuc|rg .t
.|te.r.t|ve |rste.J`
c. || t|e eccrcny |s p.cJuc|rg .t .|te.r.t|ve , .|.t |s t|e cppc.tur|ty ccst tc |t c| p.cJuc|rg .t
.|te.r.t|ve | |rste.J`
J. |s |t pcss|b|e |c. t||s eccrcny tc p.cJuce 30 ur|ts c| ccrsune. gccJs pe. pe.|cJ .|||e
p.cJuc|rg 1 ur|t c| c.p|t.| gccJs` \cu|J t||s ccnb|r.t|cr c| gccJs .ep.esert e|c|ert c.
|re|c|ert p.cJuct|cr` |xp|.|r.
e. \||c| pc|rt, b c. , .cu|J |e.J tc ||g|e. eccrcn|c g.c.t|` |xp|.|r ycu. .rs.e..
. |e ex||b|t be|c. s|c.s t|e scu.ces c| g.c.t| |r t|e |r|teJ St.tes bet.eer 1909 .rJ 1929 .rJ bet.eer
1950 .rJ 199, .ccc.J|rg tc . stuJy by |J...J |er|scr.
1
(|cte. |e scu.ces c| eccrcn|c g.c.t| ..e
cunu|.t|ve .rJ, t.ker cc||ect|ve|y, exp|.|r 100 c| tct.| g.c.t| cve. t|e pe.|cJ.)
.. App.cx|n.te|y .|.t pe.cert.ge c| |.S. g.c.t| bet.eer 1909 .rJ 1929 ..s Jue tc |rc.e.ses |r
,u.rt|t|es c| |.ctc.s c| p.cJuct|cr`
b. App.cx|n.te|y .|.t pe.cert.ge c| |.S. g.c.t| bet.eer 1909 .rJ 1929 ..s Jue tc |rc.e.ses |r
,u.||ty c| |.ctc.s c| p.cJuct|cr .rJ tec|rc|cg|c.| |np.cvenert`
54 PRINCIPLS OI CONOMICS
c. App.cx|n.te|y .|.t pe.cert.ge c| |.S. g.c.t| bet.eer 1950 .rJ 199 ..s Jue tc |rc.e.ses |r
,u.rt|t|es c| |.ctc.s c| p.cJuct|cr`
J. App.cx|n.te|y .|.t pe.cert.ge c| |.S. g.c.t| bet.eer 1950 .rJ 199 ..s Jue tc |rc.e.ses |r
,u.||ty c| |.ctc.s c| p.cJuct|cr .rJ tec|rc|cg|c.| |np.cvenert`
CHAP7R 2 CONIRON7ING SCARCI7Y: CHOICS IN PRODUC7ION 55
1.
ENDNOTES
|J...J |er|scr, |e .ooce o| |coo|c Oo.|| | ||e |||eJ .|o|e (|e. +c.k. cn
n|ttee |c. |ccrcn|c |eve|cpnert, 1962) .rJ |J...J |er|scr, eJ | ^e|co
Oo.|| 9299S2 (\.s||rgtcr, |... b.cck|rgs |rst|tut|crs, 1985).
56 PRINCIPLS OI CONOMICS
Chapter 1
Sampling and Data
1.1 Sampling and Data
1
1.1.1 Student Learning Outcomes
By the end of this chapter, the student should be able to:
Recognize and differentiate between key terms.
Apply various types of sampling methods to data collection.
Create and interpret frequency tables.
1.1.2 Introduction
You are probably asking yourself the question, "When and where will I use statistics?". If you read any
newspaper or watch television, or use the Internet, you will see statistical information. There are statistics
about crime, sports, education, politics, and real estate. Typically, when you read a newspaper article or
watch a news program on television, you are given sample information. With this information, you may
make a decision about the correctness of a statement, claim, or "fact." Statistical methods can help you make
the "best educated guess."
Since you will undoubtedly be given statistical information at some point in your life, you need to know
some techniques to analyze the information thoughtfully. Think about buying a house or managing a
budget. Think about your chosen profession. The elds of economics, business, psychology, education,
biology, law, computer science, police science, and early childhood development require at least one course
in statistics.
Included in this chapter are the basic ideas and words of probability and statistics. You will soon under-
stand that statistics and probability work together. You will also learn how data are gathered and what
"good" data are.
1.2 Statistics
2
The science of statistics deals with the collection, analysis, interpretation, and presentation of data. We see
and use data in our everyday lives.
1
This content is available online at <http://cnx.org/content/m16008/1.9/>.
2
This content is available online at <http://cnx.org/content/m16020/1.14/>.
13
14 CHAPTER 1. SAMPLING AND DATA
1.2.1 Optional Collaborative Classroom Exercise
In your classroom, try this exercise. Have class members write down the average time (in hours, to the
nearest half-hour) they sleep per night. Your instructor will record the data. Then create a simple graph
(called a dot plot) of the data. A dot plot consists of a number line and dots (or points) positioned above
the number line. For example, consider the following data:
5; 5.5; 6; 6; 6; 6.5; 6.5; 6.5; 6.5; 7; 7; 8; 8; 9
The dot plot for this data would be as follows:
Frequency of Average Time (in Hours) Spent Sleeping per Night
Figure 1.1
Does your dot plot look the same as or different from the example? Why? If you did the same example in
an English class with the same number of students, do you think the results would be the same? Why or
why not?
Where do your data appear to cluster? How could you interpret the clustering?
The questions above ask you to analyze and interpret your data. With this example, you have begun your
study of statistics.
In this course, you will learn how to organize and summarize data. Organizing and summarizing data is
called descriptive statistics. Two ways to summarize data are by graphing and by numbers (for example,
nding an average). After you have studied probability and probability distributions, you will use formal
methods for drawing conclusions from "good" data. The formal methods are called inferential statistics.
Statistical inference uses probability to determine how condent we can be that the conclusions are correct.
Effective interpretation of data (inference) is based on good procedures for producing data and thoughtful
examination of the data. You will encounter what will seem to be too many mathematical formulas for
interpreting data. The goal of statistics is not to perform numerous calculations using the formulas, but to
gain an understanding of your data. The calculations can be done using a calculator or a computer. The
understanding must come from you. If you can thoroughly grasp the basics of statistics, you can be more
condent in the decisions you make in life.
15
1.3 Probability
3
Probability is a mathematical tool used to study randomness. It deals with the chance (the likelihood) of
an event occurring. For example, if you toss a fair coin 4 times, the outcomes may not be 2 heads and 2
tails. However, if you toss the same coin 4,000 times, the outcomes will be close to half heads and half tails.
The expected theoretical probability of heads in any one toss is
1
2
or 0.5. Even though the outcomes of a
few repetitions are uncertain, there is a regular pattern of outcomes when there are many repetitions. After
reading about the English statistician Karl Pearson who tossed a coin 24,000 times with a result of 12,012
heads, one of the authors tossed a coin 2,000 times. The results were 996 heads. The fraction
996
2000
is equal
to 0.498 which is very close to 0.5, the expected probability.
The theory of probability began with the study of games of chance such as poker. Predictions take the form
of probabilities. To predict the likelihood of an earthquake, of rain, or whether you will get an A in this
course, we use probabilities. Doctors use probability to determine the chance of a vaccination causing the
disease the vaccination is supposed to prevent. A stockbroker uses probability to determine the rate of
return on a clients investments. You might use probability to decide to buy a lottery ticket or not. In your
study of statistics, you will use the power of mathematics through probability calculations to analyze and
interpret your data.
1.4 Key Terms
4
In statistics, we generally want to study a population. You can think of a population as an entire collection
of persons, things, or objects under study. To study the larger population, we select a sample. The idea of
sampling is to select a portion (or subset) of the larger population and study that portion (the sample) to
gain information about the population. Data are the result of sampling from a population.
Because it takes a lot of time and money to examine an entire population, sampling is a very practical
technique. If you wished to compute the overall grade point average at your school, it would make sense
to select a sample of students who attend the school. The data collected from the sample would be the
students grade point averages. In presidential elections, opinion poll samples of 1,000 to 2,000 people are
taken. The opinion poll is supposed to represent the views of the people in the entire country. Manu-
facturers of canned carbonated drinks take samples to determine if a 16 ounce can contains 16 ounces of
carbonated drink.
From the sample data, we can calculate a statistic. A statistic is a number that is a property of the sample.
For example, if we consider one math class to be a sample of the population of all math classes, then the
average number of points earned by students in that one math class at the end of the term is an example of
a statistic. The statistic is an estimate of a population parameter. A parameter is a number that is a property
of the population. Since we considered all math classes to be the population, then the average number of
points earned per student over all the math classes is an example of a parameter.
One of the main concerns in the eld of statistics is how accurately a statistic estimates a parameter. The
accuracy really depends on how well the sample represents the population. The sample must contain the
characteristics of the population in order to be a representative sample. We are interested in both the
sample statistic and the population parameter in inferential statistics. In a later chapter, we will use the
sample statistic to test the validity of the established population parameter.
A variable, notated by capital letters like X and Y, is a characteristic of interest for each person or thing in
a population. Variables may be numerical or categorical. Numerical variables take on values with equal
3
This content is available online at <http://cnx.org/content/m16015/1.11/>.
4
This content is available online at <http://cnx.org/content/m16007/1.17/>.
16 CHAPTER 1. SAMPLING AND DATA
units such as weight in pounds and time in hours. Categorical variables place the person or thing into a
category. If we let X equal the number of points earned by one math student at the end of a term, then X
is a numerical variable. If we let Y be a persons party afliation, then examples of Y include Republican,
Democrat, and Independent. Y is a categorical variable. We could do some math with values of X (calculate
the average number of points earned, for example), but it makes no sense to do math with values of Y
(calculating an average party afliation makes no sense).
Data are the actual values of the variable. They may be numbers or they may be words. Datum is a single
value.
Two words that come up often in statistics are mean and proportion. If you were to take three exams in
your math classes and obtained scores of 86, 75, and 92, you calculate your mean score by adding the three
exam scores and dividing by three (your mean score would be 84.3 to one decimal place). If, in your math
class, there are 40 students and 22 are men and 18 are women, then the proportion of men students is
22
40
and the proportion of women students is
18
40
. Mean and proportion are discussed in more detail in later
chapters.
NOTE: The words "mean" and "average" are often used interchangeably. The substitution of one
word for the other is common practice. The technical term is "arithmetic mean" and "average" is
technically a center location. However, in practice among non-statisticians, "average" is commonly
accepted for "arithmetic mean."
Example 1.1
Dene the key terms from the following study: We want to know the average (mean) amount
of money rst year college students spend at ABC College on school supplies that do not include
books. We randomly survey 100 rst year students at the college. Three of those students spent
$150, $200, and $225, respectively.
Solution
The population is all rst year students attending ABC College this term.
The sample could be all students enrolled in one section of a beginning statistics course at ABC
College (although this sample may not represent the entire population).
The parameter is the average (mean) amount of money spent (excluding books) by rst year col-
lege students at ABC College this term.
The statistic is the average (mean) amount of money spent (excluding books) by rst year college
students in the sample.
The variable could be the amount of money spent (excluding books) by one rst year student.
Let X = the amount of money spent (excluding books) by one rst year student attending ABC
College.
The data are the dollar amounts spent by the rst year students. Examples of the data are $150,
$200, and $225.
1.4.1 Optional Collaborative Classroom Exercise
Do the following exercise collaboratively with up to four people per group. Find a population, a sample,
the parameter, the statistic, a variable, and data for the following study: You want to determine the average
17
(mean) number of glasses of milk college students drink per day. Suppose yesterday, in your English class,
you asked ve students how many glasses of milk they drank the day before. The answers were 1, 0, 1, 3,
and 4 glasses of milk.
1.5 Data
5
Data may come from a population or from a sample. Small letters like x or y generally are used to represent
data values. Most data can be put into the following categories:
Qualitative
Quantitative
Qualitative data are the result of categorizing or describing attributes of a population. Hair color, blood
type, ethnic group, the car a person drives, and the street a person lives on are examples of qualitative data.
Qualitative data are generally described by words or letters. For instance, hair color might be black, dark
brown, light brown, blonde, gray, or red. Blood type might be AB+, O-, or B+. Researchers often prefer to
use quantitative data over qualitative data because it lends itself more easily to mathematical analysis. For
example, it does not make sense to nd an average hair color or blood type.
Quantitative data are always numbers. Quantitative data are the result of counting or measuring attributes
of a population. Amount of money, pulse rate, weight, number of people living in your town, and the
number of students who take statistics are examples of quantitative data. Quantitative data may be either
discrete or continuous.
All data that are the result of counting are called quantitative discrete data. These data take on only certain
numerical values. If you count the number of phone calls you receive for each day of the week, you might
get 0, 1, 2, 3, etc.
All data that are the result of measuring are quantitative continuous data assuming that we can measure
accurately. Measuring angles in radians might result in the numbers

6
,

3
,

2
, ,
3
4
, etc. If you and your
friends carry backpacks with books in them to school, the numbers of books in the backpacks are discrete
data and the weights of the backpacks are continuous data.
Example 1.2: Data Sample of Quantitative Discrete Data
The data are the number of books students carry in their backpacks. You sample ve students.
Two students carry 3 books, one student carries 4 books, one student carries 2 books, and one
student carries 1 book. The numbers of books (3, 4, 2, and 1) are the quantitative discrete data.
Example 1.3: Data Sample of Quantitative Continuous Data
The data are the weights of the backpacks with the books in it. You sample the same ve students.
The weights (in pounds) of their backpacks are 6.2, 7, 6.8, 9.1, 4.3. Notice that backpacks carrying
three books can have different weights. Weights are quantitative continuous data because weights
are measured.
Example 1.4: Data Sample of Qualitative Data
The data are the colors of backpacks. Again, you sample the same ve students. One student has
a red backpack, two students have black backpacks, one student has a green backpack, and one
student has a gray backpack. The colors red, black, black, green, and gray are qualitative data.
NOTE: You may collect data as numbers and report it categorically. For example, the quiz scores
for each student are recorded throughout the term. At the end of the term, the quiz scores are
reported as A, B, C, D, or F.
5
This content is available online at <http://cnx.org/content/m16005/1.15/>.
18 CHAPTER 1. SAMPLING AND DATA
Example 1.5
Work collaboratively to determine the correct data type (quantitative or qualitative). Indicate
whether quantitative data are continuous or discrete. Hint: Data that are discrete often start with
the words "the number of."
1. The number of pairs of shoes you own.
2. The type of car you drive.
3. Where you go on vacation.
4. The distance it is from your home to the nearest grocery store.
5. The number of classes you take per school year.
6. The tuition for your classes
7. The type of calculator you use.
8. Movie ratings.
9. Political party preferences.
10. Weight of sumo wrestlers.
11. Amount of money (in dollars) won playing poker.
12. Number of correct answers on a quiz.
13. Peoples attitudes toward the government.
14. IQ scores. (This may cause some discussion.)
1.6 Sampling
6
Gathering information about an entire population often costs too much or is virtually impossible. Instead,
we use a sample of the population. A sample should have the same characteristics as the population it
is representing. Most statisticians use various methods of random sampling in an attempt to achieve this
goal. This section will describe a few of the most common methods.
There are several different methods of random sampling. In each form of random sampling, each member
of a population initially has an equal chance of being selected for the sample. Each method has pros and
cons. The easiest method to describe is called a simple random sample. Any group of n individuals is
equally likely to be chosen by any other group of n individuals if the simple random sampling technique is
used. In other words, each sample of the same size has an equal chance of being selected. For example, sup-
pose Lisa wants to form a four-person study group (herself and three other people) from her pre-calculus
class, which has 31 members not including Lisa. To choose a simple random sample of size 3 from the other
members of her class, Lisa could put all 31 names in a hat, shake the hat, close her eyes, and pick out 3
names. A more technological way is for Lisa to rst list the last names of the members of her class together
with a two-digit number as shown below.
6
This content is available online at <http://cnx.org/content/m16014/1.17/>.
19
Class Roster
ID Name
00 Anselmo
01 Bautista
02 Bayani
03 Cheng
04 Cuarismo
05 Cuningham
06 Fontecha
07 Hong
08 Hoobler
09 Jiao
10 Khan
11 King
12 Legeny
13 Lundquist
14 Macierz
15 Motogawa
16 Okimoto
17 Patel
18 Price
19 Quizon
20 Reyes
21 Roquero
22 Roth
23 Rowell
24 Salangsang
25 Slade
26 Stracher
27 Tallai
28 Tran
29 Wai
30 Wood
Table 1.1
Lisa can either use a table of random numbers (found in many statistics books as well as mathematical
handbooks) or a calculator or computer to generate random numbers. For this example, suppose Lisa
chooses to generate random numbers from a calculator. The numbers generated are:
20 CHAPTER 1. SAMPLING AND DATA
.94360; .99832; .14669; .51470; .40581; .73381; .04399
Lisa reads two-digit groups until she has chosen three class members (that is, she reads .94360 as the groups
94, 43, 36, 60). Each random number may only contribute one class member. If she needed to, Lisa could
have generated more random numbers.
The random numbers .94360 and .99832 do not contain appropriate two digit numbers. However the third
randomnumber, .14669, contains 14 (the fourth randomnumber also contains 14), the fth randomnumber
contains 05, and the seventh randomnumber contains 04. The two-digit number 14 corresponds to Macierz,
05 corresponds to Cunningham, and 04 corresponds to Cuarismo. Besides herself, Lisas group will consist
of Marcierz, and Cunningham, and Cuarismo.
Besides simple random sampling, there are other forms of sampling that involve a chance process for get-
ting the sample. Other well-known random sampling methods are the stratied sample, the cluster
sample, and the systematic sample.
To choose a stratied sample, divide the population into groups called strata and then take a proportionate
number fromeach stratum. For example, you could stratify (group) your college population by department
and then choose a proportionate simple randomsample fromeach stratum(each department) to get a strat-
ied random sample. To choose a simple random sample from each department, number each member of
the rst department, number each member of the second department and do the same for the remaining de-
partments. Then use simple random sampling to choose proportionate numbers from the rst department
and do the same for each of the remaining departments. Those numbers picked from the rst department,
picked from the second department and so on represent the members who make up the stratied sample.
To choose a cluster sample, divide the population into clusters (groups) and then randomly select some of
the clusters. All the members from these clusters are in the cluster sample. For example, if you randomly
sample four departments from your college population, the four departments make up the cluster sample.
For example, divide your college faculty by department. The departments are the clusters. Number each
department and then choose four different numbers using simple random sampling. All members of the
four departments with those numbers are the cluster sample.
To choose a systematic sample, randomly select a starting point and take every nth piece of data from a
listing of the population. For example, suppose you have to do a phone survey. Your phone book contains
20,000 residence listings. You must choose 400 names for the sample. Number the population 1 - 20,000
and then use a simple random sample to pick a number that represents the rst name of the sample. Then
choose every 50th name thereafter until you have a total of 400 names (you might have to go back to the of
your phone list). Systematic sampling is frequently chosen because it is a simple method.
A type of sampling that is nonrandom is convenience sampling. Convenience sampling involves using
results that are readily available. For example, a computer software store conducts a marketing study by
interviewing potential customers who happen to be in the store browsing through the available software.
The results of convenience sampling may be very good in some cases and highly biased (favors certain
outcomes) in others.
Sampling data should be done very carefully. Collecting data carelessly can have devastating results. Sur-
veys mailed to households and then returned may be very biased (for example, they may favor a certain
group). It is better for the person conducting the survey to select the sample respondents.
True random sampling is done with replacement. That is, once a member is picked that member goes
back into the population and thus may be chosen more than once. However for practical reasons, in most
populations, simple random sampling is done without replacement. Surveys are typically done without
replacement. That is, a member of the population may be chosen only once. Most samples are taken from
large populations and the sample tends to be small in comparison to the population. Since this is the case,
21
sampling without replacement is approximately the same as sampling with replacement because the chance
of picking the same individual more than once using with replacement is very low.
For example, in a college population of 10,000 people, suppose you want to randomly pick a sample of 1000
for a survey. For any particular sample of 1000, if you are sampling with replacement,
the chance of picking the rst person is 1000 out of 10,000 (0.1000);
the chance of picking a different second person for this sample is 999 out of 10,000 (0.0999);
the chance of picking the same person again is 1 out of 10,000 (very low).
If you are sampling without replacement,
the chance of picking the rst person for any particular sample is 1000 out of 10,000 (0.1000);
the chance of picking a different second person is 999 out of 9,999 (0.0999);
you do not replace the rst person before picking the next person.
Compare the fractions 999/10,000 and 999/9,999. For accuracy, carry the decimal answers to 4 place deci-
mals. To 4 decimal places, these numbers are equivalent (0.0999).
Sampling without replacement instead of sampling with replacement only becomes a mathematics issue
when the population is small which is not that common. For example, if the population is 25 people, the
sample is 10 and you are sampling with replacement for any particular sample,
the chance of picking the rst person is 10 out of 25 and a different second person is 9 out of 25 (you
replace the rst person).
If you sample without replacement,
the chance of picking the rst person is 10 out of 25 and then the second person (which is different) is
9 out of 24 (you do not replace the rst person).
Compare the fractions 9/25 and 9/24. To 4 decimal places, 9/25 = 0.3600 and 9/24 = 0.3750. To 4 decimal
places, these numbers are not equivalent.
When you analyze data, it is important to be aware of sampling errors and nonsampling errors. The actual
process of sampling causes sampling errors. For example, the sample may not be large enough. Factors
not related to the sampling process cause nonsampling errors. A defective counting device can cause a
nonsampling error.
In reality, a sample will never be exactly representative of the population so there will always be
some sampling error. As a rule, the larger the sample, the smaller the sampling error.
In statistics, a sampling bias is created when a sample is collected from a population and some
members of the population are not as likely to be chosen as others (remember, each member of the
population should have an equally likely chance of being chosen). When a sampling bias happens, there
can be incorrect conclusions drawn about the population that is being studied.
Example 1.6
Determine the type of sampling used (simple random, stratied, systematic, cluster, or conve-
nience).
1. A soccer coach selects 6 players from a group of boys aged 8 to 10, 7 players from a group of
boys aged 11 to 12, and 3 players from a group of boys aged 13 to 14 to form a recreational
soccer team.
2. A pollster interviews all human resource personnel in ve different high tech companies.
22 CHAPTER 1. SAMPLING AND DATA
3. A high school educational researcher interviews 50 high school female teachers and 50 high
school male teachers.
4. A medical researcher interviews every third cancer patient from a list of cancer patients at a
local hospital.
5. A high school counselor uses a computer to generate 50 random numbers and then picks
students whose names correspond to the numbers.
6. A student interviews classmates in his algebra class to determine how many pairs of jeans a
student owns, on the average.
Solution
1. stratied
2. cluster
3. stratied
4. systematic
5. simple random
6. convenience
If we were to examine two samples representing the same population, even if we used random sampling
methods for the samples, they would not be exactly the same. Just as there is variation in data, there is
variation in samples. As you become accustomed to sampling, the variability will seem natural.
Example 1.7
Suppose ABC College has 10,000 part-time students (the population). We are interested in the
average amount of money a part-time student spends on books in the fall term. Asking all 10,000
students is an almost impossible task.
Suppose we take two different samples.
First, we use convenience sampling and survey 10 students from a rst term organic chemistry
class. Many of these students are taking rst term calculus in addition to the organic chemistry
class . The amount of money they spend is as follows:
$128; $87; $173; $116; $130; $204; $147; $189; $93; $153
The second sample is taken by using a list from the P.E. department of senior citizens who take
P.E. classes and taking every 5th senior citizen on the list, for a total of 10 senior citizens. They
spend:
$50; $40; $36; $15; $50; $100; $40; $53; $22; $22
Problem 1
Do you think that either of these samples is representative of (or is characteristic of) the entire
10,000 part-time student population?
Solution
No. The rst sample probably consists of science-oriented students. Besides the chemistry course,
some of them are taking rst-term calculus. Books for these classes tend to be expensive. Most
of these students are, more than likely, paying more than the average part-time student for their
books. The second sample is a group of senior citizens who are, more than likely, taking courses
for health and interest. The amount of money they spend on books is probably much less than the
average part-time student. Both samples are biased. Also, in both cases, not all students have a
chance to be in either sample.
23
Problem 2
Since these samples are not representative of the entire population, is it wise to use the results to
describe the entire population?
Solution
No. For these samples, each member of the population did not have an equally likely chance of
being chosen.
Now, suppose we take a third sample. We choose ten different part-time students from the dis-
ciplines of chemistry, math, English, psychology, sociology, history, nursing, physical education,
art, and early childhood development. (We assume that these are the only disciplines in which
part-time students at ABC College are enrolled and that an equal number of part-time students
are enrolled in each of the disciplines.) Each student is chosen using simple random sampling.
Using a calculator, random numbers are generated and a student from a particular discipline is
selected if he/she has a corresponding number. The students spend:
$180; $50; $150; $85; $260; $75; $180; $200; $200; $150
Problem 3
Is the sample biased?
Solution
The sample is unbiased, but a larger sample would be recommended to increase the likelihood
that the sample will be close to representative of the population. However, for a biased sampling
technique, even a large sample runs the risk of not being representative of the population.
Students often ask if it is "good enough" to take a sample, instead of surveying the entire popula-
tion. If the survey is done well, the answer is yes.
1.6.1 Optional Collaborative Classroom Exercise
Exercise 1.6.1
As a class, determine whether or not the following samples are representative. If they are not,
discuss the reasons.
1. To nd the average GPA of all students in a university, use all honor students at the univer-
sity as the sample.
2. To nd out the most popular cereal among young people under the age of 10, stand outside
a large supermarket for three hours and speak to every 20th child under age 10 who enters
the supermarket.
3. To nd the average annual income of all adults in the United States, sample U.S. congress-
men. Create a cluster sample by considering each state as a stratum(group). By using simple
randomsampling, select states to be part of the cluster. Then survey every U.S. congressman
in the cluster.
4. To determine the proportion of people taking public transportation to work, survey 20 peo-
ple in New York City. Conduct the survey by sitting in Central Park on a bench and inter-
viewing every person who sits next to you.
5. To determine the average cost of a two day stay in a hospital in Massachusetts, survey 100
hospitals across the state using simple random sampling.
24 CHAPTER 1. SAMPLING AND DATA
1.7 Variation
7
1.7.1 Variation in Data
Variation is present in any set of data. For example, 16-ounce cans of beverage may contain more or less
than 16 ounces of liquid. In one study, eight 16 ounce cans were measured and produced the following
amount (in ounces) of beverage:
15.8; 16.1; 15.2; 14.8; 15.8; 15.9; 16.0; 15.5
Measurements of the amount of beverage in a 16-ounce can may vary because different people make the
measurements or because the exact amount, 16 ounces of liquid, was not put into the cans. Manufacturers
regularly run tests to determine if the amount of beverage in a 16-ounce can falls within the desired range.
Be aware that as you take data, your data may vary somewhat from the data someone else is taking for the
same purpose. This is completely natural. However, if two or more of you are taking the same data and
get very different results, it is time for you and the others to reevaluate your data-taking methods and your
accuracy.
1.7.2 Variation in Samples
It was mentioned previously that two or more samples from the same population, taken randomly, and
having close to the same characteristics of the population are different fromeach other. Suppose Doreen and
Jung both decide to study the average amount of time students at their college sleep each night. Doreen and
Jung each take samples of 500 students. Doreen uses systematic sampling and Jung uses cluster sampling.
Doreens sample will be different from Jungs sample. Even if Doreen and Jung used the same sampling
method, in all likelihood their samples would be different. Neither would be wrong, however.
Think about what contributes to making Doreens and Jungs samples different.
If Doreen and Jung took larger samples (i.e. the number of data values is increased), their sample results
(the average amount of time a student sleeps) might be closer to the actual population average. But still,
their samples would be, in all likelihood, different from each other. This variability in samples cannot be
stressed enough.
1.7.2.1 Size of a Sample
The size of a sample (often called the number of observations) is important. The examples you have seen
in this book so far have been small. Samples of only a few hundred observations, or even smaller, are
sufcient for many purposes. In polling, samples that are from 1200 to 1500 observations are considered
large enough and good enough if the survey is random and is well done. You will learn why when you
study condence intervals.
Be aware that many large samples are biased. For example, call-in surveys are invariable biased
because people choose to respond or not.
7
This content is available online at <http://cnx.org/content/m16021/1.15/>.
25
1.7.2.2 Optional Collaborative Classroom Exercise
Exercise 1.7.1
Divide into groups of two, three, or four. Your instructor will give each group one 6-sided die.
Try this experiment twice. Roll one fair die (6-sided) 20 times. Record the number of ones, twos,
threes, fours, ves, and sixes you get below ("frequency" is the number of times a particular face
of the die occurs):
First Experiment (20 rolls)
Face on Die Frequency
1
2
3
4
5
6
Table 1.2
Second Experiment (20 rolls)
Face on Die Frequency
1
2
3
4
5
6
Table 1.3
Did the two experiments have the same results? Probably not. If you did the experiment a third
time, do you expect the results to be identical to the rst or second experiment? (Answer yes or
no.) Why or why not?
Which experiment had the correct results? They both did. The job of the statistician is to see
through the variability and draw appropriate conclusions.
1.7.3 Critical Evaluation
We need to critically evaluate the statistical studies we read about and analyze before accepting the results
of the study. Common problems to be aware of include
Problems with Samples: A sample should be representative of the population. A sample that is not
representative of the population is biased. Biased samples that are not representative of the popula-
tion give results that are inaccurate and not valid.
26 CHAPTER 1. SAMPLING AND DATA
Self-Selected Samples: Responses only by people who choose to respond, such as call-in surveys are
often unreliable.
Sample Size Issues: Samples that are too small may be unreliable. Larger samples are better if possible.
In some situations, small samples are unavoidable and can still be used to draw conclusions, even
though larger samples are better. Examples: Crash testing cars, medical testing for rare conditions.
Undue inuence: Collecting data or asking questions in a way that inuences the response.
Non-response or refusal of subject to participate: The collected responses may no longer be represen-
tative of the population. Often, people with strong positive or negative opinions may answer surveys,
which can affect the results.
Causality: A relationship between two variables does not mean that one causes the other to occur.
They may both be related (correlated) because of their relationship through a different variable.
Self-Funded or Self-Interest Studies: A study performed by a person or organization in order to sup-
port their claim. Is the study impartial? Read the study carefully to evaluate the work. Do not
automatically assume that the study is good but do not automatically assume the study is bad either.
Evaluate it on its merits and the work done.
Misleading Use of Data: Improperly displayed graphs, incomplete data, lack of context.
Confounding: When the effects of multiple factors on a response cannot be separated. Confounding
makes it difcult or impossible to draw valid conclusions about the effect of each factor.
1.8 Answers and Rounding Off
8
A simple way to round off answers is to carry your nal answer one more decimal place than was present
in the original data. Round only the nal answer. Do not round any intermediate results, if possible. If it
becomes necessary to round intermediate results, carry them to at least twice as many decimal places as the
nal answer. For example, the average of the three quiz scores 4, 6, 9 is 6.3, rounded to the nearest tenth,
because the data are whole numbers. Most answers will be rounded in this manner.
It is not necessary to reduce most fractions in this course. Especially in Probability Topics (Section 3.1), the
chapter on probability, it is more helpful to leave an answer as an unreduced fraction.
1.9 Frequency
9
Twenty students were asked how many hours they worked per day. Their responses, in hours, are listed
below:
5; 6; 3; 3; 2; 4; 7; 5; 2; 3; 5; 6; 5; 4; 4; 3; 5; 2; 5; 3
Below is a frequency table listing the different data values in ascending order and their frequencies.
8
This content is available online at <http://cnx.org/content/m16006/1.7/>.
9
This content is available online at <http://cnx.org/content/m16012/1.20/>.
27
Frequency Table of Student Work Hours
DATA VALUE FREQUENCY
2 3
3 5
4 3
5 6
6 2
7 1
Table 1.4
A frequency is the number of times a given datum occurs in a data set. According to the table above,
there are three students who work 2 hours, ve students who work 3 hours, etc. The total of the frequency
column, 20, represents the total number of students included in the sample.
A relative frequency is the fraction or proportion of times an answer occurs. To nd the relative fre-
quencies, divide each frequency by the total number of students in the sample - in this case, 20. Relative
frequencies can be written as fractions, percents, or decimals.
Frequency Table of Student Work Hours w/ Relative Frequency
DATA VALUE FREQUENCY RELATIVE FREQUENCY
2 3
3
20
or 0.15
3 5
5
20
or 0.25
4 3
3
20
or 0.15
5 6
6
20
or 0.30
6 2
2
20
or 0.10
7 1
1
20
or 0.05
Table 1.5
The sum of the relative frequency column is
20
20
, or 1.
Cumulative relative frequency is the accumulation of the previous relative frequencies. To nd the cumu-
lative relative frequencies, add all the previous relative frequencies to the relative frequency for the current
row.
Frequency Table of Student Work Hours w/ Relative and Cumulative Relative Frequency
DATA VALUE FREQUENCY RELATIVE
FREQUENCY
CUMULATIVE RELA-
TIVE
FREQUENCY
continued on next page
28 CHAPTER 1. SAMPLING AND DATA
2 3
3
20
or 0.15 0.15
3 5
5
20
or 0.25 0.15 + 0.25 = 0.40
4 3
3
20
or 0.15 0.40 + 0.15 = 0.55
5 6
6
20
or 0.30 0.55 + 0.30 = 0.85
6 2
2
20
or 0.10 0.85 + 0.10 = 0.95
7 1
1
20
or 0.05 0.95 + 0.05 = 1.00
Table 1.6
The last entry of the cumulative relative frequency column is one, indicating that one hundred percent of
the data has been accumulated.
NOTE: Because of rounding, the relative frequency column may not always sumto one and the last
entry in the cumulative relative frequency column may not be one. However, they each should be
close to one.
The following table represents the heights, in inches, of a sample of 100 male semiprofessional soccer play-
ers.
Frequency Table of Soccer Player Height
HEIGHTS
(INCHES)
FREQUENCY RELATIVE
FREQUENCY
CUMULATIVE
RELATIVE
FREQUENCY
59.95 - 61.95 5
5
100
= 0.05 0.05
61.95 - 63.95 3
3
100
= 0.03 0.05 + 0.03 = 0.08
63.95 - 65.95 15
15
100
= 0.15 0.08 + 0.15 = 0.23
65.95 - 67.95 40
40
100
= 0.40 0.23 + 0.40 = 0.63
67.95 - 69.95 17
17
100
= 0.17 0.63 + 0.17 = 0.80
69.95 - 71.95 12
12
100
= 0.12 0.80 + 0.12 = 0.92
71.95 - 73.95 7
7
100
= 0.07 0.92 + 0.07 = 0.99
73.95 - 75.95 1
1
100
= 0.01 0.99 + 0.01 = 1.00
Total = 100 Total = 1.00
Table 1.7
The data in this table has been grouped into the following intervals:
59.95 - 61.95 inches
61.95 - 63.95 inches
63.95 - 65.95 inches
65.95 - 67.95 inches
67.95 - 69.95 inches
69.95 - 71.95 inches
71.95 - 73.95 inches
73.95 - 75.95 inches
29
NOTE: This example is used again in the Descriptive Statistics (Section 2.1) chapter, where the
method used to compute the intervals will be explained.
In this sample, there are 5 players whose heights are between 59.95 - 61.95 inches, 3 players whose heights
fall within the interval 61.95 - 63.95 inches, 15 players whose heights fall within the interval 63.95 - 65.95
inches, 40 players whose heights fall within the interval 65.95 - 67.95 inches, 17 players whose heights
fall within the interval 67.95 - 69.95 inches, 12 players whose heights fall within the interval 69.95 - 71.95,
7 players whose height falls within the interval 71.95 - 73.95, and 1 player whose height falls within the
interval 73.95 - 75.95. All heights fall between the endpoints of an interval and not at the endpoints.
Example 1.8
From the table, nd the percentage of heights that are less than 65.95 inches.
Solution
If you look at the rst, second, and third rows, the heights are all less than 65.95 inches. There are
5 + 3 + 15 = 23 males whose heights are less than 65.95 inches. The percentage of heights less than
65.95 inches is then
23
100
or 23%. This percentage is the cumulative relative frequency entry in the
third row.
Example 1.9
From the table, nd the percentage of heights that fall between 61.95 and 65.95 inches.
Solution
Add the relative frequencies in the second and third rows: 0.03 + 0.15 = 0.18 or 18%.
Example 1.10
Use the table of heights of the 100 male semiprofessional soccer players. Fill in the blanks and
check your answers.
1. The percentage of heights that are from 67.95 to 71.95 inches is:
2. The percentage of heights that are from 67.95 to 73.95 inches is:
3. The percentage of heights that are more than 65.95 inches is:
4. The number of players in the sample who are between 61.95 and 71.95 inches tall is:
5. What kind of data are the heights?
6. Describe how you could gather this data (the heights) so that the data are characteristic of all
male semiprofessional soccer players.
Remember, you count frequencies. To nd the relative frequency, divide the frequency by the
total number of data values. To nd the cumulative relative frequency, add all of the previous
relative frequencies to the relative frequency for the current row.
1.9.1 Optional Collaborative Classroom Exercise
Exercise 1.9.1
In your class, have someone conduct a survey of the number of siblings (brothers and sisters) each
student has. Create a frequency table. Add to it a relative frequency column and a cumulative
relative frequency column. Answer the following questions:
30 CHAPTER 1. SAMPLING AND DATA
1. What percentage of the students in your class has 0 siblings?
2. What percentage of the students has from 1 to 3 siblings?
3. What percentage of the students has fewer than 3 siblings?
Example 1.11
Nineteen people were asked how many miles, to the nearest mile they commute to work each
day. The data are as follows:
2; 5; 7; 3; 2; 10; 18; 15; 20; 7; 10; 18; 5; 12; 13; 12; 4; 5; 10
The following table was produced:
Frequency of Commuting Distances
DATA FREQUENCY RELATIVEFREQUENCY CUMULATIVERELATIVEFREQUENCY
3 3
3
19
0.1579
4 1
1
19
0.2105
5 3
3
19
0.1579
7 2
2
19
0.2632
10 3
4
19
0.4737
12 2
2
19
0.7895
13 1
1
19
0.8421
15 1
1
19
0.8948
18 1
1
19
0.9474
20 1
1
19
1.0000
Table 1.8
Problem (Solution on p. 48.)
1. Is the table correct? If it is not correct, what is wrong?
2. True or False: Three percent of the people surveyed commute 3 miles. If the statement is not
correct, what should it be? If the table is incorrect, make the corrections.
3. What fraction of the people surveyed commute 5 or 7 miles?
4. What fraction of the people surveyed commute 12 miles or more? Less than 12 miles? Be-
tween 5 and 13 miles (does not include 5 and 13 miles)?
31
1.10 Summary
10
Statistics
Deals with the collection, analysis, interpretation, and presentation of data
Probability
Mathematical tool used to study randomness
Key Terms
Population
Parameter
Sample
Statistic
Variable
Data
Types of Data
Quantitative Data (a number)
Discrete (You count it.)
Continuous (You measure it.)
Qualitative Data (a category, words)
Sampling
With Replacement: A member of the population may be chosen more than once
Without Replacement: A member of the population may be chosen only once
Random Sampling
Each member of the population has an equal chance of being selected
Sampling Methods
Random
Simple random sample
Stratied sample
Cluster sample
Systematic sample
Not Random
Convenience sample
Frequency (freq. or f)
The number of times an answer occurs
Relative Frequency (rel. freq. or RF)
The proportion of times an answer occurs
Can be interpreted as a fraction, decimal, or percent
Cumulative Relative Frequencies (cum. rel. freq. or cum RF)
An accumulation of the previous relative frequencies
10
This content is available online at <http://cnx.org/content/m16023/1.10/>.
32 CHAPTER 1. SAMPLING AND DATA
1.11 Practice: Sampling and Data
11
1.11.1 Student Learning Outcomes
The student will construct frequency tables.
The student will differentiate between key terms.
The student will compare sampling techniques.
1.11.2 Given
Studies are often done by pharmaceutical companies to determine the effectiveness of a treatment program.
Suppose that a new AIDS antibody drug is currently under study. It is given to patients once the AIDS
symptoms have revealed themselves. Of interest is the average(mean) length of time in months patients
live once starting the treatment. Two researchers each follow a different set of 40 AIDS patients from the
start of treatment until their deaths. The following data (in months) are collected.
Researcher A 3; 4; 11; 15; 16; 17; 22; 44; 37; 16; 14; 24; 25; 15; 26; 27; 33; 29; 35; 44; 13; 21; 22; 10; 12; 8; 40; 32;
26; 27; 31; 34; 29; 17; 8; 24; 18; 47; 33; 34
Researcher B 3; 14; 11; 5; 16; 17; 28; 41; 31; 18; 14; 14; 26; 25; 21; 22; 31; 2; 35; 44; 23; 21; 21; 16; 12; 18; 41; 22;
16; 25; 33; 34; 29; 13; 18; 24; 23; 42; 33; 29
1.11.3 Organize the Data
Complete the tables below using the data provided.
Researcher A
Survival
Length
(in
months)
Frequency Relative Frequency Cumulative Relative Fre-
quency
0.5 - 6.5
6.5 -
12.5
12.5 -
18.5
18.5 -
24.5
24.5 -
30.5
continued on next page
11
This content is available online at <http://cnx.org/content/m16016/1.16/>.
33
30.5 -
36.5
36.5 -
42.5
42.5 -
48.5
Table 1.9
Researcher B
Survival Length (in
months)
Frequency Relative Frequency Cumulative Relative Fre-
quency
0.5 - 6.5
6.5 - 12.5
12.5 - 18.5
18.5 - 24.5
24.5 - 30.5
30.5 - 36.5
36.5 - 42.5
42.5 - 48.5
Table 1.10
1.11.4 Key Terms
Dene the key terms based upon the above example for Researcher A.
Exercise 1.11.1
Population
Exercise 1.11.2
Sample
Exercise 1.11.3
Parameter
Exercise 1.11.4
Statistic
Exercise 1.11.5
Variable
Exercise 1.11.6
Data
34 CHAPTER 1. SAMPLING AND DATA
1.11.5 Discussion Questions
Discuss the following questions and then answer in complete sentences.
Exercise 1.11.7
List two reasons why the data may differ.
Exercise 1.11.8
Can you tell if one researcher is correct and the other one is incorrect? Why?
Exercise 1.11.9
Would you expect the data to be identical? Why or why not?
Exercise 1.11.10
How could the researchers gather random data?
Exercise 1.11.11
Suppose that the rst researcher conducted his survey by randomly choosing one state in the
nation and then randomly picking 40 patients from that state. What sampling method would that
researcher have used?
Exercise 1.11.12
Suppose that the second researcher conducted his survey by choosing 40 patients he knew. What
sampling method would that researcher have used? What concerns would you have about this
data set, based upon the data collection method?
35
1.12 Homework
12
Exercise 1.12.1 (Solution on p. 48.)
For each item below:
i. Identify the type of data (quantitative - discrete, quantitative - continuous, or qualitative) that
would be used to describe a response.
ii. Give an example of the data.
a. Number of tickets sold to a concert
b. Amount of body fat
c. Favorite baseball team
d. Time in line to buy groceries
e. Number of students enrolled at Evergreen Valley College
f. Mostwatched television show
g. Brand of toothpaste
h. Distance to the closest movie theatre
i. Age of executives in Fortune 500 companies
j. Number of competing computer spreadsheet software packages
Exercise 1.12.2
Fifty part-time students were asked how many courses they were taking this term. The (incom-
plete) results are shown below:
Part-time Student Course Loads
# of Courses Frequency Relative Frequency Cumulative Relative
Frequency
1 30 0.6
2 15
3
Table 1.11
a. Fill in the blanks in the table above.
b. What percent of students take exactly two courses?
c. What percent of students take one or two courses?
Exercise 1.12.3 (Solution on p. 48.)
Sixty adults with gum disease were asked the number of times per week they used to oss before
their diagnoses. The (incomplete) results are shown below:
Flossing Frequency for Adults with Gum Disease
# Flossing per Week Frequency Relative Frequency Cumulative Relative Freq.
0 27 0.4500
1 18
3 0.9333
6 3 0.0500
7 1 0.0167
12
This content is available online at <http://cnx.org/content/m16010/1.19/>.
36 CHAPTER 1. SAMPLING AND DATA
Table 1.12
a. Fill in the blanks in the table above.
b. What percent of adults ossed six times per week?
c. What percent ossed at most three times per week?
Exercise 1.12.4
Atness center is interested in the mean amount of time a client exercises in the center each week.
Dene the following in terms of the study. Give examples where appropriate.
a. Population
b. Sample
c. Parameter
d. Statistic
e. Variable
f. Data
Exercise 1.12.5 (Solution on p. 48.)
Ski resorts are interested in the mean age that children take their rst ski and snowboard lessons.
They need this information to optimally plan their ski classes. Dene the following in terms of the
study. Give examples where appropriate.
a. Population
b. Sample
c. Parameter
d. Statistic
e. Variable
f. Data
Exercise 1.12.6
A cardiologist is interested in the mean recovery period for her patients who have had heart
attacks. Dene the following in terms of the study. Give examples where appropriate.
a. Population
b. Sample
c. Parameter
d. Statistic
e. Variable
f. Data
Exercise 1.12.7 (Solution on p. 49.)
Insurance companies are interested in the mean health costs each year for their clients, so that
they can determine the costs of health insurance. Dene the following in terms of the study. Give
examples where appropriate.
a. Population
b. Sample
c. Parameter
d. Statistic
e. Variable
f. Data
37
Exercise 1.12.8
A politician is interested in the proportion of voters in his district that think he is doing a good
job. Dene the following in terms of the study. Give examples where appropriate.
a. Population
b. Sample
c. Parameter
d. Statistic
e. Variable
f. Data
Exercise 1.12.9 (Solution on p. 49.)
A marriage counselor is interested in the proportion the clients she counsels that stay married.
Dene the following in terms of the study. Give examples where appropriate.
a. Population
b. Sample
c. Parameter
d. Statistic
e. Variable
f. Data
Exercise 1.12.10
Political pollsters may be interested in the proportion of people that will vote for a particular
cause. Dene the following in terms of the study. Give examples where appropriate.
a. Population
b. Sample
c. Parameter
d. Statistic
e. Variable
f. Data
Exercise 1.12.11 (Solution on p. 49.)
A marketing company is interested in the proportion of people that will buy a particular product.
Dene the following in terms of the study. Give examples where appropriate.
a. Population
b. Sample
c. Parameter
d. Statistic
e. Variable
f. Data
Exercise 1.12.12
Airline companies are interested in the consistency of the number of babies on each ight, so that
they have adequate safety equipment. Suppose an airline conducts a survey. Over Thanksgiving
weekend, it surveys 6 ights from Boston to Salt Lake City to determine the number of babies on
the ights. It determines the amount of safety equipment needed by the result of that study.
a. Using complete sentences, list three things wrong with the way the survey was conducted.
b. Using complete sentences, list three ways that you would improve the survey if it were to be
repeated.
38 CHAPTER 1. SAMPLING AND DATA
Exercise 1.12.13
Suppose you want to determine the mean number of students per statistics class in your state.
Describe a possible sampling method in 3 5 complete sentences. Make the description detailed.
Exercise 1.12.14
Suppose you want to determine the mean number of cans of soda drunk each month by persons
in their twenties. Describe a possible sampling method in 3 - 5 complete sentences. Make the
description detailed.
Exercise 1.12.15 (Solution on p. 49.)
771 distance learning students at Long Beach City College responded to surveys in the 2010-
11 academic year. Highlights of the summary report are listed in the table below. (Source:
http://de.lbcc.edu/reports/2010-11/future/highlights.html#focus).
LBCC Distance Learning Survey Results
Have computer at home 96%
Unable to come to campus for classes 65%
Age 41 or over 24%
Would like LBCC to offer more DL courses 95%
Took DL classes due to a disability 17%
Live at least 16 miles from campus 13%
Took DL courses to fulll transfer requirements 71%
Table 1.13
a. What percent of the students surveyed do not have a computer at home?
b. About how many students in the survey live at least 16 miles from campus?
c. If the same survey was done at Great Basin College in Elko, Nevada, do you think the percent-
ages would be the same? Why?
Exercise 1.12.16
Nineteen immigrants to the U.S were asked how many years, to the nearest year, they have lived
in the U.S. The data are as follows:
2; 5; 7; 2; 2; 10; 20; 15; 0; 7; 0; 20; 5; 12; 15; 12; 4; 5; 10
The following table was produced:
39
Frequency of Immigrant Survey Responses
Data Frequency Relative Frequency Cumulative Relative Frequency
0 2
2
19
0.1053
2 3
3
19
0.2632
4 1
1
19
0.3158
5 3
3
19
0.1579
7 2
2
19
0.5789
10 2
2
19
0.6842
12 2
2
19
0.7895
15 1
1
19
0.8421
20 1
1
19
1.0000
Table 1.14
a. Fix the errors on the table. Also, explain how someone might have arrived at the incorrect
number(s).
b. Explain what is wrong with this statement: 47 percent of the people surveyed have lived in
the U.S. for 5 years.
c. Fix the statement above to make it correct.
d. What fraction of the people surveyed have lived in the U.S. 5 or 7 years?
e. What fraction of the people surveyed have lived in the U.S. at most 12 years?
f. What fraction of the people surveyed have lived in the U.S. fewer than 12 years?
g. What fraction of the people surveyed have lived in the U.S. from 5 to 20 years, inclusive?
Exercise 1.12.17
A random survey was conducted of 3274 people of the microprocessor generation (people
born since 1971, the year the microprocessor was invented). It was reported that 48% of those
individuals surveyed stated that if they had $2000 to spend, they would use it for computer
equipment. Also, 66% of those surveyed considered themselves relatively savvy computer users.
(Source: San Jose Mercury News)
a. Do you consider the sample size large enough for a study of this type? Why or why not?
b. Based on your gut feeling, do you believe the percents accurately reect the U.S. population
for those individuals born since 1971? If not, do you think the percents of the population are
actually higher or lower than the sample statistics? Why?
Additional information: The survey was reported by Intel Corporation of individuals who visited
the Los Angeles Convention Center to see the Smithsonian Institures road showcalled Americas
Smithsonian.
c. With this additional information, do you feel that all demographic and ethnic groups were
equally represented at the event? Why or why not?
d. With the additional information, comment on how accurately you think the sample statistics
reect the population parameters.
Exercise 1.12.18
40 CHAPTER 1. SAMPLING AND DATA
a. List some practical difculties involved in getting accurate results from a telephone survey.
b. List some practical difculties involved in getting accurate results from a mailed survey.
c. With your classmates, brainstorm some ways to overcome these problems if you needed to
conduct a phone or mail survey.
1.12.1 Try these multiple choice questions
The next four questions refer to the following: A Lake Tahoe Community College instructor is interested
in the mean number of days Lake Tahoe Community College math students are absent from class during a
quarter.
Exercise 1.12.19 (Solution on p. 49.)
What is the population she is interested in?
A. All Lake Tahoe Community College students
B. All Lake Tahoe Community College English students
C. All Lake Tahoe Community College students in her classes
D. All Lake Tahoe Community College math students
Exercise 1.12.20 (Solution on p. 49.)
Consider the following:
X = number of days a Lake Tahoe Community College math student is absent
In this case, X is an example of a:
A. Variable
B. Population
C. Statistic
D. Data
Exercise 1.12.21 (Solution on p. 49.)
The instructor takes her sample by gathering data on 5 randomly selected students from each
Lake Tahoe Community College math class. The type of sampling she used is
A. Cluster sampling
B. Stratied sampling
C. Simple random sampling
D. Convenience sampling
Exercise 1.12.22 (Solution on p. 49.)
The instructors sample produces an mean number of days absent of 3.5 days. This value is an
example of a
A. Parameter
B. Data
C. Statistic
D. Variable
The next two questions refer to the following relative frequency table on hurricanes that have made direct
hits on the U.S between 1851 and 2004. Hurricanes are given a strength category rating based on the
minimum wind speed generated by the storm. (http://www.nhc.noaa.gov/gifs/table5.gif
13
)
13
http://www.nhc.noaa.gov/gifs/table5.gif
41
Frequency of Hurricane Direct Hits
Category Number of Direct Hits Relative Frequency Cumulative Frequency
1 109 0.3993 0.3993
2 72 0.2637 0.6630
3 71 0.2601
4 18 0.9890
5 3 0.0110 1.0000
Total = 273
Table 1.15
Exercise 1.12.23 (Solution on p. 49.)
What is the relative frequency of direct hits that were category 4 hurricanes?
A. 0.0768
B. 0.0659
C. 0.2601
D. Not enough information to calculate
Exercise 1.12.24 (Solution on p. 49.)
What is the relative frequency of direct hits that were AT MOST a category 3 storm?
A. 0.3480
B. 0.9231
C. 0.2601
D. 0.3370
The next three questions refer to the following: A study was done to determine the age, number of times
per week and the duration (amount of time) of resident use of a local park in San Jose. The rst house in
the neighborhood around the park was selected randomly and then every 8th house in the neighborhood
around the park was interviewed.
Exercise 1.12.25 (Solution on p. 49.)
Number of times per week is what type of data?
A. qualitative
B. quantitative - discrete
C. quantitative - continuous
Exercise 1.12.26 (Solution on p. 49.)
The sampling method was:
A. simple random
B. systematic
C. stratied
D. cluster
Exercise 1.12.27 (Solution on p. 49.)
Duration (amount of time) is what type of data?
42 CHAPTER 1. SAMPLING AND DATA
A. qualitative
B. quantitative - discrete
C. quantitative - continuous
Exercises 28 and 29 are not multiple choice exercises.
Exercise 1.12.28 (Solution on p. 49.)
Name the sampling method used in each of the following situations:
A. A woman in the airport is handing out questionnaires to travelers asking them to evaluate the
airports service. She does not ask travelers who are hurrying through the airport with their
hands full of luggage, but instead asks all travelers sitting near gates and who are not taking
naps while they wait.
B. A teacher wants to know if her students are doing homework so she randomly selects rows 2
and 5, and then calls on all students in row 2 and all students in row 5 to present the solution
to homework problems to the class.
C. The marketing manager for an electronics chain store wants information about the ages of its
customers. Over the next two weeks, at each store location, 100 randomly selected customers
are given questionnaires to ll out which asks for information about age, as well as about
other variables of interest.
D. The librarian at a public library wants to determine what proportion of the library users are
children. The librarian has a tally sheet on which she marks whether the books are checked
out by an adult or a child. She records this data for every 4th patron who checks out books.
E. A political party wants to know the reaction of voters to a debate between the candidates. The
day after the debate, the partys polling staff calls 1200 randomly selected phone numbers.
If a registered voter answers the phone or is available to come to the phone, that registered
voter is asked who he/she intends to vote for and whether the debate changed his/her
opinion of the candidates.
** Contributed by Roberta Bloom
Exercise 1.12.29 (Solution on p. 50.)
Several online textbook retailers advertise that they have lower prices than on-campus book-
stores. However, an important factor is whether the internet retailers actually have the textbooks
that students need in stock. Students need to be able to get textbooks promptly at the beginning of
the college term. If the book is not available, then a student would not be able to get the textbook
at all, or might get a delayed delivery if the book is back ordered.
A college newspaper reporter is investigating textbook availability at online retailers. He
decides to investigate one textbook for each of the following 7 subjects: calculus, biology,
chemistry, physics, statistics, geology, and general engineering. He consults textbook industry
sales data and selects the most popular nationally used textbook in each of these subjects. He
visits websites for a random sample of major online textbook sellers and looks up each of these 7
textbooks to see if they are available in stock for quick delivery through these retailers. Based on
his investigation, he writes an article in which he draws conclusions about the overall availability
of all college textbooks through online textbook retailers.
Write an analysis of his study that addresses the following issues: Is his sample representa-
tive of the population of all college textbooks? Explain why or why not. Describe some possible
sources of bias in this study, and how it might affect the results of the study. Give some sugges-
tions about what could be done to improve the study.
** Contributed by Roberta Bloom
43
1.13 Lab 1: Data Collection
14
Class Time:
Names:
1.13.1 Student Learning Outcomes
The student will demonstrate the systematic sampling technique.
The student will construct Relative Frequency Tables.
The student will interpret results and their differences from different data groupings.
1.13.2 Movie Survey
Ask ve classmates from a different class how many movies they saw last month at the theater. Do not
include rented movies.
1. Record the data
2. In class, randomly pick one person. On the class list, mark that persons name. Move down four
peoples names on the class list. Mark that persons name. Continue doing this until you have marked
12 peoples names. You may need to go back to the start of the list. For each marked name record
below the ve data values. You now have a total of 60 data values.
3. For each name marked, record the data:
______ ______ ______ ______ ______ ______ ______ ______ ______ ______
______ ______ ______ ______ ______ ______ ______ ______ ______ ______
______ ______ ______ ______ ______ ______ ______ ______ ______ ______
______ ______ ______ ______ ______ ______ ______ ______ ______ ______
______ ______ ______ ______ ______ ______ ______ ______ ______ ______
______ ______ ______ ______ ______ ______ ______ ______ ______ ______
Table 1.16
1.13.3 Order the Data
Complete the two relative frequency tables below using your class data.
14
This content is available online at <http://cnx.org/content/m16004/1.11/>.
44 CHAPTER 1. SAMPLING AND DATA
Frequency of Number of Movies Viewed
Number of Movies Frequency Relative Frequency Cumulative Relative Frequency
0
1
2
3
4
5
6
7+
Table 1.17
Frequency of Number of Movies Viewed
Number of Movies Frequency Relative Frequency Cumulative Relative Frequency
0-1
2-3
4-5
6-7+
Table 1.18
1. Using the tables, nd the percent of data that is at most 2. Which table did you use and why?
2. Using the tables, nd the percent of data that is at most 3. Which table did you use and why?
3. Using the tables, nd the percent of data that is more than 2. Which table did you use and why?
4. Using the tables, nd the percent of data that is more than 3. Which table did you use and why?
1.13.4 Discussion Questions
1. Is one of the tables above "more correct" than the other? Why or why not?
2. In general, why would someone group the data in different ways? Are there any advantages to either
way of grouping the data?
3. Why did you switch between tables, if you did, when answering the question above?
45
1.14 Lab 2: Sampling Experiment
15
Class Time:
Names:
1.14.1 Student Learning Outcomes
The student will demonstrate the simple random, systematic, stratied, and cluster sampling tech-
niques.
The student will explain each of the details of each procedure used.
In this lab, you will be asked to pick several random samples. In each case, describe your procedure briey,
including how you might have used the random number generator, and then list the restaurants in the
sample you obtained
NOTE: The following section contains restaurants stratied by city into columns and grouped
horizontally by entree cost (clusters).
1.14.2 A Simple Random Sample
Pick a simple random sample of 15 restaurants.
1. Describe the procedure:
2.
1. __________ 6. __________ 11. __________
2. __________ 7. __________ 12. __________
3. __________ 8. __________ 13. __________
4. __________ 9. __________ 14. __________
5. __________ 10. __________ 15. __________
Table 1.19
1.14.3 A Systematic Sample
Pick a systematic sample of 15 restaurants.
1. Describe the procedure:
2.
1. __________ 6. __________ 11. __________
2. __________ 7. __________ 12. __________
3. __________ 8. __________ 13. __________
4. __________ 9. __________ 14. __________
5. __________ 10. __________ 15. __________
Table 1.20
15
This content is available online at <http://cnx.org/content/m16013/1.15/>.
46 CHAPTER 1. SAMPLING AND DATA
1.14.4 A Stratied Sample
Pick a stratied sample, by city, of 20 restaurants. Use 25% of the restaurants from each stratum. Round to
the nearest whole number.
1. Describe the procedure:
2.
1. __________ 6. __________ 11. __________ 16. __________
2. __________ 7. __________ 12. __________ 17. __________
3. __________ 8. __________ 13. __________ 18. __________
4. __________ 9. __________ 14. __________ 19. __________
5. __________ 10. __________ 15. __________ 20. __________
Table 1.21
1.14.5 A Stratied Sample
Pick a stratied sample, by entree cost, of 21 restaurants. Use 25% of the restaurants from each stratum.
Round to the nearest whole number.
1. Describe the procedure:
2.
1. __________ 6. __________ 11. __________ 16. __________
2. __________ 7. __________ 12. __________ 17. __________
3. __________ 8. __________ 13. __________ 18. __________
4. __________ 9. __________ 14. __________ 19. __________
5. __________ 10. __________ 15. __________ 20. __________
21. __________
Table 1.22
1.14.6 A Cluster Sample
Pick a cluster sample of restaurants from two cities. The number of restaurants will vary.
1. Describe the procedure:
2.
1. __________ 6. __________ 11. __________ 16. __________ 21. __________
2. __________ 7. __________ 12. __________ 17. __________ 22. __________
3. __________ 8. __________ 13. __________ 18. __________ 23. __________
4. __________ 9. __________ 14. __________ 19. __________ 24. __________
5. __________ 10. __________ 15. __________ 20. __________ 25. __________
Table 1.23
1.14.7 Restaurants Stratied by City and Entree Cost
Restaurants Used in Sample
47
Entree Cost Under $10 $10 to under $15 $15 to under $20 Over $20
San Jose El Abuelo Taq,
Pasta Mia,
Emmas Express,
Bamboo Hut
Emperors Guard,
Creekside Inn
Agenda, Gervais,
Miros
Blakes, Eulipia,
Hayes Mansion,
Germania
Palo Alto Senor Taco, Olive
Garden, Taxis
Mings, P.A. Joes,
Stickneys
Scotts Seafood,
Poolside Grill,
Fish Market
Sundance Mine,
Maddalenas,
Spagos
Los Gatos Marys Patio,
Mount Everest,
Sweet Peas,
Andele Taqueria
Lindseys, Willow
Street
Toll House Charter House, La
Maison Du Cafe
Mountain View Maharaja, New
Mas, Thai-Ric,
Garden Fresh
Amber Indian, La
Fiesta, Fiesta del
Mar, Dawit
Austins, Shivas,
Mazeh
Le Petit Bistro
Cupertino Hobees, Hung Fu,
Samrat, Panda Ex-
press
Santa Barb. Grill,
Mand. Gourmet,
Bombay Oven,
Kathmandu West
Fontanas, Blue
Pheasant
Hamasushi, He-
lios
Sunnyvale Chekijababi, Taj
India, Full Throt-
tle, Tia Juana,
Lemon Grass
Pacic Fresh,
Charley Browns,
Cafe Cameroon,
Faz, Arubas
Lion & Compass,
The Palace, Beau
Sejour
Santa Clara Rangoli, Ar-
madillo Willys,
Thai Pepper,
Pasand
Arthurs, Katies
Cafe, Pedros, La
Galleria
Birks, Truya
Sushi, Valley
Plaza
Lakeside, Mari-
anis
Table 1.24
NOTE: The original lab was designed and contributed by Carol Olmstead.
48 CHAPTER 1. SAMPLING AND DATA
Solutions to Exercises in Chapter 1
Solution to Example 1.5, Problem (p. 18)
Items 1, 5, 11, and 12 are quantitative discrete; items 4, 6, 10, and 14 are quantitative continuous; and items
2, 3, 7, 8, 9, and 13 are qualitative.
Solution to Example 1.10, Problem (p. 29)
1. 29%
2. 36%
3. 77%
4. 87
5. quantitative continuous
6. get rosters from each team and choose a simple random sample from each
Solution to Example 1.11, Problem (p. 30)
1. No. Frequency column sums to 18, not 19. Not all cumulative relative frequencies are correct.
2. False. Frequency for 3 miles should be 1; for 2 miles (left out), 2. Cumulative relative frequency
column should read: 0.1052, 0.1579, 0.2105, 0.3684, 0.4737, 0.6316, 0.7368, 0.7895, 0.8421, 0.9474, 1.
3.
5
19
4.
7
19
,
12
19
,
7
19
Solutions to Homework
Solution to Exercise 1.12.1 (p. 35)
a. quantitative - discrete
b. quantitative - continuous
c. qualitative
d. quantitative - continuous
e. quantitative - discrete
f. qualitative
g. qualitative
h. quantitative - continuous
i. quantitative - continuous
j. quantitative - discrete
Solution to Exercise 1.12.3 (p. 35)
a. Cum. Rel. Freq. for 0 is 0.4500
Rel. Freq. for 1 is 0.3000 and Cum. Rel. Freq. for 1 or less is 0.7500
Freq. for 3 is 11 and Rel. Freq. is 0.1833
Cum. Rel. Freq. for 6 or less is 0.9833
Cum. Rel. Freq. for 7 or less is 1
b. 5.00%
c. 93.33%
Solution to Exercise 1.12.5 (p. 36)
a. Children who take ski or snowboard lessons
b. A group of these children
c. The population mean
d. The sample mean
e. X = the age of one child who takes the rst ski or snowboard lesson
49
f. Values for X, such as 3, 7, etc.
Solution to Exercise 1.12.7 (p. 36)
a. The clients of the insurance companies
b. A group of the clients
c. The mean health costs of the clients
d. The mean health costs of the sample
e. X = the health costs of one client
f. Values for X, such as 34, 9, 82, etc.
Solution to Exercise 1.12.9 (p. 37)
a. All the clients of the counselor
b. A group of the clients
c. The proportion of all her clients who stay married
d. The proportion of the sample who stay married
e. X = the number of couples who stay married
f. yes, no
Solution to Exercise 1.12.11 (p. 37)
a. All people (maybe in a certain geographic area, such as the United States)
b. A group of the people
c. The proportion of all people who will buy the product
d. The proportion of the sample who will buy the product
e. X = the number of people who will buy it
f. buy, not buy
Solution to Exercise 1.12.15 (p. 38)
a: 4%
b: 100
Solution to Exercise 1.12.19 (p. 40)
D
Solution to Exercise 1.12.20 (p. 40)
A
Solution to Exercise 1.12.21 (p. 40)
B
Solution to Exercise 1.12.22 (p. 40)
C
Solution to Exercise 1.12.23 (p. 41)
B
Solution to Exercise 1.12.24 (p. 41)
B
Solution to Exercise 1.12.25 (p. 41)
B
Solution to Exercise 1.12.26 (p. 41)
B
Solution to Exercise 1.12.27 (p. 41)
C
Solution to Exercise 1.12.28 (p. 42)
A. Convenience
50 CHAPTER 1. SAMPLING AND DATA
B. Cluster
C. Stratied
D. Systematic
E. Simple Random
Solution to Exercise 1.12.29 (p. 42)
The answer below contains some of the issues that students might discuss for this problem. Individual
students answers may also identify other issues that pertain to this problem that are not included in the
answer below.
The sample is not representative of the population of all college textbooks. Two reasons why it is
not representative are that he only sampled 7 subjects and he only investigated one textbook in each
subject. There are several possible sources of bias in the study. The 7 subjects that he investigated are
all in mathematics and the sciences; there are many subjects in the humanities, social sciences, and many
other subject areas, (for example: literature, art, history, psychology, sociology, business) that he did not
investigate at all. It may be that different subject areas exhibit different patterns of textbook availability,
but his sample would not detect such results.
He also only looked at the most popular textbook in each of the subjects he investigated. The avail-
ability of the most popular textbooks may differ from the availability of other textbooks in one of two
ways:
the most popular textbooks may be more readily available online, because more new copies are
printed and more students nationwide selling back their used copies OR
the most popular textbooks may be harder to nd available online, because more student demand
exhausts the supply more quickly.
In reality, many college students do not use the most popular textbook in their subject, and this study gives
no useful information about the situation for those less popular textbooks.
He could improve this study by
expanding the selection of subjects he investigates so that it is more representative of all subjects
studied by college students and
expanding the selection of textbooks he investigates within each subject to include a mixed represen-
tation of both the popular and less popular textbooks.
Chapter 2
Descriptive Statistics
2.1 Descriptive Statistics
1
2.1.1 Student Learning Outcomes
By the end of this chapter, the student should be able to:
Display data graphically and interpret graphs: stemplots, histograms and boxplots.
Recognize, describe, and calculate the measures of location of data: quartiles and percentiles.
Recognize, describe, and calculate the measures of the center of data: mean, median, and mode.
Recognize, describe, and calculate the measures of the spread of data: variance, standard deviation,
and range.
2.1.2 Introduction
Once you have collected data, what will you do with it? Data can be described and presented in many
different formats. For example, suppose you are interested in buying a house in a particular area. You may
have no clue about the house prices, so you might ask your real estate agent to give you a sample data set
of prices. Looking at all the prices in the sample often is overwhelming. A better way might be to look
at the median price and the variation of prices. The median and variation are just two ways that you will
learn to describe data. Your agent might also provide you with a graph of the data.
In this chapter, you will study numerical and graphical ways to describe and display your data. This area
of statistics is called "Descriptive Statistics". You will learn to calculate, and even more importantly, to
interpret these measurements and graphs.
2.2 Displaying Data
2
A statistical graph is a tool that helps you learn about the shape or distribution of a sample. The graph can
be a more effective way of presenting data than a mass of numbers because we can see where data clusters
and where there are only a few data values. Newspapers and the Internet use graphs to show trends and
to enable readers to compare facts and gures quickly.
Statisticians often graph data rst to get a picture of the data. Then, more formal tools may be applied.
1
This content is available online at <http://cnx.org/content/m16300/1.9/>.
2
This content is available online at <http://cnx.org/content/m16297/1.9/>.
51
52 CHAPTER 2. DESCRIPTIVE STATISTICS
Some of the types of graphs that are used to summarize and organize data are the dot plot, the bar chart,
the histogram, the stem-and-leaf plot, the frequency polygon (a type of broken line graph), pie charts, and
the boxplot. In this chapter, we will briey look at stem-and-leaf plots, line graphs and bar graphs. Our
emphasis will be on histograms and boxplots.
2.3 Stem and Leaf Graphs (Stemplots), Line Graphs and Bar Graphs
3
One simple graph, the stem-and-leaf graph or stemplot, comes fromthe eld of exploratory data analysis.It
is a good choice when the data sets are small. To create the plot, divide each observation of data into a stem
and a leaf. The leaf consists of a nal signicant digit. For example, 23 has stem 2 and leaf 3. Four hundred
thirty-two (432) has stem43 and leaf 2. Five thousand four hundred thirty-two (5,432) has stem543 and leaf
2. The decimal 9.3 has stem 9 and leaf 3. Write the stems in a vertical line from smallest the largest. Draw a
vertical line to the right of the stems. Then write the leaves in increasing order next to their corresponding
stem.
Example 2.1
For Susan Deans spring pre-calculus class, scores for the rst exam were as follows (smallest to
largest):
33; 42; 49; 49; 53; 55; 55; 61; 63; 67; 68; 68; 69; 69; 72; 73; 74; 78; 80; 83; 88; 88; 88; 90; 92; 94; 94; 94; 94;
96; 100
Stem-and-Leaf Diagram
Stem Leaf
3 3
4 299
5 355
6 1378899
7 2348
8 03888
9 0244446
10 0
Table 2.1
The stemplot shows that most scores fell in the 60s, 70s, 80s, and 90s. Eight out of the 31 scores or
approximately 26% of the scores were in the 90s or 100, a fairly high number of As.
The stemplot is a quick way to graph and gives an exact picture of the data. You want to look for an overall
pattern and any outliers. An outlier is an observation of data that does not t the rest of the data. It is
sometimes called an extreme value. When you graph an outlier, it will appear not to t the pattern of the
graph. Some outliers are due to mistakes (for example, writing down 50 instead of 500) while others may
indicate that something unusual is happening. It takes some background information to explain outliers.
In the example above, there were no outliers.
Example 2.2
Create a stem plot using the data:
3
This content is available online at <http://cnx.org/content/m16849/1.15/>.
53
1.1; 1.5; 2.3; 2.5; 2.7; 3.2; 3.3; 3.3; 3.5; 3.8; 4.0; 4.2; 4.5; 4.5; 4.7; 4.8; 5.5; 5.6; 6.5; 6.7; 12.3
The data are the distance (in kilometers) from a home to the nearest supermarket.
Problem (Solution on p. 105.)
1. Are there any outliers?
2. Do the data seem to have any concentration of values?
HINT: The leaves are to the right of the decimal.
Another type of graph that is useful for specic data values is a line graph. In the particular line graph
shown in the example, the x-axis consists of data values and the y-axis consists of frequency points. The
frequency points are connected.
Example 2.3
In a survey, 40 mothers were asked how many times per week a teenager must be reminded to do
his/her chores. The results are shown in the table and the line graph.
Number of times teenager is reminded Frequency
0 2
1 5
2 8
3 14
4 7
5 4
Table 2.2
Bar graphs consist of bars that are separated from each other. The bars can be rectangles or they can be
rectangular boxes and they can be vertical or horizontal.
The bar graph shown in Example 4 has age groups represented on the x-axis and proportions on the y-axis.
54 CHAPTER 2. DESCRIPTIVE STATISTICS
Example 2.4
By the end of 2011, in the United States, Facebook had over 146 million users. The table
shows three age groups, the number of users in each age group and the proportion (%) of
users in each age group. Source: http://www.kenburbary.com/2011/03/facebook-demographics-
revisited-2011-statistics-2/
Age groups Number of Facebook users Proportion (%) of Facebook users
13 - 25 65,082,280 45%
26 - 44 53,300,200 36%
45 - 64 27,885,100 19%
Table 2.3
Example 2.5
The columns in the table below contain the race/ethnicity of U.S. Public Schools: High School
Class of 2011, percentages for the Advanced Placement Examinee Population for that class
and percentages for the Overall Student Population. The 3-dimensional graph shows the
Race/Ethnicity of U.S. Public Schools (qualitative data) on the x-axis and Advanced Placement
Examinee Population percentages on the y-axis. (Source: http://www.collegeboard.com and
Source: http://apreport.collegeboard.org/goals-and-ndings/promoting-equity)
Race/Ethnicity AP Examinee Population Overall Student Population
1 = Asian, Asian American or Pa-
cic Islander
10.3% 5.7%
continued on next page
55
2 = Black or African American 9.0% 14.7%
3 = Hispanic or Latino 17.0% 17.6%
4 = American Indian or Alaska
Native
0.6% 1.1%
5 = White 57.1% 59.2%
6 = Not reported/other 6.0% 1.7%
Table 2.4
Go to Outcomes of Education Figure 22
4
for an example of a bar graph that shows unemployment rates of
persons 25 years and older for 2009.
NOTE: This book contains instructions for constructing a histogram and a box plot for the TI-83+
and TI-84 calculators. You can nd additional instructions for using these calculators on the Texas
Instruments (TI) website
5
.
2.4 Histograms
6
For most of the work you do in this book, you will use a histogram to display the data. One advantage of a
histogram is that it can readily display large data sets. A rule of thumb is to use a histogram when the data
set consists of 100 values or more.
A histogram consists of contiguous boxes. It has both a horizontal axis and a vertical axis. The horizontal
axis is labeled with what the data represents (for instance, distance from your home to school). The vertical
axis is labeled either Frequency or relative frequency. The graph will have the same shape with either
label. The histogram (like the stemplot) can give you the shape of the data, the center, and the spread of the
data. (The next section tells you how to calculate the center and the spread.)
4
http://nces.ed.gov/pubs2011/2011015_5.pdf
5
http://education.ti.com/educationportal/sites/US/sectionHome/support.html
6
This content is available online at <http://cnx.org/content/m16298/1.13/>.
56 CHAPTER 2. DESCRIPTIVE STATISTICS
The relative frequency is equal to the frequency for an observed value of the data divided by the total
number of data values in the sample. (In the chapter on Sampling and Data (Section 1.1), we dened
frequency as the number of times an answer occurs.) If:
f = frequency
n = total number of data values (or the sum of the individual frequencies), and
RF = relative frequency,
then:
RF =
f
n
(2.1)
For example, if 3 students in Mr. Ahabs English class of 40 students received from 90% to 100%, then,
f = 3 , n = 40 , and RF =
f
n
=
3
40
= 0.075
Seven and a half percent of the students received 90% to 100%. Ninety percent to 100 % are quantitative
measures.
To construct a histogram, rst decide how many bars or intervals, also called classes, represent the data.
Many histograms consist of from 5 to 15 bars or classes for clarity. Choose a starting point for the rst
interval to be less than the smallest data value. A convenient starting point is a lower value carried out
to one more decimal place than the value with the most decimal places. For example, if the value with the
most decimal places is 6.1 and this is the smallest value, a convenient starting point is 6.05 (6.1 - 0.05 = 6.05).
We say that 6.05 has more precision. If the value with the most decimal places is 2.23 and the lowest value
is 1.5, a convenient starting point is 1.495 (1.5 - 0.005 = 1.495). If the value with the most decimal places is
3.234 and the lowest value is 1.0, a convenient starting point is 0.9995 (1.0 - .0005 = 0.9995). If all the data
happen to be integers and the smallest value is 2, then a convenient starting point is 1.5 (2 - 0.5 = 1.5). Also,
when the starting point and other boundaries are carried to one additional decimal place, no data value
will fall on a boundary.
Example 2.6
The following data are the heights (in inches to the nearest half inch) of 100 male semiprofessional
soccer players. The heights are continuous data since height is measured.
60; 60.5; 61; 61; 61.5
63.5; 63.5; 63.5
64; 64; 64; 64; 64; 64; 64; 64.5; 64.5; 64.5; 64.5; 64.5; 64.5; 64.5; 64.5
66; 66; 66; 66; 66; 66; 66; 66; 66; 66; 66.5; 66.5; 66.5; 66.5; 66.5; 66.5; 66.5; 66.5; 66.5; 66.5; 66.5; 67; 67;
67; 67; 67; 67; 67; 67; 67; 67; 67; 67; 67.5; 67.5; 67.5; 67.5; 67.5; 67.5; 67.5
68; 68; 69; 69; 69; 69; 69; 69; 69; 69; 69; 69; 69.5; 69.5; 69.5; 69.5; 69.5
70; 70; 70; 70; 70; 70; 70.5; 70.5; 70.5; 71; 71; 71
72; 72; 72; 72.5; 72.5; 73; 73.5
74
The smallest data value is 60. Since the data with the most decimal places has one decimal (for
instance, 61.5), we want our starting point to have two decimal places. Since the numbers 0.5,
0.05, 0.005, etc. are convenient numbers, use 0.05 and subtract it from 60, the smallest value, for
the convenient starting point.
57
60 - 0.05 = 59.95 which is more precise than, say, 61.5 by one decimal place. The starting point is,
then, 59.95.
The largest value is 74. 74+ 0.05 = 74.05 is the ending value.
Next, calculate the width of each bar or class interval. To calculate this width, subtract the starting
point from the ending value and divide by the number of bars (you must choose the number of
bars you desire). Suppose you choose 8 bars.
74.05 59.95
8
= 1.76 (2.2)
NOTE: We will round up to 2 and make each bar or class interval 2 units wide. Rounding up to 2 is
one way to prevent a value from falling on a boundary. Rounding to the next number is necessary
even if it goes against the standard rules of rounding. For this example, using 1.76 as the width
would also work.
The boundaries are:
59.95
59.95 + 2 = 61.95
61.95 + 2 = 63.95
63.95 + 2 = 65.95
65.95 + 2 = 67.95
67.95 + 2 = 69.95
69.95 + 2 = 71.95
71.95 + 2 = 73.95
73.95 + 2 = 75.95
The heights 60 through 61.5 inches are in the interval 59.95 - 61.95. The heights that are 63.5 are
in the interval 61.95 - 63.95. The heights that are 64 through 64.5 are in the interval 63.95 - 65.95.
The heights 66 through 67.5 are in the interval 65.95 - 67.95. The heights 68 through 69.5 are in the
interval 67.95 - 69.95. The heights 70 through 71 are in the interval 69.95 - 71.95. The heights 72
through 73.5 are in the interval 71.95 - 73.95. The height 74 is in the interval 73.95 - 75.95.
The following histogram displays the heights on the x-axis and relative frequency on the y-axis.
58 CHAPTER 2. DESCRIPTIVE STATISTICS
Example 2.7
The following data are the number of books bought by 50 part-time college students at ABC
College. The number of books is discrete data since books are counted.
1; 1; 1; 1; 1; 1; 1; 1; 1; 1; 1
2; 2; 2; 2; 2; 2; 2; 2; 2; 2
3; 3; 3; 3; 3; 3; 3; 3; 3; 3; 3; 3; 3; 3; 3; 3
4; 4; 4; 4; 4; 4
5; 5; 5; 5; 5
6; 6
Eleven students buy 1 book. Ten students buy 2 books. Sixteen students buy 3 books. Six students
buy 4 books. Five students buy 5 books. Two students buy 6 books.
Because the data are integers, subtract 0.5 from 1, the smallest data value and add 0.5 to 6, the
largest data value. Then the starting point is 0.5 and the ending value is 6.5.
Problem (Solution on p. 105.)
Next, calculate the width of each bar or class interval. If the data are discrete and there are not too
many different values, a width that places the data values in the middle of the bar or class interval
is the most convenient. Since the data consist of the numbers 1, 2, 3, 4, 5, 6 and the starting point is
0.5, a width of one places the 1 in the middle of the interval from 0.5 to 1.5, the 2 in the middle of
the interval from 1.5 to 2.5, the 3 in the middle of the interval from 2.5 to 3.5, the 4 in the middle of
the interval from _______ to _______, the 5 in the middle of the interval from _______ to _______,
and the _______ in the middle of the interval from _______ to _______ .
59
Calculate the number of bars as follows:
6.5 0.5
bars
= 1 (2.3)
where 1 is the width of a bar. Therefore, bars = 6.
The following histogram displays the number of books on the x-axis and the frequency on the
y-axis.
2.4.1 Optional Collaborative Exercise
Count the money (bills and change) in your pocket or purse. Your instructor will record the amounts. As a
class, construct a histogram displaying the data. Discuss how many intervals you think is appropriate. You
may want to experiment with the number of intervals. Discuss, also, the shape of the histogram.
Record the data, in dollars (for example, 1.25 dollars).
Construct a histogram.
2.5 Box Plots
7
Box plots or box-whisker plots give a good graphical image of the concentration of the data. They also
show how far from most of the data the extreme values are. The box plot is constructed from ve values:
the smallest value, the rst quartile, the median, the third quartile, and the largest value. The median, the
rst quartile, and the third quartile will be discussed here, and then again in the section on measuring data
in this chapter. We use these values to compare how close other data values are to them.
The median, a number, is a way of measuring the "center" of the data. You can think of the median as the
"middle value," although it does not actually have to be one of the observed values. It is a number that
separates ordered data into halves. Half the values are the same number or smaller than the median and
half the values are the same number or larger. For example, consider the following data:
7
This content is available online at <http://cnx.org/content/m16296/1.12/>.
60 CHAPTER 2. DESCRIPTIVE STATISTICS
1; 11.5; 6; 7.2; 4; 8; 9; 10; 6.8; 8.3; 2; 2; 10; 1
Ordered from smallest to largest:
1; 1; 2; 2; 4; 6; 6.8; 7.2; 8; 8.3; 9; 10; 10; 11.5
The median is between the 7th value, 6.8, and the 8th value 7.2. To nd the median, add the two values
together and divide by 2.
6.8 +7.2
2
= 7 (2.4)
The median is 7. Half of the values are smaller than 7 and half of the values are larger than 7.
Quartiles are numbers that separate the data into quarters. Quartiles may or may not be part of the data.
To nd the quartiles, rst nd the median or second quartile. The rst quartile is the middle value of the
lower half of the data and the third quartile is the middle value of the upper half of the data. To get the
idea, consider the same data set shown above:
1; 1; 2; 2; 4; 6; 6.8; 7.2; 8; 8.3; 9; 10; 10; 11.5
The median or second quartile is 7. The lower half of the data is 1, 1, 2, 2, 4, 6, 6.8. The middle value of the
lower half is 2.
1; 1; 2; 2; 4; 6; 6.8
The number 2, which is part of the data, is the rst quartile. One-fourth of the values are the same or less
than 2 and three-fourths of the values are more than 2.
The upper half of the data is 7.2, 8, 8.3, 9, 10, 10, 11.5. The middle value of the upper half is 9.
7.2; 8; 8.3; 9; 10; 10; 11.5
The number 9, which is part of the data, is the third quartile. Three-fourths of the values are less than 9
and one-fourth of the values are more than 9.
To construct a box plot, use a horizontal number line and a rectangular box. The smallest and largest data
values label the endpoints of the axis. The rst quartile marks one end of the box and the third quartile
marks the other end of the box. The middle fty percent of the data fall inside the box. The "whiskers"
extend from the ends of the box to the smallest and largest data values. The box plot gives a good quick
picture of the data.
NOTE: You may encounter box and whisker plots that have dots marking outlier values. In those
cases, the whiskers are not extending to the minimum and maximum values.
Consider the following data:
1; 1; 2; 2; 4; 6; 6.8 ; 7.2; 8; 8.3; 9; 10; 10; 11.5
The rst quartile is 2, the median is 7, and the third quartile is 9. The smallest value is 1 and the largest
value is 11.5. The box plot is constructed as follows (see calculator instructions in the back of this book or
on the TI web site
8
):
8
http://education.ti.com/educationportal/sites/US/sectionHome/support.html
61
The two whiskers extend from the rst quartile to the smallest value and from the third quartile to the
largest value. The median is shown with a dashed line.
Example 2.8
The following data are the heights of 40 students in a statistics class.
59; 60; 61; 62; 62; 63; 63; 64; 64; 64; 65; 65; 65; 65; 65; 65; 65; 65; 65; 66; 66; 67; 67; 68; 68; 69; 70; 70; 70;
70; 70; 71; 71; 72; 72; 73; 74; 74; 75; 77
Construct a box plot with the following properties:
Smallest value = 59
Largest value = 77
Q1: First quartile = 64.5
Q2: Second quartile or median= 66
Q3: Third quartile = 70
a. Each quarter has 25% of the data.
b. The spreads of the four quarters are 64.5 - 59 = 5.5 (rst quarter), 66 - 64.5 = 1.5 (second quarter),
70 - 66 = 4 (3rd quarter), and 77 - 70 = 7 (fourth quarter). So, the second quarter has the
smallest spread and the fourth quarter has the largest spread.
c. Interquartile Range: IQR = Q3 Q1 = 70 64.5 = 5.5.
d. The interval 59 through 65 has more than 25% of the data so it has more data in it than the
interval 66 through 70 which has 25% of the data.
e. The middle 50% (middle half) of the data has a range of 5.5 inches.
For some sets of data, some of the largest value, smallest value, rst quartile, median, and third
quartile may be the same. For instance, you might have a data set in which the median and the
third quartile are the same. In this case, the diagram would not have a dotted line inside the box
displaying the median. The right side of the box would display both the third quartile and the
median. For example, if the smallest value and the rst quartile were both 1, the median and the
third quartile were both 5, and the largest value was 7, the box plot would look as follows:
62 CHAPTER 2. DESCRIPTIVE STATISTICS
Example 2.9
Test scores for a college statistics class held during the day are:
99; 56; 78; 55.5; 32; 90; 80; 81; 56; 59; 45; 77; 84.5; 84; 70; 72; 68; 32; 79; 90
Test scores for a college statistics class held during the evening are:
98; 78; 68; 83; 81; 89; 88; 76; 65; 45; 98; 90; 80; 84.5; 85; 79; 78; 98; 90; 79; 81; 25.5
Problem (Solution on p. 105.)
What are the smallest and largest data values for each data set?
What is the median, the rst quartile, and the third quartile for each data set?
Create a boxplot for each set of data.
Which boxplot has the widest spread for the middle 50% of the data (the data between the
rst and third quartiles)? What does this mean for that set of data in comparison to the other
set of data?
For each data set, what percent of the data is between the smallest value and the rst quar-
tile? (Answer: 25%) the rst quartile and the median? (Answer: 25%) the median and the
third quartile? the third quartile and the largest value? What percent of the data is between
the rst quartile and the largest value? (Answer: 75%)
The rst data set (the top box plot) has the widest spread for the middle 50% of the data. IQR =
Q3 Q1 is 82.5 56 = 26.5 for the rst data set and 89 78 = 11 for the second data set.
So, the rst set of data has its middle 50% of scores more spread out.
25% of the data is between M and Q3 and 25% is between Q3 and Xmax.
2.6 Measures of the Location of the Data
9
The common measures of location are quartiles and percentiles (%iles). Quartiles are special percentiles.
The rst quartile, Q
1
is the same as the 25th percentile (25th %ile) and the third quartile, Q
3
, is the same as
the 75th percentile (75th %ile). The median, M, is called both the second quartile and the 50th percentile
(50th %ile).
To calculate quartiles and percentiles, the data must be ordered from smallest to largest. Recall that
quartiles divide ordered data into quarters. Percentiles divide ordered data into hundredths. To score in
the 90th percentile of an exam does not mean, necessarily, that you received 90% on a test. It means that
90% of test scores are the same or less than your score and 10% of the test scores are the same or greater
than your test score.
9
This content is available online at <http://cnx.org/content/m16314/1.17/>.
63
Percentiles are useful for comparing values. For this reason, universities and colleges use percentiles
extensively.
Percentiles are mostly used with very large populations. Therefore, if you were to say that 90% of
the test scores are less (and not the same or less) than your score, it would be acceptable because removing
one particular data value is not signicant.
The interquartile range is a number that indicates the spread of the middle half or the middle 50% of the
data. It is the difference between the third quartile (Q
3
) and the rst quartile (Q
1
).
IQR = Q
3
Q
1
(2.5)
The IQR can help to determine potential outliers. A value is suspected to be a potential outlier if it is
less than (1.5) (IQR) below the rst quartile or more than (1.5) (IQR) above the third quartile. Potential
outliers always need further investigation.
Example 2.10
For the following 13 real estate prices, calculate the IQR and determine if any prices are outliers.
Prices are in dollars. (Source: San Jose Mercury News)
389,950; 230,500; 158,000; 479,000; 639,000; 114,950; 5,500,000; 387,000; 659,000; 529,000; 575,000;
488,800; 1,095,000
Solution
Order the data from smallest to largest.
114,950; 158,000; 230,500; 387,000; 389,950; 479,000; 488,800; 529,000; 575,000; 639,000; 659,000;
1,095,000; 5,500,000
M = 488, 800
Q
1
=
230500+387000
2
= 308750
Q
3
=
639000+659000
2
= 649000
IQR = 649000 308750 = 340250
(1.5) (IQR) = (1.5) (340250) = 510375
Q
1
(1.5) (IQR) = 308750 510375 = 201625
Q
3
+ (1.5) (IQR) = 649000 +510375 = 1159375
No house price is less than -201625. However, 5,500,000 is more than 1,159,375. Therefore,
5,500,000 is a potential outlier.
Example 2.11
For the two data sets in the test scores example (p. 62), nd the following:
a. The interquartile range. Compare the two interquartile ranges.
b. Any outliers in either set.
c. The 30th percentile and the 80th percentile for each set. How much data falls below the 30th
percentile? Above the 80th percentile?
64 CHAPTER 2. DESCRIPTIVE STATISTICS
Example 2.12: Finding Quartiles and Percentiles Using a Table
Fifty statistics students were asked how much sleep they get per school night (rounded to the
nearest hour). The results were (student data):
AMOUNT OF SLEEP
PER SCHOOL NIGHT
(HOURS)
FREQUENCY RELATIVE FRE-
QUENCY
CUMULATIVE RELA-
TIVE FREQUENCY
4 2 0.04 0.04
5 5 0.10 0.14
6 7 0.14 0.28
7 12 0.24 0.52
8 14 0.28 0.80
9 7 0.14 0.94
10 3 0.06 1.00
Table 2.5
Find the 28th percentile: Notice the 0.28 in the "cumulative relative frequency" column. 28% of 50
data values = 14. There are 14 values less than the 28th %ile. They include the two 4s, the ve 5s,
and the seven 6s. The 28th %ile is between the last 6 and the rst 7. The 28th %ile is 6.5.
Find the median: Look again at the "cumulative relative frequency " column and nd 0.52. The
median is the 50th %ile or the second quartile. 50% of 50 = 25. There are 25 values less than the
median. They include the two 4s, the ve 5s, the seven 6s, and eleven of the 7s. The median or
50th %ile is between the 25th (7) and 26th (7) values. The median is 7.
Find the third quartile: The third quartile is the same as the 75th percentile. You can "eyeball" this
answer. If you look at the "cumulative relative frequency" column, you nd 0.52 and 0.80. When
you have all the 4s, 5s, 6s and 7s, you have 52% of the data. When you include all the 8s, you have
80% of the data. The 75th %ile, then, must be an 8 . Another way to look at the problem is to nd
75% of 50 (= 37.5) and round up to 38. The third quartile, Q
3
, is the 38th value which is an 8. You
can check this answer by counting the values. (There are 37 values below the third quartile and 12
values above.)
Example 2.13
Using the table:
1. Find the 80th percentile.
2. Find the 90th percentile.
3. Find the rst quartile. What is another name for the rst quartile?
4. Construct a box plot of the data.
Collaborative Classroom Exercise: Your instructor or a member of the class will ask everyone in class how
many sweaters they own. Answer the following questions.
1. How many students were surveyed?
2. What kind of sampling did you do?
65
3. Find the mean and standard deviation.
4. Find the mode.
5. Construct 2 different histograms. For each, starting value = _____ ending value = ____.
6. Find the median, rst quartile, and third quartile.
7. Construct a box plot.
8. Construct a table of the data to nd the following:
The 10th percentile
The 70th percentile
The percent of students who own less than 4 sweaters
Interpreting Percentiles, Quartiles, and Median
A percentile indicates the relative standing of a data value when data are sorted into numerical order, from
smallest to largest. p% of data values are less than or equal to the pth percentile. For example, 15% of data
values are less than or equal to the 15th percentile.
Low percentiles always correspond to lower data values.
High percentiles always correspond to higher data values.
A percentile may or may not correspond to a value judgment about whether it is "good" or "bad". The
interpretation of whether a certain percentile is good or bad depends on the context of the situation to
which the data applies. In some situations, a low percentile would be considered "good; in other contexts
a high percentile might be considered "good". In many situations, there is no value judgment that applies.
Understanding how to properly interpret percentiles is important not only when describing data,
but is also important in later chapters of this textbook when calculating probabilities.
Guideline:
When writing the interpretation of a percentile in the context of the given data, the sentence should
contain the following information:
information about the context of the situation being considered,
the data value (value of the variable) that represents the percentile,
the percent of individuals or items with data values below the percentile.
Additionally, you may also choose to state the percent of individuals or items with data values above
the percentile.
Example 2.14
On a timed math test, the rst quartile for times for nishing the exam was 35 minutes. Interpret
the rst quartile in the context of this situation.
25% of students nished the exam in 35 minutes or less.
75% of students nished the exam in 35 minutes or more.
A low percentile could be considered good, as nishing more quickly on a timed exam is
desirable. (If you take too long, you might not be able to nish.)
Example 2.15
On a 20 question math test, the 70th percentile for number of correct answers was 16. Interpret
the 70th percentile in the context of this situation.
70% of students answered 16 or fewer questions correctly.
30% of students answered 16 or more questions correctly.
Note: A high percentile could be considered good, as answering more questions correctly is
desirable.
66 CHAPTER 2. DESCRIPTIVE STATISTICS
Example 2.16
At a certain community college, it was found that the 30th percentile of credit units that students
are enrolled for is 7 units. Interpret the 30th percentile in the context of this situation.
30% of students are enrolled in 7 or fewer credit units
70% of students are enrolled in 7 or more credit units
In this example, there is no "good" or "bad" value judgment associated with a higher or
lower percentile. Students attend community college for varied reasons and needs, and their
course load varies according to their needs.
Do the following Practice Problems for Interpreting Percentiles
Exercise 2.6.1 (Solution on p. 106.)
a. For runners in a race, a low time means a faster run. The winners in a race have the shortest
running times. Is it more desirable to have a nish time with a high or a low percentile when
running a race?
b. The 20th percentile of run times in a particular race is 5.2 minutes. Write a sentence interpreting
the 20th percentile in the context of the situation.
c. A bicyclist in the 90th percentile of a bicycle race between two towns completed the race in 1
hour and 12 minutes. Is he among the fastest or slowest cyclists in the race? Write a sentence
interpreting the 90th percentile in the context of the situation.
Exercise 2.6.2 (Solution on p. 107.)
a. For runners in a race, a higher speed means a faster run. Is it more desirable to have a speed
with a high or a low percentile when running a race?
b. The 40th percentile of speeds in a particular race is 7.5 miles per hour. Write a sentence inter-
preting the 40th percentile in the context of the situation.
Exercise 2.6.3 (Solution on p. 107.)
On an exam, would it be more desirable to earn a grade with a high or low percentile? Explain.
Exercise 2.6.4 (Solution on p. 107.)
Mina is waiting in line at the Department of Motor Vehicles (DMV). Her wait time of 32 minutes
is the 85th percentile of wait times. Is that good or bad? Write a sentence interpreting the 85th
percentile in the context of this situation.
Exercise 2.6.5 (Solution on p. 107.)
In a survey collecting data about the salaries earned by recent college graduates, Li found that her
salary was in the 78th percentile. Should Li be pleased or upset by this result? Explain.
Exercise 2.6.6 (Solution on p. 107.)
In a study collecting data about the repair costs of damage to automobiles in a certain type of
crash tests, a certain model of car had $1700 in damage and was in the 90th percentile. Should the
manufacturer and/or a consumer be pleased or upset by this result? Explain. Write a sentence
that interprets the 90th percentile in the context of this problem.
Exercise 2.6.7 (Solution on p. 107.)
The University of California has two criteria used to set admission standards for freshman to be
admitted to a college in the UC system:
a. Students GPAs and scores on standardized tests (SATs and ACTs) are entered into a formula
that calculates an "admissions index" score. The admissions index score is used to set eligi-
bility standards intended to meet the goal of admitting the top 12% of high school students
in the state. In this context, what percentile does the top 12% represent?
67
b. Students whose GPAs are at or above the 96th percentile of all students at their high school
are eligible (called eligible in the local context), even if they are not in the top 12% of all
students in the state. What percent of students from each high school are "eligible in the
local context"?
Exercise 2.6.8 (Solution on p. 107.)
Suppose that you are buying a house. You and your realtor have determined that the most expen-
sive house you can afford is the 34th percentile. The 34th percentile of housing prices is $240,000
in the town you want to move to. In this town, can you afford 34% of the houses or 66% of the
houses?
**With contributions from Roberta Bloom
2.7 Measures of the Center of the Data
10
The "center" of a data set is also a way of describing location. The two most widely used measures of the
"center" of the data are the mean (average) and the median. To calculate the mean weight of 50 people,
add the 50 weights together and divide by 50. To nd the median weight of the 50 people, order the data
and nd the number that splits the data into two equal parts (previously discussed under box plots in this
chapter). The median is generally a better measure of the center when there are extreme values or outliers
because it is not affected by the precise numerical values of the outliers. The mean is the most common
measure of the center.
NOTE: The words "mean" and "average" are often used interchangeably. The substitution of one
word for the other is common practice. The technical term is "arithmetic mean" and "average" is
technically a center location. However, in practice among non-statisticians, "average" is commonly
accepted for "arithmetic mean."
The mean can also be calculated by multiplying each distinct value by its frequency and then dividing the
sum by the total number of data values. The letter used to represent the sample mean is an x with a bar
over it (pronounced "x bar"): x.
The Greek letter (pronounced "mew") represents the population mean. One of the requirements for the
sample mean to be a good estimate of the population mean is for the sample taken to be truly random.
To see that both ways of calculating the mean are the same, consider the sample:
1; 1; 1; 2; 2; 3; 4; 4; 4; 4; 4
x =
1 +1 +1 +2 +2 +3 +4 +4 +4 +4 +4
11
= 2.7 (2.6)
x =
3 1 +2 2 +1 3 +5 4
11
= 2.7 (2.7)
In the second example, the frequencies are 3, 2, 1, and 5.
You can quickly nd the location of the median by using the expression
n+1
2
.
The letter n is the total number of data values in the sample. If n is an odd number, the median is the middle
value of the ordered data (ordered smallest to largest). If n is an even number, the median is equal to the
two middle values added together and divided by 2 after the data has been ordered. For example, if the
total number of data values is 97, then
n+1
2
=
97+1
2
= 49. The median is the 49th value in the ordered data.
10
This content is available online at <http://cnx.org/content/m17102/1.12/>.
68 CHAPTER 2. DESCRIPTIVE STATISTICS
If the total number of data values is 100, then
n+1
2
=
100+1
2
= 50.5. The median occurs midway between the
50th and 51st values. The location of the median and the value of the median are not the same. The upper
case letter M is often used to represent the median. The next example illustrates the location of the median
and the value of the median.
Example 2.17
AIDS data indicating the number of months an AIDS patient lives after taking a new antibody
drug are as follows (smallest to largest):
3; 4; 8; 8; 10; 11; 12; 13; 14; 15; 15; 16; 16; 17; 17; 18; 21; 22; 22; 24; 24; 25; 26; 26; 27; 27; 29; 29; 31; 32;
33; 33; 34; 34; 35; 37; 40; 44; 44; 47
Calculate the mean and the median.
Solution
The calculation for the mean is:
x =
[3+4+(8)(2)+10+11+12+13+14+(15)(2)+(16)(2)+...+35+37+40+(44)(2)+47]
40
= 23.6
To nd the median, M, rst use the formula for the location. The location is:
n+1
2
=
40+1
2
= 20.5
Starting at the smallest value, the median is located between the 20th and 21st values (the two
24s):
3; 4; 8; 8; 10; 11; 12; 13; 14; 15; 15; 16; 16; 17; 17; 18; 21; 22; 22; 24; 24; 25; 26; 26; 27; 27; 29; 29; 31; 32;
33; 33; 34; 34; 35; 37; 40; 44; 44; 47
M =
24+24
2
= 24
The median is 24.
Example 2.18
Suppose that, in a small town of 50 people, one person earns $5,000,000 per year and the other 49
each earn $30,000. Which is the better measure of the "center," the mean or the median?
Solution
x =
5000000+4930000
50
= 129400
M = 30000
(There are 49 people who earn $30,000 and one person who earns $5,000,000.)
The median is a better measure of the "center" than the mean because 49 of the values are 30,000
and one is 5,000,000. The 5,000,000 is an outlier. The 30,000 gives us a better sense of the middle of
the data.
Another measure of the center is the mode. The mode is the most frequent value. If a data set has two
values that occur the same number of times, then the set is bimodal.
Example 2.19: Statistics exam scores for 20 students are as follows
Statistics exam scores for 20 students are as follows:
69
50 ; 53 ; 59 ; 59 ; 63 ; 63 ; 72 ; 72 ; 72 ; 72 ; 72 ; 76 ; 78 ; 81 ; 83 ; 84 ; 84 ; 84 ; 90 ; 93
Problem
Find the mode.
Solution
The most frequent score is 72, which occurs ve times. Mode = 72.
Example 2.20
Five real estate exam scores are 430, 430, 480, 480, 495. The data set is bimodal because the scores
430 and 480 each occur twice.
When is the mode the best measure of the "center"? Consider a weight loss programthat advertises
a mean weight loss of six pounds the rst week of the program. The mode might indicate that most
people lose two pounds the rst week, making the program less appealing.
NOTE: The mode can be calculated for qualitative data as well as for quantitative data.
Statistical software will easily calculate the mean, the median, and the mode. Some graphing
calculators can also make these calculations. In the real world, people make these calculations
using software.
2.7.1 The Law of Large Numbers and the Mean
The Law of Large Numbers says that if you take samples of larger and larger size from any population,
then the mean x of the sample is very likely to get closer and closer to . This is discussed in more detail in
The Central Limit Theorem.
NOTE: The formula for the mean is located in the Summary of Formulas (Section 2.10) section
course.
2.7.2 Sampling Distributions and Statistic of a Sampling Distribution
You can think of a sampling distribution as a relative frequency distribution with a great many samples.
(See Sampling and Data for a review of relative frequency). Suppose thirty randomly selected students
were asked the number of movies they watched the previous week. The results are in the relative frequency
table shown below.
# of movies Relative Frequency
0 5/30
1 15/30
2 6/30
3 4/30
4 1/30
Table 2.6
70 CHAPTER 2. DESCRIPTIVE STATISTICS
If you let the number of samples get very large (say, 300 million or more), the relative frequency table
becomes a relative frequency distribution.
A statistic is a number calculated from a sample. Statistic examples include the mean, the median and the
mode as well as others. The sample mean x is an example of a statistic which estimates the population
mean .
2.8 Skewness and the Mean, Median, and Mode
11
Consider the following data set:
4 ; 5 ; 6 ; 6 ; 6 ; 7 ; 7 ; 7 ; 7 ; 7 ; 7 ; 8 ; 8 ; 8 ; 9 ; 10
This data set produces the histogram shown below. Each interval has width one and each value is located
in the middle of an interval.
The histogram displays a symmetrical distribution of data. A distribution is symmetrical if a vertical line
can be drawn at some point in the histogram such that the shape to the left and the right of the vertical
line are mirror images of each other. The mean, the median, and the mode are each 7 for these data. In a
perfectly symmetrical distribution, the mean and the median are the same. This example has one mode
(unimodal) and the mode is the same as the mean and median. In a symmetrical distribution that has two
modes (bimodal), the two modes would be different from the mean and median.
The histogram for the data:
4 ; 5 ; 6 ; 6 ; 6 ; 7 ; 7 ; 7 ; 7 ; 8
is not symmetrical. The right-hand side seems "chopped off" compared to the left side. The shape distribu-
tion is called skewed to the left because it is pulled out to the left.
11
This content is available online at <http://cnx.org/content/m17104/1.9/>.
71
The mean is 6.3, the median is 6.5, and the mode is 7. Notice that the mean is less than the median and
they are both less than the mode. The mean and the median both reect the skewing but the mean more
so.
The histogram for the data:
6 ; 7 ; 7 ; 7 ; 7 ; 8 ; 8 ; 8 ; 9 ; 10
is also not symmetrical. It is skewed to the right.
The mean is 7.7, the median is 7.5, and the mode is 7. Of the three statistics, the mean is the largest, while
the mode is the smallest. Again, the mean reects the skewing the most.
To summarize, generally if the distribution of data is skewed to the left, the mean is less than the median,
which is often less than the mode. If the distribution of data is skewed to the right, the mode is often less
than the median, which is less than the mean.
Skewness and symmetry become important when we discuss probability distributions in later chapters.
72 CHAPTER 2. DESCRIPTIVE STATISTICS
2.9 Measures of the Spread of the Data
12
An important characteristic of any set of data is the variation in the data. In some data sets, the data values
are concentrated closely near the mean; in other data sets, the data values are more widely spread out from
the mean. The most common measure of variation, or spread, is the standard deviation.
The standard deviation is a number that measures how far data values are from their mean.
The standard deviation
provides a numerical measure of the overall amount of variation in a data set
can be used to determine whether a particular data value is close to or far from the mean
The standard deviation provides a measure of the overall variation in a data set
The standard deviation is always positive or 0. The standard deviation is small when the data are all
concentrated close to the mean, exhibiting little variation or spread. The standard deviation is larger when
the data values are more spread out from the mean, exhibiting more variation.
Suppose that we are studying waiting times at the checkout line for customers at supermarket A and
supermarket B; the average wait time at both markets is 5 minutes. At market A, the standard deviation
for the waiting time is 2 minutes; at market B the standard deviation for the waiting time is 4 minutes.
Because market B has a higher standard deviation, we know that there is more variation in the wait-
ing times at market B. Overall, wait times at market B are more spread out from the average; wait times at
market A are more concentrated near the average.
The standard deviation can be used to determine whether a data value is close to or far from the mean.
Suppose that Rosa and Binh both shop at Market A. Rosa waits for 7 minutes and Binh waits for 1 minute
at the checkout counter. At market A, the mean wait time is 5 minutes and the standard deviation is 2
minutes. The standard deviation can be used to determine whether a data value is close to or far from the
mean.
Rosa waits for 7 minutes:
7 is 2 minutes longer than the average of 5; 2 minutes is equal to one standard deviation.
Rosas wait time of 7 minutes is 2 minutes longer than the average of 5 minutes.
Rosas wait time of 7 minutes is one standard deviation above the average of 5 minutes.
Binh waits for 1 minute.
1 is 4 minutes less than the average of 5; 4 minutes is equal to two standard deviations.
Binhs wait time of 1 minute is 4 minutes less than the average of 5 minutes.
Binhs wait time of 1 minute is two standard deviations below the average of 5 minutes.
A data value that is two standard deviations from the average is just on the borderline for what many
statisticians would consider to be far from the average. Considering data to be far from the mean if it
is more than 2 standard deviations away is more of an approximate "rule of thumb" than a rigid rule.
In general, the shape of the distribution of the data affects how much of the data is further away than
2 standard deviations. (We will learn more about this in later chapters.)
The number line may help you understand standard deviation. If we were to put 5 and 7 on a number line,
7 is to the right of 5. We say, then, that 7 is one standard deviation to the right of 5 because
5 + (1) (2) = 7.
12
This content is available online at <http://cnx.org/content/m17103/1.14/>.
73
If 1 were also part of the data set, then 1 is two standard deviations to the left of 5 because
5 + (2) (2) = 1.
In general, a value = mean + (#ofSTDEV)(standard deviation)
where #ofSTDEVs = the number of standard deviations
7 is one standard deviation more than the mean of 5 because: 7=5+(1)(2)
1 is two standard deviations less than the mean of 5 because: 1=5+(2)(2)
The equation value = mean + (#ofSTDEVs)(standard deviation) can be expressed for a sample and for a
population:
sample: x = x + (#o f STDEV) (s)
Population: x = + (#o f STDEV) ()
The lower case letter s represents the sample standard deviation and the Greek letter (sigma, lower case)
represents the population standard deviation.
The symbol x is the sample mean and the Greek symbol is the population mean.
Calculating the Standard Deviation
If x is a number, then the difference "x - mean" is called its deviation. In a data set, there are as many
deviations as there are items in the data set. The deviations are used to calculate the standard deviation.
If the numbers belong to a population, in symbols a deviation is x . For sample data, in symbols a
deviation is x x .
The procedure to calculate the standard deviation depends on whether the numbers are the entire popula-
tion or are data from a sample. The calculations are similar, but not identical. Therefore the symbol used
to represent the standard deviation depends on whether it is calculated from a population or a sample.
The lower case letter s represents the sample standard deviation and the Greek letter (sigma, lower case)
represents the population standard deviation. If the sample has the same characteristics as the population,
then s should be a good estimate of .
To calculate the standard deviation, we need to calculate the variance rst. The variance is an average of
the squares of the deviations (the x x values for a sample, or the x values for a population). The
symbol
2
represents the population variance; the population standard deviation is the square root of
the population variance. The symbol s
2
represents the sample variance; the sample standard deviation s is
the square root of the sample variance. You can think of the standard deviation as a special average of the
deviations.
If the numbers come from a census of the entire population and not a sample, when we calculate the aver-
age of the squared deviations to nd the variance, we divide by N, the number of items in the population.
If the data are from a sample rather than a population, when we calculate the average of the squared devi-
ations, we divide by n-1, one less than the number of items in the sample. You can see that in the formulas
below.
74 CHAPTER 2. DESCRIPTIVE STATISTICS
Formulas for the Sample Standard Deviation
s =
_
(xx)
2
n1
or s =
_
f (xx)
2
n1
For the sample standard deviation, the denominator is n-1, that is the sample size MINUS 1.
Formulas for the Population Standard Deviation
=
_
(x)
2
N
or =
_
f (x)
2
N
For the population standard deviation, the denominator is N, the number of items in the population.
In these formulas, f represents the frequency with which a value appears. For example, if a value appears
once, f is 1. If a value appears three times in the data set or population, f is 3.
Sampling Variability of a Statistic
The statistic of a sampling distribution was discussed in Descriptive Statistics: Measuring the Center of
the Data. Howmuch the statistic varies fromone sample to another is known as the sampling variability of
a statistic. You typically measure the sampling variability of a statistic by its standard error. The standard
error of the mean is an example of a standard error. It is a special standard deviation and is known as the
standard deviation of the sampling distribution of the mean. You will cover the standard error of the mean
in The Central Limit Theorem (not now). The notation for the standard error of the mean is

n
where is
the standard deviation of the population and n is the size of the sample.
NOTE: In practice, USE A CALCULATOR OR COMPUTER SOFTWARE TO CALCULATE
THE STANDARD DEVIATION. If you are using a TI-83,83+,84+ calculator, you need to select
the appropriate standard deviation
x
or s
x
from the summary statistics. We will concentrate on
using and interpreting the information that the standard deviation gives us. However you should
study the following step-by-step example to help you understand how the standard deviation
measures variation from the mean.
Example 2.21
In a fth grade class, the teacher was interested in the average age and the sample standard
deviation of the ages of her students. The following data are the ages for a SAMPLE of n = 20 fth
grade students. The ages are rounded to the nearest half year:
9 ; 9.5 ; 9.5 ; 10 ; 10 ; 10 ; 10 ; 10.5 ; 10.5 ; 10.5 ; 10.5 ; 11 ; 11 ; 11 ; 11 ; 11 ; 11 ; 11.5 ; 11.5 ; 11.5
x =
9 +9.5 2 +10 4 +10.5 4 +11 6 +11.5 3
20
= 10.525 (2.8)
The average age is 10.53 years, rounded to 2 places.
The variance may be calculated by using a table. Then the standard deviation is calculated by
taking the square root of the variance. We will explain the parts of the table after calculating s.
Data Freq. Deviations Deviations
2
(Freq.)(Deviations
2
)
x f (x x) (x x)
2
( f ) (x x)
2
9 1 9 10.525 = 1.525 (1.525)
2
= 2.325625 1 2.325625 = 2.325625
9.5 2 9.5 10.525 = 1.025 (1.025)
2
= 1.050625 2 1.050625 = 2.101250
10 4 10 10.525 = 0.525 (0.525)
2
= 0.275625 4 .275625 = 1.1025
10.5 4 10.5 10.525 = 0.025 (0.025)
2
= 0.000625 4 .000625 = .0025
11 6 11 10.525 = 0.475 (0.475)
2
= 0.225625 6 .225625 = 1.35375
11.5 3 11.5 10.525 = 0.975 (0.975)
2
= 0.950625 3 .950625 = 2.851875
75
Table 2.7
The sample variance, s
2
, is equal to the sumof the last column (9.7375) divided by the total number
of data values minus one (20 - 1):
s
2
=
9.7375
201
= 0.5125
The sample standard deviation s is equal to the square root of the sample variance:
s =

0.5125 = .0715891 Rounded to two decimal places, s = 0.72


Typically, you do the calculation for the standard deviation on your calculator or computer. The
intermediate results are not rounded. This is done for accuracy.
Problem 1
Verify the mean and standard deviation calculated above on your calculator or computer.
Solution
For the TI-83,83+,84+, enter data into the list editor.
Put the data values in list L1 and the frequencies in list L2.
STAT CALC 1-VarStats L1, L2
x=10.525
Use Sx because this is sample data (not a population): Sx=.715891
For the following problems, recall that value = mean + (#ofSTDEVs)(standard deviation)
For a sample: x = x + (#ofSTDEVs)(s)
For a population: x = + (#ofSTDEVs)( )
For this example, use x = x + (#ofSTDEVs)(s) because the data is from a sample
Problem 2
Find the value that is 1 standard deviation above the mean. Find (x +1s).
Solution
(x +1s) = 10.53 + (1) (0.72) = 11.25
Problem 3
Find the value that is two standard deviations below the mean. Find (x 2s).
Solution
(x 2s) = 10.53 (2) (0.72) = 9.09
Problem 4
Find the values that are 1.5 standard deviations from (below and above) the mean.
Solution
(x 1.5s) = 10.53 (1.5) (0.72) = 9.45
(x +1.5s) = 10.53 + (1.5) (0.72) = 11.61
76 CHAPTER 2. DESCRIPTIVE STATISTICS
Explanation of the standard deviation calculation shown in the table
The deviations show how spread out the data are about the mean. The data value 11.5 is farther from the
mean than is the data value 11. The deviations 0.97 and 0.47 indicate that. Apositive deviation occurs when
the data value is greater than the mean. A negative deviation occurs when the data value is less than the
mean; the deviation is -1.525 for the data value 9. If you add the deviations, the sum is always zero. (For
this example, there are n=20 deviations.) So you cannot simply add the deviations to get the spread of the
data. By squaring the deviations, you make them positive numbers, and the sum will also be positive. The
variance, then, is the average squared deviation.
The variance is a squared measure and does not have the same units as the data. Taking the square root
solves the problem. The standard deviation measures the spread in the same units as the data.
Notice that instead of dividing by n=20, the calculation divided by n-1=20-1=19 because the data is a sam-
ple. For the sample variance, we divide by the sample size minus one (n 1). Why not divide by n? The
answer has to do with the population variance. The sample variance is an estimate of the population vari-
ance. Based on the theoretical mathematics that lies behind these calculations, dividing by (n 1) gives a
better estimate of the population variance.
NOTE: Your concentration should be on what the standard deviation tells us about the data. The
standard deviation is a number which measures how far the data are spread from the mean. Let a
calculator or computer do the arithmetic.
The standard deviation, s or , is either zero or larger than zero. When the standard deviation is 0, there is
no spread; that is, the all the data values are equal to each other. The standard deviation is small when the
data are all concentrated close to the mean, and is larger when the data values show more variation from
the mean. When the standard deviation is a lot larger than zero, the data values are very spread out about
the mean; outliers can make s or very large.
The standard deviation, when rst presented, can seem unclear. By graphing your data, you can get a
better "feel" for the deviations and the standard deviation. You will nd that in symmetrical distributions,
the standard deviation can be very helpful but in skewed distributions, the standard deviation may not be
much help. The reason is that the two sides of a skewed distribution have different spreads. In a skewed
distribution, it is better to look at the rst quartile, the median, the third quartile, the smallest value, and
the largest value. Because numbers can be confusing, always graph your data.
NOTE: The formula for the standard deviation is at the end of the chapter.
Example 2.22
Use the following data (rst exam scores) from Susan Deans spring pre-calculus class:
33; 42; 49; 49; 53; 55; 55; 61; 63; 67; 68; 68; 69; 69; 72; 73; 74; 78; 80; 83; 88; 88; 88; 90; 92; 94; 94; 94; 94;
96; 100
a. Create a chart containing the data, frequencies, relative frequencies, and cumulative relative
frequencies to three decimal places.
b. Calculate the following to one decimal place using a TI-83+ or TI-84 calculator:
i. The sample mean
ii. The sample standard deviation
iii. The median
iv. The rst quartile
v. The third quartile
77
vi. IQR
c. Construct a box plot and a histogram on the same set of axes. Make comments about the box
plot, the histogram, and the chart.
Solution
a.
Data Frequency Relative Frequency Cumulative Relative Frequency
33 1 0.032 0.032
42 1 0.032 0.064
49 2 0.065 0.129
53 1 0.032 0.161
55 2 0.065 0.226
61 1 0.032 0.258
63 1 0.032 0.29
67 1 0.032 0.322
68 2 0.065 0.387
69 2 0.065 0.452
72 1 0.032 0.484
73 1 0.032 0.516
74 1 0.032 0.548
78 1 0.032 0.580
80 1 0.032 0.612
83 1 0.032 0.644
88 3 0.097 0.741
90 1 0.032 0.773
92 1 0.032 0.805
94 4 0.129 0.934
96 1 0.032 0.966
100 1 0.032 0.998 (Why isnt this value 1?)
Table 2.8
b. i. The sample mean = 73.5
ii. The sample standard deviation = 17.9
iii. The median = 73
iv. The rst quartile = 61
v. The third quartile = 90
vi. IQR = 90 - 61 = 29
c. The x-axis goes from 32.5 to 100.5; y-axis goes from -2.4 to 15 for the histogram; number of
intervals is 5 for the histogram so the width of an interval is (100.5 - 32.5) divided by 5 which
is equal to 13.6. Endpoints of the intervals: starting point is 32.5, 32.5+13.6 = 46.1, 46.1+13.6 =
59.7, 59.7+13.6 = 73.3, 73.3+13.6 = 86.9, 86.9+13.6 = 100.5 = the ending value; No data values
fall on an interval boundary.
78 CHAPTER 2. DESCRIPTIVE STATISTICS
Figure 2.1
The long left whisker in the box plot is reected in the left side of the histogram. The spread of
the exam scores in the lower 50% is greater (73 - 33 = 40) than the spread in the upper 50% (100 -
73 = 27). The histogram, box plot, and chart all reect this. There are a substantial number of A
and B grades (80s, 90s, and 100). The histogram clearly shows this. The box plot shows us that the
middle 50% of the exam scores (IQR = 29) are Ds, Cs, and Bs. The box plot also shows us that the
lower 25% of the exam scores are Ds and Fs.
Comparing Values from Different Data Sets
The standard deviation is useful when comparing data values that come fromdifferent data sets. If the data
sets have different means and standard deviations, it can be misleading to compare the data values directly.
For each data value, calculate how many standard deviations the value is away from its mean.
Use the formula: value = mean + (#ofSTDEVs)(standard deviation); solve for #ofSTDEVs.
#o f STDEVs =
valuemean
standard deviation
Compare the results of this calculation.
#ofSTDEVs is often called a "z-score"; we can use the symbol z. In symbols, the formulas become:
Sample x = x + z s z =
xx
s
Population x = + z z =
x

Table 2.9
79
Example 2.23
Two students, John and Ali, from different high schools, wanted to nd out who had the highest
G.P.A. when compared to his school. Which student had the highest G.P.A. when compared to his
school?
Student GPA School Mean GPA School Standard Deviation
John 2.85 3.0 0.7
Ali 77 80 10
Table 2.10
Solution
For each student, determine how many standard deviations (#ofSTDEVs) his GPA is away from
the average, for his school. Pay careful attention to signs when comparing and interpreting the
answer.
#o f STDEVs =
valuemean
standard deviation
; z =
x

For John, z = #o f STDEVs =


2.853.0
0.7
= 0.21
For Ali, z = #ofSTDEVs =
7780
10
= 0.3
John has the better G.P.A. when compared to his school because his G.P.A. is 0.21 standard
deviations below his schools mean while Alis G.P.A. is 0.3 standard deviations below his
schools mean.
Johns z-score of 0.21 is higher than Alis z-score of 0.3 . For GPA, higher values are
better, so we conclude that John has the better GPA when compared to his school.
The following lists give a few facts that provide a little more insight into what the standard deviation tells
us about the distribution of the data.
For ANY data set, no matter what the distribution of the data is:
At least 75% of the data is within 2 standard deviations of the mean.
At least 89% of the data is within 3 standard deviations of the mean.
At least 95% of the data is within 4 1/2 standard deviations of the mean.
This is known as Chebyshevs Rule.
For data having a distribution that is MOUND-SHAPED and SYMMETRIC:
Approximately 68% of the data is within 1 standard deviation of the mean.
Approximately 95% of the data is within 2 standard deviations of the mean.
More than 99% of the data is within 3 standard deviations of the mean.
This is known as the Empirical Rule.
It is important to note that this rule only applies when the shape of the distribution of the data is
mound-shaped and symmetric. We will learn more about this when studying the "Normal" or "Gaus-
sian" probability distribution in later chapters.
**With contributions from Roberta Bloom
80 CHAPTER 2. DESCRIPTIVE STATISTICS
2.10 Summary of Formulas
13
Commonly Used Symbols
The symbol means to add or to nd the sum.
n = the number of data values in a sample
N = the number of people, things, etc. in the population
x = the sample mean
s = the sample standard deviation
= the population mean
= the population standard deviation
f = frequency
x = numerical value
Commonly Used Expressions
x f = A value multiplied by its respective frequency
x = The sum of the values
x f = The sum of values multiplied by their respective frequencies
(x x) or (x ) = Deviations from the mean (how far a value is from the mean)
(x x)
2
or (x )
2
= Deviations squared
f (x x)
2
or f (x )
2
= The deviations squared and multiplied by their frequencies
Mean Formulas:
x =
x
n
or x =
f x
n
=
x
N
or =
f x
N
Standard Deviation Formulas:
s =
_
(xx)
2
n1
or s =
_
f (xx)
2
n1
=
_
(x)
2
N
or =
_
f (x)
2
N
Formulas Relating a Value, the Mean, and the Standard Deviation:
value = mean + (#ofSTDEVs)(standard deviation), where #ofSTDEVs = the number of standard devi-
ations
x = x+ (#ofSTDEVs)(s)
x = + (#ofSTDEVs)()
13
This content is available online at <http://cnx.org/content/m16310/1.9/>.
81
2.11 Practice 1: Center of the Data
14
2.11.1 Student Learning Outcomes
The student will calculate and interpret the center, spread, and location of the data.
The student will construct and interpret histograms an box plots.
2.11.2 Given
Sixty-ve randomly selected car salespersons were asked the number of cars they generally sell in one
week. Fourteen people answered that they generally sell three cars; nineteen generally sell four cars; twelve
generally sell ve cars; nine generally sell six cars; eleven generally sell seven cars.
2.11.3 Complete the Table
Data Value (# cars) Frequency Relative Frequency Cumulative Relative Frequency
Table 2.11
2.11.4 Discussion Questions
Exercise 2.11.1 (Solution on p. 107.)
What does the frequency column sum to? Why?
Exercise 2.11.2 (Solution on p. 107.)
What does the relative frequency column sum to? Why?
Exercise 2.11.3
What is the difference between relative frequency and frequency for each data value?
Exercise 2.11.4
What is the difference between cumulative relative frequency and relative frequency for each data
value?
2.11.5 Enter the Data
Enter your data into your calculator or computer.
14
This content is available online at <http://cnx.org/content/m16312/1.12/>.
82 CHAPTER 2. DESCRIPTIVE STATISTICS
2.11.6 Construct a Histogram
Determine appropriate minimum and maximum x and y values and the scaling. Sketch the histogram
below. Label the horizontal and vertical axes with words. Include numerical scaling.
2.11.7 Data Statistics
Calculate the following values:
Exercise 2.11.5 (Solution on p. 107.)
Sample mean = x =
Exercise 2.11.6 (Solution on p. 107.)
Sample standard deviation = s
x
=
Exercise 2.11.7 (Solution on p. 107.)
Sample size = n =
2.11.8 Calculations
Use the table in section 2.11.3 to calculate the following values:
Exercise 2.11.8 (Solution on p. 107.)
Median =
Exercise 2.11.9 (Solution on p. 107.)
Mode =
Exercise 2.11.10 (Solution on p. 107.)
First quartile =
Exercise 2.11.11 (Solution on p. 108.)
Second quartile = median = 50th percentile =
Exercise 2.11.12 (Solution on p. 108.)
Third quartile =
Exercise 2.11.13 (Solution on p. 108.)
Interquartile range (IQR) = _____ - _____ = _____
Exercise 2.11.14 (Solution on p. 108.)
10th percentile =
Exercise 2.11.15 (Solution on p. 108.)
70th percentile =
83
Exercise 2.11.16 (Solution on p. 108.)
Find the value that is 3 standard deviations:
a. Above the mean
b. Below the mean
2.11.9 Box Plot
Construct a box plot below. Use a ruler to measure and scale accurately.
2.11.10 Interpretation
Looking at your box plot, does it appear that the data are concentrated together, spread out evenly, or
concentrated in some areas, but not in others? How can you tell?
84 CHAPTER 2. DESCRIPTIVE STATISTICS
2.12 Practice 2: Spread of the Data
15
2.12.1 Student Learning Outcomes
The student will calculate measures of the center of the data.
The student will calculate the spread of the data.
2.12.2 Given
The population parameters below describe the full-time equivalent number of students (FTES) each year
at Lake Tahoe Community College from 1976-77 through 2004-2005. (Source: Graphically Speaking by Bill
King, LTCC Institutional Research, December 2005).
Use these values to answer the following questions:
= 1000 FTES
Median = 1014 FTES
= 474 FTES
First quartile = 528.5 FTES
Third quartile = 1447.5 FTES
n = 29 years
2.12.3 Calculate the Values
Exercise 2.12.1 (Solution on p. 108.)
A sample of 11 years is taken. About how many are expected to have a FTES of 1014 or above?
Explain how you determined your answer.
Exercise 2.12.2 (Solution on p. 108.)
75% of all years have a FTES:
a. At or below:
b. At or above:
Exercise 2.12.3 (Solution on p. 108.)
The population standard deviation =
Exercise 2.12.4 (Solution on p. 108.)
What percent of the FTES were from 528.5 to 1447.5? How do you know?
Exercise 2.12.5 (Solution on p. 108.)
What is the IQR? What does the IQR represent?
Exercise 2.12.6 (Solution on p. 108.)
How many standard deviations away from the mean is the median?
Additional Information: The population FTES for 2005-2006 through 2010-2011 was given in an updated
report. (Source: http://www.ltcc.edu/data/ResourcePDF/LTCC_FactBook_2010-11.pdf). The data are re-
ported here.
Year 2005-06 2006-07 2007-08 2008-09 2009-10 2010-11
Total FTES 1585 1690 1735 1935 2021 1890
15
This content is available online at <http://cnx.org/content/m17105/1.12/>.
85
Table 2.12
Exercise 2.12.7 (Solution on p. 108.)
Calculate the mean, median, standard deviation, rst quartile, the third quartile and the IQR.
Round to one decimal place.
Exercise 2.12.8
Construct a boxplot for the FTES for 2005-2006 through 2010-2011 and a boxplot for the FTES for
1976-1977 through 2004-2005.
Exercise 2.12.9 (Solution on p. 108.)
Compare the IQR for the FTES for 1976-77 through 2004-2005 with the IQR for the FTES for 2005-
2006 through 2010-2011. Why do you suppose the IQRs are so different?
86 CHAPTER 2. DESCRIPTIVE STATISTICS
2.13 Homework
16
Exercise 2.13.1 (Solution on p. 108.)
Twenty-ve randomly selected students were asked the number of movies they watched the pre-
vious week. The results are as follows:
# of movies Frequency Relative Frequency Cumulative Relative Frequency
0 5
1 9
2 6
3 4
4 1
Table 2.13
a. Find the sample mean x
b. Find the sample standard deviation, s
c. Construct a histogram of the data.
d. Complete the columns of the chart.
e. Find the rst quartile.
f. Find the median.
g. Find the third quartile.
h. Construct a box plot of the data.
i. What percent of the students saw fewer than three movies?
j. Find the 40th percentile.
k. Find the 90th percentile.
l. Construct a line graph of the data.
m. Construct a stem plot of the data.
Exercise 2.13.2
The median age for U.S. blacks currently is 30.9 years; for U.S. whites it is 42.3
years. ((Source: http://www.usatoday.com/news/nation/story/2012-05-17/minority-births-
census/55029100/1) )
a. Based upon this information, give two reasons why the black median age could be lower than
the white median age.
b. Does the lower median age for blacks necessarily mean that blacks die younger than whites?
Why or why not?
c. How might it be possible for blacks and whites to die at approximately the same age, but for
the median age for whites to be higher?
Exercise 2.13.3 (Solution on p. 109.)
Forty randomly selected students were asked the number of pairs of sneakers they owned. Let X
= the number of pairs of sneakers owned. The results are as follows:
16
This content is available online at <http://cnx.org/content/m16801/1.25/>.
87
X Frequency Relative Frequency Cumulative Relative Frequency
1 2
2 5
3 8
4 12
5 12
7 1
Table 2.14
a. Find the sample mean x
b. Find the sample standard deviation, s
c. Construct a histogram of the data.
d. Complete the columns of the chart.
e. Find the rst quartile.
f. Find the median.
g. Find the third quartile.
h. Construct a box plot of the data.
i. What percent of the students owned at least ve pairs?
j. Find the 40th percentile.
k. Find the 90th percentile.
l. Construct a line graph of the data
m. Construct a stem plot of the data
Exercise 2.13.4
600 adult Americans were asked by telephone poll, What do you think constitutes a middle-class
income? The results are below. Also, include left endpoint, but not the right endpoint. (Source:
Time magazine; survey by Yankelovich Partners, Inc.)
NOTE: "Not sure" answers were omitted from the results.
Salary ($) Relative Frequency
< 20,000 0.02
20,000 - 25,000 0.09
25,000 - 30,000 0.19
30,000 - 40,000 0.26
40,000 - 50,000 0.18
50,000 - 75,000 0.17
75,000 - 99,999 0.02
100,000+ 0.01
Table 2.15
a. What percent of the survey answered "not sure" ?
88 CHAPTER 2. DESCRIPTIVE STATISTICS
b. What percent think that middle-class is from $25,000 - $50,000 ?
c. Construct a histogram of the data
a. Should all bars have the same width, based on the data? Why or why not?
b. How should the <20,000 and the 100,000+ intervals be handled? Why?
d. Find the 40th and 80th percentiles
e. Construct a bar graph of the data
Exercise 2.13.5 (Solution on p. 109.)
Following are the published weights (in pounds) of all of the team members of the San Francisco
49ers from a previous year (Source: San Jose Mercury News)
177; 205; 210; 210; 232; 205; 185; 185; 178; 210; 206; 212; 184; 174; 185; 242; 188; 212; 215; 247; 241;
223; 220; 260; 245; 259; 278; 270; 280; 295; 275; 285; 290; 272; 273; 280; 285; 286; 200; 215; 185; 230;
250; 241; 190; 260; 250; 302; 265; 290; 276; 228; 265
a. Organize the data from smallest to largest value.
b. Find the median.
c. Find the rst quartile.
d. Find the third quartile.
e. Construct a box plot of the data.
f. The middle 50% of the weights are from _______ to _______.
g. If our population were all professional football players, would the above data be a sample of
weights or the population of weights? Why?
h. If our population were the San Francisco 49ers, would the above data be a sample of weights
or the population of weights? Why?
i. Assume the population was the San Francisco 49ers. Find:
i. the population mean, .
ii. the population standard deviation, .
iii. the weight that is 2 standard deviations below the mean.
iv. When Steve Young, quarterback, played football, he weighed 205 pounds. How many
standard deviations above or below the mean was he?
j. That same year, the mean weight for the Dallas Cowboys was 240.08 pounds with a standard
deviation of 44.38 pounds. Emmit Smith weighed in at 209 pounds. With respect to his team,
who was lighter, Smith or Young? How did you determine your answer?
Exercise 2.13.6
An elementary school class ran 1 mile with a mean of 11 minutes and a standard deviation of 3
minutes. Rachel, a student in the class, ran 1 mile in 8 minutes. A junior high school class ran 1
mile with a mean of 9 minutes and a standard deviation of 2 minutes. Kenji, a student in the class,
ran 1 mile in 8.5 minutes. A high school class ran 1 mile with a mean of 7 minutes and a standard
deviation of 4 minutes. Nedda, a student in the class, ran 1 mile in 8 minutes.
a. Why is Kenji considered a better runner than Nedda, even though Nedda ran faster than he?
b. Who is the fastest runner with respect to his or her class? Explain why.
Exercise 2.13.7
In a survey of 20 year olds in China, Germany and America, people were asked the number of
foreign countries they had visited in their lifetime. The following box plots display the results.
89
a. In complete sentences, describe what the shape of each box plot implies about the distribution
of the data collected.
b. Explain howit is possible that more Americans than Germans surveyed have been to over eight
foreign countries.
c. Compare the three box plots. What do they imply about the foreign travel of twenty year old
residents of the three countries when compared to each other?
Exercise 2.13.8
One hundred teachers attended a seminar on mathematical problem solving. The attitudes of
a representative sample of 12 of the teachers were measured before and after the seminar. A
positive number for change in attitude indicates that a teachers attitude toward math became
more positive. The twelve change scores are as follows:
3; 8; -1; 2; 0; 5; -3; 1; -1; 6; 5; -2
a. What is the mean change score?
b. What is the standard deviation for this population?
c. What is the median change score?
d. Find the change score that is 2.2 standard deviations below the mean.
Exercise 2.13.9 (Solution on p. 109.)
Three students were applying to the same graduate school. They came fromschools with different
grading systems. Which student had the best G.P.A. when compared to his school? Explain how
you determined your answer.
Student G.P.A. School Ave. G.P.A. School Standard Deviation
Thuy 2.7 3.2 0.8
Vichet 87 75 20
Kamala 8.6 8 0.4
Table 2.16
90 CHAPTER 2. DESCRIPTIVE STATISTICS
Exercise 2.13.10
Given the following box plot:
a. Which quarter has the smallest spread of data? What is that spread?
b. Which quarter has the largest spread of data? What is that spread?
c. Find the Inter Quartile Range (IQR).
d. Are there more data in the interval 5 - 10 or in the interval 10 - 13? How do you know this?
e. Which interval has the fewest data in it? How do you know this?
I. 0-2
II. 2-4
III. 10-12
IV. 12-13
Exercise 2.13.11
Given the following box plot:
a. Think of an example (in words) where the data might t into the above box plot. In 2-5 sen-
tences, write down the example.
b. What does it mean to have the rst and second quartiles so close together, while the second to
fourth quartiles are far apart?
Exercise 2.13.12
Santa Clara County, CA, has approximately 27,873 Japanese-Americans. Their ages are as follows.
(Source: West magazine)
Age Group Percent of Community
0-17 18.9
18-24 8.0
25-34 22.8
35-44 15.0
45-54 13.1
55-64 11.9
65+ 10.3
Table 2.17
a. Construct a histogram of the Japanese-American community in Santa Clara County, CA. The
bars will not be the same width for this example. Why not?
91
b. What percent of the community is under age 35?
c. Which box plot most resembles the information above?
Exercise 2.13.13
Suppose that three book publishers were interested in the number of ction paperbacks adult
consumers purchase per month. Each publisher conducted a survey. In the survey, each asked
adult consumers the number of ction paperbacks they had purchased the previous month. The
results are below.
Publisher A
# of books Freq. Rel. Freq.
0 10
1 12
2 16
3 12
4 8
5 6
6 2
8 2
Table 2.18
92 CHAPTER 2. DESCRIPTIVE STATISTICS
Publisher B
# of books Freq. Rel. Freq.
0 18
1 24
2 24
3 22
4 15
5 10
7 5
9 1
Table 2.19
Publisher C
# of books Freq. Rel. Freq.
0-1 20
2-3 35
4-5 12
6-7 2
8-9 1
Table 2.20
a. Find the relative frequencies for each survey. Write them in the charts.
b. Using either a graphing calculator, computer, or by hand, use the frequency column to construct
a histogram for each publishers survey. For Publishers A and B, make bar widths of 1. For
Publisher C, make bar widths of 2.
c. In complete sentences, give two reasons why the graphs for Publishers Aand B are not identical.
d. Would you have expected the graph for Publisher C to look like the other two graphs? Why or
why not?
e. Make new histograms for Publisher A and Publisher B. This time, make bar widths of 2.
f. Now, compare the graph for Publisher C to the new graphs for Publishers A and B. Are the
graphs more similar or more different? Explain your answer.
Exercise 2.13.14
Often, cruise ships conduct all on-board transactions, with the exception of gambling, on a cash-
less basis. At the end of the cruise, guests pay one bill that covers all on-board transactions. Sup-
pose that 60 single travelers and 70 couples were surveyed as to their on-board bills for a seven-day
cruise from Los Angeles to the Mexican Riviera. Below is a summary of the bills for each group.
93
Singles
Amount($) Frequency Rel. Frequency
51-100 5
101-150 10
151-200 15
201-250 15
251-300 10
301-350 5
Table 2.21
Couples
Amount($) Frequency Rel. Frequency
100-150 5
201-250 5
251-300 5
301-350 5
351-400 10
401-450 10
451-500 10
501-550 10
551-600 5
601-650 5
Table 2.22
a. Fill in the relative frequency for each group.
b. Construct a histogram for the Singles group. Scale the x-axis by $50. widths. Use relative
frequency on the y-axis.
c. Construct a histogram for the Couples group. Scale the x-axis by $50. Use relative frequency on
the y-axis.
d. Compare the two graphs:
i. List two similarities between the graphs.
ii. List two differences between the graphs.
iii. Overall, are the graphs more similar or different?
e. Construct a new graph for the Couples by hand. Since each couple is paying for two indi-
viduals, instead of scaling the x-axis by $50, scale it by $100. Use relative frequency on the
y-axis.
f. Compare the graph for the Singles with the new graph for the Couples:
i. List two similarities between the graphs.
ii. Overall, are the graphs more similar or different?
94 CHAPTER 2. DESCRIPTIVE STATISTICS
i. By scaling the Couples graph differently, how did it change the way you compared it to the
Singles?
j. Based on the graphs, do you think that individuals spend the same amount, more or less, as
singles as they do person by person in a couple? Explain why in one or two complete sen-
tences.
Exercise 2.13.15 (Solution on p. 109.)
Refer to the following histograms and box plot. Determine which of the following are true and
which are false. Explain your solution to each part in complete sentences.
a. The medians for all three graphs are the same.
b. We cannot determine if any of the means for the three graphs is different.
c. The standard deviation for (b) is larger than the standard deviation for (a).
d. We cannot determine if any of the third quartiles for the three graphs is different.
Exercise 2.13.16
Refer to the following box plots.
95
a. In complete sentences, explain why each statement is false.
i. Data 1 has more data values above 2 than Data 2 has above 2.
ii. The data sets cannot have the same mode.
iii. For Data 1, there are more data values below 4 than there are above 4.
b. For which group, Data 1 or Data 2, is the value of 7 more likely to be an outlier? Explain why
in complete sentences
Exercise 2.13.17 (Solution on p. 110.)
In a recent issue of the IEEE Spectrum, 84 engineering conferences were announced. Four con-
ferences lasted two days. Thirty-six lasted three days. Eighteen lasted four days. Nineteen lasted
ve days. Four lasted six days. One lasted seven days. One lasted eight days. One lasted nine
days. Let X = the length (in days) of an engineering conference.
a. Organize the data in a chart.
b. Find the median, the rst quartile, and the third quartile.
c. Find the 65th percentile.
d. Find the 10th percentile.
e. Construct a box plot of the data.
f. The middle 50% of the conferences last from _______ days to _______ days.
g. Calculate the sample mean of days of engineering conferences.
h. Calculate the sample standard deviation of days of engineering conferences.
i. Find the mode.
j. If you were planning an engineering conference, which would you choose as the length of the
conference: mean; median; or mode? Explain why you made that choice.
k. Give two reasons why you think that 3 - 5 days seem to be popular lengths of engineering
conferences.
Exercise 2.13.18
A survey of enrollment at 35 community colleges across the United States yielded the following
gures (source: Microsoft Bookshelf ):
6414; 1550; 2109; 9350; 21828; 4300; 5944; 5722; 2825; 2044; 5481; 5200; 5853; 2750; 10012; 6357;
27000; 9414; 7681; 3200; 17500; 9200; 7380; 18314; 6557; 13713; 17768; 7493; 2771; 2861; 1263; 7285;
28165; 5080; 11622
a. Organize the data into a chart with ve intervals of equal width. Label the two columns "En-
rollment" and "Frequency."
b. Construct a histogram of the data.
96 CHAPTER 2. DESCRIPTIVE STATISTICS
c. If you were to build a new community college, which piece of information would be more
valuable: the mode or the mean?
d. Calculate the sample mean.
e. Calculate the sample standard deviation.
f. A school with an enrollment of 8000 would be how many standard deviations away from the
mean?
Exercise 2.13.19 (Solution on p. 110.)
The median age of the U.S. population in 1980 was 30.0 years. In 1991, the median age was 33.1
years. (Source: Bureau of the Census)
a. What does it mean for the median age to rise?
b. Give two reasons why the median age could rise.
c. For the median age to rise, is the actual number of children less in 1991 than it was in 1980?
Why or why not?
Exercise 2.13.20
A survey was conducted of 130 purchasers of new BMW 3 series cars, 130 purchasers of new
BMW 5 series cars, and 130 purchasers of new BMW 7 series cars. In it, people were asked the age
they were when they purchased their car. The following box plots display the results.
a. In complete sentences, describe what the shape of each box plot implies about the distribution
of the data collected for that car series.
b. Which group is most likely to have an outlier? Explain how you determined that.
c. Compare the three box plots. What do they imply about the age of purchasing a BMW from the
series when compared to each other?
d. Look at the BMW 5 series. Which quarter has the smallest spread of data? What is that spread?
e. Look at the BMW 5 series. Which quarter has the largest spread of data? What is that spread?
f. Look at the BMW 5 series. Estimate the Inter Quartile Range (IQR).
g. Look at the BMW 5 series. Are there more data in the interval 31-38 or in the interval 45-55?
How do you know this?
h. Look at the BMW 5 series. Which interval has the fewest data in it? How do you know this?
i. 31-35
ii. 38-41
iii. 41-64
97
Exercise 2.13.21 (Solution on p. 110.)
The following box plot shows the U.S. population for 1990, the latest available year. (Source:
Bureau of the Census, 1990 Census)
a. Are there fewer or more children (age 17 and under) than senior citizens (age 65 and over)?
How do you know?
b. 12.6% are age 65 and over. Approximately what percent of the population are of working age
adults (above age 17 to age 65)?
Exercise 2.13.22
Javier and Ercilia are supervisors at a shopping mall. Each was given the task of estimating the
mean distance that shoppers live from the mall. They each randomly surveyed 100 shoppers. The
samples yielded the following information:
Javier Ercilla
x 6.0 miles 6.0 miles
s 4.0 miles 7.0 miles
Table 2.23
a. How can you determine which survey was correct ?
b. Explain what the difference in the results of the surveys implies about the data.
c. If the two histograms depict the distribution of values for each supervisor, which one depicts
Ercilias sample? How do you know?
Figure 2.2
d. If the two box plots depict the distribution of values for each supervisor, which one depicts
Ercilias sample? How do you know?
98 CHAPTER 2. DESCRIPTIVE STATISTICS
Figure 2.3
Exercise 2.13.23 (Solution on p. 110.)
Student grades on a chemistry exam were:
77, 78, 76, 81, 86, 51, 79, 82, 84, 99
a. Construct a stem-and-leaf plot of the data.
b. Are there any potential outliers? If so, which scores are they? Why do you consider them
outliers?
2.13.1 Try these multiple choice questions (Exercises 24 - 30).
The next three questions refer to the following information. We are interested in the number of years
students in a particular elementary statistics class have lived in California. The information in the following
table is from the entire section.
Number of years Frequency
7 1
14 3
15 1
18 1
19 4
20 3
22 1
23 1
26 1
40 2
42 2
Total = 20
Table 2.24
Exercise 2.13.24 (Solution on p. 110.)
What is the IQR?
A. 8
99
B. 11
C. 15
D. 35
Exercise 2.13.25 (Solution on p. 110.)
What is the mode?
A. 19
B. 19.5
C. 14 and 20
D. 22.65
Exercise 2.13.26 (Solution on p. 110.)
Is this a sample or the entire population?
A. sample
B. entire population
C. neither
The next two questions refer to the following table. X = the number of days per week that 100 clients use
a particular exercise facility.
x Frequency
0 3
1 12
2 33
3 28
4 11
5 9
6 4
Table 2.25
Exercise 2.13.27 (Solution on p. 110.)
The 80th percentile is:
A. 5
B. 80
C. 3
D. 4
Exercise 2.13.28 (Solution on p. 110.)
The number that is 1.5 standard deviations BELOW the mean is approximately:
A. 0.7
B. 4.8
C. -2.8
D. Cannot be determined
100 CHAPTER 2. DESCRIPTIVE STATISTICS
The next two questions refer to the following histogram. Suppose one hundred eleven people who
shopped in a special T-shirt store were asked the number of T-shirts they own costing more than $19 each.
Exercise 2.13.29 (Solution on p. 110.)
The percent of people that own at most three (3) T-shirts costing more than $19 each is approxi-
mately:
A. 21
B. 59
C. 41
D. Cannot be determined
Exercise 2.13.30 (Solution on p. 110.)
If the data were collected by asking the rst 111 people who entered the store, then the type of
sampling is:
A. cluster
B. simple random
C. stratied
D. convenience
Exercise 2.13.31 (Solution on p. 110.)
Below are the 2010 obesity rates by U.S. states and Washington, DC.(Source:
http://www.cdc.gov/obesity/data/adult.html) )
101
State Percent (%) State Percent (%)
Alabama 32.2 Montana 23.0
Alaska 24.5 Nebraska 26.9
Arizona 24.3 Nevada 22.4
Arkansas 30.1 New Hampshire 25.0
California 24.0 New Jersey 23.8
Colorado 21.0 New Mexico 25.1
Connecticut 22.5 New York 23.9
Delaware 28.0 North Carolina 27.8
Washington, DC 22.2 North Dakota 27.2
Florida 26.6 Ohio 29.2
Georgia 29.6 Oklahoma 30.4
Hawaii 22.7 Oregon 26.8
Idaho 26.5 Pennsylvania 28.6
Illinois 28.2 Rhode Island 25.5
Indiana 29.6 South Carolina 31.5
Iowa 28.4 South Dakota 27.3
Kansas 29.4 Tennessee 30.8
Kentucky 31.3 Texas 31.0
Louisiana 31.0 Utah 22.5
Maine 26.8 Vermont 23.2
Maryland 27.1 Virginia 26.0
Massachusetts 23.0 Washington 25.5
Michigan 30.9 West Virginia 32.5
Minnesota 24.8 Wisconsin 26.3
Mississippi 34.0 Wyoming 25.1
Missouri 30.5
Table 2.26
a.. Construct a bar graph of obesity rates of your state and the four states closest to your state.
Hint: Label the x-axis with the states.
b.. Use a randomnumber generator to randomly pick 8 states. Construct a bar graph of the obesity
rates of those 8 states.
c.. Construct a bar graph for all the states beginning with the letter "A."
d.. Construct a bar graph for all the states beginning with the letter "M."
Exercise 2.13.32 (Solution on p. 111.)
A music school has budgeted to purchase 3 musical instruments. They plan to purchase a piano
costing $3000, a guitar costing $550, and a drum set costing $600. The mean cost for a piano is
102 CHAPTER 2. DESCRIPTIVE STATISTICS
$4,000 with a standard deviation of $2,500. The mean cost for a guitar is $500 with a standard
deviation of $200. The mean cost for drums is $700 with a standard deviation of $100. Which cost
is the lowest, when compared to other instruments of the same type? Which cost is the highest
when compared to other instruments of the same type. Justify your answer numerically.
Exercise 2.13.33 (Solution on p. 111.)
Suppose that a publisher conducted a survey asking adult consumers the number of ction pa-
perback books they had purchased in the previous month. The results are summarized in the table
below. (Note that this is the data presented for publisher B in homework exercise 13).
Publisher B
# of books Freq. Rel. Freq.
0 18
1 24
2 24
3 22
4 15
5 10
7 5
9 1
Table 2.27
a. Are there any outliers in the data? Use an appropriate numerical test involving the IQR to
identify outliers, if any, and clearly state your conclusion.
b. If a data value is identied as an outlier, what should be done about it?
c. Are any data values further than 2 standard deviations away from the mean? In some situ-
ations, statisticians may use this criteria to identify data values that are unusual, compared
to the other data values. (Note that this criteria is most appropriate to use for data that is
mound-shaped and symmetric, rather than for skewed data.)
d. Do parts (a) and (c) of this problem give the same answer?
e. Examine the shape of the data. Which part, (a) or (c), of this question gives a more appropri-
ate result for this data?
f. Based on the shape of the data which is the most appropriate measure of center for this data:
mean, median or mode?
**Exercises 32 and 33 contributed by Roberta Bloom
103
2.14 Lab: Descriptive Statistics
17
Class Time:
Names:
2.14.1 Student Learning Outcomes
The student will construct a histogram and a box plot.
The student will calculate univariate statistics.
The student will examine the graphs to interpret what the data implies.
2.14.2 Collect the Data
Record the number of pairs of shoes you own:
1. Randomly survey 30 classmates. Record their values.
Survey Results
_____ _____ _____ _____ _____
_____ _____ _____ _____ _____
_____ _____ _____ _____ _____
_____ _____ _____ _____ _____
_____ _____ _____ _____ _____
_____ _____ _____ _____ _____
Table 2.28
2. Construct a histogram. Make 5-6 intervals. Sketch the graph using a ruler and pencil. Scale the axes.
17
This content is available online at <http://cnx.org/content/m16299/1.13/>.
104 CHAPTER 2. DESCRIPTIVE STATISTICS
Figure 2.4
3. Calculate the following:
x =
s =
4. Are the data discrete or continuous? How do you know?
5. Describe the shape of the histogram. Use complete sentences.
6. Are there any potential outliers? Which value(s) is (are) it (they)? Use a formula to check the end
values to determine if they are potential outliers.
2.14.3 Analyze the Data
1. Determine the following:
Minimum value =
Median =
Maximum value =
First quartile =
Third quartile =
IQR =
2. Construct a box plot of data
3. What does the shape of the box plot imply about the concentration of data? Use complete sentences.
4. Using the box plot, how can you determine if there are potential outliers?
5. How does the standard deviation help you to determine concentration of the data and whether or not
there are potential outliers?
6. What does the IQR represent in this problem?
7. Show your work to nd the value that is 1.5 standard deviations:
a. Above the mean:
b. Below the mean:
105
Solutions to Exercises in Chapter 2
Solution to Example 2.2, Problem (p. 53)
The value 12.3 may be an outlier. Values appear to concentrate at 3 and 4 kilometers.
Stem Leaf
1 1 5
2 3 5 7
3 2 3 3 5 8
4 0 2 5 5 7 8
5 5 6
6 5 7
7
8
9
10
11
12 3
Table 2.29
Solution to Example 2.7, Problem (p. 58)
3.5 to 4.5
4.5 to 5.5
6
5.5 to 6.5
Solution to Example 2.9, Problem (p. 62)
First Data Set
Xmin = 32
Q1 = 56
M = 74.5
Q3 = 82.5
Xmax = 99
Second Data Set
Xmin = 25.5
Q1 = 78
M = 81
Q3 = 89
Xmax = 98
106 CHAPTER 2. DESCRIPTIVE STATISTICS
Solution to Example 2.11, Problem (p. 63)
For the IQRs, see the answer to the test scores example (Solution to Example 2.9: p. 105). The rst data set
has the larger IQR, so the scores between Q3 and Q1 (middle 50%) for the rst data set are more spread out
and not clustered about the median.
First Data Set

_
3
2
_
(IQR) =
_
3
2
_
(26.5) = 39.75
Xmax Q3 = 99 82.5 = 16.5
Q1 Xmin = 56 32 = 24
_
3
2
_
(IQR) = 39.75 is larger than 16.5 and larger than 24, so the rst set has no outliers.
Second Data Set

_
3
2
_
(IQR) =
_
3
2
_
(11) = 16.5
Xmax Q3 = 98 89 = 9
Q1 Xmin = 78 25.5 = 52.5
_
3
2
_
(IQR) = 16.5 is larger than 9 but smaller than 52.5, so for the second set 45 and 25.5 are outliers.
To nd the percentiles, create a frequency, relative frequency, and cumulative relative frequency chart (see
"Frequency" from the Sampling and Data Chapter (Section 1.9)). Get the percentiles from that chart.
First Data Set
30th %ile (between the 6th and 7th values) =
(56 + 59)
2
= 57.5
80th %ile (between the 16th and 17th values) =
(84 + 84.5)
2
= 84.25
Second Data Set
30th %ile (7th value) = 78
80th %ile (18th value) = 90
30% of the data falls below the 30th %ile, and 20% falls above the 80th %ile.
Solution to Example 2.13, Problem (p. 64)
1.
(8 + 9)
2
= 8.5
2. 9
3. 6
4. First Quartile = 25th %ile
Solution to Exercise 2.6.1 (p. 66)
a. For runners in a race it is more desirable to have a low percentile for nish time. A low percentile means
a short time, which is faster.
107
b. INTERPRETATION: 20% of runners nished the race in 5.2 minutes or less. 80% of runners nished the
race in 5.2 minutes or longer.
c. He is among the slowest cyclists (90% of cyclists were faster than him.) INTERPRETATION: 90% of
cyclists had a nish time of 1 hour, 12 minutes or less.Only 10% of cyclists had a nish time of 1 hour,
12 minutes or longer
Solution to Exercise 2.6.2 (p. 66)
a. For runners in a race it is more desirable to have a high percentile for speed. A high percentile means a
higher speed, which is faster.
b. INTERPRETATION: 40% of runners ran at speeds of 7.5 miles per hour or less (slower). 60% of runners
ran at speeds of 7.5 miles per hour or more (faster).
Solution to Exercise 2.6.3 (p. 66)
On an exam you would prefer a high percentile; higher percentiles correspond to higher grades on the
exam.
Solution to Exercise 2.6.4 (p. 66)
When waiting in line at the DMV, the 85th percentile would be a long wait time compared to the other
people waiting. 85% of people had shorter wait times than you did. In this context, you would prefer a
wait time corresponding to a lower percentile. INTERPRETATION: 85% of people at the DMV waited 32
minutes or less. 15% of people at the DMV waited 32 minutes or longer.
Solution to Exercise 2.6.5 (p. 66)
Li should be pleased. Her salary is relatively high compared to other recent college grads. 78% of recent
college graduates earn less than Li does. 22% of recent college graduates earn more than Li does.
Solution to Exercise 2.6.6 (p. 66)
The manufacturer and the consumer would be upset. This is a large repair cost for the damages, compared
to the other cars in the sample. INTERPRETATION: 90% of the crash tested cars had damage repair costs
of $1700 or less; only 10% had damage repair costs of $1700 or more.
Solution to Exercise 2.6.7 (p. 66)
a. The top 12% of students are those who are at or above the 88th percentile of admissions index scores.
b. The top 4%of students GPAs are at or above the 96th percentile, making the top 4%of students "eligible
in the local context".
Solution to Exercise 2.6.8 (p. 67)
You can afford 34% of houses. 66% of the houses are too expensive for your budget. INTERPRETATION:
34% of houses cost $240,000 or less. 66% of houses cost $240,000 or more.
Solutions to Practice 1: Center of the Data
Solution to Exercise 2.11.1 (p. 81)
65
Solution to Exercise 2.11.2 (p. 81)
1
Solution to Exercise 2.11.5 (p. 82)
4.75
Solution to Exercise 2.11.6 (p. 82)
1.39
Solution to Exercise 2.11.7 (p. 82)
65
Solution to Exercise 2.11.8 (p. 82)
4
Solution to Exercise 2.11.9 (p. 82)
4
108 CHAPTER 2. DESCRIPTIVE STATISTICS
Solution to Exercise 2.11.10 (p. 82)
4
Solution to Exercise 2.11.11 (p. 82)
4
Solution to Exercise 2.11.12 (p. 82)
6
Solution to Exercise 2.11.13 (p. 82)
6 4 = 2
Solution to Exercise 2.11.14 (p. 82)
3
Solution to Exercise 2.11.15 (p. 82)
6
Solution to Exercise 2.11.16 (p. 83)
a. 8.93
b. 0.58
Solutions to Practice 2: Spread of the Data
Solution to Exercise 2.12.1 (p. 84)
6
Solution to Exercise 2.12.2 (p. 84)
a. 1447.5
b. 528.5
Solution to Exercise 2.12.3 (p. 84)
474 FTES
Solution to Exercise 2.12.4 (p. 84)
50%
Solution to Exercise 2.12.5 (p. 84)
919
Solution to Exercise 2.12.6 (p. 84)
0.03
Solution to Exercise 2.12.7 (p. 85)
mean = 1809.3
median = 1812.5
standard deviation = 151.2
First quartile = 1690
Third quartile = 1935
IQR = 245
Solution to Exercise 2.12.9 (p. 85)
Hint: Think about the number of years covered by each time period and what happened to higher educa-
tion during those periods.
Solutions to Homework
Solution to Exercise 2.13.1 (p. 86)
a. 1.48
b. 1.12
e. 1
f. 1
109
g. 2
h.
i. 80%
j. 1
k. 3
Solution to Exercise 2.13.3 (p. 86)
a. 3.78
b. 1.29
e. 3
f. 4
g. 5
h.
i. 32.5%
j. 4
k. 5
Solution to Exercise 2.13.5 (p. 88)
b. 241
c. 205.5
d. 272.5
e.
f. 205.5, 272.5
g. sample
h. population
i. i. 236.34
ii. 37.50
iii. 161.34
iv. 0.84 std. dev. below the mean
j. Young
Solution to Exercise 2.13.9 (p. 89)
Kamala
Solution to Exercise 2.13.15 (p. 94)
a. True
110 CHAPTER 2. DESCRIPTIVE STATISTICS
b. True
c. True
d. False
Solution to Exercise 2.13.17 (p. 95)
b. 4,3,5
c. 4
d. 3
e.
f. 3,5
g. 3.94
h. 1.28
i. 3
j. mode
Solution to Exercise 2.13.19 (p. 96)
c. Maybe
Solution to Exercise 2.13.21 (p. 97)
a. more children
b. 62.4%
Solution to Exercise 2.13.23 (p. 98)
b. 51,99
Solution to Exercise 2.13.24 (p. 98)
A
Solution to Exercise 2.13.25 (p. 99)
A
Solution to Exercise 2.13.26 (p. 99)
B
Solution to Exercise 2.13.27 (p. 99)
D
Solution to Exercise 2.13.28 (p. 99)
A
Solution to Exercise 2.13.29 (p. 100)
C
Solution to Exercise 2.13.30 (p. 100)
D
Solution to Exercise 2.13.31 (p. 100)
Example solution for b using the random number generator for the Ti-84 Plus to generate a simple random
sample of 8 states. Instructions are below.
Number the entries in the table 1 - 51 (Includes Washington, DC; Numbered vertically)
Press MATH
Arrow over to PRB
Press 5:randInt(
111
Enter 51,1,8)
Eight numbers are generated (use the right arrow key to scroll through the numbers). The
numbers correspond to the numbered states (for this example: {47 21 9 23 51 13 25 4}. If
any numbers are repeated, generate a different number by using 5:randInt(51,1)). Here, the
states (and Washington DC) are {Arkansas, Washington DC, Idaho, Maryland, Michigan, Missis-
sippi, Virginia, Wyoming}. Corresponding percents are {28.7 21.8 24.5 26 28.9 32.8 25 24.6}.
Solution to Exercise 2.13.32 (p. 101)
For pianos, the cost of the piano is 0.4 standard deviations BELOW the mean. For guitars, the cost of the
guitar is 0.25 standard deviations ABOVE the mean. For drums, the cost of the drum set is 1.0 standard
deviations BELOW the mean. Of the three, the drums cost the lowest in comparison to the cost of other
instruments of the same type. The guitar cost the most in comparison to the cost of other instruments of the
same type.
Solution to Exercise 2.13.33 (p. 102)
IQR = 4 1 = 3 ; Q1 1.5*IQR = 1 1.5(3) = -3.5 ; Q3 + 1.5*IQR = 4 + 1.5(3) = 8.5 ;The data value of 9 is
larger than 8.5. The purchase of 9 books in one month is an outlier.
The outlier should be investigated to see if there is an error or some other problem in the data; then a
decision whether to include or exclude it should be made based on the particular situation. If it was
a correct value then the data value should remain in the data set. If there is a problem with this data
value, then it should be corrected or removed from the data. For example: If the data was recorded
incorrectly (perhaps a 9 was miscoded and the correct value was 6) then the data should be corrected.
If it was an error but the correct value is not known it should be removed from the data set.
xbar 2s = 2.45 2*1.88 = -1.31 ; xbar + 2s = 2.45 + 2*1.88 = 6.21 ; Using this method, the ve data values
of 7 books purchased and the one data value of 9 books purchased would be considered unusual.
No: part (a) identies only the value of 9 to be an outlier but part (c) identies both 7 and 9.
The data is skewed (to the right). It would be more appropriate to use the method involving the IQR
in part (a), identifying only the one value of 9 books purchased as an outlier. Note that part (c) remarks
that identifying unusual data values by using the criteria of being further than 2 standard deviations
away from the mean is most appropriate when the data are mound-shaped and symmetric.
The data are skewed to the right. For skewed data it is more appropriate to use the median as a
measure of center.
112 CHAPTER 2. DESCRIPTIVE STATISTICS
Chapter 12
Linear Regression and Correlation
12.1 Linear Regression and Correlation
1
12.1.1 Student Learning Outcomes
By the end of this chapter, the student should be able to:
Discuss basic ideas of linear regression and correlation.
Create and interpret a line of best t.
Calculate and interpret the correlation coefcient.
Calculate and interpret outliers.
12.1.2 Introduction
Professionals often want to know how two or more numeric variables are related. For example, is there a
relationship between the grade on the second math exam a student takes and the grade on the nal exam?
If there is a relationship, what is it and how strong is the relationship?
In another example, your income may be determined by your education, your profession, your years of
experience, and your ability. The amount you pay a repair person for labor is often determined by an initial
amount plus an hourly fee. These are all examples in which regression can be used.
The type of data described in the examples is bivariate data - "bi" for two variables. In reality, statisticians
use multivariate data, meaning many variables.
In this chapter, you will be studying the simplest form of regression, "linear regression" with one indepen-
dent variable (x). This involves data that ts a line in two dimensions. You will also study correlation which
measures how strong the relationship is.
12.2 Linear Equations
2
Linear regression for two variables is based on a linear equation with one independent variable. It has the
form:
y = a +bx (12.1)
1
This content is available online at <http://cnx.org/content/m17089/1.6/>.
2
This content is available online at <http://cnx.org/content/m17086/1.4/>.
515
516 CHAPTER 12. LINEAR REGRESSION AND CORRELATION
where a and b are constant numbers.
x is the independent variable, and y is the dependent variable. Typically, you choose a value to substitute
for the independent variable and then solve for the dependent variable.
Example 12.1
The following examples are linear equations.
y = 3 +2x (12.2)
y = 0.01 +1.2x (12.3)
The graph of a linear equation of the form y = a +bx is a straight line. Any line that is not vertical can be
described by this equation.
Example 12.2
Figure 12.1: Graph of the equation y = 1 +2x.
Linear equations of this form occur in applications of life sciences, social sciences, psychology, business,
economics, physical sciences, mathematics, and other areas.
Example 12.3
Aarons Word Processing Service (AWPS) does word processing. Its rate is $32 per hour plus a
$31.50 one-time charge. The total cost to a customer depends on the number of hours it takes to
do the word processing job.
Problem
Find the equation that expresses the total cost in terms of the number of hours required to nish
the word processing job.
Solution
Let x = the number of hours it takes to get the job done.
Let y = the total cost to the customer.
The $31.50 is a xed cost. If it takes x hours to complete the job, then (32) (x) is the cost of the
word processing only. The total cost is:
517
y = 31.50 +32x
12.3 Slope and Y-Intercept of a Linear Equation
3
For the linear equation y = a + bx, b = slope and a = y-intercept.
From algebra recall that the slope is a number that describes the steepness of a line and the y-intercept is
the y coordinate of the point (0, a) where the line crosses the y-axis.
(a) (b) (c)
Figure 12.2: Three possible graphs of y = a + bx. (a) If b > 0, the line slopes upward to the right. (b) If
b = 0, the line is horizontal. (c) If b < 0, the line slopes downward to the right.
Example 12.4
Svetlana tutors to make extra money for college. For each tutoring session, she charges a one
time fee of $25 plus $15 per hour of tutoring. A linear equation that expresses the total amount of
money Svetlana earns for each session she tutors is y = 25 +15x.
Problem
What are the independent and dependent variables? What is the y-intercept and what is the
slope? Interpret them using complete sentences.
Solution
The independent variable (x) is the number of hours Svetlana tutors each session. The dependent
variable (y) is the amount, in dollars, Svetlana earns for each session.
The y-intercept is 25 (a = 25). At the start of the tutoring session, Svetlana charges a one-time fee
of $25 (this is when x = 0). The slope is 15 (b = 15). For each session, Svetlana earns $15 for each
hour she tutors.
3
This content is available online at <http://cnx.org/content/m17083/1.5/>.
518 CHAPTER 12. LINEAR REGRESSION AND CORRELATION
12.4 Scatter Plots
4
Before we take up the discussion of linear regression and correlation, we need to examine a way to display
the relation between two variables x and y. The most common and easiest way is a scatter plot. The
following example illustrates a scatter plot.
Example 12.5
From an article in the Wall Street Journal : In Europe and Asia, m-commerce is popular. M-
commerce users have special mobile phones that work like electronic wallets as well as provide
phone and Internet services. Users can do everything from paying for parking to buying a TV set
or soda from a machine to banking to checking sports scores on the Internet. For the years 2000
through 2004, was there a relationship between the year and the number of m-commerce users?
Construct a scatter plot. Let x = the year and let y = the number of m-commerce users, in millions.
x (year) y (# of users)
2000 0.5
2002 20.0
2003 33.0
2004 47.0
(a)
(b)
Figure 12.3: (a) Table showing the number of m-commerce users (in millions) by year. (b) Scatter plot
showing the number of m-commerce users (in millions) by year.
A scatter plot shows the direction and strength of a relationship between the variables. A clear direction
happens when there is either:
High values of one variable occurring with high values of the other variable or low values of one
variable occurring with low values of the other variable.
High values of one variable occurring with low values of the other variable.
You can determine the strength of the relationship by looking at the scatter plot and seeing how close the
points are to a line, a power function, an exponential function, or to some other type of function.
When you look at a scatterplot, you want to notice the overall pattern and any deviations from the pattern.
The following scatterplot examples illustrate these concepts.
4
This content is available online at <http://cnx.org/content/m17082/1.8/>.
519
(a) Positive Linear Pattern (Strong) (b) Linear Pattern w/ One Deviation
Figure 12.4
(a) Negative Linear Pattern (Strong) (b) Negative Linear Pattern (Weak)
Figure 12.5
(a) Exponential Growth Pattern (b) No Pattern
Figure 12.6
In this chapter, we are interested in scatter plots that show a linear pattern. Linear patterns are quite com-
mon. The linear relationship is strong if the points are close to a straight line. If we think that the points
show a linear relationship, we would like to draw a line on the scatter plot. This line can be calculated
through a process called linear regression. However, we only calculate a regression line if one of the vari-
ables helps to explain or predict the other variable. If x is the independent variable and y the dependent
variable, then we can use a regression line to predict y for a given value of x.
520 CHAPTER 12. LINEAR REGRESSION AND CORRELATION
12.5 The Regression Equation
5
Data rarely t a straight line exactly. Usually, you must be satised with rough predictions. Typically, you
have a set of data whose scatter plot appears to "t" a straight line. This is called a Line of Best Fit or Least
Squares Line.
12.5.1 Optional Collaborative Classroom Activity
If you know a persons pinky (smallest) nger length, do you think you could predict that persons height?
Collect data from your class (pinky nger length, in inches). The independent variable, x, is pinky nger
length and the dependent variable, y, is height.
For each set of data, plot the points on graph paper. Make your graph big enough and use a ruler. Then
"by eye" draw a line that appears to "t" the data. For your line, pick two convenient points and use them
to nd the slope of the line. Find the y-intercept of the line by extending your lines so they cross the y-axis.
Using the slopes and the y-intercepts, write your equation of "best t". Do you think everyone will have
the same equation? Why or why not?
Using your equation, what is the predicted height for a pinky length of 2.5 inches?
Example 12.6
A random sample of 11 statistics students produced the following data where x is the third exam
score, out of 80, and y is the nal exam score, out of 200. Can you predict the nal exam score of a
random student if you know the third exam score?
5
This content is available online at <http://cnx.org/content/m17090/1.15/>.
521
x (third exam score) y (nal exam score)
65 175
67 133
71 185
71 163
66 126
75 198
67 153
70 163
71 159
69 151
69 159
(a)
(b)
Figure 12.7: (a) Table showing the scores on the nal exambased on scores fromthe third exam. (b) Scatter
plot showing the scores on the nal exam based on scores from the third exam.
The third exam score, x, is the independent variable and the nal exam score, y, is the dependent variable.
We will plot a regression line that best "ts" the data. If each of you were to t a line "by eye", you would
draw different lines. We can use what is called a least-squares regression line to obtain the best t line.
Consider the following diagram. Each point of data is of the the form (x, y)and each point of the line of
best t using least-squares linear regression has the form
_
x,
^
y
_
.
The
^
y
is read "y hat" and is the estimated value of y. It is the value of y obtained using the regression line.
It is not generally equal to y from data.
522 CHAPTER 12. LINEAR REGRESSION AND CORRELATION
Figure 12.8
The term y
0

^
y
0
=
0
is called the "error" or residual. It is not an error in the sense of a mistake. The
absolute value of a residual measures the vertical distance between the actual value of y and the estimated
value of y. In other words, it measures the vertical distance between the actual data point and the predicted
point on the line.
If the observed data point lies above the line, the residual is positive, and the line underestimates the
actual data value for y. If the observed data point lies below the line, the residual is negative, and the line
overestimates that actual data value for y.
In the diagram above, y
0

^
y
0
=
0
is the residual for the point shown. Here the point lies above the line
and the residual is positive.
= the Greek letter epsilon
For each data point, you can calculate the residuals or errors, y
i

^
y
i
=
i
for i = 1, 2, 3, ..., 11.
Each || is a vertical distance.
For the example about the third exam scores and the nal exam scores for the 11 statistics students, there
are 11 data points. Therefore, there are 11 values. If you square each and add, you get
(
1
)
2
+ (
2
)
2
+... + (
11
)
2
=
11

i = 1

2
This is called the Sum of Squared Errors (SSE).
Using calculus, you can determine the values of a and b that make the SSE a minimum. When you make
the SSE a minimum, you have determined the points that are on the line of best t. It turns out that the line
of best t has the equation:
^
y
= a +bx (12.4)
523
where a = y b x and b =
(xx)(yy)
(xx)
2
.
x and y are the sample means of the x values and the y values, respectively. The best t line always passes
through the point (x, y).
The slope b can be written as b = r
_
s
y
s
x
_
where s
y
= the standard deviation of the y values and s
x
= the
standard deviation of the x values. r is the correlation coefcient which is discussed in the next section.
Least Squares Criteria for Best Fit
The process of tting the best t line is called linear regression. The idea behind nding the best t line is
based on the assumption that the data are scattered about a straight line. The criteria for the best t line is
that the sum of the squared errors (SSE) is minimized, that is made as small as possible. Any other line you
might choose would have a higher SSE than the best t line. This best t line is called the least squares
regression line .
NOTE: Computer spreadsheets, statistical software, and many calculators can quickly calculate the
best t line and create the graphs. The calculations tend to be tedious if done by hand. Instructions
to use the TI-83, TI-83+, and TI-84+ calculators to nd the best t line and create a scatterplot are
shown at the end of this section.
THIRD EXAM vs FINAL EXAM EXAMPLE:
The graph of the line of best t for the third exam/nal exam example is shown below:
Figure 12.9
The least squares regression line (best t line) for the third exam/nal exam example has the equation:
^
y
= 173.51 +4.83x (12.5)
524 CHAPTER 12. LINEAR REGRESSION AND CORRELATION
NOTE:
Remember, it is always important to plot a scatter diagram rst. If the scatter plot indicates that
there is a linear relationship between the variables, then it is reasonable to use a best t line
to make predictions for y given x within the domain of x-values in the sample data, but not
necessarily for x-values outside that domain.
You could use the line to predict the nal exam score for a student who earned a grade of 73 on
the third exam.
You should NOT use the line to predict the nal exam score for a student who earned a grade of
50 on the third exam, because 50 is not within the domain of the x-values in the sample data,
which are between 65 and 75.
UNDERSTANDING SLOPE
The slope of the line, b, describes how changes in the variables are related. It is important to interpret
the slope of the line in the context of the situation represented by the data. You should be able to write a
sentence interpreting the slope in plain English.
INTERPRETATION OF THE SLOPE: The slope of the best t line tells us how the dependent variable (y)
changes for every one unit increase in the independent (x) variable, on average.
THIRD EXAM vs FINAL EXAM EXAMPLE
Slope: The slope of the line is b = 4.83.
Interpretation: For a one point increase in the score on the third exam, the nal exam score increases by
4.83 points, on average.
12.5.2 Using the TI-83+ and TI-84+ Calculators
Using the Linear Regression T Test: LinRegTTest
Step 1. In the STAT list editor, enter the X data in list L1 and the Y data in list L2, paired so that the corre-
sponding (x,y) values are next to each other in the lists. (If a particular pair of values is repeated, enter
it as many times as it appears in the data.)
Step 2. On the STAT TESTS menu, scroll down with the cursor to select the LinRegTTest. (Be careful to select
LinRegTTest as some calculators may also have a different item called LinRegTInt.)
Step 3. On the LinRegTTest input screen enter: Xlist: L1 ; Ylist: L2 ; Freq: 1
Step 4. On the next line, at the prompt or , highlight "= 0" and press ENTER
Step 5. Leave the line for "RegEq:" blank
Step 6. Highlight Calculate and press ENTER.
525
Figure 12.10
The output screen contains a lot of information. For now we will focus on a few items from the output, and
will return later to the other items.
The second line says y=a+bx. Scroll down to nd the values a=-173.513, and b=4.8273 ; the equation of the
best t line is
^
y
= 173.51 +4.83x
The two items at the bottom are r
2
= .43969 and r=.663. For now, just note where to nd these values; we
will discuss them in the next two sections.
Graphing the Scatterplot and Regression Line
Step 1. We are assuming your X data is already entered in list L1 and your Y data is in list L2
Step 2. Press 2nd STATPLOT ENTER to use Plot 1
Step 3. On the input screen for PLOT 1, highlight On and press ENTER
Step 4. For TYPE: highlight the very rst icon which is the scatterplot and press ENTER
Step 5. Indicate Xlist: L1 and Ylist: L2
Step 6. For Mark: it does not matter which symbol you highlight.
Step 7. Press the ZOOM key and then the number 9 (for menu item "ZoomStat") ; the calculator will t the
window to the data
Step 8. To graph the best t line, press the "Y=" key and type the equation -173.5+4.83X into equation Y1.
(The X key is immediately left of the STAT key). Press ZOOM 9 again to graph it.
Step 9. Optional: If you want to change the viewing window, press the WINDOW key. Enter your desired
window using Xmin, Xmax, Ymin, Ymax
**With contributions from Roberta Bloom
526 CHAPTER 12. LINEAR REGRESSION AND CORRELATION
12.6 Correlation Coefcient and Coefcient of Determination
6
12.6.1 The Correlation Coefcient r
Besides looking at the scatter plot and seeing that a line seems reasonable, how can you tell if the line is a
good predictor? Use the correlation coefcient as another indicator (besides the scatterplot) of the strength
of the relationship between x and y.
The correlation coefcient, r, developed by Karl Pearson in the early 1900s, is a numerical measure of the
strength of association between the independent variable x and the dependent variable y.
The correlation coefcient is calculated as
r =
n x y (x) (y)
_
_
n x
2
(x)
2
_

_
n y
2
(y)
2
_
(12.6)
where n = the number of data points.
If you suspect a linear relationship between x and y, then r can measure how strong the linear relationship
is.
What the VALUE of r tells us:
The value of r is always between -1 and +1: 1 r 1.
The size of the correlation r indicates the strength of the linear relationship between x and y. Values
of r close to -1 or to +1 indicate a stronger linear relationship between x and y.
If r = 0 there is absolutely no linear relationship between x and y (no linear correlation).
If r = 1, there is perfect positive correlation. If r = 1, there is perfect negative correlation. In both
these cases, all of the original data points lie on a straight line. Of course, in the real world, this will
not generally happen.
What the SIGN of r tells us
A positive value of r means that when x increases, y tends to increase and when x decreases, y tends
to decrease (positive correlation).
A negative value of r means that when x increases, y tends to decrease and when x decreases, y tends
to increase (negative correlation).
The sign of r is the same as the sign of the slope, b, of the best t line.
NOTE: Strong correlation does not suggest that x causes y or y causes x. We say "correlation does
not imply causation." For example, every person who learned math in the 17th century is dead.
However, learning math does not necessarily cause death!
6
This content is available online at <http://cnx.org/content/m17092/1.12/>.
527
(a) Positive Correlation (b) Negative Correlation (c) Zero Correlation
Figure 12.11: (a) A scatter plot showing data with a positive correlation. 0 < r < 1 (b) A scatter plot
showing data with a negative correlation. 1 < r < 0 (c) Ascatter plot showing data with zero correlation.
r=0
The formula for r looks formidable. However, computer spreadsheets, statistical software, and many cal-
culators can quickly calculate r. The correlation coefcient r is the bottom item in the output screens for the
LinRegTTest on the TI-83, TI-83+, or TI-84+ calculator (see previous section for instructions).
12.6.2 The Coefcient of Determination
r
2
is called the coefcient of determination. r
2
is the square of the correlation coefcient , but is usually
stated as a percent, rather than in decimal form. r
2
has an interpretation in the context of the data:
r
2
, when expressed as a percent, represents the percent of variation in the dependent variable y that
can be explained by variation in the independent variable x using the regression (best t) line.
1-r
2
, when expressed as a percent, represents the percent of variation in y that is NOT explained by
variation in x using the regression line. This can be seen as the scattering of the observed data points
about the regression line.
Consider the third exam/nal exam example introduced in the previous section
The line of best t is:
^
y
= 173.51 +4.83x
The correlation coefcient is r = 0.6631
The coefcient of determination is r
2
= 0.6631
2
= 0.4397
Interpretation of r
2
in the context of this example:
Approximately 44% of the variation (0.4397 is approximately 0.44) in the nal exam grades can be ex-
plained by the variation in the grades on the third exam, using the best t regression line.
Therefore approximately 56% of the variation (1 - 0.44 = 0.56) in the nal exam grades can NOT be ex-
plained by the variation in the grades on the third exam, using the best t regression line. (This is
seen as the scattering of the points about the line.)
**With contributions from Roberta Bloom.
528 CHAPTER 12. LINEAR REGRESSION AND CORRELATION
12.7 Testing the Signicance of the Correlation Coefcient
7
12.7.1 Testing the Signicance of the Correlation Coefcient
The correlation coefcient, r, tells us about the strength of the linear relationship between x and y. However,
the reliability of the linear model also depends on how many observed data points are in the sample. We
need to look at both the value of the correlation coefcient r and the sample size n, together.
We perform a hypothesis test of the "signicance of the correlation coefcient" to decide whether the
linear relationship in the sample data is strong enough to use to model the relationship in the population.
The sample data is used to compute r, the correlation coefcient for the sample. If we had data for the
entire population, we could nd the population correlation coefcient. But because we only have sample
data, we can not calculate the population correlation coefcient. The sample correlation coefcient, r, is our
estimate of the unknown population correlation coefcient.
The symbol for the population correlation coefcient is , the Greek letter "rho".
= population correlation coefcient (unknown)
r = sample correlation coefcient (known; calculated from sample data)
The hypothesis test lets us decide whether the value of the population correlation coefcient is "close to
0" or "signicantly different from 0". We decide this based on the sample correlation coefcient r and the
sample size n.
If the test concludes that the correlation coefcient is signicantly different from 0, we say that the
correlation coefcient is "signicant".
Conclusion: "There is sufcient evidence to conclude that there is a signicant linear relationship
between x and y because the correlation coefcient is signicantly different from 0."
What the conclusion means: There is a signicant linear relationship between x and y. We can use the
regression line to model the linear relationship between x and y in the population.
If the test concludes that the correlation coefcient is not signicantly different from 0 (it is close to 0),
we say that correlation coefcient is "not signicant".
Conclusion: "There is insufcient evidence to conclude that there is a signicant linear relationship
between x and y because the correlation coefcient is not signicantly different from 0."
What the conclusion means: There is not a signicant linear relationship between x and y. Therefore
we can NOT use the regression line to model a linear relationship between x and y in the population.
NOTE:
If r is signicant and the scatter plot shows a linear trend, the line can be used to predict the
value of y for values of x that are within the domain of observed x values.
If r is not signicant OR if the scatter plot does not show a linear trend, the line should not be
used for prediction.
If r is signicant and if the scatter plot shows a linear trend, the line may NOT be appropriate
or reliable for prediction OUTSIDE the domain of observed x values in the data.
PERFORMING THE HYPOTHESIS TEST
SETTING UP THE HYPOTHESES:
Null Hypothesis: H
o
: = 0
7
This content is available online at <http://cnx.org/content/m17077/1.15/>.
529
Alternate Hypothesis: H
a
: = 0
What the hypotheses mean in words:
Null Hypothesis H
o
: The population correlation coefcient IS NOT signicantly different from 0.
There IS NOT a signicant linear relationship(correlation) between x and y in the population.
Alternate Hypothesis H
a
: The population correlation coefcient IS signicantly DIFFERENT FROM
0. There IS A SIGNIFICANT LINEAR RELATIONSHIP (correlation) between x and y in the popula-
tion.
DRAWING A CONCLUSION:
There are two methods to make the decision. Both methods are equivalent and give the same result.
Method 1: Using the p-value
Method 2: Using a table of critical values
In this chapter of this textbook, we will always use a signicance level of 5%, = 0.05
Note: Using the p-value method, you could choose any appropriate signicance level you want; you are
not limited to using = 0.05. But the table of critical values provided in this textbook assumes that
we are using a signicance level of 5%, = 0.05. (If we wanted to use a different signicance level
than 5% with the critical value method, we would need different tables of critical values that are not
provided in this textbook.)
METHOD 1: Using a p-value to make a decision
The linear regression t-test LinRegTTEST on the TI-83+ or TI-84+ calculators calculates the p-value.
On the LinRegTTEST input screen, on the line prompt for or , highlight "= 0"
The output screen shows the p-value on the line that reads "p =".
(Most computer statistical software can calculate the p-value.)
If the p-value is less than the signicance level ( = 0.05):
Decision: REJECT the null hypothesis.
Conclusion: "There is sufcient evidence to conclude that there is a signicant linear relationship
between x and y because the correlation coefcient is signicantly different from 0."
If the p-value is NOT less than the signicance level ( = 0.05)
Decision: DO NOT REJECT the null hypothesis.
Conclusion: "There is insufcient evidence to conclude that there is a signicant linear relationship
between x and y because the correlation coefcient is NOT signicantly different from 0."
Calculation Notes:
You will use technology to calculate the p-value. The following describe the calculations to compute the
test statistics and the p-value:
The p-value is calculated using a t-distribution with n 2 degrees of freedom.
The formula for the test statistic is t =
r

n2

1r
2
. The value of the test statistic, t, is shown in the computer
or calculator output along with the p-value. The test statistic t has the same sign as the correlation
coefcient r.
The p-value is the combined area in both tails.
An alternative way to calculate the p-value (p) given by LinRegTTest is the command 2*tcdf(abs(t),10^99,
n-2) in 2nd DISTR.
THIRD EXAM vs FINAL EXAM EXAMPLE: p value method
530 CHAPTER 12. LINEAR REGRESSION AND CORRELATION
Consider the third exam/nal exam example.
The line of best t is:
^
y
= 173.51 +4.83x with r = 0.6631 and there are n = 11 data points.
Can the regression line be used for prediction? Given a third exam score (x value), can we use the
line to predict the nal exam score (predicted y value)?
H
o
: = 0
H
a
: = 0
= 0.05
The p-value is 0.026 (from LinRegTTest on your calculator or from computer software)
The p-value, 0.026, is less than the signicance level of = 0.05
Decision: Reject the Null Hypothesis H
o
Conclusion: There is sufcient evidence to conclude that there is a signicant linear relationship between
x and y because the correlation coefcient is signicantly different from 0.
Because r is signicant and the scatter plot shows a linear trend, the regression line can be used to
predict nal exam scores.
METHOD 2: Using a table of Critical Values to make a decision
The 95% Critical Values of the Sample Correlation Coefcient Table (Section 12.10) at the end of this
chapter (before the Summary (Section 12.11)) may be used to give you a good idea of whether the com-
puted value of r is signicant or not. Compare r to the appropriate critical value in the table. If r is not
between the positive and negative critical values, then the correlation coefcient is signicant. If r is signif-
icant, then you may want to use the line for prediction.
Example 12.7
Suppose you computed r = 0.801 using n = 10 data points. df = n 2 = 10 2 = 8. The
critical values associated with df = 8 are -0.632 and + 0.632. If r< negative critical value or r >
positive critical value, then r is signicant. Since r = 0.801 and 0.801 > 0.632, r is signicant and
the line may be used for prediction. If you view this example on a number line, it will help you.
Figure 12.12: r is not signicant between -0.632 and +0.632. r = 0.801 > +0.632. Therefore, r is signicant.
Example 12.8
Suppose you computed r = 0.624 with 14 data points. df = 14 2 = 12. The critical values are
-0.532 and 0.532. Since 0.624<0.532, r is signicant and the line may be used for prediction
Figure 12.13: r = 0.624<0.532. Therefore, r is signicant.
531
Example 12.9
Suppose you computed r = 0.776 and n = 6. df = 6 2 = 4. The critical values are -0.811
and 0.811. Since 0.811< 0.776 < 0.811, r is not signicant and the line should not be used for
prediction.
Figure 12.14: 0.811<r = 0.776<0.811. Therefore, r is not signicant.
THIRD EXAM vs FINAL EXAM EXAMPLE: critical value method
Consider the third exam/nal exam example.
The line of best t is:
^
y
= 173.51 +4.83x with r = 0.6631 and there are n = 11 data points.
Can the regression line be used for prediction? Given a third exam score (x value), can we use the
line to predict the nal exam score (predicted y value)?
H
o
: = 0
H
a
: = 0
= 0.05
Use the "95% Critical Value" table for r with df = n 2 = 11 2 = 9
The critical values are -0.602 and +0.602
Since 0.6631 > 0.602, r is signicant.
Decision: Reject H
o
:
Conclusion:There is sufcient evidence to conclude that there is a signicant linear relationship between
x and y because the correlation coefcient is signicantly different from 0.
Because r is signicant and the scatter plot shows a linear trend, the regression line can be used to
predict nal exam scores.
Example 12.10: Additional Practice Examples using Critical Values
Suppose you computed the following correlation coefcients. Using the table at the end of the
chapter, determine if r is signicant and the line of best t associated with each r can be used to
predict a y value. If it helps, draw a number line.
1. r = 0.567 and the sample size, n, is 19. The df = n 2 = 17. The critical value is -0.456.
0.567<0.456 so r is signicant.
2. r = 0.708 and the sample size, n, is 9. The df = n 2 = 7. The critical value is 0.666.
0.708 > 0.666 so r is signicant.
3. r = 0.134 and the sample size, n, is 14. The df = 14 2 = 12. The critical value is 0.532.
0.134 is between -0.532 and 0.532 so r is not signicant.
4. r = 0 and the sample size, n, is 5. No matter what the dfs are, r = 0 is between the two
critical values so r is not signicant.
532 CHAPTER 12. LINEAR REGRESSION AND CORRELATION
12.7.2 Assumptions in Testing the Signicance of the Correlation Coefcient
Testing the signicance of the correlation coefcient requires that certain assumptions about the data are
satised. The premise of this test is that the data are a sample of observed points taken from a larger
population. We have not examined the entire population because it is not possible or feasible to do so. We
are examining the sample to draw a conclusion about whether the linear relationship that we see between
x and y in the sample data provides strong enough evidence so that we can conclude that there is a linear
relationship between x and y in the population.
The regression line equation that we calculate from the sample data gives the best t line for our particular
sample. We want to use this best t line for the sample as an estimate of the best t line for the population.
Examining the scatterplot and testing the signicance of the correlation coefcient helps us determine if it
is appropriate to do this.
The assumptions underlying the test of signicance are:
There is a linear relationship in the population that models the average value of y for varying values
of x. In other words, the expected value of y for each particular value lies on a straight line in the
population. (We do not know the equation for the line for the population. Our regression line from
the sample is our best estimate of this line in the population.)
The y values for any particular x value are normally distributed about the line. This implies that
there are more y values scattered closer to the line than are scattered farther away. Assumption (1)
above implies that these normal distributions are centered on the line: the means of these normal
distributions of y values lie on the line.
The standard deviations of the population y values about the line are equal for each value of x. In
other words, each of these normal distributions of y values has the same shape and spread about the
line.
The residual errors are mutually independent (no pattern).
Figure 12.15: The y values for each x value are normally distributed about the line with the same standard
deviation. For each x value, the mean of the y values lies on the regression line. More y values lie near the
line than are scattered further away from the line.
**With contributions from Roberta Bloom
533
12.8 Prediction
8
Recall the third exam/nal exam example.
We examined the scatterplot and showed that the correlation coefcient is signicant. We found the equa-
tion of the best t line for the nal exam grade as a function of the grade on the third exam. We can now
use the least squares regression line for prediction.
Suppose you want to estimate, or predict, the nal exam score of statistics students who received 73 on the
third exam. The exam scores (x-values) range from 65 to 75. Since 73 is between the x-values 65 and 75,
substitute x = 73 into the equation. Then:
^
y
= 173.51 +4.83 (73) = 179.08 (12.8)
We predict that statistic students who earn a grade of 73 on the third exam will earn a grade of 179.08 on
the nal exam, on average.
Example 12.11
Recall the third exam/nal exam example.
Problem 1
What would you predict the nal exam score to be for a student who scored a 66 on the third
exam?
Solution
145.27
Problem 2 (Solution on p. 571.)
What would you predict the nal exam score to be for a student who scored a 90 on the third
exam?
**With contributions from Roberta Bloom
12.9 Outliers
9
In some data sets, there are values (observed data points) called outliers. Outliers are observed data
points that are far from the least squares line. They have large "errors", where the "error" or residual is the
vertical distance from the line to the point.
Outliers need to be examined closely. Sometimes, for some reason or another, they should not be included
in the analysis of the data. It is possible that an outlier is a result of erroneous data. Other times, an outlier
may hold valuable information about the population under study and should remain included in the data.
The key is to carefully examine what causes a data point to be an outlier.
Besides outliers, a sample may contain one or a few points that are called inuential points. Inuential
points are observed data points that are far from the other observed data points in the horizontal direction.
These points may have a big effect on the slope of the regression line. To begin to identify an inuential
point, you can remove it from the data set and see if the slope of the regression line is changed signicantly.
8
This content is available online at <http://cnx.org/content/m17095/1.8/>.
9
This content is available online at <http://cnx.org/content/m17094/1.14/>.
534 CHAPTER 12. LINEAR REGRESSION AND CORRELATION
Computers and many calculators can be used to identify outliers from the data. Computer output for
regression analysis will often identify both outliers and inuential points so that you can examine them.
Identifying Outliers
We could guess at outliers by looking at a graph of the scatterplot and best t line. However we would like
some guideline as to how far away a point needs to be in order to be considered an outlier. As a rough rule
of thumb, we can ag any point that is located further than two standard deviations above or below the
best t line as an outlier. The standard deviation used is the standard deviation of the residuals or errors.
We can do this visually in the scatterplot by drawing an extra pair of lines that are two standard deviations
above and below the best t line. Any data points that are outside this extra pair of lines are agged as
potential outliers. Or we can do this numerically by calculating each residual and comparing it to twice the
standard deviation. On the TI-83, 83+, or 84+, the graphical approach is easier. The graphical procedure
is shown rst, followed by the numerical calculations. You would generally only need to use one of these
methods.
Example 12.12
In the third exam/nal exam example, you can determine if there is an outlier or not. If there is
an outlier, as an exercise, delete it and t the remaining data to a new line. For this example, the
new line ought to t the remaining data better. This means the SSE should be smaller and the
correlation coefcient ought to be closer to 1 or -1.
Solution
Graphical Identication of Outliers
With the TI-83,83+,84+ graphing calculators, it is easy to identify the outlier graphically and visu-
ally. If we were to measure the vertical distance from any data point to the corresponding point
on the line of best t and that distance was equal to 2s or farther, then we would consider the data
point to be "too far" from the line of best t. We need to nd and graph the lines that are two
standard deviations below and above the regression line. Any points that are outside these two
lines are outliers. We will call these lines Y2 and Y3:
As we did with the equation of the regression line and the correlation coefcient, we will use
technology to calculate this standard deviation for us. Using the LinRegTTest with this data,
scroll down through the output screens to nd s=16.412
Line Y2=-173.5+4.83x-2(16.4) and line Y3=-173.5+4.83x+2(16.4)
where
^
y
=-173.5+4.83x is the line of best t. Y2 and Y3 have the same slope as the line of
best t.
Graph the scatterplot with the best t line in equation Y1, then enter the two extra lines as Y2 and
Y3 in the "Y="equation editor and press ZOOM 9. You will nd that the only data point that is not
between lines Y2 and Y3 is the point x=65, y=175. On the calculator screen it is just barely outside
these lines. The outlier is the student who had a grade of 65 on the third exam and 175 on the nal
exam; this point is further than 2 standard deviations away from the best t line.
Sometimes a point is so close to the lines used to ag outliers on the graph that it is difcult to tell
if the point is between or outside the lines. On a computer, enlarging the graph may help; on a
small calculator screen, zooming in may make the graph clearer. Note that when the graph does
not give a clear enough picture, you can use the numerical comparisons to identify outliers.
535
Figure 12.16
Numerical Identication of Outliers
In the table below, the rst two columns are the third exam and nal exam data. The third
column shows the predicted
^
y
values calculated from the line of best t:
^
y
=-173.5+4.83x. The
residuals, or errors, have been calculated in the fourth column of the table: observed y value
predicted y value = y
^
y
.
s is the standard deviation of all the y
^
y
= values where n = the total number of data points. If
each residual is calculated and squared, and the results are added, we get the SSE. The standard
deviation of the residuals is calculated from the SSE as:
s =
_
SSE
n2
Rather than calculate the value of s ourselves, we can nd s using the computer or calculator. For
this example, the calculator function LinRegTTest found s = 16.4 as the standard deviation of the
residuals 35; -17; 16; -6; -19; 9; 3; -1; -10; -9; -1.
536 CHAPTER 12. LINEAR REGRESSION AND CORRELATION
x y
^
y
y
^
y
65 175 140 175 140 = 35
67 133 150 133 150 = 17
71 185 169 185 169 = 16
71 163 169 163 169 = 6
66 126 145 126 145 = 19
75 198 189 198 189 = 9
67 153 150 153 150 = 3
70 163 164 163 164 = 1
71 159 169 159 169 = 10
69 151 160 151 160 = 9
69 159 160 159 160 = 1
Table 12.1
We are looking for all data points for which the residual is greater than 2s=2(16.4)=32.8 or less than
-32.8. Compare these values to the residuals in column 4 of the table. The only such data point is
the student who had a grade of 65 on the third exam and 175 on the nal exam; the residual for
this student is 35.
How does the outlier affect the best t line?
Numerically and graphically, we have identied the point (65,175) as an outlier. We should re-
examine the data for this point to see if there are any problems with the data. If there is an error
we should x the error if possible, or delete the data. If the data is correct, we would leave it in
the data set. For this problem, we will suppose that we examined the data and found that this
outlier data was an error. Therefore we will continue on and delete the outlier, so that we can
explore how it affects the results, as a learning experience.
Compute a new best-t line and correlation coefcient using the 10 remaining points:
On the TI-83, TI-83+, TI-84+ calculators, delete the outlier from L1 and L2. Using the LinRegTTest,
the new line of best t and the correlation coefcient are:
^
y
= 355.19 +7.39x and r = 0.9121
The new line with r = 0.9121 is a stronger correlation than the original (r=0.6631) because r =
0.9121 is closer to 1. This means that the new line is a better t to the 10 remaining data values.
The line can better predict the nal exam score given the third exam score.
Numerical Identication of Outliers: Calculating s and Finding Outliers Manually
If you do not have the function LinRegTTest, then you can calculate the outlier in the rst example by
doing the following.
First, square each |y
^
y
| (See the TABLE above):
537
The squares are 35
2
; 17
2
; 16
2
; 6
2
; 19
2
; 9
2
; 3
2
; 1
2
; 10
2
; 9
2
; 1
2
Then, add (sum) all the |y
^
y
| squared terms using the formula
11

i = 1
_
|y
i

^
y
i
|
_
2
=
11

i = 1

i
2
(Recall that y
i

^
y
i
=
i
.)
= 35
2
+17
2
+16
2
+6
2
+19
2
+9
2
+3
2
+1
2
+10
2
+9
2
+1
2
= 2440 = SSE. The result, SSE is the Sum of Squared Errors.
Next, calculate s, the standard deviation of all the y
^
y
= values where n = the total number of data
points.
The calculation is s =
_
SSE
n2
For the third exam/nal exam problem, s =
_
2440
112
= 16.47
Next, multiply s by 1.9:
(1.9) (16.47) = 31.29
31.29 is almost 2 standard deviations away from the mean of the y
^
y
values.
If we were to measure the vertical distance from any data point to the corresponding point on the line of
best t and that distance is at least 1.9s, then we would consider the data point to be "too far" from the line
of best t. We call that point a potential outlier.
For the example, if any of the |y
^
y
| values are at least 31.29, the corresponding (x, y) data point is a
potential outlier.
For the third exam/nal exam problem, all the |y
^
y
|s are less than 31.29 except for the rst one which is
35.
35 > 31.29 That is, |y
^
y
| (1.9) (s)
The point which corresponds to |y
^
y
| = 35 is (65, 175). Therefore, the data point (65, 175) is a potential
outlier. For this example, we will delete it. (Remember, we do not always delete an outlier.)
The next step is to compute a new best-t line using the 10 remaining points. The new line of best
t and the correlation coefcient are:
^
y
= 355.19 +7.39x and r = 0.9121
Example 12.13
Using this new line of best t (based on the remaining 10 data points), what would a student
who receives a 73 on the third exam expect to receive on the nal exam? Is this the same as the
prediction made using the original line?
538 CHAPTER 12. LINEAR REGRESSION AND CORRELATION
Solution
Using the new line of best t,
^
y
= 355.19 +7.39(73) = 184.28. A student who scored 73 points
on the third exam would expect to earn 184 points on the nal exam.
The original line predicted
^
y
= 173.51 + 4.83(73) = 179.08 so the prediction using the
new line with the outlier eliminated differs from the original prediction.
Example 12.14
(From The Consumer Price Indexes Web site) The Consumer Price Index (CPI) measures the aver-
age change over time in the prices paid by urban consumers for consumer goods and services. The
CPI affects nearly all Americans because of the many ways it is used. One of its biggest uses is as
a measure of ination. By providing information about price changes in the Nations economy to
government, business, and labor, the CPI helps them to make economic decisions. The President,
Congress, and the Federal Reserve Board use the CPIs trends to formulate monetary and scal
policies. In the following table, x is the year and y is the CPI.
Data:
x y
1915 10.1
1926 17.7
1935 13.7
1940 14.7
1947 24.1
1952 26.5
1964 31.0
1969 36.7
1975 49.3
1979 72.6
1980 82.4
1986 109.6
1991 130.7
1999 166.6
Table 12.2
Problem
Make a scatterplot of the data.
Calculate the least squares line. Write the equation in the form
^
y
= a +bx.
Draw the line on the scatterplot.
Find the correlation coefcient. Is it signicant?
What is the average CPI for the year 1990?
539
Solution
Scatter plot and line of best t.

^
y
= 3204 +1.662x is the equation of the line of best t.
r = 0.8694
The number of data points is n = 14. Use the 95% Critical Values of the Sample Correlation
Coefcient table at the end of Chapter 12. n 2 = 12. The corresponding critical value is
0.532. Since 0.8694 > 0.532, r is signicant.

^
y
= 3204 +1.662 (1990) = 103.4 CPI
Using the calculator LinRegTTest, we nd that s = 25.4 ; graphing the lines Y2=-3204+1.662X-
2(25.4) and Y3=-3204+1.662X+2(25.4) shows that no data values are outside those lines, iden-
tifying no outliers. (Note that the year 1999 was very close to the upper line, but still inside
it.)
Figure 12.17
NOTE: In the example, notice the pattern of the points compared to the line. Although the correla-
tion coefcient is signicant, the pattern in the scatterplot indicates that a curve would be a more
appropriate model to use than a line. In this example, a statistician should prefer to use other
methods to t a curve to this data, rather than model the data with the line we found. In addition
to doing the calculations, it is always important to look at the scatterplot when deciding whether
a linear model is appropriate.
If you are interested in seeing more years of data, visit the Bureau of Labor Statistics CPI website
ftp://ftp.bls.gov/pub/special.requests/cpi/cpiai.txt ; our data is taken from the column entitled
"Annual Avg." (third column from the right). For example you could add more current years of
data. Try adding the more recent years 2004 : CPI=188.9, 2008 : CPI=215.3 and 2011: CPI=224.9.
See how it affects the model. (Check:
^
y
= 4436 +2.295x. r = 0.9018. Is r signicant? Is the t
better with the addition of the new points?)
**With contributions from Roberta Bloom
540 CHAPTER 12. LINEAR REGRESSION AND CORRELATION
12.10 95% Critical Values of the Sample Correlation Coefcient Table
10
10
This content is available online at <http://cnx.org/content/m17098/1.6/>.
541
Degrees of Freedom: n 2 Critical Values: (+ and )
1 0.997
2 0.950
3 0.878
4 0.811
5 0.754
6 0.707
7 0.666
8 0.632
9 0.602
10 0.576
11 0.555
12 0.532
13 0.514
14 0.497
15 0.482
16 0.468
17 0.456
18 0.444
19 0.433
20 0.423
21 0.413
22 0.404
23 0.396
24 0.388
25 0.381
26 0.374
continued on next page
542 CHAPTER 12. LINEAR REGRESSION AND CORRELATION
27 0.367
28 0.361
29 0.355
30 0.349
40 0.304
50 0.273
60 0.250
70 0.232
80 0.217
90 0.205
100 0.195
Table 12.3
543
12.11 Summary
11
Bivariate Data: Each data point has two values. The form is (x, y).
Line of Best Fit or Least Squares Line (LSL):
^
y
= a +bx
x = independent variable; y = dependent variable
Residual: Actual y value predicted y value = y
^
y
Correlation Coefcient r:
1. Used to determine whether a line of best t is good for prediction.
2. Between -1 and 1 inclusive. The closer r is to 1 or -1, the closer the original points are to a straight line.
3. If r is negative, the slope is negative. If r is positive, the slope is positive.
4. If r = 0, then the line is horizontal.
Sum of Squared Errors (SSE): The smaller the SSE, the better the original set of points ts the line of best
t.
Outlier: A point that does not seem to t the rest of the data.
11
This content is available online at <http://cnx.org/content/m17081/1.4/>.
544 CHAPTER 12. LINEAR REGRESSION AND CORRELATION
12.12 Practice: Linear Regression
12
12.12.1 Student Learning Outcomes
The student will evaluate bivariate data and determine if a line is an appropriate t to the data.
12.12.2 Given
Below are real data for the rst two decades of AIDS reporting. (Source: Centers for Disease Control and
Prevention, National Center for HIV, STD, and TB Prevention)
Adults and Adolescents only, United States
Year # AIDS cases diagnosed # AIDS deaths
Pre-1981 91 29
1981 319 121
1982 1,170 453
1983 3,076 1,482
1984 6,240 3,466
1985 11,776 6,878
1986 19,032 11,987
1987 28,564 16,162
1988 35,447 20,868
1989 42,674 27,591
1990 48,634 31,335
1991 59,660 36,560
1992 78,530 41,055
1993 78,834 44,730
1994 71,874 49,095
1995 68,505 49,456
1996 59,347 38,510
1997 47,149 20,736
1998 38,393 19,005
1999 25,174 18,454
2000 25,522 17,347
2001 25,643 17,402
2002 26,464 16,371
Total 802,118 489,093
Table 12.4
12
This content is available online at <http://cnx.org/content/m17088/1.12/>.
545
NOTE: We will use the columns year and # AIDS cases diagnosed for all questions unless
otherwise stated.
12.12.3 Graphing
Graph year vs. # AIDS cases diagnosed. Plot the points on the graph located below in the section
titled "Plot" . Do not include pre-1981. Label both axes with words. Scale both axes.
12.12.4 Data
Exercise 12.12.1
Enter your data into your calculator or computer. The pre-1981 data should not be included. Why
is that so?
12.12.5 Linear Equation
Write the linear equation below, rounding to 4 decimal places:
NOTE: For any prediction questions, the answers are calculated using the least squares (best t)
line equation cited in the solution.
Exercise 12.12.2 (Solution on p. 571.)
Calculate the following:
a. a =
b. b =
c. corr. =
d. n =(# of pairs)
Exercise 12.12.3 (Solution on p. 571.)
equation:
^
y
=
12.12.6 Solve
Exercise 12.12.4 (Solution on p. 571.)
Solve.
a. When x = 1985,
^
y
=
b. When x = 1990,
^
y
=
546 CHAPTER 12. LINEAR REGRESSION AND CORRELATION
12.12.7 Plot
Plot the 2 above points on the graph below. Then, connect the 2 points to form the regression line.
Obtain the graph on your calculator or computer.
12.12.8 Discussion Questions
Look at the graph above.
Exercise 12.12.5
Does the line seem to t the data? Why or why not?
Exercise 12.12.6
Do you think a linear t is best? Why or why not?
Exercise 12.12.7
Hand draw a smooth curve on the graph above that shows the ow of the data.
Exercise 12.12.8
What does the correlation imply about the relationship between time (years) and the number of
diagnosed AIDS cases reported in the U.S.?
Exercise 12.12.9
Why is year the independent variable and # AIDS cases diagnosed. the dependent variable
(instead of the reverse)?
Exercise 12.12.10 (Solution on p. 571.)
Solve.
a. When x = 1970,
^
y
=:
b. Why doesnt this answer make sense?
547
12.13 Homework
13
Exercise 12.13.1 (Solution on p. 571.)
For each situation below, state the independent variable and the dependent variable.
a. A study is done to determine if elderly drivers are involved in more motor vehicle fatalities
than all other drivers. The number of fatalities per 100,000 drivers is compared to the age of
drivers.
b. A study is done to determine if the weekly grocery bill changes based on the number of family
members.
c. Insurance companies base life insurance premiums partially on the age of the applicant.
d. Utility bills vary according to power consumption.
e. A study is done to determine if a higher education reduces the crime rate in a population.
NOTE: For any prediction questions, the answers are calculated using the least squares (best t)
line equation cited in the solution.
Exercise 12.13.2
Recently, the annual number of driver deaths per 100,000
for the selected age groups was as follows (Source: http://
http://www.census.gov/compendia/statab/cats/transportation/motor_vehicle_accidents_and_fatalities.html
14
):
Age Number of Driver Deaths per 100,000
16-19 38
20-24 36
25-34 24
35-54 20
55-74 18
75+ 28
Table 12.5
a. For each age group, pick the midpoint of the interval for the x value. (For the 75+ group, use
80.)
b. Using ages as the independent variable and Number of driver deaths per 100,000 as the
dependent variable, make a scatter plot of the data.
c. Calculate the least squares (bestt) line. Put the equation in the form of:
^
y
= a +bx
d. Find the correlation coefcient. Is it signicant?
e. Pick two ages and nd the estimated fatality rates.
f. Use the two points in (e) to plot the least squares line on your graph from (b).
g. Based on the above data, is there a linear relationship between age of a driver and driver fatality
rate?
h. What is the slope of the least squares (best-t) line? Interpret the slope.
13
This content is available online at <http://cnx.org/content/m17085/1.14/>.
14
http://www.census.gov/compendia/statab/cats/transportation/motor_vehicle_accidents_and_fatalities.html
548 CHAPTER 12. LINEAR REGRESSION AND CORRELATION
Exercise 12.13.3 (Solution on p. 571.)
The average number of people in a family that received welfare for various years is given below.
(Source: House Ways and Means Committee, Health and Human Services Department )
Year Welfare family size
1969 4.0
1973 3.6
1975 3.2
1979 3.0
1983 3.0
1988 3.0
1991 2.9
Table 12.6
a. Using year as the independent variable and welfare family size as the dependent variable,
make a scatter plot of the data.
b. Calculate the least squares line. Put the equation in the form of:
^
y
= a +bx
c. Find the correlation coefcient. Is it signicant?
d. Pick two years between 1969 and 1991 and nd the estimated welfare family sizes.
e. Use the two points in (d) to plot the least squares line on your graph from (b).
f. Based on the above data, is there a linear relationship between the year and the average number
of people in a welfare family?
g. Using the least squares line, estimate the welfare family sizes for 1960 and 1995. Does the least
squares line give an accurate estimate for those years? Explain why or why not.
h. Are there any outliers in the above data?
i. What is the estimated average welfare family size for 1986? Does the least squares line give an
accurate estimate for that year? Explain why or why not.
j. What is the slope of the least squares (best-t) line? Interpret the slope.
Exercise 12.13.4
Use the AIDS data from the practice for this section (Section 12.12.2: Given), but this time use the
columns year # and # new AIDS deaths in U.S. Answer all of the questions from the practice
again, using the new columns.
Exercise 12.13.5 (Solution on p. 572.)
The height (sidewalk to roof) of notable tall buildings in America is compared to the number of
stories of the building (beginning at street level). (Source: Microsoft Bookshelf )
549
Height (in feet) Stories
1050 57
428 28
362 26
529 40
790 60
401 22
380 38
1454 110
1127 100
700 46
Table 12.7
a. Using stories as the independent variable and height as the dependent variable, make a
scatter plot of the data.
b. Does it appear from inspection that there is a relationship between the variables?
c. Calculate the least squares line. Put the equation in the form of:
^
y
= a +bx
d. Find the correlation coefcient. Is it signicant?
e. Find the estimated heights for 32 stories and for 94 stories.
f. Use the two points in (e) to plot the least squares line on your graph from (b).
g. Based on the above data, is there a linear relationship between the number of stories in tall
buildings and the height of the buildings?
h. Are there any outliers in the above data? If so, which point(s)?
i. What is the estimated height of a building with 6 stories? Does the least squares line give an
accurate estimate of height? Explain why or why not.
j. Based on the least squares line, adding an extra story is predicted to add about how many feet
to a building?
k. What is the slope of the least squares (best-t) line? Interpret the slope.
Exercise 12.13.6
Below is the life expectancy for an individual born in the United States in certain years. (Source:
National Center for Health Statistics)
550 CHAPTER 12. LINEAR REGRESSION AND CORRELATION
Year of Birth Life Expectancy
1930 59.7
1940 62.9
1950 70.2
1965 69.7
1973 71.4
1982 74.5
1987 75
1992 75.7
2010 78.7
Table 12.8
a. Decide which variable should be the independent variable and which should be the dependent
variable.
b. Draw a scatter plot of the ordered pairs.
c. Calculate the least squares line. Put the equation in the form of:
^
y
= a +bx
d. Find the correlation coefcient. Is it signicant?
e. Find the estimated life expectancy for an individual born in 1950 and for one born in 1982.
f. Why arent the answers to part (e) the values on the above chart that correspond to those years?
g. Use the two points in (e) to plot the least squares line on your graph from (b).
h. Based on the above data, is there a linear relationship between the year of birth and life ex-
pectancy?
i. Are there any outliers in the above data?
j. Using the least squares line, nd the estimated life expectancy for an individual born in 1850.
Does the least squares line give an accurate estimate for that year? Explain why or why not.
k. What is the slope of the least squares (best-t) line? Interpret the slope.
Exercise 12.13.7 (Solution on p. 572.)
The percent of female wage and salary workers who are paid hourly rates is given below for the
years 1979 - 1992. (Source: Bureau of Labor Statistics, U.S. Dept. of Labor)
551
Year Percent of workers paid hourly rates
1979 61.2
1980 60.7
1981 61.3
1982 61.3
1983 61.8
1984 61.7
1985 61.8
1986 62.0
1987 62.7
1990 62.8
1992 62.9
Table 12.9
a. Using year as the independent variable and percent as the dependent variable, make a
scatter plot of the data.
b. Does it appear from inspection that there is a relationship between the variables? Why or why
not?
c. Calculate the least squares line. Put the equation in the form of:
^
y
= a +bx
d. Find the correlation coefcient. Is it signicant?
e. Find the estimated percents for 1991 and 1988.
f. Use the two points in (e) to plot the least squares line on your graph from (b).
g. Based on the above data, is there a linear relationship between the year and the percent of
female wage and salary earners who are paid hourly rates?
h. Are there any outliers in the above data?
i. What is the estimated percent for the year 2050? Does the least squares line give an accurate
estimate for that year? Explain why or why not?
j. What is the slope of the least squares (best-t) line? Interpret the slope.
Exercise 12.13.8
The maximum discount value of the Entertainment card for the Fine Dining section, Edition
10, for various pages is given below.
552 CHAPTER 12. LINEAR REGRESSION AND CORRELATION
Page number Maximum value ($)
4 16
14 19
25 15
32 17
43 19
57 15
72 16
85 15
90 17
Table 12.10
a. Decide which variable should be the independent variable and which should be the dependent
variable.
b. Draw a scatter plot of the ordered pairs.
c. Calculate the least squares line. Put the equation in the form of:
^
y
= a +bx
d. Find the correlation coefcient. Is it signicant?
e. Find the estimated maximum values for the restaurants on page 10 and on page 70.
f. Use the two points in (e) to plot the least squares line on your graph from (b).
g. Does it appear that the restaurants giving the maximum value are placed in the beginning of
the Fine Dining section? How did you arrive at your answer?
h. Suppose that there were 200 pages of restaurants. What do you estimate to be the maximum
value for a restaurant listed on page 200?
i. Is the least squares line valid for page 200? Why or why not?
j. What is the slope of the least squares (best-t) line? Interpret the slope.
The next two questions refer to the following data: The cost of a leading liquid laundry detergent in
different sizes is given below.
Size (ounces) Cost ($) Cost per ounce
16 3.99
32 4.99
64 5.99
200 10.99
Table 12.11
Exercise 12.13.9 (Solution on p. 572.)
a. Using size as the independent variable and cost as the dependent variable, make a scatter
plot.
b. Does it appear from inspection that there is a relationship between the variables? Why or why
not?
553
c. Calculate the least squares line. Put the equation in the form of:
^
y
= a +bx
d. Find the correlation coefcient. Is it signicant?
e. If the laundry detergent were sold in a 40 ounce size, nd the estimated cost.
f. If the laundry detergent were sold in a 90 ounce size, nd the estimated cost.
g. Use the two points in (e) and (f) to plot the least squares line on your graph from (a).
h. Does it appear that a line is the best way to t the data? Why or why not?
i. Are there any outliers in the above data?
j. Is the least squares line valid for predicting what a 300 ounce size of the laundry detergent
would cost? Why or why not?
k. What is the slope of the least squares (best-t) line? Interpret the slope.
Exercise 12.13.10
a. Complete the above table for the cost per ounce of the different sizes.
b. Using Size as the independent variable and Cost per ounce as the dependent variable,
make a scatter plot of the data.
c. Does it appear from inspection that there is a relationship between the variables? Why or why
not?
d. Calculate the least squares line. Put the equation in the form of:
^
y
= a +bx
e. Find the correlation coefcient. Is it signicant?
f. If the laundry detergent were sold in a 40 ounce size, nd the estimated cost per ounce.
g. If the laundry detergent were sold in a 90 ounce size, nd the estimated cost per ounce.
h. Use the two points in (f) and (g) to plot the least squares line on your graph from (b).
i. Does it appear that a line is the best way to t the data? Why or why not?
j. Are there any outliers in the above data?
k. Is the least squares line valid for predicting what a 300 ounce size of the laundry detergent
would cost per ounce? Why or why not?
l. What is the slope of the least squares (best-t) line? Interpret the slope.
Exercise 12.13.11 (Solution on p. 572.)
According to yer by a Prudential Insurance Company representative, the costs of approximate
probate fees and taxes for selected net taxable estates are as follows:
Net Taxable Estate ($) Approximate Probate Fees and Taxes ($)
600,000 30,000
750,000 92,500
1,000,000 203,000
1,500,000 438,000
2,000,000 688,000
2,500,000 1,037,000
3,000,000 1,350,000
Table 12.12
a. Decide which variable should be the independent variable and which should be the dependent
variable.
b. Make a scatter plot of the data.
554 CHAPTER 12. LINEAR REGRESSION AND CORRELATION
c. Does it appear from inspection that there is a relationship between the variables? Why or why
not?
d. Calculate the least squares line. Put the equation in the form of:
^
y
= a +bx
e. Find the correlation coefcient. Is it signicant?
f. Find the estimated total cost for a net taxable estate of $1,000,000. Find the cost for $2,500,000.
g. Use the two points in (f) to plot the least squares line on your graph from (b).
h. Does it appear that a line is the best way to t the data? Why or why not?
i. Are there any outliers in the above data?
j. Based on the above, what would be the probate fees and taxes for an estate that does not have
any assets?
k. What is the slope of the least squares (best-t) line? Interpret the slope.
Exercise 12.13.12
The following are advertised sale prices of color televisions at Andersons.
Size (inches) Sale Price ($)
9 147
20 197
27 297
31 447
35 1177
40 2177
60 2497
Table 12.13
a. Decide which variable should be the independent variable and which should be the dependent
variable.
b. Make a scatter plot of the data.
c. Does it appear from inspection that there is a relationship between the variables? Why or why
not?
d. Calculate the least squares line. Put the equation in the form of:
^
y
= a +bx
e. Find the correlation coefcient. Is it signicant?
f. Find the estimated sale price for a 32 inch television. Find the cost for a 50 inch television.
g. Use the two points in (f) to plot the least squares line on your graph from (b).
h. Does it appear that a line is the best way to t the data? Why or why not?
i. Are there any outliers in the above data?
j. What is the slope of the least squares (best-t) line? Interpret the slope.
Exercise 12.13.13 (Solution on p. 572.)
Below are the average heights for American boys. (Source: Physicians Handbook, 1990)
555
Age (years) Height (cm)
birth 50.8
2 83.8
3 91.4
5 106.6
7 119.3
10 137.1
14 157.5
Table 12.14
a. Decide which variable should be the independent variable and which should be the dependent
variable.
b. Make a scatter plot of the data.
c. Does it appear from inspection that there is a relationship between the variables? Why or why
not?
d. Calculate the least squares line. Put the equation in the form of:
^
y
= a +bx
e. Find the correlation coefcient. Is it signicant?
f. Find the estimated average height for a one yearold. Find the estimated average height for an
eleven yearold.
g. Use the two points in (f) to plot the least squares line on your graph from (b).
h. Does it appear that a line is the best way to t the data? Why or why not?
i. Are there any outliers in the above data?
j. Use the least squares line to estimate the average height for a sixtytwo yearold man. Do you
think that your answer is reasonable? Why or why not?
k. What is the slope of the least squares (best-t) line? Interpret the slope.
Exercise 12.13.14
The following chart gives the gold medal times for every other Summer Olympics for the womens
100 meter freestyle (swimming).
Year Time (seconds)
1912 82.2
1924 72.4
1932 66.8
1952 66.8
1960 61.2
1968 60.0
1976 55.65
1984 55.92
1992 54.64
2000 53.8
2008 53.1
556 CHAPTER 12. LINEAR REGRESSION AND CORRELATION
Table 12.15
a. Decide which variable should be the independent variable and which should be the dependent
variable.
b. Make a scatter plot of the data.
c. Does it appear from inspection that there is a relationship between the variables? Why or why
not?
d. Calculate the least squares line. Put the equation in the form of:
^
y
= a +bx
e. Find the correlation coefcient. Is the decrease in times signicant?
f. Find the estimated gold medal time for 1932. Find the estimated time for 1984.
g. Why are the answers from (f) different from the chart values?
h. Use the two points in (f) to plot the least squares line on your graph from (b).
i. Does it appear that a line is the best way to t the data? Why or why not?
j. Use the least squares line to estimate the gold medal time for the next Summer Olympics. Do
you think that your answer is reasonable? Why or why not?
The next three questions use the following state information.
State # letters in name Year entered the
Union
Rank for entering
the Union
Area (square
miles)
Alabama 7 1819 22 52,423
Colorado 1876 38 104,100
Hawaii 1959 50 10,932
Iowa 1846 29 56,276
Maryland 1788 7 12,407
Missouri 1821 24 69,709
New Jersey 1787 3 8,722
Ohio 1803 17 44,828
South Carolina 13 1788 8 32,008
Utah 1896 45 84,904
Wisconsin 1848 30 65,499
Table 12.16
Exercise 12.13.15 (Solution on p. 573.)
We are interested in whether or not the number of letters in a state name depends upon the year
the state entered the Union.
a. Decide which variable should be the independent variable and which should be the dependent
variable.
b. Make a scatter plot of the data.
c. Does it appear from inspection that there is a relationship between the variables? Why or why
not?
d. Calculate the least squares line. Put the equation in the form of:
^
y
= a +bx
e. Find the correlation coefcient. What does it imply about the signicance of the relationship?
557
f. Find the estimated number of letters (to the nearest integer) a state would have if it entered
the Union in 1900. Find the estimated number of letters a state would have if it entered the
Union in 1940.
g. Use the two points in (f) to plot the least squares line on your graph from (b).
h. Does it appear that a line is the best way to t the data? Why or why not?
i. Use the least squares line to estimate the number of letters a new state that enters the Union this
year would have. Can the least squares line be used to predict it? Why or why not?
Exercise 12.13.16
We are interested in whether there is a relationship between the ranking of a state and the area of
the state.
a. Let rank be the independent variable and area be the dependent variable.
b. What do you think the scatter plot will look like? Make a scatter plot of the data.
c. Does it appear from inspection that there is a relationship between the variables? Why or why
not?
d. Calculate the least squares line. Put the equation in the form of:
^
y
= a +bx
e. Find the correlation coefcient. What does it imply about the signicance of the relationship?
f. Find the estimated areas for Alabama and for Colorado. Are they close to the actual areas?
g. Use the two points in (f) to plot the least squares line on your graph from (b).
h. Does it appear that a line is the best way to t the data? Why or why not?
i. Are there any outliers?
j. Use the least squares line to estimate the area of a new state that enters the Union. Can the least
squares line be used to predict it? Why or why not?
k. Delete Hawaii and substitute Alaska for it. Alaska is the fortieth state with an area of
656,424 square miles.
l. Calculate the new least squares line.
m. Find the estimated area for Alabama. Is it closer to the actual area with this new least squares
line or with the previous one that included Hawaii? Why do you think thats the case?
n. Do you think that, in general, newer states are larger than the original states?
Exercise 12.13.17 (Solution on p. 573.)
We are interested in whether there is a relationship between the rank of a state and the year it
entered the Union.
a. Let year be the independent variable and rank be the dependent variable.
b. What do you think the scatter plot will look like? Make a scatter plot of the data.
c. Why must the relationship be positive between the variables?
d. Calculate the least squares line. Put the equation in the form of:
^
y
= a +bx
e. Find the correlation coefcient. What does it imply about the signicance of the relationship?
f. Lets say a fty-rst state entered the union. Based upon the least squares line, when should
that have occurred?
g. Using the least squares line, how many states do we currently have?
h. Why isnt the least squares line a good estimator for this year?
Exercise 12.13.18
Below are the percents of the U.S. labor force (excluding self-employed and unemployed ) that
are members of a union. We are interested in whether the decrease is signicant. (Source: Bureau
of Labor Statistics, U.S. Dept. of Labor)
558 CHAPTER 12. LINEAR REGRESSION AND CORRELATION
Year Percent
1945 35.5
1950 31.5
1960 31.4
1970 27.3
1980 21.9
1993 15.8
2011 11.8
Table 12.17
a. Let year be the independent variable and percent be the dependent variable.
b. What do you think the scatter plot will look like? Make a scatter plot of the data.
c. Why will the relationship between the variables be negative?
d. Calculate the least squares line. Put the equation in the form of:
^
y
= a +bx
e. Find the correlation coefcient. What does it imply about the signicance of the relationship?
f. Based on your answer to (e), do you think that the relationship can be said to be decreasing?
g. If the trend continues, when will there no longer be any union members? Do you think that
will happen?
The next two questions refer to the following information: The data below reects the 1991-92 Reunion
Class Giving. (Source: SUNY Albany alumni magazine)
Class Year Average Gift Total Giving
1922 41.67 125
1927 60.75 1,215
1932 83.82 3,772
1937 87.84 5,710
1947 88.27 6,003
1952 76.14 5,254
1957 52.29 4,393
1962 57.80 4,451
1972 42.68 18,093
1976 49.39 22,473
1981 46.87 20,997
1986 37.03 12,590
Table 12.18
Exercise 12.13.19 (Solution on p. 573.)
We will use the columns class year and total giving for all questions, unless otherwise stated.
559
a. What do you think the scatter plot will look like? Make a scatter plot of the data.
b. Calculate the least squares line. Put the equation in the form of:
^
y
= a +bx
c. Find the correlation coefcient. What does it imply about the signicance of the relationship?
d. For the class of 1930, predict the total class gift.
e. For the class of 1964, predict the total class gift.
f. For the class of 1850, predict the total class gift. Why doesnt this value make any sense?
Exercise 12.13.20
We will use the columns class year and average gift for all questions, unless otherwise stated.
a. What do you think the scatter plot will look like? Make a scatter plot of the data.
b. Calculate the least squares line. Put the equation in the form of:
^
y
= a +bx
c. Find the correlation coefcient. What does it imply about the signicance of the relationship?
d. For the class of 1930, predict the average class gift.
e. For the class of 1964, predict the average class gift.
f. For the class of 2010, predict the average class gift. Why doesnt this value make any sense?
Exercise 12.13.21 (Solution on p. 573.)
We are interested in exploring the relationship between the weight of a vehicle and its fuel ef-
ciency (gasoline mileage). The data in the table show the weights, in pounds, and fuel efciency,
measured in miles per gallon, for a sample of 12 vehicles.
Weight Fuel Efciency
2715 24
2570 28
2610 29
2750 38
3000 25
3410 22
3640 20
3700 26
3880 21
3900 18
4060 18
4710 15
Table 12.19
a. Graph a scatterplot of the data.
b. Find the correlation coefcient and determine if it is signicant.
c. Find the equation of the best t line.
d. Write the sentence that interprets the meaning of the slope of the line in the context of the data.
e. What percent of the variation in fuel efciency is explained by the variation in the weight of the
vehicles, using the regression line? (State your answer in a complete sentence in the context
of the data.)
560 CHAPTER 12. LINEAR REGRESSION AND CORRELATION
f. Accurately graph the best t line on your scatterplot.
g. For the vehicle that weights 3000 pounds, nd the residual (y-yhat). Does the value predicted
by the line underestimate or overestimate the observed data value?
h. Identify any outliers, using either the graphical or numerical procedure demonstrated in the
textbook.
i. The outlier is a hybrid car that runs on gasoline and electric technology, but all other vehicles
in the sample have engines that use gasoline only. Explain why it would be appropriate to
remove the outlier from the data in this situation. Remove the outlier from the sample data.
Find the new correlation coefcient, coefcient of determination, and best t line.
j. Compare the correlation coefcients and coefcients of determination before and after removing
the outlier, and explain in complete sentences what these numbers indicate about how the
model has changed.
Exercise 12.13.22 (Solution on p. 573.)
The four data sets below were created by statistician Francis Anscomb. They show why it is im-
portant to examine the scatterplots for your data, in addition to nding the correlation coefcient,
in order to evaluate the appropriateness of tting a linear model.
Set 1 Set 2 Set 3 Set 4
x y x y x y x y
10 8.04 10 9.14 10 7.46 8 6.58
8 6.95 8 8.14 8 6.77 8 5.76
13 7.58 13 8.74 13 12.74 8 7.71
9 8.81 9 8.77 9 7.11 8 8.84
11 8.33 11 9.26 11 7.81 8 8.47
14 9.96 14 8.10 14 8.84 8 7.04
6 7.24 6 6.13 6 6.08 8 5.25
4 4.26 4 3.10 4 5.39 19 12.50
12 10.84 12 9.13 12 8.15 8 5.56
7 4.82 7 7.26 7 6.42 8 7.91
5 5.68 5 4.74 5 5.73 8 6.89
Table 12.20
a. For each data set, nd the least squares regression line and the correlation coefcient. What did
you discover about the lines and values of r?
For each data set, create a scatter plot and graph the least squares regression line. Use the graphs
to answer the following questions:
b. For which data set does it appear that a curve would be a more appropriate model than a line?
c. Which data set has an inuential point (point close to or on the line that greatly inuences the
best t line)?
d. Which data set has an outlier (obviously visible on the scatter plot with best t line graphed)?
e. Which data set appears to be the most appropriate to model using the least squares regression
line?
561
12.13.1 Try these multiple choice questions
Exercise 12.13.23 (Solution on p. 574.)
A correlation coefcient of -0.95 means there is a ____________ between the two variables.
A. Strong positive correlation
B. Weak negative correlation
C. Strong negative correlation
D. No Correlation
Exercise 12.13.24 (Solution on p. 574.)
According to the data reported by the New York State Department of Health regarding West Nile
Virus (http://www.health.state.ny.us/nysdoh/westnile/update/update.htm) for the years 2000-
2008, the least squares line equation for the number of reported dead birds (x) versus the number
of human West Nile virus cases (y) is
^
y
= 10.2638 +0.0491x. If the number of dead birds reported
in a year is 732, how many human cases of West Nile virus can be expected? r = 0.5490
A. No prediction can be made.
B. 19.6
C. 15
D. 38.1
The next three questions refer to the following data: (showing the number of hurricanes by category to
directly strike the mainland U.S. each decade) obtained from www.nhc.noaa.gov/gifs/table6.gif
15
A major
hurricane is one with a strength rating of 3, 4 or 5.
Decade Total Number of Hurricanes Number of Major Hurricanes
1941-1950 24 10
1951-1960 17 8
1961-1970 14 6
1971-1980 12 4
1981-1990 15 5
1991-2000 14 5
2001 2004 9 3
Table 12.21
Exercise 12.13.25 (Solution on p. 574.)
Using only completed decades (1941 2000), calculate the least squares line for the number of
major hurricanes expected based upon the total number of hurricanes.
A.
^
y
= 1.67x +0.5
B.
^
y
= 0.5x 1.67
C.
^
y
= 0.94x 1.67
D.
^
y
= 2x +1
15
http://www.nhc.noaa.gov/gifs/table6.gif
562 CHAPTER 12. LINEAR REGRESSION AND CORRELATION
Exercise 12.13.26 (Solution on p. 574.)
The correlation coefcient is 0.942. Is this considered signicant? Why or why not?
A. No, because 0.942 is greater than the critical value of 0.707
B. Yes, because 0.942 is greater than the critical value of 0.707
C. No, because 0942 is greater than the critical value of 0.811
D. Yes, because 0.942 is greater than the critical value of 0.811
Exercise 12.13.27 (Solution on p. 574.)
The data for 2001-2004 show 9 hurricanes have hit the mainland United States. The line of best t
predicts 2.83 major hurricanes to hit mainland U.S. Can the least squares line be used to make this
prediction?
A. No, because 9 lies outside the independent variable values
B. Yes, because, in fact, there have been 3 major hurricanes this decade
C. No, because 2.83 lies outside the dependent variable values
D. Yes, because how else could we predict what is going to happen this decade.
**Exercises 21 and 22 contributed by Roberta Bloom
563
12.14 Lab 1: Regression (Distance from School)
16
Class Time:
Names:
12.14.1 Student Learning Outcomes:
The student will calculate and construct the line of best t between two variables.
The student will evaluate the relationship between two variables to determine if that relationship is
signicant.
12.14.2 Collect the Data
Use 8 members of your class for the sample. Collect bivariate data (distance an individual lives fromschool,
the cost of supplies for the current term).
1. Complete the table.
Distance from school Cost of supplies this term
Table 12.22
2. Which variable should be the dependent variable and which should be the independent variable?
Why?
3. Graph distance vs. cost. Plot the points on the graph. Label both axes with words. Scale both
axes.
16
This content is available online at <http://cnx.org/content/m17080/1.11/>.
564 CHAPTER 12. LINEAR REGRESSION AND CORRELATION
Figure 12.18
12.14.3 Analyze the Data
Enter your data into your calculator or computer. Write the linear equation below, rounding to 4 decimal
places.
1. Calculate the following:
a. a =
b. b=
c. correlation =
d. n =
e. equation:
^
y
=
f. Is the correlation signicant? Why or why not? (Answer in 1-3 complete sentences.)
2. Supply an answer for the following senarios:
a. For a person who lives 8 miles from campus, predict the total cost of supplies this term:
b. For a person who lives 80 miles from campus, predict the total cost of supplies this term:
3. Obtain the graph on your calculator or computer. Sketch the regression line below.
565
Figure 12.19
12.14.4 Discussion Questions
1. Answer each with 1-3 complete sentences.
a. Does the line seem to t the data? Why?
b. What does the correlation imply about the relationship between the distance and the cost?
2. Are there any outliers? If so, which point is an outlier?
3. Should the outlier, if it exists, be removed? Why or why not?
566 CHAPTER 12. LINEAR REGRESSION AND CORRELATION
12.15 Lab 2: Regression (Textbook Cost)
17
Class Time:
Names:
12.15.1 Student Learning Outcomes:
The student will calculate and construct the line of best t between two variables.
The student will evaluate the relationship between two variables to determine if that relationship is
signicant.
12.15.2 Collect the Data
Survey 10 textbooks. Collect bivariate data (number of pages in a textbook, the cost of the textbook).
1. Complete the table.
Number of pages Cost of textbook
Table 12.23
2. Which variable should be the dependent variable and which should be the independent variable?
Why?
3. Graph distance vs. cost. Plot the points on the graph in "Analyze the Data". Label both axes with
words. Scale both axes.
12.15.3 Analyze the Data
Enter your data into your calculator or computer. Write the linear equation below, rounding to 4 decimal
places.
1. Calculate the following:
a. a =
b. b =
c. correlation =
d. n =
17
This content is available online at <http://cnx.org/content/m17087/1.9/>.
567
e. equation: y =
f. Is the correlation signicant? Why or why not? (Answer in 1-3 complete sentences.)
2. Supply an answer for the following senarios:
a. For a textbook with 400 pages, predict the cost:
b. For a textbook with 600 pages, predict the cost:
3. Obtain the graph on your calculator or computer. Sketch the regression line below.
Figure 12.20
12.15.4 Discussion Questions
1. Answer each with 1-3 complete sentences.
a. Does the line seem to t the data? Why?
b. What does the correlation imply about the relationship between the number of pages and the cost?
2. Are there any outliers? If so, which point(s) is an outlier?
3. Should the outlier, if it exists, be removed? Why or why not?
568 CHAPTER 12. LINEAR REGRESSION AND CORRELATION
12.16 Lab 3: Regression (Fuel Efciency)
18
Class Time:
Names:
12.16.1 Student Learning Outcomes:
The student will calculate and construct the line of best t between two variables.
The student will evaluate the relationship between two variables to determine if that relationship is
signicant.
12.16.2 Collect the Data
Use the most recent April issue of Consumer Reports. It will give the total fuel efciency (in miles per
gallon) and weight (in pounds) of new model cars with automatic transmissions. We will use this data to
determine the relationship, if any, between the fuel efciency of a car and its weight.
1. Which variable should be the independent variable and which should be the dependent variable?
Explain your answer in one or two complete sentences.
2. Using your random number generator, randomly select 20 cars from the list and record their weights
and fuel efciency into the table below.
Weight Fuel Efciency
18
This content is available online at <http://cnx.org/content/m17079/1.8/>.
569
Table 12.24
3. Which variable should be the dependent variable and which should be the independent variable?
Why?
4. By hand, do a scatterplot of weight vs. fuel efciency. Plot the points on graph paper. Label both
axes with words. Scale both axes accurately.
Figure 12.21
12.16.3 Analyze the Data
Enter your data into your calculator or computer. Write the linear equation below, rounding to 4 decimal
places.
1. Calculate the following:
a. a =
b. b =
c. correlation =
d. n =
e. equation:
^
y
=
570 CHAPTER 12. LINEAR REGRESSION AND CORRELATION
2. Obtain the graph of the regression line on your calculator. Sketch the regression line on the same axes
as your scatterplot.
12.16.4 Discussion Questions
1. Is the correlation signicant? Explain how you determined this in complete sentences.
2. Is the relationship a positive one or a negative one? Explain how you can tell and what this means in
terms of weight and fuel efciency.
3. In one or two complete sentences, what is the practical interpretation of the slope of the least squares
line in terms of fuel efciency and weight?
4. For a car that weighs 4000 pounds, predict its fuel efciency. Include units.
5. Can we predict the fuel efciency of a car that weighs 10000 pounds using the least squares line?
Explain why or why not.
6. Questions. Answer each in 1 to 3 complete sentences.
a. Does the line seem to t the data? Why or why not?
b. What does the correlation imply about the relationship between fuel efciency and weight of a
car? Is this what you expected?
7. Are there any outliers? If so, which point is an outlier?
** This lab was designed and contributed by Diane Mathios.
571
Solutions to Exercises in Chapter 12
Solution to Example 12.11, Problem 2 (p. 533)
The x values in the data are between 65 and 75. 90 is outside of the domain of the observed x values in
the data (independent variable), so you cannot reliably predict the nal exam score for this student. (Even
though it is possible to enter x into the equation and calculate a y value, you should not do so!)
To really understand how unreliable the prediction can be outside of the observed x values in the
data, make the substitution x = 90 into the equation.
^
y
= 173.51 +4.83 (90) = 261.19
The nal exam score is predicted to be 261.19. The largest the nal exam score can be is 200.
NOTE: The process of predicting inside of the observed x values in the data is called interpolation.
The process of predicting outside of the observed x values in the data is called extrapolation.
Solutions to Practice: Linear Regression
Solution to Exercise 12.12.2 (p. 545)
a. a = -3,448,225
b. b = 1750
c. corr. = 0.4526
d. n = 22
Solution to Exercise 12.12.3 (p. 545)
^
y
= -3,448,225 +1750x
Solution to Exercise 12.12.4 (p. 545)
a. 25,525
b. 34,275
Solution to Exercise 12.12.10 (p. 546)
a. -725
Solutions to Homework
Solution to Exercise 12.13.1 (p. 547)
a. Independent: Age; Dependent: Fatalities
d. Independent: Power Consumption; Dependent: Utility
Solution to Exercise 12.13.3 (p. 548)
b.
^
y
= 88.7206 0.0432x
c. -0.8533, Yes
g. No
h. No.
i. 2.93, Yes
j. slope = -0.0432. As the year increases by one, the welfare family size tends to decrease by 0.0432 people.
572 CHAPTER 12. LINEAR REGRESSION AND CORRELATION
Solution to Exercise 12.13.5 (p. 548)
b. Yes
c.
^
y
= 102.4287 +11.7585x
d. 0.9436; yes
e. 478.70 feet; 1207.73 feet
g. Yes
h. Yes; (57, 1050)
i. 172.98; No
j. 11.7585 feet
k. slope = 11.7585. As the number of stories increases by one, the height of the building tends to increase
by 11.7585 feet.
Solution to Exercise 12.13.7 (p. 550)
b. Yes
c.
^
y
= 266.8863 +0.1656x
d. 0.9448; Yes
e. 62.8233; 62.3265
h. yes; (1987, 62.7)
i. 72.5937; No
j. slope = 0.1656. As the year increases by one, the percent of workers paid hourly rates tends to increase
by 0.1656.
Solution to Exercise 12.13.9 (p. 552)
b. Yes
c.
^
y
= 3.5984 +0.0371x
d. 0.9986; Yes
e. $5.08
f. $6.93
i. No
j. Not valid
k. slope = 0.0371. As the number of ounces increases by one, the cost of liquid detergent tends to increase
by $0.0371 or is predicted to increase by $0.0371 (about 4 cents).
Solution to Exercise 12.13.11 (p. 553)
c. Yes
d.
^
y
= 337, 424.6478 +0.5463x
e. 0.9964; Yes
f. $208,875.35; $1,028,325.35
h. Yes
i. No
k. slope = 0.5463. As the net taxable estate increases by one dollar, the approximate probate fees and taxes
tend to increase by 0.5463 dollars (about 55 cents).
Solution to Exercise 12.13.13 (p. 554)
c. Yes
d.
^
y
= 65.0876 +7.0948x
e. 0.9761; yes
f. 72.2 cm; 143.13 cm
573
h. Yes
i. No
j. 505.0 cm; No
k. slope = 7.0948. As the age of an American boy increases by one year, the average height tends to increase
by 7.0948 cm.
Solution to Exercise 12.13.15 (p. 556)
c. No
d.
^
y
= 47.03 0.0216x
e. -0.4280
f. 6; 5
Solution to Exercise 12.13.17 (p. 557)
d.
^
y
= 480.5845 +0.2748x
e. 0.9553
f. 1934
Solution to Exercise 12.13.19 (p. 558)
b.
^
y
= 569, 770.2796 +296.0351x
c. 0.8302
d. $1577.46
e. $11,642.66
f. -$22,105.34
Solution to Exercise 12.13.21 (p. 559)
b. r = -0.8, signicant
c. yhat = 48.4-0.00725x
d. For every one pound increase in weight, the fuel efciency tends to decrease (or is predicted to decrease)
by 0.00725 miles per gallon. (For every one thousand pounds increase in weight, the fuel efciency
tends to decrease by 7.25 miles per gallon.)
e. 64% of the variation in fuel efciency is explained by the variation in weight using the regression line.
g. yhat=48.4-0.00725(3000)=26.65 mpg. y-yhat=25-26.65=-1.65. Because yhat=26.5 is greater than y=25, the
line overestimates the observed fuel efciency.
h. (2750,38) is the outlier. Be sure you know how to justify it using the requested graphical or numerical
methods, not just by guessing.
i. yhat = 42.4-0.00578x
j. Without outlier, r=-0.885, rsquare=0.76; with outlier, r=-0.8, rsquare=0.64. The new linear model is a
better t, after the outlier is removed from the data, because the new correlation coefcient is farther
from 0 and the new coefcient of determination is larger.
Solution to Exercise 12.13.22 (p. 560)
a. All four data sets have the same correlation coefcient r=0.816 and the same least squares regression line
yhat=3+0.5x
b. Set 2 ; c. Set 4 ; d. Set 3 ; e. Set 1
574 CHAPTER 12. LINEAR REGRESSION AND CORRELATION
Figure 12.22
Solution to Exercise 12.13.23 (p. 561)
C
Solution to Exercise 12.13.24 (p. 561)
A
Solution to Exercise 12.13.25 (p. 561)
B
Solution to Exercise 12.13.26 (p. 562)
D
Solution to Exercise 12.13.27 (p. 562)
A
markets
|e |rst|tut|crs t|.t b.|rg
tcget|e. buye.s .rJ se||e.s.
| A | | | 3
Demand and Supply
S7AR7 UP: CRA7Y IOR COII
St..bucks c|ee cnp.ry .evc|ut|cr|.eJ t|e cc|eeJ.|rk|rg |.b|ts c| n||||crs c| Ane.|c.rs. St..bucks, .|cse
b.|g|t g.eer.rJ.||te |cgc |s .|ncst .s |.n|||.. .s t|e gc|Jer ..c|es c| Vc|cr.|Js, beg.r |r Se.tt|e |r 191. |||
teer ye..s |.te. |t |.J g.c.r |rtc . c|.|r c| |cu. stc.es |r t|e Se.tt|e ..e.. |er |r 198 |c...J Sc|u|t., . |c.ne.
St..bucks enp|cyee, .|c |.J beccne er.nc.eJ .|t| t|e cu|tu.e c| |t.||.r cc|ee b..s Ju.|rg . t.|p tc |t.|y,
bcug|t t|e ccnp.ry |.cn |ts |curJe.s |c. 3.8 n||||cr. |r 2008, Ane.|c.rs .e.e .||||rg|y p.y|rg 3 c. nc.e |c. .
c.ppucc|rc c. . |.tte, .rJ St..bucks |.J g.c.r tc beccne .r |rte.r.t|cr.| c|.|r, .|t| cve. 16,000 stc.es ..curJ
t|e .c.|J.
|e c|.rge |r Ane.|c.r ccrsune.s t.ste |c. cc|ee .rJ t|e p.c|ts ..keJ |r by St..bucks |u.eJ ct|e. ccnp.r
|es tc get |rtc t|e g.ne. |et.||e.s suc| .s Se.tt|es best c|ee .rJ C|c.|. 'e.rs c|ees erte.eJ t|e n..ket, .rJ
tcJ.y t|e.e ..e t|cus.rJs c| cc|ee b..s, c..ts, J.|vet|.cug|s, .rJ k|csks |r Jc.rtc.rs, n.||s, .rJ .|.pc.ts .||
..curJ t|e ccurt.y. |ver Vc|cr.|Js beg.r se|||rg spec|.|ty cc|ees.
but cve. t|e |.st Jec.Je t|e p.|ce c| cc|ee be.rs |.s beer ,u|te vc|.t||e. 'ust .s ccrsune.s .e.e g.c.|rg .c
custcneJ tc t|e|. c.ppucc|rcs .rJ |.ttes, |r 199, t|e p.|ce c| cc|ee be.rs s|ct up. |xcess|ve ..|r .rJ |.bc. st.|kes
|r cc|eeg.c.|rg ..e.s c| Scut| Ane.|c. |.J .eJuceJ t|e supp|y c| cc|ee, |e.J|rg tc . .|se |r |ts p.|ce. |r t|e e..|y
2000s, \|etr.n |ccJeJ t|e n..ket .|t| cc|ee, .rJ t|e p.|ce c| cc|ee be.rs p|unneteJ. Vc.e .ecert|y, .e.t|e.
ccrJ|t|crs |r v..|cus cc|eeg.c.|rg ccurt.|es .eJuceJ supp|y, .rJ t|e p.|ce c| cc|ee be.rs .ert b.ck up.
Markets, t|e |rst|tut|crs t|.t b.|rg tcget|e. buye.s .rJ se||e.s, ..e .|..ys .espcrJ|rg tc everts, suc| .s b.J
|..vests .rJ c|.rg|rg ccrsune. t.stes t|.t .|ect t|e p.|ces .rJ ,u.rt|t|es c| p..t|cu|.. gccJs. |e Jen.rJ |c.
scne gccJs |rc.e.ses, .|||e t|e Jen.rJ |c. ct|e.s Jec.e.ses. |e supp|y c| scne gccJs .|ses, .|||e t|e supp|y c|
ct|e.s |.||s. As suc| everts ur|c|J, p.|ces .Just tc keep n..kets |r b.|.rce. ||s c|.pte. exp|.|rs |c. t|e n..ket
|c.ces c| Jen.rJ .rJ supp|y |rte..ct tc Jete.n|re e,u|||b.|un p.|ces .rJ e,u|||b.|un ,u.rt|t|es c| gccJs .rJ se.
v|ces. \e .||| see |c. p.|ces .rJ ,u.rt|t|es .Just tc c|.rges |r Jen.rJ .rJ supp|y .rJ |c. c|.rges |r p.|ces
se.ve .s s|gr.|s tc buye.s .rJ se||e.s.
|e ncJe| c| Jen.rJ .rJ supp|y t|.t .e s|.|| Jeve|cp |r t||s c|.pte. |s cre c| t|e ncst pc.e.|u| tcc|s |r .||
c| eccrcn|c .r.|ys|s. +cu .||| be us|rg |t t|.cug|cut ycu. stuJy c| eccrcn|cs. \e .||| |.st |cck .t t|e v..|.b|es
t|.t |r|uerce Jen.rJ. |er .e .||| tu.r tc supp|y, .rJ |r.||y .e .||| put Jen.rJ .rJ supp|y tcget|e. tc exp|c.e
|c. t|e ncJe| c| Jen.rJ .rJ supp|y cpe..tes. As .e ex.n|re t|e ncJe|, be.. |r n|rJ t|.t Jen.rJ |s . .ep.es
ert.t|cr c| t|e be|.v|c. c| buye.s .rJ t|.t supp|y |s . .ep.esert.t|cr c| t|e be|.v|c. c| se||e.s. buye.s n.y be ccr
sune.s pu.c|.s|rg g.cce.|es c. p.cJuce.s pu.c|.s|rg |.cr c.e tc n.ke stee|. Se||e.s n.y be |.ns se|||rg c..s c.
|cuse|c|Js se|||rg t|e|. |.bc. se.v|ces. \e s|.|| see t|.t t|e |Je.s c| Jen.rJ .rJ supp|y .pp|y, .|.teve. t|e |Jer
t|ty c| t|e buye.s c. se||e.s .rJ .|.teve. t|e gccJ c. se.v|ce be|rg exc|.rgeJ |r t|e n..ket. |r t||s c|.pte., .e
s|.|| |ccus cr buye.s .rJ se||e.s c| gccJs .rJ se.v|ces.
quantity demanded
|e ,u.rt|ty buye.s ..e
.||||rg .rJ .b|e tc buy c| .
gccJ c. se.v|ce .t . p..t|cu|..
p.|ce Ju.|rg . p..t|cu|..
pe.|cJ, .|| ct|e. t||rgs
urc|.rgeJ.
demand scheduIe
A t.b|e t|.t s|c.s t|e
,u.rt|t|es c| . gccJ c.
se.v|ce Jen.rJeJ .t
J||e.ert p.|ces Ju.|rg .
p..t|cu|.. pe.|cJ, .|| ct|e.
t||rgs urc|.rgeJ.
1. DEMAND
L A R N I N G O 8 1 C 7 I V S
1. Dene the quantity demanded of a good or service and iIIustrate it using a demand scheduIe
and a demand curve.
2. Distinguish between the foIIowing pairs of concepts: demand and quantity demanded, de-
mand scheduIe and demand curve, movement aIong and shift in a demand curve.
3. Identify demand shifters and determine whether a change in a demand shifter causes the de-
mand curve to shift to the right or to the Ieft.
How many pizzas will people eat this yeai: How many doctoi visits will people make: How many
houses will people buy:
Each good oi seivice has its own special chaiacteiistics that deteimine the quantity people aie will-
ing and able to consume. One is the piice of the good oi seivice itself. Othei independent vaiiables that
aie impoitant deteiminants of demand include consumei piefeiences, piices of ielated goods and sei-
vices, income, demogiaphic chaiacteiistics such as population size, and buyei expectations. The num-
bei of pizzas people will puichase, foi example, depends veiy much on whethei they like pizza. It also
depends on the piices foi alteinatives such as hambuigeis oi spaghetti. The numbei of doctoi visits is
likely to vaiy with income-people with highei incomes aie likely to see a doctoi moie often than
people with lowei incomes. The demands foi pizza, foi doctoi visits, and foi housing aie ceitainly
afected by the age distiibution of the population and its size.
While difeient vaiiables play difeient ioles in infuencing the demands foi difeient goods and
seivices, economists pay special attention to one: the piice of the good oi seivice. Given the values of all
the othei vaiiables that afect demand, a highei piice tends to ieduce the quantity people demand, and
a lowei piice tends to inciease it. A medium pizza typically sells foi $3 to $10. Suppose the piice weie
$30. Chances aie, you would buy fewei pizzas at that piice than you do now. Suppose pizzas typically
sold foi $2 each. At that piice, people would be likely to buy moie pizzas than they do now.
We will discuss fist how piice afects the quantity demanded of a good oi seivice and then how
othei vaiiables afect demand.
1.1 Piice and the Demand Cuive
Because people will puichase difeient quantities of a good oi seivice at difeient piices, economists
must be caieful when speaking of the demand" foi something. They have theiefoie developed some
specifc teims foi expiessing the geneial concept of demand.
The quantity demanded of a good oi seivice is the quantity buyeis aie willing and able to buy at
a paiticulai piice duiing a paiticulai peiiod, all othei things unchanged. (As we leained, we can substi-
tute the Latin phiase ceteiis paiibus" foi all othei things unchanged.") Suppose, foi example, that
100,000 movie tickets aie sold each month in a paiticulai town at a piice of $8 pei ticket. That quant-
ity-100,000-is the quantity of movie admissions demanded pei month at a piice of $8. If the piice
weie $12, we would expect the quantity demanded to be less. If it weie $4, we would expect the quant-
ity demanded to be gieatei. The quantity demanded at each piice would be difeient if othei things that
might afect it, such as the population of the town, weie to change. That is why we add the qualifei that
othei things have not changed to the defnition of quantity demanded.
A demand schedule is a table that shows the quantities of a good oi seivice demanded at difei-
ent piices duiing a paiticulai peiiod, all othei things unchanged. To intioduce the concept of a de-
mand schedule, let us considei the demand foi cofee in the United States. We will ignoie difeiences
among types of cofee beans and ioasts, and speak simply of cofee. The table in Figuie 3.1 shows
quantities of cofee that will be demanded each month at piices ianging fiom $9 to $4 pei pound; the
table is a demand schedule. We see that the highei the piice, the lowei the quantity demanded.
58 PRINCIPLS OI CONOMICS
demand curve
A g..p||c.| .ep.esert.t|cr c|
. Jen.rJ sc|eJu|e.
change in quantity
demanded
A ncvenert .|crg .
Jen.rJ cu.ve t|.t .esu|ts
|.cn . c|.rge |r p.|ce.
Iaw of demand
|c. v|.tu.||y .|| gccJs .rJ
se.v|ces, . ||g|e. p.|ce |e.Js
tc . .eJuct|cr |r ,u.rt|ty
Jen.rJeJ .rJ . |c.e. p.|ce
|e.Js tc .r |rc.e.se |r
,u.rt|ty Jen.rJeJ.
II GUR 3. 1 A Demand ScheduIe and a Demand Curve
|e t.b|e |s . Jen.rJ sc|eJu|e, |t s|c.s ,u.rt|t|es c| cc|ee Jen.rJeJ pe. ncrt| |r t|e |r|teJ St.tes .t
p..t|cu|.. p.|ces, .|| ct|e. t||rgs urc|.rgeJ. |ese J.t. ..e t|er p|ctteJ cr t|e Jen.rJ cu.ve. At pc|rt A cr t|e
cu.ve, 25 n||||cr pcurJs c| cc|ee pe. ncrt| ..e Jen.rJeJ .t . p.|ce c| 6 pe. pcurJ. At pc|rt b, 30 n||||cr
pcurJs c| cc|ee pe. ncrt| ..e Jen.rJeJ .t . p.|ce c| 5 pe. pcurJ.
The infoimation given in a demand schedule can be piesented with a demand curve, which is a
giaphical iepiesentation of a demand schedule. A demand cuive thus shows the ielationship between
the piice and quantity demanded of a good oi seivice duiing a paiticulai peiiod, all othei things un-
changed. The demand cuive in Figuie 3.1 shows the piices and quantities of cofee demanded that aie
given in the demand schedule. At point A, foi example, we see that 23 million pounds of cofee pei
month aie demanded at a piice of $6 pei pound. By convention, economists giaph piice on the veitical
axis and quantity on the hoiizontal axis.
Piice alone does not deteimine the quantity of cofee oi any othei good that people buy. To isolate
the efect of changes in piice on the quantity of a good oi seivice demanded, howevei, we show the
quantity demanded at each piice, assuming that those othei vaiiables iemain unchanged. We do the
same thing in diawing a giaph of the ielationship between any two vaiiables; we assume that the values
of othei vaiiables that may afect the vaiiables shown in the giaph (such as income oi population) ie-
main unchanged foi the peiiod undei consideiation.
A change in piice, with no change in any of the othei vaiiables that afect demand, iesults in a
movement along the demand cuive. Foi example, if the piice of cofee falls fiom $6 to $3 pei pound,
consumption iises fiom 23 million pounds to 30 million pounds pei month. That is a movement fiom
point A to point B along the demand cuive in Figuie 3.1. A movement along a demand cuive that ies-
ults fiom a change in piice is called a change in quantity demanded. Note that a change in quantity
demanded is not a change oi shift in the demand cuive; it is a movement along the demand cuive.
The negative slope of the demand cuive in Figuie 3.1 suggests a key behavioial ielationship of eco-
nomics. All othei things unchanged, the law of demand holds that, foi viitually all goods and sei-
vices, a highei piice leads to a ieduction in quantity demanded and a lowei piice leads to an inciease in
quantity demanded.
The law of demand is called a law because the iesults of countless studies aie consistent with it.
Undoubtedly, you have obseived one manifestation of the law. When a stoie fnds itself with an ovei-
stock of some item, such as iunning shoes oi tomatoes, and needs to sell these items quickly, what does
it do: It typically has a sale, expecting that a lowei piice will inciease the quantity demanded. In genei-
al, we expect the law of demand to hold. Given the values of othei vaiiables that infuence demand, a
highei piice ieduces the quantity demanded. A lowei piice incieases the quantity demanded. Demand
cuives, in shoit, slope downwaid.
CHAP7R 3 DMAND AND SUPPLY 59
change in demand
A s|||t |r . Jen.rJ cu.ve.
1.2 Changes in Demand
Of couise, piice alone does not deteimine the quantity of a good oi seivice that people consume.
Cofee consumption, foi example, will be afected by such vaiiables as income and population. Piefei-
ences also play a iole. The stoiy at the beginning of the chaptei illustiates as much. Staibucks tuined
people on" to cofee. We also expect othei piices to afect cofee consumption. People often eat dough-
nuts oi bagels with theii cofee, so a ieduction in the piice of doughnuts oi bagels might induce people
to diink moie cofee. An alteinative to cofee is tea, so a ieduction in the piice of tea might iesult in the
consumption of moie tea and less cofee. Thus, a change in any one of the vaiiables held constant in
constiucting a demand schedule will change the quantities demanded at each piice. The iesult will be a
shift in the entiie demand cuive iathei than a movement along the demand cuive. A shift in a demand
cuive is called a change in demand.
Suppose, foi example, that something happens to inciease the quantity of cofee demanded at each
piice. Seveial events could pioduce such a change: an inciease in incomes, an inciease in population,
oi an inciease in the piice of tea would each be likely to inciease the quantity of cofee demanded at
each piice. Any such change pioduces a new demand schedule. Figuie 3.2 shows such a change in the
demand schedule foi cofee. We see that the quantity of cofee demanded pei month is gieatei at each
piice than befoie. We show that giaphically as a shift in the demand cuive. The oiiginal cuive, labeled
D
1
, shifts to the iight to D
2
. At a piice of $6 pei pound, foi example, the quantity demanded iises fiom
23 million pounds pei month (point A) to 33 million pounds pei month (point A).
II GUR 3. 2 An Increase in Demand
Ar |rc.e.se |r t|e ,u.rt|ty c| . gccJ c. se.v|ce Jen.rJeJ .t e.c| p.|ce |s s|c.r .s .r |rc.e.se |r Jen.rJ. |e.e,
t|e c.|g|r.| Jen.rJ cu.ve |
1
s|||ts tc |
2
. |c|rt A cr |
1
cc..espcrJs tc . p.|ce c| 6 pe. pcurJ .rJ . ,u.rt|ty
Jen.rJeJ c| 25 n||||cr pcurJs c| cc|ee pe. ncrt|. Or t|e re. Jen.rJ cu.ve |
2
, t|e ,u.rt|ty Jen.rJeJ .t t||s
p.|ce .|ses tc 35 n||||cr pcurJs c| cc|ee pe. ncrt| (pc|rt A).
Just as demand can inciease, it can deciease. In the case of cofee, demand might fall as a iesult of
events such as a ieduction in population, a ieduction in the piice of tea, oi a change in piefeiences. Foi
example, a defnitive fnding that the cafeine in cofee contiibutes to heait disease, which is cuiiently
being debated in the scientifc community, could change piefeiences and ieduce the demand foi cofee.
A ieduction in the demand foi cofee is illustiated in Figuie 3.3. The demand schedule shows that
less cofee is demanded at each piice than in Figuie 3.1. The iesult is a shift in demand fiom the oiigin-
al cuive D
1
to D
3
. The quantity of cofee demanded at a piice of $6 pei pound falls fiom 23 million
pounds pei month (point A) to 13 million pounds pei month (point A). Note, again, that a change in
quantity demanded, ceteiis paiibus, iefeis to a movement along the demand cuive, while a change in
demand iefeis to a shift in the demand cuive.
60 PRINCIPLS OI CONOMICS
demand shifter
A v..|.b|e t|.t c.r c|.rge
t|e ,u.rt|ty c| . gccJ c.
se.v|ce Jen.rJeJ .t e.c|
p.|ce.
compIements
.c gccJs |c. .||c| .r
|rc.e.se |r p.|ce c| cre
.eJuces t|e Jen.rJ |c. t|e
ct|e..
substitutes
.c gccJs |c. .||c| .r
|rc.e.se |r p.|ce c| cre
|rc.e.ses t|e Jen.rJ |c. t|e
ct|e..
II GUR 3. 3 A Reduction in Demand
A .eJuct|cr |r Jen.rJ cccu.s .|er t|e ,u.rt|t|es c| . gccJ c. se.v|ce Jen.rJeJ |.|| .t e.c| p.|ce. |e.e, t|e
Jen.rJ sc|eJu|e s|c.s . |c.e. ,u.rt|ty c| cc|ee Jen.rJeJ .t e.c| p.|ce t|.r .e |.J |r ||gu.e 3.1. |e
.eJuct|cr s|||ts t|e Jen.rJ cu.ve |c. cc|ee tc |
3
|.cn |
1
. |e ,u.rt|ty Jen.rJeJ .t . p.|ce c| 6 pe. pcurJ, |c.
ex.np|e, |.||s |.cn 25 n||||cr pcurJs pe. ncrt| (pc|rt A) tc 15 n||||cr pcurJs c| cc|ee pe. ncrt| (pc|rt A).
A vaiiable that can change the quantity of a good oi seivice demanded at each piice is called a de-
mand shifter. When these othei vaiiables change, the all-othei-things-unchanged conditions behind
the oiiginal demand cuive no longei hold. Although difeient goods and seivices will have difeient de-
mand shifteis, the demand shifteis aie likely to include (1) consumei piefeiences, (2) the piices of ie-
lated goods and seivices, (3) income, (4) demogiaphic chaiacteiistics, and (3) buyei expectations. Next
we look at each of these.
Preferences
Changes in piefeiences of buyeis can have impoitant consequences foi demand. We have alieady seen
how Staibucks supposedly incieased the demand foi cofee. Anothei example is ieduced demand foi
cigaiettes caused by concein about the efect of smoking on health. A change in piefeiences that makes
one good oi seivice moie populai will shift the demand cuive to the iight. A change that makes it less
populai will shift the demand cuive to the left.
Prices of ReIated Goods and Services
Suppose the piice of doughnuts weie to fall. Many people who diink cofee enjoy dunking doughnuts
in theii cofee; the lowei piice of doughnuts might theiefoie inciease the demand foi cofee, shifting
the demand cuive foi cofee to the iight. A lowei piice foi tea, howevei, would be likely to ieduce
cofee demand, shifting the demand cuive foi cofee to the left.
In geneial, if a ieduction in the piice of one good incieases the demand foi anothei, the two goods
aie called complements. If a ieduction in the piice of one good ieduces the demand foi anothei, the
two goods aie called substitutes. These defnitions hold in ieveise as well: two goods aie complements
if an inciease in the piice of one ieduces the demand foi the othei, and they aie substitutes if an in-
ciease in the piice of one incieases the demand foi the othei. Doughnuts and cofee aie complements;
tea and cofee aie substitutes.
Complementaiy goods aie goods used in conjunction with one anothei. Tennis iackets and tennis
balls, eggs and bacon, and stationeiy and postage stamps aie complementaiy goods. Substitute goods
aie goods used instead of one anothei. iPODs, foi example, aie likely to be substitutes foi CD playeis.
Bieakfast ceieal is a substitute foi eggs. A fle attachment to an e-mail is a substitute foi both a fax ma-
chine and postage stamps.
CHAP7R 3 DMAND AND SUPPLY 61
normaI good
A gccJ |c. .||c| Jen.rJ
|rc.e.ses .|er |rccne
|rc.e.ses.
inferior good
A gccJ |c. .||c| Jen.rJ
Jec.e.ses .|er |rccne
|rc.e.ses.
II GUR 3. 4
Income
As incomes iise, people inciease theii consumption of many goods and seivices, and as incomes fall,
theii consumption of these goods and seivices falls. Foi example, an inciease in income is likely to iaise
the demand foi gasoline, ski tiips, new cais, and jeweliy. Theie aie, howevei, goods and seivices foi
which consumption falls as income iises-and iises as income falls. As incomes iise, foi example,
people tend to consume moie fiesh fiuit but less canned fiuit.
A good foi which demand incieases when income incieases is called a normal good. A good foi
which demand decieases when income incieases is called an inferior good. An inciease in income
shifts the demand cuive foi fiesh fiuit (a noimal good) to the iight; it shifts the demand cuive foi
canned fiuit (an infeiioi good) to the left.
Demographic Characteristics
The numbei of buyeis afects the total quantity of a good oi seivice that will be bought; in geneial, the
gieatei the population, the gieatei the demand. Othei demogiaphic chaiacteiistics can afect demand
as well. As the shaie of the population ovei age 63 incieases, the demand foi medical seivices, ocean
ciuises, and motoi homes incieases. The biith iate in the United States fell shaiply between 1933 and
1973 but has giadually incieased since then. That inciease has iaised the demand foi such things as in-
fant supplies, elementaiy school teacheis, soccei coaches, in-line skates, and college education. De-
mand can thus shift as a iesult of changes in both the numbei and chaiacteiistics of buyeis.
8uyer xpectations
The consumption of goods that can be easily stoied, oi whose consumption can be postponed, is
stiongly afected by buyei expectations. The expectation of newei TV technologies, such as high-defni-
tion TV, could slow down sales of iegulai TVs. If people expect gasoline piices to iise tomoiiow, they
will fll up theii tanks today to tiy to beat the piice inciease. The same will be tiue foi goods such as
automobiles and washing machines: an expectation of highei piices in the futuie will lead to moie pui-
chases today. If the piice of a good is expected to fall, howevei, people aie likely to ieduce theii pui-
chases today and await tomoiiow's lowei piices. The expectation that computei piices will fall, foi ex-
ample, can ieduce cuiient demand.
62 PRINCIPLS OI CONOMICS
Heads Up!
|t |s c.uc|.| tc J|st|rgu|s| bet.eer . c|.rge |r ,u.rt|ty Jen.rJeJ, .||c| |s . ncvenert .|crg t|e Jen.rJ
cu.ve c.useJ by . c|.rge |r p.|ce, .rJ . c|.rge |r Jen.rJ, .||c| |np||es . s|||t c| t|e Jen.rJ cu.ve |tse||.
A c|.rge |r Jen.rJ |s c.useJ by . c|.rge |r . Jen.rJ s|||te.. Ar |rc.e.se |r Jen.rJ |s . s|||t c| t|e Je
n.rJ cu.ve tc t|e .|g|t. A Jec.e.se |r Jen.rJ |s . s|||t |r t|e Jen.rJ cu.ve tc t|e |e|t. ||s J...|rg c| . Je
n.rJ cu.ve ||g|||g|ts t|e J||e.erce.
k Y 7 A k A W A Y S
< |e ,u.rt|ty Jen.rJeJ c| . gccJ c. se.v|ce |s t|e ,u.rt|ty buye.s ..e .||||rg .rJ .b|e tc buy .t .
p..t|cu|.. p.|ce Ju.|rg . p..t|cu|.. pe.|cJ, .|| ct|e. t||rgs urc|.rgeJ.
< A Jen.rJ sc|eJu|e |s . t.b|e t|.t s|c.s t|e ,u.rt|t|es c| . gccJ c. se.v|ce Jen.rJeJ .t J||e.ert p.|ces
Ju.|rg . p..t|cu|.. pe.|cJ, .|| ct|e. t||rgs urc|.rgeJ.
< A Jen.rJ cu.ve s|c.s g..p||c.||y t|e ,u.rt|t|es c| . gccJ c. se.v|ce Jen.rJeJ .t J||e.ert p.|ces Ju.|rg
. p..t|cu|.. pe.|cJ, .|| ct|e. t||rgs urc|.rgeJ.
< A|| ct|e. t||rgs urc|.rgeJ, t|e |.. c| Jen.rJ |c|Js t|.t, |c. v|.tu.||y .|| gccJs .rJ se.v|ces, . ||g|e.
p.|ce |rJuces . .eJuct|cr |r ,u.rt|ty Jen.rJeJ .rJ . |c.e. p.|ce |rJuces .r |rc.e.se |r ,u.rt|ty
Jen.rJeJ.
< A c|.rge |r t|e p.|ce c| . gccJ c. se.v|ce c.uses . c|.rge |r t|e ,u.rt|ty Jen.rJeJ. ncvenert
o|o t|e Jen.rJ cu.ve.
< A c|.rge |r . Jen.rJ s|||te. c.uses . c|.rge |r Jen.rJ, .||c| |s s|c.r .s . |||| c| t|e Jen.rJ cu.ve.
|en.rJ s|||te.s |rc|uJe p.e|e.erces, t|e p.|ces c| .e|.teJ gccJs .rJ se.v|ces, |rccne, Jencg..p||c
c|...cte.|st|cs, .rJ buye. expect.t|crs.
< .c gccJs ..e subst|tutes || .r |rc.e.se |r t|e p.|ce c| cre c.uses .r |rc.e.se |r t|e Jen.rJ |c. t|e
ct|e.. .c gccJs ..e ccnp|enerts || .r |rc.e.se |r t|e p.|ce c| cre c.uses . Jec.e.se |r t|e Jen.rJ |c.
t|e ct|e..
< A gccJ |s . rc.n.| gccJ || .r |rc.e.se |r |rccne c.uses .r |rc.e.se |r Jen.rJ. A gccJ |s .r |r|e.|c. gccJ
|| .r |rc.e.se |r |rccne c.uses . Jec.e.se |r Jen.rJ.
7 R Y I 7 !
A|| ct|e. t||rgs urc|.rgeJ, .|.t |.ppers tc t|e Jen.rJ cu.ve |c. |\| .ert.|s || t|e.e |s (.) .r |rc.e.se |r
t|e p.|ce c| ncv|e t|e.te. t|ckets, (b) . Jec.e.se |r |.n||y |rccne, c. (c) .r |rc.e.se |r t|e p.|ce c| |\| .ert
.|s` |r .rs.e.|rg t||s .rJ ct|e. .y |t! p.cb|ens |r t||s c|.pte., J... .rJ c..e|u||y |.be| . set c| .xes. Or t|e
|c.|.crt.| .x|s c| ycu. g..p|, s|c. t|e ,u.rt|ty c| |\| .ert.|s. |t |s recess..y tc spec||y t|e t|ne pe.|cJ tc
.||c| ycu. ,u.rt|ty pe.t.|rs (e.g., pe. pe.|cJ, pe. .eek, c. pe. ye..). Or t|e ve.t|c.| .x|s s|c. t|e p.|ce
pe. |\| .ert.|. S|rce ycu Jc rct |.ve spec||c J.t. cr p.|ces .rJ ,u.rt|t|es Jen.rJeJ, n.ke . |.ee|.rJ
J...|rg c| t|e cu.ve c. cu.ves ycu ..e .skeJ tc ex.n|re. |ccus cr t|e gere..| s|.pe .rJ pcs|t|cr c| t|e
cu.ve(s) be|c.e .rJ .|te. everts cccu.. |... re. cu.ve(s) tc s|c. .|.t |.ppers |r e.c| c| t|e c|.cunst.rces
g|ver. |e cu.ves ccu|J s|||t tc t|e |e|t c. tc t|e .|g|t, c. st.y .|e.e t|ey ..e.
CHAP7R 3 DMAND AND SUPPLY 63
Case in Point: SoIving Campus Parking ProbIems Without Adding More Parking
Spaces
2010 jupiterimages Corporation
|r|ess ycu .tterJ . v|.tu.| c.npus, c|.rces ..e ycu |.ve erg.geJ |r nc.e t|.r cre ccrve.s.t|cr .bcut
|c. |..J |t |s tc |rJ . p|.ce tc p..k cr c.npus. |rJeeJ, .ccc.J|rg tc |..k |e.., . |c.ne. p.es|Jert c| t|e
|r|ve.s|ty c| .|||c.r|. systen, . ur|ve.s|ty |s best urJe.stccJ .s . g.cup c| pecp|e |e|J tcget|e. by . ccn
ncr g.|ev.rce cve. p..k|rg.
|e..|y, t|e Jen.rJ |c. c.npus p..k|rg sp.ces |.s g.c.r subst.rt|.||y cve. t|e p.st |e. Jec.Jes. |r su.veys
ccrJucteJ by |.r|e| |errey, ||c..Jc |uncrt, .rJ C|rge. |errey, .|c .c.k |c. t|e c.npus Jes|gr ccn
p.ry S.s.k| .rJ Asscc|.tes, |t ..s |curJ t|.t cut c| 10 stuJerts c.r t|e|. c.r c..s. |ey |.ve |rte.v|e.eJ
n.ry stuJerts .|c ccr|esseJ tc J.|v|rg |.cn t|e|. Jc.n|tc.|es tc c|.sses t|.t .e.e . |ven|rute ..|k ...y,
.rJ t|ey ..gue t|.t t|e Jete.|c..t|cr c| cc||ege erv|.crnerts |s |..ge|y .tt.|but.b|e tc t|e |rc.e.seJ use c|
c..s cr c.npus .rJ t|.t cc||eges ccu|J bette. se.v|ce t|e|. n|ss|crs by rct .JJ|rg nc.e p..k|rg sp.ces.
S|rce |e. ur|ve.s|t|es c|..ge ercug| |c. p..k|rg tc ever ccve. t|e ccst c| bu||J|rg .rJ n.|rt.|r|rg p..k|rg
|cts, t|e .est |s p.|J |c. by .|| stuJerts .s p..t c| tu|t|cr. |e|. .ese..c| s|c.s t|.t |c. eve.y 1,000 p..k|rg
sp.ces, t|e neJ|.r |rst|tut|cr |cses .|ncst 400,000 . ye.. |c. su.|.ce p..k|rg, .rJ nc.e t|.r 1,200,000 |c.
st.uctu..| p..k|rg. |e.. c| . b.ck|.s| |.cn stuJerts .rJ t|e|. p..erts, .s .e|| .s |.cn |.cu|ty .rJ st.|, seens
tc exp|.|r .|y c.npus .Jn|r|st..tc.s Jc rct s|np|y ..|se t|e p.|ce c| p..k|rg cr c.npus.
\|||e |errey .rJ ||s cc||e.gues Jc .Jvcc.te ..|s|rg p..k|rg |ees, || rct .|| .t crce t|er cve. t|ne, t|ey .|sc
suggest scne subt|e., .rJ pe.|.ps pc||t|c.||y nc.e p.|.t.b|e, ne.su.es|r p..t|cu|.., s|||t|rg t|e Jen.rJ |c.
p..k|rg sp.ces tc t|e |e|t by |c.e.|rg t|e p.|ces c| subst|tutes.
.c ex.np|es t|ey rcteJ .e.e .t t|e |r|ve.s|ty c| \.s||rgtcr .rJ t|e |r|ve.s|ty c| c|c..Jc .t bcu|Je.. At
t|e |r|ve.s|ty c| \.s||rgtcr, c.. pcc|e.s n.y p..k |c. |.ee. ||s |rrcv.t|cr |.s .eJuceJ pu.c|.ses c| s|rg|e
cccup.rcy p..k|rg pe.n|ts by 32 cve. . Jec.Je. Accc.J|rg tc |r|ve.s|ty c| \.s||rgtcr .ss|st.rt J|.ectc. c|
t..rspc.t.t|cr se.v|ces |ete. |e.ey, \|t|cut v|gc.cus|y n.r.g|rg cu. p..k|rg .rJ p.cv|J|rg ccnnute. .|
te.r.t|ves, t|e ur|ve.s|ty .cu|J |.ve beer |.ceJ .|t| .JJ|rg .pp.cx|n.te|y 3,600 p..k|rg sp.ces, .t . ccst c|
cve. 100 n||||cr.|e ur|ve.s|ty |.s c.e.teJ cppc.tur|t|es tc n.ke c.p|t.| |rvestnerts |r bu||J|rgs suppc.t
|rg eJuc.t|cr |rste.J c| st.uctu.es |c. c..s. At t|e |r|ve.s|ty c| c|c..Jc, |.ee pub||c t..rs|t |.s |rc.e.seJ use
c| buses .rJ ||g|t ..|| |.cn 300,000 tc 2 n||||cr t.|ps pe. ye.. cve. t|e |.st Jec.Je. |e |rc.e.seJ use c| n.ss
t..rs|t |.s .||c.eJ t|e ur|ve.s|ty tc .vc|J ccrst.uct|rg re..|y 2,000 p..k|rg sp.ces, .||c| |.s s.veJ .bcut
3.6 n||||cr .rru.||y.
.ooce |o|e| | |ee,. |o. |o .o|.e oo |o|| |o|e /|||oo| ^JJ| /oe |o||.' |e |o|c|e o| |||e |Joco||o. /oc| 26.
2004. .ec||o |. |22|23
64 PRINCIPLS OI CONOMICS
quantity suppIied
|e ,u.rt|ty se||e.s ..e
.||||rg tc se|| c| . gccJ c.
se.v|ce .t . p..t|cu|.. p.|ce
Ju.|rg . p..t|cu|.. pe.|cJ, .||
ct|e. t||rgs urc|.rgeJ.
A N S W R 7 O 7 R Y I 7 ! P R O 8 L M
S|rce gc|rg tc t|e ncv|es |s . subst|tute |c. ..tc||rg . |\| .t |cne, .r |rc.e.se |r t|e p.|ce c| gc|rg tc t|e
ncv|es s|cu|J c.use nc.e pecp|e tc s.|tc| |.cn gc|rg tc t|e ncv|es tc st.y|rg .t |cne .rJ .ert|rg |\|s.
|us, t|e Jen.rJ cu.ve |c. |\| .ert.|s .||| s|||t tc t|e .|g|t .|er t|e p.|ce c| ncv|e t|e.te. t|ckets |rc.e.ses
|.re| (.).
A Jec.e.se |r |.n||y |rccne .||| c.use t|e Jen.rJ cu.ve tc s|||t tc t|e |e|t || |\| .ert.|s ..e . rc.n.| gccJ
but tc t|e .|g|t || |\| .ert.|s ..e .r |r|e.|c. gccJ. |e |.tte. n.y be t|e c.se |c. scne |.n|||es, s|rce st.y|rg
.t |cne .rJ ..tc||rg |\|s |s . c|e.pe. |c.n c| erte.t.|rnert t|.r t.k|rg t|e |.n||y tc t|e ncv|es. |c.
ncst ct|e.s, |c.eve., |\| .ert.|s ..e p.cb.b|y . rc.n.| gccJ |.re| (b).
Ar |rc.e.se |r t|e p.|ce c| |\| .ert.|s Jces rct s|||t t|e Jen.rJ cu.ve |c. |\| .ert.|s .t .||, ..t|e., .r |r
c.e.se |r p.|ce, s.y |.cn |
1
tc |
2
, |s . ncvenert up...J tc t|e |e|t .|crg t|e Jen.rJ cu.ve. At . ||g|e. p.|ce,
pecp|e .||| .ert |e.e. |\|s, s.y (
2
|rste.J c| (
1
, cete.|s p..|bus |.re| (c).
2. SUPPLY
L A R N I N G O 8 1 C 7 I V S
1. Dene the quantity suppIied of a good or service and iIIustrate it using a suppIy scheduIe and a
suppIy curve.
2. Distinguish between the foIIowing pairs of concepts: suppIy and quantity suppIied, suppIy
scheduIe and suppIy curve, movement aIong and shift in a suppIy curve.
3. Identify suppIy shifters and determine whether a change in a suppIy shifter causes the suppIy
curve to shift to the right or to the Ieft.
What deteimines the quantity of a good oi seivice selleis aie willing to ofei foi sale: Piice is one
factoi; ceteiis paiibus, a highei piice is likely to induce selleis to ofei a gieatei quantity of a good oi
seivice. Pioduction cost is anothei deteiminant of supply. Vaiiables that afect pioduction cost include
the piices of factois used to pioduce the good oi seivice, ietuins fiom alteinative activities, technology,
the expectations of selleis, and natuial events such as weathei changes. Still anothei factoi afecting the
quantity of a good that will be ofeied foi sale is the numbei of selleis-the gieatei the numbei of
selleis of a paiticulai good oi seivice, the gieatei will be the quantity ofeied at any piice pei time
peiiod.
2.1 Piice and the Supply Cuive
The quantity supplied of a good oi seivice is the quantity selleis aie willing to sell at a paiticulai
piice duiing a paiticulai peiiod, all othei things unchanged. Ceteiis paiibus, the ieceipt of a highei
piice incieases piofts and induces selleis to inciease the quantity they supply.
In geneial, when theie aie many selleis of a good, an inciease in piice iesults in an inciease in
quantity supplied, and this ielationship is often iefeiied to as the law of supply. We will see, though,
thiough oui exploiation of micioeconomics, that theie aie a numbei of exceptions to this ielationship.
Theie aie cases in which a highei piice will not induce an inciease in quantity supplied. Goods that
cannot be pioduced, such as additional land on the coinei of Paik Avenue and 36th Stieet in Manhat-
tan, aie fxed in supply-a highei piice cannot induce an inciease in the quantity supplied. Theie aie
CHAP7R 3 DMAND AND SUPPLY 65
suppIy scheduIe
A t.b|e t|.t s|c.s ,u.rt|t|es
supp||eJ .t J||e.ert p.|ces
Ju.|rg . p..t|cu|.. pe.|cJ, .||
ct|e. t||rgs urc|.rgeJ.
suppIy curve
A g..p||c.| .ep.esert.t|cr c|
. supp|y sc|eJu|e.
change in quantity
suppIied
Vcvenert .|crg t|e supp|y
cu.ve c.useJ by . c|.rge |r
p.|ce.
change in suppIy
A s|||t |r t|e supp|y cu.ve.
even cases, which we investigate in micioeconomic analysis, in which a highei piice induces a ieduc-
tion in the quantity supplied.
Geneially speaking, howevei, when theie aie many selleis of a good, an inciease in piice iesults in
a gieatei quantity supplied. The ielationship between piice and quantity supplied is suggested in a
supply schedule, a table that shows quantities supplied at difeient piices duiing a paiticulai peiiod,
all othei things unchanged. Figuie 3.8 gives a supply schedule foi the quantities of cofee that will be
supplied pei month at vaiious piices, ceteiis paiibus. At a piice of $4 pei pound, foi example, piodu-
ceis aie willing to supply 13 million pounds of cofee pei month. A highei piice, say $6 pei pound, in-
duces selleis to supply a gieatei quantity-23 million pounds of cofee pei month.
II GUR 3. 8 A SuppIy ScheduIe and a SuppIy Curve
|e supp|y sc|eJu|e s|c.s t|e ,u.rt|ty c| cc|ee t|.t .||| be supp||eJ |r t|e |r|teJ St.tes e.c| ncrt| .t
p..t|cu|.. p.|ces, .|| ct|e. t||rgs urc|.rgeJ. |e s.ne |r|c.n.t|cr |s g|ver g..p||c.||y |r t|e supp|y cu.ve. |e
v.|ues g|ver |e.e suggest . pcs|t|ve .e|.t|crs||p bet.eer p.|ce .rJ ,u.rt|ty supp||eJ.
A supply curve is a giaphical iepiesentation of a supply schedule. It shows the ielationship between
piice and quantity supplied duiing a paiticulai peiiod, all othei things unchanged. Because the iela-
tionship between piice and quantity supplied is geneially positive, supply cuives aie geneially upwaid
sloping. The supply cuive foi cofee in Figuie 3.8 shows giaphically the values given in the supply
schedule.
A change in piice causes a movement along the supply cuive; such a movement is called a change
in quantity supplied. As is the case with a change in quantity demanded, a change in quantity sup-
plied does not shift the supply cuive. By defnition, it is a movement along the supply cuive. Foi ex-
ample, if the piice iises fiom $6 pei pound to $7 pei pound, the quantity supplied iises fiom 23 million
pounds pei month to 30 million pounds pei month. That's a movement fiom point A to point B along
the supply cuive in Figuie 3.8.
2.2 Changes in Supply
When we diaw a supply cuive, we assume that othei vaiiables that afect the willingness of selleis to
supply a good oi seivice aie unchanged. It follows that a change in any of those vaiiables will cause a
change in supply, which is a shift in the supply cuive. A change that incieases the quantity of a good
oi seivice supplied at each piice shifts the supply cuive to the iight. Suppose, foi example, that the
piice of feitilizei falls. That will ieduce the cost of pioducing cofee and thus inciease the quantity of
cofee pioduceis will ofei foi sale at each piice. The supply schedule in Figuie 3.9 shows an inciease in
the quantity of cofee supplied at each piice. We show that inciease giaphically as a shift in the supply
cuive fiom S
1
to S
2
. We see that the quantity supplied at each piice incieases by 10 million pounds of
cofee pei month. At point A on the oiiginal supply cuive S
1
, foi example, 23 million pounds of cofee
pei month aie supplied at a piice of $6 pei pound. Aftei the inciease in supply, 33 million pounds pei
month aie supplied at the same piice (point A on cuive S
2
).
66 PRINCIPLS OI CONOMICS
II GUR 3. 9 An Increase in SuppIy
|| t|e.e |s . c|.rge |r supp|y t|.t |rc.e.ses t|e ,u.rt|ty supp||eJ .t e.c| p.|ce, .s |s t|e c.se |r t|e supp|y sc|eJu|e
|e.e, t|e supp|y cu.ve s|||ts tc t|e .|g|t. At . p.|ce c| 6 pe. pcurJ, |c. ex.np|e, t|e ,u.rt|ty supp||eJ .|ses |.cn
t|e p.ev|cus |eve| c| 25 n||||cr pcurJs pe. ncrt| cr supp|y cu.ve .
1
(pc|rt A) tc 35 n||||cr pcurJs pe. ncrt| cr
supp|y cu.ve .
2
(pc|rt A).
An event that ieduces the quantity supplied at each piice shifts the supply cuive to the left. An inciease
in pioduction costs and excessive iain that ieduces the yields fiom cofee plants aie examples of events
that might ieduce supply. Figuie 3.10 shows a ieduction in the supply of cofee. We see in the supply
schedule that the quantity of cofee supplied falls by 10 million pounds of cofee pei month at each
piice. The supply cuive thus shifts fiom S
1
to S
3
.
II GUR 3. 10 A Reduction in SuppIy
A c|.rge |r supp|y t|.t .eJuces t|e ,u.rt|ty supp||eJ .t e.c| p.|ce s|||ts t|e supp|y cu.ve tc t|e |e|t. At . p.|ce c|
6 pe. pcurJ, |c. ex.np|e, t|e c.|g|r.| ,u.rt|ty supp||eJ ..s 25 n||||cr pcurJs c| cc|ee pe. ncrt| (pc|rt A).
\|t| . re. supp|y cu.ve .
3
, t|e ,u.rt|ty supp||eJ .t t|.t p.|ce |.||s tc 15 n||||cr pcurJs c| cc|ee pe. ncrt| (pc|rt
A).
CHAP7R 3 DMAND AND SUPPLY 67
suppIy shifter
A v..|.b|e t|.t c.r c|.rge
t|e ,u.rt|ty c| . gccJ c.
se.v|ce supp||eJ .t e.c| p.|ce.
A vaiiable that can change the quantity of a good oi seivice supplied at each piice is called a supply
shifter. Supply shifteis include (1) piices of factois of pioduction, (2) ietuins fiom alteinative activit-
ies, (3) technology, (4) sellei expectations, (3) natuial events, and (6) the numbei of selleis. When these
othei vaiiables change, the all-othei-things-unchanged conditions behind the oiiginal supply cuive no
longei hold. Let us look at each of the supply shifteis.
Prices of Iactors of Production
A change in the piice of laboi oi some othei factoi of pioduction will change the cost of pioducing any
given quantity of the good oi seivice. This change in the cost of pioduction will change the quantity
that supplieis aie willing to ofei at any piice. An inciease in factoi piices should deciease the quantity
supplieis will ofei at any piice, shifting the supply cuive to the left. A ieduction in factoi piices in-
cieases the quantity supplieis will ofei at any piice, shifting the supply cuive to the iight.
Suppose cofee gioweis must pay a highei wage to the woikeis they hiie to haivest cofee oi must
pay moie foi feitilizei. Such incieases in pioduction cost will cause them to pioduce a smallei quantity
at each piice, shifting the supply cuive foi cofee to the left. A ieduction in any of these costs incieases
supply, shifting the supply cuive to the iight.
Returns from AIternative Activities
To pioduce one good oi seivice means foigoing the pioduction of anothei. The concept of oppoitunity
cost in economics suggests that the value of the activity foigone is the oppoitunity cost of the activity
chosen; this cost should afect supply. Foi example, one oppoitunity cost of pioducing eggs is not
selling chickens. An inciease in the piice people aie willing to pay foi fiesh chicken would make it
moie pioftable to sell chickens and would thus inciease the oppoitunity cost of pioducing eggs. It
would shift the supply cuive foi eggs to the left, iefecting a deciease in supply.
7echnoIogy
A change in technology alteis the combinations of inputs oi the types of inputs iequiied in the pioduc-
tion piocess. An impiovement in technology usually means that fewei and/oi less costly inputs aie
needed. If the cost of pioduction is lowei, the piofts available at a given piice will inciease, and piodu-
ceis will pioduce moie. With moie pioduced at eveiy piice, the supply cuive will shift to the iight,
meaning an inciease in supply.
Impiessive technological changes have occuiied in the computei industiy in iecent yeais. Com-
puteis aie much smallei and aie fai moie poweiful than they weie only a few yeais ago-and they aie
much cheapei to pioduce. The iesult has been a huge inciease in the supply of computeis, shifting the
supply cuive to the iight.
While we usually think of technology as enhancing pioduction, declines in pioduction due to
pioblems in technology aie also possible. Outlawing the use of ceitain equipment without pollution-
contiol devices has incieased the cost of pioduction foi many goods and seivices, theieby ieducing
piofts available at any piice and shifting these supply cuives to the left.
SeIIer xpectations
All supply cuives aie based in pait on sellei expectations about futuie maiket conditions. Many de-
cisions about pioduction and selling aie typically made long befoie a pioduct is ieady foi sale. Those
decisions necessaiily depend on expectations. Changes in sellei expectations can have impoitant efects
on piice and quantity.
Considei, foi example, the owneis of oil deposits. Oil pumped out of the giound and used today
will be unavailable in the futuie. If a change in the inteinational political climate leads many owneis to
expect that oil piices will iise in the futuie, they may decide to leave theii oil in the giound, planning to
sell it latei when the piice is highei. Thus, theie will be a deciease in supply; the supply cuive foi oil
will shift to the left.
NaturaI vents
Stoims, insect infestations, and diought afect agiicultuial pioduction and thus the supply of agiicul-
tuial goods. If something destioys a substantial pait of an agiicultuial ciop, the supply cuive will shift
to the left. The teiiible cyclone that killed moie than 30,000 people in Myanmai in 2008 also destioyed
some of the countiy's piime iice giowing land. That shifted the supply cuive foi iice to the left. If theie
is an unusually good haivest, the supply cuive will shift to the iight.
68 PRINCIPLS OI CONOMICS
7he Number of SeIIers
The supply cuive foi an industiy, such as cofee, includes all the selleis in the industiy. A change in the
numbei of selleis in an industiy changes the quantity available at each piice and thus changes supply.
An inciease in the numbei of selleis supplying a good oi seivice shifts the supply cuive to the iight; a
ieduction in the numbei of selleis shifts the supply cuive to the left.
The maiket foi cellulai phone seivice has been afected by an inciease in the numbei of fims
ofeiing the seivice. Ovei the past decade, new cellulai phone companies emeiged, shifting the supply
cuive foi cellulai phone seivice to the iight.
Heads Up!
|e.e ..e t.c spec|.| t||rgs tc rcte .bcut supp|y cu.ves. |e |.st |s s|n||.. tc t|e |e.Js |p! cr Jen.rJ
cu.ves. |t |s |npc.t.rt tc J|st|rgu|s| c..e|u||y bet.eer c|.rges |r supp|y .rJ c|.rges |r ,u.rt|ty supp||eJ. A
c|.rge |r supp|y .esu|ts |.cn . c|.rge |r . supp|y s|||te. .rJ |np||es . s|||t c| t|e supp|y cu.ve tc t|e .|g|t c.
|e|t. A c|.rge |r p.|ce p.cJuces . c|.rge |r ,u.rt|ty supp||eJ .rJ |rJuces . ncvenert .|crg t|e supp|y
cu.ve. A c|.rge |r p.|ce Jces rct s|||t t|e supp|y cu.ve.
|e seccrJ c.ut|cr .e|.tes tc t|e |rte.p.et.t|cr c| |rc.e.ses .rJ Jec.e.ses |r supp|y. |ct|ce t|.t |r ||gu.e 3.9
.r |rc.e.se |r supp|y |s s|c.r .s . s|||t c| t|e supp|y cu.ve tc t|e .|g|t, t|e cu.ve s|||ts |r t|e J|.ect|cr c| |r
c.e.s|rg ,u.rt|ty .|t| .espect tc t|e |c.|.crt.| .x|s. |r ||gu.e 3.10 . .eJuct|cr |r supp|y |s s|c.r .s . s|||t c|
t|e supp|y cu.ve tc t|e |e|t, t|e cu.ve s|||ts |r t|e J|.ect|cr c| Jec.e.s|rg ,u.rt|ty .|t| .espect tc t|e |c.|
.crt.| .x|s.
bec.use t|e supp|y cu.ve |s up...J s|cp|rg, . s|||t tc t|e .|g|t p.cJuces . re. cu.ve t|.t |r . serse ||es
be|c. t|e c.|g|r.| cu.ve. StuJerts scnet|nes n.ke t|e n|st.ke c| t||rk|rg c| suc| . s|||t .s . s|||t Jc.r
.rJ t|e.e|c.e .s . .eJuct|cr |r supp|y. S|n||..|y, |t |s e.sy tc n.ke t|e n|st.ke c| s|c.|rg .r |rc.e.se |r sup
p|y .|t| . re. cu.ve t|.t ||es .bcve t|e c.|g|r.| cu.ve. but t|.t |s . .eJuct|cr |r supp|y!
c .vc|J suc| e..c.s, |ccus cr t|e |.ct t|.t .r |rc.e.se |r supp|y |s .r |rc.e.se |r t|e ,u.rt|ty supp||eJ .t e.c|
p.|ce .rJ s|||ts t|e supp|y cu.ve |r t|e J|.ect|cr c| |rc.e.seJ ,u.rt|ty cr t|e |c.|.crt.| .x|s. S|n||..|y, . .eJuc
t|cr |r supp|y |s . .eJuct|cr |r t|e ,u.rt|ty supp||eJ .t e.c| p.|ce .rJ s|||ts t|e supp|y cu.ve |r t|e J|.ect|cr
c| . |c.e. ,u.rt|ty cr t|e |c.|.crt.| .x|s.
k Y 7 A k A W A Y S
< |e ,u.rt|ty supp||eJ c| . gccJ c. se.v|ce |s t|e ,u.rt|ty se||e.s ..e .||||rg tc se|| .t . p..t|cu|.. p.|ce
Ju.|rg . p..t|cu|.. pe.|cJ, .|| ct|e. t||rgs urc|.rgeJ.
< A supp|y sc|eJu|e s|c.s t|e ,u.rt|t|es supp||eJ .t J||e.ert p.|ces Ju.|rg . p..t|cu|.. pe.|cJ, .|| ct|e.
t||rgs urc|.rgeJ. A supp|y cu.ve s|c.s t||s s.ne |r|c.n.t|cr g..p||c.||y.
< A c|.rge |r t|e p.|ce c| . gccJ c. se.v|ce c.uses . c|.rge |r t|e ,u.rt|ty supp||eJ. ncvenert o|o
t|e supp|y cu.ve.
< A c|.rge |r . supp|y s|||te. c.uses . c|.rge |r supp|y, .||c| |s s|c.r .s . |||| c| t|e supp|y cu.ve.
Supp|y s|||te.s |rc|uJe p.|ces c| |.ctc.s c| p.cJuct|cr, .etu.rs |.cn .|te.r.t|ve .ct|v|t|es, tec|rc|cgy, se||e.
expect.t|crs, r.tu..| everts, .rJ t|e runbe. c| se||e.s.
< Ar |rc.e.se |r supp|y |s s|c.r .s . s|||t tc t|e .|g|t c| . supp|y cu.ve, . Jec.e.se |r supp|y |s s|c.r .s .
s|||t tc t|e |e|t.
CHAP7R 3 DMAND AND SUPPLY 69
7 R Y I 7 !
|| .|| ct|e. t||rgs ..e urc|.rgeJ, .|.t |.ppers tc t|e supp|y cu.ve |c. |\| .ert.|s || t|e.e |s (.) .r |rc.e.se
|r ..ges p.|J tc |\| .ert.| stc.e c|e.ks, (b) .r |rc.e.se |r t|e p.|ce c| |\| .ert.|s, c. (c) .r |rc.e.se |r t|e
runbe. c| |\| .ert.| stc.es` |... . g..p| t|.t s|c.s .|.t |.ppers tc t|e supp|y cu.ve |r e.c| c|.cun
st.rce. |e supp|y cu.ve c.r s|||t tc t|e |e|t c. tc t|e .|g|t, c. st.y .|e.e |t |s. |enenbe. tc |.be| t|e .xes .rJ
cu.ves, .rJ .enenbe. tc spec||y t|e t|ne pe.|cJ (e.g., |\|s .erteJ pe. .eek).
Case in Point: 7he Monks of St. 8enedict's Get Out of the gg 8usiness
2010 jupiterimages Corporation
|t ..s ccck|es t|.t |u.eJ t|e ncrks c| St. bereJ|cts cut c| t|e egg bus|ress, .rJ rc. p.|v.te .et.e.t spcrsc.
s||p |s |u.|rg t|en ...y |.cn ccck|es.
St. bereJ|cts |s . bereJ|ct|re ncr.ste.y, rest|eJ cr . ..rc| ||g| |r t|e c|c..Jc |cck|es, .bcut 20 n||es
Jc.r t|e .c.J |.cn Asper. |e ncr.ste.ys 15 ncrks cpe..te t|e ..rc| tc suppc.t t|ense|ves .rJ tc
p.cv|Je |e|p |c. pcc. pecp|e |r t|e ..e.. |ey |e.se cut .bcut 3,500 .c.es c| t|e|. |.rJ tc c.tt|e .rJ s|eep
g...e.s, p.cJuce ccck|es, .rJ spcrsc. p.|v.te .et.e.ts. |ey useJ tc p.cJuce eggs.
Att..cteJ by pctert|.| p.c|ts .rJ t|e pe.ce|u| r.tu.e c| t|e .c.k, t|e ncrks .ert |rtc t|e egg bus|ress |r
196. |ey |.J 10,000 c||ckers p.cJuc|rg t|e|. Vcr.ste.y |ggs b..rJ. |c. . .|||e, bus|ress ..s gccJ. \e.y
gccJ. |er, |r t|e |.te 190s, t|e p.|ce c| c||cker |eeJ st..teJ tc .|se ..p|J|y.
\|er .e st..teJ |r t|e bus|ress, .e .e.e p.y|rg 60 tc 80 . tcr |c. |eeJJe||ve.eJ, .ec.||s t|e ncr.s
te.ys .bbct, |.t|e. 'csep| bcy|e. by t|e |.te 190s, cu. ccst |.J nc.e t|.r Jcub|eJ. \e .e.e p.y|rg 160 tc
200 . tcr. |.t .e.||y |u.t, bec.use |eeJ .ep.eserts . |..ge p..t c| t|e ccst c| p.cJuc|rg eggs.
|e ncrks .JusteJ tc t|e b|c.. \|er g..|r p.|ces .e.e |c.e., .eJ pu|| . |er c| |c. . |e. .eeks tc nc|t,
t|er .etu.r |e. tc |.y|rg. A|te. g..|r p.|ces .ert up, |t ..s 12 ncrt|s c| |.y|rg .rJ |rtc t|e scup pct, |.t|e.
'csep| s.ys.
C..|r p.|ces ccrt|rueJ tc .|se |r t|e 1980s .rJ |rc.e.seJ t|e ccsts c| p.cJuct|cr |c. .|| egg p.cJuce.s. |t
c.useJ t|e supp|y c| eggs tc |.||. |en.rJ |e|| .t t|e s.ne t|ne, .s Ane.|c.rs .c..|eJ .bcut t|e c|c|este.c| |r
eggs. |nes gct tcug|e. |r t|e egg bus|ress.
\e .e.e st||| n.k|rg ncrey |r t|e |r.rc|.| serse, |.t|e. 'csep| s.ys. but .e t.|eJ .r expe.|nert |r 1985
p.cJuc|rg ccck|es, .rJ |t ..s . success. \e |r.||y Jec|JeJ t|.t Jevct|rg cu. t|ne .rJ ere.gy tc t|e ccck|es
.cu|J p.y c| bette. t|.r t|e egg bus|ress, sc .e ,u|t t|e egg bus|ress |r 1986.
70 PRINCIPLS OI CONOMICS
|e n.||c.Je. ccck|e bus|ress ..s gccJ tc t|e ncrks. |ey sc|J 200,000 curces c| Vcr.ste.y cck|es |r
198.
by 1998, |c.eve., t|ey |.J ||n|teJ t|e|. p.cJuct|cr c| ccck|es, se|||rg cr|y |cc.||y .rJ tc g||t s|cps. S|rce
2000, t|ey |.ve s.|tc|eJ tc p.cv|J|rg p.|v.te .et.e.ts |c. |rJ|v|Ju.|s .rJ g.cups.bcut 40 pecp|e pe.
ncrt|, .ccc.J|rg tc b.ct|e. |..|es.
|e ncrks c.|cu|.t|cr c| t|e|. cppc.tur|ty ccsts .eve.|eJ t|.t t|ey .cu|J e..r . ||g|e. .etu.r t|.cug| spcr
sc.s||p c| p.|v.te .et.e.ts t|.r |r e|t|e. ccck|es c. eggs. ||s p.cect|cr |.s p.cveJ cc..ect.
ArJ t|e.e |s .rct|e. .Jv.rt.ge .s .e||.
|e c||ckers J|Jrt stcp |.y|rg eggs cr SurJ.y, |.t|e. 'csep| c|uck|es. \|er .e s|||teJ tc ccck|es .e
ccu|J t.ke SurJ.ys c|. \e .e.ert |enneJ |r t|e ..y .e .e.e .|t| t|e c||ckers. |e ncve tc p.cv|J|rg
.et.e.ts |s ever bette. |r t||s .eg..J. S|rce guests p.cv|Je t|e|. c.r ne.|s, ncst c| t|e ncr.ste.ys e|c.t gces
|rtc p|.rr|rg .rJ sc|eJu||rg, .||c| |.ees up ever nc.e c| t|e|. t|ne |c. ct|e. .c.|J|y .s .e|| .s sp|.|tu.|
pu.su|ts.
.ooce |eoo| ||e.|e.
A N S W R 7 O 7 R Y I 7 ! P R O 8 L M
|\| .ert.| stc.e c|e.ks ..e . |.ctc. c| p.cJuct|cr |r t|e |\| .ert.| n..ket. Ar |rc.e.se |r t|e|. ..ges ..|ses
t|e ccst c| p.cJuct|cr, t|e.eby c.us|rg t|e supp|y cu.ve c| |\| .ert.|s tc s|||t tc t|e |e|t |.re| (.). (oo||o.
|t |s pcss|b|e t|.t ycu t|cug|t c| t|e ..ge |rc.e.se .s .r |rc.e.se |r |rccne, . Jen.rJ s|||te., t|.t .cu|J
|e.J tc .r |rc.e.se |r Jen.rJ, but t||s .cu|J be |rcc..ect. |e ,uest|cr .e|e.s cr|y tc ..ges c| |\| .ert.|
stc.e c|e.ks. |ey n.y .ert scne |\|, but t|e|. |np.ct cr tct.| Jen.rJ .cu|J be reg||g|b|e. bes|Jes, .e
|.ve rc |r|c.n.t|cr cr .|.t |.s |.ppereJ cve..|| tc |rccnes c| pecp|e .|c .ert |\|s. \e Jc krc.,
|c.eve., t|.t t|e ccst c| . |.ctc. c| p.cJuct|cr, .||c| |s . supp|y s|||te., |rc.e.seJ.)
Ar |rc.e.se |r t|e p.|ce c| |\| .ert.|s Jces rct s|||t t|e supp|y cu.ve .t .||, ..t|e., |t cc..espcrJs tc . ncve
nert up...J tc t|e .|g|t .|crg t|e supp|y cu.ve. At . ||g|e. p.|ce c| |
2
|rste.J c| |
1
, . g.e.te. ,u.rt|ty c|
|\| .ert.|s, s.y (
2
|rste.J c| (
1
, .||| be supp||eJ |.re| (b).
Ar |rc.e.se |r t|e runbe. c| stc.es .ert|rg |\|s .||| c.use t|e supp|y cu.ve tc s|||t tc t|e .|g|t |.re| (c).
CHAP7R 3 DMAND AND SUPPLY 71
modeI of demand and
suppIy
VcJe| t|.t uses Jen.rJ .rJ
supp|y cu.ves tc exp|.|r t|e
Jete.n|r.t|cr c| p.|ce .rJ
,u.rt|ty |r . n..ket.
equiIibrium price
|e p.|ce .t .||c| ,u.rt|ty
Jen.rJeJ e,u.|s ,u.rt|ty
supp||eJ.
equiIibrium quantity
|e ,u.rt|ty Jen.rJeJ .rJ
supp||eJ .t t|e e,u|||b.|un
p.|ce.
3. DEMAND, SUPPLY, AND EQUILIBRIUM
L A R N I N G O 8 1 C 7 I V S
1. Use demand and suppIy to expIain how equiIibrium price and quantity are determined in a
market.
2. Understand the concepts of surpIuses and shortages and the pressures on price they generate.
3. xpIain the impact of a change in demand or suppIy on equiIibrium price and quantity.
4. xpIain how the circuIar 0ow modeI provides an overview of demand and suppIy in product
and factor markets and how the modeI suggests ways in which these markets are Iinked.
In this section we combine the demand and supply cuives we have just studied into a new model. The
model of demand and supply uses demand and supply cuives to explain the deteimination of piice
and quantity in a maiket.
3.1 The Deteimination of Piice and Quantity
The logic of the model of demand and supply is simple. The demand cuive shows the quantities of a
paiticulai good oi seivice that buyeis will be willing and able to puichase at each piice duiing a spe-
cifed peiiod. The supply cuive shows the quantities that selleis will ofei foi sale at each piice duiing
that same peiiod. By putting the two cuives togethei, we should be able to fnd a piice at which the
quantity buyeis aie willing and able to puichase equals the quantity selleis will ofei foi sale.
Figuie 3.14 combines the demand and supply data intioduced in Figuie 3.1 and Figuie 3.8 Notice
that the two cuives inteisect at a piice of $6 pei pound-at this piice the quantities demanded and sup-
plied aie equal. Buyeis want to puichase, and selleis aie willing to ofei foi sale, 23 million pounds of
cofee pei month. The maiket foi cofee is in equilibiium. Unless the demand oi supply cuive shifts,
theie will be no tendency foi piice to change. The equilibrium price in any maiket is the piice at
which quantity demanded equals quantity supplied. The equilibiium piice in the maiket foi cofee is
thus $6 pei pound. The equilibrium quantity is the quantity demanded and supplied at the equilibii-
um piice.
II GUR 3. 14 7he Determination of quiIibrium Price and Quantity
\|er .e ccnb|re t|e Jen.rJ .rJ supp|y cu.ves |c. . gccJ |r . s|rg|e g..p|, t|e pc|rt .t .||c| t|ey |rte.sect
|Jert||es t|e e,u|||b.|un p.|ce .rJ e,u|||b.|un ,u.rt|ty. |e.e, t|e e,u|||b.|un p.|ce |s 6 pe. pcurJ. crsune.s
Jen.rJ, .rJ supp||e.s supp|y, 25 n||||cr pcurJs c| cc|ee pe. ncrt| .t t||s p.|ce.
With an upwaid-sloping supply cuive and a downwaid-sloping demand cuive, theie is only a single
piice at which the two cuives inteisect. This means theie is only one piice at which equilibiium is
72 PRINCIPLS OI CONOMICS
surpIus
|e .ncurt by .||c| t|e
,u.rt|ty supp||eJ exceeJs
t|e ,u.rt|ty Jen.rJeJ .t
t|e cu..ert p.|ce.
achieved. It follows that at any piice othei than the equilibiium piice, the maiket will not be in equilib-
iium. We next examine what happens at piices othei than the equilibiium piice.
SurpIuses
Figuie 3.13 shows the same demand and supply cuives we have just examined, but this time the initial
piice is $8 pei pound of cofee. Because we no longei have a balance between quantity demanded and
quantity supplied, this piice is not the equilibiium piice. At a piice of $8, we iead ovei to the demand
cuive to deteimine the quantity of cofee consumeis will be willing to buy-13 million pounds pei
month. The supply cuive tells us what selleis will ofei foi sale-33 million pounds pei month. The
difeience, 20 million pounds of cofee pei month, is called a suiplus. Moie geneially, a surplus is the
amount by which the quantity supplied exceeds the quantity demanded at the cuiient piice. Theie is,
of couise, no suiplus at the equilibiium piice; a suiplus occuis only if the cuiient piice exceeds the
equilibiium piice.
II GUR 3. 15 A SurpIus in the Market for Coffee
At . p.|ce c| 8, t|e ,u.rt|ty supp||eJ |s 35 n||||cr pcurJs c| cc|ee pe. ncrt| .rJ t|e ,u.rt|ty Jen.rJeJ |s 15
n||||cr pcurJs pe. ncrt|, t|e.e |s . su.p|us c| 20 n||||cr pcurJs c| cc|ee pe. ncrt|. C|ver . su.p|us, t|e p.|ce
.||| |.|| ,u|ck|y tc...J t|e e,u|||b.|un |eve| c| 6.
A suiplus in the maiket foi cofee will not last long. With unsold cofee on the maiket, selleis will begin
to ieduce theii piices to cleai out unsold cofee. As the piice of cofee begins to fall, the quantity of
cofee supplied begins to decline. At the same time, the quantity of cofee demanded begins to iise. Re-
membei that the ieduction in quantity supplied is a movement along the supply cuive-the cuive itself
does not shift in iesponse to a ieduction in piice. Similaily, the inciease in quantity demanded is a
movement along the demand cuive-the demand cuive does not shift in iesponse to a ieduction in
piice. Piice will continue to fall until it ieaches its equilibiium level, at which the demand and supply
cuives inteisect. At that point, theie will be no tendency foi piice to fall fuithei. In geneial, suipluses in
the maiketplace aie shoit-lived. The piices of most goods and seivices adjust quickly, eliminating the
suiplus. Latei on, we will discuss some maikets in which adjustment of piice to equilibiium may occui
only veiy slowly oi not at all.
CHAP7R 3 DMAND AND SUPPLY 73
shortage
|e .ncurt by .||c| t|e
,u.rt|ty Jen.rJeJ exceeJs
t|e ,u.rt|ty supp||eJ .t t|e
cu..ert p.|ce.
Shortages
Just as a piice above the equilibiium piice will cause a suiplus, a piice below equilibiium will cause a
shoitage. A shortage is the amount by which the quantity demanded exceeds the quantity supplied at
the cuiient piice.
Figuie 3.16 shows a shoitage in the maiket foi cofee. Suppose the piice is $4 pei pound. At that
piice, 13 million pounds of cofee would be supplied pei month, and 33 million pounds would be de-
manded pei month. When moie cofee is demanded than supplied, theie is a shoitage.
II GUR 3. 16 A Shortage in the Market for Coffee
At . p.|ce c| 4 pe. pcurJ, t|e ,u.rt|ty c| cc|ee Jen.rJeJ |s 35 n||||cr pcurJs pe. ncrt| .rJ t|e ,u.rt|ty
supp||eJ |s 15 n||||cr pcurJs pe. ncrt|. |e .esu|t |s . s|c.t.ge c| 20 n||||cr pcurJs c| cc|ee pe. ncrt|.
In the face of a shoitage, selleis aie likely to begin to iaise theii piices. As the piice iises, theie will be
an inciease in the quantity supplied (but not a change in supply) and a ieduction in the quantity de-
manded (but not a change in demand) until the equilibiium piice is achieved.
74 PRINCIPLS OI CONOMICS
3.2 Shifts in Demand and Supply
II GUR 3. 17 Changes in Demand and SuppIy
A c|.rge |r Jen.rJ c. |r supp|y c|.rges t|e e,u|||b.|un sc|ut|cr |r t|e ncJe|. |.re|s (.) .rJ (b) s|c. .r
|rc.e.se .rJ . Jec.e.se |r Jen.rJ, .espect|ve|y, |.re|s (c) .rJ (J) s|c. .r |rc.e.se .rJ . Jec.e.se |r supp|y,
.espect|ve|y.
A change in one of the vaiiables (shifteis) held constant in any model of demand and supply will cieate
a change in demand oi supply. A shift in a demand oi supply cuive changes the equilibiium piice and
equilibiium quantity foi a good oi seivice. Figuie 3.17 combines the infoimation about changes in the
demand and supply of cofee piesented in Figuie 3.2 Figuie 3.3 Figuie 3.9 and Figuie 3.10 In each case,
the oiiginal equilibiium piice is $6 pei pound, and the coiiesponding equilibiium quantity is 23 mil-
lion pounds of cofee pei month. Figuie 3.17 shows what happens with an inciease in demand, a ieduc-
tion in demand, an inciease in supply, and a ieduction in supply. We then look at what happens if both
cuives shift simultaneously. Each of these possibilities is discussed in tuin below.
An Increase in Demand
An inciease in demand foi cofee shifts the demand cuive to the iight, as shown in Panel (a) of Figuie
3.17. The equilibiium piice iises to $7 pei pound. As the piice iises to the new equilibiium level, the
quantity supplied incieases to 30 million pounds of cofee pei month. Notice that the supply cuive does
not shift; iathei, theie is a movement along the supply cuive.
CHAP7R 3 DMAND AND SUPPLY 75
Demand shifteis that could cause an inciease in demand include a shift in piefeiences that leads to
gieatei cofee consumption; a lowei piice foi a complement to cofee, such as doughnuts; a highei piice
foi a substitute foi cofee, such as tea; an inciease in income; and an inciease in population. A change
in buyei expectations, peihaps due to piedictions of bad weathei loweiing expected yields on cofee
plants and incieasing futuie cofee piices, could also inciease cuiient demand.
A Decrease in Demand
Panel (b) of Figuie 3.17 shows that a deciease in demand shifts the demand cuive to the left. The equi-
libiium piice falls to $3 pei pound. As the piice falls to the new equilibiium level, the quantity supplied
decieases to 20 million pounds of cofee pei month.
Demand shifteis that could ieduce the demand foi cofee include a shift in piefeiences that makes
people want to consume less cofee; an inciease in the piice of a complement, such as doughnuts; a ie-
duction in the piice of a substitute, such as tea; a ieduction in income; a ieduction in population; and a
change in buyei expectations that leads people to expect lowei piices foi cofee in the futuie.
An Increase in SuppIy
An inciease in the supply of cofee shifts the supply cuive to the iight, as shown in Panel (c) of Figuie
3.17. The equilibiium piice falls to $3 pei pound. As the piice falls to the new equilibiium level, the
quantity of cofee demanded incieases to 30 million pounds of cofee pei month. Notice that the de-
mand cuive does not shift; iathei, theie is movement along the demand cuive.
Possible supply shifteis that could inciease supply include a ieduction in the piice of an input such
as laboi, a decline in the ietuins available fiom alteinative uses of the inputs that pioduce cofee, an
impiovement in the technology of cofee pioduction, good weathei, and an inciease in the numbei of
cofee-pioducing fims.
A Decrease in SuppIy
Panel (d) of Figuie 3.17 shows that a deciease in supply shifts the supply cuive to the left. The equilibii-
um piice iises to $7 pei pound. As the piice iises to the new equilibiium level, the quantity demanded
decieases to 20 million pounds of cofee pei month.
Possible supply shifteis that could ieduce supply include an inciease in the piices of inputs used in
the pioduction of cofee, an inciease in the ietuins available fiom alteinative uses of these inputs, a de-
cline in pioduction because of pioblems in technology (peihaps caused by a iestiiction on pesticides
used to piotect cofee beans), a ieduction in the numbei of cofee-pioducing fims, oi a natuial event,
such as excessive iain.
Heads Up!
+cu ..e ||ke|y tc be g|ver p.cb|ens |r .||c| ycu .||| |.ve tc s|||t . Jen.rJ c. supp|y cu.ve.
Suppcse ycu ..e tc|J t|.t .r |rv.s|cr c| pcJc.urc||rg |rsects |.s gcbb|eJ up |.|| t|e c.cp c| |.es| pe.s,
.rJ ycu ..e .skeJ tc use Jen.rJ .rJ supp|y .r.|ys|s tc p.eJ|ct .|.t .||| |.pper tc t|e p.|ce .rJ ,u.rt|ty c|
pe.s Jen.rJeJ .rJ supp||eJ. |e.e ..e scne suggest|crs.
|ut t|e ,u.rt|ty c| t|e gccJ ycu ..e .skeJ tc .r.|y.e cr t|e |c.|.crt.| .x|s .rJ |ts p.|ce cr t|e ve.t|c.| .x|s.
|... . Jc.r...Js|cp|rg ||re |c. Jen.rJ .rJ .r up...Js|cp|rg ||re |c. supp|y. |e |r|t|.| e,u|||b.|un p.|ce
|s Jete.n|reJ by t|e |rte.sect|cr c| t|e t.c cu.ves. |.be| t|e e,u|||b.|un sc|ut|cr. +cu n.y |rJ |t |e|p|u| tc
use . runbe. |c. t|e e,u|||b.|un p.|ce |rste.J c| t|e |ette. |. ||ck . p.|ce t|.t seens p|.us|b|e, s.y, 9 pe.
pcurJ. |c rct .c..y .bcut t|e p.ec|se pcs|t|crs c| t|e Jen.rJ .rJ supp|y cu.ves, ycu c.rrct be expecteJ
tc krc. .|.t t|ey ..e.
76 PRINCIPLS OI CONOMICS
Step 2 c.r be t|e ncst J||cu|t step, t|e p.cb|en |s tc Jec|Je .||c| cu.ve tc s|||t. |e key |s tc .enenbe. t|e
J||e.erce bet.eer . c|.rge |r Jen.rJ c. supp|y .rJ . c|.rge |r ,u.rt|ty Jen.rJeJ c. supp||eJ. At e.c|
p.|ce, .sk ycu.se|| .|et|e. t|e g|ver evert .cu|J c|.rge t|e ,u.rt|ty Jen.rJeJ. \cu|J t|e |.ct t|.t . bug
|.s .tt.ckeJ t|e pe. c.cp c|.rge t|e ,u.rt|ty Jen.rJeJ .t . p.|ce c|, s.y, 9 pe. pcurJ` |e..|y rct, rcre
c| t|e Jen.rJ s|||te.s |.ve c|.rgeJ. |e evert .cu|J, |c.eve., .eJuce t|e ,u.rt|ty supp||eJ .t t||s p.|ce,
.rJ t|e supp|y cu.ve .cu|J s|||t tc t|e |e|t. |e.e |s . c|.rge |r supp|y .rJ . .eJuct|cr |r t|e ,u.rt|ty Je
n.rJeJ. |e.e |s rc c|.rge |r Jen.rJ.
|ext c|eck tc see .|et|e. t|e .esu|t ycu |.ve cbt.|reJ n.kes serse. |e g..p| |r Step 2 n.kes serse, |t
s|c.s p.|ce .|s|rg .rJ ,u.rt|ty Jen.rJeJ |.|||rg.
|t |s e.sy tc n.ke . n|st.ke suc| .s t|e cre s|c.r |r t|e t||.J |gu.e c| t||s |e.Js |p! Ore n|g|t, |c. ex
.np|e, .e.scr t|.t .|er |e.e. pe.s ..e .v.||.b|e, |e.e. .||| be Jen.rJeJ, .rJ t|e.e|c.e t|e Jen.rJ cu.ve
.||| s|||t tc t|e |e|t. ||s suggests t|e p.|ce c| pe.s .||| |.||but t|.t Jces rct n.ke serse. || cr|y |.|| .s n.ry
|.es| pe.s .e.e .v.||.b|e, t|e|. p.|ce .cu|J su.e|y .|se. |e e..c. |e.e ||es |r ccr|us|rg . c|.rge |r ,u.rt|ty Je
n.rJeJ .|t| . c|.rge |r Jen.rJ. +es, buye.s .||| erJ up buy|rg |e.e. pe.s. but rc, t|ey .||| rct Jen.rJ
|e.e. pe.s .t e.c| p.|ce t|.r be|c.e, t|e Jen.rJ cu.ve Jces rct s|||t.
SimuItaneous Shifts
As we have seen, when either the demand oi the supply cuive shifts, the iesults aie unambiguous; that
is, we know what will happen to both equilibiium piice and equilibiium quantity, so long as we know
whethei demand oi supply incieased oi decieased. Howevei, in piactice, seveial events may occui at
aiound the same time that cause both the demand and supply cuives to shift. To fguie out what hap-
pens to equilibiium piice and equilibiium quantity, we must know not only in which diiection the de-
mand and supply cuives have shifted but also the ielative amount by which each cuive shifts. Of
couise, the demand and supply cuives could shift in the same diiection oi in opposite diiections, de-
pending on the specifc events causing them to shift.
Foi example, all thiee panels of Figuie 3.19 show a deciease in demand foi cofee (caused peihaps
by a deciease in the piice of a substitute good, such as tea) and a simultaneous deciease in the supply of
cofee (caused peihaps by bad weathei). Since ieductions in demand and supply, consideied sepaiately,
each cause the equilibiium quantity to fall, the impact of both cuives shifting simultaneously to the left
means that the new equilibiium quantity of cofee is less than the old equilibiium quantity. The efect
on the equilibiium piice, though, is ambiguous. Whethei the equilibiium piice is highei, lowei, oi un-
changed depends on the extent to which each cuive shifts.
CHAP7R 3 DMAND AND SUPPLY 77
II GUR 3. 19 SimuItaneous Decreases in Demand and SuppIy
bct| t|e Jen.rJ .rJ t|e supp|y c| cc|ee Jec.e.se. S|rce Jec.e.ses |r Jen.rJ .rJ supp|y, ccrs|Je.eJ sep...te|y,
e.c| c.use e,u|||b.|un ,u.rt|ty tc |.||, t|e |np.ct c| bct| Jec.e.s|rg s|nu|t.recus|y ne.rs t|.t . re.
e,u|||b.|un ,u.rt|ty c| cc|ee nust be |ess t|.r t|e c|J e,u|||b.|un ,u.rt|ty. |r |.re| (.), t|e Jen.rJ cu.ve s|||ts
|..t|e. tc t|e |e|t t|.r Jces t|e supp|y cu.ve, sc e,u|||b.|un p.|ce |.||s. |r |.re| (b), t|e supp|y cu.ve s|||ts |..t|e. tc
t|e |e|t t|.r Jces t|e Jen.rJ cu.ve, sc t|e e,u|||b.|un p.|ce .|ses. |r |.re| (c), bct| cu.ves s|||t tc t|e |e|t by t|e
s.ne .ncurt, sc e,u|||b.|un p.|ce st.ys t|e s.ne.
If the demand cuive shifts faithei to the left than does the supply cuive, as shown in Panel (a) of Figuie
3.19, then the equilibiium piice will be lowei than it was befoie the cuives shifted. In this case the new
equilibiium piice falls fiom $6 pei pound to $3 pei pound. If the shift to the left of the supply cuive is
gieatei than that of the demand cuive, the equilibiium piice will be highei than it was befoie, as shown
in Panel (b). In this case, the new equilibiium piice iises to $7 pei pound. In Panel (c), since both
cuives shift to the left by the same amount, equilibiium piice does not change; it iemains $6 pei
pound.
Regaidless of the scenaiio, changes in equilibiium piice and equilibiium quantity iesulting fiom
two difeient events need to be consideied sepaiately. If both events cause equilibiium piice oi quantity
to move in the same diiection, then cleaily piice oi quantity can be expected to move in that diiection.
If one event causes piice oi quantity to iise while the othei causes it to fall, the extent by which each
cuive shifts is ciitical to fguiing out what happens. Figuie 3.20 summaiizes what may happen to equi-
libiium piice and quantity when demand and supply both shift.
78 PRINCIPLS OI CONOMICS
circuIar 0ow modeI
VcJe| t|.t p.cv|Jes . |cck .t
|c. n..kets .c.k .rJ |c.
t|ey ..e .e|.teJ tc e.c|
ct|e..
II GUR 3. 20 SimuItaneous Shifts in Demand and SuppIy
|| s|nu|t.recus s|||ts |r Jen.rJ .rJ supp|y c.use e,u|||b.|un p.|ce c. ,u.rt|ty tc ncve |r t|e s.ne J|.ect|cr,
t|er e,u|||b.|un p.|ce c. ,u.rt|ty c|e..|y ncves |r t|.t J|.ect|cr. || t|e s|||t |r cre c| t|e cu.ves c.uses e,u|||b.|un
p.|ce c. ,u.rt|ty tc .|se .|||e t|e s|||t |r t|e ct|e. cu.ve c.uses e,u|||b.|un p.|ce c. ,u.rt|ty tc |.||, t|er t|e
.e|.t|ve .ncurt by .||c| e.c| cu.ve s|||ts |s c.|t|c.| tc |gu.|rg cut .|.t |.ppers tc t|.t v..|.b|e.
As demand and supply cuives shift, piices adjust to maintain a balance between the quantity of a good
demanded and the quantity supplied. If piices did not adjust, this balance could not be maintained.
Notice that the demand and supply cuives that we have examined in this chaptei have all been
diawn as lineai. This simplifcation of the ieal woild makes the giaphs a bit easiei to iead without sac-
iifcing the essential point: whethei the cuives aie lineai oi nonlineai, demand cuives aie downwaid
sloping and supply cuives aie geneially upwaid sloping. As ciicumstances that shift the demand cuive
oi the supply cuive change, we can analyze what will happen to piice and what will happen to quantity.
3.3 An Oveiview of Demand and Supply: The Ciiculai Flow Model
Implicit in the concepts of demand and supply is a constant inteiaction and adjustment that econom-
ists illustiate with the ciiculai fow model. The circular ow model piovides a look at how maikets
woik and how they aie ielated to each othei. It shows fows of spending and income thiough the
economy.
A gieat deal of economic activity can be thought of as a piocess of exchange between households
and fims. Fiims supply goods and seivices to households. Households buy these goods and seivices
fiom fims. Households supply factois of pioduction-laboi, capital, and natuial iesouices-that fims
iequiie. The payments fims make in exchange foi these factois iepiesent the incomes households
eain.
The fow of goods and seivices, factois of pioduction, and the payments they geneiate is illustiated
in Figuie 3.21. This ciiculai fow model of the economy shows the inteiaction of households and fims
as they exchange goods and seivices and factois of pioduction. Foi simplicity, the model heie shows
only the piivate domestic economy; it omits the goveinment and foieign sectois.
CHAP7R 3 DMAND AND SUPPLY 79
product markets
V..kets |r .||c| |.ns supp|y
gccJs .rJ se.v|ces
Jen.rJeJ by |cuse|c|Js.
factor markets
V..kets |r .||c| |cuse|c|Js
supp|y |.ctc.s c|
p.cJuct|cr|.bc., c.p|t.|,
.rJ r.tu..|
.escu.cesJen.rJeJ by
|.ns.
II GUR 3. 21 7he CircuIar IIow of conomic Activity
||s s|np|||eJ c|.cu|.. |c. ncJe| s|c.s |c.s c| sperJ|rg bet.eer |cuse|c|Js .rJ |.ns t|.cug| p.cJuct .rJ
|.ctc. n..kets. |e |rre. ...c.s s|c. gccJs .rJ se.v|ces |c.|rg |.cn |.ns tc |cuse|c|Js .rJ |.ctc.s c|
p.cJuct|cr |c.|rg |.cn |cuse|c|Js tc |.ns. |e cute. |c.s s|c. t|e p.ynerts |c. gccJs, se.v|ces, .rJ |.ctc.s
c| p.cJuct|cr. |ese |c.s, |r tu.r, .ep.esert n||||crs c| |rJ|v|Ju.| n..kets |c. p.cJucts .rJ |.ctc.s c| p.cJuct|cr.
The ciiculai fow model shows that goods and seivices that households demand aie supplied by fims
in product markets. The exchange foi goods and seivices is shown in the top half of Figuie 3.21. The
bottom half of the exhibit illustiates the exchanges that take place in factoi maikets. factor markets
aie maikets in which households supply factois of pioduction-laboi, capital, and natuial ie-
souices-demanded by fims.
Oui model is called a ciiculai fow model because households use the income they ieceive fiom
theii supply of factois of pioduction to buy goods and seivices fiom fims. Fiims, in tuin, use the pay-
ments they ieceive fiom households to pay foi theii factois of pioduction.
The demand and supply model developed in this chaptei gives us a basic tool foi undeistanding
what is happening in each of these pioduct oi factoi maikets and also allows us to see how these mai-
kets aie inteiielated. In Figuie 3.21, maikets foi thiee goods and seivices that households want-blue
jeans, haiicuts, and apaitments-cieate demands by fims foi textile woikeis, baibeis, and apaitment
buildings. The equilibiium of supply and demand in each maiket deteimines the piice and quantity of
that item. Moieovei, a change in equilibiium in one maiket will afect equilibiium in ielated maikets.
Foi example, an inciease in the demand foi haiicuts would lead to an inciease in demand foi baibeis.
Equilibiium piice and quantity could iise in both maikets. Foi some puiposes, it will be adequate to
simply look at a single maiket, wheieas at othei times we will want to look at what happens in ielated
maikets as well.
In eithei case, the model of demand and supply is one of the most widely used tools of economic
analysis. That widespiead use is no accident. The model yields iesults that aie, in fact, bioadly consist-
ent with what we obseive in the maiketplace. Youi masteiy of this model will pay big dividends in youi
study of economics.
80 PRINCIPLS OI CONOMICS
k Y 7 A k A W A Y S
< |e e,u|||b.|un p.|ce |s t|e p.|ce .t .||c| t|e ,u.rt|ty Jen.rJeJ e,u.|s t|e ,u.rt|ty supp||eJ. |t |s
Jete.n|reJ by t|e |rte.sect|cr c| t|e Jen.rJ .rJ supp|y cu.ves.
< A su.p|us ex|sts || t|e ,u.rt|ty c| . gccJ c. se.v|ce supp||eJ exceeJs t|e ,u.rt|ty Jen.rJeJ .t t|e cu..ert
p.|ce, |t c.uses Jc.r...J p.essu.e cr p.|ce. A s|c.t.ge ex|sts || t|e ,u.rt|ty c| . gccJ c. se.v|ce
Jen.rJeJ exceeJs t|e ,u.rt|ty supp||eJ .t t|e cu..ert p.|ce, |t c.uses up...J p.essu.e cr p.|ce.
< Ar |rc.e.se |r Jen.rJ, .|| ct|e. t||rgs urc|.rgeJ, .||| c.use t|e e,u|||b.|un p.|ce tc .|se, ,u.rt|ty
supp||eJ .||| |rc.e.se. A Jec.e.se |r Jen.rJ .||| c.use t|e e,u|||b.|un p.|ce tc |.||, ,u.rt|ty supp||eJ .|||
Jec.e.se.
< Ar |rc.e.se |r supp|y, .|| ct|e. t||rgs urc|.rgeJ, .||| c.use t|e e,u|||b.|un p.|ce tc |.||, ,u.rt|ty
Jen.rJeJ .||| |rc.e.se. A Jec.e.se |r supp|y .||| c.use t|e e,u|||b.|un p.|ce tc .|se, ,u.rt|ty Jen.rJeJ
.||| Jec.e.se.
< c Jete.n|re .|.t |.ppers tc e,u|||b.|un p.|ce .rJ e,u|||b.|un ,u.rt|ty .|er bct| t|e supp|y .rJ
Jen.rJ cu.ves s|||t, ycu nust krc. |r .||c| J|.ect|cr e.c| c| t|e cu.ves s|||ts .rJ t|e extert tc .||c|
e.c| cu.ve s|||ts.
< |e c|.cu|.. |c. ncJe| p.cv|Jes .r cve.v|e. c| Jen.rJ .rJ supp|y |r p.cJuct .rJ |.ctc. n..kets .rJ
suggests |c. t|ese n..kets ..e ||rkeJ tc cre .rct|e..
7 R Y I 7 !
\|.t |.ppers tc t|e e,u|||b.|un p.|ce .rJ t|e e,u|||b.|un ,u.rt|ty c| |\| .ert.|s || t|e p.|ce c| ncv|e t|e.t
e. t|ckets |rc.e.ses .rJ ..ges p.|J tc |\| .ert.| stc.e c|e.ks |rc.e.se, .|| ct|e. t||rgs urc|.rgeJ` be su.e tc
s|c. .|| pcss|b|e scer..|cs, .s ..s Jcre |r ||gu.e 3.19. Ag.|r, ycu Jc rct reeJ .ctu.| runbe.s tc ...|ve .t .r
.rs.e.. 'ust |ccus cr t|e gere..| pcs|t|cr c| t|e cu.ve(s) be|c.e .rJ .|te. everts cccu..eJ.
Case in Point: Demand, SuppIy, and Obesity
2010 jupiterimages Corporation
\|y ..e sc n.ry Ane.|c.rs |.t` |ut sc c.uJe|y, t|e ,uest|cr n.y seen .uJe, but, |rJeeJ, t|e runbe. c|
cbese Ane.|c.rs |.s |rc.e.seJ by nc.e t|.r 50 cve. t|e |.st gere..t|cr, .rJ cbes|ty n.y rc. be t|e r.
t|crs runbe. cre |e.|t| p.cb|en. Accc.J|rg tc Stu.n |c|.rJ |r . .ecert |A|| c.pc..t|cr stuJy, Obes|ty
.ppe..s tc |.ve . st.crge. .sscc|.t|cr .|t| t|e cccu..erce c| c|.cr|c neJ|c.| ccrJ|t|crs, .eJuceJ p|ys|c.|
|e.|t|.e|.teJ ,u.||ty c| |||e .rJ |rc.e.seJ |e.|t| c..e .rJ neJ|c.t|cr experJ|tu.es t|.r snck|rg c. p.cb|en
J.|rk|rg.
CHAP7R 3 DMAND AND SUPPLY 81
V.ry exp|.r.t|crs c| .|s|rg cbes|ty suggest ||g|e. Jen.rJ |c. |ccJ. \|.t nc.e .pt p|ctu.e c| cu. seJert..y
|||e sty|e |s t|e.e t|.r sperJ|rg t|e .|te.rccr ..tc||rg . b.||g.ne cr \, .|||e e.t|rg c||ps .rJ s.|s., |c|
|c.eJ by . J|rre. c| . |.v|s||y tcppeJ, t.kecut p|...` ||g|e. |rccne |.s .|sc urJcubteJ|y ccrt.|buteJ tc .
.|g|t...J s|||t |r t|e Jen.rJ cu.ve |c. |ccJ. ||us, .ry .JJ|t|cr.| |ccJ |rt.ke t..rs|.tes |rtc nc.e .e|g|t |r
c.e.se bec.use .e sperJ sc |e. c.|c.|es p.ep..|rg |t, e|t|e. J|.ect|y c. |r t|e p.ccess c| e..r|rg t|e |rccne tc
buy |t. A stuJy by eccrcn|sts |..|us |.kJ...||. .rJ cn.s |||||pscr suggests t|.t .bcut 60 c| t|e .ecert
g.c.t| |r .e|g|t n.y be exp|.|reJ |r t||s ..yt|.t |s, Jen.rJ |.s s|||teJ tc t|e .|g|t, |e.J|rg tc .r |r
c.e.se |r t|e e,u|||b.|un ,u.rt|ty c| |ccJ ccrsuneJ .rJ, g|ver cu. |ess st.erucus |||e sty|es, ever nc.e
.e|g|t g.|r t|.r c.r be exp|.|reJ s|np|y by t|e |rc.e.seJ .ncurt .e ..e e.t|rg.
\|.t .cccurts |c. t|e .en.|r|rg 40 c| t|e .e|g|t g.|r` |.kJ...||. .rJ |||||pscr |u.t|e. .e.scr t|.t . .|g|t
...J s|||t |r Jen.rJ .cu|J by |tse|| |e.J tc .r |rc.e.se |r t|e ,u.rt|ty c| |ccJ .s .e|| .s .r |rc.e.se |r t|e
p.|ce c| |ccJ. |e p.cb|en t|ey |.ve .|t| t||s exp|.r.t|cr |s t|.t cve. t|e pcst\c.|J \.. || pe.|cJ, t|e .e|.t
|ve p.|ce c| |ccJ |.s Jec||reJ by .r .ve..ge c| 0.2 pe.cert.ge pc|rts pe. ye... |ey exp|.|r t|e |.|| |r t|e p.|ce
c| |ccJ by ..gu|rg t|.t .g.|cu|tu..| |rrcv.t|cr |.s |eJ tc . subst.rt|.| .|g|t...J s|||t |r t|e supp|y cu.ve c|
|ccJ. As s|c.r, |c.e. |ccJ p.|ces .rJ . ||g|e. e,u|||b.|un ,u.rt|ty c| |ccJ |.ve .esu|teJ |.cn s|nu|t.recus
.|g|t...J s|||ts |r Jen.rJ .rJ supp|y .rJ t|.t t|e .|g|t...J s|||t |r t|e supp|y c| |ccJ |.cn .
1
tc .
2
|.s
beer subst.rt|.||y |..ge. t|.r t|e .|g|t...J s|||t |r t|e Jen.rJ cu.ve |.cn |
1
tc |
2
.
.ooce |o|oJ. .|o. |e ||ec| o| e||,. .o||. oJ |o|e |||| o |o|c /eJ|co| |o|e oJ |eo||| oe o|.' |eo||| ^|o|.
2002. 2(2: 245253 |o|Jo.o||o. |o|o oJ oo |||||o. |e Oo.|| o| e||, oJ ec|o|o|co| |oe ^ |eoe||co| oJ |||co|
|o|o||o.' |o||oo| |oeoo o| |coo|c |eeoc| /o|| |oe o .S946. /o, 2002
82 PRINCIPLS OI CONOMICS
A N S W R 7 O 7 R Y I 7 ! P R O 8 L M
Ar |rc.e.se |r t|e p.|ce c| ncv|e t|e.te. t|ckets (. subst|tute |c. |\| .ert.|s) .||| c.use t|e Jen.rJ cu.ve |c.
|\| .ert.|s tc s|||t tc t|e .|g|t. Ar |rc.e.se |r t|e ..ges p.|J tc |\| .ert.| stc.e c|e.ks (.r |rc.e.se |r t|e
ccst c| . |.ctc. c| p.cJuct|cr) s|||ts t|e supp|y cu.ve tc t|e |e|t. |.c| evert t.ker sep...te|y c.uses e,u|||b.|
un p.|ce tc .|se. \|et|e. e,u|||b.|un ,u.rt|ty .||| be ||g|e. c. |c.e. JeperJs cr .||c| cu.ve s|||teJ nc.e.
|| t|e Jen.rJ cu.ve s|||teJ nc.e, t|er t|e e,u|||b.|un ,u.rt|ty c| |\| .ert.|s .||| .|se |.re| (.).
|| t|e supp|y cu.ve s|||teJ nc.e, t|er t|e e,u|||b.|un ,u.rt|ty c| |\| .ert.|s .||| |.|| |.re| (b).
|| t|e cu.ves s|||teJ by t|e s.ne .ncurt, t|er t|e e,u|||b.|un ,u.rt|ty c| |\| .ert.|s .cu|J rct c|.rge
|.re| (c).
4. REVIEW AND PRACTICE
Summary
|r t||s c|.pte. .e |.ve ex.n|reJ t|e ncJe| c| Jen.rJ .rJ supp|y. \e |curJ t|.t . Jen.rJ cu.ve s|c.s
t|e ,u.rt|ty Jen.rJeJ .t e.c| p.|ce, .|| ct|e. t||rgs urc|.rgeJ. |e |.. c| Jen.rJ .sse.ts t|.t .r |rc.e.se
|r p.|ce .eJuces t|e ,u.rt|ty Jen.rJeJ .rJ . Jec.e.se |r p.|ce |rc.e.ses t|e ,u.rt|ty Jen.rJeJ, .|| ct|e.
t||rgs urc|.rgeJ. |e supp|y cu.ve s|c.s t|e ,u.rt|ty c| . gccJ c. se.v|ce t|.t se||e.s .||| c|e. .t v..|cus
p.|ces, .|| ct|e. t||rgs urc|.rgeJ. Supp|y cu.ves ..e gere..||y up...J s|cp|rg. .r |rc.e.se |r p.|ce gere..||y
|rc.e.ses t|e ,u.rt|ty supp||eJ, .|| ct|e. t||rgs urc|.rgeJ.
|e e,u|||b.|un p.|ce cccu.s .|e.e t|e Jen.rJ .rJ supp|y cu.ves |rte.sect. At t||s p.|ce, t|e ,u.rt|ty Je
n.rJeJ e,u.|s t|e ,u.rt|ty supp||eJ. A p.|ce ||g|e. t|.r t|e e,u|||b.|un p.|ce |rc.e.ses t|e ,u.rt|ty sup
p||eJ .rJ .eJuces t|e ,u.rt|ty Jen.rJeJ, c.us|rg . su.p|us. A p.|ce |c.e. t|.r t|e e,u|||b.|un p.|ce |r
c.e.ses t|e ,u.rt|ty Jen.rJeJ .rJ .eJuces t|e ,u.rt|ty supp||eJ, c.us|rg . s|c.t.ge. |su.||y, n..ket su.
p|uses .rJ s|c.t.ges ..e s|c.t||veJ. |.rges |r Jen.rJ c. supp|y, c.useJ by c|.rges |r t|e Jete.n|r.rts c|
Jen.rJ .rJ supp|y ct|e..|se |e|J ccrst.rt |r t|e .r.|ys|s, c|.rge t|e e,u|||b.|un p.|ce .rJ cutput. |e c|.
cu|.. |c. ncJe| .||c.s us tc see |c. Jen.rJ .rJ supp|y |r v..|cus n..kets ..e .e|.teJ tc cre .rct|e..
CHAP7R 3 DMAND AND SUPPLY 83
C O N C P 7 P R O 8 L M S
1. \|.t Jc ycu t||rk |.ppers tc t|e Jen.rJ |c. p|...s Ju.|rg t|e Supe. bc.|` \|y`
2. \||c| c| t|e |c||c.|rg gccJs ..e ||ke|y tc be c|.ss||eJ .s rc.n.| gccJs c. se.v|ces` |r|e.|c.` |e|erJ ycu.
.rs.e..
.. be.rs
b. uxeJcs
c. |seJ c..s
J. |seJ c|ct||rg
e. cnpute.s
|. bccks .ev|e.eJ |r |e |e. `o| |e
g. V.c..cr| .rJ c|eese
|. .|cu|.tc.s
|. |g..ettes
. .v|..
k. |eg.| se.v|ces
3. \||c| c| t|e |c||c.|rg p.|.s c| gccJs ..e ||ke|y tc be c|.ss||eJ .s subst|tutes` cnp|enerts` |e|erJ ycu.
.rs.e..
.. |e.rut butte. .rJ e||y
b. |ggs .rJ |.n
c. ||ke b..rJ .rJ |eebck b..rJ sre.ke.s
J. |bV .rJ App|e V.c|rtcs| b..rJ ccnpute.s
e. |.ess s||.ts .rJ t|es
|. A|.||re t|ckets .rJ |cte|s
g. C.sc||re .rJ t|.es
|. bee. .rJ .|re
|. |.xes .rJ |.stc|.ss n.||
. e.e.| .rJ n||k
k. e.e.| .rJ eggs
4. A stuJy |curJ t|.t |c.e. .|.|..es |eJ scne pecp|e tc subst|tute |y|rg |c. J.|v|rg tc t|e|. v.c.t|cr
Jest|r.t|crs. ||s .eJuceJ t|e Jen.rJ |c. c.. t..ve| .rJ |eJ tc .eJuceJ t..|c |.t.||t|es, s|rce .|. t..ve| |s
s.|e. pe. p.sserge. n||e t|.r c.. t..ve|. |s|rg t|e |cg|c suggesteJ by t|.t stuJy, suggest |c. e.c| c| t|e
|c||c.|rg everts .cu|J .|ect t|e runbe. c| ||g|..y |.t.||t|es |r .ry cre ye...
.. Ar |rc.e.se |r t|e p.|ce c| g.sc||re
b. A |..ge .eJuct|cr |r .ert.| ..tes |c. p.sserge. v.rs
c. Ar |rc.e.se |r .|.|..es
5. |||J.er urJe. .ge 2 ..e rc. .||c.eJ tc |y |.ee cr |.S. .|.||res, t|ey usu.||y s|t |r t|e|. p..erts |.ps. Scne
s.|ety .Jvcc.tes |.ve u.geJ t|.t t|ey be .e,u|.eJ tc be st..ppeJ |r |r|.rt se.ts, .||c| .cu|J ne.r t|e|.
p..erts .cu|J |.ve tc pu.c|.se t|ckets |c. t|en. Scne eccrcn|sts |.ve ..gueJ t|.t suc| . ne.su.e
.cu|J .ctu.||y |rc.e.se |r|.rt |.t.||t|es. .r ycu s.y .|y`
6. |e g..p|s be|c. s|c. |cu. pcss|b|e s|||ts |r Jen.rJ c. |r supp|y t|.t ccu|J cccu. |r p..t|cu|.. n..kets.
|e|.te e.c| c| t|e everts Jesc.|beJ be|c. tc cre c| t|en.
84 PRINCIPLS OI CONOMICS
.. |c. J|J t|e |e.vy ..|rs |r Scut| Ane.|c. |r 199 .|ect t|e n..ket |c. cc|ee`
b. |e Su.gecr Cere..| Jec|Jes |.erc| |.|es ..e rct b.J |c. ycu. |e.|t| .|te. .|| .rJ |ssues . .epc.t
erJc.s|rg t|e|. use. \|.t |.ppers tc t|e n..ket |c. |.erc| |.|es`
c. |c. Jc ycu t||rk .|s|rg |rccnes .|ect t|e n..ket |c. sk| v.c.t|crs`
J. A re. tec|r|,ue |s J|sccve.eJ |c. n.ru|.ctu.|rg ccnpute.s t|.t g.e.t|y |c.e.s t|e|. p.cJuct|cr
ccst. \|.t |.ppers tc t|e n..ket |c. ccnpute.s`
e. |c. .cu|J . b.r cr snck|rg |r pub||c .|ect t|e n..ket |c. c|g..ettes`
. As |c.c..b J|ets |rc.e.seJ |r pcpu|..|ty, egg p.|ces .cse s|..p|y. |c. n|g|t t||s .|ect t|e ncrks supp|y
c| ccck|es c. p.|v.te .et.e.ts` (See t|e .se |r |c|rt cr t|e Vcrks c| St. bereJ|cts.)
8. C.sc||re p.|ces typ|c.||y .|se Ju.|rg t|e sunne., . t|ne c| |e.vy tcu.|st t..|c. A st.eet t.|k |e.tu.e cr .
..J|c st.t|cr scug|t tcu.|st .e.ct|cr tc ||g|e. g.sc||re p.|ces. |e.e ..s cre .espcrse. | Jcrt ||ke en t|e
||g|e. p.|ces nuc|. | t||rk t|e g.s ccnp.r|es ust use .ry excuse tc .ck up p.|ces, .rJ t|ey.e Jc|rg |t
.g.|r rc.. |c. Jces t||s tcu.|sts pe.spect|ve J||e. |.cn t|.t c| eccrcn|sts .|c use t|e ncJe| c|
Jen.rJ .rJ supp|y`
9. |e |rt.cJuct|cr tc t|e c|.pte. ..gues t|.t p.e|e.erces |c. cc|ee c|.rgeJ |r t|e 1990s .rJ t|.t
excess|ve ..|r |u.t y|e|Js |.cn cc|ee p|.rts. S|c. .rJ exp|.|r t|e e|ects c| t|ese t.c c|.cunst.rces cr
t|e cc|ee n..ket.
10. \|t| p.e|e.erces |c. cc|ee .en.|r|rg st.crg |r t|e e..|y p..t c| t|e certu.y, \|etr.n erte.eJ t|e n..ket
.s . n.c. expc.te. c| cc|ee. S|c. .rJ exp|.|r t|e e|ects c| t|ese t.c c|.cunst.rces cr t|e cc|ee
n..ket.
11. |e stuJy cr t|e eccrcn|cs c| cbes|ty J|scusseJ |r t|e .se |r |c|rt |r t||s c|.pte. cr t|.t tcp|c .|sc
rcteJ t|.t .rct|e. |.ctc. be||rJ .|s|rg cbes|ty |s t|e Jec||re |r c|g..ette snck|rg .s t|e p.|ce c|
c|g..ettes |.s .|ser. S|c. .rJ exp|.|r t|e e|ect c| ||g|e. c|g..ette p.|ces cr t|e n..ket |c. |ccJ. \|.t
Jces t||s |rJ|rg |np|y .bcut t|e .e|.t|crs||p bet.eer c|g..ettes .rJ |ccJ`
12. |r 2004, |e |e. `o| |e .epc.teJ t|.t |rJ|. n|g|t be |cs|rg |ts cutscu.c|rg eJge Jue tc .|s|rg ..ges
1
|e .epc.te. rcteJ t|.t . .ecert .epc.t p.cecteJ t|.t || |rJ|. ccrt|rueJ tc p.cJuce cc||ege g..Ju.tes .t
t|e cu..ert ..te, Jen.rJ .cu|J exceeJ supp|y by 20 |r t|e n.|r cutscu.c|rg n..kets by 2008. |s|rg
t|e te.n|rc|cgy ycu |e..reJ |r t||s c|.pte., exp|.|r .|.t |e ne.rt tc s.y ..s |.pper|rg |r t|e n..ket
|c. |rJ|.r .c.ke.s |r cutscu.c|rg cbs. |r p..t|cu|.., |s Jen.rJ |c. |rJ|.r .c.ke.s |rc.e.s|rg c.
Jec.e.s|rg` |s t|e supp|y c| |rJ|.r .c.ke.s |rc.e.s|rg c. Jec.e.s|rg` \||c| |s s|||t|rg |.ste.` |c. Jc ycu
krc.`
13. |c. nc.e t|.r . certu.y, n||k p.cJuce.s |.ve p.cJuceJ sk|n n||k, .||c| ccrt.|rs v|.tu.||y rc |.t, .|crg
.|t| .egu|.. n||k, .||c| ccrt.|rs 4 |.t. but . certu.y .gc, sk|n n||k .cccurteJ |c. cr|y .bcut 1 c| tct.|
p.cJuct|cr, .rJ nuc| c| |t ..s |eJ tc |cgs. cJ.y, sk|n .rJ ct|e. .eJuceJ|.t n||ks n.ke up t|e bu|k c|
n||k s.|es. \|.t cu.ve s|||teJ, .rJ .|.t |.ctc. s|||teJ |t`
14. Suppcse |.ns |r t|e eccrcny .e.e tc p.cJuce |e.e. gccJs .rJ se.v|ces. |c. Jc ycu t||rk t||s .cu|J
.|ect |cuse|c|J sperJ|rg cr gccJs .rJ se.v|ces` (||| |se t|e c|.cu|.. |c. ncJe| tc .r.|y.e t||s
,uest|cr.)
CHAP7R 3 DMAND AND SUPPLY 85
N U M R I C A L P R O 8 L M S
|.cb|ens 15 ..e b.seJ cr t|e g..p| be|c..
1. At . p.|ce c| 1.50 pe. Jc.er, |c. n.ry b.ge|s ..e Jen.rJeJ pe. ncrt|`
2. At . p.|ce c| 1.50 pe. Jc.er, |c. n.ry b.ge|s ..e supp||eJ pe. ncrt|`
3. At . p.|ce c| 3.00 pe. Jc.er, |c. n.ry b.ge|s ..e Jen.rJeJ pe. ncrt|`
4. At . p.|ce c| 3.00 pe. Jc.er, |c. n.ry b.ge|s ..e supp||eJ pe. ncrt|`
5. \|.t |s t|e e,u|||b.|un p.|ce c| b.ge|s` \|.t |s t|e e,u|||b.|un ,u.rt|ty pe. ncrt|`
|.cb|ens 69 ..e b.seJ cr t|e ncJe| c| Jen.rJ .rJ supp|y |c. cc|ee .s s|c.r |r ||gu.e 3.1 +cu c.r
g..p| t|e |r|t|.| Jen.rJ .rJ supp|y cu.ves by us|rg t|e |c||c.|rg v.|ues, .|t| .|| ,u.rt|t|es |r n||||crs c|
pcurJs c| cc|ee pe. ncrt|.
Price Quantity demanded Quantity suppIied
3 40 10
4 35 15
5 30 20
6 25 25
20 30
8 15 35
9 10 40
6. Suppcse t|e ,u.rt|ty Jen.rJeJ .|ses by 20 n||||cr pcurJs c| cc|ee pe. ncrt| .t e.c| p.|ce. |... t|e
|r|t|.| Jen.rJ .rJ supp|y cu.ves b.seJ cr t|e v.|ues g|ver |r t|e t.b|e .bcve. |er J... t|e re.
Jen.rJ cu.ve g|ver by t||s c|.rge, .rJ s|c. t|e re. e,u|||b.|un p.|ce .rJ ,u.rt|ty.
. Suppcse t|e ,u.rt|ty Jen.rJeJ |.||s, .e|.t|ve tc t|e v.|ues g|ver |r t|e .bcve t.b|e, by 20 n||||cr pcurJs
pe. ncrt| .t p.|ces bet.eer 4 .rJ 6 pe. pcurJ, .t p.|ces bet.eer .rJ 9 pe. pcurJ, t|e ,u.rt|ty
Jen.rJeJ beccnes .e.c. |... t|e re. Jen.rJ cu.ve .rJ s|c. t|e re. e,u|||b.|un p.|ce .rJ
,u.rt|ty.
8. Suppcse t|e ,u.rt|ty supp||eJ .|ses by 20 n||||cr pcurJs pe. ncrt| .t e.c| p.|ce, .|||e t|e ,u.rt|t|es
Jen.rJeJ .et.|r t|e v.|ues s|c.r |r t|e t.b|e .bcve. |... t|e re. supp|y cu.ve .rJ s|c. t|e re.
e,u|||b.|un p.|ce .rJ ,u.rt|ty.
9. Suppcse t|e ,u.rt|ty supp||eJ |.||s, .e|.t|ve tc t|e v.|ues g|ver |r t|e t.b|e .bcve, by 20 n||||cr pcurJs
pe. ncrt| .t p.|ces .bcve 5, .t . p.|ce c| 5 c. |ess pe. pcurJ, t|e ,u.rt|ty supp||eJ beccnes .e.c.
|... t|e re. supp|y cu.ve .rJ s|c. t|e re. e,u|||b.|un p.|ce .rJ ,u.rt|ty.
86 PRINCIPLS OI CONOMICS
|.cb|ens 1015 ..e b.seJ cr t|e Jen.rJ .rJ supp|y sc|eJu|es |c. g.sc||re be|c. (.|| ,u.rt|t|es ..e |r t|cu
s.rJs c| g.||crs pe. .eek).
Price per gaIIon Quantity demanded Quantity suppIied
1 8 0
2 1
3 6 2
4 5 3
5 4 4
6 3 5
2 6
8 1
10. C..p| t|e Jen.rJ .rJ supp|y cu.ves .rJ s|c. t|e e,u|||b.|un p.|ce .rJ ,u.rt|ty.
11. At . p.|ce c| 3 pe. g.||cr, .cu|J t|e.e be . su.p|us c. s|c.t.ge c| g.sc||re` |c. nuc| .cu|J t|e su.p|us
c. s|c.t.ge be` |rJ|c.te t|e su.p|us c. s|c.t.ge cr t|e g..p|.
12. At . p.|ce c| 6 pe. g.||cr, .cu|J t|e.e be . su.p|us c. s|c.t.ge c| g.sc||re` |c. nuc| .cu|J t|e su.p|us
c. s|c.t.ge be` S|c. t|e su.p|us c. s|c.t.ge cr t|e g..p|.
13. Suppcse t|e ,u.rt|ty Jen.rJeJ |rc.e.seJ by 2,000 g.||crs pe. ncrt| .t e.c| p.|ce. At . p.|ce c| 3 pe.
g.||cr, |c. nuc| .cu|J t|e su.p|us c. s|c.t.ge be` C..p| t|e Jen.rJ .rJ supp|y cu.ves .rJ s|c. t|e
su.p|us c. s|c.t.ge.
14. Suppcse t|e ,u.rt|ty supp||eJ Jec.e.seJ by 2,000 g.||crs pe. ncrt| .t e.c| p.|ce |c. p.|ces bet.eer 4
.rJ 8 pe. g.||cr. At p.|ces |ess t|.r 4 pe. g.||cr t|e ,u.rt|ty supp||eJ beccnes .e.c, .|||e t|e
,u.rt|t|es Jen.rJeJ .et.|r t|e v.|ues s|c.r |r t|e t.b|e. At . p.|ce c| 4 pe. g.||cr, |c. nuc| .cu|J
t|e su.p|us c. s|c.t.ge be` C..p| t|e Jen.rJ .rJ supp|y cu.ves .rJ s|c. t|e su.p|us c. s|c.t.ge.
15. || t|e Jen.rJ cu.ve s|||ts .s |r p.cb|en 13 .rJ t|e supp|y cu.ve s|||ts .s |r p.cb|en 14, .|t|cut J...|rg
. g..p| c. ccrsu|t|rg t|e J.t., c.r ycu p.eJ|ct .|et|e. e,u|||b.|un p.|ce |rc.e.ses c. Jec.e.ses` \|.t
.bcut e,u|||b.|un ,u.rt|ty` |c. J... . g..p| t|.t s|c.s .|.t t|e re. e,u|||b.|un p.|ce .rJ ,u.rt|ty
..e.
CHAP7R 3 DMAND AND SUPPLY 87
1.
ENDNOTES
|c.n Sc|e|be., As . erte. |c. Outscu.c|rg, |rJ|. cu|J be |cs|rg |ts |Jge, |e.
`o| |e, V.y 9, 2004, p. b|3.
88 PRINCIPLS OI CONOMICS
| A | | | 4
Applications of Demand and
Supply
S7AR7 UP: A COMPOSR LOGS ON
S|rce t|e .ge c| sever, | kre. t|.t | .cu|J be . nus|c|.r. ArJ |.cn .ge |cu.teer, | kre. t|.t | .cu|J be . ccn
pcse., s.ys |s..e||bc.r O|e. berAncts. \|.t |e J|J rct krc. ..s t|.t |e .cu|J use ccnpute.s tc c...y cut ||s
.c.k. |e |s rc. . p.c|essc. c| nus|c .t c|c..Jc c||ege, .rJ |.. berAnctss ccnpcs|t|crs .rJ cpe..s |.ve beer
pe.|c.neJ |r t|e |r|teJ St.tes, |u.cpe, .rJ '.p.r.
|c. cve. 15 ye..s, |e |.s useJ nus|c.| rct.t|cr sc|t...e tc |e|p |r ccnpcs|rg nus|c. |e cutput |s ext.ene|y
e|eg.rt. |e.|c.ne.s ercy |cck|rg .t suc| . c|e.. .rJ c|e.r scc.e. |e c.e.t|cr c| p..ts cut c| . |u|| scc.e |s .s e.sy
.s p.ess|rg t|e <||||> key cr t|e keybc..J. |.rges c.r e.s||y be |rse.teJ |rtc t|e rct.t|cr ||e, .||c| e||n|r
.tes t|e reeJ |c. .eccpy|rg. |r .JJ|t|cr, |.. berAncts uses ccnpute.s |c. p|.yb.ck. | c.r ||ster tc . .e|.t|ve|y .c
cu..te J|g|t.| pe.|c.n.rce c| t|e scc.e .t .ry g|ver pc|rt, .|t| .ry tenpc c. |rst.unert.t|cr | c|ccse. |e scurJ
,u.||ty |.s |np.cveJ sc nuc| t|.t J|g|t.| ||es scurJ .|ncst |Jert|c.| tc .e.| pe.|c.n.rce. |e c.r .|sc p.cJuce
|s cr ||s c.r .rJ c.e.te |cJc.sts sc t|.t .rycre |r t|e .c.|J c.r |e.. ||s nus|c. |e erg.ges |r se||pub||c.t|cr
c| scc.es .rJ se||n..ket|rg. |r ny c.se, | get tc keep t|e ccpy.|g|ts cr .|| c| ny nus|c. ||s .cu|J |.ve beer |n
pcss|b|e ter tc t.e|ve ye..s .gc .|er ccnpcse.s t..rs|e..eJ t|e|. .|g|ts tc pub||s|e.s. |cne p.ges cr t|e \c.|J
\|Je \eb .||c. ne tc p.cncte ny c.r .c.k. |.c|essc. berAncts .|sc c|.rgeJ t|e ..y |e te.c|es nus|c ccn
pcs|t|cr. |e. .pp||c.t|cr sc|t...e, suc| .s C...geb.rJ, |.s cpereJ t|e ..y |c. .rycre |rte.esteJ tc t.y tc ccn
pcse nus|c. \|e.e.s ||s nus|c ccnpcs|t|cr c|.sses useJ tc |.ve nus|c t|ec.y p.e.e,u|s|tes, tcJ.y ||s c|.sses ..e
cper tc .||.
|.. berAncts st..teJ cut |r 1989 .|t| . V.c|rtcs| S|30 t|.t |.J 4 neg.bytes c| ..rJcn .ccess nenc.y
(|AV) .rJ .r 80neg.byte |..J J.|ve. |t ccst ||n .bcut 3,000. cJ.y, |e uses . V.c|rtcs| |c.e.bcck C4 |.ptcp
.|t| 1.5 g|g.bytes c| nenc.y, bu||t|r |\|/| bu.re., .rJ .|.e|ess |rte.ret ccrrect|crs. ||s re. ccnpute. ccst
.bcut 2,000. |c. pe.scr.| ccnpute.s .cse sc J..n.t|c.||y |r pc.e. .s t|ey |e|| sc steep|y |r p.|ce |s ust cre c|
t|e stc.|es .bcut n..kets .e .||| te|| |r t||s c|.pte., .||c| .|ns tc |e|p ycu urJe.st.rJ |c. t|e ncJe| c| Jen.rJ
.rJ supp|y .pp||es tc t|e .e.| .c.|J.
|r t|e |.st sect|cr c| t||s c|.pte., .e .||| |cck .t seve..| n..kets t|.t ycu ..e ||ke|y tc |.ve p..t|c|p.teJ |r c. be
|.n|||.. .|t|t|e n..ket |c. pe.scr.| ccnpute.s, t|e n..kets |c. c.uJe c|| .rJ |c. g.sc||re, .rJ t|e stcck n..ket.
+cu p.cb.b|y c.r c. |.ve .ccess tc . ccnpute.. |.c| c| us ..s .|ecteJ by t|e s|..p .|se |r c.uJe c|| .rJ g.sc||re
p.|ces |.cn 2004 tc n|J2008. |e pe.|c.n.rce c| t|e stcck n..ket |s .|..ys . n.c. re.s |ten .rJ n.y .|ect
ycu pe.scr.||y, || rct rc., t|er |r t|e |utu.e. |e ccrcepts c| Jen.rJ .rJ supp|y gc . |crg ..y |r exp|.|r|rg t|e
be|.v|c. c| e,u|||b.|un p.|ces .rJ ,u.rt|t|es |r .|| c| t|ese n..kets. |e pu.pcse c| t||s sect|cr |s tc .||c. ycu tc
p..ct|ce us|rg t|e ncJe| c| Jen.rJ .rJ supp|y .rJ get ycu tc st..t t||rk|rg .bcut t|e ny.|.J ..ys t|e ncJe| c|
Jen.rJ .rJ supp|y c.r be .pp||eJ.
|r t|e seccrJ p..t c| t|e c|.pte. .e .||| |cck .t n..kets |r .||c| t|e gcve.rnert |.s ||stc.|c.||y p|.yeJ .
|..ge .c|e |r .egu|.t|rg p.|ces. by |eg|s|.t|rg n.x|nun c. n|r|nun p.|ces, t|e gcve.rnert |.s kept t|e p.|ces c|
ce.t.|r gccJs be|c. c. .bcve e,u|||b.|un. \e .||| |cck .t t|e ..gunerts |c. J|.ect gcve.rnert |rte.vert|cr |r
ccrt.c|||rg p.|ces .s .e|| .s t|e ccrse,uerces c| suc| pc||c|es. As .e s|.|| see, p.evert|rg t|e p.|ce c| . gccJ |.cn
|rJ|rg |ts c.r e,u|||b.|un c|ter |.s ccrse,uerces t|.t n.y be .t cJJs .|t| t|e |rtert|crs c| t|e pc||cy n.ke.s
.|c put t|e .egu|.t|crs |r p|.ce.
|r t|e t||.J sect|cr c| t|e c|.pte. .e .||| |cck .t t|e n..ket |c. |e.|t| c..e. ||s n..ket |s |rte.est|rg bec.use
|c. .e|| (c. pcc.|y) |t .c.ks c.r be . n.tte. c| |||e .rJ Je.t| .rJ bec.use |t |.s spec|.| c|...cte.|st|cs. |r p..t|cu
|.., n..kets |r .||c| p..t|c|p.rts Jc rct p.y |c. gccJs J|.ect|y, but ..t|e. p.y |rsu.e.s .|c t|er p.y t|e supp||e.s
c| t|e gccJs, cpe..te scne.|.t J||e.ert|y |.cn t|cse |r .||c| p..t|c|p.rts p.y J|.ect|y |c. t|e|. pu.c|.ses. ||s
exters|cr c| Jen.rJ .rJ supp|y .r.|ys|s .eve.|s nuc| .bcut |c. suc| n..kets cpe..te.
1. PUTTING DEMAND AND SUPPLY TO WORK
L A R N I N G O 8 1 C 7 I V S
1. Learn how to appIy the modeI of demand and suppIy to expIaining the behavior of equiIibrium
prices and quantities in a variety of markets.
2. xpIain how technoIogicaI change can be represented using the modeI of demand and suppIy.
3. xpIain how the modeI of demand and suppIy can be used to expIain changes in prices of
shares of stock.
A shift in eithei demand oi supply, oi in both, leads to a change in equilibiium piice and equilibiium
quantity. We begin this chaptei by examining maikets in which piices adjust quickly to changes in de-
mand oi supply: the maiket foi peisonal computeis, the maikets foi ciude oil and gasoline, and the
stock maiket. These maikets aie thus diiect applications of the model of demand and supply.
1.1 The Peisonal Computei Maiket
In the 1960s, to speak of computeis was to speak of IBM, the dominant makei of laige mainfiame com-
puteis used by business and goveinment agencies. Then between 1976, when Apple Computei intio-
duced its fist desktop computei, and 1981, when IBM pioduced its fist peisonal computeis (PCs), the
old woild was tuined upside down. In 1984, just 8.2 of U.S. households owned a peisonal computei.
By 2007, Google estimates that 78 did. The tools of demand and supply tell the stoiy fiom an eco-
nomic peispective.
Technological change has been bieathtakingly swift in the computei industiy. Because peisonal
computeis have changed so diamatically in peifoimance and in the iange of the functions they pei-
foim, we shall speak of quality-adjusted" peisonal computeis. The piice pei unit of quality-adjusted
desktop computeis fell by about half eveiy 30 months duiing the peiiod 1976-1989. In the fist half of
the 1990s, those piices fell by half eveiy 28 months. In the second half of the 1990s, the halving time"
fell to eveiy 24 months.
[1]
Considei anothei indicatoi of the phenomenal change in computeis. Between 1993 and 1998, the
Buieau of Laboi Statistics estimates that cential piocessing unit (CPU) speed iose 1,263, system
memoiy incieased 1,300, haid diive capacity soaied by 3,700, and monitoi size went up 13. It
seems safe to say that the dizzying pace of change iecoided in the 1990s has incieased in this centuiy. A
computei" today is not the same good as a computei" even fve yeais ago. To make them compai-
able, we must adjust foi these changes in quality.
Initially, most peisonal computeis weie manufactuied by Apple oi Compaq; both companies weie
veiy pioftable. The potential foi piofts attiacted IBM and othei fims to the industiy. Unlike laige
90 PRINCIPLS OI CONOMICS
mainfiame computeis, peisonal computei clones tuined out to be faiily easy things to manufactuie. As
shown in Table 4.1, the top fve peisonal computei manufactuieis pioduced only 48 of the peisonal
computeis sold in the woild in 2003, and the laigest manufactuiei, Dell, sold only about 19 of the
total in that yeai. This is a fai ciy fiom the moie than 90 of the mainfiame computei maiket that
IBM once held. The maiket has become fai moie competitive.
7A8L 4. 1 PersonaI Computer Shipments, Market Percentage Shares by Vendors, WorId and United
States
Company % of WorId Shipments Company % of U.S. Shipments
|e|| 18.9 |e|| 34
|e.|ett|.ck..J 15.4 |e.|ett|.ck..J 18.2
|bV 5.1 C.te..y 5.
|u|tsu Se|ners 4.6 |bV 4.3
Ace. 4 App|e 3.9
Ot|e.s 52 Ot|e.s 34
7otaI 100.0 7otaI 100.0
Source IDC-Press Release 15 Apr 2005 PC Market Approaches 11 Growth as International Demand Remains Strong, According to IDC
(http//www.idc.com/getdoc.jsp?containerId=pr2005_04_14_17070722) (Totals may not add due to rounding)
Figuie 4.1 illustiates the changes that have occuiied in the computei maiket. The hoiizontal axis shows
the quantity of quality-adjusted peisonal computeis. Thus, the quantity axis can be thought of as a unit
of computing powei. Similaily, the piice axis shows the piice pei unit of computing powei. The iapid
inciease in the numbei of fims, togethei with diamatic technological impiovements, led to an inciease
in supply, shifting the supply cuive in Figuie 4.1 to the iight fiom S
1
to S
2
.
II GUR 4. 1 7he PersonaI Computer Market
|e supp|y cu.ve |c. ,u.||ty.JusteJ pe.scr.| ccnpute.s |.s s|||teJ n..keJ|y tc t|e .|g|t, .eJuc|rg t|e
e,u|||b.|un p.|ce |.cn |
1
tc |
2
.rJ |rc.e.s|rg t|e e,u|||b.|un ,u.rt|ty |.cn (
1
tc (
2
|r 2005.
Demand also shifted to the iight fiom D
1
to D
2
, as incomes iose and new uses foi computeis, fiom e-
mail and social netwoiking to Voice ovei Inteinet Piotocol (VoIP) and Radio Fiequency ID (RFID)
tags (which allow wiieless tiacking of commeicial shipments via desktop computeis), alteied the pief-
eiences of consumei and business useis. Because we obseive a fall in equilibiium piice and an inciease
in equilibiium quantity, we conclude that the iightwaid shift in supply has outweighed the iightwaid
shift in demand. The powei of maiket foices has piofoundly afected the way we live and woik.
CHAP7R 4 APPLICA7IONS OI DMAND AND SUPPLY 91
1.2 The Maikets foi Ciude Oil and foi Gasoline
The maiket foi ciude oil took a iadical tuin in 1973. The piice pei baiiel of ciude oil quadiupled in
1973 and 1974. Piice iemained high until the eaily 1980s but then fell back diastically and iemained
low foi about two decades. In 2004, the piice of oil began to move upwaid and by 2008 had ieached
$147 pei baiiel.
What caused the diamatic inciease in gasoline and oil piices in 2008: It appeaied to be incieasing
woildwide demand outpacing pioduceis' ability-oi willingness-to inciease pioduction much. This
inciease in demand is illustiated in Figuie 4.2.
II GUR 4. 2 7he Increasing Demand for Crude OiI
|e p.|ce c| c|| ..s 35 pe. b...e| .t t|e beg|rr|rg c| 2004, .s Jete.n|reJ by t|e |rte.sect|cr c| .c.|J Jen.rJ,
|
1
, .rJ .c.|J supp|y, .
1
. |rc.e.s|rg .c.|J Jen.rJ, p.cnpteJ |..ge|y by |rc.e.s|rg Jen.rJ |.cn ||r. .s .e|| .s
|.cn ct|e. ccurt.|es, s|||teJ .c.|J Jen.rJ tc |
2
, pus||rg t|e p.|ce .s ||g| .s 140 pe. b...e| by t|e n|JJ|e c|
2008.
Highei oil piices also inciease the cost of pioducing viitually eveiy good oi seivice, as at a minimum,
the pioduction of most goods iequiies tianspoitation. These costs inevitably tianslate into highei
piices foi neaily all goods and seivices. Supply cuives of the goods and seivices thus afected shift to
the left, putting downwaid piessuie on output and upwaid piessuie on piices.
Giaphically, the impact of highei gasoline piices on businesses that use gasoline is illustiated in
Figuie 4.3. Because highei gasoline piices inciease the cost of doing business, they shift the supply
cuives foi neaily all businesses to the left, putting upwaid piessuie on piices and downwaid piessuie
on output. In the case shown heie, the supply cuive in a typical industiy shifts fiom S
1
to S
2
. This in-
cieases the equilibiium piice fiom P
1
to P
2
and ieduces the equilibiium quantity fiom Q
1
to Q
2
.
92 PRINCIPLS OI CONOMICS
soIe proprietorship
A |.n c.reJ by cre
|rJ|v|Ju.|.
partnership
A |.n c.reJ by seve..|
|rJ|v|Ju.|s.
corporation
A |.n c.reJ by
s|..e|c|Je.s .|c c.r stcck
|r t|e |.n.
corporate stock
S|..es |r t|e c.re.s||p c| .
cc.pc..t|cr.
stock market
|e set c| |rst|tut|crs |r
.||c| s|..es c| stcck ..e
bcug|t .rJ sc|J.
II GUR 4. 3 7he Impact of Higher GasoIine Prices
||g|e. g.sc||re p.|ces |rc.e.se t|e ccst c| p.cJuc|rg v|.tu.||y eve.y gccJ c. se.v|ce. |r t|e c.se s|c.r |e.e, t|e
supp|y cu.ve |r . typ|c.| |rJust.y s|||ts |.cn .
1
tc .
2
. ||s |rc.e.ses t|e e,u|||b.|un p.|ce |.cn |
1
tc |
2
.rJ .eJuces
e,u|||b.|un ,u.rt|ty |.cn (
1
tc (
2
.
Then, as the woild economy slowed diamatically in the second half of 2008, the demand cuive foi oil
shifted back to the left. By Novembei 2008, the piice pei baiiel had diopped back to below $60 pei bai-
iel. As gas piices also subsided, so did the thieat of highei piices in othei industiies.
1.3 The Stock Maiket
The ciiculai fow model suggests that capital, like othei factois of pioduction, is supplied by house-
holds to fims. Fiims, in tuin, pay income to those households foi the use of theii capital. Geneially
speaking, howevei, capital is actually owned by fims themselves. Geneial Motois owns its assembly
plants, and Wal-Mait owns its stoies; these fims theiefoie own theii capital. But fims, in tuin, aie
owned by people-and those people, of couise, live in households. It is thiough theii owneiship of
fims that households own capital.
A fim may be owned by one individual (a sole proprietorship), by seveial individuals (a part-
nership), oi by shaieholdeis who own stock in the fim (a corporation). Although most fims in the
United States aie sole piopiietoiships oi paitneiships, the bulk of the nation's total output (about 90)
is pioduced by coipoiations. Coipoiations also own most of the capital (machines, plants, buildings,
and the like).
This section desciibes how the piices of shaies of corporate stock, shaies in the owneiship of a
coipoiation, aie deteimined by the inteiaction of demand and supply. Ultimately, the same foices that
deteimine the value of a fim's stock deteimine the value of a sole piopiietoiship oi paitneiship.
When a coipoiation needs funds to inciease its capital oi foi othei ieasons, one means at its dis-
posal is to issue new stock in the coipoiation. (Othei means include boiiowing funds oi using past
piofts.) Once the new shaies have been sold in what is called an initial public ofeiing (IPO), the coi-
poiation ieceives no fuithei funding as shaies of its stock aie bought and sold on the secondaiy mai-
ket. The secondaiy maiket is the maiket foi stocks that have been issued in the past, and the daily news
iepoits about stock piices almost always iefei to activity in the secondaiy maiket. Geneially, the coi-
poiations whose shaies aie tiaded aie not involved in these tiansactions.
The stock market is the set of institutions in which shaies of stock aie bought and sold. The New
Yoik Stock Exchange (NYSE) is one such institution. Theie aie many otheis all ovei the woild, such as
the DAX in Geimany and the Bolsa in Mexico. To buy oi sell a shaie of stock, one places an oidei with
a stockbiokei who ielays the oidei to one of the tiadeis at the NYSE oi at some othei exchange.
The piocess thiough which shaies of stock aie bought and sold can seem chaotic. At many ex-
changes, tiadeis with oideis fiom customeis who want to buy stock shout out the piices those custom-
eis aie willing to pay. Tiadeis with oideis fiom customeis who want to sell shout out ofeis of piices at
which theii customeis aie willing to sell. Some exchanges use electionic tiading, but the piinciple is the
same: if the piice someone is willing to pay matches the piice at which someone else is willing to sell,
CHAP7R 4 APPLICA7IONS OI DMAND AND SUPPLY 93
II GUR 4. 4 Demand and SuppIy in the
Stock Market
|e e,u|||b.|un p.|ce c| stcck s|..es |r |rte|
c.pc..t|cr |s |r|t|.||y 25, Jete.n|reJ by t|e
|rte.sect|cr c| Jen.rJ .rJ supp|y cu.ves |
1
.rJ .
1
, .t .||c| (
1
n||||cr s|..es ..e t..JeJ
e.c| J.y.
retained earnings
|.c|ts kept by . ccnp.ry.
dividends
|.c|ts J|st.|buteJ tc
s|..e|c|Je.s.
the tiade is made. The most iecent piice at which a stock has tiaded is iepoited almost instantaneously
thioughout the woild.
Figuie 4.4 applies the model of demand and supply to the deteimination of stock piices. Suppose
the demand cuive foi shaies in Intel Coipoiation is given by D
1
and the supply by S
1
. (Even though the
total numbei of shaies outstanding is fxed at any point in time, the supply cuive is not veitical. Rathei,
the supply cuive is upwaid sloping because it iepiesents how many shaies cuiient owneis aie piepaied
to sell at each piice, and that numbei will be gieatei at highei piices.) Suppose that these cuives intei-
sect at a piice of $23, at which Q
1
shaies aie tiaded each day. If the piice weie highei, moie shaies
would be ofeied foi sale than would be demanded, and the piice would quickly fall. If the piice weie
lowei, moie shaies would be demanded than would be supplied, and the piice would quickly iise. In
geneial, we can expect the piices of shaies of stock to move quickly to theii equilibiium levels.
The inteisection of the demand and supply cuives foi shaies of stock in a paiticu-
lai company deteimines the equilibiium piice foi a shaie of stock. But what deteimines
the demand and supply foi shaies of a company's stock:
The ownei of a shaie of a company's stock owns a shaie of the company, and,
hence, a shaie of its piofts; typically, a coipoiation will ietain and ieinvest some of its
piofts to inciease its futuie pioftability. The piofts kept by a company aie called re-
tained earnings. Piofts distiibuted to shaieholdeis aie called dividends. Because a
shaie of stock gives its ownei a claim on pait of a company's futuie piofts, it follows
that the expected level of futuie piofts plays a iole in deteimining the value of its stock.
Of couise, those futuie piofts cannot be known with ceitainty; investois can only
piedict what they might be, based on infoimation about futuie demand foi the com-
pany's pioducts, futuie costs of pioduction, infoimation about the soundness of a com-
pany's management, and so on. Stock piices in the ieal woild thus iefect estimates of a
company's piofts piojected into the futuie.
The downwaid slope of the demand cuive suggests that at lowei piices foi the
stock, moie people calculate that the fim's futuie eainings will justify the stock's pui-
chase. The upwaid slope of the supply cuive tells us that as the piice of the stock iises,
moie people conclude that the fim's futuie eainings do not justify holding the stock
and theiefoie ofei to sell it. At the equilibiium piice, the numbei of shaies supplied by
people who think holding the stock no longei makes sense just balances the numbei of
shaies demanded by people who think it does.
What factois, then, cause the demand oi supply cuives foi shaies of stocks to shift:
The most impoitant factoi is a change in the expectations of a company's futuie piofts.
Suppose Intel announces a new geneiation of computei chips that will lead to fastei
computeis with laigei memoiies. Cuiient owneis of Intel stock would adjust upwaid
theii estimates of what the value of a shaie of Intel stock should be. At the old equilibiium piice of $23
fewei owneis of Intel stock would be willing to sell. Since this would be tiue at eveiy possible shaie
piice, the supply cuive foi Intel stock would shift to the left, as shown in Figuie 4.3. Just as the expecta-
tion that a company will be moie pioftable shifts the supply cuive foi its stock to the left, that same
change in expectations will cause moie people to want to puichase the stock, shifting the demand cuive
to the iight. In Figuie 4.3, we see the supply cuive shifting to the left, fiom S
1
to S
2
, while the demand
cuive shifts to the iight, fiom D
1
to D
2
.
94 PRINCIPLS OI CONOMICS
II GUR 4. 5 A Change in xpectations
Affects the Price of Corporate Stock
|| |r.rc|.| |rvestc.s Jec|Je t|.t . ccnp.ry |s
||ke|y tc be nc.e p.c|t.b|e, t|er t|e supp|y c|
t|e stcck s|||ts tc t|e |e|t (|r t||s c.se, |.cn .
1
tc .
2
), .rJ t|e Jen.rJ |c. t|e stcck s|||ts tc
t|e .|g|t (|r t||s c.se, |.cn |
1
tc |
2
), .esu|t|rg
|r .r |rc.e.se |r p.|ce |.cn |
1
tc |
2
.
Othei factois may altei the piice of an individual coipoiation's shaie of stock oi
the level of stock piices in geneial. Foi example, demogiaphic change and iising in-
comes have afected the demand foi stocks in iecent yeais. Foi example, with a laige
piopoition of the U.S. population neaiing ietiiement age and beginning to think about
and plan foi theii lives duiing ietiiement, the demand foi stocks has iisen.
Infoimation on the economy as a whole is also likely to afect stock piices. If the
economy oveiall is doing well and people expect that to continue, they may become
moie optimistic about how pioftable companies will be in geneial, and thus the piices
of stocks will iise. Conveisely, expectations of a sluggish economy, as happened in the
fall of 2008, could cause stock piices in geneial to fall.
The stock maiket is bombaided with new infoimation eveiy minute of eveiy day.
Fiims announce theii piofts of the pievious quaitei. They announce that they plan to
move into a new pioduct line oi sell theii goods in anothei countiy. We leain that the
piice of Company A's good, which is a substitute foi one sold by Company B, has iisen.
We leain that countiies sign tiade agieements, launch wais, oi make peace. All of this
infoimation may afect stock piices because any infoimation can afect how buyeis and
selleis value companies.
k Y 7 A k A W A Y S
< ec|rc|cg|c.| c|.rge, .||c| |.s c.useJ t|e supp|y cu.ve |c. ccnput|rg pc.e. tc s|||t tc t|e .|g|t, |s t|e
n.|r .e.scr |c. t|e ..p|J |rc.e.se |r e,u|||b.|un ,u.rt|ty .rJ Jec.e.se |r e,u|||b.|un p.|ce c| pe.scr.|
ccnpute.s.
< |e |rc.e.se |r c.uJe c|| .rJ g.sc||re p.|ces |r 2008 ..s J.|ver p.|n..||y by |rc.e.seJ Jen.rJ |c. c.uJe
c||, .r |rc.e.se t|.t ..s c.e.teJ by eccrcn|c g.c.t| t|.cug|cut t|e .c.|J. .uJe c|| .rJ g.s p.|ces |e||
n..keJ|y .s .c.|J eccrcn|c g.c.t| subs|JeJ |.te. |r t|e ye...
< ||g|e. g.sc||re p.|ces |rc.e.seJ t|e ccst c| p.cJuc|rg v|.tu.||y eve.y gccJ .rJ se.v|ce, s|||t|rg supp|y
cu.ves |c. ncst gccJs .rJ se.v|ces tc t|e |e|t. ||s terJeJ tc pus| p.|ces up .rJ cutput Jc.r.
< |en.rJ .rJ supp|y Jete.n|re p.|ces c| s|..es c| cc.pc..te stcck. |e e,u|||b.|un p.|ce c| . s|..e c|
stcck st.|kes . b.|.rce bet.eer t|cse .|c t||rk t|e stcck |s .c.t| nc.e .rJ t|cse .|c t||rk |t |s .c.t|
|ess t|.r t|e cu..ert p.|ce.
< || . ccnp.rys p.c|ts ..e expecteJ tc |rc.e.se, t|e Jen.rJ cu.ve |c. |ts stcck s|||ts tc t|e .|g|t .rJ t|e
supp|y cu.ve s|||ts tc t|e |e|t, c.us|rg e,u|||b.|un p.|ce tc .|se. |e cppcs|te .cu|J cccu. || . ccnp.rys
p.c|ts .e.e expecteJ tc Jec.e.se.
< Ot|e. |.ctc.s t|.t |r|uerce t|e p.|ce c| cc.pc..te stcck |rc|uJe Jencg..p||c .rJ |rccne c|.rges .rJ
t|e cve..|| |e.|t| c| t|e eccrcny.
7 R Y I 7 !
Suppcse .r .|.||re .rrcurces t|.t |ts e..r|rgs t||s ye.. ..e |c.e. t|.r expecteJ Jue tc .eJuceJ t|cket s.|es.
|e .|.||re spckespe.scr g|ves rc |r|c.n.t|cr cr |c. t|e ccnp.ry p|.rs tc tu.r t||rgs ..curJ. |se t|e
ncJe| c| Jen.rJ .rJ supp|y tc s|c. .rJ exp|.|r .|.t |s ||ke|y tc |.pper tc t|e p.|ce c| t|e .|.||res stcck.
CHAP7R 4 APPLICA7IONS OI DMAND AND SUPPLY 95
Case in Point: 9/11 and the Stock Market
2010 jupiterimages Corporation
|e |.ck|rg c| |cu. .|.p|.res .rJ t|e stee.|rg c| t|en |rtc bu||J|rgs |s pe.|.ps t|e cr|y J|s.ste. t|.t |.s be
ccne ur|ve.s.||y krc.r by |ts J.te. Septenbe. 11, 2001|erce, 9/11. 9/11 .||| .en.|r etc|eJ |r cu. cc|
|ect|ve nenc.y |c. . g.e.t n.ry gere..t|crs.
||s.ste.s suc| .s 9/11 .ep.esert t|e k|rJ c| ccnp|ete su.p.|ses t|.t J..n.t|c.||y .|ect stcck p.|ces, || cr|y
tenpc...||y. |e |e. +c.k Stcck |xc|.rge ..s c|cseJ cr t|e J.y c| t|e .tt.ck .rJ .en.|reJ c|cseJ |c. s|x
J.ys. Or t|e J.y t|e n..ket cpereJ, t|e |c. 'cres |rJust.|.| Ave..ge (t|e |O\, . .|Je|y useJ g.uge c|
stcck p.|ces) |e|| re..|y 685 pc|rts tc 8,920. |t ..s cre c| t|e b|ggest creJ.y Jec||re |r |.S. ||stc.y.
\|y J|J t|e .tt.cks cr Septenbe. 11, 2001, |.ve suc| . J..n.t|c s|c.tte.n |np.ct cr t|e stcck n..ket` |e
.tt.cks c| 9/11 p|urgeJ t|e |r|teJ St.tes .rJ nuc| c| t|e .est c| t|e .c.|J |rtc . ve.y |.|g|ter|rg ...
.g.|rst te..c.|sn. |e .e.||..t|cr t|.t te..c.|sts ccu|J st.|ke .ryt|ne .rJ |r .ry p|.ce s.ppeJ ccrsune. .rJ
bus|ress ccr|Jerce .||ke .rJ .|ecteJ bct| t|e Jen.rJ .rJ supp|y c| ncst stccks. |e .tt.cks cr 9/11 p.c
vckeJ |e.. .rJ urce.t.|rtyt.c t||rgs t|.t ..e ce.t.|r tc b.|rg stcck p.|ces Jc.r, .t |e.st urt|| ct|e. everts
.rJ nc.e |r|c.n.t|cr c.use expect.t|crs tc c|.rge .g.|r |r t||s ve.y .espcrs|ve n..ket.
96 PRINCIPLS OI CONOMICS
price 0oor
A n|r|nun .||c..b|e p.|ce
set .bcve t|e e,u|||b.|un
p.|ce.
A N S W R 7 O 7 R Y I 7 ! P R O 8 L M
|e |r|c.n.t|cr g|ver |r t|e p.cb|en suggests t|.t t|e .|.||res p.c|ts ..e ||ke|y tc |.|| be|c. expect.t|crs.
u..ert c.re.s c| t|e .|.||res stcck .rJ pctert|.| buye.s c| t|e stcck .cu|J .Just Jc.r...J t|e|. est|n.tes
c| .|.t t|e v.|ue c| t|e cc.pc..t|crs stcck s|cu|J be. As . .esu|t t|e supp|y cu.ve |c. t|e stcck .cu|J |r
c.e.se, s|||t|rg |t tc t|e .|g|t, .|||e t|e Jen.rJ cu.ve |c. t|e stcck .cu|J Jec.e.se, s|||t|rg |t tc t|e |e|t. As .
.esu|t, e,u|||b.|un p.|ce c| t|e stcck |.||s |.cn |
1
tc |
2
. \|.t |.ppers tc e,u|||b.|un ,u.rt|ty JeperJs cr t|e
extert tc .||c| e.c| cu.ve s|||ts. |r t|e J|.g..n, e,u|||b.|un ,u.rt|ty |s s|c.r tc Jec.e.se |.cn (
1
tc (
2
.
2. GOVERNMENT INTERVENTION IN MARKET PRICES:
PRICE FLOORS AND PRICE CEILINGS
L A R N I N G O 8 1 C 7 I V S
1. Use the modeI of demand and suppIy to expIain what happens when the government imposes
price 0oors or price ceiIings.
2. Discuss the reasons why governments sometimes choose to controI prices and the con-
sequences of price controI poIicies.
So fai in this chaptei and in the pievious chaptei, we have leained that maikets tend to move towaid
theii equilibiium piices and quantities. Suipluses and shoitages of goods aie shoit-lived as piices ad-
just to equate quantity demanded with quantity supplied.
In some maikets, howevei, goveinments have been called on by gioups of citizens to inteivene to
keep piices of ceitain items highei oi lowei than what would iesult fiom the maiket fnding its own
equilibiium piice. In this section we will examine agiicultuial maikets and apaitment iental mai-
kets-two maikets that have often been subject to piice contiols. Thiough these examples, we will
identify the efects of contiolling piices. In each case, we will look at ieasons why goveinments have
chosen to contiol piices in these maikets and the consequences of these policies.
2.1 Agiicultuial Piice Floois
Goveinments often seek to assist faimeis by setting piice foois in agiicultuial maikets. A minimum al-
lowable piice set above the equilibiium piice is a price oor. With a piice fooi, the goveinment foi-
bids a piice below the minimum. (Notice that, if the piice fooi weie foi whatevei ieason set below the
equilibiium piice, it would be iiielevant to the deteimination of the piice in the maiket since nothing
would piohibit the piice fiom iising to equilibiium.) A piice fooi that is set above the equilibiium
piice cieates a suiplus.
Figuie 4.8 shows the maiket foi wheat. Suppose the goveinment sets the piice of wheat at P
F
.
Notice that P
F
is above the equilibiium piice of P
E
. At P
F
, we iead ovei to the demand cuive to fnd
that the quantity of wheat that buyeis will be willing and able to puichase is V
1
bushels. Reading ovei
to the supply cuive, we fnd that selleis will ofei V
2
bushels of wheat at the piice fooi of P
F
. Because
CHAP7R 4 APPLICA7IONS OI DMAND AND SUPPLY 97
II GUR 4. 8 Price IIoors in Wheat
Markets
A p.|ce |cc. |c. .|e.t c.e.tes . su.p|us c|
.|e.t e,u.| tc (/
2
/
1
) bus|e|s.
II GUR 4. 9 SuppIy and Demand Shifts
for AgricuIturaI Products
A .e|.t|ve|y |..ge |rc.e.se |r t|e supp|y c|
.g.|cu|tu..| p.cJucts, .cccnp.r|eJ by .
.e|.t|ve|y sn.|| |rc.e.se |r Jen.rJ, |.s
.eJuceJ t|e p.|ce .ece|veJ by |..ne.s .rJ
|rc.e.seJ t|e ,u.rt|ty c| .g.|cu|tu..| gccJs.
P
F
is above the equilibiium piice, theie is a suiplus of wheat equal to (V
2
V
1
) bushels. The suiplus
peisists because the goveinment does not allow the piice to fall.
Why have many goveinments aiound the woild set piice foois in agiicultuial
maikets: Faiming has changed diamatically ovei the past two centuiies. Technological
impiovements in the foim of new equipment, feitilizeis, pesticides, and new vaiieties
of ciops have led to diamatic incieases in ciop output pei acie. Woildwide pioduction
capacity has expanded maikedly. As we have leained, technological impiovements
cause the supply cuive to shift to the iight, ieducing the piice of food. While such piice
ieductions have been celebiated in computei maikets, faimeis have successfully lob-
bied foi goveinment piogiams aimed at keeping theii piices fiom falling.
While the supply cuive foi agiicultuial goods has shifted to the iight, the demand
has incieased with iising population and with iising income. But as incomes iise,
people spend a smallei and smallei fiaction of theii incomes on food. While the de-
mand foi food has incieased, that inciease has not been neaily as gieat as the inciease
in supply. Figuie 4.9 shows that the supply cuive has shifted much faithei to the iight,
fiom S
1
to S
2
, than the demand cuive has, fiom D
1
to D
2
. As a iesult, equilibiium
quantity has iisen diamatically, fiom Q
1
to Q
2
, and equilibiium piice has fallen, fiom
P
1
to P
2
.
On top of this long-teim histoiical tiend in agiicultuie, agiicultuial piices aie sub-
ject to wide swings ovei shoitei peiiods. Dioughts oi fieezes can shaiply ieduce sup-
plies of paiticulai ciops, causing sudden incieases in piices. Demand foi agiicultuial
goods of one countiy can suddenly diy up if the goveinment of anothei countiy im-
poses tiade iestiictions against its pioducts, and piices can fall. Such diamatic shifts in
piices and quantities make incomes of faimeis unstable.
The Gieat Depiession of the 1930s led to a majoi fedeial iole in agiicultuie. The
Depiession afected the entiie economy, but it hit faimeis paiticulaily haid. Piices ie-
ceived by faimeis plunged neaily two-thiids fiom 1930 to 1933. Many faimeis had a
tough time keeping up moitgage payments. By 1932, moie than half of all faim loans
weie in default.
Faim legislation passed duiing the Gieat Depiession has been modifed many
times, but the fedeial goveinment has continued its diiect involvement in agiicultuial
maikets. This has meant a vaiiety of goveinment piogiams that guaiantee a minimum
piice foi some types of agiicultuial pioducts. These piogiams have been accompanied
by goveinment puichases of any suiplus, by iequiiements to iestiict acieage in oidei to
limit those suipluses, by ciop oi pioduction iestiictions, and the like.
To see how such policies woik, look back at Figuie 4.8. At P
F
, V
2
bushels of wheat
will be supplied. With that much wheat on the maiket, theie is maiket piessuie on the
piice of wheat to fall. To pievent piice fiom falling, the goveinment buys the suiplus of
(V
2
- V
1
) bushels of wheat, so that only V
1
bushels aie actually available to piivate
consumeis foi puichase on the maiket. The goveinment can stoie the suipluses oi fnd
special uses foi them. Foi example, suipluses geneiated in the United States have been
shipped to developing countiies as giants-in-aid oi distiibuted to local school lunch
piogiams. As a vaiiation on this piogiam, the goveinment can iequiie faimeis who
want to paiticipate in the piice suppoit piogiam to ieduce acieage in oidei to limit the
size of the suipluses.
Aftei 1973, the goveinment stopped buying the suipluses (with some exceptions)
and simply guaianteed faimeis a taiget piice." If the aveiage maiket piice foi a ciop
fell below the ciop's taiget piice, the goveinment paid the difeience. If, foi example, a
ciop had a maiket piice of $3 pei unit and a taiget piice of $4 pei unit, the goveinment
would give faimeis a payment of $1 foi each unit sold. Faimeis would thus ieceive the maiket piice of
$3 plus a goveinment payment of $1 pei unit. Foi faimeis to ieceive these payments, they had to agiee
to iemove acies fiom pioduction and to comply with ceitain conseivation piovisions. These iestiic-
tions sought to ieduce the size of the suiplus geneiated by the taiget piice, which acted as a kind of
piice fooi.
What aie the efects of such faim suppoit piogiams: The intention is to boost and stabilize faim
incomes. But, with piice foois, consumeis pay moie foi food than they would otheiwise, and govein-
ments spend heavily to fnance the piogiams. With the taiget piice appioach, consumeis pay less, but
goveinment fnancing of the piogiam continues. U.S. fedeial spending foi agiicultuie aveiaged well
ovei $22 billion pei yeai between 2003 and 2007, ioughly $70 pei peison.
Help to faimeis has sometimes been justifed on the giounds that it boosts incomes of small"
faimeis. Howevei, since faim aid has geneially been allotted on the basis of how much faims pioduce
iathei than on a pei-faim basis, most fedeial faim suppoit has gone to the laigest faims. If the goal is
to eliminate poveity among faimeis, faim aid could be iedesigned to supplement the incomes of small
oi pooi faimeis iathei than to undeimine the functioning of agiicultuial maikets.
98 PRINCIPLS OI CONOMICS
II GUR 4. 10 ffect of a Price CeiIing on
the Market for Apartments
A p.|ce ce|||rg cr .p..tnert .erts t|.t |s set
be|c. t|e e,u|||b.|un .ert c.e.tes . s|c.t.ge
c| .p..tnerts e,u.| tc (^
2
^
1
) .p..tnerts.
price ceiIing
A n.x|nun .||c..b|e p.|ce.
In 1996, the U.S. Congiess passed the Fedeial Agiicultuie Impiovement and Refoim Act of 1996,
oi FAIR. The thiust of the new legislation was to do away with the vaiious piogiams of piice suppoit
foi most ciops and hence piovide incentives foi faimeis to iespond to maiket piice signals. To piotect
faimeis thiough a tiansition peiiod, the act piovided foi continued payments that weie scheduled to
decline ovei a seven-yeai peiiod. Howevei, with piices foi many ciops falling in 1998, the U.S. Con-
giess passed an emeigency aid package that incieased payments to faimeis. In 2008, as faim piices
ieached iecoid highs, Congiess passed a faim bill that incieased subsidy payments to $40 billion. It did,
howevei, foi the fist time limit payments to the wealthiest faimeis. Individual faimeis whose faim in-
comes exceed $730,000 (oi $1.3 million foi couples) would be ineligible foi some subsidy piogiams.
2.2 Rental Piice Ceilings
The puipose of ient contiol is to make iental units cheapei foi tenants than they would otheiwise be.
Unlike agiicultuial piice contiols, ient contiol in the United States has been laigely a local phenomen-
on, although theie weie national ient contiols in efect duiing Woild Wai II. Cuiiently, about 200 cit-
ies and counties have some type of ient contiol piovisions, and about 10 of iental units in the United
States aie now subject to piice contiols. New Yoik City's ient contiol piogiam, which began in 1943, is
among the oldest in the countiy. Many othei cities in the United States adopted some foim of ient con-
tiol in the 1970s. Rent contiols have been peivasive in Euiope since Woild Wai I, and many laige cities
in pooiei countiies have also adopted ient contiols.
Rent contiols in difeient cities difei in teims of theii fexibility. Some cities allow ient incieases
foi specifed ieasons, such as to make impiovements in apaitments oi to allow ients to keep pace with
piice incieases elsewheie in the economy. Often, iental housing constiucted aftei the imposition of the
ient contiol oidinances is exempted. Apaitments that aie vacated may also be decontiolled. Foi simpli-
city, the model piesented heie assumes that apaitment ients aie contiolled at a piice that does not
change.
Figuie 4.10 shows the maiket foi iental apaitments. Notice that the demand and
supply cuives aie diawn to look like all the othei demand and supply cuives you have
encounteied so fai in this text: the demand cuive is downwaid-sloping and the supply
cuive is upwaid-sloping.
The demand cuive shows that a highei piice (ient) ieduces the quantity of apait-
ments demanded. Foi example, with highei ients, moie young people will choose to
live at home with theii paients. With lowei ients, moie will choose to live in apait-
ments. Highei ients may encouiage moie apaitment shaiing; lowei ients would induce
moie people to live alone.
The supply cuive is diawn to show that as ient incieases, piopeity owneis will be
encouiaged to ofei moie apaitments to ient. Even though an aeiial photogiaph of a
city would show apaitments to be fxed at a point in time, owneis of those piopeities
will decide how many to ient depending on the amount of ient they anticipate. Highei
ients may also induce some homeowneis to ient out apaitment space. In addition,
ienting out apaitments implies a ceitain level of seivice to ienteis, so that low ients
may lead some piopeity owneis to keep some apaitments vacant.
Rent contiol is an example of a price ceiling, a maximum allowable piice. With a
piice ceiling, the goveinment foibids a piice above the maximum. A piice ceiling that is
set below the equilibiium piice cieates a shoitage that will peisist.
Suppose the goveinment sets the piice of an apaitment at P
C
in Figuie 4.10. Notice
that P
C
is below the equilibiium piice of P
E
. At P
C
, we iead ovei to the supply cuive to
fnd that selleis aie willing to ofei A
1
apaitments. Reading ovei to the demand cuive,
we fnd that consumeis would like to ient A
2
apaitments at the piice ceiling of P
C
. Be-
cause P
C
is below the equilibiium piice, theie is a shoitage of apaitments equal to (A
2
- A
1
). (Notice
that if the piice ceiling weie set above the equilibiium piice it would have no efect on the maiket since
the law would not piohibit the piice fiom settling at an equilibiium piice that is lowei than the piice
ceiling.)
CHAP7R 4 APPLICA7IONS OI DMAND AND SUPPLY 99
II GUR 4. 11 7he Unintended
Consequences of Rent ControI
crt.c|||rg .p..tnert .erts .t |

c.e.tes .
s|c.t.ge c| (^
2
^
1
) .p..tnerts. |c. ^
1
.p..tnerts, ccrsune.s ..e .||||rg .rJ .b|e tc
p.y |
b
, .||c| |e.Js tc v..|cus b.ckJcc.
p.ynerts tc .p..tnert c.re.s.
If ient contiol cieates a shoitage of apaitments, why do some citizens nonetheless
clamoi foi ient contiol and why do goveinments often give in to the demands: The
ieason geneially given foi ient contiol is to keep apaitments afoidable foi low- and
middle-income tenants.
But the ieduced quantity of apaitments supplied must be iationed in some way,
since, at the piice ceiling, the quantity demanded would exceed the quantity supplied.
Cuiient occupants may be ieluctant to leave theii dwellings because fnding othei
apaitments will be dimcult. As apaitments do become available, theie will be a line of
potential ienteis waiting to fll them, any of whom is willing to pay the contiolled piice
of P
C
oi moie. In fact, ieading up to the demand cuive in Figuie 4.11 fiom A
1
apait-
ments, the quantity available at P
C
, you can see that foi A
1
apaitments, theie aie poten-
tial ienteis willing and able to pay P
B
. This often leads to vaiious backdooi" payments
to apaitment owneis, such as laige secuiity deposits, payments foi things ienteis may
not want (such as fuinituie), so-called key" payments (The monthly ient is $300 and
the key piice is $3,000"), oi simple biibes.
In the end, ient contiols and othei piice ceilings often end up huiting some of the
people they aie intended to help. Many people will have tiouble fnding apaitments to
ient. Iionically, some of those who do fnd apaitments may actually end up paying
moie than they would have paid in the absence of ient contiol. And many of the people
that the ient contiols do help (piimaiily cuiient occupants, iegaidless of theii income,
and those lucky enough to fnd apaitments) aie not those they aie intended to help (the
pooi). Theie aie also costs in goveinment administiation and enfoicement.
Because New Yoik City has the longest histoiy of ient contiols of any city in the
United States, its piogiam has been widely studied. Theie is geneial agieement that the
ient contiol piogiam has ieduced tenant mobility, led to a substantial gap between
ients on contiolled and uncontiolled units, and favoied long-teim iesidents at the expense of new-
comeis to the city.
[2]
These distoitions have giown ovei time, anothei fiequent consequence of piice
contiols.
A moie diiect means of helping pooi tenants, one that would avoid inteifeiing with the function-
ing of the maiket, would be to subsidize theii incomes. As with piice foois, inteifeiing with the maiket
mechanism may solve one pioblem, but it cieates many otheis at the same time.
k Y 7 A k A W A Y S
< |.|ce |cc.s c.e.te su.p|uses by |x|rg t|e p.|ce .bcve t|e e,u|||b.|un p.|ce. At t|e p.|ce set by t|e |cc.,
t|e ,u.rt|ty supp||eJ exceeJs t|e ,u.rt|ty Jen.rJeJ.
< |r .g.|cu|tu.e, p.|ce |cc.s |.ve c.e.teJ pe.s|stert su.p|uses c| . .|Je ..rge c| .g.|cu|tu..| ccnncJ|t|es.
Ccve.rnerts typ|c.||y pu.c|.se t|e .ncurt c| t|e su.p|us c. |npcse p.cJuct|cr .est.|ct|crs |r .r
.ttenpt tc .eJuce t|e su.p|us.
< |.|ce ce|||rgs c.e.te s|c.t.ges by sett|rg t|e p.|ce be|c. t|e e,u|||b.|un. At t|e ce|||rg p.|ce, t|e ,u.rt|ty
Jen.rJeJ exceeJs t|e ,u.rt|ty supp||eJ.
< |ert ccrt.c|s ..e .r ex.np|e c| . p.|ce ce|||rg, .rJ t|us t|ey c.e.te s|c.t.ges c| .ert.| |cus|rg.
< |t |s scnet|nes t|e c.se t|.t .ert ccrt.c|s c.e.te b.ckJcc. ....rgenerts, ..rg|rg |.cn .e,u|.enerts
t|.t ter.rts .ert |tens t|.t t|ey Jc rct ..rt tc cut.|g|t b.|bes, t|.t .esu|t |r .erts ||g|e. t|.r .cu|J
ex|st |r t|e .bserce c| t|e ce|||rg.
7 R Y I 7 !
A n|r|nun ..ge |.. |s .rct|e. ex.np|e c| . p.|ce |cc.. |... Jen.rJ .rJ supp|y cu.ves |c. ursk|||eJ |.bc..
|e |c.|.crt.| .x|s .||| s|c. t|e ,u.rt|ty c| ursk|||eJ |.bc. pe. pe.|cJ .rJ t|e ve.t|c.| .x|s .||| s|c. t|e
|cu.|y ..ge ..te |c. ursk|||eJ .c.ke.s, .||c| |s t|e p.|ce c| ursk|||eJ |.bc.. S|c. .rJ exp|.|r t|e e|ect c| .
n|r|nun ..ge t|.t |s .bcve t|e e,u|||b.|un ..ge.
100 PRINCIPLS OI CONOMICS
Case in Point: Corn: It Is Not 1ust Iood Any More
2010 jupiterimages Corporation
Ccve.rnert suppc.t |c. cc.r J.tes b.ck tc t|e Ag.|cu|tu..| Act c| 1938 .rJ, |r cre |c.n c. .rct|e., |.s beer
p..t c| .g.|cu|tu..| |eg|s|.t|cr eve. s|rce. ypes c| suppc.ts |.ve ..rgeJ |.cn gcve.rnert pu.c|.ses c| su.
p|uses tc t..get p.|c|rg, |.rJ set .s|Jes, .rJ |c.r gu...rtees. Accc.J|rg tc cre est|n.te, t|e |.S. gcve.rnert
spert re..|y 42 b||||cr tc suppc.t cc.r bet.eer 1995 .rJ 2004.
|er, Ju.|rg t|e pe.|cJ c| .|s|rg c|| p.|ces c| t|e |.te 190s .rJ ncurt|rg ccrce.rs .bcut JeperJerce cr
|c.e|gr c|| |.cn vc|.t||e .eg|crs |r t|e .c.|J, suppc.t |c. cc.r, rct .s . |ccJ, but ..t|e. .s .r |rput |rtc t|e
p.cJuct|cr c| et|.rc|.r .|te.r.t|ve tc c||b.seJ |ue|beg.r. |t|.rc| t.x c.eJ|ts .e.e p..t c| t|e |re.gy
Act c| 198. S|rce 1980, . t..|| c| 50 pe. g.||cr .g.|rst |npc.teJ et|.rc|, ever ||g|e. tcJ.y, |.s se.veJ tc
p.ctect Jcnest|c cc.rb.seJ et|.rc| |.cn |npc.teJ et|.rc|, |r p..t|cu|.. |.cn sug..c.reb.seJ et|.rc|
|.cn b...||.
|e |re.gy |c||cy Act c| 2005 ..s .rct|e. n||estcre |r et|.rc| |eg|s|.t|cr. |.cug| |c.r gu...rtees, suppc.t
|c. .ese..c| .rJ Jeve|cpnert, .rJ t.x c.eJ|ts, |t n.rJ.teJ t|.t 4 b||||cr g.||crs c| et|.rc| be useJ by 2006
.rJ .5 b||||cr g.||crs by 2012. |t|.rc| p.cJuct|cr |.J .|.e.Jy .e.c|eJ 6.5 b||||cr g.||crs by 200, sc re. |e
g|s|.t|cr |r 200 uppeJ t|e .rte tc 15 b||||cr g.||crs by 2015.
beycrJ t|e |rc.e.seJ .ncurt t|e gcve.rnert |s sperJ|rg tc suppc.t cc.r .rJ cc.rb.seJ et|.rc|, c.|t|c|sn
c| t|e pc||cy |.s t|.ee n.c. p.crgs.
1. c.rb.seJ et|.rc| Jces ||tt|e tc .eJuce |.S. JeperJerce cr |c.e|gr c|| bec.use t|e ere.gy .e,u|.eJ
tc p.cJuce . g.||cr c| cc.rb.seJ et|.rc| |s ,u|te ||g|. A 2006 |.t|cr.| Ac.Jeny c| Sc|erces p.pe.
est|n.teJ t|.t cre g.||cr c| et|.rc| |s reeJeJ tc b.|rg 1.25 g.||crs c| |t tc n..ket. Ot|e. stuJ|es
s|c. .r ever |ess |.vc..b|e ..t|c.
2. b|c|ue|s, suc| .s cc.rb.seJ et|.rc|, ..e |.v|rg Jet.|nert.| e|ects cr t|e erv|.crnert, .|t|
|rc.e.seJ Je|c.est.t|cr, stenn|rg |.cn nc.e |.rJ be|rg useJ tc g.c. |ue| |rputs, ccrt.|but|rg tc
g|cb.| ...n|rg.
3. |e J|ve.s|cr c| cc.r .rJ ct|e. c.cps |.cn |ccJ tc |ue| |s ccrt.|but|rg tc .|s|rg |ccJ p.|ces .rJ .r
|rc.e.se |r .c.|J |urge.. . |c.J |urge .rJ ber.n|r Ser.ue. ..cte |r |oe| ^|o| t|.t ever
sn.|| |rc.e.ses |r p.|ces c| |ccJ st.p|es |.ve seve.e ccrse,uerces cr t|e ve.y pcc. c| t|e .c.|J, .rJ
|||||rg t|e 25g.||cr t.rk c| .r S|\ .|t| pu.e et|.rc| .e,u|.es cve. 450 pcurJs c| cc.r.||c|
ccrt.|rs ercug| c.|c.|es tc |eeJ cre pe.scr |c. . ye...
Scne c| t|ese c.|t|c|sns n.y be ccrtesteJ .s ex.gge..teJ. \||| t|e ..t|c c| ere.gy|r tc ere.gycut |np.cve
.s re. tec|rc|cg|es ene.ge |c. p.cJuc|rg et|.rc|` ||J rct ct|e. |.ctc.s, suc| .s .e.t|e. .rJ .|s|rg |ccJ
Jen.rJ .c.|J.|Je, ccrt.|bute tc ||g|e. g..|r p.|ces` |cret|e|ess, |t |s c|e.. t|.t cc.rb.seJ et|.rc| |s rc
|.ee |urc|. |t |s .|sc c|e.. t|.t t|e erJ c| gcve.rnert suppc.t |c. cc.r |s rc.|e.e tc be seer.
CHAP7R 4 APPLICA7IONS OI DMAND AND SUPPLY 101
.ooce ^|ee| |o|ooe.o. /oo|o| o| o oJ o .eo o| |o .o|J|e.' |e. `o| |e. |o.ee 9. 2005. o||e .e|o. |o.|J |eJJoo.
|||Je o| ||e o.' |o||oo| |e.|e. ||e. /o, 6. 200S. |oJ |oe oJ |e,o| .eooe. |o. ||o|oe| oo|J .|o.e ||e |oo.' |oe|
^|o|. /o,.oe 200.. o||e .e|o. /|c|oe| Oo.o|J. |e |eo |e, .co.' |e .4 (^|| .. 200S: 4045
A N S W R 7 O 7 R Y I 7 ! P R O 8 L M
A n|r|nun ..ge (/
n|r
) t|.t |s set .bcve t|e e,u|||b.|un ..ge .cu|J c.e.te . su.p|us c| ursk|||eJ |.bc.
e,u.| tc (|
2
|
1
). |.t |s, |
2
ur|ts c| ursk|||eJ |.bc. ..e c|e.eJ .t t|e n|r|nun ..ge, but ccnp.r|es cr|y
..rt tc use |
1
ur|ts .t t|.t ..ge. bec.use ursk|||eJ .c.ke.s ..e . subst|tute |c. . sk|||eJ .c.ke.s, |c.c|rg t|e
p.|ce c| ursk|||eJ .c.ke.s ||g|e. .cu|J |rc.e.se t|e Jen.rJ |c. sk|||eJ |.bc. .rJ t|us |rc.e.se t|e|. ..ges.
3. THE MARKET FOR HEALTH-CARE SERVICES
L A R N I N G O 8 1 C 7 I V
1. Use the modeI of demand and suppIy to expIain the eects of third-party payers on the heaIth-
care market and on heaIth-care spending.
Theie has been much discussion ovei the past thiee decades about the health-caie pioblem in the Un-
ited States. Much of this discussion has focused on iising spending foi health caie. In this section, we
will apply the model of demand and supply to health caie to see what we can leain about some of the
ieasons behind iising spending in this impoitant sectoi of the economy.
Figuie 4.14 shows the shaie of U.S. output devoted to health caie since 1960. In 1960, about 3 of
total output was devoted to health caie; by 2004 this shaie had iisen to 13.4. That has meant that we
aie devoting moie of oui spending to health caie, and less to othei goods and seivices, than we would
be had health-caie spending not iisen so much.
102 PRINCIPLS OI CONOMICS
II GUR 4. 14 HeaIth-Care Spending as a Percentage of U.S. Output, 1960-2003
|e.|t| c..es s|..e c| tct.| |.S. cutput .cse |.cn .bcut 5 |r 1960 tc 15.3 |r 2003.
Data for period 190-1992 from Health Care Finance Association (which was the predecessor to the Centers for Medicare and Medicaid Services),
Data for period 1993-2003 from Centers for Medicare and Medicaid Services, Opce of the Actuary ^ational Health Statistics Group
http//www.cms.hhs.gov/statistics/nhe/historical/t1.asp.
Why weie Ameiicans willing to inciease theii spending on health caie so diamatically: The model of
demand and supply gives us pait of the answei. As we apply the model to this pioblem, we will also
gain a bettei undeistanding of the iole of piices in a maiket economy.
3.1 The Demand and Supply foi Health Caie
When we speak of health caie," we aie speaking of the entiie health-caie industiy. This industiy pio-
duces seivices ianging fiom heait tiansplant opeiations to theiapeutic massages; it pioduces goods
ianging fiom X-iay machines to aspiiin tablets. Cleaily each of these goods and seivices is exchanged
in a paiticulai maiket. To assess the maiket foices afecting health caie, we will focus fist on just one
of these maikets: the maiket foi physician omce visits. When you go to the doctoi, you aie pait of the
demand foi these visits. Youi doctoi, by seeing you, is pait of the supply.
Figuie 4.13 shows the maiket, assuming that it opeiates in a fashion similai to othei maikets. The
demand cuive D
1
and the supply cuive S
1
inteisect at point E, with an equilibiium piice of $30 pei
omce visit. The equilibiium quantity of omce visits pei week is 1,000,000.
CHAP7R 4 APPLICA7IONS OI DMAND AND SUPPLY 103
II GUR 4. 15 7otaI Spending for
Physician Office Visits
ct.| sperJ|rg cr p|ys|c|.r c|ce v|s|ts |s 30
pe. v|s|t nu|t|p||eJ by 1,000,000 v|s|ts pe. .eek,
.||c| e,u.|s 30,000,000. |t |s t|e s|.JeJ ..e.
bcurJeJ by p.|ce .rJ ,u.rt|ty.
third-party payer
Ar .gert ct|e. t|.r t|e
se||e. c. t|e buye. .|c p.ys
p..t c| t|e p.|ce c| . gccJ c.
se.v|ce.
We can use the demand and supply giaph to show total spending, which equals the
piice pei unit (in this case, $30 pei visit) times the quantity consumed (in this case,
1,000,000 visits pei week). Total spending foi physician omce visits thus equals
$30,000,000 pei week ($30 times 1,000,000 visits). We show total spending as the aiea
of a iectangle bounded by the piice and the quantity. It is the shaded iegion in Figuie
4.13.
The pictuie in Figuie 4.13 misses a ciucial featuie of the maiket. Most people in
the United States have health insuiance, piovided eithei by piivate fims, by piivate
puichases, oi by the goveinment. With health insuiance, people agiee to pay a fxed
amount to the insuiei in exchange foi the insuiei's agieement to pay foi most of the
health-caie expenses they incui. While insuiance plans difei in theii specifc piovi-
sions, let us suppose that all individuals have plans that iequiie them to pay $10 foi an
omce visit; the insuiance company will pay the iest.
How will this insuiance afect the maiket foi physician omce visits: If it costs only
$10 foi a visit instead of $30, people will visit theii doctois moie often. The quantity of
omce visits demanded will inciease. In Figuie 4.16, this is shown as a movement along
the demand cuive. Think about youi own choices. When you get a cold, do you go to
the doctoi: Piobably not, if it is a minoi cold. But if you feel like you aie dying, oi wish
you weie, you piobably head foi the doctoi. Cleaily, theie aie lots of colds in between
these two extiemes. Whethei you diag youiself to the doctoi will depend on the sevei-
ity of youi cold and what you will pay foi a visit. At a lowei piice, you aie moie likely to
go to the doctoi; at a highei piice, you aie less likely to go.
In the case shown, the quantity of omce visits iises to 1,300,000 pei week. But that
suggests a potential pioblem. The quantity of visits supplied at a piice of $30 pei visit was 1,000,000.
Accoiding to supply cuive S
1
, it will take a piice of $30 pei visit to inciease the quantity supplied to
1,300,000 visits (Point F on S
1
). But consumeis-patients-pay only $10.
Insuieis make up the difeience between the fees doctois ieceive and the piice patients pay. In oui
example, insuieis pay $40 pei visit of insuied patients to supplement the $10 that patients pay. When
an agent othei than the sellei oi the buyei pays pait of the piice of a good oi seivice, we say that the
agent is a third-party payer.
Notice how the piesence of a thiid-paity payei afects total spending on omce visits. When people
paid foi theii own visits, and the piice equaled $30 pei visit, total spending equaled $30 million pei
week. Now doctois ieceive $30 pei visit and piovide 1,300,000 visits pei week. Total spending has iisen
to $73 million pei week ($30 times 1,300,000 visits, shown by the daikly shaded iegion plus the lightly
shaded iegion).
II GUR 4. 16 7otaI Spending for Physician Office Visits Covered by Insurance
\|t| |rsu..rce, t|e ,u.rt|ty c| p|ys|c|.r c|ce v|s|ts Jen.rJeJ .|ses tc 1,500,000. |e supp|y cu.ve s|c.s t|.t |t
t.kes . p.|ce c| 50 pe. v|s|t tc |rc.e.se t|e ,u.rt|ty supp||eJ tc 1,500,000 v|s|ts. |.t|erts p.y 10 pe. v|s|t .rJ
|rsu..rce p.ys 40 pe. v|s|t. ct.| sperJ|rg .|ses tc 5,000,000 pe. .eek, s|c.r by t|e J..k|y s|.JeJ .eg|cr p|us
t|e ||g|t|y s|.JeJ .eg|cr.
104 PRINCIPLS OI CONOMICS
The iesponse desciibed in Figuie 4.16 holds foi many difeient types of goods and seivices coveied by
insuiance oi otheiwise paid foi by thiid-paity payeis. Foi example, the availability of scholaiships and
subsidized tuition at public and piivate univeisities incieases the quantity of education demanded and
the total expendituies on highei education. In maikets with thiid-paity payeis, an equilibiium is
achieved, but it is not at the inteisection of the demand and supply cuives. The efect of thiid-paity
payeis is to deciease the piice that consumeis diiectly pay foi the goods and seivices they consume and
to inciease the piice that supplieis ieceive. Consumeis use moie than they would in the absence of
thiid-paity payeis, and piovideis aie encouiaged to supply moie than they otheiwise would. The iesult
is incieased total spending.
k Y 7 A k A W A Y S
< |e .|s|rg s|..e c| t|e cutput c| t|e |r|teJ St.tes JevcteJ tc |e.|t| c..e .ep.eserts . .|s|rg cppc.tur|ty
ccst. Vc.e sperJ|rg cr |e.|t| c..e ne.rs |ess sperJ|rg cr ct|e. gccJs .rJ se.v|ces, ccnp..eJ tc .|.t
.cu|J |.ve t..rsp|.eJ |.J |e.|t|c..e sperJ|rg rct .|ser sc nuc|.
< |e ncJe| c| Jen.rJ .rJ supp|y c.r be useJ tc s|c. t|e e|ect c| t||.Jp..ty p.ye.s cr tct.| sperJ|rg.
\|t| t||.Jp..ty p.ye.s (|c. ex.np|e, |e.|t| |rsu.e.s), t|e ,u.rt|ty c| se.v|ces ccrsuneJ .|ses, .s Jces
sperJ|rg.
7 R Y I 7 !
|e p.cv|s|cr c| ur|ve.s|ty eJuc.t|cr t|.cug| t.xp.ye.suppc.teJ st.te ur|ve.s|t|es |s .rct|e. ex.np|e c| .
n..ket .|t| . t||.Jp..ty p.ye.. |se t|e ncJe| c| Jen.rJ .rJ supp|y tc J|scuss t|e |np.ct t||s |.s cr t|e
||g|e. eJuc.t|cr n..ket. Spec||c.||y, J... . g..p| s|n||.. tc ||gu.e 4.16. |c. .cu|J ycu |.be| t|e .xes`
S|c. t|e e,u|||b.|un p.|ce .rJ ,u.rt|ty |r t|e .bserce c| . t||.Jp..ty p.ye. .rJ |rJ|c.te tct.| sperJ|rg cr
eJuc.t|cr. |c. s|c. t|e |np.ct c| |c.e. tu|t|cr As . .esu|t c| st.te suppc.t |c. eJuc.t|cr. |c. nuc| eJuc.
t|cr Jc stuJerts Jen.rJ .t t|e |c.e. tu|t|cr` |c. nuc| tu|t|cr nust eJuc.t|cr.| |rst|tut|crs .ece|ve tc p.c
Juce t|.t nuc| eJuc.t|cr` |c. nuc| sperJ|rg cr eJuc.t|cr .||| cccu.` cnp..e tct.| sperJ|rg be|c.e
.rJ .|te. . t||.Jp..ty p.ye. erte.s t||s n..ket.
Case in Point: 7he Oregon PIan
2010 jupiterimages Corporation
CHAP7R 4 APPLICA7IONS OI DMAND AND SUPPLY 105
|e |e.|t|c..e |rJust.y p.eserts us .|t| . J||enn.. |e..|y, |t n.kes serse |c. pecp|e tc |.ve |e.|t| |rsu.
.rce. 'ust .s c|e..|y, |e.|t| |rsu..rce gere..tes . subst.rt|.| |rc.e.se |r sperJ|rg |c. |e.|t| c..e. || t|.t
sperJ|rg |s tc be ||n|teJ, scne nec|.r|sn nust be c|cser tc Jc |t. Ore nec|.r|sn .cu|J be tc .e,u|.e p.
t|erts tc p.y . |..ge. s|..e c| t|e|. c.r |e.|t|c..e ccrsunpt|cr J|.ect|y, .eJuc|rg t|e p.ynerts n.Je by
t||.Jp..ty p.ye.s. A||c.|rg pecp|e tc .ccunu|.te t.x|.ee p.|v.te neJ|c.| s.v|rgs .cccurts |s cre ..y tc Jc
t||s. Arct|e. cpt|cr |s tc ccrt|rue t|e cu..ert t.erJ tc use |rsu..rce ccnp.r|es .s t|e .gerts t|.t ||n|t
sperJ|rg. A t||.J cpt|cr |s gcve.rnert .egu|.t|cr, t||s .se |r |c|rt Jesc.|bes |c. t|e st.te c| O.egcr t.|eJ
tc ||n|t |e.|t|c..e sperJ|rg by essert|.||y .e|us|rg tc be . t||.Jp..ty p.ye. |c. ce.t.|r se.v|ces.
||ke .|| ct|e. st.tes, O.egcr |.s ..est|eJ .|t| t|e p.cb|en c| sc..|rg VeJ|c.|J ccsts. |ts sc|ut|cr tc t|e p.cb
|en |||ust..tes scne c| t|e c|c|ces scc|ety n|g|t n.ke |r seek|rg tc .eJuce |e.|t|c..e ccsts.
O.egcr useJ tc |.ve . p|.r s|n||.. tc p|.rs |r n.ry ct|e. st.tes. |cuse|c|Js .|cse |rccnes .e.e |c.e. t|.r
50 c| t|e pcve.ty ||re ,u.|||eJ |c. VeJ|c.|J. |r 198, t|e st.te beg.r .r e|c.t tc n.r.ge |ts VeJ|c.|J ccsts.
|t Jec|JeJ t|.t |t .cu|J rc |crge. |urJ c.g.r t..rsp|.rts .rJ t|.t |t .cu|J use t|e ncrey s.veJ tc g|ve bet
te. c..e tc p.egr.rt .cner. |e Jec|s|cr tu.reJ cut tc be . p.|r|u| cre, t|e |.st ye.., . severye..c|J bcy
.|t| |euken|., .|c n|g|t |.ve beer s.veJ .|t| . bcre n...c. t..rsp|.rt, J|eJ. but st.te c|c|.|s ..gueJ
t|.t t|e s|||t c| experJ|tu.es tc p.egr.rt .cner .cu|J u|t|n.te|y s.ve nc.e ||ves.
|e st.te g..Ju.||y exp.rJeJ |ts ccrcept c| Jete.n|r|rg .|.t se.v|ces tc |urJ .rJ .|.t se.v|ces rct tc |urJ.
|t cc||.pseJ . ||st c| 10,000 J||e.ert J|.grcses t|.t |.J beer subn|tteJ tc |ts VeJ|c.|J p.cg..n |r t|e p.st |r
tc . ||st c| nc.e t|.r 00 ccrJ|t|crt.e.tnert p.|.s. Ore suc| p.|., |c. ex.np|e, |s .pperJ|c|t|s
.pperJectcny. |e.|t|c..e c|c|.|s t|er ..rkeJ t|ese p.|.s |r c.Je. c| p.|c.|ty. |e ..rk|rgs .e.e b.seJ cr
suc| |.ctc.s .s t|e se.|cusress c| . p..t|cu|.. ccrJ|t|cr .rJ t|e ccst .rJ e|c.cy c| t.e.tnerts. |e st.te .r
rcurceJ t|.t |t .cu|J p.cv|Je VeJ|c.|J tc .|| |cuse|c|Js be|c. t|e pcve.ty ||re, but t|.t |t .cu|J rct |urJ
.ry p.cceJu.e ..rkeJ be|c. . ce.t.|r |eve|, |r|t|.||y runbe. 588 cr |ts ||st. |e p|.r .|sc set . buJget ||n|t |c.
.ry cre ye.., || sperJ|rg .cse .bcve t|.t ||n|t, t|e |eg|s|.tu.e nust .pp.cp.|.te .JJ|t|cr.| ncrey c. J.cp .J
J|t|cr.| p.cceJu.es |.cn t|e ||st c| t|cse ccve.eJ by t|e p|.r. |e O.egcr |e.|t| ||.r c|c|.||y beg.r cpe..
t|cr |r 1994.
\|||e t|e O.egcr p|.r |.s beer .pp||eJ cr|y tc |cuse|c|Js be|c. t|e pcve.ty ||re t|.t ..e rct ccve.eJ by
ct|e. p.cg..ns, |t suggests . ne.rs c| .eJuc|rg |e.|t|c..e sperJ|rg. |e..|y, || p..t c| t|e |e.|t|c..e p.cb
|en |s excess|ve p.cv|s|cr c| se.v|ces, . systen Jes|greJ tc cut se.v|ces nust Jete.n|re .|.t t.e.tnerts rct
tc |urJ.
|.c|essc.s 'cr.t|.r Obe.|.rJe., |ecJc.e V..nc., .rJ |...erce '.ccbs stuJ|eJ t|e |np.ct c| t||s p|.r |r
p..ct|ce t|.cug| t|e ye.. 2000 .rJ |curJ t|.t, |r ccrt..st tc |r|t|.| expect.t|crs, exc|uJeJ p.cceJu.es .e.e
gere..||y cres c| n..g|r.| neJ|c.| v.|ue, sc t|e ||re |r t|e s.rJ |.J ||tt|e p..ct|c.| s|gr||c.rce. |r .JJ|t|cr,
t|ey |curJ t|.t p.t|erts .e.e c|ter .b|e tc .ece|ve suppcseJ|y exc|uJeJ se.v|ces .|er p|ys|c|.rs, |c. ex
.np|e, t.e.teJ .r urccve.eJ |||ress |r ccrurct|cr .|t| . ccve.eJ cre. |u.|rg t|e pe.|cJ c| t|e stuJy, t|e
runbe. c| pecp|e ccve.eJ by t|e p|.r exp.rJeJ subst.rt|.||y .rJ yet ..t|cr|rg c| se.v|ces essert|.||y J|J rct
cccu.. |c. Jc t|ey exp|.|r t||s seen|rg ccrt..J|ct|cr` u|te s|np|y. st.te gcve.rnert |rc.e.seJ .everues
|.cn v..|cus scu.ces tc suppc.t t|e p|.r. |rJeeJ, t|ey ..gue t|.t, bec.use t.e.tnerts t|.t n|g|t rct be |r
c|uJeJ .e.e exp||c|t|y st.teJ, pc||t|c.| p.essu.e n.Je exc|uJ|rg t|en ever nc.e J||cu|t .rJ n.y |.ve |r.J
ve.tert|y |rc.e.seJ t|e ccst c| t|e p.cg..n.
|r t|e e..|y 2000s, O.egcr, ||ke n.ry ct|e. st.tes, ccr|.crteJ seve.e buJget..y p.essu.es. c ||n|t sperJ|rg, |t
c|cse t|e pe.|.ps |ess v|s|b|e st..tegy c| .eJuc|rg t|e runbe. c| pecp|e ccve.eJ t|.cug| t|e p|.r. Orce
se.v|rg nc.e t|.r 100,000 pecp|e, buJget cuts .eJuceJ t|e runbe. se.veJ tc .bcut 1,000. \|e.e.s |r 1996,
11 c| O.egcr|.rs |.ckeJ |e.|t| |rsu..rce, |r 2008 16 J|J.
..||b|..|rg .g.|r, |r 2008 O.egcr .e.||.eJ t|.t |ts buJget .||c.eJ .ccn |c. ccve..ge |c. . |e. t|cus.rJ .JJ|
t|cr.| pecp|e. but |c. tc c|ccse .ncrg t|e 130,000 e||g|b|es` |e sc|ut|cr. tc |c|J . |ctte.y. Vc.e t|.r
90,000 pecp|e ,ueueJ up, |cp|rg tc be |ucky .|rre.s.
.ooce .oo||o e|oJe. |eoJoe /oo. oJ |o.ece .oco. |o||o| /eJ|co| oe ||e|o|c oJ |eo|||, | ||e eo |eo||| ||o.'
ooJ|o /eJ|co| ^oc|o||o .ooo| 64 (/o, 29. 200: 5S35S.. /||||o`oJ|e,. |o.| |o| |o |eo||| oe.' |e |e.`o| |e. /oc|
3. 200S ^2
106 PRINCIPLS OI CONOMICS
A N S W R 7 O 7 R Y I 7 ! P R O 8 L M
\|t|cut . t||.Jp..ty p.ye. |c. eJuc.t|cr, t|e g..p| s|c.s e,u|||b.|un tu|t|cr c| |
1
.rJ e,u|||b.|un ,u.rt|ty
c| eJuc.t|cr c| (
1
. St.te suppc.t |c. eJuc.t|cr |c.e.s tu|t|cr t|.t stuJerts p.y tc |
2
. As . .esu|t, stuJerts Je
n.rJ (
2
ccu.ses pe. ye... c p.cv|Je t|.t .ncurt c| eJuc.t|cr, eJuc.t|cr.| |rst|tut|crs .e,u|.e tu|t|cr pe.
ccu.se c| |
3
. \|t|cut . t||.Jp..ty p.ye., sperJ|rg cr eJuc.t|cr |s 0|
1
|(
1
. \|t| . t||.Jp..ty p.ye., sperJ|rg
.|ses tc 0|
3
|(
2
.
4. REVIEW AND PRACTICE
Summary
|r t||s c|.pte. .e useJ t|e tcc|s c| Jen.rJ .rJ supp|y tc urJe.st.rJ . .|Je v..|ety c| n..ket cutccnes. \e
|e..reJ t|.t tec|rc|cg|c.| c|.rge .rJ t|e ert.y c| re. se||e.s |.s c.useJ t|e supp|y cu.ve c| pe.scr.| ccn
pute.s tc s|||t n..keJ|y tc t|e .|g|t, t|e.eby .eJuc|rg e,u|||b.|un p.|ce .rJ |rc.e.s|rg e,u|||b.|un ,u.rt|ty.
V..ket |c.ces |.ve n.Je pe.scr.| ccnpute.s . ccnncr |ten |r c|ces .rJ |cnes.
.uJe c|| .rJ g.sc||re p.|ces sc..eJ |r 2008 .rJ t|er |e|| b.ck. \e |cckeJ .t t|e c.uses c| t|ese |rc.e.ses .s
.e|| .s t|e|. |np.cts. .uJe c|| p.|ces .cse |r |..ge p..t As . .esu|t c| |rc.e.seJ Jen.rJ, p..t|cu|..|y |.cn |
|r.. ||g|e. p.|ces |c. c.uJe c|| |eJ tc ||g|e. p.|ces |c. g.sc||re. |cse ||g|e. p.|ces rct cr|y |u.t ccrsune.s c|
g.sc||re, t|ey .|sc put up...J p.essu.e cr t|e p.|ces c| . .|Je ..rge c| gccJs .rJ se.v|ces. .uJe c|| .rJ
g.sc||re p.|ces t|er Jec.e.seJ J..n.t|c.||y |r t|e |.st p..t c| 2008, .s .c.|J g.c.t| Jec||reJ.
|e ncJe| c| Jen.rJ .rJ supp|y .|sc exp|.|rs t|e Jete.n|r.t|cr c| stcck p.|ces. |e p.|ce pe. s|..e c| cc.
pc..te stcck .e|ects t|e n..kets est|n.te c| t|e expecteJ p.c|t.b|||ty c| t|e |.n. Ary |r|c.n.t|cr .bcut t|e
|.n t|.t c.uses pctert|.| buye.s c. cu..ert c.re.s c| cc.pc..te stcck tc .eev.|u.te |c. p.c|t.b|e t|ey t||rk
t|e |.n |s, c. .||| be, .||| c.use t|e e,u|||b.|un p.|ce c| t|e stcck tc c|.rge.
\e t|er ex.n|reJ n..kets |r .||c| scne |c.n c| gcve.rnert p.|ce ccrt.c| keeps p.|ce pe.n.rert|y .bcve
c. be|c. e,u|||b.|un. A p.|ce |cc. |e.Js tc pe.s|stert su.p|uses bec.use |t |s set .bcve t|e e,u|||b.|un p.|ce,
.|e.e.s . p.|ce ce|||rg, bec.use |t |s set be|c. t|e e,u|||b.|un p.|ce, |e.Js tc pe.s|stert s|c.t.ges. \e s..
t|.t |rte.|e.|rg .|t| t|e n..ket nec|.r|sn n.y sc|ve cre p.cb|en but c|ter c.e.tes ct|e. p.cb|ens .t t|e
s.ne t|ne. \e J|scusseJ .|.t scne c| t|ese ur|rterJeJ ccrse,uerces n|g|t be. |c. ex.np|e, .g.|cu|tu..|
p.|ce |cc.s .|neJ .t bccst|rg |..n |rccne |.ve .|sc ..|seJ p.|ces |c. ccrsune.s .rJ ccst t.xp.ye.s Je..|y,
.rJ t|e bu|k c| gcve.rnert p.ynerts |.ve gcre tc |..ge |..ns. |ert ccrt.c|s |.ve |c.e.eJ .erts, but t|ey
|.ve .|sc .eJuceJ t|e ,u.rt|ty c| .ert.| |cus|rg supp||eJ, c.e.teJ s|c.t.ges, .rJ scnet|nes |eJ tc v..|cus
|c.ns c| b.ckJcc. p.ynerts, .||c| scnet|nes |c.ce t|e p.|ce c| .ert.| |cus|rg .bcve .|.t .cu|J ex|st |r
t|e .bserce c| ccrt.c|s.
||r.||y, .e |cckeJ .t t|e n..ket |c. |e.|t| c..e .rJ . spec|.| |e.tu.e be||rJ Jen.rJ .rJ supp|y |r t||s n..ket
t|.t |e|ps tc exp|.|r .|y t|e s|..e c| cutput c| t|e |r|teJ St.tes t|.t |s JevcteJ tc |e.|t| c..e |.s .|ser.
|e.|t| c..e |s .r ex.np|e c| . n..ket |r .||c| t|e.e ..e t||.Jp..ty p.ye.s (p.|n..||y p.|v.te |rsu.e.s .rJ t|e
gcve.rnert). \|t| t||.Jp..ty p.ye.s t|e ,u.rt|ty c| |e.|t|c..e se.v|ces ccrsuneJ .|ses, .s Jces |e.|t|c..e
sperJ|rg.
CHAP7R 4 APPLICA7IONS OI DMAND AND SUPPLY 107
C O N C P 7 P R O 8 L M S
1. ||ke pe.scr.| ccnpute.s, J|g|t.| c.ne..s |.ve beccne . ccnncr |cuse|c|J |ten. ||g|t.| c.ne.. p.|ces
|.ve p|urgeJ |r t|e |.st 10 ye..s. |se t|e ncJe| c| Jen.rJ .rJ supp|y tc exp|.|r t|e |.|| |r p.|ce .rJ
|rc.e.se |r ,u.rt|ty.
2. |r.cr c.p. ..s cre c| seve..| cc.pc..t|crs ccrv|cteJ c| |..uJ |r |ts .cccurt|rg p..ct|ces Ju.|rg t|e e..|y
p..t c| t||s Jec.Je. |t |.J c.e.teJ Junny cc.pc..t|crs tc ||Je n.ss|ve bc..c.|rg .rJ tc g|ve |t t|e
.ppe...rce c| ext..c.J|r..y p.c|t.b|||ty. |se t|e ncJe| c| Jen.rJ .rJ supp|y tc exp|.|r t|e ||ke|y
|np.ct c| suc| ccrv|ct|crs cr t|e stccks c| ct|e. cc.pc..t|crs.
3. |u.|rg \c.|J \.. || t|e.e ..s . |.ee.e cr ..ges, .rJ cc.pc..t|crs |curJ t|ey ccu|J ev.Je t|e |.ee.e by
p.cv|J|rg ct|e. |.|rge bere|ts suc| .s .et|.enert |urJs |c. t|e|. enp|cyees. |e O|ce c| |.|ce
AJn|r|st..t|cr, .||c| .Jn|r|ste.eJ t|e ..ge |.ee.e, .u|eJ t|.t t|e c|e. c| .et|.enert |urJs ..s rct .
v|c|.t|cr c| t|e |.ee.e. |e |rte.r.| |everue Se.v|ce .ert .|crg .|t| t||s .rJ .u|eJ t|.t enp|cye.
|r.rceJ .et|.enert p|.rs .e.e rct t.x.b|e |rccne. \.s t|e ..ge |.ee.e .r ex.np|e c| . p.|ce |cc. c. .
p.|ce ce|||rg` |se t|e ncJe| c| Jen.rJ .rJ supp|y tc exp|.|r .|y enp|cye.s beg.r tc c|e. suc|
bere|ts tc t|e|. enp|cyees.
4. |e text ..gues t|.t pc||t|c.| |rst.b|||ty |r pctert|.| supp||e.s c| c|| suc| .s |.., .rJ \ere.ue|. .cccurts |c.
. .e|.t|ve|y steep supp|y cu.ve |c. c.uJe c|| suc| .s t|e cre s|c.r |r ||gu.e 4.2 Suppcse t|.t t||s
|rst.b|||ty e.ses ccrs|Je..b|y .rJ t|.t t|e .c.|J supp|y cu.ve |c. c.uJe c|| beccnes nuc| |.tte.. |...
suc| . cu.ve, .rJ exp|.|r |ts |np||c.t|crs |c. t|e .c.|J eccrcny .rJ |c. typ|c.| ccrsune.s.
5. Suppcse t|.t tec|rc|cg|c.| c|.rge .|ects t|e J.|.y |rJust.y |r t|e s.ne ..y |t |.s .|ecteJ t|e
ccnpute. |rJust.y. |c.eve., suppcse t|.t J.|.y p.|ce suppc.ts .en.|r |r p|.ce. |c. .cu|J t||s .|ect
gcve.rnert sperJ|rg cr t|e J.|.y p.cg..n` |se t|e ncJe| c| Jen.rJ .rJ supp|y tc suppc.t ycu.
.rs.e..
6. |ecp|e c|ter ..gue t|.t t|e.e |s . s|c.t.ge c| c|||J c..e. |s|rg t|e ncJe| c| Jen.rJ .rJ supp|y,
ev.|u.te .|et|e. t||s ..gunert |s ||ke|y tc be cc..ect.
. |u.|rg ncst c| t|e p.st 50 ye..s t|e |r|teJ St.tes |.s |.J . su.p|us c| |..ne.s, .rJ t||s |.s beer t|e
.cct c| t|e |..n p.cb|en. cnnert.
8. Suppcse t|e |ep..tnert c| Ag.|cu|tu.e c.Je.eJ .|| |..ne.s tc .eJuce t|e .c.e.ge t|ey p|.rt by 10.
\cu|J ycu expect . 10 .eJuct|cr |r |ccJ p.cJuct|cr` \|y c. .|y rct`
9. |e text ..gues t|.t t|e |rc.e.se |r g.sc||re p.|ces |.J . p..t|cu|..|y st.crg |np.ct cr |c.|rccne
pecp|e. |.ne scne ct|e. gccJs .rJ se.v|ces |c. .||c| . s|..p |rc.e.se |r p.|ce .cu|J |.ve . s|n||..
|np.ct cr pecp|e .|t| |c. |rccnes.
10. Suppcse t|.t t|e |r|teJ St.tes .rJ t|e |u.cpe.r |r|cr |npcse . p.|ce ce|||rg cr c.uJe c|| c| 25 pe.
b...e|. |xp|.|r, .rJ |||ust..te g..p||c.||y, |c. t||s .cu|J .|ect t|e n..kets |c. c.uJe c|| .rJ |c. g.sc||re |r
t|e |r|teJ St.tes .rJ |r t|e |u.cpe.r |r|cr.
11. C|ver t|.t .ert ccrt.c|s c.r .ctu.||y |u.t |c.|rccne pecp|e, Jev|se . |cus|rg st..tegy t|.t .cu|J
p.cv|Je .|c.J.b|e |cus|rg |c. t|cse .|cse |rccnes |.|| be|c. t|e pcve.ty ||re (|r 2004, t||s ..s .bcut
19,000 |c. . |.n||y c| |cu.).
12. |s|rg t|e ncJe| c| Jen.rJ .rJ supp|y, s|c. .rJ exp|.|r |c. .r |rc.e.se |r t|e s|..e |rJ|v|Ju.|s nust
p.y J|.ect|y |c. neJ|c.| c..e .|ects t|e ,u.rt|ty t|ey ccrsune. |xp|.|r |c. t||s .cu|J .JJ.ess t|e tct.|
.ncurt c| sperJ|rg cr |e.|t| c..e.
13. C|ver t|.t pecp|e p.y p.en|uns |c. t|e|. |e.|t| |rsu..rce, |c. c.r .e s.y t|.t |rsu..rce |c.e.s t|e
p.|ces pecp|e p.y |c. |e.|t|c..e se.v|ces`
14. Suppcse t|.t p|ys|c|.rs rc. c|..ge 30 |c. .r c|ce v|s|t .rJ |rsu..rce pc||c|es .e,u|.e p.t|erts tc p.y
33 1/3 c| t|e .ncurt t|ey p.y t|e p|ys|c|.rs, sc t|e cutc|pccket ccst tc ccrsune.s |s 10 pe. v|s|t. |r
.r e|c.t tc ccrt.c| ccsts, t|e gcve.rnert |npcses . p.|ce ce|||rg c| 2 pe. c|ce v|s|t. |s|rg . Jen.rJ
.rJ supp|y ncJe|, s|c. |c. t||s pc||cy .cu|J .|ect t|e n..ket |c. |e.|t| c..e.
15. |c ycu t||rk t|e |.S. |e.|t|c..e systen .e,u|.es .e|c.n` \|y c. .|y rct` || ycu t||rk .e|c.n |s |r c.Je.,
exp|.|r t|e .pp.c.c| tc .e|c.n ycu .Jvcc.te.
108 PRINCIPLS OI CONOMICS
N U M R I C A L P R O 8 L M S
|.cb|ens 14 ..e b.seJ cr t|e |c||c.|rg Jen.rJ .rJ supp|y sc|eJu|es |c. cc.r (.|| ,u.rt|t|es ..e |r n||||crs
c| bus|e|s pe. ye..).
Price per busheI Quantity demanded Quantity suppIied
0 6 0
1 5 1
2 4 2
3 3 3
4 2 4
5 1 5
6 0 6
1. |... t|e Jen.rJ .rJ supp|y cu.ves |c. cc.r. \|.t |s t|e e,u|||b.|un p.|ce` |e e,u|||b.|un ,u.rt|ty`
2. Suppcse t|e gcve.rnert rc. |npcses . p.|ce |cc. .t 4 pe. bus|e|. S|c. t|e e|ect c| t||s p.cg..n
g..p||c.||y. |c. |..ge |s t|e su.p|us c| cc.r`
3. \|t| t|e p.|ce |cc., |c. nuc| Jc |..ne.s .ece|ve |c. t|e|. cc.r` |c. nuc| .cu|J t|ey |.ve .ece|veJ ||
t|e.e .e.e rc p.|ce |cc.`
4. || t|e gcve.rnert buys .|| t|e su.p|us .|e.t, |c. nuc| .||| |t sperJ`
|.cb|ens 59 ..e b.seJ cr t|e |c||c.|rg |ypct|et|c.| Jen.rJ .rJ supp|y cu.ves |c. .p..tnerts
Rent/Month Number of Apts.
Demanded/Month
Number of Apts.
SuppIied/Month
0 120,000 0
200 100,000 20,000
400 80,000 40,000
600 60,000 60,000
800 40,000 80,000
1000 20,000 100,000
1200 0 120,000
5. |... t|e Jen.rJ .rJ supp|y cu.ves |c. .p..tnerts.
6. \|.t |s t|e e,u|||b.|un .ert pe. ncrt|` At t||s .ert, .|.t |s t|e runbe. c| .p..tnerts Jen.rJeJ .rJ
supp||eJ pe. ncrt|`
. Suppcse . ce|||rg cr .erts |s set .t 400 pe. ncrt|. |...cte.|.e t|e s|tu.t|cr t|.t .esu|ts |.cn t||s pc||cy.
8. At t|e .ert ce|||rg, |c. n.ry .p..tnerts ..e Jen.rJeJ` |c. n.ry ..e supp||eJ`
9. |c. nuc| ..e pecp|e .||||rg tc p.y |c. t|e runbe. c| .p..tnerts supp||eJ .t t|e ce|||rg` |esc.|be t|e
....rgenerts tc .||c| t||s s|tu.t|cr n|g|t |e.J.
CHAP7R 4 APPLICA7IONS OI DMAND AND SUPPLY 109
1.
2.
ENDNOTES
||kk. ucn|, |e ||ves .rJ |e.t| c| Vcc.es |... |ttp.//....|.stncrJ.y.c.g/
|ssues/|ssue_11/tucn|/|rJex. ||| /oJo, (|ttp.//....|.stncrJ.y.c.g) |s . pee.
.ev|e.eJ cu.r.| cr t|e |rte.ret.
||c|..J A.rctt, |ne |c. |ev|s|cr|sn cr |ert crt.c|, .ooo| o| |coo|c |eec|
|.e 9(1) (\|rte., 1995). 99120.
110 PRINCIPLS OI CONOMICS
eIasticity
|e ..t|c c| t|e pe.cert.ge
c|.rge |r . JeperJert
v..|.b|e tc . pe.cert.ge
c|.rge |r .r |rJeperJert
v..|.b|e.
| A | | | 5
Elasticity: A Measure of
Response
S7AR7 UP: RAIS IARS1 LOWR IARS1 WHA7'S A
PU8LIC 7RANSI7 MANAGR 7O DO1
|n.g|re t|.t ycu ..e t|e n.r.ge. c| t|e pub||c t..rspc.t.t|cr systen |c. . |..ge net.cpc||t.r ..e.. Ope..t|rg
ccsts |c. t|e systen |.ve sc..eJ |r t|e |.st |e. ye..s, .rJ ycu ..e urJe. p.essu.e tc bccst .everues. \|.t Jc ycu
Jc`
Ar cbv|cus c|c|ce .cu|J be tc ..|se |..es. |.t .||| n.ke ycu. custcne.s .rg.y, but .t |e.st |t .||| gere..te t|e
ext.. .everue ycu reeJc. .||| |t` |e |.. c| Jen.rJ s.ys t|.t ..|s|rg |..es .||| .eJuce t|e runbe. c| p.sserge.s
.|J|rg cr ycu. systen. || t|e runbe. c| p.sserge.s |.||s cr|y . ||tt|e, t|er t|e ||g|e. |..es t|.t ycu. .en.|r|rg p.s
serge.s ..e p.y|rg n|g|t p.cJuce t|e ||g|e. .everues ycu reeJ. but .|.t || t|e runbe. c| p.sserge.s |.||s by sc
nuc| t|.t ycu. ||g|e. |..es .ctu.||y .eJuce ycu. .everues` || t|.t |.ppers, ycu .||| |.ve n.Je ycu. custcne.s
n.J .rJ ycu. |r.rc|.| p.cb|en .c.se!
V.ybe ycu s|cu|J .eccnnerJ |o.e |..es. A|te. .||, t|e |.. c| Jen.rJ .|sc s.ys t|.t |c.e. |..es .||| |rc.e.se
t|e runbe. c| p.sserge.s. |.v|rg nc.e pecp|e use t|e pub||c t..rspc.t.t|cr systen ccu|J nc.e t|.r c|set .
|c.e. |..e ycu cc||ect |.cn e.c| pe.scr. but |t n|g|t rct. \|.t .||| ycu Jc`
+cu. cb .rJ t|e |sc.| |e.|t| c| t|e pub||c t..rs|t systen ..e .|J|rg cr ycu. n.k|rg t|e cc..ect Jec|s|cr. c Jc
sc, ycu reeJ tc krc. ust |c. .espcrs|ve t|e ,u.rt|ty Jen.rJeJ |s tc . p.|ce c|.rge. +cu reeJ . ne.su.e c|
.espcrs|veress.
|ccrcn|sts use . ne.su.e c| .espcrs|veress c.||eJ e|.st|c|ty. Iasticity |s t|e ..t|c c| t|e pe.cert.ge c|.rge
|r . JeperJert v..|.b|e tc . pe.cert.ge c|.rge |r .r |rJeperJert v..|.b|e. || t|e JeperJert v..|.b|e |s ,, .rJ t|e
|rJeperJert v..|.b|e |s , t|er t|e e|.st|c|ty c| , .|t| .espect tc . c|.rge |r |s g|ver by.
e
y, x
=
change in y
change in x
A v..|.b|e suc| .s , |s s.|J tc be nc.e e|.st|c (.espcrs|ve) || t|e pe.cert.ge c|.rge |r , |s |..ge .e|.t|ve tc t|e
pe.cert.ge c|.rge |r . |t |s |ess e|.st|c || t|e .eve.se |s t.ue.
As n.r.ge. c| t|e pub||c t..rs|t systen, |c. ex.np|e, ycu .||| ..rt tc krc. |c. .espcrs|ve t|e runbe. c|
p.sserge.s cr ycu. systen (t|e JeperJert v..|.b|e) .||| be tc . c|.rge |r |..es (t|e |rJeperJert v..|.b|e). |e
ccrcept c| e|.st|c|ty .||| |e|p ycu sc|ve ycu. pub||c t..rs|t p.|c|rg p.cb|en .rJ . g.e.t n.ry ct|e. |ssues |r ecc
rcn|cs. \e .||| ex.n|re seve..| e|.st|c|t|es |r t||s c|.pte..|| .||| te|| us |c. .espcrs|ve cre v..|.b|e |s tc .
c|.rge |r .rct|e..
price eIasticity of demand
|e pe.cert.ge c|.rge |r
,u.rt|ty Jen.rJeJ c| .
p..t|cu|.. gccJ c. se.v|ce
J|v|JeJ by t|e pe.cert.ge
c|.rge |r t|e p.|ce c| t|.t
gccJ c. se.v|ce, .|| ct|e.
t||rgs urc|.rgeJ.
1. THE PRICE ELASTICITY OF DEMAND
L A R N I N G O 8 1 C 7 I V S
1. xpIain the concept of price eIasticity of demand and its caIcuIation.
2. xpIain what it means for demand to be price ineIastic, unit price eIastic, price eIastic, perfectIy
price ineIastic, and perfectIy price eIastic.
3. xpIain how and why the vaIue of the price eIasticity of demand changes aIong a Iinear de-
mand curve.
4. Understand the reIationship between totaI revenue and price eIasticity of demand.
5. Discuss the determinants of price eIasticity of demand.
We know fiom the law of demand how the quantity demanded will iespond to a piice change: it will
change in the opposite diiection. But how much will it change: It seems ieasonable to expect, foi ex-
ample, that a 10 change in the piice chaiged foi a visit to the doctoi would yield a difeient peicent-
age change in quantity demanded than a 10 change in the piice of a Foid Mustang. But how much is
this difeience:
To show how iesponsive quantity demanded is to a change in piice, we apply the concept of elasti-
city. The price elasticity of demand foi a good oi seivice, e
D
, is the peicentage change in quantity
demanded of a paiticulai good oi seivice divided by the peicentage change in the piice of that good oi
seivice, all othei things unchanged. Thus we can wiite
QUA7I ON 5. 2
e
D
=
change in quantity demanded
change in piice
Because the piice elasticity of demand shows the iesponsiveness of quantity demanded to a piice
change, assuming that othei factois that infuence demand aie unchanged, it iefects movements along
a demand cuive. With a downwaid-sloping demand cuive, piice and quantity demanded move in op-
posite diiections, so the piice elasticity of demand is always negative. A positive peicentage change in
piice implies a negative peicentage change in quantity demanded, and vice veisa. Sometimes you will
see the absolute value of the piice elasticity measuie iepoited. In essence, the minus sign is ignoied be-
cause it is expected that theie will be a negative (inveise) ielationship between quantity demanded and
piice. In this text, howevei, we will ietain the minus sign in iepoiting piice elasticity of demand and
will say the absolute value of the piice elasticity of demand" when that is what we aie desciibing.
Heads Up!
be c..e|u| rct tc ccr|use e|.st|c|ty .|t| s|cpe. |e s|cpe c| . ||re |s t|e c|.rge |r t|e v.|ue c| t|e v..|.b|e cr
t|e ve.t|c.| .x|s J|v|JeJ by t|e c|.rge |r t|e v.|ue c| t|e v..|.b|e cr t|e |c.|.crt.| .x|s bet.eer t.c pc|rts.
||.st|c|ty |s t|e ..t|c c| t|e pe.cert.ge c|.rges. |e s|cpe c| . Jen.rJ cu.ve, |c. ex.np|e, |s t|e ..t|c c| t|e
c|.rge |r p.|ce tc t|e c|.rge |r ,u.rt|ty bet.eer t.c pc|rts cr t|e cu.ve. |e p.|ce e|.st|c|ty c| Jen.rJ |s
t|e ..t|c c| t|e pe.cert.ge c|.rge |r ,u.rt|ty tc t|e pe.cert.ge c|.rge |r p.|ce. As .e .||| see, .|er ccn
put|rg e|.st|c|ty .t J||e.ert pc|rts cr . ||re.. Jen.rJ cu.ve, t|e s|cpe |s ccrst.rtt|.t |s, |t Jces rct
c|.rgebut t|e v.|ue |c. e|.st|c|ty .||| c|.rge.
1.1 Computing the Piice Elasticity of Demand
Finding the piice elasticity of demand iequiies that we fist compute peicentage changes in piice and
in quantity demanded. We calculate those changes between two points on a demand cuive.
Figuie 3.1 shows a paiticulai demand cuive, a lineai demand cuive foi public tiansit iides. Sup-
pose the initial piice is $0.80, and the quantity demanded is 40,000 iides pei day; we aie at point A on
the cuive. Now suppose the piice falls to $0.70, and we want to iepoit the iesponsiveness of the quant-
ity demanded. We see that at the new piice, the quantity demanded iises to 60,000 iides pei day (point
B). To compute the elasticity, we need to compute the peicentage changes in piice and in quantity de-
manded between points A and B.
112 PRINCIPLS OI CONOMICS
arc eIasticity
Ve.su.e c| e|.st|c|ty b.seJ
cr pe.cert.ge c|.rges
.e|.t|ve tc t|e .ve..ge v.|ue
c| e.c| v..|.b|e bet.eer t.c
pc|rts.
II GUR 5. 1 Responsiveness and Demand
|e Jen.rJ cu.ve s|c.s |c. c|.rges |r p.|ce |e.J tc c|.rges |r t|e ,u.rt|ty Jen.rJeJ. A ncvenert |.cn
pc|rt A tc pc|rt b s|c.s t|.t . 0.10 .eJuct|cr |r p.|ce |rc.e.ses t|e runbe. c| .|Jes pe. J.y by 20,000. A
ncvenert |.cn b tc A |s . 0.10 |rc.e.se |r p.|ce, .||c| .eJuces ,u.rt|ty Jen.rJeJ by 20,000 .|Jes pe. J.y.
We measuie the peicentage change between two points as the change in the vaiiable divided by the av-
erage value of the vaiiable between the two points. Thus, the peicentage change in quantity between
points A and B in Figuie 3.1 is computed ielative to the average of the quantity values at points A and
B: (60,000 + 40,000)/2 = 30,000. The peicentage change in quantity, then, is 20,000/30,000, oi 40.
Likewise, the peicentage change in piice between points A and B is based on the average of the two
piices: ($0.80+$0.70)/2=$0.73, and so we have a peicentage change of 0.10/0.73, oi 13.33. The
piice elasticity of demand between points A and B is thus 40/(13.33)=3.00.
This measuie of elasticity, which is based on peicentage changes ielative to the aveiage value of
each vaiiable between two points, is called arc elasticity. The aic elasticity method has the advantage
that it yields the same elasticity whethei we go fiom point A to point B oi fiom point B to point A. It is
the method we shall use to compute elasticity.
Foi the aic elasticity method, we calculate the piice elasticity of demand using the aveiage value of
piice,

P, and the aveiage value of quantity demanded,

Q. We shall use the Gieek lettei A to mean


change in," so the change in quantity between two points is AQ and the change in piice is AP. Now we
can wiite the foimula foi the piice elasticity of demand as
QUA7I ON 5. 3
e
D
=
AQ
/

Q
AP
/

P
The piice elasticity of demand between points A and B is thus:
e
D
=
20,000
(40,000 + 60,000)/2
-$0.10
($0.80 + $0.70)/2
=
40
-13.33
= -3.00
With the aic elasticity foimula, the elasticity is the same whethei we move fiom point A to point B oi
fiom point B to point A. If we stait at point B and move to point A, we have:
e
D
=
-20,000
(60,000 + 40,000)/2
0.10
($0.70 + $0.80)/2
=
-40
13.33
= -3.00
The aic elasticity method gives us an estimate of elasticity. It gives the value of elasticity at the mid-
point ovei a iange of change, such as the movement between points A and B. Foi a piecise
CHAP7R 5 LAS7ICI7Y: A MASUR OI RSPONS 113
computation of elasticity, we would need to considei the iesponse of a dependent vaiiable to an ex-
tiemely small change in an independent vaiiable. The fact that aic elasticities aie appioximate suggests
an impoitant piactical iule in calculating aic elasticities: we should considei only small changes in in-
dependent vaiiables. We cannot apply the concept of aic elasticity to laige changes.
Anothei aigument foi consideiing only small changes in computing piice elasticities of demand
will become evident in the next section. We will investigate what happens to piice elasticities as we
move fiom one point to anothei along a lineai demand cuive.
Heads Up!
|ct|ce t|.t |r t|e ..c e|.st|c|ty |c.nu|., t|e net|cJ |c. ccnput|rg . pe.cert.ge c|.rge J||e.s |.cn t|e
st.rJ..J net|cJ .|t| .||c| ycu n.y be |.n|||... |.t net|cJ ne.su.es t|e pe.cert.ge c|.rge |r . v..|.b|e
.e|.t|ve tc |ts c.|g|r.| v.|ue. |c. ex.np|e, us|rg t|e st.rJ..J net|cJ, .|er .e gc |.cn pc|rt A tc pc|rt b, .e
.cu|J ccnpute t|e pe.cert.ge c|.rge |r ,u.rt|ty .s 20,000/40,00050. |e pe.cert.ge c|.rge |r p.|ce
.cu|J be 0.10/0.8012.5. |e p.|ce e|.st|c|ty c| Jen.rJ .cu|J t|er be 50/(12.5)4.00. Cc|rg
|.cn pc|rt b tc pc|rt A, |c.eve., .cu|J y|e|J . J||e.ert e|.st|c|ty. |e pe.cert.ge c|.rge |r ,u.rt|ty .cu|J
be 20,000/60,000, c. 33.33. |e pe.cert.ge c|.rge |r p.|ce .cu|J be 0.10/0.014.29. |e p.|ce
e|.st|c|ty c| Jen.rJ .cu|J t|us be 33.33/14.292.33. by us|rg t|e .ve..ge ,u.rt|ty .rJ .ve..ge p.|ce
tc c.|cu|.te pe.cert.ge c|.rges, t|e ..c e|.st|c|ty .pp.c.c| .vc|Js t|e recess|ty tc spec||y t|e J|.ect|cr c|
t|e c|.rge .rJ, t|e.eby, g|ves us t|e s.ne .rs.e. .|et|e. .e gc |.cn A tc b c. |.cn b tc A.
1.2 Piice Elasticities Along a Lineai Demand Cuive
What happens to the piice elasticity of demand when we tiavel along the demand cuive: The answei
depends on the natuie of the demand cuive itself. On a lineai demand cuive, such as the one in Figuie
3.2, elasticity becomes smallei (in absolute value) as we tiavel downwaid and to the iight.
II GUR 5. 2 Price Iasticities of Demand for a Linear Demand Curve
|e p.|ce e|.st|c|ty c| Jen.rJ v..|es bet.eer J||e.ert p.|.s c| pc|rts .|crg . ||re.. Jen.rJ cu.ve. |e |c.e. t|e p.|ce .rJ t|e g.e.te. t|e ,u.rt|ty
Jen.rJeJ, t|e |c.e. t|e .bsc|ute v.|ue c| t|e p.|ce e|.st|c|ty c| Jen.rJ.
Figuie 3.2 shows the same demand cuive we saw in Figuie 3.1. We have alieady calculated the piice
elasticity of demand between points A and B; it equals 3.00. Notice, howevei, that when we use the
same method to compute the piice elasticity of demand between othei sets of points, oui answei vaiies.
Foi each of the paiis of points shown, the changes in piice and quantity demanded aie the same (a
$0.10 deciease in piice and 20,000 additional iides pei day, iespectively). But at the high piices and low
quantities on the uppei pait of the demand cuive, the peicentage change in quantity is ielatively laige,
wheieas the peicentage change in piice is ielatively small. The absolute value of the piice elasticity of
demand is thus ielatively laige. As we move down the demand cuive, equal changes in quantity iepies-
ent smallei and smallei peicentage changes, wheieas equal changes in piice iepiesent laigei and laigei
peicentage changes, and the absolute value of the elasticity measuie declines. Between points C and D,
114 PRINCIPLS OI CONOMICS
totaI revenue
A |.ns cutput nu|t|p||eJ by
t|e p.|ce .t .||c| |t se||s t|.t
cutput.
foi example, the piice elasticity of demand is 1.00, and between points E and F the piice elasticity of
demand is 0.33.
On a lineai demand cuive, the piice elasticity of demand vaiies depending on the inteival ovei
which we aie measuiing it. Foi any lineai demand cuive, the absolute value of the piice elasticity of de-
mand will fall as we move down and to the iight along the cuive.
1.3 The Piice Elasticity of Demand and Changes in Total Revenue
Suppose the public tiansit authoiity is consideiing iaising faies. Will its total ievenues go up oi down:
Total revenue is the piice pei unit times the numbei of units sold.
[1]
In this case, it is the faie times
the numbei of iideis. The tiansit authoiity will ceitainly want to know whethei a piice inciease will
cause its total ievenue to iise oi fall. In fact, deteimining the impact of a piice change on total ievenue
is ciucial to the analysis of many pioblems in economics.
We will do two quick calculations befoie geneializing the piinciple involved. Given the demand
cuive shown in Figuie 3.2, we see that at a piice of $0.80, the tiansit authoiity will sell 40,000 iides pei
day. Total ievenue would be $32,000 pei day ($0.80 times 40,000). If the piice weie loweied by $0.10 to
$0.70, quantity demanded would inciease to 60,000 iides and total ievenue would inciease to $42,000
($0.70 times 60,000). The ieduction in faie increases total ievenue. Howevei, if the initial piice had
been $0.30 and the tiansit authoiity ieduced it by $0.10 to $0.20, total ievenue would decrease fiom
$42,000 ($0.30 times 140,000) to $32,000 ($0.20 times 160,000). So it appeais that the impact of a piice
change on total ievenue depends on the initial piice and, by implication, the oiiginal elasticity. We
geneialize this point in the iemaindei of this section.
The pioblem in assessing the impact of a piice change on total ievenue of a good oi seivice is that
a change in piice always changes the quantity demanded in the opposite diiection. An inciease in piice
ieduces the quantity demanded, and a ieduction in piice incieases the quantity demanded. The ques-
tion is how much. Because total ievenue is found by multiplying the piice pei unit times the quantity
demanded, it is not cleai whethei a change in piice will cause total ievenue to iise oi fall.
We have alieady made this point in the context of the tiansit authoiity. Considei the following
thiee examples of piice incieases foi gasoline, pizza, and diet cola.
Suppose that 1,000 gallons of gasoline pei day aie demanded at a piice of $4.00 pei gallon. Total
ievenue foi gasoline thus equals $4,000 pei day (=1,000 gallons pei day times $4.00 pei gallon). If an
inciease in the piice of gasoline to $4.23 ieduces the quantity demanded to 930 gallons pei day, total
ievenue iises to $4,037.30 pei day (=930 gallons pei day times $4.23 pei gallon). Even though people
consume less gasoline at $4.23 than at $4.00, total ievenue iises because the highei piice moie than
makes up foi the diop in consumption.
Next considei pizza. Suppose 1,000 pizzas pei week aie demanded at a piice of $9 pei pizza. Total
ievenue foi pizza equals $9,000 pei week (=1,000 pizzas pei week times $9 pei pizza). If an inciease in
the piice of pizza to $10 pei pizza ieduces quantity demanded to 900 pizzas pei week, total ievenue will
still be $9,000 pei week (=900 pizzas pei week times $10 pei pizza). Again, when piice goes up, con-
sumeis buy less, but this time theie is no change in total ievenue.
Now considei diet cola. Suppose 1,000 cans of diet cola pei day aie demanded at a piice of $0.30
pei can. Total ievenue foi diet cola equals $300 pei day (=1,000 cans pei day times $0.30 pei can). If an
inciease in the piice of diet cola to $0.33 pei can ieduces quantity demanded to 880 cans pei month,
total ievenue foi diet cola falls to $484 pei day (=880 cans pei day times $0.33 pei can). As in the case
of gasoline, people will buy less diet cola when the piice iises fiom $0.30 to $0.33, but in this example
total ievenue diops.
In oui fist example, an inciease in piice incieased total ievenue. In the second, a piice inciease left
total ievenue unchanged. In the thiid example, the piice iise ieduced total ievenue. Is theie a way to
piedict how a piice change will afect total ievenue: Theie is; the efect depends on the piice elasticity
of demand.
CHAP7R 5 LAS7ICI7Y: A MASUR OI RSPONS 115
price eIastic
S|tu.t|cr |r .||c| t|e
.bsc|ute v.|ue c| t|e p.|ce
e|.st|c|ty c| Jen.rJ |s
g.e.te. t|.r 1.
unit price eIastic
S|tu.t|cr |r .||c| t|e
.bsc|ute v.|ue c| t|e p.|ce
e|.st|c|ty c| Jen.rJ |s e,u.|
tc 1.
price ineIastic
S|tu.t|cr |r .||c| t|e
.bsc|ute v.|ue c| t|e p.|ce c|
e|.st|c|ty c| Jen.rJ |s |ess
t|.r 1.
Iastic, Unit Iastic, and IneIastic Demand
To deteimine how a piice change will afect total ievenue, economists place piice elasticities of demand
in thiee categoiies, based on theii absolute value. If the absolute value of the piice elasticity of demand
is gieatei than 1, demand is teimed price elastic. If it is equal to 1, demand is unit price elastic. And
if it is less than 1, demand is price inelastic.
ReIating Iasticity to Changes in 7otaI Revenue
When the piice of a good oi seivice changes, the quantity demanded changes in the opposite diiection.
Total ievenue will move in the diiection of the vaiiable that changes by the laigei peicentage. If the
vaiiables move by the same peicentage, total ievenue stays the same. If quantity demanded changes by
a laigei peicentage than piice (i.e., if demand is piice elastic), total ievenue will change in the diiection
of the quantity change. If piice changes by a laigei peicentage than quantity demanded (i.e., if demand
is piice inelastic), total ievenue will move in the diiection of the piice change. If piice and quantity de-
manded change by the same peicentage (i.e., if demand is unit piice elastic), then total ievenue does
not change.
When demand is piice inelastic, a given peicentage change in piice iesults in a smallei peicentage
change in quantity demanded. That implies that total ievenue will move in the diiection of the piice
change: a ieduction in piice will ieduce total ievenue, and an inciease in piice will inciease it.
Considei the piice elasticity of demand foi gasoline. In the example above, 1,000 gallons of gasol-
ine weie puichased each day at a piice of $4.00 pei gallon; an inciease in piice to $4.23 pei gallon ie-
duced the quantity demanded to 930 gallons pei day. We thus had an aveiage quantity of 973 gallons
pei day and an aveiage piice of $4.123. We can thus calculate the aic piice elasticity of demand foi
gasoline:
Peicentage change in quantity demanded = 30 / 973 = 3.1
Peicentage change in piice = 0.23 / 4.123=6.06
Piice elasticity of demand = 3.1/ 6.06 = 0.84
The demand foi gasoline is piice inelastic, and total ievenue moves in the diiection of the piice change.
When piice iises, total ievenue iises. Recall that in oui example above, total spending on gasoline
(which equals total ievenues to selleis) iose fiom $4,000 pei day (=1,000 gallons pei day times $4.00) to
$4037.30 pei day (=930 gallons pei day times $4.23 pei gallon).
When demand is piice inelastic, a given peicentage change in piice iesults in a smallei peicentage
change in quantity demanded. That implies that total ievenue will move in the diiection of the piice
change: an inciease in piice will inciease total ievenue, and a ieduction in piice will ieduce it.
Considei again the example of pizza that we examined above. At a piice of $9 pei pizza, 1,000 piz-
zas pei week weie demanded. Total ievenue was $9,000 pei week (=1,000 pizzas pei week times $9 pei
pizza). When the piice iose to $10, the quantity demanded fell to 900 pizzas pei week. Total ievenue
iemained $9,000 pei week (=900 pizzas pei week times $10 pei pizza). Again, we have an aveiage
quantity of 930 pizzas pei week and an aveiage piice of $9.30. Using the aic elasticity method, we can
compute:
Peicentage change in quantity demanded = 100 / 930 = 10.3
Peicentage change in piice = $1.00 / $9.30 = 10.3
Piice elasticity of demand = 10.3/ 10.3 = 1.0
Demand is unit piice elastic, and total ievenue iemains unchanged. Quantity demanded falls by the
same peicentage by which piice incieases.
Considei next the example of diet cola demand. At a piice of $0.30 pei can, 1,000 cans of diet cola
weie puichased each day. Total ievenue was thus $300 pei day (=$0.30 pei can times 1,000 cans pei
day). An inciease in piice to $0.33 ieduced the quantity demanded to 880 cans pei day. We thus have
an aveiage quantity of 940 cans pei day and an aveiage piice of $0.323 pei can. Computing the piice
elasticity of demand foi diet cola in this example, we have:
Peicentage change in quantity demanded = 120 / 940 = 12.8
Peicentage change in piice = $0.03 / $0.323 = 9.3
Piice elasticity of demand = 12.8/ 9.3 = 1.3
The demand foi diet cola is piice elastic, so total ievenue moves in the diiection of the quantity change.
It falls fiom $300 pei day befoie the piice inciease to $484 pei day aftei the piice inciease.
A demand cuive can also be used to show changes in total ievenue. Figuie 3.3 shows the demand
cuive fiom Figuie 3.1 and Figuie 3.2. At point A, total ievenue fiom public tiansit iides is given by the
aiea of a iectangle diawn with point A in the uppei iight-hand coinei and the oiigin in the lowei left-
116 PRINCIPLS OI CONOMICS
hand coinei. The height of the iectangle is piice; its width is quantity. We have alieady seen that total
ievenue at point A is $32,000 ($0.80 40,000). When we ieduce the piice and move to point B, the
iectangle showing total ievenue becomes shoitei and widei. Notice that the aiea gained in moving to
the iectangle at B is gieatei than the aiea lost; total ievenue iises to $42,000 ($0.70 60,000). Recall
fiom Figuie 3.2 that demand is elastic between points A and B. In geneial, demand is elastic in the up-
pei half of any lineai demand cuive, so total ievenue moves in the diiection of the quantity change.
II GUR 5. 3 Changes in 7otaI Revenue and a Linear Demand Curve
Vcv|rg |.cn pc|rt A tc pc|rt b |np||es . .eJuct|cr |r p.|ce .rJ .r |rc.e.se |r t|e ,u.rt|ty Jen.rJeJ. |en.rJ |s
e|.st|c bet.eer t|ese t.c pc|rts. ct.| .everue, s|c.r by t|e ..e.s c| t|e .ect.rg|es J...r |.cn pc|rts A .rJ b
tc t|e c.|g|r, .|ses. \|er .e ncve |.cn pc|rt | tc pc|rt |, .||c| |s |r t|e |re|.st|c .eg|cr c| t|e Jen.rJ cu.ve,
tct.| .everue |.||s.
A movement fiom point E to point F also shows a ieduction in piice and an inciease in quantity de-
manded. This time, howevei, we aie in an inelastic iegion of the demand cuive. Total ievenue now
moves in the diiection of the piice change-it falls. Notice that the iectangle diawn fiom point F is
smallei in aiea than the iectangle diawn fiom point E, once again confiming oui eailiei calculation.
CHAP7R 5 LAS7ICI7Y: A MASUR OI RSPONS 117
perfectIy ineIastic
S|tu.t|cr |r .||c| t|e p.|ce
e|.st|c|ty c| Jen.rJ |s .e.c.
II GUR 5. 4
We have noted that a lineai demand cuive is moie elastic wheie piices aie ielatively high and quantit-
ies ielatively low and less elastic wheie piices aie ielatively low and quantities ielatively high. We can
be even moie specifc. For any linear demand curve, demand will be price elastic in the upper half of the
curve and price inelastic in its lower half. At the midpoint of a linear demand curve, demand is unit price
elastic.
1.4 Constant Piice Elasticity of Demand Cuives
Figuie 3.3 shows foui demand cuives ovei which piice elasticity of demand is the same at all points.
The demand cuive in Panel (a) is veitical. This means that piice changes have no efect on quantity de-
manded. The numeiatoi of the foimula given in Equation 3.2 foi the piice elasticity of demand
(peicentage change in quantity demanded) is zeio. The piice elasticity of demand in this case is theie-
foie zeio, and the demand cuive is said to be perfectly inelastic. This is a theoietically extieme case,
and no good that has been studied empiiically exactly fts it. A good that comes close, at least ovei a
specifc piice iange, is insulin. A diabetic will not consume moie insulin as its piice falls but, ovei some
piice iange, will consume the amount needed to contiol the disease.
118 PRINCIPLS OI CONOMICS
perfectIy eIastic
S|tu.t|cr |r .||c| t|e p.|ce
e|.st|c|ty c| Jen.rJ |s
|r|r|te.
II GUR 5. 5 Demand Curves with Constant Price Iasticities
|e Jen.rJ cu.ve |r |.re| (.) |s pe.|ect|y |re|.st|c. |e Jen.rJ cu.ve |r |.re| (b) |s pe.|ect|y e|.st|c. |.|ce e|.st|c|ty
c| Jen.rJ |s 1.00 .|| .|crg t|e Jen.rJ cu.ve |r |.re| (c), .|e.e.s |t |s 0.50 .|| .|crg t|e Jen.rJ cu.ve |r |.re|
(J).
As illustiated in Figuie 3.3, seveial othei types of demand cuives have the same elasticity at eveiy point
on them. The demand cuive in Panel (b) is hoiizontal. This means that even the smallest piice changes
have enoimous efects on quantity demanded. The denominatoi of the foimula given in Equation 3.2
foi the piice elasticity of demand (peicentage change in piice) appioaches zeio. The piice elasticity of
demand in this case is theiefoie infnite, and the demand cuive is said to be perfectly elastic.
[2]
This is
the type of demand cuive faced by pioduceis of standaidized pioducts such as wheat. If the wheat of
othei faims is selling at $4 pei bushel, a typical faim can sell as much wheat as it wants to at $4 but
nothing at a highei piice and would have no ieason to ofei its wheat at a lowei piice.
The nonlineai demand cuives in Panels (c) and (d) have piice elasticities of demand that aie neg-
ative; but, unlike the lineai demand cuive discussed above, the value of the piice elasticity is constant
all along each demand cuive. The demand cuive in Panel (c) has piice elasticity of demand equal to
1.00 thioughout its iange; in Panel (d) the piice elasticity of demand is equal to 0.30 thioughout its
iange. Empiiical estimates of demand often show cuives like those in Panels (c) and (d) that have the
same elasticity at eveiy point on the cuive.
CHAP7R 5 LAS7ICI7Y: A MASUR OI RSPONS 119
Heads Up!
|c rct ccr|use p.|ce |re|.st|c Jen.rJ .rJ pe.|ect|y |re|.st|c Jen.rJ. |e.|ect|y |re|.st|c Jen.rJ ne.rs t|.t
t|e c|.rge |r ,u.rt|ty |s .e.c |c. .ry pe.cert.ge c|.rge |r p.|ce, t|e Jen.rJ cu.ve |r t||s c.se |s ve.t|c.|.
|.|ce |re|.st|c Jen.rJ ne.rs cr|y t|.t t|e pe.cert.ge c|.rge |r ,u.rt|ty |s |ess t|.r t|e pe.cert.ge c|.rge
|r p.|ce, rct t|.t t|e c|.rge |r ,u.rt|ty |s .e.c. \|t| p.|ce |re|.st|c (.s cppcseJ tc pe.|ect|y |re|.st|c) Jen.rJ,
t|e Jen.rJ cu.ve |tse|| |s st||| Jc.r...J s|cp|rg.
1.3 Deteiminants of the Piice Elasticity of Demand
The gieatei the absolute value of the piice elasticity of demand, the gieatei the iesponsiveness of
quantity demanded to a piice change. What deteimines whethei demand is moie oi less piice elastic:
The most impoitant deteiminants of the piice elasticity of demand foi a good oi seivice aie the avail-
ability of substitutes, the impoitance of the item in household budgets, and time.
AvaiIabiIity of Substitutes
The piice elasticity of demand foi a good oi seivice will be gieatei in absolute value if many close sub-
stitutes aie available foi it. If theie aie lots of substitutes foi a paiticulai good oi seivice, then it is easy
foi consumeis to switch to those substitutes when theie is a piice inciease foi that good oi seivice. Sup-
pose, foi example, that the piice of Foid automobiles goes up. Theie aie many close substitutes foi
Foids-Cheviolets, Chiysleis, Toyotas, and so on. The availability of close substitutes tends to make
the demand foi Foids moie piice elastic.
If a good has no close substitutes, its demand is likely to be somewhat less piice elastic. Theie aie
no close substitutes foi gasoline, foi example. The piice elasticity of demand foi gasoline in the intei-
mediate teim of, say, thiee-nine months is geneially estimated to be about 0.3. Since the absolute
value of piice elasticity is less than 1, it is piice inelastic. We would expect, though, that the demand foi
a paiticulai biand of gasoline will be much moie piice elastic than the demand foi gasoline in geneial.
Importance in HousehoId 8udgets
One ieason piice changes afect quantity demanded is that they change how much a consumei can buy;
a change in the piice of a good oi seivice afects the puichasing powei of a consumei's income and
thus afects the amount of a good the consumei will buy. This efect is stiongei when a good oi seivice
is impoitant in a typical household's budget.
A change in the piice of jeans, foi example, is piobably moie impoitant in youi budget than a
change in the piice of pencils. Suppose the piices of both weie to double. You had planned to buy foui
paiis of jeans this yeai, but now you might decide to make do with two new paiis. A change in pencil
piices, in contiast, might lead to veiy little ieduction in quantity demanded simply because pencils aie
not likely to loom laige in household budgets. The gieatei the impoitance of an item in household
budgets, the gieatei the absolute value of the piice elasticity of demand is likely to be.
7ime
Suppose the piice of electiicity iises tomoiiow moining. What will happen to the quantity demanded:
The answei depends in laige pait on how much time we allow foi a iesponse. If we aie inteiested
in the ieduction in quantity demanded by tomoiiow afteinoon, we can expect that the iesponse will be
veiy small. But if we give consumeis a yeai to iespond to the piice change, we can expect the iesponse
to be much gieatei. We expect that the absolute value of the piice elasticity of demand will be gieatei
when moie time is allowed foi consumei iesponses.
Considei the piice elasticity of ciude oil demand. Economist John C. B. Coopei estimated shoit-
and long-iun piice elasticities of demand foi ciude oil foi 23 industiialized nations foi the peiiod
1971-2000. Piofessoi Coopei found that foi viitually eveiy countiy, the piice elasticities weie negative,
and the long-iun piice elasticities weie geneially much gieatei (in absolute value) than weie the shoit-
iun piice elasticities. His iesults aie iepoited in Table 3.1. As you can see, the ieseaich was iepoited in
a jouinal published by OPEC (Oiganization of Petioleum Expoiting Countiies), an oiganization whose
membeis have piofted gieatly fiom the inelasticity of demand foi theii pioduct. By iestiicting supply,
OPEC, which pioduces about 43 of the woild's ciude oil, is able to put upwaid piessuie on the piice
of ciude. That incieases OPEC's (and all othei oil pioduceis') total ievenues and ieduces total costs.
120 PRINCIPLS OI CONOMICS
7A8L 5. 1 Short- and Long-Run Price Iasticities of the Demand for Crude OiI in 23 Countries
|c. ncst ccurt.|es, p.|ce e|.st|c|ty c| Jen.rJ |c. c.uJe c|| terJs tc be g.e.te. (|r .bsc|ute v.|ue) |r t|e |crg .ur
t|.r |r t|e s|c.t .ur.
Country Short-Run Price Iasticity of Demand Long-Run Price Iasticity of Demand
Aust..||. 0.034 0.068
Aust.|. 0.059 0.092
.r.J. 0.041 0.352
||r. 0.001 0.005
|ern..k 0.026 0.191
||r|.rJ 0.016 0.033
|..rce 0.069 0.568
Ce.n.ry 0.024 0.29
C.eece 0.055 0.126
|ce|.rJ 0.109 0.452
|.e|.rJ 0.082 0.196
|t.|y 0.035 0.208
'.p.r 0.01 0.35
|c.e. 0.094 0.18
|et|e.|.rJs 0.05 0.244
|e. e.|.rJ 0.054 0.326
|c...y 0.026 0.036
|c.tug.| 0.023 0.038
Sp.|r 0.08 0.146
S.eJer 0.043 0.289
S.|t.e.|.rJ 0.030 0.056
|r|teJ ||rgJcn 0.068 0.182
|r|teJ St.tes 0.061 0.453
Source john C. B. Cooper, Price Elasticity of Demand for Crude Oil Estimates from 23 Countries, OPEC Review Energy Economics o Related
Issues, 271 (March 2003) 4. The estimates are based on data for the period 1971-2000, except for China and South Korea, where the period is
1979-2000. Vhile the price elasticities for China and Portugal were positive, they were not statistically signifcant.
k Y 7 A k A W A Y S
< |e p.|ce e|.st|c|ty c| Jen.rJ ne.su.es t|e .espcrs|veress c| ,u.rt|ty Jen.rJeJ tc c|.rges |r p.|ce, |t
|s c.|cu|.teJ by J|v|J|rg t|e pe.cert.ge c|.rge |r ,u.rt|ty Jen.rJeJ by t|e pe.cert.ge c|.rge |r p.|ce.
< |en.rJ |s p.|ce |re|.st|c || t|e .bsc|ute v.|ue c| t|e p.|ce e|.st|c|ty c| Jen.rJ |s |ess t|.r 1, |t |s ur|t p.|ce
e|.st|c || t|e .bsc|ute v.|ue |s e,u.| tc 1, .rJ |t |s p.|ce e|.st|c || t|e .bsc|ute v.|ue |s g.e.te. t|.r 1.
< |en.rJ |s p.|ce e|.st|c |r t|e uppe. |.|| c| .ry ||re.. Jen.rJ cu.ve .rJ p.|ce |re|.st|c |r t|e |c.e. |.||. |t
|s ur|t p.|ce e|.st|c .t t|e n|Jpc|rt.
< \|er Jen.rJ |s p.|ce |re|.st|c, tct.| .everue ncves |r t|e J|.ect|cr c| . p.|ce c|.rge. \|er Jen.rJ |s
ur|t p.|ce e|.st|c, tct.| .everue Jces rct c|.rge |r .espcrse tc . p.|ce c|.rge. \|er Jen.rJ |s p.|ce
e|.st|c, tct.| .everue ncves |r t|e J|.ect|cr c| . ,u.rt|ty c|.rge.
< |e .bsc|ute v.|ue c| t|e p.|ce e|.st|c|ty c| Jen.rJ |s g.e.te. .|er subst|tutes ..e .v.||.b|e, .|er t|e
gccJ |s |npc.t.rt |r |cuse|c|J buJgets, .rJ .|er buye.s |.ve nc.e t|ne tc .Just tc c|.rges |r t|e
p.|ce c| t|e gccJ.
CHAP7R 5 LAS7ICI7Y: A MASUR OI RSPONS 121
7 R Y I 7 !
+cu ..e rc. .e.Jy tc p|.y t|e p..t c| t|e n.r.ge. c| t|e pub||c t..rs|t systen. +cu. |r.rce c|ce. |.s ust
.Jv|seJ ycu t|.t t|e systen |.ces . Je|c|t. +cu. bc..J Jces rct ..rt ycu tc cut se.v|ce, .||c| ne.rs t|.t
ycu c.rrct cut ccsts. +cu. cr|y |cpe |s tc |rc.e.se .everue. \cu|J . |..e |rc.e.se bccst .everue`
+cu ccrsu|t t|e eccrcn|st cr ycu. st.| .|c |.s .ese..c|eJ stuJ|es cr pub||c t..rspc.t.t|cr e|.st|c|t|es. S|e
.epc.ts t|.t t|e est|n.teJ p.|ce e|.st|c|ty c| Jen.rJ |c. t|e |.st |e. ncrt|s .|te. . p.|ce c|.rge |s .bcut
0.3, but t|.t .|te. seve..| ye..s, |t .||| be .bcut 1.5.
1. |xp|.|r .|y t|e est|n.teJ v.|ues |c. p.|ce e|.st|c|ty c| Jen.rJ J||e..
2. cnpute .|.t .||| |.pper tc .|Je.s||p .rJ .everue cve. t|e rext |e. ncrt|s || ycu Jec|Je tc ..|se |..es
by 5.
3. cnpute .|.t .||| |.pper tc .|Je.s||p .rJ .everue cve. t|e rext |e. ye..s || ycu Jec|Je tc ..|se |..es by
5.
4. \|.t |.ppers tc tct.| .everue rc. .rJ .|te. seve..| ye..s || ycu c|ccse tc ..|se |..es`
Case in Point: Iasticity and Stop Lights
2010 jupiterimages Corporation
\e .|| |.ce t|e s|tu.t|cr eve.y J.y. +cu ..e .pp.c.c||rg .r |rte.sect|cr. |e ye||c. ||g|t ccnes cr. +cu krc.
t|.t ycu ..e suppcseJ tc s|c. Jc.r, but ycu ..e |r . b|t c| . |u..y. Sc, ycu speeJ up . ||tt|e tc t.y tc n.ke t|e
||g|t. but t|e .eJ ||g|t |.s|es cr ust be|c.e ycu get tc t|e |rte.sect|cr. S|cu|J ycu .|sk |t .rJ gc t|.cug|`
V.ry pecp|e |.ceJ .|t| t|.t s|tu.t|cr t.ke t|e .|sky c|c|ce. |r 1998, 2,000 pecp|e |r t|e |r|teJ St.tes J|eJ .s
. .esu|t c| J.|ve.s .urr|rg .eJ ||g|ts .t |rte.sect|crs. |r .r e|c.t tc .eJuce t|e runbe. c| J.|ve.s .|c n.ke
suc| c|c|ces, n.ry ..e.s |.ve |rst.||eJ c.ne..s .t |rte.sect|crs. |.|ve.s .|c .ur .eJ ||g|ts |.ve t|e|. p|ctu.es
t.ker .rJ .ece|ve c|t.t|crs |r t|e n.||. ||s er|c.cenert net|cJ, tcget|e. .|t| .ecert |rc.e.ses |r t|e |res
|c. J.|v|rg t|.cug| .eJ ||g|ts .t |rte.sect|crs, |.s |eJ tc .r |rt.|gu|rg .pp||c.t|cr c| t|e ccrcept c| e|.st|c|ty.
|ccrcn|sts Avre. b..||.r c| t|e |r|ve.s|ty c| |.||. |r |s..e| .rJ b.uce S.ce.Jcte c| |..tncut| |r|ve.s|ty |.ve
est|n.teJ .|.t |s, |r e|ect, t|e p.|ce e|.st|c|ty |c. J.|v|rg t|.cug| stcp||g|ts .|t| .espect tc t..|c |res .t |r
te.sect|crs |r |s..e| .rJ |r S.r |..rc|scc.
|r |ecenbe. 1996, |s..e| s|..p|y |rc.e.seJ t|e |re |c. J.|v|rg t|.cug| . .eJ ||g|t. |e c|J |re c| 400 s|eke|s
(t||s ..s e,u.| .t t|.t t|ne tc 122 |r t|e |r|teJ St.tes) ..s |rc.e.seJ tc 1,000 s|eke|s (305). |r '.ru..y
1998, .|||c.r|. ..|seJ |ts |re |c. t|e c|erse |.cn 104 tc 21. |e ccurt.y c| |s..e| .rJ t|e c|ty c| S.r |..r
c|scc |rst.||eJ c.ne..s .t seve..| |rte.sect|crs. |.|ve.s .|c |grc.eJ stcp||g|ts gct t|e|. p|ctu.es t.ker .rJ
.utcn.t|c.||y .ece|veJ c|t.t|crs |npcs|rg t|e re. ||g|e. |res.
122 PRINCIPLS OI CONOMICS
\e c.r t||rk c| J.|v|rg t|.cug| .eJ ||g|ts .s .r .ct|v|ty |c. .||c| t|e.e |s . Jen.rJ.|te. .||, |grc.|rg . .eJ
||g|t speeJs up cres t.|p. |t n.y .|sc gere..te s.t|s|.ct|cr tc pecp|e .|c ercy J|scbey|rg t..|c |..s. |e
ccrcept c| e|.st|c|ty g|ves us . ..y tc s|c. ust |c. .espcrs|ve J.|ve.s .e.e tc t|e |rc.e.se |r |res.
|.c|essc.s b..||.r .rJ S.ce.Jcte cbt.|reJ |r|c.n.t|cr cr .|| t|e J.|ve.s c|teJ .t 3 |rte.sect|crs |r |s..e| .rJ
e|g|t |rte.sect|crs |r S.r |..rc|scc. |c. |s..e|, |c. ex.np|e, t|ey Je|reJ t|e pe.|cJ '.ru..y 1992 tc 'ure 1996
.s t|e be|c.e pe.|cJ. |ey ccnp..eJ t|e runbe. c| v|c|.t|crs Ju.|rg t|e be|c.e pe.|cJ tc t|e runbe. c| v|
c|.t|crs |.cn 'u|y 1996 tc |ecenbe. 1999t|e .|te. pe.|cJ.rJ |curJ t|e.e ..s . .eJuct|cr |r t|ckets pe.
J.|ve. c| 31.5 pe. cert. Spec||c.||y, t|e .ve..ge runbe. c| t|ckets pe. J.|ve. ..s 0.03 Ju.|rg t|e pe.|cJ be
|c.e t|e |rc.e.se, |t |e|| tc 0.050 .|te. t|e |rc.e.se. |e |rc.e.se |r t|e |re ..s 150 pe. cert. (|cte t|.t, be
c.use t|ey .e.e n.k|rg . be|c.e .rJ .|te. c.|cu|.t|cr, t|e .ut|c.s useJ t|e st.rJ..J net|cJ Jesc.|beJ |r
t|e |e.Js |p! cr ccnput|rg . pe.cert.ge c|.rge|.e., t|ey ccnputeJ t|e pe.cert.ge c|.rges |r ccnp..
|scr tc t|e c.|g|r.| v.|ues |rste.J c| t|e .ve..ge v.|ue c| t|e v..|.b|es.) |e e|.st|c|ty c| c|t.t|crs .|t| .espect
tc t|e |re ..s t|us 0.21 ( 31.5/150).
|e eccrcn|sts est|n.teJ e|.st|c|t|es |c. p..t|cu|.. g.cups c| pecp|e. |c. ex.np|e, ycurg pecp|e (.ge 130)
|.J .r e|.st|c|ty c| 0.36, pecp|e cve. t|e .ge c| 30 |.J .r e|.st|c|ty c| 0.16. |r gere..|, e|.st|c|t|es |e|| |r .b
sc|ute v.|ue .s |rccne .cse. |c. S.r |..rc|scc .rJ |s..e| ccnb|reJ, t|e e|.st|c|ty ..s bet.eer 0.26 .rJ
0.33.
|r gere..|, t|e .esu|ts s|c.eJ t|.t pecp|e .espcrJeJ ..t|cr.||y tc t|e |rc.e.ses |r |res. |rc.e.s|rg t|e p.|ce
c| . p..t|cu|.. be|.v|c. .eJuceJ t|e |.e,uercy c| t|.t be|.v|c.. |e stuJy .|sc pc|rts cut t|e e|ect|veress c|
c.ne..s .s .r er|c.cenert tec|r|,ue. \|t| c.ne..s, v|c|.tc.s c.r be ce.t.|r t|ey .||| be c|teJ || t|ey |grc.e .
.eJ ||g|t. ArJ .eJuc|rg t|e runbe. c| pecp|e .urr|rg .eJ ||g|ts c|e..|y s.ves ||ves.
.ooce ^.e |o||o oJ |oce .oceJo|e |e |eoe o| ||o| oJ |o||o| |o ||e' .ooo| o| |o. oJ |coo|c. 4. (^|| 2004:
.
A N S W R S 7 O 7 R Y I 7 ! P R O 8 L M S
1. |e .bsc|ute v.|ue c| p.|ce e|.st|c|ty c| Jen.rJ terJs tc be g.e.te. .|er nc.e t|ne |s .||c.eJ |c.
ccrsune.s tc .espcrJ. Ove. t|ne, .|Je.s c| t|e ccnnute. ..|| systen c.r c.g.r|.e c.. pcc|s, ncve, c.
ct|e..|se .Just tc t|e |..e |rc.e.se.
2. |s|rg t|e |c.nu|. |c. p.|ce e|.st|c|ty c| Jen.rJ .rJ p|ugg|rg |r v.|ues |c. t|e est|n.te c| p.|ce e|.st|c|ty
(0.5) .rJ t|e pe.cert.ge c|.rge |r p.|ce (5) .rJ t|er .e....rg|rg te.ns, .e c.r sc|ve |c. t|e
pe.cert.ge c|.rge |r ,u.rt|ty Jen.rJeJ .s. e
|
/ |r (// |r |, 0.5 / |r (/5, (0.5)(5) /
|r ( 2.5. ||Je.s||p |.||s by 2.5 |r t|e |.st |e. ncrt|s.
3. |s|rg t|e |c.nu|. |c. p.|ce e|.st|c|ty c| Jen.rJ .rJ p|ugg|rg |r v.|ues |c. t|e est|n.te c| p.|ce e|.st|c|ty
cve. . |e. ye..s (1.5) .rJ t|e pe.cert.ge c|.rge |r p.|ce (5), .e c.r sc|ve |c. t|e pe.cert.ge c|.rge
|r ,u.rt|ty Jen.rJeJ .s e
|
/ |r (// |r | , 1.5 / |r (/5, (1.5)(5) / |r ( .5.
||Je.s||p |.||s by .5 cve. . |e. ye..s.
4. ct.| .everue .|ses |nneJ|.te|y .|te. t|e |..e |rc.e.se, s|rce Jen.rJ cve. t|e |nneJ|.te pe.|cJ |s p.|ce
|re|.st|c. ct.| .everue |.||s .|te. . |e. ye..s, s|rce Jen.rJ c|.rges .rJ beccnes p.|ce e|.st|c.
2. RESPONSIVENESS OF DEMAND TO OTHER FACTORS
L A R N I N G O 8 1 C 7 I V S
1. xpIain the concept of income eIasticity of demand and its caIcuIation.
2. CIassify goods as normaI or inferior depending on their income eIasticity of demand.
3. xpIain the concept of cross price eIasticity of demand and its caIcuIation.
4. CIassify goods as substitutes or compIements depending on their cross price eIasticity of
demand.
Although the iesponse of quantity demanded to changes in piice is the most widely used measuie of
elasticity, economists aie inteiested in the iesponse to changes in the demand shifteis as well. Two of
the most impoitant measuies show how demand iesponds to changes in income and to changes in the
piices of ielated goods and seivices.
CHAP7R 5 LAS7ICI7Y: A MASUR OI RSPONS 123
income eIasticity of
demand
|e pe.cert.ge c|.rge |r
,u.rt|ty Jen.rJeJ .t .
spec||c p.|ce J|v|JeJ by t|e
pe.cert.ge c|.rge |r
|rccne t|.t p.cJuceJ t|e
Jen.rJ c|.rge, .|| ct|e.
t||rgs urc|.rgeJ.
2.1 Income Elasticity of Demand
We saw in the chaptei that intioduced the model of demand and supply that the demand foi a good oi
seivice is afected by income. We measuie the income elasticity of demand, e
Y
, as the peicentage
change in quantity demanded at a specifc price divided by the peicentage change in income that pio-
duced the demand change, all othei things unchanged:
QUA7I ON 5. 4
e
Y
=
change in quantity demanded
change in income
The symbol Y is often used in economics to iepiesent income. Because income elasticity of de-
mand iepoits the iesponsiveness of quantity demanded to a change in income, all othei things un-
changed (including the piice of the good), it iefects a shift in the demand cuive at a given piice. Re-
membei that piice elasticity of demand iefects movements along a demand cuive in iesponse to a
change in piice.
A positive income elasticity of demand means that income and demand move in the same diiec-
tion-an inciease in income incieases demand, and a ieduction in income ieduces demand. As we
leained, a good whose demand iises as income iises is called a noimal good.
Studies show that most goods and seivices aie noimal, and thus theii income elasticities aie posit-
ive. Goods and seivices foi which demand is likely to move in the same diiection as income include
housing, seafood, iock conceits, and medical seivices.
If a good oi seivice is infeiioi, then an inciease in income ieduces demand foi the good. That im-
plies a negative income elasticity of demand. Goods and seivices foi which the income elasticity of de-
mand is likely to be negative include used clothing, beans, and uiban public tiansit. Foi example, the
studies we have alieady cited conceining the demands foi uiban public tiansit in Fiance and in Madiid
found the long-iun income elasticities of demand to be negative (0.23 in Fiance and 0.23 in Mad-
iid).
[3]
II GUR 5. 7
When we compute the income elasticity of demand, we aie looking at the change in the quantity de-
manded at a specifc piice. We aie thus dealing with a change that shifts the demand cuive. An inciease
in income shifts the demand foi a noimal good to the iight; it shifts the demand foi an infeiioi good to
the left.
124 PRINCIPLS OI CONOMICS
cross price eIasticity of
demand
|t e,u.|s t|e pe.cert.ge
c|.rge |r t|e ,u.rt|ty
Jen.rJeJ c| cre gccJ c.
se.v|ce .t . spec||c p.|ce
J|v|JeJ by t|e pe.cert.ge
c|.rge |r t|e p.|ce c| .
.e|.teJ gccJ c. se.v|ce.
2.2 Cioss Piice Elasticity of Demand
The demand foi a good oi seivice is afected by the piices of ielated goods oi seivices. A ieduction in
the piice of salsa, foi example, would inciease the demand foi chips, suggesting that salsa is a comple-
ment of chips. A ieduction in the piice of chips, howevei, would ieduce the demand foi peanuts, sug-
gesting that chips aie a substitute foi peanuts.
The measuie economists use to desciibe the iesponsiveness of demand foi a good oi seivice to a
change in the piice of anothei good oi seivice is called the cross price elasticity of demand, e
A, B
. It
equals the peicentage change in the quantity demanded of one good oi seivice at a specifc price di-
vided by the peicentage change in the piice of a ielated good oi seivice. We aie vaiying the piice of a
ielated good when we considei the cioss piice elasticity of demand, so the iesponse of quantity deman-
ded is shown as a shift in the demand cuive.
The cioss piice elasticity of the demand foi good A with iespect to the piice of good B is given by:
QUA7I ON 5. 5
e
A, B
=
change in quantity demanded of good A
change in piice of good B
Cioss piice elasticities of demand defne whethei two goods aie substitutes, complements, oi unie-
lated. If two goods aie substitutes, an inciease in the piice of one will lead to an inciease in the demand
foi the othei-the cioss piice elasticity of demand is positive. If two goods aie complements, an in-
ciease in the piice of one will lead to a ieduction in the demand foi the othei-the cioss piice elasticity
of demand is negative. If two goods aie unielated, a change in the piice of one will not afect the de-
mand foi the othei-the cioss piice elasticity of demand is zeio.
II GUR 5. 8
An examination of the demand foi local television adveitising with iespect to the piice of local iadio
adveitising ievealed that the two goods aie cleaily substitutes. A 10 pei cent inciease in the piice of loc-
al iadio adveitising led to a 10 pei cent inciease in demand foi local television adveitising, so that the
cioss piice elasticity of demand foi local television adveitising with iespect to changes in the piice of
iadio adveitising was 1.0.
[4]
CHAP7R 5 LAS7ICI7Y: A MASUR OI RSPONS 125
Heads Up!
|ct|ce t|.t .|t| |rccne e|.st|c|ty c| Jen.rJ .rJ c.css p.|ce e|.st|c|ty c| Jen.rJ .e ..e p.|n..||y ccrce.reJ
.|t| .|et|e. t|e ne.su.eJ v.|ue c| t|ese e|.st|c|t|es |s pcs|t|ve c. reg.t|ve. |r t|e c.se c| |rccne e|.st|c|ty c|
Jen.rJ t||s te||s us .|et|e. t|e gccJ c. se.v|ce |s rc.n.| c. |r|e.|c.. |r t|e c.se c| c.css p.|ce e|.st|c|ty c| Je
n.rJ |t te||s us .|et|e. t.c gccJs ..e subst|tutes c. ccnp|enerts. \|t| p.|ce e|.st|c|ty c| Jen.rJ .e .e.e
ccrce.reJ .|t| .|et|e. t|e ne.su.eJ .bsc|ute v.|ue c| t||s e|.st|c|ty ..s g.e.te. t|.r, |ess t|.r, c. e,u.| tc
1, bec.use t||s g.ve us |r|c.n.t|cr .bcut .|.t |.ppers tc tct.| .everue .s p.|ce c|.rges. |e te.ns e|.st|c
.rJ |re|.st|c .pp|y tc p.|ce e|.st|c|ty c| Jen.rJ. |ey ..e rct useJ tc Jesc.|be |rccne e|.st|c|ty c| Jen.rJ c.
c.css p.|ce e|.st|c|ty c| Jen.rJ.
k Y 7 A k A W A Y S
< |e |rccne e|.st|c|ty c| Jen.rJ .e|ects t|e .espcrs|veress c| Jen.rJ tc c|.rges |r |rccne. |t |s t|e
pe.cert.ge c|.rge |r ,u.rt|ty Jen.rJeJ o| o ec||c |ce J|v|JeJ by t|e pe.cert.ge c|.rge |r |rccne,
cete.|s p..|bus.
< |rccne e|.st|c|ty |s pcs|t|ve |c. rc.n.| gccJs .rJ reg.t|ve |c. |r|e.|c. gccJs.
< |e c.css p.|ce e|.st|c|ty c| Jen.rJ ne.su.es t|e ..y Jen.rJ |c. cre gccJ c. se.v|ce .espcrJs tc
c|.rges |r t|e p.|ce c| .rct|e.. |t |s t|e pe.cert.ge c|.rge |r t|e ,u.rt|ty Jen.rJeJ c| cre gccJ c.
se.v|ce o| o ec||c |ce J|v|JeJ by t|e pe.cert.ge c|.rge |r t|e p.|ce c| .rct|e. gccJ c. se.v|ce, .||
ct|e. t||rgs urc|.rgeJ.
< .css p.|ce e|.st|c|ty |s pcs|t|ve |c. subst|tutes, reg.t|ve |c. ccnp|enerts, .rJ .e.c |c. gccJs c. se.v|ces
.|cse Jen.rJs ..e ur.e|.teJ.
7 R Y I 7 !
Suppcse t|.t .|er t|e p.|ce c| b.ge|s .|ses by 10, t|e Jen.rJ |c. c.e.n c|eese |.||s by 3 .t t|e cu..ert
p.|ce, .rJ t|.t .|er |rccne .|ses by 10, t|e Jen.rJ |c. b.ge|s |rc.e.ses by 1 .t t|e cu..ert p.|ce. .|cu
|.te t|e c.css p.|ce e|.st|c|ty c| Jen.rJ |c. c.e.n c|eese .|t| .espect tc t|e p.|ce c| b.ge|s .rJ te|| .|et|e.
b.ge|s .rJ c.e.n c|eese ..e subst|tutes c. ccnp|enerts. .|cu|.te t|e |rccne e|.st|c|ty c| Jen.rJ .rJ te||
.|et|e. b.ge|s ..e rc.n.| c. |r|e.|c..
126 PRINCIPLS OI CONOMICS
Case in Point: 7een Smoking and Iasticity
2010 jupiterimages Corporation
cb.ccc k|||s nc.e pecp|e t|.r .ry ct|e. subst.rce c. J|se.se. \c.|J.|Je, t|e .rru.| Je.t| tc|| |.cn tc
b.ccc |s .e|| cve. 3 n||||cr pecp|e pe. ye... |r t|e |r|teJ St.tes .|cre, 400,000 pecp|e J|e e.c| ye.. .s . .es
u|t c| t|e|. use c| tcb.ccc.
Vc.e t|.r t.ct||.Js c| sncke.s |rJ|c.teJ |r 1995 t|.t t|ey .cu|J p.e|e. tc ,u|t snck|rg but .e.e ur.b|e tc
Jc sc, .ccc.J|rg tc t|e erte.s |c. ||se.se crt.c| .rJ |.evert|cr. |r |.ct, |ess t|.r 2.5 pe. cert c| sncke.s
succeeJ |r ,u|tt|rg e.c| ye...
Vcst sncke.s beg|r us|rg tcb.ccc .s teer.ge.s. eers terJ tc urJe.est|n.te t|e J.rge. c| snck|rg .rJ tc
cve.est|n.te t|e|. ||ke|y .b|||ty tc ,u|t snck|rg .|er t|ey c|ccse tc Jc sc. Ore c.r, t|e.e|c.e, ..gue t|.t t|e
Jec|s|cr tc sncke n.y rct be . ..t|cr.| cre, .rJ |t |s cre t|.t |npcses subst.rt|.| ccsts cr t|e .est c| scc|ety.
bec.use |t ..|ses |e.|t|c..e ccsts, |t ..|ses |e.|t| |rsu..rce ..tes. ArJ t|e ev|Jerce |s ncurt|rg t|.t seccrJ
|.rJ sncke |npcses se.|cus |e.|t| ccrse,uerces cr rcrsncke.s. bec.use snck|rg |s suc| . se.|cus p.cb
|en |c. cu. scc|ety, .rJ bec.use t|e Jec|s|cr tc sncke |s typ|c.||y n.Je .|er cre |s . teer.ge., |e.|t| ecc
rcn|sts terJ tc |ccus cr ne.su.es tc p.evert ycurg pecp|e |.cn snck|rg.
Ore p|.ce tc beg|r |r ||n|t|rg teer snck|rg |s p.|ce. |e p.|ce e|.st|c|ty c| Jen.rJ |c. teer.ge sncke.s |s
g.e.te. (|r .bsc|ute v.|ue) t|.r t|.t |c. t|e pcpu|.t|cr |r gere..| bec.use t|e ccst c| tcb.ccc .ep.eserts .
g.e.te. pe.cert.ge c| teer |rccnes t|.r c| .Ju|t |rccnes. |c. .|| sncke.s, t|e p.|ce e|.st|c|ty c| Jen.rJ ..s
est|n.teJ by eccrcn|sts V.tt|e. . |...e||y, e..y |. |ec|.cek, .rJ |..rk '. |.|cupk. tc be 0.32. |e.|t|
c..e eccrcn|sts est|n.te t|.t t|e p.|ce e|.st|c|ty c| Jen.rJ |c. c|g..ettes |c. teer.ge.s |s bet.eer 0.9 .rJ
1.5.
|r 1998, t|e tcb.ccc |rJust.y .e.c|eJ . sett|enert .|t| 46 st.tes t|.t |.J ||eJ |..su|ts .g.|rst t|e |rJust.y,
c|..g|rg t|.t t|e tcb.ccc |rJust.y |.J |npcseJ |uge |e.|t|c..e ccsts. |e V.ste. Sett|enert Ag.eenert
(VSA) c.||eJ |c. . p.ynert c| 205 b||||cr cve. . pe.|cJ c| 25 ye..s (t|e ct|e. |cu. st.tes .e.c|eJ sep...te
.g.eenerts .|t| t|e |rJust.y |r 199 .rJ 1998). |e VSA |eJ tc .r |rc.e.se |r t|e p.|ce c| c|g..ettes by 48
pe. cert bet.eer 199 .rJ 1999. |e pe.cert.ge c| ||g| sc|cc| stuJerts .|c snckeJ |e|| s|gr||c.rt|y by
2000, |rJ|c.t|rg . subst.rt|.| .espcrs|veress c| teer.ge.s tc p.|ce c|.rges.
|e VSA .|sc .e,u|.eJ t|.t st.tes use scne c| t|e ncrey t|ey .ece|ve |.cn tcb.ccc |.ns tc c...y cut .rt|s
nck|rg p.cg..ns. |e r.tu.e .rJ sccpe c| t|ese p.cg..ns v..y .|Je|y. St.te exc|se t.xes, .|sc v..y|rg .|Je|y,
..rge |.cn 2.5 pe. p.ck |r \|.g|r|. (. tcb.cccp.cJuc|rg st.te) tc 1.51 |r V.ss.c|usetts. C|ver t|e g.e.te.
.espcrs|veress c| teer.ge.s tc t|e p.|ce c| c|g..ettes, exc|se t.xes s|cu|J p.cve .r e|ect|ve Jev|ce.
CHAP7R 5 LAS7ICI7Y: A MASUR OI RSPONS 127
Ore c.ve.t, |c.eve., |r ev.|u.t|rg t|e |np.ct c| . t.x ||ke cr teer snck|rg |s t|.t scne teers n|g|t s.|tc|
|.cn c|g..ettes tc sncke|ess tcb.ccc, .||c| |s .sscc|.teJ .|t| . ||g|e. .|sk c| c..| c.rce.. |t |s est|n.teJ t|.t
|c. ycurg n.|es t|e c.css p.|ce e|.st|c|ty c| sncke|ess tcb.ccc .|t| .espect tc t|e p.|ce c| c|g..ettes |s 1.2.
10 |rc.e.se |r c|g..ette p.|ces |e.Js tc . 12 |rc.e.se |r ycurg n.|es us|rg sncke|ess tcb.ccc.
.ooce /o|||e. |oe||,. e, | |ec|oce|. oJ |o| . |o|oo|o. |e |oc| o| oocco o|o| |oo|eJ||oe o ^eo|e |oe||e
.o|e 9S2000.' .ooo| o| |eo||| |coo|c 225 (.e|ee 2003: S43S59. /|c|oe| Ooo. |oe||e oe.' |o||c |eo||| |eo| 24
(.o|,^oo| 99.: 29029.. |oo |o oJ |o| . |o|oo|o. |e ||ec| o| |o||c |o||c|e oJ ||ce o `oo|| .o||.' .oo||e |coo|c
.ooo| .04 (^|| 2004: .96S5. .o| ^ ooo |o||c |o||c, oJ .o|| eo||o oo `oo ^Jo|| | ||e |||eJ .|o|e.' |eo||| |o||c,. 6S3
(.oe 2004: 32332
A N S W R 7 O 7 R Y I 7 ! P R O 8 L M
|s|rg t|e |c.nu|. |c. c.css p.|ce e|.st|c|ty c| Jen.rJ, .e |rJ t|.t e
Ab
(3)/(10) 0.3. S|rce t|e e
^|
|s
reg.t|ve, b.ge|s .rJ c.e.n c|eese ..e ccnp|enerts. |s|rg t|e |c.nu|. |c. |rccne e|.st|c|ty c| Jen.rJ, .e
|rJ t|.t e
`
(+1)/(10) +0.1. S|rce e
`
|s pcs|t|ve, b.ge|s ..e . rc.n.| gccJ.
3. PRICE ELASTICITY OF SUPPLY
L A R N I N G O 8 1 C 7 I V S
1. xpIain the concept of eIasticity of suppIy and its caIcuIation.
2. xpIain what it means for suppIy to be price ineIastic, unit price eIastic, price eIastic, perfectIy
price ineIastic, and perfectIy price eIastic.
3. xpIain why time is an important determinant of price eIasticity of suppIy.
4. AppIy the concept of price eIasticity of suppIy to the Iabor suppIy curve.
The elasticity measuies encounteied so fai in this chaptei all ielate to the demand side of the maiket. It
is also useful to know how iesponsive quantity supplied is to a change in piice.
Suppose the demand foi apaitments iises. Theie will be a shoitage of apaitments at the old level of
apaitment ients and piessuie on ients to iise. All othei things unchanged, the moie iesponsive the
quantity of apaitments supplied is to changes in monthly ients, the lowei the inciease in ient iequiied
to eliminate the shoitage and to biing the maiket back to equilibiium. Conveisely, if quantity supplied
is less iesponsive to piice changes, piice will have to iise moie to eliminate a shoitage caused by an in-
ciease in demand.
This is illustiated in Figuie 3.10. Suppose the ient foi a typical apaitment had been R
0
and the
quantity Q
0
when the demand cuive was D
1
and the supply cuive was eithei S
1
(a supply cuive in
which quantity supplied is less iesponsive to piice changes) oi S
2
(a supply cuive in which quantity
supplied is moie iesponsive to piice changes). Note that with eithei supply cuive, equilibiium piice
and quantity aie initially the same. Now suppose that demand incieases to D
2
, peihaps due to popula-
tion giowth. With supply cuive S
1
, the piice (ient in this case) will iise to R
1
and the quantity of apait-
ments will iise to Q
1
. If, howevei, the supply cuive had been S
2
, the ient would only have to iise to R
2
to biing the maiket back to equilibiium. In addition, the new equilibiium numbei of apaitments would
be highei at Q
2
. Supply cuive S
2
shows gieatei iesponsiveness of quantity supplied to piice change
than does supply cuive S
1
.
128 PRINCIPLS OI CONOMICS
II GUR 5. 10 Increase in Apartment
Rents Depends on How Responsive SuppIy
Is
|e nc.e .espcrs|ve t|e supp|y c| .p..tnerts
|s tc c|.rges |r p.|ce (.ert |r t||s c.se), t|e |ess
.erts .|se .|er t|e Jen.rJ |c. .p..tnerts
|rc.e.ses.
price eIasticity of suppIy
|e ..t|c c| t|e pe.cert.ge
c|.rge |r ,u.rt|ty supp||eJ
c| . gccJ c. se.v|ce tc t|e
pe.cert.ge c|.rge |r |ts
p.|ce, .|| ct|e. t||rgs
urc|.rgeJ.
We measuie the price elasticity of supply (e
S
) as the iatio of the peicentage
change in quantity supplied of a good oi seivice to the peicentage change in its piice,
all othei things unchanged:
QUA7I ON 5. 6
e
S
=
change in quantity supplied
change in piice
Because piice and quantity supplied usually move in the same diiection, the piice
elasticity of supply is usually positive. The laigei the piice elasticity of supply, the moie
iesponsive the fims that supply the good oi seivice aie to a piice change.
Supply is piice elastic if the piice elasticity of supply is gieatei than 1, unit piice
elastic if it is equal to 1, and piice inelastic if it is less than 1. A veitical supply cuive, as
shown in Panel (a) of Figuie 3.11, is peifectly inelastic; its piice elasticity of supply is
zeio. The supply of Beatles' songs is peifectly inelastic because the band no longei ex-
ists. A hoiizontal supply cuive, as shown in Panel (b) of Figuie 3.11, is peifectly elastic;
its piice elasticity of supply is infnite. It means that supplieis aie willing to supply any
amount at a ceitain piice.
II GUR 5. 11 SuppIy Curves and 7heir Price Iasticities
|e supp|y cu.ve |r |.re| (.) |s pe.|ect|y |re|.st|c. |r |.re| (b), t|e supp|y cu.ve |s pe.|ect|y e|.st|c.
3.1 Time: An Impoitant Deteiminant of the Elasticity of Supply
Time plays a veiy impoitant iole in the deteimination of the piice elasticity of supply. Look again at
the efect of ient incieases on the supply of apaitments. Suppose apaitment ients in a city iise. If we aie
looking at a supply cuive of apaitments ovei a peiiod of a few months, the ient inciease is likely to in-
duce apaitment owneis to ient out a ielatively small numbei of additional apaitments. With the highei
ients, apaitment owneis may be moie vigoious in ieducing theii vacancy iates, and, indeed, with moie
people looking foi apaitments to ient, this should be faiily easy to accomplish. Attics and basements
aie easy to ienovate and ient out as additional units. In a shoit peiiod of time, howevei, the supply ie-
sponse is likely to be faiily modest, implying that the piice elasticity of supply is faiily low. A supply
cuive coiiesponding to a shoit peiiod of time would look like S
1
in Figuie 3.10. It is duiing such peii-
ods that theie may be calls foi ient contiols.
If the peiiod of time undei consideiation is a few yeais iathei than a few months, the supply cuive
is likely to be much moie piice elastic. Ovei time, buildings can be conveited fiom othei uses and new
apaitment complexes can be built. A supply cuive coiiesponding to a longei peiiod of time would look
like S
2
in Figuie 3.10.
CHAP7R 5 LAS7ICI7Y: A MASUR OI RSPONS 129
3.2 Elasticity of Laboi Supply: A Special Application
The concept of piice elasticity of supply can be applied to laboi to show how the quantity of laboi sup-
plied iesponds to changes in wages oi salaiies. What makes this case inteiesting is that it has some-
times been found that the measuied elasticity is negative, that is, that an inciease in the wage iate is as-
sociated with a ieduction in the quantity of laboi supplied.
In most cases, laboi supply cuives have theii noimal upwaid slope: highei wages induce people to
woik moie. Foi them, having the additional income fiom woiking moie is piefeiable to having moie
leisuie time. Howevei, wage incieases may lead some people in veiy highly paid jobs to cut back on the
numbei of houis they woik because theii incomes aie alieady high and they would iathei have moie
time foi leisuie activities. In this case, the laboi supply cuive would have a negative slope. The ieasons
foi this phenomenon aie explained moie fully in a latei chaptei.
This chaptei has coveied a vaiiety of elasticity measuies. All iepoit the degiee to which a depend-
ent vaiiable iesponds to a change in an independent vaiiable. As we have seen, the degiee of this ie-
sponse can play a ciitically impoitant iole in deteimining the outcomes of a wide iange of economic
events. Table 3.2
[3]
piovides examples of some estimates of elasticities.
130 PRINCIPLS OI CONOMICS
7A8L 5. 2 SeIected Iasticity stimates
Product Iasticity Product Iasticity Product Iasticity
Price Iasticity
of Demand
Cross Price Iasticity of
Demand
Income Iasticity of
Demand
.uJe c|| (|.S.) 0.06 A|cc|c| .|t| .espect tc p.|ce c|
|e.c|r
0.05 SpeeJ|rg c|t.t|crs 0.26 tc
0.33
C.sc||re 0.1 |ue| .|t| .espect tc p.|ce c|
t..rspc.t
0.48 |.b.r |ub||c .ust |r
|..rce .rJ V.J.|J
(.espect|ve|y)
0.23,
0.26
SpeeJ|rg
c|t.t|crs
0.21 A|cc|c| .|t| .espect tc p.|ce c|
|ccJ
0.16 C.curJ bee| 0.19
.bb.ge 0.25 V..u.r. .|t| .espect tc p.|ce
c| |e.c|r (s|n||.. |c. ccc.|re)
0.01 |ctte.y |rst.rt g.ne
s.|es |r c|c..Jc
0.06
cc.|re (t.c
est|n.tes)
0.28,
1.0
bee. .|t| .espect tc p.|ce c|
.|re J|st|||eJ ||,uc. (ycurg
J.|rke.s)
0.0 |e.c|r 0.00
A|cc|c| 0.30 bee. .|t| .espect tc p.|ce c|
J|st|||eJ ||,uc. (ycurg J.|rke.s)
0.0 V..u.r., .|cc|c|,
ccc.|re
+0.00
|e.c|es 0.38 |c.k .|t| .espect tc p.|ce c|
pcu|t.y
0.06 |ct.tces 0.15
V..u.r. 0.4 |c.k .|t| .espect tc p.|ce c|
g.curJ bee|
0.23 |ccJ 0.2
|g..ettes (.||
sncke.s, t.c
est|n.tes)
0.4,
0.32
C.curJ bee| .|t| .espect tc
p.|ce c| pcu|t.y
0.24 |ct||rg 0.3
.uJe c|| (|.S.) 0.45 C.curJ bee| .|t| .espect tc
p.|ce c| pc.k
0.35 bee. 0.4
V||k (t.c
est|n.tes)
0.49,
0.63
cke .|t| .espect tc p.|ce c|
|eps|
0.61 |ggs 0.5
C.sc||re
(|rte.neJ|.te
te.n)
0.5 |eps| .|t| .espect tc p.|ce c|
cke
0.80 cke 0.60
Sc|t J.|rks 0.55 |cc.| te|ev|s|cr .Jve.t|s|rg .|t|
.espect tc p.|ce c| ..J|c
.Jve.t|s|rg
1.0 S|e|te. 0.
..rspc.t.t|cr 0.6 Sncke|ess tcb.ccc .|t| .espect
tc p.|ce c| c|g..ettes (ycurg
n.|es)
1.2 bee| (t.b|e cutsrct
g.curJ)
0.81
|ccJ 0. Price Iasticity of SuppIy O..rges 0.83
bee. 0. tc
0.9
||ys|c|.rs (Spec|.||st) 0.3 App|es 1.32
|g..ettes
(teer.ge.s, t.c
est|n.tes)
0.9 tc
1.5
||ys|c|.rs (|.|n..y ..e) 0.0 |e|su.e 1.4
|e.c|r 0.94 ||ys|c|.rs (+curg n.|e) 0.2 |e.c|es 1.43
C.curJ bee| 1.0 ||ys|c|.rs (+curg |en.|e) 0.5 |e.|t| c..e 1.6
ctt.ge c|eese 1.1 V||k 0.36 ||g|e. eJuc.t|cr 1.6
C.sc||re 1.5 V||k 0.5
cke 1.1 |||J c..e |.bc. 2
..rspc.t.t|cr 1.9
|eps| 2.08
|.es| tcn.tces 2.22
|ccJ 2.3
|ettuce 2.58
Note: "=short-run, ""=Iong-run
CHAP7R 5 LAS7ICI7Y: A MASUR OI RSPONS 131
k Y 7 A k A W A Y S
< |e p.|ce e|.st|c|ty c| supp|y ne.su.es t|e .espcrs|veress c| ,u.rt|ty supp||eJ tc c|.rges |r p.|ce. |t |s t|e
pe.cert.ge c|.rge |r ,u.rt|ty supp||eJ J|v|JeJ by t|e pe.cert.ge c|.rge |r p.|ce. |t |s usu.||y pcs|t|ve.
< Supp|y |s p.|ce |re|.st|c || t|e p.|ce e|.st|c|ty c| supp|y |s |ess t|.r 1, |t |s ur|t p.|ce e|.st|c || t|e p.|ce
e|.st|c|ty c| supp|y |s e,u.| tc 1, .rJ |t |s p.|ce e|.st|c || t|e p.|ce e|.st|c|ty c| supp|y |s g.e.te. t|.r 1. A
ve.t|c.| supp|y cu.ve |s s.|J tc be pe.|ect|y |re|.st|c. A |c.|.crt.| supp|y cu.ve |s s.|J tc be pe.|ect|y e|.st|c.
< |e p.|ce e|.st|c|ty c| supp|y |s g.e.te. .|er t|e |ergt| c| t|ne urJe. ccrs|Je..t|cr |s |crge. bec.use
cve. t|ne p.cJuce.s |.ve nc.e cpt|crs |c. .Just|rg tc t|e c|.rge |r p.|ce.
< \|er .pp||eJ tc |.bc. supp|y, t|e p.|ce e|.st|c|ty c| supp|y |s usu.||y pcs|t|ve but c.r be reg.t|ve. || ||g|e.
..ges |rJuce pecp|e tc .c.k nc.e, t|e |.bc. supp|y cu.ve |s up...J s|cp|rg .rJ t|e p.|ce e|.st|c|ty c|
supp|y |s pcs|t|ve. |r scne ve.y ||g|p.y|rg p.c|ess|crs, t|e |.bc. supp|y cu.ve n.y |.ve . reg.t|ve s|cpe,
.||c| |e.Js tc . reg.t|ve p.|ce e|.st|c|ty c| supp|y.
7 R Y I 7 !
|r t|e |.te 1990s, |t ..s .epc.teJ cr t|e re.s t|.t t|e ||g|tec| |rJust.y ..s .c..|eJ .bcut be|rg .b|e tc
|rJ ercug| .c.ke.s .|t| ccnpute..e|.teJ expe.t|se. 'cb c|e.s |c. .ecert cc||ege g..Ju.tes .|t| Jeg.ees |r
ccnpute. sc|erce .ert .|t| ||g| s.|..|es. |t ..s .|sc .epc.teJ t|.t nc.e urJe.g..Ju.tes t|.r eve. .e.e n.
c.|rg |r ccnpute. sc|erce. cnp..e t|e p.|ce e|.st|c|ty c| supp|y c| ccnpute. sc|ert|sts .t t|.t pc|rt |r t|ne
tc t|e p.|ce e|.st|c|ty c| supp|y c| ccnpute. sc|ert|sts cve. . |crge. pe.|cJ c|, s.y, 1999 tc 2009.
Case in Point: A Variety of Labor SuppIy Iasticities
2010 jupiterimages Corporation
StuJ|es suppc.t t|e |Je. t|.t |.bc. supp|y |s |ess e|.st|c |r ||g|p.y|rg cbs t|.r |r |c.e.p.y|rg cres.
|c. ex.np|e, |.v|J V. b|.u est|n.teJ t|e |.bc. supp|y c| c|||Jc..e .c.ke.s tc be ve.y p.|ce e|.st|c, .|t| es
t|n.teJ p.|ce e|.st|c|ty c| |.bc. supp|y c| .bcut 2.0. ||s ne.rs t|.t . 10 |rc.e.se |r ..ges |e.Js tc . 20 |r
c.e.se |r t|e ,u.rt|ty c| |.bc. supp||eJ. 'c|r bu.kett est|n.teJ t|e |.bc. supp|y c| bct| ru.s|rg .ss|st.rts
.rJ ru.ses tc be p.|ce e|.st|c, .|t| t|.t c| ru.s|rg .ss|st.rts tc be 1.9 (ve.y c|cse tc t|.t c| c|||Jc..e .c.ke.s)
.rJ c| ru.ses tc be 1.1. |cte t|.t t|e p.|ce e|.st|c|ty c| |.bc. supp|y c| t|e ||g|e.p.|J ru.ses |s . b|t |c.e.
t|.r t|.t c| |c.e.p.|J ru.s|rg .ss|st.rts.
132 PRINCIPLS OI CONOMICS
|r ccrt..st, 'c|r ||..c .rJ |.v|J b|unert|.| est|n.teJ t|e p.|ce e|.st|c|ty c| |.bc. supp|y |c. ycurg p|ys|
c|.rs (urJe. t|e .ge c| 40) tc be .bcut 0.3. ||s ne.rs t|.t . 10 |rc.e.se |r ..ges |e.Js tc .r |rc.e.se |r
t|e ,u.rt|ty c| |.bc. supp||eJ c| cr|y .bcut 3. |r .JJ|t|cr, .|er ||..c .rJ b|unert|.| |cckeJ .t |.bc. sup
p|y e|.st|c|t|es by gerJe., t|ey |curJ t|e |en.|e p|ys|c|.rs |.bc. supp|y p.|ce e|.st|c|ty tc be . b|t ||g|e. (.t
.bcut 0.5) t|.r t|.t c| t|e n.|es (.t .bcut 0.2) |r t|e s.np|e. bec.use e..r|rgs c| |en.|e p|ys|c|.rs |r t|e
s.np|e .e.e |c.e. t|.r e..r|rgs c| t|e n.|e p|ys|c|.rs |r t|e s.np|e, t||s J||e.erce |r |.bc. supp|y e|.st|c|t
|es ..s expecteJ. Vc.ecve., s|rce t|e s.np|e ccrs|steJ c| p|ys|c|.rs |r t|e e..|y p|.ses c| t|e|. c..ee.s, t|e
pcs|t|ve, t|cug| sn.||, p.|ce e|.st|c|t|es .e.e .|sc expecteJ. V.ry c| t|e |rJ|v|Ju.|s |r t|e s.np|e .|sc |.J
||g| Jebt |eve|s, c|ter |.cn eJuc.t|cr.| |c.rs. |us, t|e c|.rce tc e..r nc.e by .c.k|rg nc.e |s .r cppc.
tur|ty tc .ep.y eJuc.t|cr.| .rJ ct|e. |c.rs.
|r .rct|e. stuJy c| p|ys|c|.rs |.bc. supp|y t|.t ..s rct .est.|cteJ tc ycurg p|ys|c|.rs, |cug|.s V. b.c.r
|curJ t|e |.bc. supp|y p.|ce e|.st|c|ty |c. p.|n..y c..e p|ys|c|.rs tc be c|cse tc .e.c .rJ t|.t c| spec|.||sts tc
be reg.t|ve, .t .bcut 0.3. |us, |c. t||s s.np|e c| p|ys|c|.rs, |rc.e.ses |r ..ges |.ve ||tt|e c. rc e|ect cr t|e
.ncurt t|e p.|n..y c..e Jcctc.s .c.k, .|||e . 10 |rc.e.se |r ..ges |c. spec|.||sts eJoce t|e|. ,u.rt|ty c|
|.bc. supp||eJ by .bcut 3. bec.use t|e e..r|rgs c| spec|.||sts exceeJ t|cse c| p.|n..y c..e Jcctc.s, t||s
e|.st|c|ty J||e.ert|.| .|sc n.kes serse.
.ooce |o.|J / ||oo. |e .o|, o| |||J oe |oo.' .ooo| o| |oo |coo|c 2 (^|| 993: 32434.. |o.|J / |o.. |e ||| o| o|
||,|c|o .e.|ce ^ oec||o oJ ||e|o o .o|,.' |e.|e. o| |coo|c oJ .|o||||c .6 (2: (/o, 994: 3S9393. .o| | |o|e||. |e |oo
.o|, o| |oe oJ |o| ^||o| | ||e |||eJ .|o|e.' |o|e |coo|c .ooo| 3(4: (|o|| 2005: 5S5599. .o| ^ ||o oJ |oo| ||oe||o|
||,|c|o |oo .o|, |o |coe ||ec| /o||e.' .ooo| o| |eo||| |coo|c 34 (|ecee 994: 433453
A N S W R 7 O 7 R Y I 7 ! P R O 8 L M
\|||e .t . pc|rt |r t|ne t|e supp|y c| pecp|e .|t| Jeg.ees |r ccnpute. sc|erce |s ve.y p.|ce |re|.st|c, cve.
t|ne t|e e|.st|c|ty s|cu|J .|se. |.t nc.e stuJerts .e.e n.c.|rg |r ccnpute. sc|erce |erJs c.eJerce tc t||s
p.eJ|ct|cr. As supp|y beccnes nc.e p.|ce e|.st|c, s.|..|es |r t||s |e|J s|cu|J .|se nc.e s|c.|y.
4. REVIEW AND PRACTICE
Summary
||s c|.pte. |rt.cJuceJ . re. tcc|. t|e ccrcept c| e|.st|c|ty. ||.st|c|ty |s . ne.su.e c| t|e Jeg.ee tc .||c| .
JeperJert v..|.b|e .espcrJs tc . c|.rge |r .r |rJeperJert v..|.b|e. |t |s t|e pe.cert.ge c|.rge |r t|e Je
perJert v..|.b|e J|v|JeJ by t|e pe.cert.ge c|.rge |r t|e |rJeperJert v..|.b|e, .|| ct|e. t||rgs urc|.rgeJ.
|e ncst .|Je|y useJ e|.st|c|ty ne.su.e |s t|e p.|ce e|.st|c|ty c| Jen.rJ, .||c| .e|ects t|e .espcrs|veress c|
,u.rt|ty Jen.rJeJ tc c|.rges |r p.|ce. |en.rJ |s s.|J tc be p.|ce e|.st|c || t|e .bsc|ute v.|ue c| t|e p.|ce
e|.st|c|ty c| Jen.rJ |s g.e.te. t|.r 1, ur|t p.|ce e|.st|c || |t |s e,u.| tc 1, .rJ p.|ce |re|.st|c || |t |s |ess t|.r 1.
|e p.|ce e|.st|c|ty c| Jen.rJ |s use|u| |r |c.ec.st|rg t|e .espcrse c| ,u.rt|ty Jen.rJeJ tc p.|ce c|.rges, |t
|s .|sc use|u| |c. p.eJ|ct|rg t|e |np.ct . p.|ce c|.rge .||| |.ve cr tct.| .everue. ct.| .everue ncves |r t|e
J|.ect|cr c| t|e ,u.rt|ty c|.rge || Jen.rJ |s p.|ce e|.st|c, |t ncves |r t|e J|.ect|cr c| t|e p.|ce c|.rge || Je
n.rJ |s p.|ce |re|.st|c, .rJ |t Jces rct c|.rge || Jen.rJ |s ur|t p.|ce e|.st|c. |e ncst |npc.t.rt Jete.n|r
.rts c| t|e p.|ce e|.st|c|ty c| Jen.rJ ..e t|e .v.||.b|||ty c| subst|tutes, t|e |npc.t.rce c| t|e |ten |r |cuse
|c|J buJgets, .rJ t|ne.
.c ct|e. e|.st|c|ty ne.su.es ccnncr|y useJ |r ccrurct|cr .|t| Jen.rJ ..e |rccne e|.st|c|ty .rJ c.css
p.|ce e|.st|c|ty. |e s|grs c| t|ese e|.st|c|ty ne.su.es p|.y |npc.t.rt .c|es. A pcs|t|ve |rccne e|.st|c|ty te||s us
t|.t . gccJ |s rc.n.|, . reg.t|ve |rccne e|.st|c|ty te||s us t|e gccJ |s |r|e.|c.. A pcs|t|ve c.css p.|ce e|.st|c|ty
te||s us t|.t t.c gccJs ..e subst|tutes, . reg.t|ve c.css p.|ce e|.st|c|ty te||s us t|ey ..e ccnp|enerts.
||.st|c|ty c| supp|y ne.su.es t|e .espcrs|veress c| ,u.rt|ty supp||eJ tc c|.rges |r p.|ce. |e v.|ue c| p.|ce
e|.st|c|ty c| supp|y |s gere..||y pcs|t|ve. Supp|y |s c|.ss||eJ .s be|rg p.|ce e|.st|c, ur|t p.|ce e|.st|c, c. p.|ce |r
e|.st|c || p.|ce e|.st|c|ty |s g.e.te. t|.r 1, e,u.| tc 1, c. |ess t|.r 1, .espect|ve|y. |e |ergt| c| t|ne cve. .||c|
supp|y |s be|rg ccrs|Je.eJ |s .r |npc.t.rt Jete.n|r.rt c| t|e p.|ce e|.st|c|ty c| supp|y.
CHAP7R 5 LAS7ICI7Y: A MASUR OI RSPONS 133
C O N C P 7 P R O 8 L M S
1. |xp|.|r .|y t|e p.|ce e|.st|c|ty c| Jen.rJ |s gere..||y . reg.t|ve runbe., except |r t|e c.ses .|e.e t|e
Jen.rJ cu.ve |s pe.|ect|y e|.st|c c. pe.|ect|y |re|.st|c. \|.t .cu|J be |np||eJ by . pcs|t|ve p.|ce e|.st|c|ty
c| Jen.rJ`
2. |xp|.|r .|y t|e s|gr (pcs|t|ve c. reg.t|ve) c| t|e c.css p.|ce e|.st|c|ty c| Jen.rJ |s |npc.t.rt.
3. |xp|.|r .|y t|e s|gr (pcs|t|ve c. reg.t|ve) c| t|e |rccne e|.st|c|ty c| Jen.rJ |s |npc.t.rt.
4. |ccrcn|sts |.|e |e|er .rJ .t|y |c|e|n \esse||s |curJ t|.t t|e p.|ce e|.st|c|ty c| Jen.rJ |c. |.es| n||k
|s 0.63 .rJ t|e p.|ce e|.st|c|ty c| Jen.rJ |c. cctt.ge c|eese |s 1.1.
6
\|y Jc ycu t||rk t|e e|.st|c|ty
est|n.tes J||e.`
5. |e p.|ce e|.st|c|ty c| Jen.rJ |c. |e.|t| c..e |.s beer est|n.teJ tc be 0.2. |...cte.|.e t||s Jen.rJ .s
p.|ce e|.st|c, ur|t p.|ce e|.st|c, c. p.|ce |re|.st|c. |e text ..gues t|.t t|e g.e.te. t|e |npc.t.rce c| .r |ten
|r ccrsune. buJgets, t|e g.e.te. |ts e|.st|c|ty. |e.|t|c..e ccsts .cccurt |c. . .e|.t|ve|y |..ge s|..e c|
|cuse|c|J buJgets. |c. ccu|J t|e p.|ce e|.st|c|ty c| Jen.rJ |c. |e.|t| c..e be suc| . sn.|| runbe.`
6. Suppcse ycu ..e .b|e tc c.g.r|.e .r .|||.rce t|.t |rc|uJes .|| |..ne.s. |ey .g.ee tc |c||c. t|e g.cups
|rst.uct|crs .|t| .espect tc t|e ,u.rt|ty c| .g.|cu|tu..| p.cJucts t|ey p.cJuce. \|.t n|g|t t|e g.cup
seek tc Jc` \|y`
. Suppcse ycu ..e t|e c||e| execut|ve c|ce. c| . |.n, .rJ ycu |.ve beer p|.rr|rg tc .eJuce ycu. p.|ces.
+cu. n..ket|rg n.r.ge. .epc.ts t|.t t|e p.|ce e|.st|c|ty c| Jen.rJ |c. ycu. p.cJuct |s 0.65. |c. .||| t||s
re.s .|ect ycu. p|.rs`
8. Suppcse t|e |rccne e|.st|c|ty c| t|e Jen.rJ |c. be.rs |s 0.8. |rte.p.et t||s runbe..
9. ..rspc.t.t|cr eccrcn|sts gere..||y .g.ee t|.t t|e c.css p.|ce e|.st|c|ty c| Jen.rJ |c. .utcncb||e use
.|t| .espect tc t|e p.|ce c| bus |..es |s .bcut 0. |xp|.|r .|.t t||s runbe. ne.rs.
10. Suppcse t|e p.|ce e|.st|c|ty c| supp|y c| tcn.tces .s ne.su.eJ cr . g|ver J.y |r 'u|y |s 0. |rte.p.et t||s
runbe..
11. |e p.|ce e|.st|c|ty c| supp|y |c. c|||Jc..e .c.ke.s ..s .epc.teJ tc be ,u|te ||g|, .bcut 2. \|.t .|||
|.pper tc t|e ..ges c| c|||Jc..e .c.ke.s .s Jen.rJ |c. t|en |rc.e.ses, ccnp..eJ tc .|.t .cu|J
|.pper || t|e ne.su.eJ p.|ce e|.st|c|ty c| supp|y .e.e |c.e.`
12. |e .se |r |c|rt cr c|g..ette t.xes .rJ teer snck|rg suggests t|.t . ||g|e. t.x cr c|g..ettes .cu|J
.eJuce teer snck|rg .rJ p.en.tu.e Je.t|s. S|cu|J c|g..ette t.xes t|e.e|c.e be ..|seJ`
134 PRINCIPLS OI CONOMICS
N U M R I C A L P R O 8 L M S
1. |ccrcn|st |.v|J |cne. |curJ t|.t |r |rt.cJuctc.y eccrcn|cs c|.sses . 10 |rc.e.se |r c|.ss .tterJ.rce
|s .sscc|.teJ .|t| . 4 |rc.e.se |r ccu.se g..Je.

\|.t |s t|e e|.st|c|ty c| ccu.se g..Je .|t| .espect tc


c|.ss .tterJ.rce`
2. |e|e. tc ||gu.e 5.2 .rJ
.. |s|rg t|e ..c e|.st|c|ty c| Jen.rJ |c.nu|., ccnpute t|e p.|ce e|.st|c|ty c| Jen.rJ bet.eer
pc|rts b .rJ .
b. |s|rg t|e ..c e|.st|c|ty c| Jen.rJ |c.nu|., ccnpute t|e p.|ce e|.st|c|ty c| Jen.rJ bet.eer
pc|rts | .rJ |.
c. |c. Jc t|e v.|ues c| p.|ce e|.st|c|ty c| Jen.rJ ccnp..e` \|y ..e t|ey t|e s.ne c. J||e.ert`
J. cnpute t|e s|cpe c| t|e Jen.rJ cu.ve bet.eer pc|rts b .rJ .
e. cnpute. t|e s|cpe c| t|e Jen.rJ cu.ve bet.eer pc|rts | .rJ |.
|. |c. Jc t|e s|cpes ccnp..e` \|y ..e t|ey t|e s.ne c. J||e.ert`
3. crs|Je. t|e |c||c.|rg ,ucte |.cn |e /o|| .|ee| .ooo| A bunpe. c.cp c| c..rges |r ||c.|J. |.st ye..
J.cve Jc.r c..rge p.|ces. As u|ce n..kete.s ccsts |e||, t|ey cut p.|ces by .s nuc| .s 15. |.t ..s
ercug| tc tenpt scne v.|uec.|erteJ custcne.s. ur|t vc|une c| |.c.er u|ces .ctu.||y .cse .bcut 6
Ju.|rg t|e ,u..te..
.. C|ver t|ese runbe.s, .rJ .ssun|rg t|e.e .e.e rc c|.rges |r Jen.rJ s|||te.s |c. |.c.er c..rge
u|ce, .|.t ..s t|e p.|ce e|.st|c|ty c| Jen.rJ |c. |.c.er c..rge u|ce`
b. \|.t Jc ycu t||rk |.ppereJ tc tct.| sperJ|rg cr |.c.er c..rge u|ce` \|y`
4. Suppcse ycu ..e t|e n.r.ge. c| . .est.u..rt t|.t se.ves .r .ve..ge c| 400 ne.|s pe. J.y .t .r .ve..ge
p.|ce pe. ne.| c| 20. Or t|e b.s|s c| . su.vey, ycu |.ve Jete.n|reJ t|.t .eJuc|rg t|e p.|ce c| .r .ve..ge
ne.| tc 18 .cu|J |rc.e.se t|e ,u.rt|ty Jen.rJeJ tc 450 pe. J.y.
.. cnpute t|e p.|ce e|.st|c|ty c| Jen.rJ bet.eer t|ese t.c pc|rts.
b. \cu|J ycu expect tct.| .everues tc .|se c. |.||` |xp|.|r.
c. Suppcse ycu |.ve .eJuceJ t|e .ve..ge p.|ce c| . ne.| tc 18 .rJ ..e ccrs|Je.|rg . |u.t|e.
.eJuct|cr tc 16. Arct|e. su.vey s|c.s t|.t t|e ,u.rt|ty Jen.rJeJ c| ne.|s .||| |rc.e.se |.cn
450 tc 500 pe. J.y. cnpute t|e p.|ce e|.st|c|ty c| Jen.rJ bet.eer t|ese t.c pc|rts.
J. \cu|J ycu expect tct.| .everue tc .|se c. |.|| .s . .esu|t c| t||s seccrJ p.|ce .eJuct|cr` |xp|.|r.
e. cnpute tct.| .everue .t t|e t|.ee ne.| p.|ces. |c t|ese tct.|s ccr|.n ycu. .rs.e.s |r (b) .rJ
(J) .bcve`
5. |e text rctes t|.t, |c. .ry ||re.. Jen.rJ cu.ve, Jen.rJ |s p.|ce e|.st|c |r t|e uppe. |.|| .rJ p.|ce
|re|.st|c |r t|e |c.e. |.||. crs|Je. t|e |c||c.|rg Jen.rJ cu.ves.
|e t.b|e g|ves t|e p.|ces .rJ ,u.rt|t|es cc..espcrJ|rg tc e.c| c| t|e pc|rts s|c.r cr t|e t.c Jen.rJ
cu.ves.
CHAP7R 5 LAS7ICI7Y: A MASUR OI RSPONS 135
Demand curve |
1
[PaneI (a)] Demand curve |
2
[PaneI (b)]
Price Quantity Price Quantity
A 80 2 | 8 20
b 0 3 | 30
30 C 3 0
| 20 8 | 2 80
.. cnpute t|e p.|ce e|.st|c|ty c| Jen.rJ bet.eer pc|rts A .rJ b .rJ bet.eer pc|rts .rJ | cr
Jen.rJ cu.ve |
1
|r |.re| (.). A.e ycu. .esu|ts ccrs|stert .|t| t|e rct|cr t|.t . ||re.. Jen.rJ
cu.ve |s p.|ce e|.st|c |r |ts uppe. |.|| .rJ p.|ce |re|.st|c |r |ts |c.e. |.||`
b. cnpute t|e p.|ce e|.st|c|ty c| Jen.rJ bet.eer pc|rts | .rJ | .rJ bet.eer pc|rts C .rJ | cr
Jen.rJ cu.ve |
2
|r |.re| (b). A.e ycu. .esu|ts ccrs|stert .|t| t|e rct|cr t|.t . ||re.. Jen.rJ
cu.ve |s p.|ce e|.st|c |r |ts uppe. |.|| .rJ p.|ce |re|.st|c |r |ts |c.e. |.||`
c. cnp..e tct.| sperJ|rg .t pc|rts A .rJ b cr |
1
|r |.re| (.). |s ycu. .esu|t ccrs|stert .|t| ycu.
|rJ|rg .bcut t|e p.|ce e|.st|c|ty c| Jen.rJ bet.eer t|cse t.c pc|rts`
J. cnp..e tct.| sperJ|rg .t pc|rts .rJ | cr |
1
|r |.re| (.). |s ycu. .esu|t ccrs|stert .|t| ycu.
|rJ|rg .bcut t|e p.|ce e|.st|c|ty c| Jen.rJ bet.eer t|cse t.c pc|rts`
e. cnp..e tct.| sperJ|rg .t pc|rts | .rJ | cr |
2
|r |.re| (b). |s ycu. .esu|t ccrs|stert .|t| ycu.
|rJ|rg .bcut t|e p.|ce e|.st|c|ty c| Jen.rJ bet.eer t|cse t.c pc|rts`
|. cnp..e tct.| sperJ|rg .t pc|rts C .rJ | cr |
2
|r |.re| (b). |s ycu. .esu|t ccrs|stert .|t| ycu.
|rJ|rg .bcut t|e p.|ce e|.st|c|ty c| Jen.rJ bet.eer t|cse t.c pc|rts`
6. Suppcse '.r|ce buys t|e |c||c.|rg .ncurts c| v..|cus |ccJ |tens JeperJ|rg cr |e. .eek|y |rccne.
WeekIy Income Hamburgers Pizza Ice Cream Sundaes
500 3 3 2
50 4 2 2
.. cnpute '.r|ces |rccne e|.st|c|ty c| Jen.rJ |c. |.nbu.ge.s.
b. cnpute '.r|ces |rccne e|.st|c|ty c| Jen.rJ |c. p|....
c. cnpute '.r|ces |rccne e|.st|c|ty c| Jen.rJ |c. |ce c.e.n surJ.es.
J. |.ss||y e.c| gccJ .s rc.n.| c. |r|e.|c..
. Suppcse t|e |c||c.|rg t.b|e Jesc.|bes 'cce|yrs .eek|y sr.ck pu.c|.ses, .||c| v..y JeperJ|rg cr t|e
p.|ce c| . b.g c| c||ps.
Price of bag of chips 8ags of chips Containers of saIsa 8ags of pretzeIs Cans of soda
1.00 2 3 1 4
1.50 1 2 2 4
.. cnpute t|e c.css p.|ce e|.st|c|ty c| s.|s. .|t| .espect tc t|e p.|ce c| . b.g c| c||ps.
b. cnpute t|e c.css p.|ce e|.st|c|ty c| p.et.e|s .|t| .espect tc t|e p.|ce c| . b.g c| c||ps.
c. cnpute t|e c.css p.|ce e|.st|c|ty c| scJ. .|t| .espect tc t|e p.|ce c| . b.g c| c||ps.
J. A.e c||ps .rJ s.|s. subst|tutes c. ccnp|enerts` |c. Jc ycu krc.`
e. A.e c||ps .rJ p.et.e|s subst|tutes c. ccnp|enerts` |c. Jc ycu krc.`
|. A.e c||ps .rJ scJ. subst|tutes c. ccnp|enerts` |c. Jc ycu krc.`
8. |e t.b|e be|c. Jesc.|bes t|e supp|y cu.ve |c. ||g|t bu|bs.
Price per Iight buIb Quantity suppIied per day
1.00 500
1.50 3,000
2.00 4,000
2.50 4,500
3.00 4,500
136 PRINCIPLS OI CONOMICS
cnpute t|e p.|ce e|.st|c|ty c| supp|y .rJ Jete.n|re .|et|e. supp|y |s p.|ce e|.st|c, p.|ce |re|.st|c,
pe.|ect|y e|.st|c, pe.|ect|y |re|.st|c, c. ur|t e|.st|c.
.. .|er t|e p.|ce c| . ||g|t bu|b |rc.e.ses |.cn 1.00 tc 1.50.
b. .|er t|e p.|ce c| . ||g|t bu|b |rc.e.ses |.cn 1.50 tc 2.00.
c. .|er t|e p.|ce c| . ||g|t bu|b |rc.e.ses |.cn 2.00 tc 2.50.
J. .|er t|e p.|ce c| . ||g|t bu|b |rc.e.ses |.cn 2.50 tc 3.00.
CHAP7R 5 LAS7ICI7Y: A MASUR OI RSPONS 137
1.
2.
3.
4.
5.
6.
.
ENDNOTES
|ct|ce t|.t s|rce t|e runbe. c| ur|ts sc|J c| . gccJ |s t|e s.ne .s t|e runbe. c|
ur|ts bcug|t, t|e Je|r|t|cr |c. tct.| .everue ccu|J .|sc be useJ tc Je|re tct.|
sperJ|rg. \||c| te.n .e use JeperJs cr t|e ,uest|cr .t |.rJ. || .e ..e t.y|rg tc
Jete.n|re .|.t |.ppers tc .everues c| se||e.s, t|er .e ..e .sk|rg .bcut tct.| .ev
erue. || .e ..e t.y|rg tc Jete.n|re |c. nuc| ccrsune.s sperJ, t|er .e ..e .sk|rg
.bcut tct.| sperJ|rg.
||v|s|cr by .e.c .esu|ts |r .r urJe|reJ sc|ut|cr. S.y|rg t|.t t|e p.|ce e|.st|c|ty c|
Jen.rJ |s |r|r|te .e,u|.es t|.t .e s.y t|e Jercn|r.tc. .pp.c.c|es .e.c.
See Cec.ges b.esscr, 'cyce |..g.y, 'e.r|cup V.J.e, .rJ A|.|r ||.ctte, |ccrcn|c
.rJ St.uctu..| |ete.n|r.rts c| t|e |en.rJ |c. |.erc| ..rspc.t. Ar Ar.|ys|s cr .
|.re| c| |.erc| |.b.r A.e.s |s|rg S|.|rk.ge |st|n.tc.s. oo|o||o |eeoc|
|o| ^ 38. 4 (V.y 2004). 269285. See .|sc Arr. V.t.s. |en.rJ .rJ |everue |n
p||c.t|crs c| .r |rteg..teJ ..rspc.t |c||cy. |e .se c| V.J.|J. oo| |e.|e.
24.2 (V..c| 2004). 19521.
|cbe.t b. |ke|urJ, S. |c.J, .rJ 'c|r |. '.ckscr. A.e |cc.| \ V..kets Sep...te V..
kets` ||eo||oo| .ooo| o| ||e |coo|c o| |o|e .1 (2000). 99.
A|t|cug| c|cse tc .e.c |r .|| c.ses, t|e s|gr||c.rt .rJ pcs|t|ve s|grs c| |rccne e|.st|
c|ty |c. n..u.r., .|cc|c|, .rJ ccc.|re suggest t|.t t|ey ..e rc.n.| gccJs, but s|g
r||c.rt .rJ reg.t|ve s|grs, |r t|e c.se c| |e.c|r, suggest t|.t |e.c|r |s .r |r|e.|c.
gccJ, S.|e. .rJ |.|cupk. (c|teJ be|c.) suggest t|e e|ects c| |rccne |c. .|| |cu.
subst.rces n|g|t be .|ecteJ by eJuc.t|cr.
Scu.ces. 'c|r A. .u..s. |ub||c |c||cy .rJ Snck|rg ess.t|cr .ncrg +curg
AJu|ts |r t|e |r|teJ St.tes, |eo||| |o||c,, 68.3 ('ure 2004). 321332. Cec.ges
b.esscr, 'cyce |..g.y, 'e.r|cup V.J.e, .rJ A|.|r ||.ctte, |ccrcn|c .rJ St.uctu..|
|ete.n|r.rts c| t|e |en.rJ |c. |.erc| ..rspc.t. Ar Ar.|ys|s cr . |.re| c| |.erc|
|.b.r A.e.s |s|rg S|.|rk.ge |st|n.tc.s, oo|o||o |eeoc| |o| ^ 38.4 (V.y
2004). 269285, Avre. b..||.r .rJ b.uce S.ce.Jcte, |e |espcrse c| .|n|r.|s .rJ
|cr.|n|r.|s tc ||res, .ooo| o| |o. oJ |coo|c, 4.1 (Ap.|| 2004). 11, |.r.
|css .rJ |..rk '. |.|cupk., |e ||ect c| |ub||c |c||c|es .rJ |.|ces cr +cut|
Snck|rg, .oo||e |coo|c .ooo| 0.4 (Ap.|| 2004). 96815, Arr. V.t.s,
|en.rJ .rJ |everue |np||c.t|crs c| .r |rteg..teJ ..rspc.t |c||cy. |e .se c|
V.J.|J, oo| |e.|e., 24.2 (V..c| 2004). 19521, V.tt|e. . |...e||y, e..y |.
|ec|.cek, .rJ |..rk '. |.|cupk., |e |np.ct c| cb.ccc crt.c| |.cg..n |x
perJ|tu.es cr Agg.eg.te |g..ette S.|es. 19812000, .ooo| o| |eo||| |coo|c
22.5 (Septenbe. 2003). 843859, |cbe.t b. |ke|urJ, S. |c.J, .rJ 'c|r |. '.ckscr. A.e
|cc.| \ V..kets Sep...te V..kets` ||eo||oo| .ooo| o| ||e |coo|c o| |o|e
.1 (2000). 99, |er.y S.|e. .rJ |..rk |.|cupk., |e |en.rJ |c. ||||c|t |.ugs,
|coo|c |o|, 3(3) ('u|y, 1999). 401411, |cbe.t \. |cge|, .tc||rg |p \|t| t|e
|ccrcny, ^e|co |coo|c |e.|e. 89(1) (V..c|, 1999).121, V|c|.e| C.cssn.r,
A Su.vey c| |ccrcn|c VcJe|s c| AJJ|ct|ve be|.v|c., .ooo| o| |o |oe 28.3
(Sunne. 1998).631643, S.r|b b|uy.r .rJ ||gcbe.tc A. |cpe., O||gcpc|y |c.e. |r
t|e |ccJ .rJ cb.ccc |rJust.|es, ^e|co .ooo| o| ^|co||oo| |coo|c 9
(August 199).10351043, V|c|.e| C.cssn.r, |g..ette .xes, |o||c |eo||| |eo|
112.4 ('u|y/August 199). 29029, Arr |.rser, |e .x |rc|Jerce c| t|e c|c..Jc
St.te |ctte.y |rst.rt C.ne, |o||c ||oce (oo|e|, 23(3) ('u|y, 1995).385398,
|.r|e| b. Su|ts, Ag.|cu|tu.e, |r \.|te. AJ.ns .rJ '.nes b.cck, eJs., |e .|oc|oe o|
^e|co |Jo|,, 9
t|
eJ. (|rg|e.ccJ |||s. |.ert|ce |.||, , 1995), pp. 133, |erret|
C. ||.|rg., bee., |r \.|te. AJ.ns .rJ '.nes b.cck, eJs., |e .|oc|oe o| ^e|co |
Jo|,, 9
t|
eJ. (|rg|e.ccJ |||s. |.ert|ce |.||, 1995), pp. 119151, 'c|r A. ||..c .rJ
|.v|J b|unert|.|, ||ys|c|.r |.bc. Supp|y. |c |rccne ||ects V.tte.` .ooo| o|
|eo||| |coo|c 13(4) (|ecenbe. 1994).433453, |cug|.s V. b.c.r, |e ||s|rg
|.|ce c| ||ys|c|.rs Se.v|ces. A c..ect|cr .rJ |xters|cr cr Supp|y, |e.|e. o| |co
o|c oJ .|o||||c 6(2) (V.y 1994).389393, Cec.ge . |.v|s .rJ V|c|.e| |. \c||
ger.rt, |en.rJ ||.st|c|t|es |.cn . ||sc.ete |c|ce VcJe|. |e |.tu..| |.|stn.s
.ee V..ket, .ooo| o| ^|co||oo| |coo|c 5(3) (August 1993).3038, |.v|J V.
b|.u, |e Supp|y c| |||J ..e |.bc., .ooo| o| |oo |coo|c 2(11) (Ap.||
1993).32434, ||c|..J b|urJe|| e| o|., \|.t |c \e |e..r Abcut crsune. |en.rJ
|.tte.rs |.cn V|c.c |.t.`, ^e|co |coo|c |e.|e. 83(3) ('ure 1993).5059, |.
C.sn|, e| o|., |ccrcnet.|c Ar.|ys|s c| c||us|ve be|.v|c. |r . Sc|t|.|rk V..ket,
.ooo| o| |coo|c oJ /ooee| .|o|e, (Sunne. 1992), pp. 2311, V.|.
b.ye, |.\. '.rser, .rJ '.\. |ee, AJve.t|s|rg ||ects |r cnp|ete |en.rJ Systens,
^||eJ |coo|c 24 (1992).1081096, C..y \. b.este. .rJ V|c|.e| |. \c||ger.rt,
|st|n.t|rg |rte..e|.teJ |en.rJs |c. Ve.ts |s|rg |e. Ve.su.es |c. C.curJ .rJ
.b|e ut bee|, ^e|co .ooo| o| ^|co||oo| |coo|c 3 (|cvenbe.
1991).11821194, AJesc|, O. AJe|.., |.|ce |.rges, Supp|y ||.st|c|t|es, |rJust.y O.
g.r|..t|cr, .rJ |.|.y Output ||st.|but|cr, ^e|co .ooo| o| ^|co||oo| |coo|c
3.1 (|eb.u..y 1991).89102, V..k A. |. ||e|rn.r, /oooo o| o| ^oe. o| o|
o|o| (|+.C.eer.ccJ |.ess, 1989), 'u|es V. |ev|re, e| o|., |e |en.rJ |c. ||g|e.
|Juc.t|cr |r |.ee V|JAt|.rt|c St.tes, |e. `o| |coo|c |e.|e. 18 (|.||
1988).320, |.|e |e|er .rJ .t|y |c|e|n \esse||s, |e |en.rJ |c. |.|.y |.cJucts.
St.uctu.e, |.eJ|ct|cr, .rJ |eccnpcs|t|cr, ^e|co .ooo| o| ^|co||oe |coo|c
(V.y 1988).219228, V|c|.e| C.cssn.r .rJ |er.y S.|e., bee. .xes, t|e |eg.|
|.|rk|rg Age, .rJ +cut| Vctc. \e||c|e |.t.||t|es, .ooo| o| |eo| .|oJ|e 16(2) ('ure
198).35134, '.nes V. C.||r .rJ |er.y b. Stee|e, |e, |coo|c oJ |o||c,
(|e. +c.k. Ac.Jen|c |.ess, 1980), p. 232.
|.|e V. |e|er .rJ .t|y |c|e|n \esse|s, |e |en.rJ |c. |.|.y |.cJucts. St.uc
tu.e, |.eJ|ct|cr, .rJ |eccnpcs|t|cr, ^e|co .ooo| o| ^|co||oo| |coo|c 0.2
(V.y 1988). 219228.
|.v|J |cne., |c StuJerts Cc tc |.ss` S|cu|J |ey` .ooo| o| |coo|c |eec|
|.e .3 (Sunne. 1993). 1614.
138 PRINCIPLS OI CONOMICS
| A | | | 6
Markets, Maximizers, and
Efficiency
S7AR7 UP: A DRIV IN 7H COUN7RY
Suppcse ycu Jec|Je tc t.ke . J.|ve. |c. pu.pcses c| t||s ex.np|e, .e .||| .ssune t|.t ycu |.ve . c.. .v.||.b|e, t|.t
t|e .e.t|e. |s p|e.s.rt, .rJ t|.t t|e.e |s .r ..e. re..by t|.t .||| be pe.|ect |c. ycu. J.|ve.
+cu. Jec|s|cr tc t.ke t||s J.|ve |s . c|c|ce. S|rce eccrcn|cs Je.|s .|t| c|c|ces, .e c.r put eccrcn|cs tc .c.k
|r t||rk|rg .bcut |t. |ccrcn|sts .ssune t|.t pecp|e n.ke c|c|ces t|.t n.x|n|.e t|e v.|ue c| scne cbect|ve. +cu
..e . ccrsune., .e .ssune t|.t t.k|rg . J.|ve |s . c|c|ce t|.t n.x|n|.es ycu. ut|||tyt|e s.t|s|.ct|cr ycu cbt.|r
|.cn ycu. use c| gccJs .rJ se.v|ces .rJ |.cn t|e .ct|v|t|es ycu pu.sue.
+cu ce.t.|r|y p|.r tc ercy t|e J.|ve, t|.t ercynert |s t|e bere|t ycu expect |.cn |t. but ycu .||| g|ve up
scne t||rgs .s .e||. +cu. J.|ve .||| t.ke scne t|ne, t|ne ycu ccu|J |.ve spert Jc|rg scnet||rg e|se. |t .||| t.ke
scne g.sc||re, .|.t ycu sperJ |c. t|e g.sc||re ccu|J |.ve beer useJ |c. scnet||rg e|se. |e J.|ve .||| .|sc ger
e..te scne .e.. .rJ te.. cr ycu. c... |.t .||| ccst ycu t|e p.|ce c| .ep.|. .rJ n.|rter.rce .rJ .eJuceJ .es.|e
v.|ue c| ycu. c... |e cppc.tur|ty ccst c| ycu. J.|ve .||| t|us |rc|uJe t|e v.|ue c| t|e best ct|e. use c| ycu. t|ne
.rJ t|e v.|ue c| t|e best ct|e. use c| t|e |urJs ycu. J.|ve .||| .e,u|.e. c n.x|n|.e ut|||ty ycu .||| .e|g| t|e be
re|ts c| t|e J.|ve .g.|rst t|e ccst c| t|e J.|ve .rJ n.x|n|.e t|e J||e.erce bet.eer t|cse bere|ts .rJ ccsts.
||s c|.pte. |rt.cJuces t|e net|cJ t|.cug| .||c| n.x|n|.|rg c|c|ces c.r be n.Je. ||s net|cJ .pp||es rct
ust tc ycu. Jec|s|cr tc t.ke . J.|ve, but .|sc tc \.|V..ts Jec|s|cr tc ||.e ext.. .c.ke.s .rJ tc |S` c.pc..t|crs
tc p.cJuce ext.. stee|. |e net|cJ .e .||| |e..r c.r be .pp||eJ tc t|e .r.|ys|s c| .ry c|c|ce, .e .||| use |t
t|.cug|cut cu. |rvest|g.t|cr c| n|c.ceccrcn|cs.
\e .||| .|sc see |c. n.x|n|.|rg c|c|ces by |rJ|v|Ju.|s .rJ by |.ns c.r |e.J tc .r .||cc.t|cr c| .escu.ces t|.t
gere..tes t|e g.e.test g.|rs pcss|b|e |c. t|e eccrcny .s . .|c|e. |r t||s .r.|ys|s, .e .||| put . re. |ten |r cu.
tcc|k|t, t|e net|cJ t|.cug| .||c| |rJ|v|Ju.|s .rJ |.ns n.x|n|.e, tcget|e. .|t| Jen.rJ .rJ supp|y .r.|ys|s, tc
see |c. t|e n..ketp|.ce c.r gu|Je .escu.ces tc t|e|. best uses.
\e .||| .|sc ex.n|re c.ses |r .||c| n.x|n|.|rg c|c|ces Jc rct gu|Je .escu.ces tc t|e|. best uses. |.t pcss|b
|||ty |s suggesteJ by .rct|e. .spect c| ycu. c|c|ce tc t.ke . J.|ve. |r .JJ|t|cr tc t|e ccsts ycu .||| ccrs|Je., t|e.e
.||| be ccsts |npcseJ cr ct|e.s. +cu. J.|ve .||| pc||ute t|e .|., sc p..t c| t|e cppc.tur|ty ccst c| t|e J.|ve .||| be
t|e v.|ue c| t|e s||g|t|y c|e.re. .|. pecp|e |r ycu. ..e. n|g|t |.ve |.J. |escu.ces suc| .s t|e .|. .e b.e.t|e .||| .|
ncst ce.t.|r|y be n|s.||cc.teJ .s t|e .esu|t c| n.x|n|.|rg c|c|ces. \e .||| see ust |c. n|s.||cc.t|cr c| .r ecc
rcnys .escu.ces c.r cccu. .rJ |c. t||s n|s.||cc.t|cr ccu|J be |xeJ.
economic prot
|e J||e.erce bet.eer tct.|
.everue .rJ tct.| ccst.
net benet
|e tct.| bere|t c| .r .ct|v|ty
n|rus |ts cppc.tur|ty ccst.
marginaI benet
|e .ncurt by .||c| .r
.JJ|t|cr.| ur|t c| .r .ct|v|ty
|rc.e.ses |ts tct.| bere|t.
marginaI cost
|e .ncurt by .||c| .r
.JJ|t|cr.| ur|t c| .r .ct|v|ty
|rc.e.ses |ts tct.| ccst.
marginaI decision ruIe
|| t|e n..g|r.| bere|t c| .r
.JJ|t|cr.| ur|t c| .r .ct|v|ty
exceeJs t|e n..g|r.| ccst,
t|e ,u.rt|ty c| t|e .ct|v|ty
s|cu|J be |rc.e.seJ. || t|e
n..g|r.| bere|t |s |ess t|.r
t|e n..g|r.| ccst, t|e
,u.rt|ty s|cu|J be .eJuceJ.
1. THE LOGIC OF MAXIMIZING BEHAVIOR
L A R N I N G O 8 1 C 7 I V S
1. xpIain the maximization assumption that economists make in expIaining the behavior of con-
sumers and rms.
2. xpIain and iIIustrate the concepts of marginaI benet and marginaI cost and appIy them to
understanding the marginaI decision ruIe.
To say that individuals maximize is to say that they pick some objective and then seek to maximize its
value. A spiintei might want to maximize his oi hei speed; a politician might want to maximize the
piobability that he oi she will win the next election. Economists pay special attention to two gioups of
maximizeis: consumeis and fims. We assume that consumeis seek to maximize utility and that fims
seek to maximize economic pront, which is the difeience between total ievenue and total cost. The
costs involved in this concept of economic pioft aie computed in the economic sense-as the oppoi-
tunity costs, oi value of the best oppoitunity foigone.
The assumption of maximizing behavioi lies at the heait of economic analysis. As we exploie its
implications, howevei, we must keep in mind the distinction between models and the ieal woild. Oui
model assumes that individuals make choices in a way that achieves a maximum value foi some cleaily
defned objective. In using such a model, economists do not assume that people actually go thiough the
calculations we will desciibe. What economists do aigue is that people's behavioi is bioadly consistent
with such a model. People may not consciously seek to maximize anything, but they behave as though
they do.
1.1 The Analysis of Maximizing Behavioi
The activities of consumeis and fims have benefts, and they also have oppoitunity costs. We assume
that given these benefts and costs, consumeis and fims will make choices that maximize the net be-
nent of each activity-the total beneft of the activity minus its oppoitunity cost. The specifc measuies
of beneft and cost vaiy with the kind of choice being made. In the case of a fim's choices in pioduc-
tion, foi example, the total beneft of pioduction is the ievenue a fim ieceives fiom selling the pioduct;
the total cost is the oppoitunity cost the fim incuis by pioducing it. The net beneft is thus total ieven-
ue minus total oppoitunity cost, oi economic pioft.
Economists maintain that in oidei to maximize net beneft, consumeis and fims evaluate each
activity at the maigin-they considei the additional beneft and the additional cost of anothei unit of
the activity. Should you supeisize" youi oidei at McDonald's: Will the additional beveiage and the
additional fiench fiies be woith the extia cost: Should a fim hiie one moie woikei: Will the benefts
to the fim of hiiing this woikei be woith the additional cost of hiiing him oi hei:
The marginal benent is the amount by which an additional unit of an activity incieases its total
beneft. It is the amount by which the extia fiench fiies inciease youi satisfaction, oi the extia ievenue
the fim expects to biing in by hiiing anothei woikei. The marginal cost is the amount by which an
additional unit of an activity incieases its total cost. You will pay moie to supeisize youi McDonald's
oidei; the fim's laboi costs will iise when it hiies anothei woikei.
To deteimine the quantity of any activity that will maximize its net beneft, we apply the marginal
decision rule: If the maiginal beneft of an additional unit of an activity exceeds the maiginal cost, the
quantity of the activity should be incieased. If the maiginal beneft is less than the maiginal cost, the
quantity should be ieduced. Net beneft is maximized at the point at which maiginal beneft equals
maiginal cost. The maiginal decision iule is at the heait of the economic way of thinking. The iule ba-
sically says this: If the additional beneft of one moie unit exceeds the extia cost, do it; if not, do not.
This simple logic gives us a poweiful tool foi the analysis of choice. Peihaps moie than any othei iule
in economic analysis, the maiginal decision iule typifes the way in which economists analyze piob-
lems. We shall apply it in eveiy chaptei that follows in the micioeconomics poition of this text.
140 PRINCIPLS OI CONOMICS
constraint
A bcurJ..y t|.t ||n|ts t|e
..rge c| c|c|ces t|.t c.r be
n.Je.
Maximizing choices must be made within the paiameteis imposed by some constraint, which is a
boundaiy that limits the iange of choices that can be made. We assume that a consumei seeks the
gieatest satisfaction possible within the limits of his oi hei income oi budget. A fim cannot pioduce
beyond the limits of its pioduction capacity at a point in time.
The maiginal decision iule foims the foundation foi the stiuctuie economists use to analyze all
choices. At fist glance, it may seem that a consumei seeking satisfaction fiom, say, pizza has little in
common with an entiepieneui seeking pioft fiom the pioduction of custom-designed semiconduct-
ois. But maximizing choices always follow the maiginal decision iule-and that iule holds iegaidless of
what is being maximized oi who is doing the maximizing.
To see how the logic of maximizing choices woiks, we will examine a specifc pioblem. We will
then extend that pioblem to the geneial analysis of maximizing choices.
A ProbIem in Maximization
Suppose a college student, Lauiie Phan, faces two midteims tomoiiow, one in economics and anothei
in accounting. She has alieady decided to spend 3 houis studying foi the two examinations. This de-
cision imposes a constiaint on the pioblem. Suppose that Ms. Phan's goal is to allocate hei 3 houis of
study so that she incieases hei total scoie foi the two exams by as much as possible.
Ms. Phan expects the ielationship between the time she spends studying foi the economics exam
and the total gain in hei scoie to be as given by the second iow of the table in Panel (a) of Figuie 6.1.
We inteipiet the expected total gain in hei scoie as the total beneft of study. She expects that 1 houi of
study will iaise hei scoie by 18 points; 2 houis will iaise it by 32 points, and so on. These values aie
plotted in Panel (b). Notice that the total beneft cuive iises, but by smallei and smallei amounts, as she
studies moie and moie. The slope of the cuive, which in this case tells us the iate at which hei expected
scoie iises with incieased study time, falls as we tiavel up and to the iight along the cuive.
CHAP7R 6 MARk7S, MAXIMI7RS, AND IIICINCY 141
II GUR 6. 1 7he 8enefits of Studying conomics
|e t.b|e |r |.re| (.) s|c.s t|e tct.| bere|t .rJ n..g|r.| bere|t c| t|e t|ne |.u.|e ||.r sperJs stuJy|rg |c. |e.
eccrcn|cs ex.n. |.re| (b) s|c.s t|e tct.| bere|t cu.ve. |.re| (c) s|c.s t|e n..g|r.| bere|t cu.ve, .||c| |s g|ver
by t|e s|cpe c| t|e tct.| bere|t cu.ve |r |.re| (b).
142 PRINCIPLS OI CONOMICS
II GUR 6. 2 7he MarginaI 8enefits of
Studying Accounting
|e n..g|r.| bere|t |.u.|e ||.r expects |.cn
stuJy|rg |c. |e. .cccurt|rg ex.n |s s|c.r by
t|e n..g|r.| bere|t cu.ve. |e |.st |cu. c|
stuJy |rc.e.ses |e. expecteJ scc.e by 14
pc|rts, t|e seccrJ |cu. by 10 pc|rts, t|e t||.J
by 6 pc|rts, .rJ sc cr.
Now look at the thiid iow in the table in Panel (a). It tells us the amount by which each additional houi
of study incieases hei expected scoie; it gives the maiginal beneft of studying foi the economics exam.
Maiginal beneft equals the amount by which total beneft iises with each additional houi of study. Be-
cause these maiginal benefts aie given by the changes in total benefts fiom additional houis of study,
they equal the slope of the total beneft cuive. We see this in the ielationship between Panels (b) and (c)
of Figuie 6.1. The decieasing slope of the total beneft cuive in Panel (b) gives us the downwaid-slop-
ing maiginal beneft cuive in Panel (c).
The maiginal beneft cuive tells us what happens when we pass fiom one point to anothei on the
total beneft cuive, so we have plotted maiginal benefts at the midpoints of the houily inteivals in
Panel (c). Foi example, the total beneft cuive in Panel (b) tells us that, when Ms. Phan incieases hei
time studying foi the economics exam fiom 2 houis to 3 houis, hei total beneft iises fiom 32 points to
42 points. The inciease of 10 points is the maiginal beneft of incieasing study time foi the economics
exam fiom 2 houis to 3 houis. We maik the point foi a maiginal beneft of 10 points midway between
2 and 3 houis. Because maiginal values tell us what happens as we pass fiom one quantity to the next,
we shall always plot them at the midpoints of inteivals of the vaiiable on the hoiizontal axis.
We can peifoim the same kind of analysis to obtain the maiginal beneft cuive foi studying foi the
accounting exam. Figuie 6.2 piesents this cuive. Like the maiginal beneft cuive foi studying econom-
ics, it slopes downwaid. Once again, we have plotted maiginal values at the midpoints of the inteivals.
Incieasing study time in accounting fiom 0 to 1 houi incieases Ms. Phan's expected accounting scoie
by 14 points.
Ms. Phan's maiginal beneft cuives foi studying typify a geneial phenomenon in
economics. Maiginal beneft cuives foi viitually all activities, including the activities of
consumeis and of fims, slope downwaid. Think about youi own expeiience with
studying. On a given day, the fist houi spent studying a ceitain subject piobably genei-
ates a gieatei maiginal beneft than the second, and the second houi piobably geneiates
a gieatei maiginal beneft than the thiid. You may ieach a point at which an extia houi
of study is unlikely to yield any beneft at all. Of couise, oui example of Lauiie Phan's
expected exam scoies is a highly stylized one. One could haidly expect a student to
have a piecise set of numbeis to guide him oi hei in allocating study time. But it is cei-
tainly the case that students have a iough idea of the likely payof of study time in
difeient subjects. If you weie faced with exams in two subjects, it is likely that you
would set aside a ceitain amount of study time, just as Ms. Phan did in oui example.
And it is likely that youi own expeiience would seive as a guide in deteimining how to
allocate that time. Economists do not assume that people have numeiical scales in theii
heads with which to diaw maiginal beneft and maiginal cost cuives. They meiely as-
sume that people act as if they did.
The natuie of maiginal benefts can change with difeient applications. Foi a ies-
tauiant, the maiginal beneft of seiving one moie meal can be defned as the ievenue
that meal pioduces. Foi a consumei, the maiginal beneft of one moie slice of pizza can
be consideied in teims of the additional satisfaction the pizza will cieate. But whatevei
the natuie of the beneft, maiginal benefts geneially fall as quantities inciease.
Ms. Phan's falling maiginal beneft fiom houis spent studying accounting has spe-
cial signifcance foi oui analysis of hei choice conceining how many houis to devote to
economics. In oui pioblem, she had decided to devote 3 houis to studying the two sub-
jects. That means that the oppoitunity cost of an houi spent studying economics equals
the beneft she would have gotten spending that houi studying accounting.
Suppose, foi example, that she weie to considei spending all 3 houis studying ac-
counting. The maiginal beneft cuive foi studying foi hei accounting exam tells us that
she expects that the ffth houi will add nothing to hei scoie. Shifting that houi to economics would cost
nothing. We can say that the maiginal cost of the fist houi spent studying economics is zeio. We ob-
tained this value fiom the maiginal beneft cuive foi studying accounting in Figuie 6.2.
Similaily, we can fnd the maiginal cost of the second houi studying economics. That iequiies giv-
ing up the fouith houi spent on accounting. Figuie 6.2 tells us that the maiginal beneft of that houi
equals 2-that is the maiginal cost of spending the second houi studying economics.
Figuie 6.3 shows the maiginal cost cuive of studying economics. We see that at fist, time devoted
to studying economics has a low maiginal cost. As time spent studying economics incieases, howevei,
it iequiies hei to give up study time in accounting that she expects will be moie and moie pioductive.
The maiginal cost cuive foi studying economics can thus be deiived fiom the maiginal beneft cuive
foi studying accounting. Figuie 6.3 also shows the maiginal beneft cuive foi studying economics that
we deiived in Panel (b) of Figuie 6.1.
CHAP7R 6 MARk7S, MAXIMI7RS, AND IIICINCY 143
II GUR 6. 3 7he MarginaI 8enefits and
MarginaI Costs of Studying conomics
|e n..g|r.| bere|t cu.ve |.cn |.re| (c) c|
||gu.e 6.1 |s s|c.r tcget|e. .|t| t|e n..g|r.|
ccsts c| stuJy|rg eccrcn|cs. |e n..g|r.|
ccst cu.ve |s Je.|veJ |.cn t|e n..g|r.| bere|t
cu.ve |c. stuJy|rg .cccurt|rg s|c.r |r ||gu.e
6.2.
Just as maiginal beneft cuives geneially slope downwaid, maiginal cost cuives
geneially slope upwaid, as does the one in Figuie 6.3. In the case of allocating time, the
phenomenon of iising maiginal cost iesults fiom the simple fact that, the moie time a
peison devotes to one activity, the less time is available foi anothei. And the moie one
ieduces the second activity, the gieatei the foigone maiginal benefts aie likely to be.
That means the maiginal cost cuive foi that fist activity iises.
Because we now have maiginal beneft and maiginal cost cuives foi studying eco-
nomics, we can apply the maiginal decision iule. This iule says that, to maximize the
net beneft of an activity, a decision makei should inciease an activity up to the point at
which maiginal beneft equals maiginal cost. That occuis wheie the maiginal beneft
and maiginal cost cuives inteisect, with 3 houis spent studying economics and 2 houis
spent studying accounting.
Using MarginaI 8enefit and MarginaI Cost Curves to Iind Net 8enefits
We can use maiginal beneft and maiginal cost cuives to show the total beneft, the
total cost, and the net beneft of an activity. We will see that equating maiginal beneft
to maiginal cost does, indeed, maximize net beneft. We will also develop anothei tool
to use in inteipieting maiginal beneft and cost cuives.
Panel (a) of Figuie 6.4 shows the maiginal beneft cuive we deiived in Panel (c) of
Figuie 6.1. The coiiesponding point on the maiginal beneft cuive gives the maiginal
beneft of the fist houi of study foi the economics exam, 18 points. This same value
equals the aiea of the iectangle bounded by 0 and 1 houi of study and the maiginal be-
neft of 18. Similaily, the maiginal beneft of the second houi, 14 points, is shown by
the coiiesponding point on the maiginal beneft cuive and by the aiea of the shaded
iectangle bounded by 1 and 2 houis of study. The total beneft of 2 houis of study
equals the sum of the aieas of the fist two iectangles, 32 points. We continue this pio-
ceduie thiough the ffth houi of studying economics; the aieas foi each of the shaded
iectangles aie shown in the giaph.
II GUR 6. 4 7he 8enefits and Costs of Studying conomics
|.re| (.) s|c.s t|e n..g|r.| bere|t cu.ve c| ||gu.e 6.1. |e tct.| bere|t c| stuJy|rg eccrcn|cs .t .ry g|ver
,u.rt|ty c| stuJy t|ne |s g|ver .pp.cx|n.te|y by t|e s|.JeJ ..e. be|c. t|e n..g|r.| bere|t cu.ve up tc t|.t |eve|
c| stuJy. |.re| (b) s|c.s t|e n..g|r.| ccst cu.ve |.cn ||gu.e 6.3. |e tct.| ccst c| stuJy|rg eccrcn|cs .t .ry g|ver
,u.rt|ty c| stuJy |s g|ver .pp.cx|n.te|y by t|e s|.JeJ ..e. be|c. t|e n..g|r.| ccst cu.ve up tc t|.t |eve| c| stuJy.
Two featuies of the cuive in Panel (a) of Figuie 6.4 aie paiticulaily impoitant. Fiist, note that the sum
of the aieas of the fve iectangles, 30 points, equals the total beneft of 3 houis of study given in the
table in Panel (a) of Figuie 6.1. Second, notice that the shaded aieas aie appioximately equal to the aiea
undei the maiginal beneft cuive between 0 and 3 houis of study. We can pick any quantity of study
time, and the total beneft of that quantity equals the sum of the shaded iectangles between zeio and
that quantity. Thus, the total beneft of 2 houis of study equals 32 points, the sum of the aieas of the
fist two iectangles.
144 PRINCIPLS OI CONOMICS
deadweight Ioss
|e |css |r ret bere|ts
.esu|t|rg |.cn . |.||u.e tc
c...y cut .r .ct|v|ty .t t|e
ncst e|c|ert |eve|.
Now considei the maiginal cost cuive in Panel (b) of Figuie 6.4. The aieas of the shaded iectangles
equal the values of maiginal cost. The maiginal cost of the fist houi of study equals zeio; theie is thus
no iectangle undei the cuive. The maiginal cost of the second houi of study equals 2 points; that is the
aiea of the iectangle bounded by 1 and 2 houis of study and a maiginal cost of 2. The maiginal cost of
the thiid houi of study is 6 points; this is the aiea of the shaded iectangle bounded by 2 and 3 houis of
study and a maiginal cost of 6.
Looking at the iectangles in Panel (b) ovei the iange of 0 to 3 houis of study, we see that the aieas
of the fve iectangles total 32, the total cost of spending all 3 houis studying economics. And looking at
the iectangles, we see that theii aiea is appioximately equal to the aiea undei the maiginal cost cuive
between 0 and 3 houis of study.
We have seen that the aieas of the iectangles diawn with Lauiie Phan's maiginal beneft and mai-
ginal cost cuives equal the total beneft and total cost of studying economics. We have also seen that
these aieas aie ioughly equal to the aieas undei the cuives themselves. We can make this last statement
much stiongei. Suppose, instead of thinking in inteivals of whole houis, we think in teims of smallei
inteivals, say, of 12 minutes. Then each iectangle would be only one-ffth as wide as the iectangles we
diew in Figuie 6.4. Theii aieas would still equal the total beneft and total cost of study, and the sum of
those aieas would be closei to the aiea undei the cuives. We have done this foi Ms. Phan's maiginal
beneft cuive in Figuie 6.3; notice that the aieas of the iectangles closely appioximate the aiea undei
the cuive. They still stick out" fiom eithei side of the cuive as did the iectangles we diew in Figuie 6.4,
but you almost need a magnifying glass to see that. The smallei the inteival we choose, the closei the
aieas undei the maiginal beneft and maiginal cost cuives will be to total beneft and total cost. Foi
puiposes of oui model, we can imagine that the inteivals aie as small as we like. Ovei a paiticulai iange
of quantity, the aiea undei a maiginal beneft cuive equals the total beneft of that quantity, and the
aiea undei the maiginal cost cuive equals the total cost of that quantity.
II GUR 6. 5 7he MarginaI 8enefit Curve and 7otaI 8enefit
\|er t|e |rc.enerts useJ tc ne.su.e t|ne .||cc.teJ tc stuJy|rg eccrcn|cs ..e n.Je sn.||e., |r t||s c.se 12
n|rutes |rste.J c| .|c|e |cu.s, t|e ..e. urJe. t|e n..g|r.| bere|t cu.ve |s c|cse. tc t|e tct.| bere|t c| stuJy|rg
t|.t .ncurt c| t|ne.
Panel (a) of Figuie 6.6 shows maiginal beneft and maiginal cost cuives foi studying economics, this
time without numbeis. We have the usual downwaid-sloping maiginal beneft cuive and upwaid-slop-
ing maiginal cost cuive. The maiginal decision iule tells us to choose D houis studying economics, the
quantity at which maiginal beneft equals maiginal cost at point C. We know that the total beneft of
study equals the aiea undei the maiginal beneft cuive ovei the iange fiom A to D houis of study, the
aiea ABCD. Total cost equals the aiea undei the maiginal cost cuive ovei the same iange, oi ACD. The
difeience between total beneft and total cost equals the aiea between maiginal beneft and maiginal
CHAP7R 6 MARk7S, MAXIMI7RS, AND IIICINCY 145
cost between A and D houis of study; it is the gieen-shaded tiiangle ABC. This difeience is the net be-
neft of time spent studying economics. Panel (b) of Figuie 6.6 intioduces anothei impoitant concept.
If an activity is caiiied out at a level less than the emcient level, then net benefts aie foigone. The loss
in net benefts iesulting fiom a failuie to caiiy out an activity at the emcient level is called a dead-
weight loss.
II GUR 6. 6 Using MarginaI 8enefit and MarginaI Cost Curves to Determine Net 8enefit
|r |.re| (.) ret bere|ts ..e g|ver by t|e J||e.erce bet.eer tct.| bere|ts (.s ne.su.eJ by t|e ..e. urJe. t|e
n..g|r.| bere|t cu.ve up tc .ry g|ver |eve| c| .ct|v|ty) .rJ tct.| ccsts (.s ne.su.eJ by t|e ..e. urJe. t|e n..g|r.|
ccst cu.ve up tc .ry g|ver |eve| c| .ct|v|ty). V.x|nun ret bere|ts ..e |curJ .|e.e t|e n..g|r.| bere|t cu.ve
|rte.sects t|e n..g|r.| ccst cu.ve .t .ct|v|ty |eve| |. |.re| (b) s|c.s t|.t || t|e |eve| c| t|e .ct|v|ty |s .est.|cteJ tc
.ct|v|ty |eve| |, ret bere|ts ..e .eJuceJ |.cn t|e ||g|tg.eer s|.JeJ t.|.rg|e Ab |r |.re| (.) tc t|e sn.||e. ..e.
AbC|. |e |c.gcre ret bere|ts, c. Je.J.e|g|t |css, |s g|ver by t|e pu.p|es|.JeJ ..e. |C. || t|e .ct|v|ty |eve| |s
|rc.e.seJ |.cn | tc ', .s s|c.r |r |.re| (c), ret bere|ts Jec||reJ by t|e Je.J.e|g|t |css ne.su.eJ by t|e ..e.
||.
Now suppose a peison incieases study time fiom D to J houis as shown in Panel (c). The aiea undei
the maiginal cost cuive between D and J gives the total cost of incieasing study time; it is DCHJ. The
total beneft of incieasing study time equals the aiea undei the maiginal beneft cuive between D and J;
it is DCL. The cost of incieasing study time in economics fiom D houis to J houis exceeds the beneft.
This gives us a deadweight loss of CHI. The net beneft of spending J houis studying economics equals
the net beneft of studying foi D houis less the deadweight loss, oi ABC minus CHI. Only by studying
up to the point at which maiginal beneft equals maiginal cost do we achieve the maximum net beneft
shown in Panel (a).
We can apply the maiginal decision iule to the pioblem in Figuie 6.6 in anothei way. In Panel (b),
a peison studies economics foi E houis. Reading up to the maiginal beneft cuive, we ieach point G.
Reading up to the maiginal cost cuive, we ieach point F. Maiginal beneft at G exceeds maiginal cost at
F; the maiginal decision iule says economics study should be incieased, which would take us towaid
the inteisection of the maiginal beneft and maiginal cost cuives. Spending J houis studying
146 PRINCIPLS OI CONOMICS
economics, as shown in Panel (c), is too much. Reading up to the maiginal beneft and maiginal cost
cuives, we see that maiginal cost exceeds maiginal beneft, suggesting that study time be ieduced.
This completes oui intioduction to the maiginal decision iule and the use of maiginal beneft and
maiginal cost cuives. We will spend the iemaindei of the chaptei applying the model.
Heads Up!
|t |s e.sy tc n.ke t|e n|st.ke c| .ssun|rg t|.t || .r .ct|v|ty |s c...|eJ cut up tc t|e pc|rt .|e.e n..g|r.| be
re|t e,u.|s n..g|r.| ccst, t|er ret bere|ts nust be .e.c. |enenbe. t|.t |c||c.|rg t|e n..g|r.| Jec|s|cr
.u|e .rJ e,u.t|rg n..g|r.| bere|ts .rJ ccsts o||e ret bere|ts. |t n.kes t|e J||eece bet.eer tct.|
bere|ts .rJ tct.| ccst .s |..ge .s pcss|b|e.
k Y 7 A k A W A Y S
< |ccrcn|sts .ssune t|.t Jec|s|cr n.ke.s n.ke c|c|ces |r t|e ..y t|.t n.x|n|.es t|e v.|ue c| scne
cbect|ve.
< V.x|n|..t|cr |rvc|ves Jete.n|r|rg t|e c|.rge |r tct.| bere|t .rJ t|e c|.rge |r tct.| ccst .sscc|.teJ
.|t| e.c| ur|t c| .r .ct|v|ty. |ese c|.rges ..e c.||eJ n..g|r.| bere|t .rJ n..g|r.| ccst, .espect|ve|y.
< || t|e n..g|r.| bere|t c| .r .ct|v|ty exceeJs t|e n..g|r.| ccst, t|e Jec|s|cr n.ke. .||| g.|r by |rc.e.s|rg
t|e .ct|v|ty.
< || t|e n..g|r.| ccst c| .r .ct|v|ty exceeJs t|e n..g|r.| bere|t, t|e Jec|s|cr n.ke. .||| g.|r by .eJuc|rg
t|e .ct|v|ty.
< |e ..e. urJe. t|e n..g|r.| bere|t cu.ve |c. .r .ct|v|ty g|ves |ts tct.| bere|t, t|e ..e. urJe. t|e n..g|r.|
ccst cu.ve g|ves t|e .ct|v|tys tct.| ccst. |et bere|t e,u.|s tct.| bere|t |ess tct.| ccst.
< |e n..g|r.| bere|t .u|e te||s us t|.t .e c.r n.x|n|.e t|e ret bere|t c| .ry .ct|v|ty by c|ccs|rg t|e
,u.rt|ty .t .||c| n..g|r.| bere|t e,u.|s n..g|r.| ccst. At t||s ,u.rt|ty, t|e ret bere|t c| t|e .ct|v|ty |s
n.x|n|.eJ.
7 R Y I 7 !
Suppcse Vs. ||.r st||| |.ces t|e ex.ns |r eccrcn|cs .rJ |r .cccurt|rg, .rJ s|e st||| p|.rs tc sperJ . tct.| c|
5 |cu.s stuJy|rg |c. t|e t.c ex.ns. |c.eve., s|e .ev|ses |e. expect.t|crs .bcut t|e Jeg.ee tc .||c| stuJy
|rg eccrcn|cs .rJ .cccurt|rg .||| .|ect |e. scc.es cr t|e t.c ex.ns. S|e expects stuJy|rg eccrcn|cs .|||
.JJ scne.|.t |ess tc |e. scc.e, .rJ s|e expects stuJy|rg .cccurt|rg .||| .JJ nc.e. |e .esu|t |s t|e t.b|e
be|c. c| expecteJ tct.| bere|ts .rJ tct.| ccsts c| |cu.s spert stuJy|rg eccrcn|cs. |ct|ce t|.t seve..| v.|ues
|r t|e t.b|e |.ve beer cn|tteJ. |||| |r t|e n|ss|rg v.|ues |r t|e t.b|e. |c. n.ry |cu.s c| stuJy s|cu|J Vs.
||.r Jevcte tc eccrcn|cs tc n.x|n|.e |e. ret bere|t`
|cu.s stuJy|rg eccrcn|cs 0 1 2 3 4 5
ct.| bere|t 0 14 24 30 32
ct.| ccst 0 2 8 32 50
|et bere|t 0 12 12 0 18
|c. ccnpute t|e n..g|r.| bere|ts .rJ ccsts c| |cu.s JevcteJ tc stuJy|rg eccrcn|cs, ccnp|et|rg t|e t.b|e
be|c..
|... t|e n..g|r.| bere|t .rJ n..g|r.| ccst cu.ves |c. stuJy|rg eccrcn|cs (.enenbe. tc p|ct n..g|r.|
v.|ues .t t|e n|Jpc|rts c| t|e .espect|ve |cu.|y |rte.v.|s). |c ycu. cu.ves |rte.sect .t t|e .|g|t runbe. c|
|cu.s c| stuJyt|e runbe. t|.t n.x|n|.es t|e ret bere|t c| stuJy|rg eccrcn|cs`
CHAP7R 6 MARk7S, MAXIMI7RS, AND IIICINCY 147
Case in Point: Preventing OiI SpiIIs
2010 jupiterimages Corporation
|c .e sp||| ercug| c|| |r cu. cce.rs .rJ ..te...ys` |t |s . ,uest|cr t|.t pe.|.ps cr|y eccrcn|sts .cu|J
.sk.rJ, .s eccrcn|sts, .e s|cu|J .sk |t.
|e.e |s, c| ccu.se, rc v|.tue |r .r c|| sp|||. |t Jest.cys .||J|||e .rJ |cu|s s|c.e||res. |e.rup ccsts c.r be t.e
nerJcus. |c.eve., |.evert|rg c|| Sp|||s |.s ccsts .s .e||. g.e.te. er|c.cenert experJ|tu.es .rJ ||g|e. ccsts
tc s||ppe.s c| c|| .rJ, t|e.e|c.e, ||g|e. ccsts c| gccJs suc| .s g.sc||re tc custcne.s. |e cr|y ..y tc p.evert
c|| sp|||s ccnp|ete|y |s tc stcp s||pp|rg c||. |.t |s . ccst |e. pecp|e .cu|J .ccept. but .|.t |s t|e .|g|t b.|
.rce bet.eer erv|.crnert.| p.ctect|cr .rJ t|e s.t|s|.ct|cr c| ccrsune. Jen.rJ |c. c||`
\.rJe.b||t |r|ve.s|ty eccrcn|st V..k c|er ex.n|reJ t|e |.S. c.st Cu..Js e|c.ts tc .eJuce c|| sp|||s
t|.cug| |ts er|c.cenert c| s||pp|rg .egu|.t|crs |r cc.st.| ..te.s .rJ cr .|ve.s. |e |ccuseJ cr t|e ccsts .rJ
bere|ts .esu|t|rg |.cn t|e c.st Cu..Js er|c.cenert e|c.ts |r 1981. Or t|e b.s|s c| t|e |.e,uercy c| c||
sp|||s be|c.e t|e c.st Cu..J beg.r |ts er|c.cenert, V.. c|er est|n.teJ t|.t t|e c.st Cu..J p.everteJ
1,159,352 g.||crs c| c|| |.cn be|rg sp|||eJ |r 1981.
C|ver t|.t t|e.e ..s . tct.| c| 824,921 g.||crs c| c|| .ctu.||y sp|||eJ |r 1981, s|cu|J t|e c.st Cu..J |.ve .t
tenpteJ tc p.evert ever nc.e sp|||.ge` V.. c|er est|n.teJ t|.t t|e n..g|r.| bere|t c| p.evert|rg cre
nc.e g.||cr |.cn be|rg sp|||eJ ..s .2 (3.42 |r c|e.rup ccsts, 3 |ess |r erv|.crnert.| J.n.ge, .rJ 0.85
.c.t| c| c|| s.veJ). |e n..g|r.| ccst c| p.evert|rg cre nc.e g.||cr |.cn be|rg sp|||eJ ..s 5.50. V.. c|er
suggests t|.t bec.use t|e n..g|r.| bere|t c| nc.e v|gc.cus er|c.cenert exceeJeJ t|e n..g|r.| ccst, nc.e
v|gc.cus c.st Cu..J e|c.ts .cu|J |.ve beer ust||eJ.
Vc.e v|gc.cus e|c.ts |.ve, |rJeeJ, beer pu.sueJ. |r 1989, t|e |xxcr c|| t.rke. |xxcr \.|Je. ..r .g.curJ,
sp||||rg 10.8 n||||cr g.||crs c| c|| c| t|e cc.st c| A|.sk.. |e sp||| J.n.geJ t|e s|c.e||re c| . r.t|cr.| |c.est,
|cu. r.t|cr.| .||J|||e .e|uges, t|.ee r.t|cr.| p..ks, |ve st.te p..ks, |cu. c.|t|c.| |.b|t.t ..e.s, .rJ . st.te g.ne
.e|uge. |xxcr ..s c.Je.eJ tc p.y 900 n||||cr |r J.n.ges, . |eJe..| u.y |curJ |xxcr .rJ t|e c.pt.|r gu||ty c|
c.|n|r.| reg||gerce .rJ |npcseJ .r .JJ|t|cr.| 5 b||||cr |r pur|t|ve J.n.ges. |r 2008, |e Sup.ene cu.t .e
JuceJ t|e .ssessnert c| pur|t|ve J.n.ges tc 50 n||||cr, .|t| t|e n.c.|ty ..gu|rg t|.t t|e c.|g|r.| |gu.e
..s tcc ||g| |r ccnp..|scr tc t|e ccnpers.tc.y J.n.ges |c. . c.se |r .||c| t|e .ct|crs c| t|e Je|erJ.rt,
|xxcr, .e.e .ep.e|ers|b|e but rct |rtert|cr.|.
|e.|.ps t|e ncst |npc.t.rt |np.ct c| t|e |xxcr \.|Je. J|s.ste. ..s t|e p.ss.ge c| t|e O|| |c||ut|cr Act c|
1990. |t |rc.e.seJ s||ppe. ||.b|||ty |.cn 14 n||||cr tc 100 n||||cr. |t .|sc .e,u|.eJ Jcub|e|u||eJ t.rke.s |c.
s||pp|rg c||.
|e |u.cpe.r |r|cr (||) |.s .|sc st.ergt|ereJ |ts st.rJ..Js |c. c|| t.rke.s. |e 2002 b.e.kup c| t|e c|| t.rke.
|.est|ge c| t|e cc.st c| Sp.|r .esu|teJ |r t|e sp|||.ge c| 3.2 n||||cr g.||crs c| c||. |e || |.J p|.rreJ tc b.r
s|rg|e|u||eJ t.rke.s, p|.s|rg |r t|e b.r bet.eer 2003 .rJ 2015. |e s|rk|rg c| t|e |.est|ge |eJ t|e || tc
ncve up t|.t Je.J||re.
148 PRINCIPLS OI CONOMICS
Sp||| c.|ses |.ve |eJ bct| t|e |r|teJ St.tes .rJ t|e |u.cpe.r |r|cr tc t|g|ter up t|e|. .egu|.t|crs c| c||
t.rke.s. |e .esu|t |.s beer . .eJuct|cr |r t|e ,u.rt|ty c| c|| sp|||eJ, .||c| ..s p.ec|se|y .|.t eccrcn|c .e
se..c| |.J ccrc|uJeJ ..s reeJeJ. by 2002, t|e .ncurt c| c|| sp|||eJ pe. b...e| s||ppeJ |.J |.||er 30 |.cn
t|e |eve| t|.ee Jec.Jes e..||e..
.ooce /o| ^ o|e. |e o| oJ |ee|| o| || .||| |e.e||o oJ ||ocee|.' .ooo| o| |.|oe|o| |coo|c oJ /ooee| 3(2:
(.oe 9S6: 6.SS. ||c| . |o|. oo|o| || |o||o||o .|||. ||. oJ |o||c, |oe.' |e.|e. o| |o||c, |eeoc|. 2(2: (/oc| 2004: 2029.
|o.|J . .o.oe. .o||ce .|o| |o vo|Je veJ|c|.' |o ^e|e |e. .oe 26. 200S. ^. oJ |J.| |.o||. |ooe Oe| oo|e o ^| ||
o|e.' |o|e |ooce. 36(4S: (|ecee 2. 2002: 3334
A N S W R 7 O 7 R Y I 7 ! P R O 8 L M
|e.e ..e t|e ccnp|eteJ J.t. t.b|e .rJ t|e t.b|e s|c.|rg tct.| .rJ n..g|r.| bere|t .rJ ccst.
Vs. ||.r n.x|n|.es |e. ret bere|t by .eJuc|rg |e. t|ne stuJy|rg eccrcn|cs tc 2 |cu.s. |e c|.rge |r
|e. expect.t|crs .eJuceJ t|e bere|t .rJ |rc.e.seJ t|e ccst c| stuJy|rg eccrcn|cs. |e ccnp|eteJ g..p| c|
n..g|r.| bere|t .rJ n..g|r.| ccst |s .t t|e |.. |e|t. |ct|ce t|.t .rs.e.|rg t|e ,uest|cr us|rg t|e n..g|r.| Je
c|s|cr .u|e g|ves t|e s.ne .rs.e..
2. MAXIMIZING IN THE MARKETPLACE
L A R N I N G O 8 1 C 7 I V S
1. xpIain what is meant by an emcient aIIocation of resources in an economy and describe the
market conditions that must exist to achieve this goaI.
2. Dene consumer and producer surpIus.
3. Discuss the reIationship between emciency and equity.
In peihaps the most infuential book in economics evei wiitten, An Inquiry into the ^ature and Causes
of the Vealth of ^ations, published in 1776, Adam Smith aigued that the puisuit of self-inteiest in a
maiketplace would piomote the geneial inteiest. He said iesouices would be guided, as if by an
invisible hand," to theii best uses. That invisible hand was the maiketplace.
CHAP7R 6 MARk7S, MAXIMI7RS, AND IIICINCY 149
emcient
|e .||cc.t|cr c| .escu.ces
.|er t|e ret bere|ts c| .||
eccrcn|c .ct|v|t|es ..e
n.x|n|.eJ.
property rights
A set c| .u|es t|.t spec||y t|e
..ys |r .||c| .r c.re. c.r
use . .escu.ce.
excIusive property right
A p.cpe.ty .|g|t t|.t .||c.s
|ts c.re. tc p.evert ct|e.s
|.cn us|rg t|e .escu.ce.
transferabIe property right
A p.cpe.ty .|g|t t|.t .||c.s
t|e c.re. c| . .escu.ce tc
se|| c. |e.se |t tc scnecre
e|se.
Smith's idea was iadical foi its time; he saw that the seemingly haphazaid woikings of the maiket-
place could piomote the common good. In this section, we will use the tools we have developed thus
fai to see the powei of Smith's invisible hand. Efoits by individuals to maximize theii own net beneft
can maximize net beneft foi the economy as a whole.
When the net benefts of all economic activities aie maximized, economists say the allocation of
iesouices is emcient. This concept of emciency is bioadei than the notion of emcient pioduction that
we encounteied when discussing the pioduction possibilities cuive. Theie, we saw that the economy's
factois of pioduction would be emcient in production if they weie allocated accoiding to the piinciple
of compaiative advantage. That meant pioducing as much as possible with the factois of pioduction
available. The concept of an emcient allocation of iesouices incoipoiates pioduction, as in that discus-
sion, but it includes emciency in the consumption of goods and seivices as well.
2.1 Achieving Efficiency
Imagine youiself aiiiving at the stoie to puichase some food. In youi choice, you will weigh youi own
benefts and costs to maximize youi net beneft. The faimeis, the distiibutois, and the giocei have
sought to maximize theii net benefts as well. How can we expect that all those efoits will maximize
net benefts foi the economy as a whole: How can we expect the maiketplace to achieve an emcient al-
location of food, oi of anything else:
One condition that must be met if the maiket's allocation is to be emcient is that the maiketplace
must be competitive oi function as if it weie. We will have a gieat deal moie to say about competitive
maikets veisus less competitive ones in subsequent chapteis. Foi now, we can simply note that a com-
petitive maiket is one with many buyeis and selleis in each maiket and in which entiy and exit aie
faiily easy. No one contiols the piice; the foices of demand and supply deteimine piice.
The second condition that must hold if the maiket is to achieve an emcient allocation conceins
piopeity iights. We tuin to that topic in the next section.
7he RoIe of Property Rights
A smoothly functioning maiket iequiies that pioduceis possess piopeity iights to the goods and sei-
vices they pioduce and that consumeis possess piopeity iights to the goods and seivices they buy.
Property rights aie a set of iules that specify the ways in which an ownei can use a iesouice.
Considei the tomato maiket. Faimeis who giow tomatoes have cleaily defned iights to theii land
and to the tomatoes they pioduce and sell. Distiibutois who puichase tomatoes fiom faimeis and sell
them to gioceis have cleai iights to the tomatoes until they sell them to gioceis. The gioceis who pui-
chase the tomatoes ietain iights to them until they sell them to consumeis. When you buy a tomato,
you have the exclusive iight to its use.
A system of piopeity iights foims the basis foi all maiket exchange. Befoie exchange can begin,
theie must be a cleai specifcation of who owns what. The system of piopeity iights must also show
what puichaseis aie acquiiing when they buy iights to paiticulai iesouices. Because piopeity iights
must exist if exchange is to occui, and because exchange is the piocess thiough which economic em-
ciency is achieved, a system of piopeity iights is essential to the emcient allocation of iesouices.
Imagine what would happen in the maiket foi tomatoes if piopeity iights weie not cleaily defned.
Suppose, foi example, that gioceis could not legally pievent someone fiom simply giabbing some to-
matoes and leaving without paying foi them. If that weie the case, gioceis would not be likely to ofei
tomatoes foi sale. If it weie the case foi all gioceiy items, theie would not be gioceiy stoies at all.
Although piopeity iights vaiy foi difeient iesouices, two chaiacteiistics aie iequiied if the mai-
ketplace is to achieve an emcient allocation of iesouices:
1. Piopeity iights must be exclusive. An exclusive property right is one that allows its ownei to
pievent otheis fiom using the iesouice. The ownei of a house, foi example, has the iight to
exclude otheis fiom the use of the house. If this iight did not exist, owneiship would have little
value; it is not likely that the piopeity could be exchanged in a maiket. And the inability to sell
piopeity would limit the incentive of owneis to maintain it.
2. Piopeity iights must be tiansfeiable. A transferable property right is one that allows the
ownei of a iesouice to sell oi lease it to someone else. In the absence of tiansfeiability, no
exchange could occui.
150 PRINCIPLS OI CONOMICS
emcient
|e .||cc.t|cr c| .escu.ces
.|er t|e ret bere|ts c| .||
eccrcn|c .ct|v|t|es ..e
n.x|n|.eJ.
II GUR 6. 10 Demand and SuppIy and
the fficiency Condition
|r . ccnpet|t|ve n..ket .|t| exc|us|ve .rJ
t..rs|e..b|e p.cpe.ty .|g|ts, suc| .s t|e n..ket
|c. tcn.tces, t|e e|c|ercy ccrJ|t|cr |s net.
buye.s .rJ se||e.s ..e |.ceJ .|t| .|| c| t|e
.e|ev.rt bere|ts .rJ ccsts, .rJ t|e
e,u|||b.|un p.|ce e,u.|s t|e n..g|r.| ccst tc
scc|ety c| p.cJuc|rg t|.t gccJ, |e.e 2.50 pe.
pcurJ. \e c.r |rte.p.et t|e n..ket Jen.rJ
.rJ supp|y cu.ve .s n..g|r.| bere|t .rJ
n..g|r.| ccst cu.ves, .espect|ve|y.
consumer surpIus
|e .ncurt by .||c| t|e
tct.| bere|ts tc ccrsune.s
|.cn ccrsun|rg . gccJ
exceeJ t|e|. tct.|
experJ|tu.es cr t|e gccJ.
Markets and the fficiency Condition
A competitive maiket with well-defned and tiansfeiable piopeity iights satisfes the emciency condi-
tion. If met, we can assume that the maiket's allocation of iesouices will be emcient.
Considei again youi puichase of tomatoes. Suppose the cuives of demand and supply foi tomatoes
aie those given in Figuie 6.10; the equilibiium piice equals $1.30 pei pound. Suppose fuithei that the
maiket satisfes the emciency condition. With that assumption, we can ielate the model of demand and
supply to oui analysis of maiginal benefts and costs.
The demand cuive tells us that the last pound of tomatoes was woith $1.30; we can
think of that as the maiginal beneft of the last pound of tomatoes since that is how
much consumeis weie willing to pay. We can say that about any piice on a maiket de-
mand cuive; a demand cuive can be consideied as a maiginal beneft cuive. Similaily,
the supply cuive can be consideied the maiginal cost cuive. In the case of the tomato
maiket, foi example, the piice tells us that the maiginal cost of pioducing the last
pound of tomatoes is $1.30. This maiginal cost is consideied in the economic
sense-othei goods and seivices woith $1.30 weie not pioduced in oidei to make an
additional pound of tomatoes available.
On what basis can we piesume that the piice of a pound of tomatoes equals its
maiginal cost: The answei lies in oui maiginal decision iule. Pioft-maximizing tomato
pioduceis will pioduce moie tomatoes as long as theii maiginal beneft exceeds theii
maiginal cost. What is the maiginal beneft to a pioducei of an extia pound of toma-
toes: It is the piice that the pioducei will ieceive. What is the maiginal cost: It is the
value that must be given up to pioduce an extia pound of tomatoes.
Pioduceis maximize pioft by expanding theii pioduction up to the point at which
theii maiginal cost equals theii maiginal beneft, which is the maiket piice. The piice
of $1.30 thus iefects the maiginal cost to society of making an additional pound of to-
matoes available.
At the equilibiium piice and output of tomatoes, then, the maiginal beneft of to-
matoes to consumeis, as iefected by the piice they aie willing to pay, equals the mai-
ginal cost of pioducing tomatoes. Wheie maiginal beneft equals maiginal cost, net be-
neft is maximized. The equilibiium quantity of tomatoes, as deteimined by demand
and supply, is emcient.
2.2 Pioducei and Consumei Suiplus
Think about the last thing you puichased. You bought it because you expected that its
benefts would exceed its oppoitunity cost; you expected that the puichase would make
you bettei of. The sellei sold it to you because he oi she expected that the money you paid would be
woith moie than the value of keeping the item. The sellei expected to be bettei of as a iesult of the sale.
Exchanges in the maiketplace have a iemaikable piopeity: Both buyeis and selleis expect to emeige
fiom the tiansaction bettei of.
Panel (a) of Figuie 6.11 shows a maiket demand cuive foi a paiticulai good. Suppose the piice
equals OB and the quantity equals OE. The aiea undei the demand cuive ovei the iange of quantities
fiom the oiigin at O to the quantity at E equals the total beneft of consuming OE units of the good. It
is the aiea OCDE. Consumeis pay foi this beneft; theii total expendituies equal the iectangle OBDE,
which is the daik shaded iegion in the giaph. Because the total benefts exceed total expendituies, theie
is a consumei suiplus given by the tiiangle BCD. Consumer surplus is the amount by which the total
benefts to consumeis fiom consuming a good exceed theii total expendituies on the good.
CHAP7R 6 MARk7S, MAXIMI7RS, AND IIICINCY 151
producer surpIus
|e J||e.erce bet.eer t|e
tct.| .everue .ece|veJ by
se||e.s .rJ t|e|. tct.| ccst.
II GUR 6. 11 Consumer and Producer SurpIus
crsune. su.p|us |.re| (.) ne.su.es t|e J||e.erce bet.eer tct.| bere|t c| ccrsun|rg . g|ver ,u.rt|ty c|
cutput .rJ t|e tct.| experJ|tu.es ccrsune.s p.y tc cbt.|r t|.t ,u.rt|ty. |e.e, tct.| bere|ts ..e g|ver by t|e
s|.JeJ ..e. O||, tct.| experJ|tu.es ..e g|ver by t|e .ect.rg|e Ob||. |e J||e.erce, s|c.r by t|e t.|.rg|e b|,
|s ccrsune. su.p|us. |.cJuce. su.p|us |.re| b) ne.su.es t|e J||e.erce bet.eer tct.| .everue .ece|veJ by |.ns
.t . g|ver ,u.rt|ty c| cutput .rJ t|e tct.| ccst c| p.cJuc|rg |t. |e.e, tct.| .everue |s g|ver by t|e .ect.rg|e Ob||,
.rJ tct.| ccsts ..e g|ver by t|e ..e. OA||. |e J||e.erce, s|c.r by t|e t.|.rg|e Ab| |s p.cJuce. su.p|us.
Now considei the selleis' side of tiansactions. Panel (b) of Figuie 6.11 shows a maiket supply cuive; ie-
call that it gives us maiginal cost. Suppose the maiket piice equals OB and quantity supplied is OE;
those aie the same values we had in Panel (a). The piice times the quantity equals the total ievenue ie-
ceived by selleis. It is shown as the shaded iectangle OBDE. The total ievenue ieceived by selleis equals
total expendituies by consumeis.
The total cost to selleis is the aiea undei the maiginal cost cuive; it is the aiea OADE. That cost is
less than ievenue. The difeience between the total ievenue ieceived by selleis and theii total cost is
called producer surplus. In Panel (b) it is the light-shaded tiiangle ABD.
II GUR 6. 12 Net 8enefit: 7he Sum of Consumer and Producer SurpIus
|e sun c| ccrsune. su.p|us .rJ p.cJuce. su.p|us ne.su.es t|e ret bere|t tc scc|ety c| .ry |eve| c| eccrcn|c
.ct|v|ty. |et bere|t |s n.x|n|.eJ .|er p.cJuct|cr .rJ ccrsunpt|cr ..e c...|eJ cut .t t|e |eve| .|e.e t|e
Jen.rJ .rJ supp|y cu.ves |rte.sect. |e.e, t|e ret bere|t tc scc|ety e,u.|s t|e ..e. A|. |t |s t|e sun c| ccrsune.
su.p|us, b|, .rJ p.cJuce. su.p|us, Ab|.
152 PRINCIPLS OI CONOMICS
We put the demand and supply cuives of Figuie 6.11 Panels (a) and (b) togethei in Figuie 6.12. The in-
teisection of the two cuives deteimines the equilibiium piice, OB, and the equilibiium quantity, OE.
The shaded iegions give us consumei and pioducei suiplus. The sum of these two suipluses is net be-
neft. This net beneft is maximized wheie the demand and supply cuives inteisect.
2.3 Efficiency and Equity
Consumei demands aie afected by incomes. Demand, aftei all, iefects ability as well as willingness to
pay foi goods and seivices. The maiket will be moie iesponsive to the piefeiences of people with high
incomes than to those of people with low incomes.
In a maiket that satisfes the emciency condition, an emcient allocation of iesouices will emeige
fiom any paiticulai distiibution of income. Difeient income distiibutions will iesult in difeient, but
still emcient, outcomes. Foi example, if 1 of the population contiols viitually all the income, then the
maiket will emciently allocate viitually all its pioduction to those same people.
What is a faii, oi equitable, distiibution of income: What is an unfaii distiibution: Should eveiy-
one have the same income: Is the cuiient distiibution faii: Should the iich have less and the pooi have
moie: Should the middle class have moie: Equity is veiy much in the mind of the obseivei. What may
seem equitable to one peison may seem inequitable to anothei. Theie is, howevei, no test we can apply
to deteimine whethei the distiibution of income is oi is not equitable. That question iequiies a noim-
ative judgment.
Deteimining whethei the allocation of iesouices is oi is not emcient is one pioblem. Deteimining
whethei the distiibution of income is faii is anothei. The goveinments of all nations act in some way to
iedistiibute income. That fact suggests that people geneially have concluded that leaving the distiibu-
tion of income solely to the maiket would not be faii and that some iedistiibution is desiiable. This
may take the foim of highei taxes foi people with highei incomes than foi those with lowei incomes. It
may take the foim of special piogiams, such as welfaie piogiams, foi low-income people.
Whatevei distiibution society chooses, an emcient allocation of iesouices is still piefeiied to an in-
emcient one. Because an emcient allocation maximizes net benefts, the gain in net benefts could be
distiibuted in a way that leaves all people bettei of than they would be at any inemcient allocation. If
an emcient allocation of iesouices seems unfaii, it must be because the distiibution of income is unfaii.
k Y 7 A k A W A Y S
< |r . ccnpet|t|ve systen |r .||c| t|e |rte..ct|cr c| Jen.rJ .rJ supp|y Jete.n|re p.|ces, t|e
cc..espcrJ|rg Jen.rJ .rJ supp|y cu.ves c.r be ccrs|Je.eJ n..g|r.| bere|t .rJ n..g|r.| ccst cu.ves,
.espect|ve|y.
< Ar e|c|ert .||cc.t|cr c| .escu.ces |s cre t|.t n.x|n|.es t|e ret bere|t c| e.c| .ct|v|ty. \e expect |t tc
be .c||eveJ |r n..kets t|.t s.t|s|y t|e e|c|ercy ccrJ|t|cr, .||c| .e,u|.es . ccnpet|t|ve n..ket .rJ .e||
Je|reJ, t..rs|e..b|e p.cpe.ty .|g|ts.
< crsune. su.p|us |s t|e .ncurt by .||c| t|e tct.| bere|t tc ccrsune.s |.cn scne .ct|v|ty exceeJs t|e|.
tct.| experJ|tu.es |c. |t.
< |.cJuce. su.p|us |s t|e .ncurt by .||c| t|e tct.| .everues c| p.cJuce.s exceeJ t|e|. tct.| ccsts.
< Ar |re,u|t.b|e .||cc.t|cr c| .escu.ces |np||es t|.t t|e J|st.|but|cr c| |rccne .rJ .e.|t| |s |re,u|t.b|e.
'uJgnerts .bcut e,u|ty ..e rc.n.t|ve uJgnerts.
7 R Y I 7 !
|... |ypct|et|c.| Jen.rJ .rJ supp|y cu.ves |c. . typ|c.| p.cJuct, s.y cc|ee. |c. s|c. t|e ..e.s c| ccr
sune. .rJ p.cJuce. su.p|us. |rJe. .|.t c|.cunst.rces |s t|e n..ket ||ke|y tc be e|c|ert`
CHAP7R 6 MARk7S, MAXIMI7RS, AND IIICINCY 153
Case in Point: Saving the Iephant 7hrough Property Rights
2010 jupiterimages Corporation
|e A|.|c.r e|ep|.rt, t|e .c.|Js |..gest |.rJ n.nn.|, seeneJ tc be |r J.rge. c| ext|rct|cr |r t|e 20t| cer
tu.y. |e pcpu|.t|cr c| A|.|c.r e|ep|.rts |e|| |.cn 1.3 n||||cr |r 199 tc 543,000 |r 1994. |e ncst J..n.t|c
|css c| e|ep|.rts c.ne |r |ery., .|e.e t|e pcpu|.t|cr |e|| |.cn 16,000 e..|y |r t|e 190s tc .bcut 26,000 |r
199, .ccc.J|rg tc t|e \c.|J \||J|||e |urJ. c ccnb.t t|e s|.ug|te., .r |rte.r.t|cr.| .g.eenert, t|e crver
t|cr cr |rte.r.t|cr.| ..Je |r |rJ.rge.eJ Spec|es c| \||J ||c.. .rJ |.ur. (||S), .ert |rtc e|ect |r 1989. |t
b.rreJ t|e s.|e c| |vc.y.
|esp|te ||S .rJ ..neJ p.t.c|s .|t| c.Je.s tc s|cct pc.c|e.s cr s|g|t, t|e pc.c|e.s ccrt|rueJ tc cpe..te |r
|ery., k||||rg .cug||y 200 e|ep|.rts pe. J.y. |e e|ep|.rts .e.e k|||eJ |c. t|e|. |vc.y, t|e tusks |.cn . s|rg|e
.r|n.| ccu|J be sc|J |c. 2,000 |r t|e b|.ck n..ketre..|y Jcub|e t|e .rru.| pe. c.p|t. |rccne |r |ery..
Seve..| A|.|c.r r.t|crs, |c.eve., |.ve t.ker . ..J|c.||y J||e.ert .pp.c.c|. |ey |.ve est.b||s|eJ exc|us|ve,
t..rs|e..b|e p.cpe.ty .|g|ts |r ||cerses tc |urt e|ep|.rts. |r e.c| c| t|ese r.t|crs, e|ep|.rt pcpu|.t|crs |.ve
|rc.e.seJ. |ese r.t|crs |rc|uJe bcts..r., |.n|b|., Scut| A|.|c., .r..r|., .nb|., .rJ |nb.b.e. |r bct
s..r., |c. ex.np|e, t|e e|ep|.rt pcpu|.t|cr |rc.e.seJ |.cn 20,000 |r 1981 tc 80,000 |r 2000. |nb.b.e |r
c.e.seJ |ts e|ep|.rt pcpu|.t|cr |.cn 30,000 |r 198 tc re..|y 90,000 |r 2000.
|.c|essc.s V|c|.e| A. Vc||e.scr .rJ V|c|.e| |. ||es.|.Jcny c| t|e |r|ve.s|ty c| |c.t| ex.s |.ve Jcre .
st.t|st|c.| .r.|ys|s c| t|e Jete.n|r.rts c| e|ep|.rt pcpu|.t|crs |r 35 A|.|c.r r.t|crs. |ey |curJ t|.t e|ep|.rt
pcpu|.t|crs |rc.e.seJ |r r.t|crs t|.t |.J (.) est.b||s|eJ exc|us|ve, t..rs|e..b|e p.cpe.ty .|g|ts |r ||cerses tc
|urt e|ep|.rts .rJ (b) |.J st.b|e pc||t|c.| systens. crve.se|y, e|ep|.rt pcpu|.t|crs Jec||reJ |r ccurt.|es
t|.t |.J |.||eJ tc est.b||s| p.cpe.ty .|g|ts .rJ t|.t |.J urst.b|e pc||t|c.| systens.
|e s.ne .ppe..s tc be t.ue c| t|e .||te .||rcce.cs, . c.e.tu.e .|cse |c.rs ..e ||g||y v.|ueJ |r As|. .s .r
.p|.cJ|s|.c. Scut| A|.|c. se||s pe.n|ts tc |urt t|e c.e.tu.es |c. 25,000 pe. .r|n.|. |ts .||rcce.cs |e.J |.s |r
c.e.seJ |.cn 20 |r 1900 tc nc.e t|.r ,000 by t|e |.te 1990s.
|e.e |s rc sec.et tc t|e p.ese.v.t|cr c| spec|es. |st.b||s||rg c|e..|y Je|reJ, t..rs|e..b|e p.cpe.ty .|g|ts v|.
tu.||y .ssu.es t|e p.ese.v.t|cr c| spec|es. \|et|e. |t be bu|.|ces, .||rcce.cses, c. e|ep|.rts, p.cpe.ty .|g|ts
est.b||s| . n..ket, .rJ t|.t n..ket p.ese.ves spec|es.
.ooce ||o Oo|e. ^e |e, .o|e | o |oJ.' |e |e o| |oJo (.o|, 6. 994: S. /|c|oe| ^ /c||eo oJ /|c|oe| | ||e.|oJo,.
^||co ||e|o| |e ||ec| o| |oe|, |||| oJ |o||||co| .|o||||,.' o|eoo, |coo|c |o||c,. S(: (.ooo, 2000: 426. o| oJ |o
oJ oe.o||o||.' |e |coo||. 343(S09: (/o, 3. 99.: 44
154 PRINCIPLS OI CONOMICS
market faiIure
|e |.||u.e c| p.|v.te
Jec|s|crs |r t|e n..ketp|.ce
tc .c||eve .r e|c|ert
.||cc.t|cr c| sc..ce .escu.ces.
A N S W R 7 O 7 R Y I 7 ! P R O 8 L M
Or t|e .ssunpt|cr t|.t t|e cc|ee n..ket |s ccnpet|t|ve .rJ t|.t |t |s c|...cte.|.eJ by .e||Je|reJ exc|us|ve
.rJ t..rs|e..b|e p.cpe.ty .|g|ts, t|e cc|ee n..ket neets t|e e|c|ercy ccrJ|t|cr. |.t ne.rs t|.t t|e .||cc.
t|cr c| .escu.ces s|c.r .t t|e e,u|||b.|un .||| be t|e cre t|.t n.x|n|.es t|e ret bere|t c| .|| .ct|v|t|es. |e
ret bere|t |s s|..eJ by cc|ee ccrsune.s (.s ne.su.eJ by ccrsune. su.p|us) .rJ cc|ee p.cJuce.s (.s ne.s
u.eJ by p.cJuce. su.p|us).
3. MARKET FAILURE
L A R N I N G O 8 1 C 7 I V S
1. xpIain what is meant by market faiIure and the conditions that may Iead to it.
2. Distinguish between private goods and pubIic goods and reIate them to the free rider probIem
and the roIe of government.
3. xpIain the concepts of externaI costs and benets and the roIe of government intervention
when they are present.
4. xpIain why a common property resource is unIikeIy to be aIIocated emcientIy in the
marketpIace.
Piivate decisions in the maiketplace may not be consistent with the maximization of the net beneft of
a paiticulai activity. The failuie of piivate decisions in the maiketplace to achieve an emcient allocation
of scaice iesouices is called market failure. Maikets will not geneiate an emcient allocation of ie-
souices if they aie not competitive oi if piopeity iights aie not well defned and fully tiansfeiable. Eith-
ei condition will mean that decision makeis aie not faced with the maiginal benefts and costs of theii
choices.
Think about the diive that we had you take at the beginning of this chaptei. You faced some, but
not all, of the oppoitunity costs involved in that choice. In paiticulai, youi choice to go foi a diive
would inciease aii pollution and might inciease tiamc congestion. That means that, in weighing the
maiginal benefts and maiginal costs of going foi a diive, not all of the costs would be counted. As a
iesult, the net beneft of the allocation of iesouices such as the aii might not be maximized.
3.1 Noncompetitive Maikets
The model of demand and supply assumes that maikets aie competitive. No one in these maikets has
any powei ovei the equilibiium piice; each consumei and pioducei takes the maiket piice as given and
iesponds to it. Undei such conditions, piice is deteimined by the inteisection of demand and supply.
In some maikets, howevei, individual buyeis oi selleis aie poweiful enough to infuence the mai-
ket piice. In subsequent chapteis, we will study cases in which pioduceis oi consumeis aie in a posi-
tion to afect the piices they chaige oi must pay, iespectively. We shall fnd that when individual fims
oi gioups of fims have maiket powei, which is the ability to change the maiket piice, the piice will be
distoited-it will not equal maiginal cost.
CHAP7R 6 MARk7S, MAXIMI7RS, AND IIICINCY 155
pubIic good
A gccJ |c. .||c| t|e ccst c|
exc|us|cr |s p.c||b|t|ve .rJ
|c. .||c| t|e n..g|r.| ccst
c| .r .JJ|t|cr.| use. |s .e.c.
private good
A gccJ |c. .||c| exc|us|cr |s
pcss|b|e .rJ |c. .||c| t|e
n..g|r.| ccst c| .rct|e. use.
|s pcs|t|ve.
free riders
|ecp|e c. |.ns t|.t ccrsune
. pub||c gccJ .|t|cut p.y|rg
|c. |t.
3.2 Public Goods
Some goods aie unlikely to be pioduced and exchanged in a maiket because of special chaiacteiistics of
the goods themselves. The benefts of these goods aie such that exclusion is not feasible. Once they aie
pioduced, anyone can enjoy them; theie is no piactical way to exclude people who have not paid foi
them fiom consuming them. Fuitheimoie, the maiginal cost of adding one moie consumei is zeio. A
good foi which the cost of exclusion is piohibitive and foi which the maiginal cost of an additional
usei is zeio is a public good. A good foi which exclusion is possible and foi which the maiginal cost
of anothei usei is positive is a private good.
National defense is a public good. Once defense is piovided, it is not possible to exclude people
who have not paid foi it fiom its consumption. Fuithei, the cost of an additional usei is zeio-an aimy
does not cost any moie if theie is one moie peison to be piotected. Othei examples of public goods in-
clude law enfoicement, fie piotection, and efoits to pieseive species thieatened with extinction.
Iree Riders
Suppose a piivate fim, Teiioi Aleit, Inc., develops a completely ieliable system to identify and intei-
cept 98 of any would-be teiioiists that might attempt to entei the United States fiom anywheie in the
woild. This seivice is a public good. Once it is piovided, no one can be excluded fiom the system's pio-
tection on giounds that he oi she has not paid foi it, and the cost of adding one moie peison to the
gioup piotected is zeio. Suppose that the system, by eliminating a potential thieat to U.S. secuiity,
makes the aveiage peison in the United States bettei of; the beneft to each household fiom the added
secuiity is woith $40 pei month (about the same as an eaithquake insuiance piemium). Theie aie
ioughly 113 million households in the United States, so the total beneft of the system is $4.3 billion pei
month. Assume that it will cost Teiioi Aleit, Inc., $1 billion pei month to opeiate. The benefts of the
system fai outweigh the cost.
Suppose that Teiioi Aleit installs its system and sends a bill to each household foi $20 foi the fist
month of seivice-an amount equal to half of each household's beneft. If each household pays its bill,
Teiioi Aleit will enjoy a tidy pioft; it will ieceive ievenues of moie than $2.23 billion pei month.
But will each household pay: Once the system is in place, each household would iecognize that it
will beneft fiom the secuiity piovided by Teiioi Aleit whethei it pays its bill oi not. Although some
households will voluntaiily pay theii bills, it seems unlikely that veiy many will. Recognizing the op-
poitunity to consume the good without paying foi it, most would be fiee iideis. Free riders aie people
oi fims that consume a public good without paying foi it. Even though the total beneft of the system
is $4.3 billion, Teiioi Aleit will not be faced by the maiketplace with a signal that suggests that the sys-
tem is woithwhile. It is unlikely that it will iecovei its cost of $1 billion pei month. Teiioi Aleit is not
likely to get of the giound.
The bill foi $20 fiom Teiioi Aleit sends the wiong signal, too. An emcient maiket iequiies a piice
equal to maiginal cost. But the maiginal cost of piotecting one moie household is zeio; adding one
moie household adds nothing to the cost of the system. A household that decides not to pay Teiioi
Aleit anything foi its seivice is paying a piice equal to its maiginal cost. But doing that, being a fiee
iidei, is piecisely what pievents Teiioi Aleit fiom opeiating.
Because no household can be excluded and because the cost of an extia household is zeio, the em-
ciency condition will not be met in a piivate maiket. What is tiue of Teiioi Aleit, Inc., is tiue of public
goods in geneial: they simply do not lend themselves to piivate maiket piovision.
PubIic Goods and the Government
Because many individuals who beneft fiom public goods will not pay foi them, piivate fims will pio-
duce a smallei quantity of public goods than is emcient, if they pioduce them at all. In such cases, it
may be desiiable foi goveinment agencies to step in. Goveinment can supply a gieatei quantity of the
good by diiect piovision, by puichasing the public good fiom a piivate agency, oi by subsidizing con-
sumption. In any case, the cost is fnanced thiough taxation and thus avoids the fiee-iidei pioblem.
Most public goods aie piovided diiectly by goveinment agencies. Goveinments pioduce national
defense and law enfoicement, foi example. Piivate fims undei contiact with goveinment agencies pio-
duce some public goods. Paik maintenance and fie seivices aie public goods that aie sometimes pio-
duced by piivate fims. In othei cases, the goveinment piomotes the piivate consumption oi pioduc-
tion of public goods by subsidizing them. Piivate chaiitable contiibutions often suppoit activities that
aie public goods; fedeial and state goveinments subsidize these by allowing taxpayeis to ieduce theii
tax payments by a fiaction of the amount they contiibute.
156 PRINCIPLS OI CONOMICS
II GUR 6. 15 PubIic Goods and Market
IaiIure
bec.use |.ee .|Je.s .||| p.evert |.ns |.cn
be|rg .b|e tc .e,u|.e ccrsune.s tc p.y |c. t|e
bere|ts .ece|veJ |.cn ccrsun|rg . pub||c
gccJ, cutput .||| be |ess t|.r t|e e|c|ert
|eve|. |r t|e c.se s|c.r |e.e, p.|v.te
Jcr.t|crs .c||eveJ . |eve| c| t|e pub||c gccJ
c| (
1
pe. pe.|cJ. |e e|c|ert |eve| |s (. |e
Je.J.e|g|t |css |s s|c.r by t|e t.|.rg|e Ab.
externaI cost
A ccst |npcseJ cr ct|e.s
cuts|Je c| .ry n..ket
exc|.rge.
externaI benet
Ar .ct|cr t.ker by . pe.scr
c. |.n t|.t c.e.tes bere|ts
|c. ct|e.s |r t|e .bserce c|
.ry n..ket .g.eenert.
While the maiket will pioduce some level of public goods in the absence of gov-
einment inteivention, we do not expect that it will pioduce the quantity that maxim-
izes net beneft. Figuie 6.13 illustiates the pioblem. Suppose that piovision of a public
good such as national defense is left entiiely to piivate fims. It is likely that some de-
fense seivices would be pioduced; suppose that equals Q
1
units pei peiiod. This level of
national defense might be achieved thiough individual contiibutions. But it is veiy un-
likely that contiibutions would achieve the coiiect level of defense seivices. The em-
cient quantity occuis wheie the demand, oi maiginal beneft, cuive inteisects the mai-
ginal cost cuive, at Q. The deadweight loss is the shaded aiea ABC; we can think of
this as the net beneft of goveinment inteivention to inciease the pioduction of nation-
al defense fiom Q
1
up to the emcient quantity, Q.
Heads Up!
|cte t|.t t|e Je|r|t|crs c| pub||c .rJ p.|v.te gccJs ..e b.seJ cr c|...cte.|st|cs c| t|e gccJs
t|ense|ves, rct cr .|et|e. t|ey ..e p.cv|JeJ by t|e pub||c c. t|e p.|v.te sectc.. |cst.| se.
v|ces ..e . p.|v.te gccJ p.cv|JeJ by t|e pub||c sectc.. |e |.ct t|.t t|ese gccJs ..e p.cJuceJ
by . gcve.rnert .gercy Jces rct n.ke t|en . pub||c gccJ.
3.3 Exteinal Costs and Benefits
Suppose that in the couise of pioduction, the fims in a paiticulai industiy geneiate aii
pollution. These fims thus impose costs on otheis, but they do so outside the context
of any maiket exchange-no agieement has been made between the fims and the
people afected by the pollution. The fims thus will not be faced with the costs of theii
action. A cost imposed on otheis outside of any maiket exchange is an external cost.
We saw an example of an exteinal cost in oui imaginaiy decision to go foi a diive.
Heie is anothei: violence on television, in the movies, and in video games. Many ciitics aigue that the
violence that peivades these media fosteis gieatei violence in the ieal woild. By the time a child who
spends the aveiage amount of time watching television fnishes elementaiy school, he oi she will have
seen 100,000 acts of violence, including 8,000 muideis, accoiding to the Ameiican Psychological Asso-
ciation. Thousands of studies of the ielationship between violence in the media and behavioi have con-
cluded that theie is a link between watching violence and violent behaviois. Video games aie a majoi
element of the pioblem, as young childien now spend houis each week playing them. Fifty peicent of
fouith-giade giadeis say that theii favoiite video games aie the fist peison shootei" type.
[1]
Any tendency of incieased violence iesulting fiom incieased violence in the media constitutes an
exteinal cost of such media. The Ameiican Academy of Pediatiics iepoited in 2001 that homicides
weie the fouith leading cause of death among childien between the ages of 10 and 14 and the second
leading cause of death foi people aged 13 to 24 and has iecommended a ieduction in exposuie to me-
dia violence.
[2]
It seems ieasonable to assume that at least some of these acts of violence can be con-
sideied an exteinal cost of violence in the media.
An action taken by a peison oi fim can also cieate benefts foi otheis, again in the absence of any
maiket agieement; such a beneft is called an external benent. A fim that builds a beautiful building
geneiates benefts to eveiyone who admiies it; such benefts aie exteinal.
CHAP7R 6 MARk7S, MAXIMI7RS, AND IIICINCY 157
II GUR 6. 16 xternaI Costs
\|er |.ns |r .r |rJust.y gere..te exte.r.|
ccsts, t|e supp|y cu.ve .
1
.e|ects cr|y t|e|.
p.|v.te n..g|r.| ccsts, /
|
. |c.c|rg |.ns tc
p.y t|e exte.r.| ccsts t|ey |npcse s|||ts t|e
supp|y cu.ve tc .
2
, .||c| .e|ects t|e |u||
n..g|r.| ccst c| t|e |.ns p.cJuct|cr, /
e
.
Output |s .eJuceJ .rJ p.|ce gces up. |e
Je.J.e|g|t |css t|.t cccu.s .|er |.ns ..e
rct |.ceJ .|t| t|e |u|| ccsts c| t|e|. Jec|s|crs
|s s|c.r by t|e s|.JeJ ..e. |r t|e g..p|.
common property resource
|escu.ces |c. .||c| rc
p.cpe.ty .|g|ts |.ve beer
Je|reJ.
xternaI Costs and fficiency
The case of the polluting fims is illustiated in Figuie 6.16. The industiy supply cuive
S
1
iefects piivate maiginal costs, MC
p
. The maiket piice is P
p
foi a quantity Q
p
. This is
the solution that would occui if fims geneiating exteinal costs weie not foiced to pay
those costs. If the exteinal costs geneiated by the pollution weie added, the new supply
cuive S
2
would iefect highei maiginal costs, MC
e
. Faced with those costs, the maiket
would geneiate a lowei equilibiium quantity, Q
e
. That quantity would command a
highei piice, P
e
. The failuie to confiont pioduceis with the cost of theii pollution
means that consumeis do not pay the full cost of the good they aie puichasing. The
level of output and the level of pollution aie theiefoie highei than would be economic-
ally emcient. If a way could be found to confiont pioduceis with the full cost of theii
choices, then consumeis would be faced with a highei cost as well. Figuie 6.16 shows
that consumption would be ieduced to the emcient level, Q
e
, at which demand and the
full maiginal cost cuive (MC
e
) inteisect. The deadweight loss geneiated by allowing the
exteinal cost to be geneiated with an output of Q
p
is given as the shaded iegion in the
giaph.
xternaI Costs and Government Intervention
If an activity geneiates exteinal costs, the decision makeis geneiating the activity will
not be faced with its full costs. Agents who impose these costs will caiiy out theii activ-
ities beyond the emcient level; those who consume them, facing too low a piice, will
consume too much. As a iesult, pioduceis and consumeis will caiiy out an excessive
quantity of the activity. In such cases, goveinment may tiy to inteivene to ieduce the
level of the activity towaid the emcient quantity. In the case shown in Figuie 6.16, foi
example, fims geneiating an exteinal cost have a supply cuive S
1
that iefects theii
piivate maiginal costs, MC
p
. A pei-unit pollution fee imposed on the fims would in-
ciease theii maiginal costs to MC
e
, thus shifting the supply cuive to S
2
, and the emcient
level of pioduction would emeige. Taxes oi othei iestiictions may be imposed on the
activity that geneiates the exteinal cost in an efoit to confiont decision makeis with
the costs that they aie imposing. In many aieas, fims and consumeis that pollute iiveis and lakes aie
iequiied to pay fees based on the amount they pollute. Fiims in many aieas aie iequiied to puichase
peimits in oidei to pollute the aii; the iequiiement that peimits be puichased seives to confiont the
fims with the costs of theii choices.
Anothei appioach to dealing with pioblems of exteinal costs is diiect iegulation. Foi example, a
fim may be oideied to ieduce its pollution. A peison who tuins his oi hei fiont yaid into a gaibage
dump may be oideied to clean it up. Paiticipants at a iaucous paity may be told to be quiet. Alteinative
ways of dealing with exteinal costs aie discussed latei in the text.
3.4 Common Piopeity Resouices
Common property resources
[3]
aie iesouices foi which no piopeity iights have been defned. The
dimculty with common piopeity iesouices is that individuals may not have adequate incentives to en-
gage in efoits to pieseive oi piotect them. Considei, foi example, the ielative fates of cattle and bufalo
in the United States in the nineteenth centuiy. Cattle populations incieased thioughout the centuiy,
while the bufalo neaily became extinct. The chief difeience between the two animals was that exclus-
ive piopeity iights existed foi cattle but not foi bufalo.
Owneis of cattle had an incentive to maintain heid sizes. A cattle ownei who slaughteied all of his
oi hei cattle without pioviding foi ieplacement of the heid would not have a souice of futuie income.
Cattle owneis not only maintained theii heids but also engaged in extensive efoits to bieed high-qual-
ity livestock. They invested time and efoit in the emcient management of the iesouice on which theii
livelihoods depended.
Bufalo hunteis suiely had similai conceins about the maintenance of bufalo heids, but they had
no individual stake in doing anything about them-the animals weie a common piopeity iesouice.
Thousands of individuals hunted bufalo foi a living. Anyone who cut back on hunting in oidei to help
to pieseive the heid would lose income-and face the likelihood that othei hunteis would go on hunt-
ing at the same iate as befoie.
Today, exclusive iights to bufalo have been widely established. The demand foi bufalo meat,
which is lowei in fat than beef, has been incieasing, but the numbei of bufalo in the United States is
iising iapidly. If bufalo weie still a common piopeity iesouice, that incieased demand, in the absence
of othei iestiictions on hunting of the animals, would suiely iesult in the elimination of the animal. Be-
cause theie aie exclusive, tiansfeiable piopeity iights in bufalo and because a competitive maiket
158 PRINCIPLS OI CONOMICS
biings buyeis and selleis of bufalo and bufalo pioducts togethei, we can be ieasonably confdent in
the emcient management of the animal.
When a species is thieatened with extinction, it is likely that no one has exclusive piopeity iights
to it. Whales, condois, giizzly beais, elephants in Cential Afiica-whatevei the animal that is
thieatened-aie common piopeity iesouices. In such cases a goveinment agency may impose limits on
the killing of the animal oi destiuction of its habitat. Such limits can pievent the excessive piivate use
of a common piopeity iesouice. Alteinatively, as was done in the case of the bufalo, piivate iights can
be established, giving iesouice owneis the task of pieseivation.
k Y 7 A k A W A Y S
< |ub||c sectc. |rte.vert|cr tc |rc.e.se t|e |eve| c| p.cv|s|cr c| pub||c gccJs n.y |np.cve t|e e|c|ercy c|
.escu.ce .||cc.t|cr by cve.ccn|rg t|e p.cb|en c| |.ee .|Je.s.
< Act|v|t|es t|.t gere..te exte.r.| ccsts ..e ||ke|y tc be c...|eJ cut .t |eve|s t|.t exceeJ t|cse t|.t .cu|J be
e|c|ert, t|e pub||c sectc. n.y seek tc |rte.vere tc ccr|.crt Jec|s|cr n.ke.s .|t| t|e |u|| ccsts c| t|e|.
c|c|ces.
< Scne p.|v.te .ct|v|t|es gere..te exte.r.| bere|ts.
< A ccnncr p.cpe.ty .escu.ce |s ur||ke|y tc be .||cc.teJ e|c|ert|y |r t|e n..ketp|.ce.
7 R Y I 7 !
|e n.ru|.ctu.e c| nenc.y c||ps |c. ccnpute.s gere..tes pc||ut.rts t|.t gere..||y erte. .|ve.s .rJ st.e.ns.
|se t|e ncJe| c| Jen.rJ .rJ supp|y tc s|c. t|e e,u|||b.|un p.|ce .rJ cutput c| c||ps. Assun|rg c||p n.r
u|.ctu.e.s Jc rct |.ve tc p.y t|e ccsts t|ese pc||ut.rts |npcse, .|.t c.r ycu s.y .bcut t|e e|c|ercy c| t|e
,u.rt|ty c| c||ps p.cJuceJ` S|c. t|e ..e. c| Je.J.e|g|t |css |npcseJ by t||s exte.r.| ccst. S|c. |c. . .e
,u|.enert t|.t |.ns p.y t|ese ccsts .s t|ey p.cJuce t|e c||ps .cu|J .|ect t|e e,u|||b.|un p.|ce .rJ cutput
c| c||ps. \cu|J suc| . .e,u|.enert |e|p tc s.t|s|y t|e e|c|ercy ccrJ|t|cr` |xp|.|r.
Case in Point: xternaIities and Smoking
2010 jupiterimages Corporation
CHAP7R 6 MARk7S, MAXIMI7RS, AND IIICINCY 159
Sncke.s |npcse t.enerJcus ccsts cr t|ense|ves. b.seJ sc|e|y cr t|e Jeg.ee tc .||c| snck|rg s|c.ters
t|e|. |||e expect.rcy, .||c| |s by .bcut s|x ye..s, t|e ccst pe. p.ck |s 35.64. |.t ccst, c| ccu.se, |s . p.|v.te
ccst. |r .JJ|t|cr tc t|.t p.|v.te ccst, sncke.s |npcse ccsts cr ct|e.s. |cse exte.r.| ccsts ccne |r t|.ee ..ys.
||.st, t|ey |rc.e.se |e.|t|c..e ccsts .rJ t|us |rc.e.se |e.|t| |rsu..rce p.en|uns. SeccrJ, snck|rg c.uses
|.es t|.t Jest.cy nc.e t|.r 300 n||||cr .c.t| c| p.cpe.ty e.c| ye... ||.J, nc.e t|.r 2,000 pecp|e J|e e.c|
ye.. .s . .esu|t c| seccrJ|.rJ sncke. A 1989 stuJy by t|e |A|| c.pc..t|cr est|n.teJ t|ese ccsts .t 0.53
pe. p.ck.
|r .r |npc.t.rt ..y, |c.eve., sncke.s .|sc gere..te exte.r.| bere|ts. |ey ccrt.|bute tc .et|.enert p.c
g..ns .rJ tc Scc|.| Secu.|ty, t|er J|e sccre. t|.r rcrsncke.s. |ey t|us subs|J|.e t|e .et|.enert p.cg..ns
c| t|e .est c| t|e pcpu|.t|cr. Accc.J|rg tc t|e |A|| stuJy, t|.t p.cJuces .r exte.r.| bere|t c| 0.24 pe.
p.ck, |e.v|rg . ret exte.r.| ccst c| 0.29 pe. p.ck. C|ver t|.t st.te .rJ |eJe..| exc|se t.xes .ve..geJ 0.3 |r
1989, t|e |A|| .ese..c|e.s ccrc|uJeJ t|.t sncke.s nc.e t|.r p.|J t|e|. c.r ..y.
|ccrcn|sts 'cr.t|.r C.ube. c| t|e V.ss.c|usetts |rst|tute c| ec|rc|cgy .rJ bctcrJ |cs.eg| c| t|e
|r|ve.s|ty c| .|||c.r|. .t be.ke|ey |.ve suggesteJ t|.t, |r t|e c.se c| pecp|e .|c ccrsune .JJ|ct|ve b.Js
suc| .s c|g..ettes, .r exc|se t.x cr c|g..ettes c| .s nuc| .s 4.6 pe. p.ck n.y |np.cve t|e .e||..e c|
sncke.s.
|ey b.se t|e|. ..gunert cr t|e ccrcept c| t|ne |rccrs|stercy, .||c| |s t|e t|ec.y t|.t sncke.s seek t|e
|nneJ|.te g..t||c.t|cr c| . c|g..ette .rJ t|er .eg.et t|e|. Jec|s|cr |.te.. |.c|essc.s C.ube. .rJ |cs.eg| ..gue
t|.t ||g|e. t.xes .cu|J se.ve tc .eJuce t|e ,u.rt|ty c| c|g..ettes Jen.rJeJ .rJ t|us .eJuce be|.v|c. t|.t
sncke.s .cu|J ct|e..|se .eg.et. |e|. ..gunert |s t|.t sncke.s |npcse |rte.r.||t|es cr t|ense|ves .rJ t|.t
||g|e. t.xes .cu|J .eJuce t||s.
\|e.e Jces t||s |e.J us` || sncke.s ..e ..t|cr.||y .JJ|cteJ tc snck|rg, |.e., t|ey |.ve .e|g|eJ t|e bere|ts
.rJ ccsts c| snck|rg .rJ |.ve c|cser tc sncke, t|er t|e cr|y p.cb|en |c. pub||c pc||cy |s tc .ssu.e t|.t
sncke.s ..e ccr|.crteJ .|t| t|e exte.r.| ccsts t|ey |npcse. |r t|.t c.se, t|e p.cb|en |s sc|veJ. t|.cug| ex
c|se t.xes, sncke.s nc.e t|.r p.y t|e|. c.r ..y. but, || t|e Jec|s|cr tc sncke |s .r |...t|cr.| cre, |t n.y be
|np.cveJ t|.cug| ||g|e. exc|se t.xes cr snck|rg.
.ooce .oo||o Ooe oJ |o|oJ |oe|. ^ |eo, o| Oo.ee| |eo|o||o o| ^JJ|c||.e |oJ ||o| o |e.e| oJ o |c|Jece |o
|oe||e |c|e oo||o.' |||| /o|| |oe S.... |eoo, 2002. /|||oJ O /o| e| o|. |e oe o| .| |o .o|e oJ |||e |o, |e|
/o,.' .ooo| o| ||e ^e|co /eJ|co| ^oc|o||o. 26 (/oc| .. 9S9: 604609
A N S W R 7 O 7 R Y I 7 ! P R O 8 L M
|r t|e .bserce c| .ry .egu|.t|cr, c||p p.cJuce.s ..e rct |.ceJ .|t| t|e ccsts c| t|e pc||ut|cr t|e|. cpe..t|crs
gere..te. |e n..ket p.|ce |s t|us |
1
.rJ t|e ,u.rt|ty (
1
. |e e|c|ercy ccrJ|t|cr |s rct net, t|e p.|ce |s
|c.e. .rJ t|e ,u.rt|ty g.e.te. t|.r .cu|J be e|c|ert. || p.cJuce.s .e.e |c.ceJ tc |.ce t|e ccst c| t|e|. pc||u
t|cr .s .e|| .s ct|e. p.cJuct|cr ccsts, t|e supp|y cu.ve .cu|J s|||t tc .
2
, t|e p.|ce .cu|J .|se tc |
2
, .rJ t|e
,u.rt|ty .cu|J |.|| tc (
2
. |e re. sc|ut|cr s.t|s|es t|e e|c|ercy ccrJ|t|cr.
160 PRINCIPLS OI CONOMICS
4. REVIEW AND PRACTICE
Summary
|ccrcn|sts |rs|st t|.t |rJ|v|Ju.|s Jc rct n.ke c|c|ces .|||yr|||y. |.t|e., eccrcn|sts .ssune t|.t |rJ|v|Ju.|s
n.ke c|c|ces |r . pu.pcse|u| ..y, cre t|.t seeks t|e n.x|nun v.|ue |c. scne cbect|ve. \e .ssune t|.t
ccrsune.s seek tc n.x|n|.e ut|||ty .rJ t|.t |.ns seek tc n.x|n|.e p.c|ts.
\|.teve. |s be|rg n.x|n|.eJ, c|c|ces ..e b.seJ cr t|e n..g|r.| Jec|s|cr .u|e. |c||c.|rg t||s .u|e .esu|ts |r .r
.||cc.t|cr t|.t .c||eves t|e g.e.test Jeg.ee c| ut|||ty c. p.c|t pcss|b|e.
|| ut|||ty .rJ p.c|tn.x|n|.|rg c|c|ces ..e n.Je |r t|e ccrtext c| . p.|ce systen t|.t ccr|.crts Jec|s|cr
n.ke.s .|t| .|| c| t|e ccsts .rJ .|| c| t|e bere|ts c| t|e|. c|c|ces, t|e .||cc.t|cr c| .escu.ces .||| be e|c|ert.
Ar e|c|ert .||cc.t|cr |s cre t|.t n.x|n|.es t|e ret bere|t c| eve.y .ct|v|ty. |e ccrcepts c| ccrsune. .rJ
p.cJuce. su.p|us s|c. us |c. t||s ret bere|t |s s|..eJ. |,u|ty |s . sep...te |ssue, cre t|.t c.||s |c. . rc.n.t
|ve ev.|u.t|cr c| t|e |.|.ress c| t|e J|st.|but|cr c| |rccne.
|e .||cc.t|cr c| .escu.ces .||| be |re|c|ert |r t|e .bserce c| ccnpet|t|ve n..kets. |t .||| .|sc be |re|c|ert ||
p.cpe.ty .|g|ts ..e rct exc|us|ve .rJ t..rs|e..b|e. |ese t.c ccrJ|t|crs b.e.k Jc.r .|er t|e.e ..e pub||c
gccJs, ccnncr p.cpe.ty .escu.ces, c. exte.r.| bere|ts c. ccsts. |r e.c| c| t|ese c.ses, pub||c sectc. |rte.
vert|cr n.y |np.cve t|e e|c|ercy c| .escu.ce .||cc.t|cr. \|er . n..ket |.||s tc .c||eve t|e e|c|ert sc|u
t|cr, ret bere|t |.||s s|c.t c| t|e n.x|nun pcss|b|e. |e.J.e|g|t |css |s t|e .ncurt by .||c| ret bere|t |.||s
be|c. t|e ret bere|t pcss|b|e .t t|e e|c|ert sc|ut|cr.
CHAP7R 6 MARk7S, MAXIMI7RS, AND IIICINCY 161
C O N C P 7 P R O 8 L M S
1. \|.t |s .c||eveJ by se|ect|rg t|e ,u.rt|ty c| .r .ct|v|ty .t .||c| n..g|r.| bere|t e,u.|s n..g|r.| ccst`
2. Suppcse t|e n..g|r.| bere|t c| .r .ct|v|ty exceeJs t|e n..g|r.| ccst. \|.t Jces t|e n..g|r.| Jec|s|cr
.u|e s.y . n.x|n|.|rg Jec|s|cr n.ke. .||| Jc`
3. Suppcse ycu ..e . J|scus |u.|e. .rJ ycu. gc.| |s tc n.x|n|.e t|e J|st.rce ycu .c||eve. +cu p.cJuce
J|scus |u.|s by p..ct|c|rg. |e tct.| bere|t c| p..ct|ce |s J|st.rce .c||eveJ, .rJ t|e |rput t|.t .c||eves
t||s J|st.rce |s |cu.s c| p..ct|ce. |esc.|be t|e tct.| bere|t cu.ve c| p..ct|ce. \|.t pc|rt cr t|e cu.ve
.cu|J ycu se|ect`
4. ||s c|.pte. ..gues t|.t ccrsune.s n.x|n|.e ut|||ty .rJ |.ns n.x|n|.e p.c|ts. \|.t Jc ycu suppcse
e.c| c| t|e |c||c.|rg n|g|t be p.esuneJ tc n.x|n|.e`
.. A n|r|ste. c. ..bb|
b. A |r|teJ St.tes Ser.tc.
c. |e n.r.ge. c| . n.c. |e.gue b.seb.|| te.n
J. |e c.re. c| . n.c. |e.gue b.seb.|| te.n
e. |e J|.ectc. c| . c|..|t.b|e c.g.r|..t|cr
5. |c. e.c| c| t|e |c||c.|rg gccJs, |rJ|c.te .|et|e. exc|us|ve, t..rs|e..b|e p.cpe.ty .|g|ts ex|st .rJ .|et|e.
t|e gccJ pcses . p.cb|en |c. pub||c pc||cy. || |t Jces, Jces t|e p.cb|en .e|.te tc . p.cb|en c| p.cpe.ty
.|g|ts`
.. |e.r .|.
b. cn.tces
c. |cus|rg
J. b|ue .|.|es
6. |e J.yc|e.r|rg |rJust.y |s . n.c. scu.ce c| .|. pc||ut|cr. \|.t c.r ycu ccrc|uJe .bcut t|e p.|ce .rJ
cutput c| J.yc|e.r|rg se.v|ces`
. |ccrcn|sts c|ter .eccnnerJ t|.t pc||ute.s suc| .s J.yc|e.r|rg est.b||s|nerts be c|..geJ |ees |c. t|e
pc||ut|cr t|ey en|t. .|t|cs c| t||s |Je. .espcrJ t|.t t|e est.b||s|nerts .cu|J s|np|y .espcrJ by p.ss|rg
t|ese c|..ges cr tc t|e|. custcne.s, |e.v|rg t|e |eve| c| pc||ut|cr urc|.rgeJ. cnnert cr t||s
cbect|cr.
8. Ccve.rnert .gerc|es c|ter .e,u|.e t|.t c|||J.er be |rccu|.teJ .g.|rst ccnnur|c.b|e J|se.ses suc| .s
pc||c .rJ ne.s|es. |.cn t|e st.rJpc|rt c| eccrcn|c e|c|ercy, |s t|e.e .ry ust||c.t|cr |c. suc| .
.e,u|.enert`
9. \||c| c| t|e |c||c.|rg gccJs c. se.v|ces ..e pub||c` \|y c. .|y rct`
.. ||b...|es
b. ||.e p.ctect|cr
c. e|ev|s|cr p.cg..ns
J. |e.|t| c..e
e. \.te. |c. |cuse|c|J ccrsunpt|cr
10. || . v|||.ge |r bcts..r. |s g..rteJ seve..| ||cerses tc k||| e|ep|.rts, |c. Jces t||s g|ve |t .r |rcert|ve tc
p.ese.ve e|ep|.rts .rJ |rc.e.se t|e s|.e c| t|e |e.J` |c. Jces t|e |rte.r.t|cr.| b.r cr |vc.y s.|es .|ect
t|e |rcert|ve |r bcts..r. tc p.ese.ve t|e e|ep|.rt`
11. |e runbe. c| |s| c.ug|t |r t|e cce.r |.s |.||er |r .ecert ye..s p..t|y .s . .esu|t c| nc.e |rters|ve
|s||rg e|c.ts .rJ t|e use c| nc.e scp||st|c.teJ e,u|pnert. ||s| |r t|e cce.r ..e . ccnncr p.cpe.ty
.escu.ce. |c. n|g|t t||s |.ct be .e|.teJ tc Jec||r|rg |s| c.tc|es` |c. Jc ycu t||rk t||s J.cp |r t|e c.tc|
.|ects t|e p.|ce c| se.|ccJ`
162 PRINCIPLS OI CONOMICS
N U M R I C A L P R O 8 L M S
1. 'ce ||gg|rs |s t||rk|rg .bcut |c. nuc| t|ne tc sperJ stuJy|rg |c. . b|c|cgy ex.n tcnc..c.. |s|rg
ut|||ty ur|ts |e ne.su.es t|e bere|ts .rJ ccsts c| stuJy, ||s c.|cu|.t|crs ..e s|c.r |r t|e |c||c.|rg
t.b|e.
.. |||| |r t|e |cu.t| .c. |c. ret bere|t |r t|e t.b|e. |se t|e n|Jpc|rt ccrvert|cr tc enp|.s|.e t|.t
t|e ret bere|t |s . n..g|r.| v.|ue s|c.|rg t|e g.|r .s |cu.s spert |rc.e.se by cre|cu.
|rc.enerts.
b. |s|rg . g..p| s|n||.. tc |.re| (.) c| ||gu.e 6.1 s|c. t|e n..g|r.| bere|t cu.ve .rJ ve.||y t|.t
t|e ..e. urJe. t|e cu.ve .t 3 |cu.s c| stuJy cc..espcrJs tc t|e tct.| bere|t c| t|.t nuc| stuJy.
(||rt. |enenbe. t|.t n..g|r.| v.|ues ..e p|ctteJ .t t|e n|Jpc|rts c| t|e cc..espcrJ|rg |rte.v.|s
cr t|e |c.|.crt.| .x|s.)
c. |se . g..p| s|n||.. tc |.re| (b) c| ||gu.e 6.1 tc s|c. t|e n..g|r.| ccst cu.ve .rJ ve.||y t|.t t|e
..e. urJe. t|e cu.ve .t 3 |cu.s c| stuJy cc..espcrJs tc t|e tct.| ccst c| t|.t nuc| stuJy.
J. |se . g..p| s|n||.. tc |.re| (.) c| ||gu.e 6.6 tc ccnb|re t|e n..g|r.| bere|t .rJ n..g|r.| ccst
cu.ves ycu J.e. |r p..ts (.) .rJ (b).
e. b.seJ cr t|e n..g|r.| Jec|s|cr .u|e, |c. n.ry |cu.s s|cu|J 'ce sperJ stuJy|rg |c. ||s b|c|cgy
ex.n`
2. |c. suppcse scne |.|erJs c| 'ces c.|| tc s.y t|ey ..e |.v|rg . p..ty tcr|g|t. 'ce c.|cu|.tes t|.t t|e p..ty
|s rc. ||s best .|te.r.t|ve tc stuJy, .rJ |e |rc.e.ses ||s est|n.te c| t|e ccst c| e.c| |cu. c| stuJy. Ore
|cu. c| stuJy rc. ccsts 0, t.c |cu.s ccst 140, t|.ee |cu.s 210, |cu. |cu.s 280, |ve |cu.s 350, .rJ s|x
|cu.s 40.
.. |... t|e re. n..g|r.| bere|t .rJ n..g|r.| ccst cu.ves .s |r |.cb|en 1, p..t (J).
b. b.seJ cr t|e n..g|r.| Jec|s|cr .u|e, |Jert||y t|e re. sc|ut|cr t|.t n.x|n|.es t|e ret bere|t c|
stuJy t|ne.
3. |e |cc.| g.sc||re n..ket |r . p..t|cu|.. c|ty |.s Jen.rJ .rJ supp|y cu.ves g|ver by t|e |c||c.|rg J.t..
(A|| ,u.rt|t|es ..e |r n||||crs c| g.||crs pe. ncrt|.)
|.|ce pe. g.||cr 1.00 1.50 2.00 2.50 3.00 3.50 4.00
u.rt|ty Jen.rJeJ 6 5 4 3 2 1 0
u.rt|ty supp||eJ 0 1 2 3 4 5 6
.. ||ct t|e Jen.rJ .rJ supp|y cu.ves, .rJ Jete.n|re t|e e,u|||b.|un p.|ce .rJ ,u.rt|ty.
b. S|c. t|e ..e.s c| ccrsune. .rJ p.cJuce. su.p|us.
c. |c. suppcse t|.t t|e ccnnur|ty Jete.n|res t|.t e.c| g.||cr c| g.sc||re ccrsuneJ |npcses
0.50 |r pc||ut|cr ccsts. Accc.J|rg|y, . 0.50pe.g.||cr t.x |s |npcseJ. |e t.x |s |npcseJ cr
se||e.s c| g.sc||re, .rJ |t |.s t|e e|ect c| |rc.e.s|rg by 0.50 t|e p.|ce .e,u|.eJ tc |rJuce t|e
,u.rt|t|es supp||eJ |r t|e t.b|e. |c. ex.np|e, . p.|ce c| 2.00 |s rc. .e,u|.eJ |c. . ,u.rt|ty c| 1
n||||cr g.||crs tc be supp||eJ e.c| ncrt|. ||ct t|e re. supp|y cu.ve.
J. App.cx|n.te t|e re. e,u|||b.|un p.|ce .rJ cutput.
e. |ces t|e p.|ce |rc.e.se by t|e |u|| .ncurt c| t|e t.x` || rct, exp|.|r .|y.
|. \cu|J ycu. .rs.e. be J||e.ert || t|e Jen.rJ |c. g.sc||re .e.e pe.|ect|y |re|.st|c`
4. |e |u v.cc|r.t|cr n..ket |.s t|e Jen.rJ .rJ supp|y cu.ves g|ver by t|e |c||c.|rg J.t.. (A|| ,u.rt|t|es
..e |r t|cus.rJs.)
|.|ce pe. v.cc|r.t|cr 10 15 20 25 30
u.rt|ty Jen.rJeJ 90 80 0 60 50
u.rt|ty supp||eJ 50 60 0 80 90
.. ||ct t|e Jen.rJ .rJ supp|y cu.ves, .rJ Jete.n|re t|e e,u|||b.|un p.|ce .rJ ,u.rt|ty.
b. S|c. t|e ..e.s c| ccrsune. .rJ p.cJuce. su.p|us.
CHAP7R 6 MARk7S, MAXIMI7RS, AND IIICINCY 163
c. |c. suppcse t|.t e.c| v.cc|r.t|cr g|ver gere..tes .r exte.r.| bere|t, .s t|cse .|c Jc rct get
v.cc|r.teJ ..e |ess ||ke|y tc get t|e |u .|er ct|e.s Jc get v.cc|r.teJ. As . .esu|t, supp||e.s
.ece|ve . 10 subs|Jy |.cn t|e gcve.rnert |c. e.c| v.cc|re. |c. ex.np|e, || ccrsune.s p.y 10
pe. v.cc|r.t|cr, supp||e.s .ece|ve 20, sc cr|y 10 |.cn ccrsune.s |s .e,u|.eJ tc |rJuce supp||e.s
tc c|e. 0,000 v.cc|r.t|crs pe. ncrt|. ||ct t|e re. supp|y cu.ve.
J. |ete.n|re t|e re. e,u|||b.|un p.|ce .rJ ,u.rt|ty.
e. |ces t|e p.|ce |.|| by t|e |u|| .ncurt c| t|e subs|Jy` || rct, exp|.|r .|y.
|. \|.t |s t|e tct.| .ncurt t|.t ccrsune.s rc. p.y |c. t|e re. e,u|||b.|un ,u.rt|ty c|
v.cc|r.t|crs`
g. \|.t |s t|e tct.| subs|Jy t|.t supp||e.s .ece|ve |.cn t|e gcve.rnert .t t|e re. e,u|||b.|un
,u.rt|ty c| v.cc|r.t|crs`
5. C|ver t|e |c||c.|rg |r|c.n.t|cr .bcut t|e supp|y c| .rJ Jen.rJ |c. .pp|es.
Price per
pound
Quantity demanded (pounds per
month)
Quantity SuppIied (pounds per
month
0.50 12,000 0
0.5 10,000 2,000
1.00 8,000 4,000
1.25 6,000 6,000
1.50 4,000 8,000
1.5 2,000 10,000
2.00 0 12,000
.. |... . g..p| s|n||.. tc ||gu.e 6.12
b. Assun|rg t|e n..ket |c. .pp|es neets t|e e|c|ercy ccrJ|t|cr, s|c. t|e e,u|||b.|un p.|ce .rJ
,u.rt|ty t|.t n.x|n|.es ret bere|t tc scc|ety.
c. |Jert||y t|e ..e. c| ccrsune. su.p|us .rJ t|e ..e. c| p.cJuce. su.p|us.
164 PRINCIPLS OI CONOMICS
1.
2.
3.
ENDNOTES
See |epc.t c| t|e cnn|ttee cr cnne.ce, Sc|erce, .rJ ..rspc.t.t|cr, |||Je
|o|ec||o |o v|o|e| |oo| ^c|, Ser.te |epc.t 106509 (Octcbe. 26, 2000),
\.s||rgtcr, |... |.S. Ccve.rnert |.|rt|rg O|ce, 2000, .rJ V|c|.e| ||c|, \|c|ert
\|Jec C.nes est|ncry, ||c.gc |ty curc||, Octcbe. 30, 2000, .t
|ttp.//......p.c.g/.Jvcc.cy/.|c|v|Jecg.nev|c|erce.pJ|.
V..k |cserbe.g, Success|u| St.te St..teg|es, AJc|escert |e.|t| |e.Je.s||p |c.un,
|ecenbe. 6, 2003, .t |ttp.//......p.c.g/.Jvcc.cy/.|p.cect/A||Success|u|
St.teSt..teg|esV|cserbe.g.pps.
cnncr p.cpe.ty .escu.ces ..e scnet|nes .e|e..eJ tc .s cper .ccess .escu.ces.
CHAP7R 6 MARk7S, MAXIMI7RS, AND IIICINCY 165
166 PRINCIPLS OI CONOMICS
rms
O.g.r|..t|crs t|.t p.cJuce
gccJs .rJ se.v|ces.
| A | | | 8
Production and Cost
S7AR7 UP: S7R7 CLANING AROUND 7H WORLD
|t |s J..r |r S|.rg|.|, ||r.. A|.e.Jy t|cus.rJs c| ||rese ..e cut c|e.r|rg t|e c|tys st.eets. |ey ..e us|rg
b.ccns.
Or t|e ct|e. s|Je c| t|e .c.|J, r|g|t |.||s |r \.s||rgtcr, |.., .|e.e t|e st.eets ..e .|sc be|rg c|e.reJby .
|.rJ|u| c| g|.rt st.eets.eep|rg n.c||res J.|ver by . |.rJ|u| c| .c.ke.s.
|e J||e.erce |r net|cJ |s rct t|e .esu|t c| . g.e.te. krc.|eJge c| ncJe.r tec|rc|cgy |r t|e |r|teJ
St.test|e ||rese krc. pe.|ect|y .e|| |c. tc bu||J st.eets.eep|rg n.c||res. |t |s . p.cJuct|cr Jec|s|cr b.seJ
cr ccsts |r t|e t.c ccurt.|es. |r ||r., .|e.e ..ges ..e .e|.t|ve|y |c., .r ..ny c| .c.ke.s ..neJ .|t| b.ccns |s
t|e |e.st expers|ve ..y tc p.cJuce c|e.r st.eets. |r \.s||rgtcr, .|e.e |.bc. ccsts ..e ||g|, |t n.kes serse tc use
nc.e n.c||re.y .rJ |ess |.bc..
A|| types c| p.cJuct|cr e|c.ts .e,u|.e c|c|ces |r t|e use c| |.ctc.s c| p.cJuct|cr. |r t||s c|.pte. .e ex.n|re
suc| c|c|ces. S|cu|J . gccJ c. se.v|ce be p.cJuceJ us|rg .e|.t|ve|y nc.e |.bc. .rJ |ess c.p|t.|` O. s|cu|J .e|.t
|ve|y nc.e c.p|t.| .rJ |ess |.bc. be useJ` \|.t .bcut t|e use c| r.tu..| .escu.ces`
|r t||s c|.pte. .e see .|y |.ns n.ke t|e p.cJuct|cr c|c|ces t|ey Jc .rJ |c. t|e|. ccsts .|ect t|e|. c|c|ces.
\e .||| .pp|y t|e n..g|r.| Jec|s|cr .u|e tc t|e p.cJuct|cr p.ccess .rJ see |c. t||s .u|e ersu.es t|.t p.cJuct|cr |s
c...|eJ cut .t t|e |c.est ccst pcss|b|e. \e ex.n|re t|e r.tu.e c| p.cJuct|cr .rJ ccsts |r c.Je. tc g.|r . bette. ur
Je.st.rJ|rg c| supp|y. \e t|us s|||t cu. |ccus tc rms, c.g.r|..t|crs t|.t p.cJuce gccJs .rJ se.v|ces. |r p.cJu
c|rg gccJs .rJ se.v|ces, |.ns ccnb|re t|e |.ctc.s c| p.cJuct|cr|.bc., c.p|t.|, .rJ r.tu..| .escu.cestc p.c
Juce v..|cus p.cJucts.
|ccrcn|sts .ssune t|.t |.ns erg.ge |r p.cJuct|cr |r c.Je. tc e..r . p.c|t .rJ t|.t t|ey seek tc n.ke t||s
p.c|t .s |..ge .s pcss|b|e. |.t |s, eccrcn|sts .ssune t|.t |.ns .pp|y t|e n..g|r.| Jec|s|cr .u|e .s t|ey seek tc
n.x|n|.e t|e|. p.c|ts. \|et|e. .e ccrs|Je. t|e cpe..tc. c| . s|ces||re st.rJ .t .r .|.pc.t c. t|e |.n t|.t p.c
Juces .|.p|.res, .e .||| |rJ t|e.e ..e b.s|c .e|.t|crs||ps bet.eer t|e use c| |.ctc.s c| p.cJuct|cr .rJ cutput
|eve|s, .rJ bet.eer cutput |eve|s .rJ ccsts, t|.t .pp|y tc .|| p.cJuct|cr. |e p.cJuct|cr c|c|ces c| |.ns .rJ t|e|.
.sscc|.teJ ccsts ..e .t t|e |curJ.t|cr c| supp|y.
short run
A p|.rr|rg pe.|cJ cve. .||c|
t|e n.r.ge.s c| . |.n nust
ccrs|Je. cre c. nc.e c| t|e|.
|.ctc.s c| p.cJuct|cr .s |xeJ
|r ,u.rt|ty.
xed factor of production
A |.ctc. c| p.cJuct|cr .|cse
,u.rt|ty c.rrct be c|.rgeJ
Ju.|rg . p..t|cu|.. pe.|cJ.
variabIe factor of
production
A |.ctc. c| p.cJuct|cr .|cse
,u.rt|ty c.r be c|.rgeJ
Ju.|rg . p..t|cu|.. pe.|cJ.
Iong run
|e p|.rr|rg pe.|cJ cve.
.||c| . |.n c.r ccrs|Je. .||
|.ctc.s c| p.cJuct|cr .s
v..|.b|e.
production function
|e .e|.t|crs||p bet.eer
|.ctc.s c| p.cJuct|cr .rJ t|e
cutput c| . |.n.
1. PRODUCTION CHOICES AND COSTS: THE SHORT
RUN
L A R N I N G O 8 1 C 7 I V S
1. Understand the terms associated with the short-run production functiontotaI product, aver-
age product, and marginaI productand expIain and iIIustrate how they are reIated to each
other.
2. xpIain the concepts of increasing, diminishing, and negative marginaI returns and expIain the
Iaw of diminishing marginaI returns.
3. Understand the terms associated with costs in the short runtotaI variabIe cost, totaI xed
cost, totaI cost, average variabIe cost, average xed cost, average totaI cost, and marginaI
costand expIain and iIIustrate how they are reIated to each other.
4. xpIain and iIIustrate how the product and cost curves are reIated to each other and to determ-
ine in what ranges on these curves marginaI returns are increasing, diminishing, or negative.
Oui analysis of pioduction and cost begins with a peiiod economists call the shoit iun. The short run
in this micioeconomic context is a planning peiiod ovei which the manageis of a fim must considei
one oi moie of theii factois of pioduction as fxed in quantity. Foi example, a iestauiant may iegaid its
building as a fxed factoi ovei a peiiod of at least the next yeai. It would take at least that much time to
fnd a new building oi to expand oi ieduce the size of its piesent facility. Decisions conceining the op-
eiation of the iestauiant duiing the next yeai must assume the building will iemain unchanged. Othei
factois of pioduction could be changed duiing the yeai, but the size of the building must be iegaided
as a constant.
When the quantity of a factoi of pioduction cannot be changed duiing a paiticulai peiiod, it is
called a nxed factor of production. Foi the iestauiant, its building is a fxed factoi of pioduction foi
at least a yeai. A factoi of pioduction whose quantity can be changed duiing a paiticulai peiiod is
called a variable factor of production; factois such as laboi and food aie examples.
While the manageis of the iestauiant aie making choices conceining its opeiation ovei the next
yeai, they aie also planning foi longei peiiods. Ovei those peiiods, manageis may contemplate altein-
atives such as modifying the building, building a new facility, oi selling the building and leaving the
iestauiant business. The planning peiiod ovei which a fim can considei all factois of pioduction as
vaiiable is called the long run.
At any one time, a fim will be making both shoit-iun and long-iun choices. The manageis may be
planning what to do foi the next few weeks and foi the next few yeais. Theii decisions ovei the next
few weeks aie likely to be shoit-iun choices. Decisions that will afect opeiations ovei the next few
yeais may be long-iun choices, in which manageis can considei changing eveiy aspect of theii opeia-
tions. Oui analysis in this section focuses on the shoit iun. We examine long-iun choices latei in this
chaptei.
1.1 The Shoit-Run Pioduction Function
A fim uses factois of pioduction to pioduce a pioduct. The ielationship between factois of pioduction
and the output of a fim is called a production function Oui fist task is to exploie the natuie of the
pioduction function.
Considei a hypothetical fim, Acme Clothing, a shop that pioduces jackets. Suppose that Acme has
a lease on its building and equipment. Duiing the peiiod of the lease, Acme's capital is its fxed factoi
of pioduction. Acme's vaiiable factois of pioduction include things such as laboi, cloth, and electiicity.
In the analysis that follows, we shall simplify by assuming that laboi is Acme's only vaiiable factoi of
pioduction.
200 PRINCIPLS OI CONOMICS
totaI product curve
C..p| t|.t s|c.s t|e
,u.rt|t|es c| cutput t|.t c.r
be cbt.|reJ |.cn J||e.ert
.ncurts c| . v..|.b|e |.ctc.
c| p.cJuct|cr, .ssun|rg
ct|e. |.ctc.s c| p.cJuct|cr
..e |xeJ.
II GUR 8. 1 Acme CIothing's 7otaI
Product Curve
|e t.b|e g|ves cutput |eve|s pe. J.y |c. Acne
|ct||rg cnp.ry .t v..|cus ,u.rt|t|es c|
|.bc. pe. J.y, .ssun|rg t|e |.ns c.p|t.| |s
|xeJ. |ese v.|ues ..e t|er p|ctteJ g..p||c.||y
.s . tct.| p.cJuct cu.ve.
marginaI product
|e .ncurt by .||c| cutput
.|ses .|t| .r .JJ|t|cr.| ur|t
c| . v..|.b|e |.ctc..
marginaI product of Iabor
|e .ncurt by .||c| cutput
.|ses .|t| .r .JJ|t|cr.| ur|t
c| |.bc..
average product
|e cutput pe. ur|t c|
v..|.b|e |.ctc..
average product of Iabor
|e ..t|c c| cutput tc t|e
runbe. c| ur|ts c| |.bc. (/
|).
7otaI, MarginaI, and Average Products
Figuie 8.1 shows the numbei of jackets Acme can obtain with vaiying amounts of laboi (in this case,
tailois) and its given level of capital. A total product curve shows the quantities of output that can be
obtained fiom difeient amounts of a vaiiable factoi of pioduction, assuming othei factois of pioduc-
tion aie fxed.
Notice what happens to the slope of the total pioduct cuive in Figuie 8.1. Between
0 and 3 units of laboi pei day, the cuive becomes steepei. Between 3 and 7 woikeis, the
cuive continues to slope upwaid, but its slope diminishes. Beyond the seventh tailoi,
pioduction begins to decline and the cuive slopes downwaid.
We measuie the slope of any cuive as the veitical change between two points di-
vided by the hoiizontal change between the same two points. The slope of the total
pioduct cuive foi laboi equals the change in output (AQ) divided by the change in
units of laboi (AL):
Slope of the total pioduct cuive = AQ/ AL
The slope of a total pioduct cuive foi any vaiiable factoi is a measuie of the change in
output associated with a change in the amount of the vaiiable factoi, with the quantities
of all othei factois held constant. The amount by which output iises with an additional
unit of a vaiiable factoi is the marginal product of the vaiiable factoi. Mathematic-
ally, maiginal pioduct is the iatio of the change in output to the change in the amount
of a vaiiable factoi. The marginal product of labor (MP
L
), foi example, is the
amount by which output iises with an additional unit of laboi. It is thus the iatio of the
change in output to the change in the quantity of laboi (AQ/AL), all othei things un-
changed. It is measuied as the slope of the total pioduct cuive foi laboi.
QUA7I ON 8. 1
MP
L
= AQ / AL
In addition we can defne the average product of a vaiiable factoi. It is the output
pei unit of vaiiable factoi. The average product of labor (AP
L
), foi example, is the
iatio of output to the numbei of units of laboi (Q/L).
QUA7I ON 8. 2
AP
L
= Q / L
The concept of aveiage pioduct is often used foi compaiing pioductivity levels ovei time oi in
compaiing pioductivity levels among nations. When you iead in the newspapei that pioductivity is
iising oi falling, oi that pioductivity in the United States is nine times gieatei than pioductivity in Ch-
ina, the iepoit is piobably iefeiiing to some measuie of the aveiage pioduct of laboi.
The total pioduct cuive in Panel (a) of Figuie 8.2 is iepeated fiom Figuie 8.1. Panel (b) shows the
maiginal pioduct and aveiage pioduct cuives. Notice that maiginal pioduct is the slope of the total
pioduct cuive, and that maiginal pioduct iises as the slope of the total pioduct cuive incieases, falls as
the slope of the total pioduct cuive declines, ieaches zeio when the total pioduct cuive achieves its
maximum value, and becomes negative as the total pioduct cuive slopes downwaid. As in othei paits
of this text, maiginal values aie plotted at the midpoint of each inteival. The maiginal pioduct of the
ffth unit of laboi, foi example, is plotted between 4 and 3 units of laboi. Also notice that the maiginal
pioduct cuive inteisects the aveiage pioduct cuive at the maximum point on the aveiage pioduct
cuive. When maiginal pioduct is above aveiage pioduct, aveiage pioduct is iising. When maiginal
pioduct is below aveiage pioduct, aveiage pioduct is falling.
CHAP7R 8 PRODUC7ION AND COS7 201
II GUR 8. 2 Irom 7otaI Product to the Average and MarginaI Product of Labor
|e |.st t.c .c.s c| t|e t.b|e g|ve t|e v.|ues |c. ,u.rt|t|es c| |.bc. .rJ tct.| p.cJuct |.cn ||gu.e 8.1. V..g|r.|
p.cJuct, g|ver |r t|e t||.J .c., |s t|e c|.rge |r cutput .esu|t|rg |.cn . creur|t |rc.e.se |r |.bc.. Ave..ge
p.cJuct, g|ver |r t|e |cu.t| .c., |s cutput pe. ur|t c| |.bc.. |.re| (.) s|c.s t|e tct.| p.cJuct cu.ve. |e s|cpe c|
t|e tct.| p.cJuct cu.ve |s n..g|r.| p.cJuct, .||c| |s p|ctteJ |r |.re| (b). \.|ues |c. n..g|r.| p.cJuct ..e p|ctteJ .t
t|e n|Jpc|rts c| t|e |rte.v.|s. Ave..ge p.cJuct .|ses .rJ |.||s. \|e.e n..g|r.| p.cJuct |s .bcve .ve..ge p.cJuct,
.ve..ge p.cJuct .|ses. \|e.e n..g|r.| p.cJuct |s be|c. .ve..ge p.cJuct, .ve..ge p.cJuct |.||s. |e n..g|r.|
p.cJuct cu.ve |rte.sects t|e .ve..ge p.cJuct cu.ve .t t|e n.x|nun pc|rt cr t|e .ve..ge p.cJuct cu.ve.
202 PRINCIPLS OI CONOMICS
increasing marginaI returns
|e ..rge cve. .||c| e.c|
.JJ|t|cr.| ur|t c| . v..|.b|e
|.ctc. .JJs nc.e tc tct.|
cutput t|.r t|e p.ev|cus
ur|t.
diminishing marginaI
returns
|e ..rge cve. .||c| e.c|
.JJ|t|cr.| ur|t c| . v..|.b|e
|.ctc. .JJs |ess tc tct.|
cutput t|.r t|e p.ev|cus
ur|t.
negative marginaI returns
|e ..rge cve. .||c|
.JJ|t|cr.| ur|ts c| . v..|.b|e
|.ctc. .eJuce tct.| cutput,
g|ver ccrst.rt ,u.rt|t|es c|
.|| ct|e. |.ctc.s.
As a student you can use youi own expeiience to undeistand the ielationship between maiginal and
aveiage values. Youi giade point aveiage (GPA) iepiesents the aveiage giade you have eained in all
youi couise woik so fai. When you take an additional couise, youi giade in that couise iepiesents the
maiginal giade. What happens to youi GPA when you get a giade that is highei than youi pievious av-
eiage: It iises. What happens to youi GPA when you get a giade that is lowei than youi pievious avei-
age: It falls. If youi GPA is a 3.0 and you eain one moie B, youi maiginal giade equals youi GPA and
youi GPA iemains unchanged.
The ielationship between aveiage pioduct and maiginal pioduct is similai. Howevei, unlike youi
couise giades, which may go up and down willy-nilly, maiginal pioduct always iises and then falls, foi
ieasons we will exploie shoitly. As soon as maiginal pioduct falls below aveiage pioduct, the aveiage
pioduct cuive slopes downwaid. While maiginal pioduct is above aveiage pioduct, whethei maiginal
pioduct is incieasing oi decieasing, the aveiage pioduct cuive slopes upwaid.
As we have leained, maximizing behavioi iequiies focusing on making decisions at the maigin.
Foi this ieason, we tuin oui attention now towaid incieasing oui undeistanding of maiginal pioduct.
Increasing, Diminishing, and Negative MarginaI Returns
Adding the fist woikei incieases Acme's output fiom 0 to 1 jacket pei day. The second tailoi adds 2
jackets to total output; the thiid adds 4. The maiginal pioduct goes up because when theie aie moie
woikeis, each one can specialize to a degiee. One woikei might cut the cloth, anothei might sew the
seams, and anothei might sew the buttonholes. Theii incieasing maiginal pioducts aie iefected by the
incieasing slope of the total pioduct cuive ovei the fist 3 units of laboi and by the upwaid slope of the
maiginal pioduct cuive ovei the same iange. The iange ovei which maiginal pioducts aie incieasing is
called the iange of increasing marginal returns. Incieasing maiginal ietuins exist in the context of a
total pioduct cuive foi laboi, so we aie holding the quantities of othei factois constant. Incieasing
maiginal ietuins may occui foi any vaiiable factoi.
The fouith woikei adds less to total output than the thiid; the maiginal pioduct of the fouith
woikei is 2 jackets. The data in Figuie 8.2 show that maiginal pioduct continues to decline aftei the
fouith woikei as moie and moie woikeis aie hiied. The additional woikeis allow even gieatei oppoi-
tunities foi specialization, but because they aie opeiating with a fxed amount of capital, each new
woikei adds less to total output. The ffth tailoi adds only a single jacket to total output. When each ad-
ditional unit of a vaiiable factoi adds less to total output, the fim is expeiiencing diminishing mar-
ginal returns. Ovei the iange of diminishing maiginal ietuins, the maiginal pioduct of the vaiiable
factoi is positive but falling. Once again, we assume that the quantities of all othei factois of pioduction
aie fxed. Diminishing maiginal ietuins may occui foi any vaiiable factoi. Panel (b) shows that Acme
expeiiences diminishing maiginal ietuins between the thiid and seventh woikeis, oi between 7 and 11
jackets pei day.
Aftei the seventh unit of laboi, Acme's fxed plant becomes so ciowded that adding anothei woik-
ei actually ieduces output. When additional units of a vaiiable factoi ieduce total output, given con-
stant quantities of all othei factois, the company expeiiences negative marginal returns. Now the
total pioduct cuive is downwaid sloping, and the maiginal pioduct cuive falls below zeio. Figuie 8.3
shows the ianges of incieasing, diminishing, and negative maiginal ietuins. Cleaily, a fim will nevei
intentionally add so much of a vaiiable factoi of pioduction that it enteis a iange of negative maiginal
ietuins.
CHAP7R 8 PRODUC7ION AND COS7 203
II GUR 8. 3 Increasing MarginaI
Returns, Diminishing MarginaI Returns, and
Negative MarginaI Returns
||s g..p| s|c.s Acnes tct.| p.cJuct cu.ve
|.cn ||gu.e 8.1 .|t| t|e ..rges c| |rc.e.s|rg
n..g|r.| .etu.rs, J|n|r|s||rg n..g|r.| .etu.rs,
.rJ reg.t|ve n..g|r.| .etu.rs n..keJ. Acne
expe.|erces |rc.e.s|rg n..g|r.| .etu.rs
bet.eer 0 .rJ 3 ur|ts c| |.bc. pe. J.y,
J|n|r|s||rg n..g|r.| .etu.rs bet.eer 3 .rJ
ur|ts c| |.bc. pe. J.y, .rJ reg.t|ve n..g|r.|
.etu.rs beycrJ t|e t| ur|t c| |.bc..
Iaw of diminishing
marginaI returns
|e n..g|r.| p.cJuct c| .ry
v..|.b|e |.ctc. c| p.cJuct|cr
.||| evertu.||y Jec||re,
.ssun|rg t|e ,u.rt|t|es c|
ct|e. |.ctc.s c| p.cJuct|cr
..e urc|.rgeJ.
variabIe costs
|e ccsts .sscc|.teJ .|t| t|e
use c| v..|.b|e |.ctc.s c|
p.cJuct|cr.
xed costs
|e ccsts .sscc|.teJ .|t| t|e
use c| |xeJ |.ctc.s c|
p.cJuct|cr.
The idea that the maiginal pioduct of a vaiiable factoi declines ovei some iange is
impoitant enough, and geneial enough, that economists state it as a law. The law of
diminishing marginal returns holds that the maiginal pioduct of any vaiiable factoi
of pioduction will eventually decline, assuming the quantities of othei factois of pio-
duction aie unchanged.
Heads Up!
|t |s e.sy tc ccr|use t|e ccrcept c| J|n|r|s||rg n..g|r.| .etu.rs .|t| t|e |Je. c| reg.t|ve n..
g|r.| .etu.rs. c s.y . |.n |s expe.|erc|rg J|n|r|s||rg n..g|r.| .etu.rs |s rct tc s.y |ts cutput
|s |.|||rg. ||n|r|s||rg n..g|r.| .etu.rs ne.r t|.t t|e n..g|r.| p.cJuct c| . v..|.b|e |.ctc. |s
Jec||r|rg. Output |s st||| |rc.e.s|rg .s t|e v..|.b|e |.ctc. |s |rc.e.seJ, but |t |s |rc.e.s|rg by
sn.||e. .rJ sn.||e. .ncurts. As .e s.. |r ||gu.e 8.2 .rJ ||gu.e 8.3, t|e ..rge c| J|n|r|s||rg
n..g|r.| .etu.rs ..s bet.eer t|e t||.J .rJ severt| .c.ke.s, cve. t||s ..rge c| .c.ke.s, cut
put .cse |.cn tc 11 .ckets. |eg.t|ve n..g|r.| .etu.rs st..teJ .|te. t|e severt| .c.ke..
To see the logic of the law of diminishing maiginal ietuins, imagine a case in which it
does not hold. Say that you have a small plot of land foi a vegetable gaiden, 10 feet by
10 feet in size. The plot itself is a fxed factoi in the pioduction of vegetables. Suppose
you aie able to hold constant all othei factois-watei, sunshine, tempeiatuie, feitilizei,
and seed-and vaiy the amount of laboi devoted to the gaiden. How much food could
the gaiden pioduce: Suppose the maiginal pioduct of laboi kept incieasing oi was con-
stant. Then you could giow an unlimited quantity of food on youi small plot-enough
to feed the entiie woild! You could add an unlimited numbei of woikeis to youi plot
and still inciease output at a constant oi incieasing iate. If you did not get enough out-
put with, say, 300 woikeis, you could use 3 million; the fve-millionth woikei would
add at least as much to total output as the fist. If diminishing maiginal ietuins to laboi
did not occui, the total pioduct cuive would slope upwaid at a constant oi incieasing
iate.
The shape of the total pioduct cuive and the shape of the iesulting maiginal pioduct cuive diawn
in Figuie 8.2 aie typical of any fim foi the shoit iun. Given its fxed factois of pioduction, incieasing
the use of a vaiiable factoi will geneiate incieasing maiginal ietuins at fist; the total pioduct cuive foi
the vaiiable factoi becomes steepei and the maiginal pioduct iises. The oppoitunity to gain fiom in-
cieased specialization in the use of the vaiiable factoi accounts foi this iange of incieasing maiginal ie-
tuins. Eventually, though, diminishing ietuins will set in. The total pioduct cuive will become fattei,
and the maiginal pioduct cuive will fall.
1.2 Costs in the Shoit Run
A fim's costs of pioduction depend on the quantities and piices of its factois of pioduction. Because
we expect a fim's output to vaiy with the fim's use of laboi in a specifc way, we can also expect the
fim's costs to vaiy with its output in a specifc way. We shall put oui infoimation about Acme's
pioduct cuives to woik to discovei how a fim's costs vaiy with its level of output.
We distinguish between the costs associated with the use of vaiiable factois of pioduction, which
aie called variable costs, and the costs associated with the use of fxed factois of pioduction, which aie
called nxed costs. Foi most fims, vaiiable costs includes costs foi iaw mateiials, salaiies of pioduc-
tion woikeis, and utilities. The salaiies of top management may be fxed costs; any chaiges set by con-
tiact ovei a peiiod of time, such as Acme's one-yeai lease on its building and equipment, aie likely to
be fxed costs. A teim commonly used foi fxed costs is overhead. Notice that fxed costs exist only in
the shoit iun. In the long iun, the quantities of all factois of pioduction aie vaiiable, so that all long-
iun costs aie vaiiable.
204 PRINCIPLS OI CONOMICS
totaI variabIe cost
cst t|.t v..|es .|t| t|e |eve|
c| cutput.
totaI xed cost
cst t|.t Jces rct v..y .|t|
cutput.
totaI cost
|e sun c| tct.| v..|.b|e ccst
.rJ tct.| |xeJ ccst.
Total variable cost (TVC) is cost that vaiies with the level of output. Total nxed cost (TFC) is
cost that does not vaiy with output. Total cost (TC) is the sum of total vaiiable cost and total fxed
cost:
QUA7I ON 8. 3
TVC + TFC = TC
Irom 7otaI Production to 7otaI Cost
Next we illustiate the ielationship between Acme's total pioduct cuive and its total costs. Acme can
vaiy the quantity of laboi it uses each day, so the cost of this laboi is a vaiiable cost. We assume capital
is a fxed factoi of pioduction in the shoit iun, so its cost is a fxed cost.
Suppose that Acme pays a wage of $100 pei woikei pei day. If laboi is the only vaiiable factoi,
Acme's total vaiiable costs pei day amount to $100 times the numbei of woikeis it employs. We can
use the infoimation given by the total pioduct cuive, togethei with the wage, to compute Acme's total
vaiiable costs.
We know fiom Figuie 8.1 that Acme iequiies 1 woikei woiking 1 day to pioduce 1 jacket. The
total vaiiable cost of a jacket thus equals $100. Thiee units of laboi pioduce 7 jackets pei day; the total
vaiiable cost of 7 jackets equals $300. Figuie 8.4 shows Acme's total vaiiable costs foi pioducing each
of the output levels given in Figuie 8.1
Figuie 8.4 gives us costs foi seveial quantities of jackets, but we need a bit moie detail. We know,
foi example, that 7 jackets have a total vaiiable cost of $300. What is the total vaiiable cost of 6 jackets:
II GUR 8. 4 Computing VariabIe Costs
|e pc|rts s|c.r g|ve t|e v..|.b|e ccsts c| p.cJuc|rg t|e ,u.rt|t|es c| .ckets g|ver |r t|e tct.| p.cJuct cu.ve |r
||gu.e 8.1 .rJ ||gu.e 8.2. Suppcse Acnes .c.ke.s e..r 100 pe. J.y. || Acne p.cJuces 0 .ckets, |t .||| use rc
|.bc.|ts v..|.b|e ccst t|us e,u.|s 0 (|c|rt A). |.cJuc|rg .ckets .e,u|.es 3 ur|ts c| |.bc., Acnes v..|.b|e ccst
e,u.|s 300 (|c|rt |).
We can estimate total vaiiable costs foi othei quantities of jackets by inspecting the total pioduct cuive
in Figuie 8.1. Reading ovei fiom a quantity of 6 jackets to the total pioduct cuive and then down sug-
gests that the Acme needs about 2.8 units of laboi to pioduce 6 jackets pei day. Acme needs 2 full-time
and 1 pait-time tailois to pioduce 6 jackets. Figuie 8.3 gives the piecise total vaiiable costs foi quantit-
ies of jackets ianging fiom 0 to 11 pei day. The numbeis in boldface type aie taken fiom Figuie 8.4; the
othei numbeis aie estimates we have assigned to pioduce a total vaiiable cost cuive that is consistent
CHAP7R 8 PRODUC7ION AND COS7 205
with oui total pioduct cuive. You should, howevei, be ceitain that you undeistand how the numbeis in
boldface type weie found.
II GUR 8. 5 7he 7otaI VariabIe Cost Curve
ct.| v..|.b|e ccsts |c. cutput |eve|s s|c.r |r Acnes tct.| p.cJuct cu.ve .e.e s|c.r |r ||gu.e 8.4. c ccnp|ete
t|e tct.| v..|.b|e ccst cu.ve, .e reeJ tc krc. t|e v..|.b|e ccst |c. e.c| |eve| c| cutput |.cn 0 tc 11 .ckets pe.
J.y. |e v..|.b|e ccsts .rJ ,u.rt|t|es c| |.bc. g|ver |r ||gu.e 8.4 ..e s|c.r |r bc|J|.ce |r t|e t.b|e |e.e .rJ .|t|
b|.ck Jcts |r t|e g..p|. |e .en.|r|rg v.|ues .e.e est|n.teJ |.cn t|e tct.| p.cJuct cu.ve |r ||gu.e 8.1 .rJ ||gu.e
8.2. |c. ex.np|e, p.cJuc|rg 6 .ckets .e,u|.es 2.8 .c.ke.s, |c. . v..|.b|e ccst c| 280.
Suppose Acme's piesent plant, including the building and equipment, is the equivalent of 20 units of
capital. Acme has signed a long-teim lease foi these 20 units of capital at a cost of $200 pei day. In the
shoit iun, Acme cannot inciease oi deciease its quantity of capital-it must pay the $200 pei day no
mattei what it does. Even if the fim cuts pioduction to zeio, it must still pay $200 pei day in the shoit
iun.
Acme's total cost is its total fxed cost of $200 plus its total vaiiable cost. We add $200 to the total
vaiiable cost cuive in Figuie 8.3 to get the total cost cuive shown in Figuie 8.6.
206 PRINCIPLS OI CONOMICS
II GUR 8. 6 Irom VariabIe Cost to 7otaI Cost
\e .JJ tct.| |xeJ ccst tc t|e tct.| v..|.b|e ccst tc cbt.|r tct.| ccst. |r t||s c.se, Acnes tct.| |xeJ ccst e,u.|s 200
pe. J.y.
Notice something impoitant about the shapes of the total cost and total vaiiable cost cuives in Figuie
8.6. The total cost cuive, foi example, staits at $200 when Acme pioduces 0 jackets-that is its total
fxed cost. The cuive iises, but at a decieasing iate, up to the seventh jacket. Beyond the seventh jacket,
the cuive becomes steepei and steepei. The slope of the total vaiiable cost cuive behaves in piecisely
the same way.
Recall that Acme expeiienced incieasing maiginal ietuins to laboi foi the fist thiee units of
laboi-oi the fist seven jackets. Up to the thiid woikei, each additional woikei added moie and moie
to Acme's output. Ovei the iange of incieasing maiginal ietuins, each additional jacket iequiies less
and less additional laboi. The fist jacket iequiied one tailoi; the second iequiied the addition of only a
pait-time tailoi; the thiid iequiied only that Acme boost that pait-time tailoi's houis to a full day. Up
to the seventh jacket, each additional jacket iequiies less and less additional laboi, and thus costs iise at
a decieasing iate; the total cost and total vaiiable cost cuives become fattei ovei the iange of incieasing
maiginal ietuins.
Acme expeiiences diminishing maiginal ietuins beyond the thiid unit of laboi-oi the seventh
jacket. Notice that the total cost and total vaiiable cost cuives become steepei and steepei beyond this
level of output. In the iange of diminishing maiginal ietuins, each additional unit of a factoi adds less
and less to total output. That means each additional unit of output iequiies laigei and laigei incieases
in the vaiiable factoi, and laigei and laigei incieases in costs.
CHAP7R 8 PRODUC7ION AND COS7 207
average totaI cost
ct.| ccst J|v|JeJ by
,u.rt|ty, |t |s t|e |.ns tct.|
ccst pe. ur|t c| cutput.
average variabIe cost
ct.| v..|.b|e ccst J|v|JeJ by
,u.rt|ty, |t |s t|e |.ns tct.|
v..|.b|e ccst pe. ur|t c|
cutput.
average xed cost
ct.| |xeJ ccst J|v|JeJ by
,u.rt|ty.
MarginaI and Average Costs
Maiginal and aveiage cost cuives, which will play an impoitant iole in the analysis of the fim, can be
deiived fiom the total cost cuive. Marginal cost shows the additional cost of each additional unit of
output a fim pioduces. This is a specifc application of the geneial concept of maiginal cost piesented
eailiei. Given the maiginal decision iule's focus on evaluating choices at the maigin, the maiginal cost
cuive takes on enoimous impoitance in the analysis of a fim's choices. The second cuive we shall de-
iive shows the fim's aveiage total cost at each level of output. Average total cost (ATC) is total cost
divided by quantity; it is the fim's total cost pei unit of output:
QUA7I ON 8. 4
ATC = TC/ Q
We shall also discuss average variable costs (AVC), which is the fim's vaiiable cost pei unit of
output; it is total vaiiable cost divided by quantity:
QUA7I ON 8. 5
AVC = TVC/ Q
We aie still assessing the choices facing the fim in the shoit iun, so we assume that at least one
factoi of pioduction is fxed. Finally, we will discuss average nxed cost (AFC), which is total fxed
cost divided by quantity:
QUA7I ON 8. 6
AFC = TFC/ Q
Maiginal cost (MC) is the amount by which total cost iises with an additional unit of output. It is
the iatio of the change in total cost to the change in the quantity of output:
QUA7I ON 8. 7
MC = ATC/ AQ
It equals the slope of the total cost cuive. Figuie 8.7 shows the same total cost cuive that was
piesented in Figuie 8.6. This time the slopes of the total cost cuive aie shown; these slopes equal the
maiginal cost of each additional unit of output. Foi example, incieasing output fiom 6 to 7 units (
AQ = 1 ) incieases total cost fiom $480 to $300 (ATC = $20 ). The seventh unit thus has a maiginal
cost of $20 (ATC/ AQ = $20 / 1 = $20 ). Maiginal cost falls ovei the iange of incieasing maiginal ie-
tuins and iises ovei the iange of diminishing maiginal ietuins.
Heads Up!
|ct|ce t|.t t|e v..|cus ccst cu.ves ..e J...r .|t| t|e ,u.rt|ty c| cutput cr t|e |c.|.crt.| .x|s. |e v..|cus
p.cJuct cu.ves ..e J...r .|t| ,u.rt|ty c| . |.ctc. c| p.cJuct|cr cr t|e |c.|.crt.| .x|s. |e .e.scr |s t|.t t|e
t.c sets c| cu.ves ne.su.e J||e.ert .e|.t|crs||ps. |.cJuct cu.ves s|c. t|e .e|.t|crs||p bet.eer cutput .rJ
t|e ,u.rt|ty c| . |.ctc., t|ey t|e.e|c.e |.ve t|e |.ctc. ,u.rt|ty cr t|e |c.|.crt.| .x|s. cst cu.ves s|c. |c.
ccsts v..y .|t| cutput .rJ t|us |.ve cutput cr t|e |c.|.crt.| .x|s.
208 PRINCIPLS OI CONOMICS
II GUR 8. 7 7otaI Cost and MarginaI Cost
V..g|r.| ccst |r |.re| (b) |s t|e s|cpe c| t|e tct.| ccst cu.ve |r |.re| (.).
Figuie 8.8 shows the computation of Acme's shoit-iun aveiage total cost, aveiage vaiiable cost, and av-
eiage fxed cost and giaphs of these values. Notice that the cuives foi shoit-iun aveiage total cost and
aveiage vaiiable cost fall, then iise. We say that these cost cuives aie U-shaped. Aveiage fxed cost
keeps falling as output incieases. This is because the fxed costs aie spiead out moie and moie as
CHAP7R 8 PRODUC7ION AND COS7 209
output expands; by defnition, they do not vaiy as laboi is added. Since aveiage total cost (ATC) is the
sum of aveiage vaiiable cost (AVC) and aveiage fxed cost (AFC), i.e.,
QUA7I ON 8. 8
AVC + AFC = ATC
the distance between the ATC and AVC cuives keeps getting smallei and smallei as the fim
spieads its oveihead costs ovei moie and moie output.
II GUR 8. 8 MarginaI Cost, Average Iixed Cost, Average VariabIe Cost, and Average 7otaI Cost in the
Short Run
ct.| ccst |gu.es |c. Acne |ct||rg ..e t.ker |.cn ||gu.e 8.. |e ct|e. v.|ues ..e Je.|veJ |.cn t|ese. Ave..ge
tct.| ccst (^) e,u.|s tct.| ccst J|v|JeJ by ,u.rt|ty p.cJuceJ, |t .|sc e,u.|s t|e sun c| t|e .ve..ge |xeJ ccst
(^|) .rJ .ve..ge v..|.b|e ccst (^v) (except|crs |r t.b|e ..e Jue tc .curJ|rg tc t|e re..est Jc||..), .ve..ge
v..|.b|e ccst |s v..|.b|e ccst J|v|JeJ by ,u.rt|ty p.cJuceJ. |e n..g|r.| ccst (/) cu.ve (|.cn ||gu.e 8.) |rte.sects
t|e ^ .rJ ^v cu.ves .t t|e |c.est pc|rts cr bct| cu.ves. |e ^| cu.ve |.||s .s ,u.rt|ty |rc.e.ses.
Figuie 8.8 includes the maiginal cost data and the maiginal cost cuive fiom Figuie 8.7. The maiginal
cost cuive inteisects the aveiage total cost and aveiage vaiiable cost cuives at theii lowest points. When
maiginal cost is below aveiage total cost oi aveiage vaiiable cost, the aveiage total and aveiage vaiiable
cost cuives slope downwaid. When maiginal cost is gieatei than shoit-iun aveiage total cost oi aveiage
vaiiable cost, these aveiage cost cuives slope upwaid. The logic behind the ielationship between mai-
ginal cost and aveiage total and vaiiable costs is the same as it is foi the ielationship between maiginal
pioduct and aveiage pioduct.
We tuin next in this chaptei to an examination of pioduction and cost in the long iun, a planning
peiiod in which the fim can considei changing the quantities of any oi all factois.
210 PRINCIPLS OI CONOMICS
k Y 7 A k A W A Y S
< |r |.re| (.), t|e tct.| p.cJuct cu.ve |c. . v..|.b|e |.ctc. |r t|e s|c.t .ur s|c.s t|.t t|e |.n expe.|erces
|rc.e.s|rg n..g|r.| .etu.rs |.cn .e.c tc |
o
ur|ts c| t|e v..|.b|e |.ctc. (.e.c tc (
o
ur|ts c| cutput),
J|n|r|s||rg n..g|r.| .etu.rs |.cn |
o
tc |

((
o
tc (

ur|ts c| cutput), .rJ reg.t|ve n..g|r.| .etu.rs


beycrJ |

ur|ts c| t|e v..|.b|e |.ctc..


< |.re| (b) s|c.s t|.t n..g|r.| p.cJuct .|ses cve. t|e ..rge c| |rc.e.s|rg n..g|r.| .etu.rs, |.||s cve. t|e
..rge c| J|n|r|s||rg n..g|r.| .etu.rs, .rJ beccnes reg.t|ve cve. t|e ..rge c| reg.t|ve n..g|r.| .etu.rs.
Ave..ge p.cJuct .|ses .|er n..g|r.| p.cJuct |s .bcve |t .rJ |.||s .|er n..g|r.| p.cJuct |s be|c. |t.
< |r |.re| (c), tct.| ccst .|ses .t . Jec.e.s|rg ..te cve. t|e ..rge c| cutput |.cn .e.c tc (
o
||s ..s t|e
..rge c| cutput t|.t ..s s|c.r |r |.re| (.) tc ex||b|t |rc.e.s|rg n..g|r.| .etu.rs. beycrJ (
o
, t|e ..rge c|
J|n|r|s||rg n..g|r.| .etu.rs, tct.| ccst .|ses .t .r |rc.e.s|rg ..te. |e tct.| ccst .t .e.c ur|ts c| cutput
(s|c.r .s t|e |rte.cept cr t|e ve.t|c.| .x|s) |s tct.| |xeJ ccst.
< |.re| (J) s|c.s t|.t n..g|r.| ccst |.||s cve. t|e ..rge c| |rc.e.s|rg n..g|r.| .etu.rs, t|er .|ses cve. t|e
..rge c| J|n|r|s||rg n..g|r.| .etu.rs. |e n..g|r.| ccst cu.ve |rte.sects t|e .ve..ge tct.| ccst .rJ
.ve..ge v..|.b|e ccst cu.ves .t t|e|. |c.est pc|rts. Ave..ge |xeJ ccst |.||s .s cutput |rc.e.ses. |cte t|.t
.ve..ge tct.| ccst e,u.|s .ve..ge v..|.b|e ccst p|us .ve..ge |xeJ ccst.
< Assun|rg |.bc. |s t|e v..|.b|e |.ctc. c| p.cJuct|cr, t|e |c||c.|rg Je|r|t|crs .rJ .e|.t|crs Jesc.|be
p.cJuct|cr .rJ ccst |r t|e s|c.t .ur.
MP
L
= AQ / AL
AP
L
= Q / L
TVC + TFC = TC
ATC = TC/ Q
AVC = TVC/ Q
AFC = TFC/ Q
MC = ATC/ AQ
CHAP7R 8 PRODUC7ION AND COS7 211
7 R Y I 7 !
1. Suppcse Acne gets scne re. e,u|pnert |c. p.cJuc|rg .ckets. |e t.b|e be|c. g|ves |ts re.
p.cJuct|cr |urct|cr. cnpute n..g|r.| p.cJuct .rJ .ve..ge p.cJuct .rJ ||| |r t|e bcttcn t.c .c.s c|
t|e t.b|e. |e|e..|rg tc ||gu.e 8.2, J... . g..p| s|c.|rg Acnes re. tct.| p.cJuct cu.ve. Or . seccrJ
g..p|, be|c. t|e cre s|c.|rg t|e tct.| p.cJuct cu.ve ycu J.e., sketc| t|e n..g|r.| .rJ .ve..ge
p.cJuct cu.ves. |enenbe. tc p|ct n..g|r.| p.cJuct .t t|e n|Jpc|rt bet.eer e.c| |rput |eve|. Or bct|
g..p|s, s|.Je t|e .eg|crs .|e.e Acne expe.|erces |rc.e.s|rg n..g|r.| .etu.rs, J|n|r|s||rg n..g|r.|
.etu.rs, .rJ reg.t|ve n..g|r.| .etu.rs.
2. |... t|e pc|rts s|c.|rg tct.| v..|.b|e ccst .t J.||y cutputs c| 0, 1, 3, , 9, 10, .rJ 11 .ckets pe. J.y .|er
Acne |.ceJ . ..ge c| 100 pe. J.y. (|se ||gu.e 8.5 .s . ncJe|.) Sketc| t|e tct.| v..|.b|e ccst cu.ve .s
s|c.r |r ||gu.e 8.4. |c. suppcse t|.t t|e ..ge .|ses tc 125 pe. J.y. Or t|e s.ne g..p|, s|c. t|e re.
pc|rts .rJ sketc| t|e re. tct.| v..|.b|e ccst cu.ve. |xp|.|r .|.t |.s |.ppereJ. \|.t .||| |.pper tc
Acnes n..g|r.| ccst cu.ve` |ts .ve..ge tct.|, .ve..ge v..|.b|e, .rJ .ve..ge |xeJ ccst cu.ves` |xp|.|r.
Case in Point: 7he Production of Iitness
|c. nuc| s|cu|J .r .t||ete t..|r`
Spc.ts p|ys|c|cg|sts c|ter ne.su.e t|e tct.| p.cJuct c| t..|r|rg .s t|e |rc.e.se |r .r .t||etes .e.cb|c c.p.
c|tyt|e c.p.c|ty tc .bsc.b cxyger |rtc t|e b|ccJst.e.n. Ar .t||ete c.r be t|cug|t c| .s p.cJuc|rg .e.cb|c
c.p.c|ty us|rg . |xeJ |.ctc. (||s c. |e. r.tu..| c.p.c|ty) .rJ . v..|.b|e |rput (exe.c|se). |e c|..t s|c.s |c.
t||s .e.cb|c c.p.c|ty v..|es .|t| t|e runbe. c| .c.kcuts pe. .eek. |e cu.ve |.s . s|.pe ve.y nuc| ||ke .
tct.| p.cJuct cu.ve.||c|, .|te. .||, |s p.ec|se|y .|.t |t |s.
|e J.t. suggest t|.t .r .t||ete expe.|erces |rc.e.s|rg n..g|r.| .etu.rs |.cn exe.c|se |c. t|e |.st t|.ee J.ys
c| t..|r|rg e.c| .eek, |rJeeJ, cve. |.|| t|e tct.| g.|r |r .e.cb|c c.p.c|ty pcss|b|e |s .c||eveJ. A pe.scr c.r
beccne ever nc.e |t by exe.c|s|rg nc.e, but t|e g.|rs beccne sn.||e. .|t| e.c| .JJeJ J.y c| t..|r|rg. |e
|.. c| J|n|r|s||rg n..g|r.| .etu.rs .pp||es tc t..|r|rg.
|e |rc.e.se |r |tress t|.t .esu|ts |.cn t|e s|xt| .rJ severt| .c.kcuts e.c| .eek |s sn.||. StuJ|es .|sc s|c.
t|.t t|e ccsts c| J.||y t..|r|rg, |r te.ns c| |rc.e.seJ ||ke|||ccJ c| |ru.y, ..e ||g|. V.ry t..|re.s .rJ cc.c|es
rc. .eccnnerJ t|.t .t||etes.t .|| |eve|s c| ccnpet|t|crt.ke . J.y c. t.c c| e.c| .eek.
.ooce .e| Oo||o.o,. Oo||o.o, |oo| o |o| (|o||o. ^ .|e||e |o||co||o. 2002:. 56
212 PRINCIPLS OI CONOMICS
A N S W R S 7 O 7 R Y I 7 ! P R O 8 L M S
1. |e |rc.e.seJ ..ge .||| s|||t t|e tct.| v..|.b|e ccst cu.ve up...J, t|e c|J .rJ re. pc|rts .rJ t|e
cc..espcrJ|rg cu.ves ..e s|c.r .t t|e .|g|t.
2. |e tct.| v..|.b|e ccst cu.ve |.s s|||teJ up...J bec.use t|e ccst c| |.bc., Acnes v..|.b|e |.ctc., |.s
|rc.e.seJ. |e n..g|r.| ccst cu.ve s|c.s t|e .JJ|t|cr.| ccst c| e.c| .JJ|t|cr.| ur|t c| cutput . |.n
p.cJuces. bec.use .r |rc.e.se |r cutput .e,u|.es nc.e |.bc., .rJ bec.use |.bc. rc. ccsts nc.e, t|e
n..g|r.| ccst cu.ve .||| s|||t up...J. |e |rc.e.se |r tct.| v..|.b|e ccst .||| |rc.e.se tct.| ccst, .ve..ge
tct.| .rJ .ve..ge v..|.b|e ccsts .||| .|se .s .e||. Ave..ge |xeJ ccst .||| rct c|.rge.
CHAP7R 8 PRODUC7ION AND COS7 213
2. PRODUCTION CHOICES AND COSTS: THE LONG RUN
L A R N I N G O 8 1 C 7 I V S
1. AppIy the marginaI decision ruIe to expIain how a rm chooses its mix of factors of production
in the Iong run.
2. Dene the Iong-run average cost curve and expIain how it reIates to economies and dis-
economies or scaIe.
In a long-iun planning peispective, a fim can considei changing the quantities of all its factois of pio-
duction. That gives the fim oppoitunities it does not have in the shoit iun. Fiist, the fim can select the
mix of factois it wishes to use. Should it choose a pioduction piocess with lots of laboi and not much
capital, like the stieet sweepeis in China: Oi should it select a piocess that uses a gieat deal of capital
and ielatively little laboi, like stieet sweepeis in the United States: The second thing the fim can select
is the scale (oi oveiall size) of its opeiations. In the shoit iun, a fim can inciease output only by in-
cieasing its use of a vaiiable factoi. But in the long iun, all factois aie vaiiable, so the fim can expand
the use of all of its factois of pioduction. The question facing the fim in the long iun is: How much of
an expansion oi contiaction in the scale of its opeiations should it undeitake: Alteinatively, it could
choose to go out of business.
In its long-iun planning, the fim not only iegaids all factois as vaiiable, but it iegaids all costs as
vaiiable as well. Theie aie no fxed costs in the long iun. Because all costs aie vaiiable, the stiuctuie of
costs in the long iun difeis somewhat fiom what we saw in the shoit iun.
2.1 Choosing the Factoi Mix
How shall a fim decide what mix of capital, laboi, and othei factois to use: We can apply the maiginal
decision iule to answei this question.
Suppose a fim uses capital and laboi to pioduce a paiticulai good. It must deteimine how to pio-
duce the good and the quantity it should pioduce. We addiess the question of how much the fim
should pioduce in subsequent chapteis, but ceitainly the fim will want to pioduce whatevei quantity it
chooses at as low a cost as possible. Anothei way of putting that goal is to say that the fim seeks the
maximum output possible at eveiy level of total cost.
At any level of total cost, the fim can vaiy its factoi mix. It could, foi example, substitute laboi foi
capital in a way that leaves its total cost unchanged. In teims of the maiginal decision iule, we can
think of the fim as consideiing whethei to spend an additional $1 on one factoi, hence $1 less on an-
othei. The maiginal decision iule says that a fim will shift spending among factois as long as the mai-
ginal beneft of such a shift exceeds the maiginal cost.
What is the maiginal beneft, say, of an additional $1 spent on capital: An additional unit of capital
pioduces the maiginal pioduct of capital. To deteimine the maiginal beneft of $1 spent on capital, we
divide capital's maiginal pioduct by its piice: MP
K
/P
K
. The piice of capital is the ient" paid foi the use
of a unit of capital foi a given peiiod. If the fim alieady owns the capital, then this ient is an oppoitun-
ity cost; it iepiesents the ietuin the fim could get by ienting the capital to anothei usei oi by selling it
and eaining inteiest on the money thus gained.
If capital and laboi aie the only factois, then spending an additional $1 on capital while holding
total cost constant means taking $1 out of laboi. The cost of that action will be the output lost fiom
cutting back $1 woith of laboi. That cost equals the iatio of the maiginal pioduct of laboi to the piice
of laboi, MP
L
/P
L
, wheie the piice of laboi is the wage.
Suppose that a fim's maiginal pioduct of laboi is 13 and the piice of laboi is $3 pei unit; the fim
gains 3 units of output by spending an additional $1 on laboi. Suppose fuithei that the maiginal
pioduct of capital is 30 and the piice of capital is $30 pei unit, so the fim would lose 1 unit of output
by spending $1 less on capital.
MP
L
P
L
>
MP
K
P
K
13
3
>
30
30
The fim achieves a net gain of 2 units of output, without any change in cost, by tiansfeiiing $1 fiom
capital to laboi. It will continue to tiansfei funds fiom capital to laboi as long as it gains moie output
fiom the additional laboi than it loses in output by ieducing capital. As the fim shifts spending in this
214 PRINCIPLS OI CONOMICS
capitaI intensive
S|tu.t|cr |r .||c| . |.n |.s
. ||g| ..t|c c| c.p|t.| tc |.bc..
Iabor intensive
S|tu.t|cr |r .||c| . |.n |.s
. |c. ..t|c c| |.bc. tc c.p|t.|.
fashion, howevei, the maiginal pioduct of laboi will fall and the maiginal pioduct of capital will iise.
At some point, the iatios of maiginal pioduct to piice will be equal foi the two factois. At this point,
the fim will obtain the maximum output possible foi a given total cost:
QUA7I ON 8. 9
MP
L
P
L
=
MP
K
P
K
Suppose that a fim that uses capital and laboi is satisfying Equation 8.9 when suddenly the piice
of laboi iises. At the cuiient usage levels of the factois, a highei piice of laboi (P
L
) loweis the iatio of
the maiginal pioduct of laboi to the piice of laboi:
MP
L
P
L
'
<
MP
K
P
K
The fim will shift funds out of laboi and into capital. It will continue to shift fiom laboi to capital until
the iatios of maiginal pioduct to piice aie equal foi the two factois. In geneial, a pioft-maximizing
fim will seek a combination of factois such that
QUA7I ON 8. 10
MP
1
P
1
=
MP
2
P
2
= ... =
MP
n
P
n
When a fim satisfes the condition given in Equation 8.10 foi emcient use, it pioduces the gieatest
possible output foi a given cost. To put it anothei way, the fim achieves the lowest possible cost foi a
given level of output.
As the piice of laboi iises, the fim will shift to a factoi mix that uses ielatively moie capital and ie-
latively less laboi. As a fim incieases its iatio of capital to laboi, we say it is becoming moie capital
intensive. A lowei piice foi laboi will lead the fim to use ielatively moie laboi and less capital, iedu-
cing its iatio of capital to laboi. As a fim ieduces its iatio of capital to laboi, we say it is becoming
moie labor intensive. The notions of laboi-intensive and capital-intensive pioduction aie puiely iel-
ative; they imply only that a fim has a highei oi lowei iatio of capital to laboi.
Sometimes economists speak of laboi-intensive veisus capital-intensive countiies in the same
mannei. One implication of the maiginal decision iule foi factoi use is that fims in countiies wheie
laboi is ielatively expensive, such as the United States, will use capital-intensive pioduction methods.
Less developed countiies, wheie laboi is ielatively cheap, will use laboi-intensive methods.
Now that we undeistand how to apply the maiginal decision iule to the pioblem of choosing the
mix of factois, we can answei the question that began this chaptei: Why does the United States employ
a capital-intensive pioduction piocess to clean stieets while China chooses a laboi-intensive piocess:
Given that the same technology-know-how-is available, both countiies could, aftei all, use the same
pioduction piocess. Suppose foi a moment that the ielative piices of laboi and capital aie the same in
China and the United States. In that case, China and the United States can be expected to use the same
method to clean stieets. But the piice of laboi ielative to the piice of capital is, in fact, fai lowei in Ch-
ina than in the United States. A lowei ielative piice foi laboi incieases the iatio of the maiginal
pioduct of laboi to its piice, making it emcient to substitute laboi foi capital. China thus fnds it cheap-
ei to clean stieets with lots of people using biooms, while the United States fnds it emcient to clean
stieets with laige machines and ielatively less laboi.
Maquiladoras, plants in Mexico wheie piocessing is done using low-cost woikeis and laboi-in-
tensive methods, allow some U.S. fims to have it both ways. They complete pait of the pioduction pio-
cess in the United States, using capital-intensive methods. They then ship the unfnished goods to ma-
quiladoras. Foi example, many U.S. clothing manufactuieis pioduce cloth at U.S. plants on laige high-
speed looms. They then ship the cloth to Mexico, wheie it is fashioned into clothing by woikeis using
sewing machines. Anothei example is plastic injection molding, which iequiies highly skilled laboi and
is made in the U.S. The paits aie molded in Texas boidei towns and aie then shipped to maquiladoras
and used in cais and computeis. The iesulting items aie shipped back to the United States, labeled
Assembled in Mexico fiom U.S. mateiials." Oveiall maquiladoias impoit 97 of the components they
use, of which 80 to 83 come fiom the U.S.
The maquiladoras have been a boon to woikeis in Mexico, who enjoy a highei demand foi theii
seivices and ieceive highei wages as a iesult. The system also benefts the U.S. fims that paiticipate and
U.S. consumeis who obtain less expensive goods than they would otheiwise. It woiks because difeient
factoi piices imply difeient mixes of laboi and capital. Companies aie able to caiiy out the capital-in-
tensive side of the pioduction piocess in the United States and the laboi-intensive side in Mexico.
[1]
CHAP7R 8 PRODUC7ION AND COS7 215
Iong-run average cost
curve
C..p| s|c.|rg t|e |.ns
|c.est ccst pe. ur|t .t e.c|
|eve| c| cutput, .ssun|rg t|.t
.|| |.ctc.s c| p.cJuct|cr ..e
v..|.b|e.
2.2 Costs in the Long Run
As in the shoit iun, costs in the long iun depend on the fim's level of output, the costs of factois, and
the quantities of factois needed foi each level of output. The chief difeience between long- and shoit-
iun costs is theie aie no fxed factois in the long iun. Theie aie thus no fxed costs. All costs aie vaii-
able, so we do not distinguish between total vaiiable cost and total cost in the long iun: total cost is
total vaiiable cost.
The long-run average cost (IRAC) curve shows the fim's lowest cost pei unit at each level of
output, assuming that all factois of pioduction aie vaiiable. The LRAC cuive assumes that the fim has
chosen the optimal factoi mix, as desciibed in the pievious section, foi pioducing any level of output.
The costs it shows aie theiefoie the lowest costs possible foi each level of output. It is impoitant to
note, howevei, that this does not mean that the minimum points of each shoit-iun ATC cuives lie on
the LRAC cuive. This ciitical point is explained in the next paiagiaph and expanded upon even fuithei
in the next section.
Figuie 8.14 shows how a fim's LRAC cuive is deiived. Suppose Lifetime Disc Co. pioduces com-
pact discs (CDs) using capital and laboi. We have alieady seen how a fim's aveiage total cost cuive can
be diawn in the shoit iun foi a given quantity of a paiticulai factoi of pioduction, such as capital. In
the shoit iun, Lifetime Disc might be limited to opeiating with a given amount of capital; it would face
one of the shoit-iun aveiage total cost cuives shown in Figuie 8.14. If it has 30 units of capital, foi ex-
ample, its aveiage total cost cuive is ATC
30
. In the long iun the fim can examine the aveiage total cost
cuives associated with vaiying levels of capital. Foui possible shoit-iun aveiage total cost cuives foi
Lifetime Disc aie shown in Figuie 8.14 foi quantities of capital of 20, 30, 40, and 30 units. The ielevant
cuives aie labeled ATC
20
, ATC
30
, ATC
40
, and ATC
30
iespectively. The LRAC cuive is deiived fiom this
set of shoit-iun cuives by fnding the lowest aveiage total cost associated with each level of output.
Again, notice that the U-shaped LRAC cuive is an envelope cuive that suiiounds the vaiious shoit-iun
ATC cuives. With the exception of ATC
40
, in this example, the lowest cost pei unit foi a paiticulai
level of output in the long iun is not the minimum point of the ielevant shoit-iun cuive.
II GUR 8. 14 ReIationship 8etween Short-Run and Long-Run Average 7otaI Costs
|e ||^ cu.ve |s |curJ by t.k|rg t|e |c.est .ve..ge tct.| ccst cu.ve .t e.c| |eve| c| cutput. |e.e, .ve..ge tct.|
ccst cu.ves |c. ,u.rt|t|es c| c.p|t.| c| 20, 30, 40, .rJ 50 ur|ts ..e s|c.r |c. t|e |||et|ne ||sc c. At . p.cJuct|cr
|eve| c| 10,000 |s pe. .eek, |||et|ne n|r|n|.es |ts ccst pe. | by p.cJuc|rg .|t| 20 ur|ts c| c.p|t.| (pc|rt A). At
20,000 |s pe. .eek, .r exp.rs|cr tc . p|.rt s|.e .sscc|.teJ .|t| 30 ur|ts c| c.p|t.| n|r|n|.es ccst pe. ur|t (pc|rt
b). |e |c.est ccst pe. ur|t |s .c||eveJ .|t| p.cJuct|cr c| 30,000 |s pe. .eek us|rg 40 ur|ts c| c.p|t.| (pc|rt ). ||
|||et|ne c|ccses tc p.cJuce 40,000 |s pe. .eek, |t .||| Jc sc ncst c|e.p|y .|t| 50 ur|ts c| c.p|t.| (pc|rt |).
216 PRINCIPLS OI CONOMICS
economies of scaIe
S|tu.t|cr |r .||c| t|e
|crg.ur .ve..ge ccst
Jec||res .s t|e |.n exp.rJs
|ts cutput.
diseconomies of scaIe
S|tu.t|cr |r .||c| t|e
|crg.ur .ve..ge ccst
|rc.e.ses .s t|e |.n exp.rJs
|ts cutput.
constant returns to scaIe
S|tu.t|cr |r .||c| t|e
|crg.ur .ve..ge ccst st.ys
t|e s.ne cve. .r cutput
..rge.
II GUR 8. 15 conomies and
Diseconomies of ScaIe and Long-Run
Average Cost
|e Jc.r...Js|cp|rg .eg|cr c| t|e |.ns
||^ cu.ve |s .sscc|.teJ .|t| eccrcn|es c|
sc.|e. |e.e n.y be . |c.|.crt.| ..rge
.sscc|.teJ .|t| ccrst.rt .etu.rs tc sc.|e. |e
up...Js|cp|rg ..rge c| t|e cu.ve |np||es
J|seccrcn|es c| sc.|e.
conomies and Diseconomies of ScaIe
Notice that the long-iun aveiage cost cuive in Figuie 8.14 fist slopes downwaid and then slopes up-
waid. The shape of this cuive tells us what is happening to aveiage cost as the fim changes its scale of
opeiations. A fim is said to expeiience economies of scale when long-iun aveiage cost declines as
the fim expands its output. A fim is said to expeiience diseconomies of scale when long-iun avei-
age cost incieases as the fim expands its output. Constant returns to scale occui when long-iun av-
eiage cost stays the same ovei an output iange.
Why would a fim expeiience economies of scale: One souice of economies of scale is gains fiom
specialization. As the scale of a fim's opeiation expands, it is able to use its factois in moie specialized
ways, incieasing theii pioductivity. Anothei souice of economies of scale lies in the economies that can
be gained fiom mass pioduction methods. As the scale of a fim's opeiation expands, the company can
begin to utilize laige-scale machines and pioduction systems that can substantially ieduce cost pei unit.
Why would a fim expeiience diseconomies of scale: At fist glance, it might seem that the answei
lies in the law of diminishing maiginal ietuins, but this is not the case. The law of diminishing maigin-
al ietuins, aftei all, tells us how output changes as a single factoi is incieased, with all othei factois of
pioduction held constant. In contiast, diseconomies of scale desciibe a situation of iising aveiage cost
even when the fim is fiee to vaiy any oi all of its factois as it wishes. Diseconomies of scale aie genei-
ally thought to be caused by management pioblems. As the scale of a fim's opeiations expands, it be-
comes haidei and haidei foi management to cooidinate and guide the activities of individual units of
the fim. Eventually, the diseconomies of management oveiwhelm any gains the fim might be achiev-
ing by opeiating with a laigei scale of plant, and long-iun aveiage costs begin iising. Fiims expeiience
constant ietuins to scale at output levels wheie theie aie neithei economies noi diseconomies of scale.
Foi the iange of output ovei which the fim expeiiences constant ietuins to scale, the long-iun aveiage
cost cuive is hoiizontal.
Fiims aie likely to expeiience all thiee situations, as shown in Figuie 8.13. At veiy
low levels of output, the fim is likely to expeiience economies of scale as it expands the
scale of its opeiations. Theie may follow a iange of output ovei which the fim expeii-
ences constant ietuins to scale-empiiical studies suggest that the iange ovei which
fims expeiience constant ietuins to scale is often veiy laige. And ceitainly theie must
be some iange of output ovei which diseconomies of scale occui; this phenomenon is
one factoi that limits the size of fims. A fim opeiating on the upwaid-sloping pait of
its LRAC cuive is likely to be undeicut in the maiket by smallei fims opeiating with
lowei costs pei unit of output.
7he Size Distribution of Iirms
Economies and diseconomies of scale have a poweiful efect on the sizes of fims that
will opeiate in any maiket. Suppose fims in a paiticulai industiy expeiience dis-
economies of scale at ielatively low levels of output. That industiy will be chaiacteiized
by a laige numbei of faiily small fims. The iestauiant maiket appeais to be such an in-
dustiy. Baibeis and beauticians aie anothei example.
If fims in an industiy expeiience economies of scale ovei a veiy wide iange of out-
put, fims that expand to take advantage of lowei cost will foice out smallei fims that
have highei costs. Such industiies aie likely to have a few laige fims instead of many
small ones. In the iefiigeiatoi industiy, foi example, the size of fim necessaiy to
achieve the lowest possible cost pei unit is laige enough to limit the maiket to only a
few fims. In most cities, economies of scale leave ioom foi only a single newspapei.
One factoi that can limit the achievement of economies of scale is the demand fa-
cing an individual fim. The scale of output iequiied to achieve the lowest unit costs possible may ie-
quiie sales that exceed the demand facing a fim. A gioceiy stoie, foi example, could minimize unit
costs with a laige stoie and a laige volume of sales. But the demand foi gioceiies in a small, isolated
community may not be able to sustain such a volume of sales. The fim is thus limited to a small scale
of opeiation even though this might involve highei unit costs.
CHAP7R 8 PRODUC7ION AND COS7 217
k Y 7 A k A W A Y S
< A |.n c|ccses |ts |.ctc. n|x |r t|e |crg .ur cr t|e b.s|s c| t|e n..g|r.| Jec|s|cr .u|e, |t seeks tc e,u.te
t|e ..t|c c| n..g|r.| p.cJuct tc p.|ce |c. .|| |.ctc.s c| p.cJuct|cr. by Jc|rg sc, |t n|r|n|.es t|e ccst c|
p.cJuc|rg . g|ver |eve| c| cutput.
< |e |crg.ur .ve..ge ccst (||^ ) cu.ve |s Je.|veJ |.cn t|e .ve..ge tct.| ccst cu.ves .sscc|.teJ .|t|
J||e.ert ,u.rt|t|es c| t|e |.ctc. t|.t |s |xeJ |r t|e s|c.t .ur. |e ||^ cu.ve s|c.s t|e |c.est ccst pe.
ur|t .t .||c| e.c| ,u.rt|ty c.r be p.cJuceJ .|er .|| |.ctc.s c| p.cJuct|cr, |rc|uJ|rg c.p|t.|, ..e v..|.b|e.
< A |.n n.y expe.|erce eccrcn|es c| sc.|e, ccrst.rt .etu.rs tc sc.|e, c. J|seccrcn|es c| sc.|e. |ccrcn|es
c| sc.|e |np|y . Jc.r...Js|cp|rg |crg.ur .ve..ge ccst (||^ ) cu.ve. crst.rt .etu.rs tc sc.|e |np|y .
|c.|.crt.| ||^ cu.ve. ||seccrcn|es c| sc.|e |np|y .r up...Js|cp|rg ||^ cu.ve.
< A |.ns .b|||ty tc exp|c|t eccrcn|es c| sc.|e |s ||n|teJ by t|e extert c| n..ket Jen.rJ |c. |ts p.cJucts.
< |e ..rge c| cutput cve. .||c| |.ns expe.|erce eccrcn|es c| sc.|e, ccrst.rt .etu.r tc sc.|e, c.
J|seccrcn|es c| sc.|e |s .r |npc.t.rt Jete.n|r.rt c| |c. n.ry |.ns .||| su.v|ve |r . p..t|cu|.. n..ket.
7 R Y I 7 !
1. Suppcse Acne |ct||rg |s cpe..t|rg .|t| 20 ur|ts c| c.p|t.| .rJ p.cJuc|rg 9 ur|ts c| cutput .t .r
.ve..ge tct.| ccst c| 6, .s s|c.r |r ||gu.e 8.8. |c. nuc| |.bc. |s |t us|rg`
2. Suppcse |t |rJs t|.t, .|t| t||s ccnb|r.t|cr c| c.p|t.| .rJ |.bc., /|
|
/|
|
> /|
|
/|
|
. \|.t .Justnert .|||
t|e |.n n.ke |r t|e |crg .ur` \|y Jces |t rct n.ke t||s s.ne .Justnert |r t|e s|c.t .ur`
Case in Point: 7eIecommunications quipment, conomies of ScaIe, and Outage
Risk
2010 jupiterimages Corporation
|c. b|g s|cu|J t|e c.|| s.|tc||rg e,u|pnert . n.c. te|eccnnur|c.t|crs ccnp.ry uses be` |.v|rg b|gge.
n.c||res .esu|ts |r eccrcn|es c| sc.|e but .|sc ..|ses t|e .|sk c| |..ge. cut.ges t|.t .||| .|ect nc.e
custcne.s.
\e.|.cr |.bc..tc.|es eccrcn|st |cr.|J |. Sn|t| ex.n|reJ bct| t|e eccrcn|es c| sc.|e .v.||.b|e |.cn |..ge.
e,u|pnert .rJ t|e g.e.te. J.rge. c| nc.e .|Jesp.e.J cut.ges. |e ccrc|uJeJ t|.t ccnp.r|es s|cu|J rct
use t|e |..gest n.c||res .v.||.b|e bec.use c| t|e cut.ge J.rge. .rJ t|.t t|ey s|cu|J rct use t|e sn.||est
s|.e bec.use t|.t .cu|J ne.r |c.gc|rg t|e pctert|.| g.|rs |.cn eccrcn|es c| sc.|e c| |..ge. s|.es.
218 PRINCIPLS OI CONOMICS
S.|tc||rg n.c||res, t|e |..ge ccnpute.s t|.t |.rJ|e c.||s |c. te|eccnnur|c.t|crs ccnp.r|es, ccne |r |cu.
b.s|c pc.t n.t.|x s|.es. |ese ..e ne.su.eJ |r te.ns c| ||g|t.| .csscrrects (|Ss). |e |cu. |S s|.es
.v.||.b|e ..e 6,000, 12,000, 24,000, .rJ 36,000 pc.ts. |||e.ert n.c||re s|.es ..e n.Je .|t| t|e s.ne ccnpcr
erts .rJ t|us |.ve essert|.||y t|e s.ne p.cb.b|||ty c| b.e.k|rg Jc.r. bec.use |..ge. n.c||res se.ve nc.e
custcne.s, |c.eve., . b.e.kJc.r |r . |..ge n.c||re |.s g.e.te. ccrse,uerces |c. t|e ccnp.ry.
|e ccsts c| .r cut.ge |.ve t|.ee e|enerts. |e |.st |s |cst .everue |.cn c.||s t|.t .cu|J ct|e..|se |.ve
beer ccnp|eteJ. SeccrJ, t|e | .e,u|.es ccnp.r|es tc p.cv|Je . c.eJ|t c| cre ncrt| c| |.ee se.v|ce .|te.
.ry cut.ge t|.t |.sts |crge. t|.r cre n|rute. ||r.||y, .r cut.ge J.n.ges . ccnp.rys .eput.t|cr .rJ |rev|t
.b|y .esu|ts |r J|ss.t|s|eJ custcne.sscne c| .|cn n.y s.|tc| tc ct|e. ccnp.r|es.
but, t|e.e ..e .Jv.rt.ges tc |..ge. n.c||res. A ccnp.ry |.s . pc.t|c||c c| s.|tc||rg n.c||res. |.v|rg |..
ge. n.c||res |c.e.s ccsts |r seve..| ..ys. ||.st, t|e |r|t|.| .c,u|s|t|cr c| t|e n.c||re gere..tes |c.e. ccst pe.
c.|| ccnp|eteJ t|e g.e.te. t|e s|.e c| t|e n.c||re. \|er t|e ccnp.ry nust n.ke upg..Jes tc t|e sc|t...e,
|.v|rg |e.e..rJ |..ge.n.c||res ne.rs |e.e. upg..Jes .rJ t|us |c.e. ccsts.
|r Jec|J|rg cr n.t.|x s|.e ccnp.r|es s|cu|J t|us ccnp..e t|e ccst .Jv.rt.ges c| . |..ge. n.t.|x .|t| t|e J|s
.Jv.rt.ges c| t|e ||g|e. cut.ge ccsts .sscc|.teJ .|t| t|cse |..ge. n.t.|xes.
V.. Sn|t| ccrc|uJeJ t|.t t|e eccrcn|es c| sc.|e cut.e|g| t|e cut.ge .|sks .s . ccnp.ry exp.rJs beycrJ
6,000 pc.ts but t|.t 36,000 pc.ts |s tcc b|g |r t|e serse t|.t t|e cut.ge ccsts cut.e|g| t|e .Jv.rt.ge c| t|e
eccrcn|es c| sc.|e. |e ev|Jerce t|us suggests t|.t . n.t.|x s|.e |r t|e ..rge c| 12,000 tc 24,000 pc.ts |s
cpt|n.|.
.ooce |oo|J | .|||. |o. || | oo ||. oJ| | ||e |coo|e o| .co|e o| |oe e|ecoo|co||o |e|.o| ||ee| ^o|| ||e ||| o|
|oe o|oe.' |ooeo .ooo| o| eo||oo| |eeoc|. .3 (: (^oo| 2006: 29932
A N S W R S 7 O 7 R Y I 7 ! P R O 8 L M S
1. c p.cJuce 9 .ckets, Acne uses 4 ur|ts c| |.bc..
2. |r t|e |crg .ur, Acne .||| subst|tute c.p|t.| |c. |.bc.. |t c.rrct n.ke t||s .Justnert |r t|e s|c.t .ur,
bec.use |ts c.p|t.| |s |xeJ |r t|e s|c.t .ur.
3. REVIEW AND PRACTICE
Summary
|r t||s c|.pte. .e |.ve ccrcert..teJ cr t|e p.cJuct|cr .rJ ccst .e|.t|crs||ps |.c|rg |.ns |r t|e s|c.t .ur
.rJ |r t|e |crg .ur.
|r t|e s|c.t .ur, . |.n |.s .t |e.st cre |.ctc. c| p.cJuct|cr t|.t |t c.rrct v..y. ||s |xeJ |.ctc. ||n|ts t|e |.ns
..rge c| |.ctc. c|c|ces. As . |.n uses nc.e .rJ nc.e c| . v..|.b|e |.ctc. (.|t| |xeJ ,u.rt|t|es c| ct|e. |.ctc.s
c| p.cJuct|cr), |t |s ||ke|y tc expe.|erce .t |.st |rc.e.s|rg, t|er J|n|r|s||rg, t|er reg.t|ve n..g|r.| .etu.rs.
|us, t|e s|c.t.ur tct.| ccst cu.ve |.s . pcs|t|ve v.|ue .t . .e.c |eve| c| cutput (t|e |.ns tct.| |xeJ ccst),
t|er s|cpes up...J .t . Jec.e.s|rg ..te (t|e ..rge c| |rc.e.s|rg n..g|r.| .etu.rs), .rJ t|er s|cpes up...J .t
.r |rc.e.s|rg ..te (t|e ..rge c| J|n|r|s||rg n..g|r.| .etu.rs).
|r .JJ|t|cr tc s|c.t.ur tct.| p.cJuct .rJ tct.| ccst cu.ves, .e Je.|veJ . |.ns n..g|r.| p.cJuct, .ve..ge
p.cJuct, .ve..ge tct.| ccst, .ve..ge v..|.b|e ccst, .ve..ge |xeJ ccst, .rJ n..g|r.| ccst cu.ves.
|| t|e |.n |s tc n.x|n|.e p.c|t |r t|e |crg .ur, |t nust se|ect t|e ccstn|r|n|.|rg ccnb|r.t|cr c| |.ctc.s |c. |ts
c|cser |eve| c| cutput. |us, t|e |.n nust t.y tc use |.ctc.s c| p.cJuct|cr |r .ccc.J.rce .|t| t|e n..g|r.|
Jec|s|cr .u|e. |.t |s, |t .||| use |.ctc.s sc t|.t t|e ..t|c c| n..g|r.| p.cJuct tc |.ctc. p.|ce |s e,u.| |c. .||
|.ctc.s c| p.cJuct|cr.
A |.ns |crg.ur .ve..ge ccst (||^) cu.ve |rc|uJes . ..rge c| eccrcn|es c| sc.|e, cve. .||c| t|e cu.ve
s|cpes Jc.r...J, .rJ . ..rge c| J|seccrcn|es c| sc.|e, cve. .||c| t|e cu.ve s|cpes up...J. |e.e n.y be
.r |rte.ver|rg ..rge c| cutput cve. .||c| t|e |.n expe.|erces ccrst.rt .etu.rs tc sc.|e, |ts ||^ cu.ve .|||
be |c.|.crt.| cve. t||s ..rge. |e s|.e c| cpe..t|crs recess..y tc .e.c| t|e |c.est pc|rt cr t|e ||^ cu.ve |.s
. g.e.t Je.| tc Jc .|t| Jete.n|r|rg t|e .e|.t|ve s|.es c| |.ns |r .r |rJust.y.
CHAP7R 8 PRODUC7ION AND COS7 219
||s c|.pte. |.s |ccuseJ cr t|e r.tu.e c| p.cJuct|cr p.ccesses .rJ t|e ccsts .sscc|.teJ .|t| t|en. |ese
|Je.s .||| p.cve use|u| |r urJe.st.rJ|rg t|e be|.v|c. c| |.ns .rJ t|e Jec|s|crs t|ey n.ke ccrce.r|rg supp|y
c| gccJs .rJ se.v|ces.
C O N C P 7 P R O 8 L M S
1. \||c| c| t|e |c||c.|rg .cu|J be ccrs|Je.eJ |crg.ur c|c|ces` \||c| ..e s|c.t.ur c|c|ces`
.. A Jert|st ||.es . re. p..tt|ne Jert.| |yg|er|st.
b. |e |cc.| c|| .e|re.y p|.rs . ccnp|ete .est.uctu.|rg c| |ts p.cJuct|cr p.ccesses, |rc|uJ|rg
.e|cc.t|rg t|e p|.rt.
c. A |..ne. |rc.e.ses t|e ,u.rt|ty c| ..te. .pp||eJ tc ||s c. |e. |e|Js.
J. A |.. p..tre.s||p s|grs . 3ye.. |e.se |c. .r c|ce ccnp|ex.
e. |e ur|ve.s|ty ||.es . re. |cctb.|| cc.c| cr . 3ye.. ccrt..ct.
2. |e.e ..e rc |xeJ ccsts |r t|e |crg .ur. |xp|.|r.
3. bus|ress |s bccn|rg .t t|e |cc.| Vc|cr.|Js .est.u..rt. |t |s ccrtenp|.t|rg .JJ|rg . re. g.||| .rJ |.erc|
|.y n.c||re, but t|e J.y supe.v|sc. suggests s|np|y ||.|rg nc.e .c.ke.s. |c. s|cu|J t|e n.r.ge. Jec|Je
.||c| .|te.r.t|ve tc pu.sue`
4. Suppcse t|.t t|e .ve..ge .ge c| stuJerts |r ycu. eccrcn|cs c|.ss |s 23. ye..s. || . re. 19ye..c|J
stuJert er.c||s |r t|e c|.ss, .||| t|e .ve..ge .ge |r t|e c|.ss .|se c. |.||` |xp|.|r |c. t||s .e|.tes tc t|e
.e|.t|crs||p bet.eer .ve..ge .rJ n..g|r.| v.|ues.
5. b...y bcrJs c..ee. |cne .ur .ve..ge |r ||s |.st 15 ye..s |r n.c. |e.gue b.seb.|| (t|.cug| 199) ..s 33
|cne .urs pe. se.scr. |r 2001, |e ||t 3 |cne .urs. \|.t |.ppereJ tc ||s c..ee. |cne .ur .ve..ge`
\|.t e|ect J|J ||s pe.|c.n.rce |r 2001 |.ve cr ||s c..ee. |cne .ur .ve..ge` |xp|.|r |c. t||s .e|.tes tc
t|e .e|.t|crs||p bet.eer .ve..ge .rJ n..g|r.| v.|ues.
6. Suppcse . |.n |s cpe..t|rg .t t|e n|r|nun pc|rt c| |ts s|c.t.ur .ve..ge tct.| ccst cu.ve, sc t|.t
n..g|r.| ccst e,u.|s .ve..ge tct.| ccst. |rJe. .|.t c|.cunst.rces .cu|J |t c|ccse tc .|te. t|e s|.e c| |ts
p|.rt` |xp|.|r.
. \|.t |.ppers tc t|e J||e.erce bet.eer .ve..ge tct.| ccst .rJ .ve..ge v..|.b|e ccst .s . |.ns cutput
exp.rJs` |xp|.|r.
8. |c. .cu|J e.c| c| t|e |c||c.|rg .|ect .ve..ge tct.| ccst, .ve..ge v..|.b|e ccst, .rJ n..g|r.| ccst`
.. Ar |rc.e.se |r t|e ccst c| t|e |e.se c| t|e |.ns bu||J|rg
b. A .eJuct|cr |r t|e p.|ce c| e|ect.|c|ty
c. A .eJuct|cr |r ..ges
J. A c|.rge |r t|e s.|..y c| t|e p.es|Jert c| t|e ccnp.ry
9. crs|Je. t|e |c||c.|rg types c| |.ns. |c. e.c| cre, t|e |crg.ur .ve..ge ccst cu.ve evertu.||y ex||b|ts
J|seccrcn|es c| sc.|e. |c. .||c| |.ns .cu|J ycu expect J|seccrcn|es c| sc.|e tc set |r .t .e|.t|ve|y |c.
|eve|s c| cutput` \|y`
.. A ccpy s|cp
b. A |..J...e stc.e
c. A J.|.y
J. A re.sp.pe.
e. Ar .utcncb||e n.ru|.ctu.e.
|. A .est.u..rt
10. As c.. n.ru|.ctu.e.s |rcc.pc..te nc.e scp||st|c.teJ ccnpute. tec|rc|cgy |r t|e|. ve||c|es, .utc.ep.|.
s|cps .e,u|.e nc.e ccnpute.|.eJ test|rg e,u|pnert, .||c| |s ,u|te expers|ve, |r c.Je. tc .ep.|. re.e.
c..s. |c. |s t||s ||ke|y tc .|ect t|e s|.pe c| t|ese |.ns |crg.ur .ve..ge tct.| ccst cu.ves` |c. |s |t ||ke|y
tc .|ect t|e runbe. c| .utc.ep.|. |.ns |r .ry n..ket`
220 PRINCIPLS OI CONOMICS
N U M R I C A L P R O 8 L M S
1. |e t.b|e be|c. s|c.s |c. t|e runbe. c| ur|ve.s|ty c|.ss.ccns c|e.reJ |r .r ever|rg v..|es .|t| t|e
runbe. c| .r|tc.s.
'.r|tc.s pe. ever|rg 0 1 2 3 4 5 6
|.ss.ccns c|e.reJ pe. ever|rg 0 3 12 16 1 1 16
.. \|.t |s t|e n..g|r.| p.cJuct c| t|e seccrJ .r|tc.`
b. \|.t |s t|e .ve..ge p.cJuct c| |cu. .r|tc.s`
c. |s t|e .JJ|t|cr c| t|e t||.J .r|tc. .sscc|.teJ .|t| |rc.e.s|rg, J|n|r|s||rg, c. reg.t|ve n..g|r.|
.etu.rs` |xp|.|r.
J. |s t|e .JJ|t|cr c| t|e |cu.t| .r|tc. .sscc|.teJ .|t| |rc.e.s|rg, J|n|r|s||rg, c. reg.t|ve n..g|r.|
.etu.rs` |xp|.|r.
e. |s t|e .JJ|t|cr c| t|e severt| .r|tc. .sscc|.teJ .|t| |rc.e.s|rg, J|n|r|s||rg, c. reg.t|ve
n..g|r.| .etu.rs` |xp|.|r.
|. |... t|e tct.| p.cJuct, .ve..ge p.cJuct, .rJ n..g|r.| p.cJuct cu.ves .rJ s|.Je t|e .eg|crs
cc..espcrJ|rg tc |rc.e.s|rg n..g|r.| .etu.rs, Jec.e.s|rg n..g|r.| .etu.rs, .rJ reg.t|ve
n..g|r.| .etu.rs.
g. .|cu|.te t|e s|cpe c| t|e tct.| p.cJuct cu.ve .s e.c| .r|tc. |s .JJeJ.
|. |...cte.|.e t|e r.tu.e c| n..g|r.| .etu.rs |r t|e .eg|cr .|e.e
1. |e s|cpe c| t|e tct.| p.cJuct cu.ve |s pcs|t|ve .rJ |rc.e.s|rg.
2. |e s|cpe c| t|e tct.| p.cJuct cu.ve |s pcs|t|ve .rJ Jec.e.s|rg.
3. |e s|cpe c| t|e tct.| p.cJuct cu.ve |s reg.t|ve.
2. Suppcse . |.n |s p.cJuc|rg 1,000 ur|ts c| cutput. |ts .ve..ge |xeJ ccsts ..e 100. |ts .ve..ge v..|.b|e
ccsts ..e 50. \|.t |s t|e tct.| ccst c| p.cJuc|rg 1,000 ur|ts c| cutput`
3. |e J|.ectc. c| . rcrp.c|t |curJ.t|cr t|.t spcrsc.s 8.eek sunne. |rst|tutes |c. g..Ju.te stuJerts
.r.|y.eJ t|e ccsts .rJ expecteJ .everues |c. t|e rext sunne. |rst|tute .rJ .eccnnerJeJ t|.t t|e
sess|cr be c.rce|eJ. |r |e. .r.|ys|s s|e |rc|uJeJ . s|..e c| t|e |curJ.t|crs cve.|e.Jt|e s.|..|es c| t|e
J|.ectc. .rJ st.| .rJ ccsts c| n.|rt.|r|rg t|e c|cetc t|e p.cg..n. S|e est|n.teJ ccsts .rJ .everues
.s |c||c.s.
|.cecteJ .everues (|.cn tu|t|cr .rJ |ees) 300,000
|o,ec|eJ co|
Ove.|e.J 50,000
|ccn .rJ bc..J |c. stuJerts 100,000
csts |c. |.cu|ty .rJ n|sce||.recus 15,000
ct.| ccsts 325,000
\|.t ..s t|e e..c. |r t|e J|.ectc.s .eccnnerJ.t|cr`
4. |e t.b|e be|c. s|c.s t|e tct.| ccst c| c|e.r|rg c|.ss.ccns.
|.ss.ccns c|e.reJ pe. ever|rg 0 3 12 16 1
ct.| ccst 100 200 300 400 500 600
.. \|.t |s t|e .ve..ge |xeJ ccst c| c|e.r|rg t|.ee c|.ss.ccns`
b. \|.t |s t|e .ve..ge v..|.b|e ccst c| c|e.r|rg t|.ee c|.ss.ccns`
c. \|.t |s t|e .ve..ge |xeJ ccst c| c|e.r|rg sever c|.ss.ccns`
J. \|.t |s t|e .ve..ge v..|.b|e ccst c| c|e.r|rg sever c|.ss.ccns`
e. \|.t |s t|e n..g|r.| ccst c| c|e.r|rg t|e severteert| c|.ss.ccn`
|. \|.t |s t|e .ve..ge tct.| ccst c| c|e.r|rg t.e|ve c|.ss.ccns`
5. |e .ve..ge tct.| ccst |c. p.|rt|rg 10,000 ccp|es c| .r |ssue c| . n.g..|re |s 0.45 pe. ccpy. |c. 20,000
ccp|es, t|e .ve..ge tct.| ccst |s 0.35 .p|ece, |c. 30,000, t|e .ve..ge tct.| ccst |s 0.30 pe. ccpy. |e
.ve..ge tct.| ccst ccrt|rues tc Jec||re s||g|t|y cve. eve.y |eve| c| cutput t|.t t|e pub||s|e.s c| t|e
n.g..|re |.ve ccrs|Je.eJ. Sketc| t|e .pp.cx|n.te s|.pes c| t|e .ve..ge .rJ n..g|r.| ccst cu.ves. \|.t
..e scne v..|.b|e ccsts c| pub||s||rg n.g..|res` Scne |xeJ ccsts`
6. |e |r|c.n.t|cr |r t|e t.b|e exp|.|rs t|e p.cJuct|cr c| sccks. Assune t|.t t|e p.|ce pe. ur|t c| t|e
v..|.b|e |.ctc. c| p.cJuct|cr (|) |s 20 .rJ t|e p.|ce pe. ur|t c| t|e |xeJ |.ctc. c| p.cJuct|cr (|) |s 5.
CHAP7R 8 PRODUC7ION AND COS7 221
Units of Iixed Iactor (|) Units of VariabIe Iactor (|) 7otaI Product (()
10 0 0
10 1 2
10 2 5
10 3 12
10 4 15
10 5 16
.. AJJ cc|unrs tc t|e t.b|e .rJ c.|cu|.te t|e v.|ues |c. . V..g|r.| |.cJuct c| |.bc. (/|
|
), ct.|
\..|.b|e cst (v), ct.| ||xeJ cst (|), ct.| cst (), Ave..ge \..|.b|e cst (^v), Ave..ge
||xeJ cst (^|), Ave..ge ct.| cst (^), .rJ V..g|r.| cst (/).
b. Or t.c sets c| .xes, g..p| t|e ct.| |.cJuct .rJ V..g|r.| |.cJuct cu.ves. be su.e tc |.be| cu.ves
.rJ .xes .rJ .enenbe. tc p|ct n..g|r.| p.cJuct us|rg t|e n|Jpc|rt ccrvert|cr. |rJ|c.te t|e
pc|rt cr e.c| g..p| .t .||c| J|n|r|s||rg n..g|r.| .etu.rs .ppe..s tc beg|r.
c. C..p| ct.| \..|.b|e cst, ct.| ||xeJ cst, .rJ ct.| cst cr .rct|e. set c| .xes. |rJ|c.te t|e
pc|rt cr t|e g..p| .t .||c| J|n|r|s||rg n..g|r.| .etu.rs .ppe..s tc beg|r.
J. C..p| t|e Ave..ge ||xeJ cst, Ave..ge \..|.b|e cst, Ave..ge ct.| cst, .rJ V..g|r.| cst
cu.ves cr .rct|e. set c| .xes. |rJ|c.te t|e pc|rt .t .||c| J|n|r|s||rg n..g|r.| .etu.rs .ppe..s
tc beg|r.
. |e t.b|e be|c. s|c.s t|e |crg.ur .ve..ge ccst c| p.cJuc|rg kr|ves.
|r|ves pe. |cu. 1,000 2,000 3,000 4,000 5,000 6,000
cst pe. kr||e 2 1.50 1.00 1.00 1.20 1.30
.. |... t|e |crg.ur .ve..ge ccst cu.ve |c. kr|ves.
b. S|.Je t|e .eg|crs cc..espcrJ|rg tc eccrcn|es c| sc.|e, ccrst.rt .etu.rs tc sc.|e, .rJ
J|seccrcn|es c| sc.|e.
c. |r t|e .eg|cr c| t|e |crg.ur .ve..ge ccst cu.ve t|.t cc..espcrJs tc eccrcn|es c| sc.|e, .|.t |s
|.pper|rg tc t|e ccst pe. kr||e`
J. |r t|e .eg|cr c| t|e |crg.ur .ve..ge ccst cu.ve t|.t cc..espcrJs tc ccrst.rt .etu.rs tc sc.|e,
.|.t |s |.pper|rg tc t|e ccst pe. kr||e`
e. |r t|e .eg|cr c| t|e |crg.ur .ve..ge ccst cu.ve t|.t cc..espcrJs tc J|seccrcn|es c| sc.|e, .|.t
|s |.pper|rg tc t|e ccst pe. kr||e`
8. Suppcse . |.n |rJs t|.t t|e n..g|r.| p.cJuct c| c.p|t.| |s 60 .rJ t|e n..g|r.| p.cJuct c| |.bc. |s 20. ||
t|e p.|ce c| c.p|t.| |s 6 .rJ t|e p.|ce c| |.bc. |s 2.50, |c. s|cu|J t|e |.n .Just |ts n|x c| c.p|t.| .rJ
|.bc.` \|.t .||| be t|e .esu|t`
9. A |.n n|r|n|.es |ts ccsts by us|rg |rputs suc| t|.t t|e n..g|r.| p.cJuct c| |.bc. |s 10 .rJ t|e n..g|r.|
p.cJuct c| c.p|t.| |s 20. |e p.|ce c| c.p|t.| |s 10 pe. ur|t. \|.t nust t|e p.|ce c| |.bc. be`
10. Suppcse t|.t t|e p.|ce c| |.bc. |s 10 pe. ur|t .rJ t|e p.|ce c| c.p|t.| |s 20 pe. ur|t.
.. Assun|rg t|e |.n |s n|r|n|.|rg |ts ccst, || t|e n..g|r.| p.cJuct c| |.bc. |s 50, .|.t nust t|e
n..g|r.| p.cJuct c| c.p|t.| be`
b. Suppcse t|e p.|ce c| c.p|t.| |rc.e.ses tc 25 pe. ur|t, .|||e t|e p.|ce c| |.bc. st.ys t|e s.ne. c
n|r|n|.e t|e ccst c| p.cJuc|rg t|e s.ne |eve| c| cutput, .cu|J t|e |.n beccne nc.e c.p|t.|
|rters|ve c. |.bc.|rters|ve` |xp|.|r.
222 PRINCIPLS OI CONOMICS
1.
ENDNOTES
|uc|rJ. \..g.s, V.,u||.Jc..s. |np.ct cr ex.s bc.Je. |t|es, |r |e |oJe
|coo,, |eJe..| |ese.ve b.rk c| |.||.s ('ure 2001). 2529, \||||.n . C.uber,
|.ve Vex|ccs V.,u||.Jc..s bcttcneJ Out`, Scut|.est |ccrcny, |eJe..| |ese.ve
b.rk c| |.||.s ('.ru..y/|eb.u..y, 2004), pp. 1415.
CHAP7R 8 PRODUC7ION AND COS7 223
224 PRINCIPLS OI CONOMICS
| A | | | 9
Competitive Markets for
Goods and Services
S7AR7 UP: LII ON 7H IARM
|ey p.cJuce . ccnncJ|ty t|.t |s essert|.| tc cu. J.||y ||ves, cre |c. .||c| t|e Jen.rJ |s v|.tu.||y .ssu.eJ. ArJ
yet n.ryever .s |..n p.|ces ..e .e.c||rg .ecc.J ||g|sseen tc ||ve cr t|e n..g|r c| |.||u.e. |cus.rJs ..e
J.|ver cut c| bus|ress e.c| ye... \e p.cv|Je b||||crs c| Jc||..s |r .|J |c. t|en, but st||| .e |e.. c| t|e |..Js||ps
n.ry c| t|en |.ce. |ey ..e cu. r.t|crs |..ne.s.
\|.t |s |t .bcut |..ne.s, .rJ |..n|rg, t|.t ..cuses cu. ccrce.r` Vuc| c| t|e .rs.e. p.cb.b|y ||es |r cu. serse
t|.t |..n|rg |s |urJ.nert.| tc t|e Ane.|c.r ..y c| |||e. Ou. ccurt.y ..s bu||t, |r |..ge p..t, by |rJeperJert ner
.rJ .cner .|c n.Je t|e|. ||v|rg |.cn t|e sc||. V.ry c| us pe.ce|ve t|e|. p||g|t .s cu. p||g|t. but p..t c| t|e .r
s.e. ||es |r t|e |.ct t|.t |..ne.s Jc, |r |.ct, |.ce . J||cu|t eccrcn|c erv|.crnert. Vcst c| t|en cpe..te |r ||g||y
ccnpet|t|ve n..kets, n..kets t|.t tc|e..te |e. n|st.kes .rJ gere..||y c|e. sn.|| .e...Js. ||r.||y, pe.|.ps cu. ccr
ce.r |s st|..eJ by cu. .eccgr|t|cr t|.t t|e |..ne.s p||g|t |s cu. b|ess|rg. |e |c. p.|ces t|.t n.ke |||e J||cu|t |c.
|..ne.s ..e t|e |c. p.|ces .e ercy .s ccrsune.s c| |ccJ.
\|.t keeps t|e .etu.rs tc |..n|rg .s |c. .s t|ey ..e` \|.t |c|Js n.ry |..ne.s |r . s|tu.t|cr |r .||c| t|ey .|
..ys seen tc be ust gett|rg by` |r t||s c|.pte. .e s|.|| see t|.t p.|ces ust ||g| ercug| tc |rJuce |.ns tc ccrt|r
ue tc p.cJuce ..e p.ec|se|y .|.t .e .cu|J expect tc p.ev.|| |r . ccnpet|t|ve n..ket. \e .||| ex.n|re . ncJe| c|
|c. ccnpet|t|ve n..kets .c.k. |ct cr|y Jces t||s ncJe| |e|p tc exp|.|r t|e s|tu.t|cr |.c|rg |..ne.s, but |t .||| .|sc
|e|p us tc urJe.st.rJ t|e Jete.n|r.t|cr c| p.|ce .rJ cutput |r . .|Je ..rge c| n..kets. A |..n |s . |.n, .rJ cu.
.r.|ys|s c| suc| . |.n |r . ccnpet|t|ve n..ket .||| g|ve us t|e tcc|s tc .r.|y.e t|e c|c|ces c| .|| |.ns cpe..t|rg |r
ccnpet|t|ve n..kets.
\e .||| put t|e ccrcepts c| n..g|r.| ccst, .ve..ge v..|.b|e ccst, .rJ .ve..ge tct.| ccst tc .c.k tc see |c.
|.ns |r . ccnpet|t|ve n..ket .espcrJ tc n..ket |c.ces. \e .||| see |c. |.ns .Just tc c|.rges |r Jen.rJ .rJ
supp|y |r t|e s|c.t .ur .rJ |r t|e |crg .ur. |r .|| c| t||s, .e .||| be ex.n|r|rg |c. |.ns use t|e n..g|r.| Jec|s|cr
.u|e.
|e ccnpet|t|ve ncJe| |rt.cJuceJ |r t||s c|.pte. ||es .t cre erJ c| . spect.un c| n..ket
ncJe|s. At t|e ct|e. erJ |s t|e ncrcpc|y ncJe|. |t .ssunes . n..ket |r .||c| t|e.e |s rc ccn
pet|t|cr, . n..ket |r .||c| cr|y . s|rg|e |.n cpe..tes. .c ncJe|s t|.t |.|| bet.eer t|e ex
t.enes c| pe.|ect ccnpet|t|cr .rJ ncrcpc|y ..e ncrcpc||st|c ccnpet|t|cr .rJ c||gcpc|y.
perfect competition
VcJe| c| t|e n..ket b.seJ
cr t|e .ssunpt|cr t|.t .
|..ge runbe. c| |.ns
p.cJuce |Jert|c.| gccJs
ccrsuneJ by . |..ge runbe.
c| buye.s.
price takers
|rJ|v|Ju.|s c. |.ns .|c nust
t.ke t|e n..ket p.|ce .s
g|ver.
1. PERFECT COMPETITION: A MODEL
L A R N I N G O 8 1 C 7 I V S
1. xpIain what economists mean by perfect competition.
2. Identify the basic assumptions of the modeI of perfect competition and expIain why they impIy
price-taking behavior.
Viitually all fims in a maiket economy face competition fiom othei fims. In this chaptei, we will be
woiking with a model of a highly idealized foim of competition called peifect" by economists.
Perfect competition is a model of the maiket based on the assumption that a laige numbei of
fims pioduce identical goods consumed by a laige numbei of buyeis. The model of peifect competi-
tion also assumes that it is easy foi new fims to entei the maiket and foi existing ones to leave. And
fnally, it assumes that buyeis and selleis have complete infoimation about maiket conditions.
As we examine these assumptions in gieatei detail, we will see that they allow us to woik with the
model moie easily. No maiket fully meets the conditions set out in these assumptions. As is always the
case with models, oui puipose is to undeistand the way things woik, not to desciibe them. And the
model of peifect competition will piove enoimously useful in undeistanding the woild of maikets.
1.1 Assumptions of the Model
The assumptions of the model of peifect competition, taken togethei, imply that individual buyeis and
selleis in a peifectly competitive maiket accept the maiket piice as given. No one buyei oi sellei has
any infuence ovei that piice. Individuals oi fims who must take the maiket piice as given aie called
price takers. A consumei oi fim that takes the maiket piice as given has no ability to infuence that
piice. A piice-taking fim oi consumei is like an individual who is buying oi selling stocks. He oi she
looks up the maiket piice and buys oi sells at that piice. The piice is deteimined by demand and sup-
ply in the maiket-not by individual buyeis oi selleis. In a peifectly competitive maiket, each fim and
each consumei is a piice takei. A piice-taking consumei assumes that he oi she can puichase any
quantity at the maiket piice-without afecting that piice. Similaily, a piice-taking fim assumes it can
sell whatevei quantity it wishes at the maiket piice without afecting the piice.
You aie a piice takei when you go into a stoie. You obseive the piices listed and make a choice to
buy oi not. Youi choice will not afect that piice. Should you sell a textbook back to youi campus book-
stoie at the end of a couise, you aie a piice-taking sellei. You aie confionted by a maiket piice and you
decide whethei to sell oi not. Youi decision will not afect that piice.
To see how the assumptions of the model of peifect competition imply piice-taking behavioi, let
us examine each of them in tuin.
IdenticaI Goods
In a peifectly competitive maiket foi a good oi seivice, one unit of the good oi seivice cannot be difei-
entiated fiom any othei on any basis. A bushel of, say, haid wintei wheat is an example. A bushel pio-
duced by one faimei is identical to that pioduced by anothei. Theie aie no biand piefeiences oi con-
sumei loyalties.
The assumption that goods aie identical is necessaiy if fims aie to be piice takeis. If one faimei's
wheat weie peiceived as having special piopeities that distinguished it fiom othei wheat, then that
faimei would have some powei ovei its piice. By assuming that all goods and seivices pioduced by
fims in a peifectly competitive maiket aie identical, we establish a necessaiy condition foi piice-taking
behavioi. Economists sometimes say that the goods oi seivices in a peifectly competitive maiket aie
homogeneous, meaning that they aie all alike. Theie aie no biand difeiences in a peifectly competitive
maiket.
A Large Number of 8uyers and SeIIers
How many buyeis and selleis aie in oui maiket: The answei iests on oui piesumption of piice-taking
behavioi. Theie aie so many buyeis and selleis that none of them has any infuence on the maiket piice
iegaidless of how much any of them puichases oi sells. A fim in a peifectly competitive maiket can ie-
act to piices, but cannot afect the piices it pays foi the factois of pioduction oi the piices it ieceives foi
its output.
226 PRINCIPLS OI CONOMICS
ase of ntry and xit
The assumption that it is easy foi othei fims to entei a peifectly competitive maiket implies an even
gieatei degiee of competition. Fiims in a maiket must deal not only with the laige numbei of compet-
ing fims but also with the possibility that still moie fims might entei the maiket.
Latei in this chaptei, we will see how ease of entiy is ielated to the sustainability of economic
piofts. If entiy is easy, then the piomise of high economic piofts will quickly attiact new fims. If
entiy is dimcult, it won't.
The model of peifect competition assumes easy exit as well as easy entiy. The assumption of easy
exit stiengthens the assumption of easy entiy. Suppose a fim is consideiing enteiing a paiticulai mai-
ket. Entiy may be easy, but suppose that getting out is dimcult. Foi example, supplieis of factois of pio-
duction to fims in the industiy might be happy to accommodate new fims but might iequiie that they
sign long-teim contiacts. Such contiacts could make leaving the maiket dimcult and costly. If that weie
the case, a fim might be hesitant to entei in the fist place. Easy exit helps make entiy easiei.
CompIete Information
We assume that all selleis have complete infoimation about piices, technology, and all othei know-
ledge ielevant to the opeiation of the maiket. No one sellei has any infoimation about pioduction
methods that is not available to all othei selleis. If one sellei had an advantage ovei othei selleis, pei-
haps special infoimation about a lowei-cost pioduction method, then that sellei could exeit some con-
tiol ovei maiket piice-the sellei would no longei be a piice takei.
We assume also that buyeis know the piices ofeied by eveiy sellei. If buyeis did not know about
piices ofeied by difeient fims in the maiket, then a fim might be able to sell a good oi seivice foi a
piice othei than the maiket piice and thus could avoid being a piice takei.
The availability of infoimation that is assumed in the model of peifect competition implies that in-
foimation can be obtained at low cost. If consumeis and fims can obtain infoimation at low cost, they
aie likely to do so. Infoimation about the maiketplace may come ovei the inteinet, ovei the aiiways in
a television commeicial, oi ovei a cup of cofee with a fiiend. Whatevei its souice, we assume that its
low cost ensuies that consumeis and fims have enough of it so that eveiyone buys oi sells goods and
seivices at maiket piices deteimined by the inteisection of demand and supply cuives.
The assumptions of the peifectly competitive model ensuie that each buyei oi sellei is a piice
takei. The maiket, not individual consumeis oi fims, deteimines piice in the model of peifect compet-
ition. No individual has enough powei in a peifectly competitive maiket to have any impact on that
piice.
1.2 Peifect Competition and the Real Woild
The assumptions of identical pioducts, a laige numbei of buyeis, easy entiy and exit, and peifect in-
foimation aie stiong assumptions. The notion that fims must sit back and let the maiket deteimine
piice seems to fy in the face of what we know about most ieal fims, which is that fims customaiily do
set piices. Yet this is the basis foi the model of demand and supply, the powei of which you have
alieady seen.
When we use the model of demand and supply, we assume that maiket foices deteimine piices. In
this model, buyeis and selleis iespond to the maiket piice. They aie piice takeis. The assumptions of
the model of peifect competition undeilie the assumption of piice-taking behavioi. Thus we aie using
the model of peifect competition whenevei we apply the model of demand and supply.
We can undeistand most maikets by applying the model of demand and supply. Even though
those maikets do not fulfll all the assumptions of the model of peifect competition, the model allows
us to undeistand some key featuies of these maikets.
Changes within youi lifetime have made many maikets moie competitive. Falling costs of tians-
poitation, togethei with diamatic advances in telecommunications, have opened the possibility of en-
teiing maikets to fims all ovei the woild. A company in South Koiea can compete in the maiket foi
steel in the United States. A fuinituie makei in New Mexico can compete in the maiket foi fuinituie in
Japan. A fim can entei the woild maiket simply by cieating a web page to adveitise its pioducts and to
take oideis.
In the iemaining sections of this chaptei, we will leain moie about the iesponse of fims to maiket
piices. We will see how fims iespond, in the shoit iun and in the long iun, to changes in demand and
to changes in pioduction costs. In shoit, we will be examining the foices that constitute the supply side
of the model of demand and supply.
We will also see how competitive maikets woik to seive consumei inteiests and how competition
acts to push economic piofts down, sometimes eliminating them entiiely. When we have fnished we
will have a bettei undeistanding of the maiket conditions facing faimeis and of the conditions that
pievail in any competitive industiy.
CHAP7R 9 COMP7I7IV MARk7S IOR GOODS AND SRVICS 227
k Y 7 A k A W A Y S
< |e cert..| c|...cte.|st|c c| t|e ncJe| c| pe.|ect ccnpet|t|cr |s t|e |.ct t|.t p.|ce |s Jete.n|reJ by t|e
|rte..ct|cr c| Jen.rJ .rJ supp|y, buye.s .rJ se||e.s ..e p.|ce t.ke.s.
< |e ncJe| .ssunes. . |..ge runbe. c| |.ns p.cJuc|rg |Jert|c.| (|cncgerecus) gccJs c. se.v|ces, . |..ge
runbe. c| buye.s .rJ se||e.s, e.sy ert.y .rJ ex|t |r t|e |rJust.y, .rJ ccnp|ete |r|c.n.t|cr .bcut p.|ces |r
t|e n..ket.
< |e ncJe| c| pe.|ect ccnpet|t|cr urJe.||es t|e ncJe| c| Jen.rJ .rJ supp|y.
7 R Y I 7 !
\||c| c| t|e |c||c.|rg gccJs .rJ se.v|ces ..e ||ke|y p.cJuceJ |r . pe.|ect|y ccnpet|t|ve |rJust.y` |e|.te ycu.
.rs.e. tc t|e .ssunpt|crs c| t|e ncJe| c| pe.|ect ccnpet|t|cr.
1. |rte.r.t|cr.| exp.ess n.|| se.v|ce
2. c.r
3. At||et|c s|ces
Case in Point: ntering and xiting the 8urkha Industry
2010 jupiterimages Corporation
Vu|.nneJ |b..||n |s|.n.J|r ..s J.|v|rg . c.b |r |.bu|, A|g|.r|st.r, .|er t|e .||b.r tcck cve. t|e ccur
t.y. |e |c.es.. t|e .ep.ess|cr t|.t .cu|J |c||c. .rJ serseJ .r cppc.tur|ty.
|e sc|J ||s t.x|c.b .rJ set up . s|cp |c. se.|rg .rJ se|||rg bu.k|.s, t|e g..nerts .e,u|.eJ c| .|| .cner ur
Je. t|e .||b.rs .u|e. V.. |s|.n.J|r |.J .r e.sy t.sk se|||rg, .s .cner c.ug|t cutJcc.s .|t| expcseJ sk|r
.e.e .cut|re|y be.ter by t|e .||b.rs .e||g|cus pc||ce. |e tc|J |e /o|| .|ee| .ooo|, ||s ..s ve.y b.J |c.
t|en, but |t ..s gccJ |c. ne.
O| ccu.se, V.. |s|.n.J|r ..s rct t|e cr|y p.cJuce. tc get |rtc t|e |rJust.y. Ot|e. A|g|.r| ne.c|.rts, .s .e||
.s ne.c|.rts |.cn |.k|st.r .rJ ||r., .|sc unpeJ .t t|e cppc.tur|ty.
|e ert.y c| re. |.ns exenp|||es .r |npc.t.rt c|...cte.|st|c c| pe.|ect ccnpet|t|cr. \|ereve. t|e.e |s .r
cppc.tur|ty tc e..r eccrcn|c p.c|tsever .r urexpecteJ cppc.tur|tyre. |.ns .||| erte., p.cv|JeJ t|.t
ert.y |s e.sy.
228 PRINCIPLS OI CONOMICS
|e ncJe| c| pe.|ect ccnpet|t|cr .|sc .ssunes t|.t ex|t .||| be e.sy || .rJ .|er . |.n expe.|erces eccrcn|c
|csses. \|er t|e .||b.r .u|e.s .e.e custeJ by t|e |r|teJ St.tes .rJ |ts .|||es |r 2001, V.. |s|.n.J|r expecteJ
t|.t t|e Jen.rJ |c. bu.k|.s .cu|J beg|r tc |.||. |t J|J. |e s.|es |e|| 50 .|ncst |nneJ|.te|y. |.|ces |e|| .s
.e||, gere..||y by .bcut 20.
|t ..s s|np|e |c. V.. |s|.n.J|r tc |e.ve t|e |rJust.y. |e g.ve ||s .en.|r|rg stcck c| bu.k|.s tc . b.ct|e. .|c
..s p.cJuc|rg t|en |r t|e ccurt.ys|Je .|e.e .cner ccrt|rueJ tc .e.. t|en. As |c. V.. |s|.n.J|r, |e |.s
n.Je p|.rs tc gc |rtc t|e g|.ss...e bus|ress. |e expects t|e Jen.rJ |c. g|.ss te.cups tc be st.crg
.|.teve. |.ppers |r A|g|.r|st.rs c.|t|c.| |utu.e.
.ooce ^Je. |||. /||| ||o|c |e. o| Ooe ||e Oo, /|o .o|J |o||o.' |e /o|| .|ee| .ooo|. |ecee 9. 200. ^
A N S W R S 7 O 7 R Y I 7 ! P R O 8 L M S
1. |ct pe.|ect|y ccnpet|t|ve|e.e ..e |e. se||e.s |r t||s n..ket (|eJex, ||S, .rJ t|e |r|teJ St.tes |cst.|
Se.v|ces ..e t|e n.|r cres |r t|e |r|teJ St.tes) p.cb.b|y bec.use c| t|e J||cu|ty c| ert.y .rJ ex|t. c
p.cv|Je t|ese se.v|ces .e,u|.es n.ry cut|ets .rJ . |..ge t..rspc.t.t|cr |eet, |c. ex.np|e.
2. |e.|ect|y ccnpet|t|ve|e.e ..e n.ry |.ns p.cJuc|rg . |..ge|y |cncgerecus p.cJuct .rJ t|e.e |s
gccJ |r|c.n.t|cr .bcut p.|ces. |rt.y .rJ ex|t |s .|sc |.|.|y e.sy .s |.ns c.r s.|tc| .ncrg . v..|ety c|
c.cps.
3. |ct pe.|ect|y ccnpet|t|ve|e n.|r .e.scr |s t|.t gccJs ..e rct |Jert|c.|.
2. OUTPUT DETERMINATION IN THE SHORT RUN
L A R N I N G O 8 1 C 7 I V S
1. Show graphicaIIy how an individuaI rm in a perfectIy competitive market can use totaI reven-
ue and totaI cost curves or marginaI revenue and marginaI cost curves to determine the IeveI of
output that wiII maximize its economic prot.
2. xpIain when a rm wiII shut down in the short run and when it wiII operate even if it is incur-
ring economic Iosses.
3. Derive the rm's suppIy curve from the rm's marginaI cost curve and the industry suppIy
curve from the suppIy curves of individuaI rms.
Oui goal in this section is to see how a fim in a peifectly competitive maiket deteimines its output
level in the shoit iun-a planning peiiod in which at least one factoi of pioduction is fxed in quantity.
We shall see that the fim can maximize economic pioft by applying the maiginal decision iule and in-
cieasing output up to the point at which the maiginal beneft of an additional unit of output is just
equal to the maiginal cost. This fact has an impoitant implication: ovei a wide iange of output, the
fim's maiginal cost cuive is its supply cuive.
2.1 Piice and Revenue
Each fim in a peifectly competitive maiket is a piice takei; the equilibiium piice and industiy output
aie deteimined by demand and supply. Figuie 9.3 shows how demand and supply in the maiket foi
iadishes, which we shall assume aie pioduced undei conditions of peifect competition, deteimine total
output and piice. The equilibiium piice is $0.40 pei pound; the equilibiium quantity is 10 million
pounds pei month.
CHAP7R 9 COMP7I7IV MARk7S IOR GOODS AND SRVICS 229
II GUR 9. 3 7he Market for Radishes
|.|ce .rJ cutput |r . ccnpet|t|ve n..ket ..e
Jete.n|reJ by Jen.rJ .rJ supp|y. |r t|e
n..ket |c. ..J|s|es, t|e e,u|||b.|un p.|ce |s
0.40 pe. pcurJ, 10 n||||cr pcurJs pe. ncrt|
..e p.cJuceJ .rJ pu.c|.seJ .t t||s p.|ce.
totaI revenue
A |.ns cutput nu|t|p||eJ by
t|e p.|ce .t .||c| |t se||s t|.t
cutput.
Because it is a piice takei, each fim in the iadish industiy assumes it can sell all the
iadishes it wants at a piice of $0.40 pei pound. No mattei how many oi how few
iadishes it pioduces, the fim expects to sell them all at the maiket piice.
The assumption that the fim expects to sell all the iadishes it wants at the maiket
piice is ciucial. If a fim did not expect to sell all of its iadishes at the maiket piice-if it
had to lowei the piice to sell some quantities-the fim would not be a piice takei. And
piice-taking behavioi is cential to the model of peifect competition.
Radish gioweis-and peifectly competitive fims in geneial-have no ieason to
chaige a piice lowei than the maiket piice. Because buyeis have complete infoimation
and because we assume each fim's pioduct is identical to that of its iivals, fims aie un-
able to chaige a piice highei than the maiket piice. Foi peifectly competitive fims, the
piice is veiy much like the weathei: they may complain about it, but in peifect competi-
tion theie is nothing any of them can do about it.
7otaI Revenue
While a fim in a peifectly competitive maiket has no infuence ovei its piice, it does
deteimine the output it will pioduce. In selecting the quantity of that output, one im-
poitant consideiation is the ievenue the fim will gain by pioducing it.
A fim's total revenue is found by multiplying its output by the piice at which it
sells that output. Foi a peifectly competitive fim, total ievenue (TR) is the maiket piice
(P) times the quantity the fim pioduces (Q), oi
QUA7I ON 9. 1
TR = P Q
The ielationship between maiket piice and the fim's total ievenue cuive is a ciucial one. Panel (a)
of Figuie 9.4 shows total ievenue cuives foi a iadish giowei at thiee possible maiket piices: $0.20,
$0.40, and $0.60 pei pound. Each total ievenue cuive is a lineai, upwaid-sloping cuive. At any piice,
the gieatei the quantity a peifectly competitive fim sells, the gieatei its total ievenue. Notice that the
gieatei the piice, the steepei the total ievenue cuive is.
II GUR 9. 4 7otaI Revenue, MarginaI Revenue, and Average Revenue
|.re| (.) s|c.s J||e.ert tct.| .everue cu.ves |c. t|.ee pcss|b|e n..ket p.|ces |r pe.|ect ccnpet|t|cr. A tct.|
.everue cu.ve |s . st..|g|t ||re ccn|rg cut c| t|e c.|g|r. |e s|cpe c| . tct.| .everue cu.ve |s /|. |t e,u.|s t|e
n..ket p.|ce (|) .rJ ^| |r pe.|ect ccnpet|t|cr. V..g|r.| .everue .rJ .ve..ge .everue ..e t|us . s|rg|e |c.|.crt.|
||re .t t|e n..ket p.|ce, .s s|c.r |r |.re| (b). |e.e |s . J||e.ert n..g|r.| .everue cu.ve |c. e.c| p.|ce.
230 PRINCIPLS OI CONOMICS
marginaI revenue
|e |rc.e.se |r tct.| .everue
|.cn . creur|t |rc.e.se |r
,u.rt|ty.
average revenue
ct.| .everue J|v|JeJ by
,u.rt|ty.
Price, MarginaI Revenue, and Average Revenue
The slope of a total ievenue cuive is paiticulaily impoitant. It equals the change in the veitical axis
(total ievenue) divided by the change in the hoiizontal axis (quantity) between any two points. The
slope measuies the iate at which total ievenue incieases as output incieases. We can think of it as the
inciease in total ievenue associated with a 1-unit inciease in output. The inciease in total ievenue fiom
a 1-unit inciease in quantity is marginal revenue. Thus maiginal ievenue (MR) equals the slope of
the total ievenue cuive.
How much additional ievenue does a iadish pioducei gain fiom selling one moie pound of
iadishes: The answei, of couise, is the maiket piice foi 1 pound. Maiginal ievenue equals the maiket
piice. Because the maiket piice is not afected by the output choice of a single fim, the maiginal ieven-
ue the fim gains by pioducing one moie unit is always the maiket piice. The maiginal ievenue cuive
shows the ielationship between maiginal ievenue and the quantity a fim pioduces. Foi a peifectly
competitive fim, the maiginal ievenue cuive is a hoiizontal line at the maiket piice. If the maiket
piice of a pound of iadishes is $0.40, then the maiginal ievenue is $0.40. Maiginal ievenue cuives foi
piices of $0.20, $0.40, and $0.60 aie given in Panel (b) of Figuie 9.4. In peifect competition, a fim's
maiginal ievenue cuive is a hoiizontal line at the maiket piice.
Piice also equals average revenue, which is total ievenue divided by quantity. Equation 9.1 gives
total ievenue, TR. To obtain aveiage ievenue (AR), we divide total ievenue by quantity, Q. Because
total ievenue equals piice (P) times quantity (Q), dividing by quantity leaves us with piice.
QUA7I ON 9. 2
AR =
TR
Q
=
P Q
Q
= P
The maiginal ievenue cuive is a hoiizontal line at the maiket piice, and aveiage ievenue equals the
maiket piice. The aveiage and maiginal ievenue cuives aie given by the same hoiizontal line. This is
consistent with what we have leained about the ielationship between maiginal and aveiage values.
When the maiginal value exceeds the aveiage value, the aveiage value will be iising. When the maigin-
al value is less than the aveiage value, the aveiage value will be falling. What happens when the aveiage
and maiginal values do not change, as in the hoiizontal cuives of Panel (b) of Figuie 9.4: The maiginal
value must equal the aveiage value; the two cuives coincide.
MarginaI Revenue, Price, and Demand for the PerfectIy Competitive Iirm
We have seen that a peifectly competitive fim's maiginal ievenue cuive is simply a hoiizontal line at
the maiket piice and that this same line is also the fim's aveiage ievenue cuive. Foi the peifectly com-
petitive fim, MR = P = AR . The maiginal ievenue cuive has anothei meaning as well. It is the de-
mand cuive facing a peifectly competitive fim.
Considei the case of a single iadish pioducei, Tony Goitaii. We assume that the iadish maiket is
peifectly competitive; Mi. Goitaii iuns a peifectly competitive fim. Suppose the maiket piice of
iadishes is $0.40 pei pound. How many pounds of iadishes can Mi. Goitaii sell at this piice: The an-
swei comes fiom oui assumption that he is a piice takei: He can sell any quantity he wishes at this
piice. How many pounds of iadishes will he sell if he chaiges a piice that exceeds the maiket piice:
None. His iadishes aie identical to those of eveiy othei fim in the maiket, and eveiyone in the maiket
has complete infoimation. That means the demand cuive facing Mi. Goitaii is a hoiizontal line at the
maiket piice as illustiated in Figuie 9.3. Notice that the cuive is labeled d to distinguish it fiom the
maiket demand cuive, D, in Figuie 9.3. The hoiizontal line in Figuie 9.3 is also Mi. Goitaii's maiginal
ievenue cuive, MR, and his aveiage ievenue cuive, AR. It is also the maiket piice, P.
Of couise, Mi. Goitaii could chaige a piice below the maiket piice, but why would he: We assume
he can sell all the iadishes he wants at the maiket piice; theie would be no ieason to chaige a lowei
piice. Mi. Goitaii faces a demand cuive that is a hoiizontal line at the maiket piice. In oui subsequent
analysis, we shall iefei to the hoiizontal line at the maiket piice simply as maiginal ievenue. We should
iemembei, howevei, that this same line gives us the maiket piice, aveiage ievenue, and the demand
cuive facing the fim.
CHAP7R 9 COMP7I7IV MARk7S IOR GOODS AND SRVICS 231
II GUR 9. 5 Price, MarginaI Revenue,
and Demand
A pe.|ect|y ccnpet|t|ve |.n |.ces . |c.|.crt.|
Jen.rJ cu.ve .t t|e n..ket p.|ce. |e.e, ..J|s|
g.c.e. cry Cc.t..| |.ces Jen.rJ cu.ve J .t
t|e n..ket p.|ce c| 0.40 pe. pcurJ. |e ccu|J
se||
1
c.
2
c. .ry ct|e. ,u.rt|ty.t . p.|ce
c| 0.40 pe. pcurJ.
Moie geneially, we can say that any peifectly competitive fim faces a hoiizontal
demand cuive at the maiket piice. We saw an example of a hoiizontal demand cuive in
the chaptei on elasticity. Such a cuive is peifectly elastic, meaning that any quantity is
demanded at a given piice.
2.2 Economic Piofit in the Shoit Run
A fim's economic pioft is the difeience between total ievenue and total cost. Recall
that total cost is the oppoitunity cost of pioducing a ceitain good oi seivice. When we
speak of economic pioft we aie speaking of a fim's total ievenue less the total oppoi-
tunity cost of its opeiations.
As we leained, a fim's total cost cuive in the shoit iun inteisects the veitical axis
at some positive value equal to the fim's total fxed costs. Total cost then iises at a de-
cieasing iate ovei the iange of incieasing maiginal ietuins to the fim's vaiiable factois.
It iises at an incieasing iate ovei the iange of diminishing maiginal ietuins. Figuie 9.6
shows the total cost cuive foi Mi. Goitaii, as well as the total ievenue cuive foi a piice
of $0.40 pei pound. Suppose that his total fxed cost is $400 pei month. Foi any given
level of output, Mi. Goitaii's economic pioft is the veitical distance between the total
ievenue cuive and the total cost cuive at that level.
II GUR 9. 6 7otaI Revenue, 7otaI Cost, and conomic Profit
|ccrcn|c p.c|t |s t|e ve.t|c.| J|st.rce bet.eer t|e tct.| .everue .rJ tct.| ccst cu.ves (.everue
n|rus ccsts). |e.e, t|e n.x|nun p.c|t .tt.|r.b|e by cry Cc.t..| |c. ||s ..J|s| p.cJuct|cr |s
938 pe. ncrt| .t .r cutput c| 6,00 pcurJs.
Let us examine the total ievenue and total cost cuives in Figuie 9.6 moie caiefully. At zeio units of out-
put, Mi. Goitaii's total cost is $400 (his total fxed cost); total ievenue is zeio. Total cost continues to
exceed total ievenue up to an output of 1,300 pounds pei month, at which point the two cuives intei-
sect. At this point, economic pioft equals zeio. As Mi. Goitaii expands output above 1,300 pounds pei
month, total ievenue becomes gieatei than total cost. We see that at a quantity of 1,300 pounds pei
month, the total ievenue cuive is steepei than the total cost cuive. Because ievenues aie iising fastei
than costs, piofts iise with incieased output. As long as the total ievenue cuive is steepei than the total
cost cuive, pioft incieases as the fim incieases its output.
232 PRINCIPLS OI CONOMICS
economic prot per unit
|e J||e.erce bet.eer p.|ce
.rJ .ve..ge tct.| ccst.
The total ievenue cuive's slope does not change as the fim incieases its output. But the total cost
cuive becomes steepei and steepei as diminishing maiginal ietuins set in. Eventually, the total cost and
total ievenue cuives will have the same slope. That happens in Figuie 9.6 at an output of 6,700 pounds
of iadishes pei month. Notice that a line diawn tangent to the total cost cuive at that quantity has the
same slope as the total ievenue cuive.
As output incieases beyond 6,700 pounds, the total cost cuive continues to become steepei. It be-
comes steepei than the total ievenue cuive, and piofts fall as costs iise fastei than ievenues. At an out-
put slightly above 8,000 pounds pei month, the total ievenue and cost cuives inteisect again, and eco-
nomic pioft equals zeio. Mi. Goitaii achieves the gieatest pioft possible by pioducing 6,700 pounds
of iadishes pei month, the quantity at which the total cost and total ievenue cuives have the same
slope. Moie geneially, we can conclude that a peifectly competitive fim maximizes economic pioft at
the output level at which the total ievenue cuive and the total cost cuive have the same slope.
2.3 Applying the Maiginal Decision Rule
The slope of the total ievenue cuive is maiginal ievenue; the slope of the total cost cuive is maiginal
cost. Economic pioft, the difeience between total ievenue and total cost, is maximized wheie maigin-
al ievenue equals maiginal cost. This is consistent with the maiginal decision iule, which holds that a
pioft-maximizing fim should inciease output until the maiginal beneft of an additional unit equals
the maiginal cost. The maiginal beneft of selling an additional unit is measuied as maiginal ievenue.
Finding the output at which maiginal ievenue equals maiginal cost is thus an application of oui mai-
ginal decision iule.
Figuie 9.7 shows how a fim can use the maiginal decision iule to deteimine its pioft-maximizing
output. Panel (a) shows the maiket foi iadishes; the maiket demand cuive (D), and supply cuive (S)
that we had in Figuie 9.3; the maiket piice is $0.40 pei pound. In Panel (b), the MR cuive is given by a
hoiizontal line at the maiket piice. The fim's maiginal cost cuive (MC) inteisects the maiginal ieven-
ue cuive at the point wheie pioft is maximized. Mi. Goitaii maximizes piofts by pioducing 6,700
pounds of iadishes pei month. That is, of couise, the iesult we obtained in Figuie 9.6, wheie we saw
that the fim's total ievenue and total cost cuives difei by the gieatest amount at the point at which the
slopes of the cuives, which equal maiginal ievenue and maiginal cost, iespectively, aie equal.
II GUR 9. 7 AppIying the MarginaI Decision RuIe
|e n..ket p.|ce |s Jete.n|reJ by t|e |rte.sect|cr c| Jen.rJ .rJ supp|y. As .|..ys, t|e |.n n.x|n|.es p.c|t by
.pp|y|rg t|e n..g|r.| Jec|s|cr .u|e. |t t.kes t|e n..ket p.|ce, 0.40 pe. pcurJ, .s g|ver .rJ se|ects .r cutput .t
.||c| /| e,u.|s /. |ccrcn|c p.c|t pe. ur|t |s t|e J||e.erce bet.eer ^ .rJ p.|ce (|e.e, 0.14 pe. pcurJ),
eccrcn|c p.c|t |s p.c|t pe. ur|t t|nes t|e ,u.rt|ty p.cJuceJ (0.146,00938).
CHAP7R 9 COMP7I7IV MARk7S IOR GOODS AND SRVICS 233
economic Ioss
|e .ncurt by .||c| . |.ns
tct.| ccst exceeJs |ts tct.|
.everue.
We can use the giaph in Figuie 9.7 to compute Mi. Goitaii's economic pioft. Economic pront per
unit is the difeience between piice and aveiage total cost. At the pioft-maximizing output of 6,700
pounds of iadishes pei month, aveiage total cost (ATC) is $0.26 pei pound, as shown in Panel (b).
Piice is $0.40 pei pound, so economic pioft pei unit is $0.14. Economic pioft is found by multiplying
economic pioft pei unit by the numbei of units pioduced; the fim's economic pioft is thus $938
($0.14 6,700). It is shown giaphically by the aiea of the shaded iectangle in Panel (b); this aiea equals
the veitical distance between maiginal ievenue (MR) and aveiage total cost (ATC) at an output of 6,700
pounds of iadishes times the numbei of pounds of iadishes pioduced, 6,700, in Figuie 9.7.
Heads Up!
|cck c..e|u||y .t t|e .ect.rg|e t|.t s|c.s eccrcn|c p.c|t |r |.re| (b) c| ||gu.e 9.. |t |s |curJ by t.k|rg t|e
p.c|tn.x|n|.|rg ,u.rt|ty, 6,00 pcurJs, t|er .e.J|rg up tc t|e ^ cu.ve .rJ t|e |.ns Jen.rJ cu.ve .t
t|e n..ket p.|ce. |ccrcn|c p.c|t pe. ur|t e,u.|s p.|ce n|rus .ve..ge tct.| ccst (| ^).
|e |.ns eccrcn|c p.c|t e,u.|s eccrcn|c p.c|t pe. ur|t t|nes t|e ,u.rt|ty p.cJuceJ. |t |s |curJ by exterJ
|rg |c.|.crt.| ||res |.cn t|e ^ .rJ /| co.e tc t|e ve.t|c.| .x|s .rJ t.k|rg t|e ..e. c| t|e .ect.rg|e |c.neJ.
|e.e |s rc .e.scr |c. t|e p.c|tn.x|n|.|rg ,u.rt|ty tc cc..espcrJ tc t|e |c.est pc|rt cr t|e ^ cu.ve, |t
Jces rct |r t||s c.se. StuJerts scnet|nes n.ke t|e n|st.ke c| c.|cu|.t|rg eccrcn|c p.c|t .s t|e J||e.erce
bet.eer t|e p.|ce .rJ t|e |c.est pc|rt cr t|e ^ cu.ve. |.t g|ves us t|e n.x|nun eccrcn|c p.c|t pe.
ur|t, but .e .ssune t|.t |.ns n.x|n|.e eccrcn|c p.c|t, rct eccrcn|c p.c|t pe. ur|t. |e |.ns eccrcn|c
p.c|t e,u.|s eccrcn|c p.c|t pe. ur|t t|nes ,u.rt|ty. |e ,u.rt|ty t|.t n.x|n|.es eccrcn|c p.c|t |s Jete.n
|reJ by t|e |rte.sect|cr c| ^ .rJ /|.
2.4 Economic Losses in the Shoit Run
In the shoit iun, a fim has one oi moie inputs whose quantities aie fxed. That means that in the shoit
iun the fim cannot leave its industiy. Even if it cannot covei all of its costs, including both its vaiiable
and fxed costs, going entiiely out of business is not an option in the shoit iun. The fim may close its
doois, but it must continue to pay its fxed costs. It is foiced to accept an economic loss, the amount
by which its total cost exceeds its total ievenue.
Suppose, foi example, that a manufactuiei has signed a 1-yeai lease on some equipment. It must
make payments foi this equipment duiing the teim of its lease, whethei it pioduces anything oi not.
Duiing the peiiod of the lease, the payments iepiesent a fxed cost foi the fim.
A fim that is expeiiencing economic losses-whose economic piofts have become negative-in
the shoit iun may eithei continue to pioduce oi shut down its opeiations, ieducing its output to zeio.
It will choose the option that minimizes its losses. The ciucial test of whethei to opeiate oi shut down
lies in the ielationship between piice and aveiage vaiiable cost.
Producing to Minimize conomic Loss
Suppose the demand foi iadishes falls to D
2
, as shown in Panel (a) of Figuie 9.8. The maiket piice foi
iadishes plunges to $0.18 pei pound, which is below aveiage total cost. Consequently Mi. Goitaii ex-
peiiences negative economic piofts-a loss. Although the new maiket piice falls shoit of aveiage total
cost, it still exceeds aveiage vaiiable cost, shown in Panel (b) as AVC. Theiefoie, Mi. Goitaii should
continue to pioduce an output at which maiginal cost equals maiginal ievenue. These cuives (labeled
MC and MR
2
) inteisect in Panel (b) at an output of 4,444 pounds of iadishes pei month.
234 PRINCIPLS OI CONOMICS
shutdown point
|e n|r|nun |eve| c|
.ve..ge v..|.b|e ccst, .||c|
cccu.s .t t|e |rte.sect|cr c|
t|e n..g|r.| ccst cu.ve .rJ
t|e .ve..ge v..|.b|e ccst
cu.ve.
II GUR 9. 8 Suffering conomic Losses in the Short Run
cry Cc.t..| expe.|erces . |css .|er p.|ce J.cps be|c. ^, .s |t Jces |r |.re| (b) .s . .esu|t c| . .eJuct|cr |r
Jen.rJ. || p.|ce |s .bcve ^v, |c.eve., |e c.r n|r|n|.e ||s |csses by p.cJuc|rg .|e.e / e,u.|s /|
2
. |e.e, t|.t
cccu.s .t .r cutput c| 4,444 pcurJs c| ..J|s|es pe. ncrt|. |e p.|ce |s 0.18 pe. pcurJ, .rJ .ve..ge tct.| ccst |s
0.23 pe. pcurJ. |e |cses 0.05 pe. pcurJ, c. 222.20 pe. ncrt|.
When pioducing 4,444 pounds of iadishes pei month, Mi. Goitaii faces an aveiage total cost of $0.23
pei pound. At a piice of $0.18 pei pound, he loses a nickel on each pound pioduced. Total economic
losses at an output of 4,444 pounds pei month aie thus $222.20 pei month (=4,444$0.03).
No pioducei likes a loss (that is, negative economic pioft), but the loss solution shown in Figuie
9.8 is the best Mi. Goitaii can attain. Any level of pioduction othei than the one at which maiginal cost
equals maiginal ievenue would pioduce even gieatei losses.
Suppose Mi. Goitaii weie to shut down and pioduce no iadishes. Ceasing pioduction would ie-
duce vaiiable costs to zeio, but he would still face fxed costs of $400 pei month (iecall that $400 was
the veitical inteicept of the total cost cuive in Figuie 9.6). By shutting down, Mi. Goitaii would lose
$400 pei month. By continuing to pioduce, he loses only $222.20.
Mi. Goitaii is bettei of pioducing wheie maiginal cost equals maiginal ievenue because at that
output piice exceeds aveiage vaiiable cost. Aveiage vaiiable cost is $0.14 pei pound, so by continuing
to pioduce he coveis his vaiiable costs, with $0.04 pei pound left ovei to apply to fxed costs. Whenevei
piice is gieatei than aveiage vaiiable cost, the fim maximizes economic pioft (oi minimizes economic
loss) by pioducing the output level at which maiginal ievenue and maiginal cost cuives inteisect.
Shutting Down to Minimize conomic Loss
Suppose piice diops below a fim's aveiage vaiiable cost. Now the best stiategy foi the fim is to shut
down, ieducing its output to zeio. The minimum level of aveiage vaiiable cost, which occuis at the in-
teisection of the maiginal cost cuive and the aveiage vaiiable cost cuive, is called the shutdown
point. Any piice below the minimum value of aveiage vaiiable cost will cause the fim to shut down. If
the fim weie to continue pioducing, not only would it lose its fxed costs, but it would also face an ad-
ditional loss by not coveiing its vaiiable costs.
Figuie 9.9 shows a case wheie the piice of iadishes diops to $0.10 pei pound. Piice is less than av-
eiage vaiiable cost, so Mi. Goitaii not only would lose his fxed cost but would also incui additional
losses by pioducing. Suppose, foi example, he decided to opeiate wheie maiginal cost equals maiginal
ievenue, pioducing 1,700 pounds of iadishes pei month. Aveiage vaiiable cost equals $0.14 pei pound,
so he would lose $0.04 on each pound he pioduces ($68) plus his fxed cost of $400 pei month. He
would lose $468 pei month. If he shut down, he would lose only his fxed cost. Because the piice of
$0.10 falls below his aveiage vaiiable cost, his best couise would be to shut down.
CHAP7R 9 COMP7I7IV MARk7S IOR GOODS AND SRVICS 235
II GUR 9. 9 Shutting Down
|e n..ket p.|ce c| ..J|s|es J.cps tc 0.10 pe.
pcurJ, sc /|
3
|s be|c. V.. Cc.t..|s ^v. |us
|e .cu|J su|e. . g.e.te. |css by ccrt|ru|rg tc
cpe..te t|.r by s|utt|rg Jc.r. \|ereve.
p.|ce |.||s be|c. .ve..ge v..|.b|e ccst, t|e |.n
.||| s|ut Jc.r, .eJuc|rg |ts p.cJuct|cr tc
.e.c.
Shutting down is not the same thing as going out of business. A fim shuts down
by closing its doois; it can ieopen them whenevei it expects to covei its vaiiable costs.
We can even think of a fim's decision to close at the end of the day as a kind of shut-
down point; the fim makes this choice because it does not anticipate that it will be able
to covei its vaiiable cost oveinight. It expects to covei those costs the next moining
when it ieopens its doois.
2.3 Maiginal Cost and Supply
In the model of peifect competition, we assume that a fim deteimines its output by
fnding the point wheie the maiginal ievenue and maiginal cost cuives inteisect.
Piovided that piice exceeds aveiage vaiiable cost, the fim pioduces the quantity de-
teimined by the inteisection of the two cuives.
A supply cuive tells us the quantity that will be pioduced at each piice, and that is
what the fim's maiginal cost cuive tells us. The fim's supply cuive in the shoit iun is
its maiginal cost cuive foi piices above the aveiage vaiiable cost. At piices below avei-
age vaiiable cost, the fim's output diops to zeio.
Panel (a) of Figuie 9.10 shows the aveiage vaiiable cost and maiginal cost cuives
foi a hypothetical astiologei, Madame LaFaige, who is in the business of pioviding as-
tiological consultations ovei the telephone. We shall assume that this industiy is pei-
fectly competitive. At any piice below $10 pei call, Madame LaFaige would shut down.
If the piice is $10 oi gieatei, howevei, she pioduces an output at which piice equals
maiginal cost. The maiginal cost cuive is thus hei supply cuive at all piices gieatei
than $10.
II GUR 9. 10 MarginaI Cost and SuppIy
|e supp|y cu.ve |c. . |.n |s t|.t pc.t|cr c| |ts / cu.ve t|.t ||es .bcve t|e ^v cu.ve, s|c.r |r
|.re| (.). c cbt.|r t|e s|c.t.ur supp|y cu.ve |c. t|e |rJust.y, .e .JJ t|e cutputs c| e.c| |.n .t e.c| p.|ce. |e
|rJust.y supp|y cu.ve |s g|ver |r |.re| (b).
Now suppose that the astiological foiecast industiy consists of Madame LaFaige and thousands of oth-
ei fims similai to heis. The maiket supply cuive is found by adding the outputs of each fim at each
piice, as shown in Panel (b) of Figuie 9.10. At a piice of $10 pei call, foi example, Madame LaFaige
supplies 14 calls pei day. Adding the quantities supplied by all the othei fims in the maiket, suppose
we get a quantity supplied of 280,000. Notice that the maiket supply cuive we have diawn is lineai;
thioughout the book we have made the assumption that maiket demand and supply cuives aie lineai
in oidei to simplify oui analysis.
Looking at Figuie 9.10, we see that pioft-maximizing choices by fims in a peifectly competitive
maiket will geneiate a maiket supply cuive that iefects maiginal cost. Piovided theie aie no exteinal
236 PRINCIPLS OI CONOMICS
benefts oi costs in pioducing a good oi seivice, a peifectly competitive maiket satisfes the emciency
condition.
k Y 7 A k A W A Y S
< |.|ce |r . pe.|ect|y ccnpet|t|ve |rJust.y |s Jete.n|reJ by t|e |rte..ct|cr c| Jen.rJ .rJ supp|y.
< |r . pe.|ect|y ccnpet|t|ve |rJust.y, . |.ns tct.| .everue cu.ve |s . st..|g|t, up...Js|cp|rg ||re .|cse
s|cpe |s t|e n..ket p.|ce. |ccrcn|c p.c|t |s n.x|n|.eJ .t t|e cutput |eve| .t .||c| t|e s|cpes c| t|e tct.|
.everue .rJ tct.| ccst cu.ves ..e e,u.|, p.cv|JeJ t|.t t|e |.n |s ccve.|rg |ts v..|.b|e ccst.
< c use t|e n..g|r.| Jec|s|cr .u|e |r p.c|t n.x|n|..t|cr, t|e |.n p.cJuces t|e cutput .t .||c| n..g|r.|
ccst e,u.|s n..g|r.| .everue. |ccrcn|c p.c|t pe. ur|t |s p.|ce n|rus .ve..ge tct.| ccst, tct.| eccrcn|c
p.c|t e,u.|s eccrcn|c p.c|t pe. ur|t t|nes ,u.rt|ty.
< || p.|ce |.||s be|c. .ve..ge tct.| ccst, but .en.|rs .bcve .ve..ge v..|.b|e ccst, t|e |.n .||| ccrt|rue tc
cpe..te |r t|e s|c.t .ur, p.cJuc|rg t|e ,u.rt|ty .|e.e MR = MC Jc|rg sc n|r|n|.es |ts |csses.
< || p.|ce |.||s be|c. .ve..ge v..|.b|e ccst, t|e |.n .||| s|ut Jc.r |r t|e s|c.t .ur, .eJuc|rg cutput tc .e.c.
|e |c.est pc|rt cr t|e .ve..ge v..|.b|e ccst cu.ve |s c.||eJ t|e s|utJc.r pc|rt.
< |e |.ns supp|y cu.ve |r t|e s|c.t .ur |s |ts n..g|r.| ccst cu.ve |c. p.|ces g.e.te. t|.r t|e n|r|nun
.ve..ge v..|.b|e ccst.
7 R Y I 7 !
Assune t|.t Acne |ct||rg, t|e |.n |rt.cJuceJ |r t|e c|.pte. cr p.cJuct|cr .rJ ccst, p.cJuces .ckets |r .
pe.|ect|y ccnpet|t|ve n..ket. Suppcse t|e Jen.rJ .rJ supp|y cu.ves |c. .ckets |rte.sect .t . p.|ce c| 81.
|c., us|rg t|e n..g|r.| ccst .rJ .ve..ge tct.| ccst cu.ves |c. Acne s|c.r |e.e.
|st|n.te Acnes p.c|tn.x|n|.|rg cutput pe. J.y (.ssune t|e |.n se|ects . .|c|e runbe.). \|.t ..e
Acnes eccrcn|c p.c|ts pe. J.y`
CHAP7R 9 COMP7I7IV MARk7S IOR GOODS AND SRVICS 237
Case in Point: Not Out of 8usiness '7iI 7hey IaII from the Sky
2010 jupiterimages Corporation
|e 66 s.te|||tes .e.e pc|seJ tc st..t |.|||rg |.cn t|e sky. |e |cpe ..s t|.t t|e p|eces .cu|J bu.r tc b|ts cr
t|e|. ..y Jc.r t|.cug| t|e .tncsp|e.e, but t|e.e ..s t|e c|.rce t|.t . bu||J|rg c. . pe.scr .cu|J t.ke .
J|.ect ||t.
|e s.te|||tes .e.e t|e p.|n..y ccnnur|c.t|cr Jev|ces c| |.|J|uns s.te|||te p|cre systen. begur |r 1998 .s
t|e |.st t.u|y g|cb.| s.te|||te systen |c. ncb||e p|cresp.cv|J|rg ccnnur|c.t|crs .c.css Jese.ts, |r t|e
n|JJ|e c| cce.rs, .rJ .t t|e pc|es|.|J|un expecteJ |ve n||||cr subsc.|be.s tc p.y . n|rute tc t.|k cr
3,000 |.rJsets. |r t|e c||n.te c| t|e |.te 1990s, use.s cpteJ |c. c|e.pe., t|cug| |ess secu.e .rJ |ess ccnp.e
|ers|ve, ce|| p|cres. by t|e erJ c| t|e Jec.Je, |.|J|un |.J Jec|..eJ b.rk.uptcy, s|ut Jc.r cpe..t|crs, .rJ
..s ust ..|t|rg |c. t|e s.te|||tes tc st..t p|urg|rg |.cn t|e|. c.b|ts ..curJ 200.
|e cr|y c|e. |c. |.|J|uns 5 b||||cr systen c.ne |.cn .r ex|O c| . ruc|e.. .e.ctc. bus|ress, |.r c|ussy,
.rJ |t ..s |c. . ne.s|y 25 n||||cr. |ts ||ke p|ck|rg up . 150,000 |c.sc|e 911 |c. 50, ..cte |.^ oJo, .e
pc.te., |ev|r V.rey.
|e pu.c|.se tu.reJ |rtc . bcr.r... |r t|e ..ke c| Septenbe. 11, 2001, .rJ t|er t|e ...s |r A|g|.r|st.r
.rJ |..,, Jen.rJ |c. secu.e ccnnur|c.t|crs |r .encte |cc.t|crs sky.ccketeJ. |e. custcne.s |rc|uJeJ t|e
|.S. .rJ b.|t|s| n|||t..|es, .s .e|| .s .epc.te.s |r |..,, .|c, .|er t..ve||rg .|t| t|e n|||t..y |.ve beer b...eJ
|.cn us|rg |ess secu.e systens t|.t ..e e.s|e. tc t..ck. |e rcrp.c|t c.g.r|..t|cr Ope..t|cr .|| |cne |.s
bcug|t t|ne tc .||c. nenbe.s c| t|e 81
st
A.nc. b.|g.Je c| t|e \.s||rgtcr |.t|cr.| Cu..J tc ccnnur|c.te
.|t| t|e|. |.n|||es .t |cne. A|.||res .rJ s||pp|rg ||res |.ve .|sc s|greJ up.
As t|e re. |.|J|un bec.ne urbu.JereJ |.cn t|e Jebt c| t|e c|J cre .rJ tec|rc|cgy |np.cveJ, t|e |c.e.
|xeJ .rJ v..|.b|e ccsts |.ve ccrt.|buteJ tc |.|J|uns .ev|v.|, but c|e..|y . c.|t|c.| e|enert |r t|e tu.r..curJ
|.s beer |rc.e.seJ Jen.rJ. |e |.urc||rg c| .r .JJ|t|cr.| sever sp..e s.te|||tes .rJ ct|e. t|rke.|rg |.ve ex
terJeJ t|e |||e c| t|e systen tc .t |e.st 2014. |e |.n ..s tenpc...||y s|ut Jc.r but, .|t| |ts re. c.re.s
.rJ re. Jen.rJ |c. |ts se.v|ces, |.s ccne .c..|rg b.ck.
238 PRINCIPLS OI CONOMICS
\|y J|J c|ussy buy |.|J|un` A tcp execut|ve |r t|e re. |.n s.|J t|.t c|ussy ust |curJ t|e e||n|r.t|cr c|
t|e s.te|||tes . te..|b|e ..ste. |e.|.ps |e |.J scne r|c|e uses |r n|rJ, .s ever be|c.e Septenbe. 11, 2001, |e
|.J begur tc er.c|| scne re. custcne.s, suc| .s t|e c|cnb|.r r.t|cr.| pc||ce, .|c rc Jcubt |curJ t|e
systen use|u| |r t|e |g|t|rg J.ug |c.Js. but |t ..s |r t|e .|te.n.t| c| 9/11 t|.t |ts subsc.|be. ||st .e.||y beg.r
tc g.c. .rJ |ts .ecper|rg ..s JeeneJ . st.cke c| ger|us. cJ.y |.|J|uns custcne.s |rc|uJe s||ps .t se.
(.||c| .cccurt |c. .bcut |.|| c| |ts bus|ress), .|.||res, n|||t..y uses, .rJ . v..|ety c| ccnne.c|.| .rJ |un.r|t
..|.r .pp||c.t|crs.
.ooce |e.| /oe,. |eee |oe ||J|o Oo| |o |o|| .o|e. |eoe |o |o| `oo |o|.' |.^ oJo,. ^|| 9. 2003 3| /|c|oe| /ec|o.
|oJ|e|J oeoc| ^ |eoec|eJ ||J|o oo| ^.|o||o. ^|||eo| ^o || Oo.|| ||e|J.' ^.|o||o /ee| oJ .oce ec|o|o,. 6 9
(.e|ee 6. 2004: 5S ||J|o .eoe co e |ooJ o| ||J|oco
A N S W R 7 O 7 R Y I 7 ! P R O 8 L M
At . p.|ce c| 81, Acnes n..g|r.| .everue cu.ve |s . |c.|.crt.| ||re .t 81. |e |.n p.cJuces t|e cutput .t
.||c| n..g|r.| ccst e,u.|s n..g|r.| .everue, t|e cu.ves |rte.sect .t . ,u.rt|ty c| 9 .ckets pe. J.y. Acnes
.ve..ge tct.| ccst .t t||s |eve| c| cutput e,u.|s 6, |c. .r eccrcn|c p.c|t pe. .cket c| 14. Acnes eccrcn|c
p.c|t pe. J.y e,u.|s .bcut 126.
3. PERFECT COMPETITION IN THE LONG RUN
L A R N I N G O 8 1 C 7 I V S
1. Distinguish between economic prot and accounting prot.
2. xpIain why in Iong-run equiIibrium in a perfectIy competitive industry rms wiII earn zero eco-
nomic prot.
3. Describe the three possibIe eects on the costs of the factors of production that expansion or
contraction of a perfectIy competitive industry may have and iIIustrate the resuIting Iong-run
industry suppIy curve in each case.
4. xpIain why under perfection competition output prices wiII change by Iess than the change in
production cost in the short run, but by the fuII amount of the change in production cost in the
Iong run.
5. xpIain the eect of a change in xed cost on price and output in the short run and in the Iong
run under perfect competition.
In the long iun, a fim is fiee to adjust all of its inputs. New fims can entei any maiket; existing fims
can leave theii maikets. We shall see in this section that the model of peifect competition piedicts that,
at a long-iun equilibiium, pioduction takes place at the lowest possible cost pei unit and that all eco-
nomic piofts and losses aie eliminated.
CHAP7R 9 COMP7I7IV MARk7S IOR GOODS AND SRVICS 239
expIicit costs
|..ges t|.t nust be p.|J |c.
|.ctc.s c| p.cJuct|cr suc| .s
|.bc. .rJ c.p|t.|.
accounting prot
|.c|t ccnputeJ us|rg cr|y
exp||c|t ccsts.
impIicit cost
A ccst t|.t |s |rc|uJeJ |r t|e
eccrcn|c ccrcept c|
cppc.tur|ty ccst but t|.t |s
rct .r exp||c|t ccst.
3.1 Economic Piofit and Economic Loss
Economic piofts and losses play a ciucial iole in the model of peifect competition. The existence of
economic piofts in a paiticulai industiy attiacts new fims to the industiy in the long iun. As new
fims entei, the supply cuive shifts to the iight, piice falls, and piofts fall. Fiims continue to entei the
industiy until economic piofts fall to zeio. If fims in an industiy aie expeiiencing economic losses,
some will leave. The supply cuive shifts to the left, incieasing piice and ieducing losses. Fiims continue
to leave until the iemaining fims aie no longei sufeiing losses-until economic piofts aie zeio.
Befoie examining the mechanism thiough which entiy and exit eliminate economic piofts and
losses, we shall examine an impoitant key to undeistanding it: the difeience between the accounting
and economic concepts of pioft and loss.
conomic Versus Accounting Concepts of Profit and Loss
Economic pioft equals total ievenue minus total cost, wheie cost is measuied in the economic sense as
oppoitunity cost. An economic loss (negative economic pioft) is incuiied if total cost exceeds total
ievenue.
Accountants include only explicit costs in theii computation of total cost. Explicit costs include
chaiges that must be paid foi factois of pioduction such as laboi and capital, togethei with an estimate
of depieciation. Pioft computed using only explicit costs is called accounting pront. It is the meas-
uie of pioft fims typically iepoit; fims pay taxes on theii accounting piofts, and a coipoiation ie-
poiting its pioft foi a paiticulai peiiod iepoits its accounting piofts. To compute his accounting
piofts, Mi. Goitaii, the iadish faimei, would subtiact explicit costs, such as chaiges foi laboi, equip-
ment, and othei supplies, fiom the ievenue he ieceives.
Economists iecognize costs in addition to the explicit costs listed by accountants. If Mi. Goitaii
weie not giowing iadishes, he could be doing something else with the land and with his own efoits.
Suppose the most valuable alteinative use of his land would be to pioduce caiiots, fiom which Mi.
Goitaii could eain $230 pei month in accounting piofts. The income he foigoes by not pioducing cai-
iots is an oppoitunity cost of pioducing iadishes. This cost is not explicit; the ietuin Mi. Goitaii could
get fiom pioducing caiiots will not appeai on a conventional accounting statement of his accounting
pioft. A cost that is included in the economic concept of oppoitunity cost, but that is not an explicit
cost, is called an implicit cost.
7he Long Run and 7ero conomic Profits
Given oui defnition of economic piofts, we can easily see why, in peifect competition, they must al-
ways equal zeio in the long iun. Suppose theie aie two industiies in the economy, and that fims in In-
dustiy A aie eaining economic piofts. By defnition, fims in Industiy A aie eaining a ietuin gieatei
than the ietuin available in Industiy B. That means that fims in Industiy B aie eaining less than they
could in Industiy A. Fiims in Industiy B aie expeiiencing economic losses.
Given easy entiy and exit, some fims in Industiy B will leave it and entei Industiy A to eain the
gieatei piofts available theie. As they do so, the supply cuive in Industiy B will shift to the left, in-
cieasing piices and piofts theie. As foimei Industiy B fims entei Industiy A, the supply cuive in In-
dustiy A will shift to the iight, loweiing piofts in A. The piocess of fims leaving Industiy B and entei-
ing A will continue until fims in both industiies aie eaining zeio economic pioft. That suggests an
impoitant long-iun iesult: Economic profts in a system of perfectly competitive markets will, in the long
run, be driven to zero in all industries.
Iiminating conomic Profit: 7he RoIe of ntry
The piocess thiough which entiy will eliminate economic piofts in the long iun is illustiated in Figuie
9.14, which is based on the situation piesented in Figuie 9.7. The piice of iadishes is $0.40 pei pound.
Mi. Goitaii's aveiage total cost at an output of 6,700 pounds of iadishes pei month is $0.26 pei pound.
Pioft pei unit is $0.14 ($0.40 $0.26 ). Mi. Goitaii thus eains a pioft of $938 pei month (
= $0.14 6, 700 ).
240 PRINCIPLS OI CONOMICS
II GUR 9. 14 Iiminating conomic Profits in the Long Run
|| |.ns |r .r |rJust.y ..e n.k|rg .r eccrcn|c p.c|t, ert.y .||| cccu. |r t|e |crg .ur. |r |.re| (b), . s|rg|e |.ns
p.c|t |s s|c.r by t|e s|.JeJ ..e.. |rt.y ccrt|rues urt|| |.ns |r t|e |rJust.y ..e cpe..t|rg .t t|e |c.est pc|rt cr
t|e|. .espect|ve .ve..ge tct.| ccst cu.ves, .rJ eccrcn|c p.c|ts |.|| tc .e.c.
Piofts in the iadish industiy attiact entiy in the long iun. Panel (a) of Figuie 9.14 shows that as fims
entei, the supply cuive shifts to the iight and the piice of iadishes falls. New fims entei as long as theie
aie economic piofts to be made-as long as piice exceeds ATC in Panel (b). As piice falls, maiginal
ievenue falls to MR
2
and the fim ieduces the quantity it supplies, moving along the maiginal cost
(MC) cuive to the lowest point on the ATC cuive, at $0.22 pei pound and an output of 3,000 pounds
pei month. Although the output of individual fims falls in iesponse to falling piices, theie aie now
moie fims, so industiy output iises to 13 million pounds pei month in Panel (a).
Iiminating Losses: 7he RoIe of xit
Just as entiy eliminates economic piofts in the long iun, exit eliminates economic losses. In Figuie
9.13, Panel (a) shows the case of an industiy in which the maiket piice P
1
is below ATC. In Panel (b), at
piice P
1
a single fim pioduces a quantity q
1
, assuming it is at least coveiing its aveiage vaiiable cost.
The fim's losses aie shown by the shaded iectangle bounded by its aveiage total cost C
1
and piice P
1
and by output q
1
.
Because fims in the industiy aie losing money, some will exit. The supply cuive in Panel (a) shifts
to the left, and it continues shifting as long as fims aie sufeiing losses. Eventually the supply cuive
shifts all the way to S
2
, piice iises to P
2
, and economic piofts ietuin to zeio.
CHAP7R 9 COMP7I7IV MARk7S IOR GOODS AND SRVICS 241
constant-cost industry
|rJust.y |r .||c| exp.rs|cr
Jces rct .|ect t|e p.|ces c|
|.ctc.s c| p.cJuct|cr.
increasing-cost industry
|rJust.y |r .||c| t|e ert.y c|
re. |.ns b|Js up t|e p.|ces
c| |.ctc.s c| p.cJuct|cr .rJ
t|us |rc.e.ses p.cJuct|cr
ccsts.
decreasing-cost industry
|rJust.y |r .||c| p.cJuct|cr
ccsts |.|| |r t|e |crg .ur .s
|.ns erte..
II GUR 9. 15 Iiminating conomic Losses in the Long Run
|.re| (b) s|c.s t|.t .t t|e |r|t|.| p.|ce |
1
, |.ns |r t|e |rJust.y c.rrct ccve. .ve..ge tct.| ccst (/|
1
|s be|c. ^).
|.t |rJuces scne |.ns tc |e.ve t|e |rJust.y, s|||t|rg t|e supp|y cu.ve |r |.re| (.) tc .
2
, .eJuc|rg |rJust.y cutput
tc (
2
.rJ ..|s|rg p.|ce tc |
2
. At t|.t p.|ce (/|
2
), |.ns e..r .e.c eccrcn|c p.c|t, .rJ ex|t |.cn t|e |rJust.y ce.ses.
|.re| (b) s|c.s t|.t t|e |.n |rc.e.ses cutput |.cn
1
tc
2
, tct.| cutput |r t|e n..ket |.||s |r |.re| (.) bec.use
t|e.e ..e |e.e. |.ns. |ct|ce t|.t |r |.re| (.) ,u.rt|ty |s Jes|gr.teJ by uppe.c.se (, .|||e |r |.re| (b) ,u.rt|ty |s
Jes|gr.teJ by |c.e.c.se . ||s ccrvert|cr |s useJ t|.cug|cut t|e text tc J|st|rgu|s| bet.eer t|e ,u.rt|ty
supp||eJ |r t|e n..ket (() .rJ t|e ,u.rt|ty supp||eJ by . typ|c.| |.n ().
ntry, xit, and Production Costs
In oui examination of entiy and exit in iesponse to economic pioft oi loss in a peifectly competitive
industiy, we assumed that the ATC cuive of a single fim does not shift as new fims entei oi existing
fims leave the industiy. That is the case when expansion oi contiaction does not afect piices foi the
factois of pioduction used by fims in the industiy. When expansion of the industiy does not afect the
piices of factois of pioduction, it is a constant-cost industry. In some cases, howevei, the entiy of
new fims may afect input piices.
As new fims entei, they add to the demand foi the factois of pioduction used by the industiy. If
the industiy is a signifcant usei of those factois, the inciease in demand could push up the maiket
piice of factois of pioduction foi all fims in the industiy. If that occuis, then entiy into an industiy
will boost aveiage costs at the same time as it puts downwaid piessuie on piice. Long-iun equilibiium
will still occui at a zeio level of economic pioft and with fims opeiating on the lowest point on the
ATC cuive, but that cost cuive will be somewhat highei than befoie entiy occuiied. Suppose, foi ex-
ample, that an inciease in demand foi new houses diives piices highei and induces entiy. That will in-
ciease the demand foi woikeis in the constiuction industiy and is likely to iesult in highei wages in the
industiy, diiving up costs.
An industiy in which the entiy of new fims bids up the piices of factois of pioduction and thus
incieases pioduction costs is called an increasing-cost industry. As such an industiy expands in the
long iun, its piice will iise.
Some industiies may expeiience ieductions in input piices as they expand with the entiy of new
fims. That may occui because fims supplying the industiy expeiience economies of scale as they in-
ciease pioduction, thus diiving input piices down. Expansion may also induce technological changes
that lowei input costs. That is cleaily the case of the computei industiy, which has enjoyed falling input
costs as it has expanded. An industiy in which pioduction costs fall as fims entei in the long iun is a
decreasing-cost industry.
Just as industiies may expand with the entiy of new fims, they may contiact with the exit of exist-
ing fims. In a constant-cost industiy, exit will not afect the input piices of iemaining fims. In an
incieasing-cost industiy, exit will ieduce the input piices of iemaining fims. And, in a decieasing-cost
industiy, input piices may iise with the exit of existing fims.
242 PRINCIPLS OI CONOMICS
Iong-run industry suppIy
curve
A cu.ve t|.t .e|.tes t|e p.|ce
c| . gccJ c. se.v|ce tc t|e
,u.rt|ty p.cJuceJ .|te. .||
|crg.ur .Justnerts tc .
p.|ce c|.rge |.ve beer
ccnp|eteJ.
The behavioi of pioduction costs as fims in an industiy expand oi ieduce theii output has im-
poitant implications foi the long-run industry supply curve, a cuive that ielates the piice of a good
oi seivice to the quantity pioduced aftei all long-iun adjustments to a piice change have been com-
pleted. Eveiy point on a long-iun supply cuive theiefoie shows a piice and quantity supplied at which
fims in the industiy aie eaining zeio economic pioft. Unlike the shoit-iun maiket supply cuive, the
long-iun industiy supply cuive does not hold factoi costs and the numbei of fims unchanged.
Figuie 9.16 shows thiee long-iun industiy supply cuives. In Panel (a), S
CC
is a long-iun supply
cuive foi a constant-cost industiy. It is hoiizontal. Neithei expansion noi contiaction by itself afects
maiket piice. In Panel (b), S
IC
is a long-iun supply cuive foi an incieasing-cost industiy. It iises as the
industiy expands. In Panel (c), S
DC
is a long-iun supply cuive foi a decieasing-cost industiy. Its down-
waid slope suggests a falling piice as the industiy expands.
II GUR 9. 16 Long-Run SuppIy Curves in Perfect Competition
|e |crg.ur supp|y cu.ve |c. . ccrst.rtccst, pe.|ect|y ccnpet|t|ve |rJust.y |s . |c.|.crt.| ||re, .

, s|c.r |r
|.re| (.). |e |crg.ur cu.ve |c. .r |rc.e.s|rgccst |rJust.y |s .r up...Js|cp|rg cu.ve, .
|
, .s |r |.re| (b). |e
Jc.r...Js|cp|rg |crg.ur supp|y cu.ve, .
|
, |c. . Jec.e.s|rg ccst |rJust.y |s g|ver |r |.re| (c).
3.2 Changes in Demand and in Pioduction Cost
The piimaiy application of the model of peifect competition is in piedicting how fims will iespond to
changes in demand and in pioduction costs. To see how fims iespond to a paiticulai change, we de-
teimine how the change afects demand oi cost conditions and then see how the pioft-maximizing
solution is afected in the shoit iun and in the long iun. Having deteimined how the pioft-maximizing
fims of the model would iespond, we can then piedict fims' iesponses to similai changes in the ieal
woild.
In the examples that follow, we shall assume, foi simplicity, that entiy oi exit do not afect the in-
put piices facing fims in the industiy. That is, we assume a constant-cost industiy with a hoiizontal
long-iun industiy supply cuive similai to S
CC
in Figuie 9.16. We shall assume that fims aie coveiing
theii aveiage vaiiable costs, so we can ignoie the possibility of shutting down.
Changes in Demand
Changes in demand can occui foi a vaiiety of ieasons. Theie may be a change in piefeiences, incomes,
the piice of a ielated good, population, oi consumei expectations. A change in demand causes a change
in the maiket piice, thus shifting the maiginal ievenue cuives of fims in the industiy.
Let us considei the impact of a change in demand foi oats. Suppose new evidence suggests that
eating oats not only helps to pievent heait disease, but also pievents baldness in males. This will, of
couise, inciease the demand foi oats. To assess the impact of this change, we assume that the industiy
is peifectly competitive and that it is initially in long-iun equilibiium at a piice of $1.70 pei bushel.
Economic piofts equal zeio.
The initial situation is depicted in Figuie 9.17. Panel (a) shows that at a piice of $1.70, industiy
output is Q
1
(point A), while Panel (b) shows that the maiket piice constitutes the maiginal ievenue,
MR
1
, facing a single fim in the industiy. The fim iesponds to that piice by fnding the output level at
which the MC and MR
1
cuives inteisect. That implies a level of output q
1
at point A.
The new medical evidence causes demand to inciease to D
2
in Panel (a). That incieases the maiket
piice to $2.30 (point B), so the maiginal ievenue cuive foi a single fim iises to MR
2
in Panel (b). The
fim iesponds by incieasing its output to q
2
in the shoit iun (point B). Notice that the fim's aveiage
total cost is slightly highei than its oiiginal level of $1.70; that is because of the U shape of the cuive.
The fim is making an economic pioft shown by the shaded iectangle in Panel (b). Othei fims in the
industiy will eain an economic pioft as well, which, in the long iun, will attiact entiy by new fims.
CHAP7R 9 COMP7I7IV MARk7S IOR GOODS AND SRVICS 243
New entiy will shift the supply cuive to the iight; entiy will continue as long as fims aie making an
economic pioft. The supply cuive in Panel (a) shifts to S
2
, diiving the piice down in the long iun to
the oiiginal level of $1.70 pei bushel and ietuining economic piofts to zeio in long-iun equilibiium. A
single fim will ietuin to its oiiginal level of output, q
1
(point A) in Panel (b), but because theie aie
moie fims in the industiy, industiy output iises to Q
3
(point C) in Panel (a).
II GUR 9. 17 Short-Run and Long-Run Ad|ustments to an Increase in Demand
|e |r|t|.| e,u|||b.|un p.|ce .rJ cutput ..e Jete.n|reJ |r t|e n..ket |c. c.ts by t|e |rte.sect|cr c| Jen.rJ .rJ
supp|y .t pc|rt A |r |.re| (.). Ar |rc.e.se |r t|e n..ket Jen.rJ |c. c.ts, |.cn |
1
tc |
2
|r |.re| (.), s|||ts t|e
e,u|||b.|un sc|ut|cr tc pc|rt b. |e p.|ce |rc.e.ses |r t|e s|c.t .ur |.cn 1.0 pe. bus|e| tc 2.30. |rJust.y cutput
.|ses tc (
2
. |c. . s|rg|e |.n, t|e |rc.e.se |r p.|ce ..|ses n..g|r.| .everue |.cn /|
1
tc /|
2
, t|e |.n .espcrJs |r t|e
s|c.t .ur by |rc.e.s|rg |ts cutput tc
2
. |t e..rs .r eccrcn|c p.c|t g|ver by t|e s|.JeJ .ect.rg|e. |r t|e |crg .ur,
t|e cppc.tur|ty |c. p.c|t .tt..cts re. |.ns. |r . ccrst.rtccst |rJust.y, t|e s|c.t.ur supp|y cu.ve s|||ts tc .
2
,
n..ket e,u|||b.|un rc. ncves tc pc|rt |r |.re| (.). |e n..ket p.|ce |.||s b.ck tc 1.0. |e |.ns Jen.rJ cu.ve
.etu.rs tc /|
1
, .rJ |ts cutput |.||s b.ck tc t|e c.|g|r.| |eve|,
1
. |rJust.y cutput |.s .|ser tc (
3
bec.use t|e.e ..e
nc.e |.ns.
A ieduction in demand would lead to a ieduction in piice, shifting each fim's maiginal ievenue cuive
downwaid. Fiims would expeiience economic losses, thus causing exit in the long iun and shifting the
supply cuive to the left. Eventually, the piice would iise back to its oiiginal level, assuming changes in
industiy output did not lead to changes in input piices. Theie would be fewei fims in the industiy, but
each fim would end up pioducing the same output as befoie.
Changes in Production Cost
A fim's costs change if the costs of its inputs change. They also change if the fim is able to take ad-
vantage of a change in technology. Changes in pioduction cost shift the ATC cuive. If a fim's vaiiable
costs aie afected, its maiginal cost cuives will shift as well. Any change in maiginal cost pioduces a
similai change in industiy supply, since it is found by adding up maiginal cost cuives foi individual
fims.
Suppose a ieduction in the piice of oil ieduces the cost of pioducing oil changes foi automobiles.
We shall assume that the oil-change industiy is peifectly competitive and that it is initially in long-iun
equilibiium at a piice of $27 pei oil change, as shown in Panel (a) of Figuie 9.18. Suppose that the ie-
duction in oil piices ieduces the cost of an oil change by $3.
244 PRINCIPLS OI CONOMICS
II GUR 9. 18 A Reduction in the Cost of Producing OiI Changes
|e |r|t|.| e,u|||b.|un p.|ce, 2, .rJ ,u.rt|ty, (
1
, c| .utcncb||e c|| c|.rges ..e Jete.n|reJ by t|e |rte.sect|cr c|
n..ket Jen.rJ, |
1
, .rJ n..ket supp|y, .
1
|r |.re| (.). |e |rJust.y |s |r |crg.ur e,u|||b.|un, . typ|c.| |.n, s|c.r
|r |.re| (b), e..rs .e.c eccrcn|c p.c|t. A .eJuct|cr |r c|| p.|ces .eJuces t|e n..g|r.| .rJ .ve..ge tct.| ccsts c|
p.cJuc|rg .r c|| c|.rge by 3. |e |.ns n..g|r.| ccst cu.ve s|||ts tc /
2
, .rJ |ts .ve..ge tct.| ccst cu.ve s|||ts tc
^
2
. |e s|c.t.ur |rJust.y supp|y cu.ve s|||ts Jc.r by 3 tc .
2
. |e n..ket p.|ce |.||s tc 26, t|e |.n |rc.e.ses |ts
cutput tc
2
.rJ e..rs .r eccrcn|c p.c|t g|ver by t|e s|.JeJ .ect.rg|e. |r t|e |crg .ur, t|e cppc.tur|ty |c. p.c|t
s|||ts t|e |rJust.y supp|y cu.ve tc .
3
. |e p.|ce |.||s tc 24, .rJ t|e |.n .eJuces |ts cutput tc t|e c.|g|r.| |eve|,
1
. |t
rc. e..rs .e.c eccrcn|c p.c|t crce .g.|r. |rJust.y cutput |r |.re| (.) .|ses tc (
3
bec.use t|e.e ..e nc.e |.ns,
p.|ce |.s |.||er by t|e |u|| .ncurt c| t|e .eJuct|cr |r p.cJuct|cr ccsts.
A ieduction in pioduction cost shifts the fim's cost cuives down. The fim's aveiage total cost and
maiginal cost cuives shift down, as shown in Panel (b). In Panel (a) the supply cuive shifts fiom S
1
to
S
2
. The industiy supply cuive is made up of the maiginal cost cuives of individual fims; because each
of them has shifted downwaid by $3, the industiy supply cuive shifts downwaid by $3.
Notice that piice in the shoit iun falls to $26; it does not fall by the $3 ieduction in cost. That is be-
cause the supply and demand cuives aie sloped. While the supply cuive shifts downwaid by $3, its in-
teisection with the demand cuive falls by less than $3. The fim in Panel (b) iesponds to the lowei piice
and lowei cost by incieasing output to q
2
, wheie MC
2
and MR
2
inteisect. That leaves fims in the in-
dustiy with an economic pioft; the economic pioft foi the fim is shown by the shaded iectangle in
Panel (b). Piofts attiact entiy in the long iun, shifting the supply cuive to the iight to S
3
in Panel (a)
Entiy will continue as long as fims aie making an economic pioft-it will thus continue until the
piice falls by the full amount of the $3 ieduction in cost. The piice falls to $24, industiy output iises to
Q
3
, and the fim's output ietuins to its oiiginal level, q
1
.
An inciease in vaiiable costs would shift the aveiage total, aveiage vaiiable, and maiginal cost
cuives upwaid. It would shift the industiy supply cuive upwaid by the same amount. The iesult in the
shoit iun would be an inciease in piice, but by less than the inciease in cost pei unit. Fiims would ex-
peiience economic losses, causing exit in the long iun. Eventually, piice would inciease by the full
amount of the inciease in pioduction cost.
Some cost incieases will not afect maiginal cost. Suppose, foi example, that an annual license fee
of $3,000 is imposed on fims in a paiticulai industiy. The fee is a fxed cost; it does not afect maiginal
cost. Imposing such a fee shifts the aveiage total cost cuive upwaid but causes no change in maiginal
cost. Theie is no change in piice oi output in the shoit iun. Because fims aie sufeiing economic
losses, theie will be exit in the long iun. Piices ultimately iise by enough to covei the cost of the fee,
leaving the iemaining fims in the industiy with zeio economic pioft.
Piice will change to iefect whatevei change we obseive in pioduction cost. A change in vaiiable
cost causes piice to change in the shoit iun. In the long iun, any change in aveiage total cost changes
piice by an equal amount.
The message of long-iun equilibiium in a competitive maiket is a piofound one. The ultimate be-
nefciaiies of the innovative efoits of fims aie consumeis. Fiims in a peifectly competitive woild eain
zeio pioft in the long-iun. While fims can eain accounting piofts in the long-iun, they cannot eain
economic piofts.
CHAP7R 9 COMP7I7IV MARk7S IOR GOODS AND SRVICS 245
k Y 7 A k A W A Y S
< |e eccrcn|c ccrcept c| p.c|t J||e.s |.cn .cccurt|rg p.c|t. |e .cccurt|rg ccrcept Je.|s cr|y .|t|
exp||c|t ccsts, .|||e t|e eccrcn|c ccrcept c| p.c|t |rcc.pc..tes exp||c|t .rJ |np||c|t ccsts.
< |e ex|sterce c| eccrcn|c p.c|ts .tt..cts ert.y, eccrcn|c |csses |e.J tc ex|t, .rJ |r |crg.ur e,u|||b.|un,
|.ns |r . pe.|ect|y ccnpet|t|ve |rJust.y .||| e..r .e.c eccrcn|c p.c|t.
< |e |crg.ur supp|y cu.ve |r .r |rJust.y |r .||c| exp.rs|cr Jces rct c|.rge |rput p.|ces (. ccrst.rt
ccst |rJust.y) |s . |c.|.crt.| ||re. |e |crg.ur supp|y cu.ve |c. .r |rJust.y |r .||c| p.cJuct|cr ccsts
|rc.e.se .s cutput .|ses (.r |rc.e.s|rgccst |rJust.y) |s up...J s|cp|rg. |e |crg.ur supp|y cu.ve |c. .r
|rJust.y |r .||c| p.cJuct|cr ccsts Jec.e.se .s cutput .|ses (. Jec.e.s|rgccst |rJust.y) |s Jc.r...J
s|cp|rg.
< |r . pe.|ect|y ccnpet|t|ve n..ket |r |crg.ur e,u|||b.|un, .r |rc.e.se |r Jen.rJ c.e.tes eccrcn|c p.c|t
|r t|e s|c.t .ur .rJ |rJuces ert.y |r t|e |crg .ur, . .eJuct|cr |r Jen.rJ c.e.tes eccrcn|c |csses
(reg.t|ve eccrcn|c p.c|ts) |r t|e s|c.t .ur .rJ |c.ces scne |.ns tc ex|t t|e |rJust.y |r t|e |crg .ur.
< \|er p.cJuct|cr ccsts c|.rge, p.|ce .||| c|.rge by |ess t|.r t|e c|.rge |r p.cJuct|cr ccst |r t|e s|c.t
.ur. |.|ce .||| .Just tc .e|ect |u||y t|e c|.rge |r p.cJuct|cr ccst |r t|e |crg .ur.
< A c|.rge |r |xeJ ccst .||| |.ve rc e|ect cr p.|ce c. cutput |r t|e s|c.t .ur. |t .||| |rJuce ert.y c. ex|t |r
t|e |crg .ur sc t|.t p.|ce .||| c|.rge by ercug| tc |e.ve |.ns e..r|rg .e.c eccrcn|c p.c|t.
7 R Y I 7 !
crs|Je. Acne |ct||rgs s|tu.t|cr |r t|e seccrJ .y |t! |r t||s c|.pte.. Suppcse t||s s|tu.t|cr |s typ|c.| c|
|.ns |r t|e .cket n..ket. |xp|.|r .|.t .||| |.pper |r t|e n..ket |c. .ckets |r t|e |crg .ur, .ssun|rg rct|
|rg |.ppers tc t|e p.|ces c| |.ctc.s c| p.cJuct|cr useJ by |.ns |r t|e |rJust.y. \|.t .||| |.pper tc t|e e,u|
||b.|un p.|ce` \|.t |s t|e e,u|||b.|un |eve| c| eccrcn|c p.c|ts`
Case in Point: Competition in the Market for Generic Prescription Drugs
2010 jupiterimages Corporation
246 PRINCIPLS OI CONOMICS
Cere.|c p.esc.|pt|cr J.ugs ..e essert|.||y |Jert|c.| subst|tutes |c. nc.e expers|ve b..rJr.ne p.esc.|pt|cr
J.ugs. S|rce t|e p.ss.ge c| t|e |.ug cnpet|t|cr .rJ |.tert e.n |estc..t|cr Act c| 1984 (ccnncr|y .e
|e..eJ tc .s t|e |.tc|\.xn.r Act) n.Je |t e.s|e. |c. n.ru|.ctu.e.s tc erte. t|e n..ket |c. gere.|c J.ugs,
t|e gere.|c J.ug |rJust.y |.s t.ker c|. Cere.|c J.ugs .ep.eserteJ 19 c| t|e p.esc.|pt|cr J.ug |rJust.y |r
1984 .rJ tcJ.y .ep.esert nc.e t|.r |.|| c| t|e |rJust.y. |.S. gere.|c s.|es .e.e 15 b||||cr |r 2002 .rJ sc..eJ
tc 192 b||||cr |r 2006. |r 2006, t|e .ve..ge p.|ce c| . b..rJeJ p.esc.|pt|cr ..s 111.02 ccnp..eJ tc 32.23
|c. . gere.|c p.esc.|pt|cr.
A crg.ess|cr.| buJget O|ce stuJy |r t|e |.te 1990s s|c.eJ t|.t ert.y |rtc t|e gere.|c J.ug |rJust.y |.s
beer t|e key tc t||s p.|ce J||e.ert|.|. As s|c.r |r t|e t.b|e, .|er t|e.e ..e cre tc |ve n.ru|.ctu.e.s se|||rg
gere.|c ccp|es c| . g|ver b..rJeJ J.ug, t|e ..t|c c| t|e gere.|c p.|ce tc t|e b..rJeJ p.|ce |s .bcut 60. \|t|
nc.e t|.r 20 ccnpet|tc.s, t|e ..t|c |.||s tc .bcut 40.
|e gere.|c J.ug |rJust.y |s |..ge|y c|...cte.|.eJ by t|e .tt.|butes c| . pe.|ect|y ccnpet|t|ve n..ket. cnpet
|tc.s |.ve gccJ |r|c.n.t|cr .bcut t|e p.cJuct .rJ se|| |Jert|c.| p.cJucts. |e |..gest gere.|c J.ug n.ru|.c
tu.e. |r t|e bO stuJy |.J . 16 s|..e c| t|e gere.|c J.ug n.ru|.ctu.|rg |rJust.y, but ncst gere.|c n.ru
|.ctu.e.s s.|es ccrst|tuteJ cr|y 1 tc 5 c| t|e n..ket. |e 1984 |eg|s|.t|cr e.seJ ert.y |rtc t||s n..ket. ArJ,
.s t|e ncJe| c| pe.|ect ccnpet|t|cr p.eJ|cts, ert.y |.s J.|ver p.|ces Jc.r, bere|t|rg ccrsune.s tc t|e ture
c| ters c| b||||crs c| Jc||..s e.c| ye...
Prlce Comorlson of Generlc onJ lnnovotor rugs, by Number of Monufocturers
Number of Generlc
Monufocturers of o
Glven lnnovotor rug
Number of
lnnovotor
rugs ln
Cotegory
Avg. kx Prlce, A//
Generlc rugs
ln Cotegory
Avg. kx Prlce, A//
lnnovotor rugs
ln Cotegory
Avg. kotlo of the Generlc
Prlce to the lnnovotor
Prlce for 5ome rug
1 tc 5 34 23.40 3.20 0.61
6 tc 10 26 26.40 42.60 0.61
11 tc 15 29 20.90 50.20 0.42
16 tc 20 19 19.90 45.00 0.46
21 tc 24 4 11.50 33.90 0.39
Averoge 22.40 43.00 0.53
.ooce oe|oo| |oJe| |ce. |o. |ceoeJ oe||||o |o Oee|c |o |o ^|ec|eJ ||ce oJ |e|o | ||e ||ooceo||co|
|Jo|,.' .o|, 99S ^.o||o|e o| ...coo.. Oee|c ||ooceo||co| |Jo|, ^||c|o|e |oo|e|||| Oo.||.' || |e..|e. /oc| .. 2004
^.o||o|e o| ...|e..|eco. 200S .|o||||co| ^|oc| o| ||e |||eJ .|o|e. o|e 30
A N S W R 7 O 7 R Y I 7 ! P R O 8 L M
|e .v.||.b|||ty c| eccrcn|c p.c|ts .||| .tt..ct re. |.ns tc t|e .cket |rJust.y |r t|e |crg .ur, s|||t|rg t|e
n..ket supp|y cu.ve tc t|e .|g|t. |rt.y .||| ccrt|rue urt|| eccrcn|c p.c|ts ..e e||n|r.teJ. |e p.|ce .||| |.||,
Acnes n..g|r.| .everue cu.ve s|||ts Jc.r. |e e,u|||b.|un |eve| c| eccrcn|c p.c|ts |r t|e |crg .ur |s .e.c.
4. REVIEW AND PRACTICE
Summary
|e .ssunpt|crs c| t|e ncJe| c| pe.|ect ccnpet|t|cr ersu.e t|.t eve.y Jec|s|cr n.ke. |s . p.|ce t.ke.t|e
|rte..ct|cr c| Jen.rJ .rJ supp|y |r t|e n..ket Jete.n|res p.|ce. A|t|cug| ncst |.ns |r .e.| n..kets |.ve
scne ccrt.c| cve. t|e|. p.|ces, t|e ncJe| c| pe.|ect ccnpet|t|cr suggests |c. c|.rges |r Jen.rJ c. |r p.c
Juct|cr ccst .||| .|ect p.|ce .rJ cutput |r . .|Je ..rge c| .e.|.c.|J c.ses.
A |.n |r pe.|ect ccnpet|t|cr n.x|n|.es p.c|t |r t|e s|c.t .ur by p.cJuc|rg .r cutput |eve| .t .||c| n..g|r
.| .everue e,u.|s n..g|r.| ccst, p.cv|JeJ n..g|r.| .everue |s .t |e.st .s g.e.t .s t|e n|r|nun v.|ue c| .ve.
.ge v..|.b|e ccst. |c. . pe.|ect|y ccnpet|t|ve |.n, n..g|r.| .everue e,u.|s p.|ce .rJ .ve..ge .everue. ||s
|np||es t|.t t|e |.ns n..g|r.| ccst cu.ve |s |ts s|c.t.ur supp|y cu.ve |c. v.|ues g.e.te. t|.r .ve..ge v..|.b|e
ccst. || p.|ce J.cps be|c. .ve..ge v..|.b|e ccst, t|e |.n s|uts Jc.r.
CHAP7R 9 COMP7I7IV MARk7S IOR GOODS AND SRVICS 247
|| |.ns |r .r |rJust.y ..e e..r|rg eccrcn|c p.c|t, ert.y by re. |.ns .||| J.|ve p.|ce Jc.r urt|| eccrcn|c
p.c|t .c||eves |ts |crg.ur e,u|||b.|un v.|ue c| .e.c. || |.ns ..e su|e.|rg eccrcn|c |csses, ex|t by ex|st|rg
|.ns .||| ccrt|rue urt|| p.|ce .|ses tc e||n|r.te t|e |csses .rJ eccrcn|c p.c|ts ..e .e.c. A |crg.ur e,u|||b.|
un n.y be c|.rgeJ by . c|.rge |r Jen.rJ c. |r p.cJuct|cr ccst, .||c| .cu|J .|ect supp|y. |e .Just
nert tc t|e c|.rge |r t|e s|c.t .ur |s ||ke|y tc .esu|t |r eccrcn|c p.c|ts c. |csses, t|ese .||| be e||n|r.teJ |r
t|e |crg .ur by ert.y c. by ex|t.
C O N C P 7 P R O 8 L M S
1. |xp|.|r |c. e.c| c| t|e .ssunpt|crs c| pe.|ect ccnpet|t|cr ccrt.|butes tc t|e |.ct t|.t .|| Jec|s|cr
n.ke.s |r pe.|ect ccnpet|t|cr ..e p.|ce t.ke.s.
2. || t|e .ssunpt|crs c| pe.|ect ccnpet|t|cr ..e rct ||ke|y tc be net |r t|e .e.| .c.|J, |c. c.r t|e ncJe| be
c| .ry use`
3. |xp|.|r t|e J||e.erce bet.eer n..g|r.| .everue, .ve..ge .everue, .rJ p.|ce |r pe.|ect ccnpet|t|cr.
4. Suppcse t|e cr|y ..y . |.n c.r |rc.e.se |ts s.|es |s tc |c.e. |ts p.|ce. |s t||s . pe.|ect|y ccnpet|t|ve |.n`
\|y c. .|y rct`
5. crs|Je. t|e |c||c.|rg gccJs .rJ se.v|ces. \||c| ..e t|e ncst ||ke|y tc be p.cJuceJ |r . pe.|ect|y
ccnpet|t|ve |rJust.y` \||c| ..e rct` |xp|.|r .|y ycu n.Je t|e c|c|ces ycu J|J, .e|.t|rg ycu. .rs.e. tc
t|e .ssunpt|crs c| t|e ncJe| c| pe.|ect ccnpet|t|cr.
.. cc.c|. .rJ |eps|
b. |ct.tces
c. |.|v.te p|ys|c|.rs |r ycu. |cc.| ccnnur|ty
J. Ccve.rnert bcrJs .rJ cc.pc..te stccks
e. .x|c.bs |r ||n., |e.u. c|ty t|.t Jces rct .est.|ct ert.y c. t|e p.|ces J.|ve.s c.r c|..ge
|. O.ts
6. |xp|.|r .|y .r eccrcn|c p.c|t c| .e.c |s .ccept.b|e tc . |.n.
. |xp|.|r .|y . pe.|ect|y ccnpet|t|ve |.n .|cse .ve..ge tct.| ccst exceeJs t|e n..ket p.|ce n.y ccrt|rue
tc cpe..te |r t|e s|c.t .ur. \|.t .bcut t|e |crg .ur`
8. +cu |.ve Jec|JeJ tc n.c. |r b|c|cgy ..t|e. t|.r ccnpute. sc|erce. A re.s .epc.t suggests t|.t t|e
s.|..|es c| ccnpute. sc|erce n.c.s ..e |rc.e.s|rg. |c. Jces t||s .|ect t|e cppc.tur|ty ccst c| ycu.
c|c|ce`
9. |xp|.|r |c. e.c| c| t|e |c||c.|rg everts .cu|J .|ect t|e n..g|r.| ccst cu.ves c| |.ns .rJ t|us t|e
supp|y cu.ve |r . pe.|ect|y ccnpet|t|ve n..ket |r t|e s|c.t .ur.
.. Ar |rc.e.se |r ..ges
b. A t.x c| 1 pe. ur|t c| cutput |npcseJ cr t|e se||e.
c. |e |rt.cJuct|cr c| ccstcutt|rg tec|rc|cgy
J. |e |npcs|t|cr c| .r .rru.| ||cerse |ee c| 1,000
10. |r . pe.|ect|y ccnpet|t|ve n..ket, .|c bere|ts |.cn .r evert t|.t |c.e.s p.cJuct|cr ccsts |c. |.ns`
11. |.yc|e.r|rg est.b||s|nerts gere..te . ccrs|Je..b|e .ncurt c| .|. pc||ut|cr |r p.cJuc|rg c|e.r|rg
se.v|ces. Suppcse t|ese |.ns ..e .||c.eJ tc pc||ute .|t|cut .est.|ct|cr .rJ t|.t .eJuc|rg t|e|. pc||ut|cr
.cu|J .JJ s|gr||c.rt|y tc t|e|. p.cJuct|cr ccsts. \|c bere|ts |.cn t|e |.ct t|.t t|ey pc||ute t|e .|.`
|c. suppcse t|e gcve.rnert .e,u|.es t|en tc .eJuce t|e|. pc||ut|cr. \|c .||| p.y |c. t|e c|e.rup`
(Assune J.y c|e.r|rg |s . pe.|ect|y ccnpet|t|ve |rJust.y, .rJ .rs.e. t|ese ,uest|crs |.cn . |crg.ur
pe.spect|ve.)
12. |e |.te cc|unr|st \||||.n |. buck|ey, ccnnert|rg cr . st.|ke by t|e e.nste.s |r|cr .g.|rst ||S |r
199, c|e.eJ t||s b|t c| eccrcn|c .r.|ys|s tc exp|.|r |c. ||S |.J succeeJeJ |r .eJuc|rg |ts .ve..ge tct.|
ccst. ||S |.s Jcre t||s by eccrcn|es c| sc.|e. |p tc . pc|rt (.|e.e t|e n..g|r.| ccst e,u.|s t|e p.|ce c|
t|e n..g|r.| ur|t), t|e |..ge. t|e bus|ress, t|e |ess t|e pe.ur|t ccst. |se t|e ccrcept c| eccrcn|es c|
sc.|e, tcget|e. .|t| t|e |r|c.n.t|cr p.eserteJ |r t||s c|.pte., tc exp|.|r t|e e..c. |r V.. buck|eys
st.tenert.
1
13. Suppcse t|.t . pe.|ect|y ccnpet|t|ve |rJust.y |s |r |crg.ur e,u|||b.|un .rJ expe.|erces .r |rc.e.se |r
p.cJuct|cr ccst. \|c .||| be.. t|e bu.Jer c| t|e |rc.e.se` |s t||s |.|.`
14. |ccrcn|sts ..gue t|.t t|e u|t|n.te bere|c|..|es c| t|e e|c.ts c| pe.|ect|y ccnpet|t|ve |.ns ..e
ccrsune.s. |r .|.t serse |s t||s t|e c.se` |c t|e c.re.s c| pe.|ect|y ccnpet|t|ve |.ns Je.|ve .ry |crg
.ur bere|t |.cn t|e|. e|c.ts`
15. |xp|.|r c..e|u||y .|y . |xeJ ||cerse |ee Jces rct s|||t . |.ns n..g|r.| ccst cu.ve |r t|e s|c.t .ur. \|.t
.bcut t|e |crg .ur`
248 PRINCIPLS OI CONOMICS
N U M R I C A L P R O 8 L M S
1. |e g..p| be|c. p.cv|Jes .everue .rJ ccst |r|c.n.t|cr |c. . pe.|ect|y ccnpet|t|ve |.n p.cJuc|rg p.pe.
c||ps.
Output 7otaI Revenue 7otaI VariabIe Cost 7otaI Iixed Cost
1 1,000 1,500 500
2 2,000 2,000 500
3 3,000 2,600 500
4 4,000 3,900 500
5 5,000 5,000 500
.. |c. nuc| ..e tct.| |xeJ ccsts`
b. Abcut |c. nuc| ..e tct.| v..|.b|e ccsts || 5,000 p.pe. c||ps ..e p.cJuceJ`
c. \|.t |s t|e p.|ce c| . p.pe. c||p`
J. \|.t |s t|e .ve..ge .everue |.cn p.cJuc|rg p.pe. c||ps`
e. \|.t |s t|e n..g|r.| .everue c| p.cJuc|rg p.pe. c||ps`
|. Ove. .|.t cutput ..rge .||| t||s |.n e..r eccrcn|c p.c|ts`
g. Ove. .|.t cutput ..rge .||| t||s |.n |rcu. eccrcn|c |csses`
|. \|.t |s t|e s|cpe c| t|e tct.| .everue cu.ve`
|. \|.t |s t|e s|cpe c| t|e tct.| ccst cu.ve .t t|e p.c|tn.x|n|.|rg runbe. c| p.pe. c||ps pe. |cu.`
. At .bcut |c. n.ry p.pe. c||ps pe. |cu. Jc eccrcn|c p.c|ts seen tc be .t . n.x|nun`
2. Suppcse .cck|rgc|.|. n.ru|.ctu.|rg |s . pe.|ect|y ccnpet|t|ve |rJust.y |r .||c| t|e.e ..e 1,000 |Jert|c.|
|.ns. |.c| |.ns tct.| ccst |s .e|.teJ tc cutput pe. J.y .s |c||c.s.
Quantity 7otaI cost Quantity 7otaI cost
0 500 5 2,200
1 1,000 6 2,00
2 1,300 3,300
3 1,500 8 4,400
4 1,800
.. |.ep..e . t.b|e t|.t s|c.s tct.| v..|.b|e ccst, .ve..ge tct.| ccst, .rJ n..g|r.| ccst .t e.c| |eve|
c| cutput.
b. ||ct t|e .ve..ge tct.| ccst, .ve..ge v..|.b|e ccst, .rJ n..g|r.| ccst cu.ves |c. . s|rg|e |.n
(.enenbe. t|.t v.|ues |c. n..g|r.| ccst ..e p|ctteJ .t t|e n|Jpc|rt c| t|e .espect|ve |rte.v.|s).
c. \|.t |s t|e |.ns supp|y cu.ve` |c. n.ry c|.|.s .cu|J t|e |.n p.cJuce .t p.|ces c| 350, 450,
550, .rJ 650` (|r ccnput|rg ,u.rt|t|es, .ssune t|.t . |.n p.cJuces . ce.t.|r runbe. c|
ccnp|eteJ c|.|.s e.c| J.y, |t Jces rct p.cJuce |..ct|crs c| . c|.|. cr .ry cre J.y.)
J. Suppcse t|e Jen.rJ cu.ve |r t|e n..ket |c. .cck|rg c|.|.s |s g|ver by t|e |c||c.|rg t.b|e.
Price Quantity of chairs Demanded/day Price Quantity of chairs Demanded/day
650 5,000 450 ,000
550 6,000 350 8,000
||ct t|e n..ket Jen.rJ cu.ve |c. c|.|.s. cnpute .rJ p|ct t|e n..ket supp|y cu.ve, us|rg t|e
|r|c.n.t|cr ycu cbt.|reJ |c. . s|rg|e |.n |r p..t (c). \|.t |s t|e e,u|||b.|un p.|ce` |e
e,u|||b.|un ,u.rt|ty`
e. C|ver ycu. sc|ut|cr |r p..t (J), p|ct t|e tct.| .everue .rJ tct.| ccst cu.ves |c. . s|rg|e |.n. |ces
ycu. g..p| cc..espcrJ tc ycu. sc|ut|cr |r p..t (c)` |xp|.|r.
3. |e |c||c.|rg t.b|e s|c.s t|e tct.| cutput, tct.| .everue, tct.| v..|.b|e ccst, .rJ tct.| |xeJ ccst c| . |.n.
\|.t |eve| c| cutput s|cu|J t|e |.n p.cJuce` S|cu|J |t s|ut Jc.r` S|cu|J |t ex|t t|e |rJust.y` |xp|.|r.
CHAP7R 9 COMP7I7IV MARk7S IOR GOODS AND SRVICS 249
Output 7otaI revenue 7otaI variabIe cost 7otaI xed cost
1 1,000 1,500 500
2 2,000 2,000 500
3 3,000 2,600 500
4 4,000 3,900 500
5 5,000 5,000 500
4. Suppcse . .|se |r |ue| ccsts |rc.e.ses t|e ccst c| p.cJuc|rg c.ts by 0.50 pe. bus|e|. |||ust..te g..p||c.||y
|c. t||s c|.rge .||| .|ect t|e c.t n..ket .rJ . s|rg|e |.n |r t|e n..ket |r t|e s|c.t .ur .rJ |r t|e |crg
.ur.
5. Suppcse t|e Jen.rJ |c. c.. ..s|es |r c||egetc.r |.||s .s . .esu|t c| . cutb.ck |r cc||ege er.c||nert.
S|c. g..p||c.||y |c. t|e p.|ce .rJ cutput |c. t|e n..ket .rJ |c. . s|rg|e |.n .||| be .|ecteJ |r t|e
s|c.t .ur .rJ |r t|e |crg .ur. Assune t|e n..ket |s pe.|ect|y ccnpet|t|ve .rJ t|.t |t |s |r|t|.||y |r |crg.ur
e,u|||b.|un .t . p.|ce c| 12 pe. c.. ..s|. Assune .|sc t|.t |rput p.|ces Jcrt c|.rge .s t|e n..ket
.espcrJs tc t|e c|.rge |r Jen.rJ.
6. Suppcse t|.t t|e n..ket |c. J.ye..se pers |s pe.|ect|y ccnpet|t|ve .rJ t|.t t|e pers ccst 1 e.c|. |e
|rJust.y |s |r |crg.ur e,u|||b.|un. |c. suppcse t|.t .r |rc.e.se |r t|e ccst c| |rk ..|ses t|e p.cJuct|cr
ccst c| t|e pers by .25 pe. per.
.. |s|rg . g..p| t|.t s|c.s t|e n..ket .s . .|c|e .rJ . typ|c.| |.n |r t||s n..ket, |||ust..te t|e
s|c.t .ur e|ects c| t|e c|.rge.
b. |s t|e p.|ce ||ke|y tc .|se by .25` \|y c. .|y rct`
c. || |t Jcesrt, ..e |.ns ||ke|y tc ccrt|rue tc cpe..te |r t|e s|c.t .ur` \|y c. .|y rct`
J. \|.t |s ||ke|y tc |.pper |r t|e |crg .ur` |||ust..te ycu. .esu|ts .|t| . |..ge, c|e..|y |.be|eJ g..p|.
250 PRINCIPLS OI CONOMICS
1.
ENDNOTES
\||||.n |. buck|ey, ..ey cck cr C.eeJ .s ||s b.tt|e .y, |e Ooe||e, 22 August
199, |e.s (. |r|ve.s.| |.ess SyrJ|c.te cc|unr).
CHAP7R 9 COMP7I7IV MARk7S IOR GOODS AND SRVICS 251
252 PRINCIPLS OI CONOMICS
monopoIy
A |.n t|.t t|.t |s t|e cr|y
p.cJuce. c| . gccJ c. se.v|ce
|c. .||c| t|e.e ..e rc c|cse
subst|tutes .rJ |c. .||c|
ert.y by pctert|.| .|v.|s |s
p.c||b|t|ve|y J||cu|t.
| A | | | 1 0
Monopoly
S7AR7 UP: SURROUNDD 8Y MONOPOLIS
|| ycu. cc||ege c. ur|ve.s|ty |s ||ke ncst, ycu sperJ . |ct c| t|ne, .rJ ncrey, Je.||rg .|t| |.ns t|.t |.ce ve.y ||tt|e
ccnpet|t|cr. +cu. c.npus bcckstc.e |s ||ke|y tc be t|e cr|y |cc.| |.n se|||rg t|e texts t|.t p.c|essc.s .e,u|.e ycu tc
.e.J. +cu. sc|cc| n.y |.ve g..rteJ .r exc|us|ve |..rc||se tc . s|rg|e |.n |c. |ccJ se.v|ce .rJ tc .rct|e. |.n |c.
verJ|rg n.c||res. A s|rg|e |.n n.y p.cv|Je ycu. ut|||t|ese|ect.|c|ty, r.tu..| g.s, .rJ ..te..
|r||ke t|e |rJ|v|Ju.| |.ns .e |.ve p.ev|cus|y stuJ|eJ t|.t cpe..te |r . ccnpet|t|ve n..ket, t.k|rg t|e p.|ce,
.||c| |s Jete.n|reJ by Jen.rJ .rJ supp|y, .s g|ver, |r t||s c|.pte. .e |rvest|g.te t|e be|.v|c. c| |.ns t|.t |.ve
t|e|. n..kets .|| tc t|ense|ves. As t|e cr|y supp||e.s c| p..t|cu|.. gccJs c. se.v|ces, t|ey |.ce t|e Jc.r...Js|cp
|rg n..ket Jen.rJ cu.ve .|cre.
\e .||| |rJ t|.t |.ns t|.t |.ve t|e|. n..kets .|| tc t|ense|ves be|.ve |r . n.rre. t|.t |s |r n.ry .espects
,u|te J||e.ert |.cn t|e be|.v|c. c| |.ns |r pe.|ect ccnpet|t|cr. Suc| |.ns ccrt|rue tc use t|e n..g|r.| Jec|s|cr
.u|e |r n.x|n|.|rg p.c|ts, but t|e|. |.eeJcn tc se|ect |.cn t|e p.|ce .rJ ,u.rt|ty ccnb|r.t|crs g|ver by t|e n..
ket Jen.rJ cu.ve .|ects t|e ..y |r .||c| t|ey .pp|y t||s .u|e.
\e .||| s|c. t|.t . ncrcpc|y |.n |s ||ke|y tc p.cJuce |ess .rJ c|..ge nc.e |c. .|.t |t p.cJuces t|.r |.ns |r
. ccnpet|t|ve |rJust.y. As . .esu|t, . ncrcpc|y sc|ut|cr |s ||ke|y tc be |re|c|ert |.cn scc|etys pe.spect|ve. \e .|||
exp|c.e t|e pc||cy .|te.r.t|ves .v.||.b|e tc gcve.rnert .gerc|es |r Je.||rg .|t| ncrcpc|y |.ns. ||.st, t|cug|, .e
.||| |cck .t c|...cte.|st|cs c| ncrcpc|y .rJ .t ccrJ|t|crs t|.t g|ve .|se tc ncrcpc||es |r t|e |.st p|.ce.
1. THE NATURE OF MONOPOLY
L A R N I N G O 8 1 C 7 I V S
1. Dene monopoIy and the reIationship between price setting and monopoIy power.
2. List and expIain the sources of monopoIy power and how they can change over time.
3. Dene what is meant by a naturaI monopoIy.
Monopoly is at the opposite end of the spectium of maiket models fiom peifect competition. A
monopoly fim has no iivals. It is the only fim in its industiy. Theie aie no close substitutes foi the
good oi seivice a monopoly pioduces. Not only does a monopoly fim have the maiket to itself, but it
also need not woiiy about othei fims enteiing. In the case of monopoly, entiy by potential iivals is
piohibitively dimcult.
A monopoly does not take the maiket piice as given; it deteimines its own piice. It selects fiom its
demand cuive the piice that coiiesponds to the quantity the fim has chosen to pioduce in oidei to
eain the maximum pioft possible. The entiy of new fims, which eliminates pioft in the long iun in a
competitive maiket, cannot occui in the monopoly model.
price setter
A |.n t|.t sets c. p|cks p.|ce
b.seJ cr |ts cutput Jec|s|cr.
monopoIy power
|e .b|||ty tc .ct .s . p.|ce
sette..
barriers to entry
|...cte.|st|c c| . p..t|cu|..
n..ket t|.t b|cck t|e ert.y c|
re. |.ns |r . ncrcpc|y
n..ket.
naturaI monopoIy
A |.n t|.t ccr|.crts
eccrcn|es c| sc.|e cve. t|e
ert|.e ..rge c| cutputs
Jen.rJeJ |r |ts |rJust.y.
A fim that sets oi picks piice based on its output decision is called a price setter. A fim that acts
as a piice settei possesses monopoly power. We shall see in the next chaptei that monopolies aie not
the only fims that have this powei; howevei, the absence of iivals in monopoly gives it much moie
piice-setting powei.
As was the case when we discussed peifect competition in the pievious chaptei, the assumptions of
the monopoly model aie iathei stiong. In assuming theie is one fim in a maiket, we assume theie aie
no othei fims pioducing goods oi seivices that could be consideied pait of the same maiket as that of
the monopoly fim. In assuming blocked entiy, we assume, foi ieasons we will discuss below, that no
othei fim can entei that maiket. Such conditions aie iaie in the ieal woild. As always with models, we
make the assumptions that defne monopoly in oidei to simplify oui analysis, not to desciibe the ieal
woild. The iesult is a model that gives us impoitant insights into the natuie of the choices of fims and
theii impact on the economy.
1.1 Souices of Monopoly Powei
Why aie some maikets dominated by single fims: What aie the souices of monopoly powei: Econom-
ists have identifed a numbei of conditions that, individually oi in combination, can lead to domina-
tion of a maiket by a single fim and cieate baiiieis that pievent the entiy of new fims.
Barriers to entry aie chaiacteiistics of a paiticulai maiket that block new fims fiom enteiing it.
They include economies of scale, special advantages of location, high sunk costs, a dominant position
in the owneiship of some of the inputs iequiied to pioduce the good, and goveinment iestiictions.
These baiiieis may be inteiielated, making entiy that much moie foimidable. Although these baiiieis
might allow one fim to gain and hold monopoly contiol ovei a maiket, theie aie often foices at woik
that can eiode this contiol.
conomies of ScaIe
Scale economies and diseconomies defne the shape of a fim's long-iun aveiage cost (LRAC) cuive as
it incieases its output. If long-iun aveiage cost declines as the level of pioduction incieases, a fim is
said to expeiience economies of scale.
A fim that confionts economies of scale ovei the entiie iange of outputs demanded in its industiy
is a natural monopoly. Utilities that distiibute electiicity, watei, and natuial gas to some maikets aie
examples. In a natuial monopoly, the LRAC of any one fim inteisects the maiket demand cuive wheie
long-iun aveiage costs aie falling oi aie at a minimum. If this is the case, one fim in the industiy will
expand to exploit the economies of scale available to it. Because this fim will have lowei unit costs than
its iivals, it can diive them out of the maiket and gain monopoly contiol ovei the industiy.
Suppose theie aie 12 fims, each opeiating at the scale shown by ATC
1
(aveiage total cost) in Fig-
uie 10.1. A fim that expanded its scale of opeiation to achieve an aveiage total cost cuive such as ATC
2
could pioduce 240 units of output at a lowei cost than could the smallei fims pioducing 20 units each.
By cutting its piice below the minimum aveiage total cost of the smallei plants, the laigei fim could
diive the smallei ones out of business. In this situation, the industiy demand is not laige enough to
suppoit moie than one fim. If anothei fim attempted to entei the industiy, the natuial monopolist
would always be able to undeisell it.
254 PRINCIPLS OI CONOMICS
sunk cost
Ar experJ|tu.e t|.t |.s
.|.e.Jy beer n.Je .rJ t|.t
c.rrct be .eccve.eJ.
II GUR 10. 1 conomies of ScaIe Lead to NaturaI MonopoIy
A |.n .|t| |.|||rg ||^ t|.cug|cut t|e ..rge c| cutputs .e|ev.rt tc ex|st|rg Jen.rJ (|) .||| ncrcpc||.e t|e
|rJust.y. |e.e, cre |.n cpe..t|rg .|t| . |..ge p|.rt (^
2
) p.cJuces 240 ur|ts c| cutput .t . |c.e. ccst t|.r t|e
ccst pe. ur|t c| t|e 12 |.ns cpe..t|rg .t . sn.||e. sc.|e (^
1
), .rJ p.cJuc|rg 20 ur|ts c| cutput e.c|.
Location
Sometimes monopoly powei is the iesult of location. Foi example, selleis in maikets isolated by dis-
tance fiom theii neaiest iivals have a degiee of monopoly powei. The local movie theatei in a small
town has a monopoly in showing fist-iun movies. Doctois, dentists, and mechanics in isolated towns
may also be monopolists.
Sunk Costs
The gieatei the cost of establishing a new business in an industiy, the moie dimcult it is to entei that
industiy. That cost will, in tuin, be gieatei if the outlays iequiied to stait a business aie unlikely to be
iecoveied if the business should fail.
Suppose, foi example, that entiy into a paiticulai industiy iequiies extensive adveitising to make
consumeis awaie of the new biand. Should the efoit fail, theie is no way to iecovei the expendituies
foi such adveitising. An expendituie that has alieady been made and that cannot be iecoveied is called
a sunk cost.
If a substantial fiaction of a fim's initial outlays will be lost upon exit fiom the industiy, exit will
be costly. Dimculty of exit can make foi dimculty of entiy. The moie fims have to lose fiom an unsuc-
cessful efoit to penetiate a paiticulai maiket, the less likely they aie to tiy. The potential foi high sunk
costs could thus contiibute to the monopoly powei of an established fim by making entiy by othei
fims moie dimcult.
Restricted Ownership of Raw MateriaIs and Inputs
In veiy few cases the souice of monopoly powei is the owneiship of stiategic inputs. If a paiticulai fim
owns all of an input iequiied foi the pioduction of a paiticulai good oi seivice, then it could emeige as
the only pioducei of that good oi seivice.
The Aluminum Company of Ameiica (ALCOA) gained monopoly powei thiough its owneiship of
viitually all the bauxite mines in the woild (bauxite is the souice of aluminum). The Inteinational
Nickel Company of Canada at one time owned viitually all the woild's nickel. De Beeis acquiied iights
to neaily all the woild's diamond pioduction, giving it enoimous powei in the maiket foi diamonds.
With new diamond supplies in Canada, Austialia, and Russia being developed and sold independently
of DeBeeis, howevei, this powei has declined, and today DeBeeis contiols a substantially smallei pei-
centage of the woild's supply.
CHAP7R 10 MONOPOLY 255
network eects
S|tu.t|crs .|e.e p.cJucts
beccne nc.e use|u| t|e
|..ge. t|e runbe. c| use.s c|
t|e p.cJuct.
Government Restrictions
Anothei impoitant basis foi monopoly powei consists of special piivileges gianted to some business
fims by goveinment agencies. State and local goveinments have commonly assigned exclusive fian-
chises-iights to conduct business in a specifc maiket-to taxi and bus companies, to cable television
companies, and to piovideis of telephone seivices, electiicity, natuial gas, and watei, although the
tiend in iecent yeais has been to encouiage competition foi many of these seivices. Goveinments
might also iegulate entiy into an industiy oi a piofession thiough licensing and ceitifcation iequiie-
ments. Goveinments also piovide patent piotection to inventois of new pioducts oi pioduction meth-
ods in oidei to encouiage innovation; these patents may afoid theii holdeis a degiee of monopoly
powei duiing the 17-yeai life of the patent.
Patents can take on extia impoitance when netwoik efects aie piesent. Network enects aiise in
situations wheie pioducts become moie useful the laigei the numbei of useis of the pioduct. Foi ex-
ample, one advantage of using the Windows computei opeiating system is that so many othei people
use it. That has advantages in teims of shaiing fles and othei infoimation.
k Y 7 A k A W A Y S
< Ar |rJust.y .|t| . s|rg|e |.n, |r .||c| ert.y |s b|cckeJ, |s c.||eJ . ncrcpc|y.
< A |.n t|.t sets c. p|cks p.|ce JeperJ|rg cr |ts cutput Jec|s|cr |s c.||eJ . p.|ce sette.. A p.|ce sette.
pcssesses ncrcpc|y pc.e..
< |e scu.ces c| ncrcpc|y pc.e. |rc|uJe eccrcn|es c| sc.|e, |cc.t|cr.| .Jv.rt.ges, ||g| surk ccsts
.sscc|.teJ .|t| ert.y, .est.|cteJ c.re.s||p c| key |rputs, .rJ gcve.rnert .est.|ct|crs, suc| .s exc|us|ve
|..rc||ses, ||cers|rg .rJ ce.t||c.t|cr .e,u|.enerts, .rJ p.terts.
< A |.n t|.t ccr|.crts eccrcn|es c| sc.|e cve. t|e ert|.e ..rge c| cutput Jen.rJeJ |r .r |rJust.y |s .
r.tu..| ncrcpc|y.
7 R Y I 7 !
\|.t |s t|e scu.ce c| ncrcpc|y pc.e.|| .ry|r e.c| c| t|e |c||c.|rg s|tu.t|crs`
1. |e |.S. |ccJ .rJ |.ug AJn|r|st..t|cr g..rteJ bu..cug|s \e||ccne exc|us|ve .|g|ts urt|| 2005 tc
n.ru|.ctu.e .rJ J|st.|bute A, . J.ug useJ |r t|e t.e.tnert c| A||S.
2. 'c|r .rJ V..y |ce .ur t|e cr|y s|ce .ep.|. s|cp |r tc.r.
3. Ore ut|||ty ccnp.ry J|st.|butes .es|Jert|.| e|ect.|c|ty |r ycu. tc.r.
4. |e .|Jesp.e.J use c| .utcn.t|c te||e. n.c||res (AVs) |.s p.cver . bccr tc ||ebc|J, t|e p.|rc|p.|
n.ru|.ctu.e. c| t|e n.c||res.
256 PRINCIPLS OI CONOMICS
Case in Point: 7he Ambassador 8ridge Iights to Maintain Its MonopoIy
2010 jupiterimages Corporation
V.tty Vc.cur ..s ,u|et|y ercy|rg ||s ncrcpc|y pc.e.. |e |s t|e c.re. c| t|e 5ye..c|J Anb.ss.Jc.
b.|Jge, . suspers|cr b.|Jge t|.t |s t|e cr|y ccrrect|cr bet.eer |et.c|t, V|c||g.r .rJ \|rJsc., Ort..|c. |e
pu.c|.seJ t|e b.|Jge |.cn \...er bu|et |r 194 |c. 30 n||||cr. |c.bes est|n.tes t|.t |t |s .c.t| nc.e t|.r
500 n||||cr tcJ.y. V.. Vc.cur rc. cve.sees t|e ..te.y cve. .||c| 100 b||||cr c| gccJscre ,u..te. c| |.S.
t..Je .|t| .r.J. .rJ 40 c| .|| t.uck s||pnerts |.cn t|e |.S.n.ke t|e|. ..y bet.eer t|e t.c ccurt.|es.
|esp|te ccnp|.|rts c| ||g| .rJ .|s|rg tc||s|e |.s nc.e t|.r Jcub|eJ |..es |c. c..s .rJ t.|p|eJ |..es |c.
t.ucksV.. Vc.cur |.s sc |.. |e|J cr. |erret| |.v|es, . |..ye. .|c c|ter b.tt|es V.. Vc.cur |r ccu.t, |s .
g.uJg|rg .Jn|.e.. |es ve.y |rte|||gert .rJ ve.y .gg.ess|ve. ||s .v..|ce .rJ g.eeJ ..e ust Ane.|c.r c.p|t.||sn
.t .c.k, |e tc|J |c.bes.
\|.t ..e t|e scu.ces c| ||s ncrcpc|y pc.e.` \|t| t|e c|csest .|te.r.t|ve b.|Jge .c.css t|e |et.c|t ||ve. t.c
|cu.s ...y, |cc.t|cr |s . b|g p|us. |r .JJ|t|cr, t|e ccst c| c.e.t|rg . re. t..rspc.t.t|cr ||rk |s ||g|. A g.cup
t|.t |s ccrs|Je.|rg ccrve.t|rg .r c|J t..|r turre| tc t.uck use .rJ bc.|rg . re. t..|r turre| scne J|st.rce
...y |s |.c|rg . 600 n||||cr p.|ce t.g |c. t|e p.cect. |r .JJ|t|cr tc |.v|rg ert.y by pctert|.| ccnpet|tc.s
b|cckeJ , |e |.s . st.tus rct s|..eJ by ncst ct|e. ncrcpc||sts. |e V|c||g.r Sup.ene cu.t .u|eJ |r 2008
t|.t t|e c|ty c| |et.c|t c.rrct .egu|.te ||s bus|ress bec.use c| t|e b.|Jges |rte.r.t|cr.| r.tu.e. .r.J|.r
ccu.ts |.ve b...eJ .ry e|c.t by .r.J|.r .ut|c.|t|es tc .egu|.te ||n. |e .||| rct ever .||c. |rspectc.s |.cn
t|e gcve.rnert c| t|e |r|teJ St.tes tc set |cct cr ||s b.|Jge.
|rc.e.seJ secu.|ty s|rce 9/11 |.s c.useJ Je|.ys, but V.. Vc.cur |.s e.seJ t|ese by |rc.e.s|rg ||s c.r sperJ
|rg cr secu.|ty tc 50,000 . .eek .rJ by bu||J|rg .JJ|t|cr.| |rspect|cr st.t|crs .rJ g||t|rg t|en tc t|e |.S.
|rspect|cr .gercy, t|e Cere..| Se.v|ces AJn|r|st..t|cr. |ver . ncrcpc||st urJe.st.rJs t|e |npc.t.rce c|
keep|rg ||s custcne.s ccrtert!
bec.use c| t|e te..c.|st .tt.cks cr 9/11 .rJ t|e ccrce.r .bcut vu|re..b|||ty .rJ secu.|ty, c.||s tc Je.| .|t| t||s
ncrcpc|y |.ve |rc.e.seJ. Scne pecp|e ..gue t|.t t|e gcve.rnert s|cu|J buy .|.t |s t|e ncst |npc.t.rt
s|rg|e |rte.r.t|cr.| ..te.|.| |r |c.t| Ane.|c., .|||e ct|e.s |.ve c.||eJ |c. nc.e .egu|.tc.y cve.s|g|t. .r.J|.r
g.cups ..e exp|c.|rg t|e Jeve|cpnert c| .|te.r.t|ve ne.rs c| b.|rg|rg t..|c bet.eer t|e |r|teJ St.tes .rJ
.r.J.. |ne .||| te|| .|et|e. V.. Vc.cur c.r |c|J crtc .|.t |oe ..|te.s Step|.re ||tc| .rJ 'c.rr Vu||e.
JubbeJ t|e best ncrcpc|y ycu reve. |e..J c|.
.ooce .|e|oe |||c| oJ .oo /o||e. |e o|| |Je ||e ||Je.' |oe .40 (|o.ee 5 2004: 3439. .o| Oo||o|e. ||o |e
|o|.o, |o |oe ^ooJo ||Je o|c.' |e|o|| |ee |e. /o, . 200S. oJ .|o|e .oee oo| .|Je .||| ^ooJo ||Je | ||o|e.'
|e|o|| |e.. /o, .. 200S
CHAP7R 10 MONOPOLY 257
A N S W R S 7 O 7 R Y I 7 ! P R O 8 L M S
1. |e gcve.rnerts g..rt c| .r exc|us|ve |..rc||se tc t|e J.ug g.ve t|e |.n ncrcpc|y pc.e..
2. \|||e 'c|r .rJ V..y |.ve t|e cr|y s|cp |r tc.r, t||s |s .r e.sy ert.y bus|ress. |u.t|e., t|e.e n.y be
ccnpet|tc.s |r t|e re..by tc.r. 'c|r .rJ V..y p.cb.b|y |.ve ncrcpc|y pc.e., but t|ey Jc rct |.ve .
ncrcpc|y.
3. |.tu..| ncrcpc|y
4. |.tert .|t| st.crg ret.c.k e|ects
2. THE MONOPOLY MODEL
L A R N I N G O 8 1 C 7 I V S
1. xpIain the reIationship between price and marginaI revenue when a rm faces a downward-
sIoping demand curve.
2. xpIain the reIationship between marginaI revenue and eIasticity aIong a Iinear demand curve.
3. AppIy the marginaI decision ruIe to expIain how a monopoIy maximizes prot.
Analyzing choices is a moie complex challenge foi a monopoly fim than foi a peifectly competitive
fim. Aftei all, a competitive fim takes the maiket piice as given and deteimines its pioft-maximizing
output. Because a monopoly has its maiket all to itself, it can deteimine not only its output but its piice
as well. What kinds of piice and output choices will such a fim make:
We will answei that question in the context of the maiginal decision iule: a fim will pioduce addi-
tional units of a good until maiginal ievenue equals maiginal cost. To apply that iule to a monopoly
fim, we must fist investigate the special ielationship between demand and maiginal ievenue foi a
monopoly.
2.1 Monopoly and Maiket Demand
Because a monopoly fim has its maiket all to itself, it faces the maiket demand cuive. Figuie 10.3 com-
paies the demand situations faced by a monopoly and a peifectly competitive fim. In Panel (a), the
equilibiium piice foi a peifectly competitive fim is deteimined by the inteisection of the demand and
supply cuives. The maiket supply cuive is found simply by summing the supply cuives of individual
fims. Those, in tuin, consist of the poitions of maiginal cost cuives that lie above the aveiage vaiiable
cost cuives. The maiginal cost cuive, MC, foi a single fim is illustiated. Notice the bieak in the hoii-
zontal axis indicating that the quantity pioduced by a single fim is a tiivially small fiaction of the
whole. In the peifectly competitive model, one fim has nothing to do with the deteimination of the
maiket piice. Each fim in a peifectly competitive industiy faces a hoiizontal demand cuive defned by
the maiket piice.
258 PRINCIPLS OI CONOMICS
II GUR 10. 3 Perfect Competition Versus MonopoIy
|.re| (.) s|c.s t|e Jete.n|r.t|cr c| e,u|||b.|un p.|ce .rJ cutput |r . pe.|ect|y ccnpet|t|ve n..ket. A typ|c.| |.n
.|t| n..g|r.| ccst cu.ve / |s . p.|ce t.ke., c|ccs|rg tc p.cJuce ,u.rt|ty .t t|e e,u|||b.|un p.|ce |. |r |.re| (b)
. ncrcpc|y |.ces . Jc.r...Js|cp|rg n..ket Jen.rJ cu.ve. As . p.c|t n.x|n|.e., |t Jete.n|res |ts p.c|t
n.x|n|.|rg cutput. Orce |t Jete.n|res t|.t ,u.rt|ty, |c.eve., t|e p.|ce .t .||c| |t c.r se|| t|.t cutput |s |curJ
|.cn t|e Jen.rJ cu.ve. |e ncrcpc|y |.n c.r se|| .JJ|t|cr.| ur|ts cr|y by |c.e.|rg p.|ce. |e pe.|ect|y
ccnpet|t|ve |.n, by ccrt..st, c.r se|| .ry ,u.rt|ty |t ..rts .t t|e n..ket p.|ce.
Contiast the situation shown in Panel (a) with the one faced by the monopoly fim in Panel (b). Be-
cause it is the only suppliei in the industiy, the monopolist faces the downwaid-sloping maiket de-
mand cuive alone. It may choose to pioduce any quantity. But, unlike the peifectly competitive fim,
which can sell all it wants at the going maiket piice, a monopolist can sell a gieatei quantity only by
cutting its piice.
Suppose, foi example, that a monopoly fim can sell quantity Q
1
units at a piice P
1
in Panel (b). If
it wants to inciease its output to Q
2
units-and sell that quantity-it must ieduce its piice to P
2
. To sell
quantity Q
3
it would have to ieduce the piice to P
3
. The monopoly fim may choose its piice and out-
put, but it is iestiicted to a combination of piice and output that lies on the demand cuive. It could not,
foi example, chaige piice P
1
and sell quantity Q
3
. To be a piice settei, a fim must face a downwaid-
sloping demand cuive.
2.2 Total Revenue and Piice Elasticity
A fim's elasticity of demand with iespect to piice has impoitant implications foi assessing the impact
of a piice change on total ievenue. Also, the piice elasticity of demand can be difeient at difeient
points on a fim's demand cuive. In this section, we shall see why a monopoly fim will always select a
piice in the elastic iegion of its demand cuive.
Suppose the demand cuive facing a monopoly fim is given by Equation 10.1, wheie Q is the
quantity demanded pei unit of time and P is the piice pei unit:
QUA7I ON 10. 1
Q = 10 P
This demand equation implies the demand schedule shown in Figuie 10.4. Total ievenue foi each
quantity equals the quantity times the piice at which that quantity is demanded. The monopoly fim's
total ievenue cuive is given in Panel (b). Because a monopolist must cut the piice of eveiy unit in oidei
to inciease sales, total ievenue does not always inciease as output iises. In this case, total ievenue
ieaches a maximum of $23 when 3 units aie sold. Beyond 3 units, total ievenue begins to decline.
CHAP7R 10 MONOPOLY 259
II GUR 10. 4 Demand, Iasticity, and 7otaI Revenue
Suppcse . ncrcpc||st |.ces t|e Jc.r...Js|cp|rg Jen.rJ cu.ve s|c.r |r |.re| (.). |r c.Je. tc |rc.e.se t|e
,u.rt|ty sc|J, |t nust cut t|e p.|ce. ct.| .everue |s |curJ by nu|t|p|y|rg t|e p.|ce .rJ ,u.rt|ty sc|J .t e.c| p.|ce.
ct.| .everue, p|ctteJ |r |.re| (b), |s n.x|n|.eJ .t 25, .|er t|e ,u.rt|ty sc|J |s 5 ur|ts .rJ t|e p.|ce |s 5. At t|.t
pc|rt cr t|e Jen.rJ cu.ve, t|e p.|ce e|.st|c|ty c| Jen.rJ e,u.|s 1.
The demand cuive in Panel (a) of Figuie 10.4 shows ianges of values of the piice elasticity of demand.
We have leained that piice elasticity vaiies along a lineai demand cuive in a special way: Demand is
piice elastic at points in the uppei half of the demand cuive and piice inelastic in the lowei half of the
demand cuive. If demand is piice elastic, a piice ieduction incieases total ievenue. To sell an additional
unit, a monopoly fim must lowei its piice. The sale of one moie unit will inciease ievenue because the
peicentage inciease in the quantity demanded exceeds the peicentage deciease in the piice. The elastic
iange of the demand cuive coiiesponds to the iange ovei which the total ievenue cuive is iising in
Panel (b) of Figuie 10.4.
If demand is piice inelastic, a piice ieduction ieduces total ievenue because the peicentage in-
ciease in the quantity demanded is less than the peicentage deciease in the piice. Total ievenue falls as
the fim sells additional units ovei the inelastic iange of the demand cuive. The downwaid-sloping
poition of the total ievenue cuive in Panel (b) coiiesponds to the inelastic iange of the demand cuive.
Finally, iecall that the midpoint of a lineai demand cuive is the point at which demand becomes
unit piice elastic. That point on the total ievenue cuive in Panel (b) coiiesponds to the point at which
total ievenue ieaches a maximum.
The ielationship among piice elasticity, demand, and total ievenue has an impoitant implication
foi the selection of the pioft-maximizing piice and output: A monopoly fim will nevei choose a piice
and output in the inelastic iange of the demand cuive. Suppose, foi example, that the monopoly fim
iepiesented in Figuie 10.4 is chaiging $3 and selling 7 units. Its total ievenue is thus $21. Because this
combination is in the inelastic poition of the demand cuive, the fim could inciease its total ievenue by
iaising its piice. It could, at the same time, ieduce its total cost. Raising piice means ieducing output; a
ieduction in output would ieduce total cost. If the fim is opeiating in the inelastic iange of its demand
cuive, then it is not maximizing piofts. The fim could eain a highei pioft by iaising piice and iedu-
cing output. It will continue to iaise its piice until it is in the elastic poition of its demand cuive. A
pioft-maximizing monopoly fim will theiefoie select a piice and output combination in the elastic
iange of its demand cuive.
Of couise, the fim could choose a point at which demand is unit piice elastic. At that point, total
ievenue is maximized. But the fim seeks to maximize pioft, not total ievenue. A solution that maxim-
izes total ievenue will not maximize pioft unless maiginal cost is zeio.
260 PRINCIPLS OI CONOMICS
2.3 Demand and Maiginal Revenue
In the peifectly competitive case, the additional ievenue a fim gains fiom selling an additional
unit-its maiginal ievenue-is equal to the maiket piice. The fim's demand cuive, which is a hoii-
zontal line at the maiket piice, is also its maiginal ievenue cuive. But a monopoly fim can sell an addi-
tional unit only by loweiing the piice. That fact complicates the ielationship between the monopoly's
demand cuive and its maiginal ievenue.
Suppose the fim in Figuie 10.4 sells 2 units at a piice of $8 pei unit. Its total ievenue is $16. Now it
wants to sell a thiid unit and wants to know the maiginal ievenue of that unit. To sell 3 units iathei
than 2, the fim must lowei its piice to $7 pei unit. Total ievenue iises to $21. The maiginal ievenue of
the thiid unit is thus $3. But the price at which the fim sells 3 units is $7. Maiginal ievenue is less than
piice.
To see why the maiginal ievenue of the thiid unit is less than its piice, we need to examine moie
caiefully how the sale of that unit afects the fim's ievenues. The fim biings in $7 fiom the sale of the
thiid unit. But selling the thiid unit iequiied the fim to chaige a piice of $7 instead of the $8 the fim
was chaiging foi 2 units. Now the fim ieceives less foi the fist 2 units. The maiginal ievenue of the
thiid unit is the $7 the fim ieceives foi that unit minus the $1 ieduction in ievenue foi each of the fist
two units. The maiginal ievenue of the thiid unit is thus $3. (In this chaptei we assume that the mono-
poly fim sells all units of output at the same piice. In the next chaptei, we will look at cases in which
fims chaige difeient piices to difeient customeis.)
Maiginal ievenue is less than piice foi the monopoly fim. Figuie 10.3 shows the ielationship
between demand and maiginal ievenue, based on the demand cuive intioduced in Figuie 10.4. As al-
ways, we follow the convention of plotting maiginal values at the midpoints of the inteivals.
II GUR 10. 5 Demand and MarginaI Revenue
|e n..g|r.| .everue cu.ve |c. t|e ncrcpc|y |.n ||es be|c. |ts Jen.rJ cu.ve. |t s|c.s t|e .JJ|t|cr.| .everue
g.|reJ |.cn se|||rg .r .JJ|t|cr.| ur|t. |ct|ce t|.t, .s .|..ys, n..g|r.| v.|ues ..e p|ctteJ .t t|e n|Jpc|rts c| t|e
.espect|ve |rte.v.|s.
When the demand cuive is lineai, as in Figuie 10.3, the maiginal ievenue cuive can be placed accoid-
ing to the following iules: the maiginal ievenue cuive is always below the demand cuive and the mai-
ginal ievenue cuive will bisect any hoiizontal line diawn between the veitical axis and the demand
cuive. To put it anothei way, the maiginal ievenue cuive will be twice as steep as the demand cuive.
The demand cuive in Figuie 10.3 is given by the equation Q = 10 P , which can be wiitten
P = 10 Q . The maiginal ievenue cuive is given by P = 10 2Q , which is twice as steep as the de-
mand cuive.
The maiginal ievenue and demand cuives in Figuie 10.3 follow these iules. The maiginal ievenue
cuive lies below the demand cuive, and it bisects any hoiizontal line diawn fiom the veitical axis to the
demand cuive. At a piice of $6, foi example, the quantity demanded is 4. The maiginal ievenue cuive
passes thiough 2 units at this piice. At a piice of 0, the quantity demanded is 10; the maiginal ievenue
cuive passes thiough 3 units at this point.
CHAP7R 10 MONOPOLY 261
Just as theie is a ielationship between the fim's demand cuive and the piice elasticity of demand,
theie is a ielationship between its maiginal ievenue cuive and elasticity. Wheie maiginal ievenue is
positive, demand is piice elastic. Wheie maiginal ievenue is negative, demand is piice inelastic. Wheie
maiginal ievenue is zeio, demand is unit piice elastic.
When marginaI revenue is . then demand is .
pcs|t|ve, p.|ce e|.st|c.
reg.t|ve, p.|ce |re|.st|c.
.e.c, ur|t p.|ce e|.st|c.
A fim would not pioduce an additional unit of output with negative maiginal ievenue. And, assuming
that the pioduction of an additional unit has some cost, a fim would not pioduce the extia unit if it
has zeio maiginal ievenue. Because a monopoly fim will geneially opeiate wheie maiginal ievenue is
positive, we see once again that it will opeiate in the elastic iange of its demand cuive.
2.4 Monopoly Equilibiium: Applying the Maiginal Decision Rule
Pioft-maximizing behavioi is always based on the maiginal decision iule: Additional units of a good
should be pioduced as long as the maiginal ievenue of an additional unit exceeds the maiginal cost.
The maximizing solution occuis wheie maiginal ievenue equals maiginal cost. As always, fims seek to
maximize economic pioft, and costs aie measuied in the economic sense of oppoitunity cost.
Figuie 10.6 shows a demand cuive and an associated maiginal ievenue cuive facing a monopoly
fim. The maiginal cost cuive is like those we deiived eailiei; it falls ovei the iange of output in which
the fim expeiiences incieasing maiginal ietuins, then iises as the fim expeiiences diminishing mai-
ginal ietuins.
II GUR 10. 6 7he MonopoIy SoIution
|e ncrcpc|y |.n n.x|n|.es p.c|t by p.cJuc|rg .r cutput (
n
.t pc|rt C, .|e.e t|e n..g|r.| .everue .rJ
n..g|r.| ccst cu.ves |rte.sect. |t se||s t||s cutput .t p.|ce |
n
.
To deteimine the pioft-maximizing output, we note the quantity at which the fim's maiginal ievenue
and maiginal cost cuives inteisect (Q
m
in Figuie 10.6). We iead up fiom Q
m
to the demand cuive to
fnd the piice P
m
at which the fim can sell Q
m
units pei peiiod. The pioft-maximizing piice and out-
put aie given by point E on the demand cuive.
Thus we can deteimine a monopoly fim's pioft-maximizing piice and output by following thiee
steps:
1. Deteimine the demand, maiginal ievenue, and maiginal cost cuives.
2. Select the output level at which the maiginal ievenue and maiginal cost cuives inteisect.
3. Deteimine fiom the demand cuive the piice at which that output can be sold.
262 PRINCIPLS OI CONOMICS
II GUR 10. 7 Computing MonopoIy
Profit
A ncrcpc|y |.ns p.c|t pe. ur|t |s t|e
J||e.erce bet.eer p.|ce .rJ .ve..ge tct.|
ccst. ct.| p.c|t e,u.|s p.c|t pe. ur|t t|nes
t|e ,u.rt|ty p.cJuceJ. ct.| p.c|t |s g|ver by
t|e ..e. c| t|e s|.JeJ .ect.rg|e ^
n
|
n
||.
Once we have deteimined the monopoly fim's piice and output, we can deteimine its
economic pioft by adding the fim's aveiage total cost cuive to the giaph showing de-
mand, maiginal ievenue, and maiginal cost, as shown in Figuie 10.7. The aveiage total
cost (ATC) at an output of Q
m
units is ATC
m
. The fim's pioft pei unit is thus P
m
-
ATC
m
. Total pioft is found by multiplying the fim's output, Q
m
, by pioft pei unit, so
total pioft equals Q
m
(P
m
- ATC
m
)-the aiea of the shaded iectangle in Figuie 10.7.
Heads Up!
DispeIIing Myths About MonopoIy
|.ee ccnncr n|sccrcept|crs .bcut ncrcpc|y ..e.
1. bec.use t|e.e ..e rc .|v.|s se|||rg t|e p.cJucts c| ncrcpc|y |.ns, t|ey c.r c|..ge
.|.teve. t|ey ..rt.
2. Vcrcpc||sts .||| c|..ge .|.teve. t|e n..ket .||| be...
3. bec.use ncrcpc|y |.ns |.ve t|e n..ket tc t|ense|ves, t|ey ..e gu...rteeJ |uge
p.c|ts.
As ||gu.e 10.6 s|c.s, crce t|e ncrcpc|y |.n Jec|Jes cr t|e runbe. c| ur|ts c| cutput t|.t
.||| n.x|n|.e p.c|t, t|e p.|ce .t .||c| |t c.r se|| t|.t n.ry ur|ts |s |curJ by .e.J|rg c| t|e
Jen.rJ cu.ve t|e p.|ce .sscc|.teJ .|t| t|.t n.ry ur|ts. || |t t.|es tc se|| (
n
ur|ts c| cutput |c.
nc.e t|.r |
n
, scne c| |ts cutput .||| gc ursc|J. |e ncrcpc|y |.n c.r set |ts p.|ce, but |s .e
st.|cteJ tc p.|ce .rJ cutput ccnb|r.t|crs t|.t ||e cr |ts Jen.rJ cu.ve. |t c.rrct ust c|..ge .|.teve. |t
..rts. ArJ || |t c|..ges .|| t|e n..ket .||| be.., |t .||| se|| e|t|e. 0 c., .t ncst, 1 ur|t c| cutput.
|e|t|e. |s t|e ncrcpc|y |.n gu...rteeJ . p.c|t. crs|Je. ||gu.e 10.. Suppcse t|e .ve..ge tct.| ccst cu.ve,
|rste.J c| |y|rg be|c. t|e Jen.rJ cu.ve |c. scne cutput |eve|s .s s|c.r, .e.e |rste.J eve.y.|e.e .bcve
t|e Jen.rJ cu.ve. |r t|.t c.se, t|e ncrcpc|y .||| |rcu. |csses rc n.tte. .|.t p.|ce |t c|ccses, s|rce .ve..ge
tct.| ccst .||| .|..ys be g.e.te. t|.r .ry p.|ce |t n|g|t c|..ge. As |s t|e c.se |c. pe.|ect ccnpet|t|cr, t|e
ncrcpc|y |.n c.r keep p.cJuc|rg |r t|e s|c.t .ur sc |crg .s p.|ce exceeJs .ve..ge v..|.b|e ccst. |r t|e |crg
.ur, |t .||| st.y |r bus|ress cr|y || |t c.r ccve. .|| c| |ts ccsts.
k Y 7 A k A W A Y S
< || . |.n |.ces . Jc.r...Js|cp|rg Jen.rJ cu.ve, n..g|r.| .everue |s |ess t|.r p.|ce.
< V..g|r.| .everue |s pcs|t|ve |r t|e e|.st|c ..rge c| . Jen.rJ cu.ve, reg.t|ve |r t|e |re|.st|c ..rge, .rJ
.e.c .|e.e Jen.rJ |s ur|t p.|ce e|.st|c.
< || . ncrcpc|y |.n |.ces . ||re.. Jen.rJ cu.ve, |ts n..g|r.| .everue cu.ve |s .|sc ||re.., ||es be|c. t|e
Jen.rJ cu.ve, .rJ b|sects .ry |c.|.crt.| ||re J...r |.cn t|e ve.t|c.| .x|s tc t|e Jen.rJ cu.ve.
< c n.x|n|.e p.c|t c. n|r|n|.e |csses, . ncrcpc|y |.n p.cJuces t|e ,u.rt|ty .t .||c| n..g|r.| ccst
e,u.|s n..g|r.| .everue. |ts p.|ce |s g|ver by t|e pc|rt cr t|e Jen.rJ cu.ve t|.t cc..espcrJs tc t||s
,u.rt|ty.
7 R Y I 7 !
|e .c|| |c.J cnp.ry |s ccrs|Je.|rg bu||J|rg . tc|| .c.J. |t est|n.tes t|.t |ts ||re.. Jen.rJ cu.ve |s .s
s|c.r be|c.. Assune t|.t t|e |xeJ ccst c| t|e .c.J |s 0.5 n||||cr pe. ye... V.|rter.rce ccsts, .||c| ..e t|e
cr|y ct|e. ccsts c| t|e .c.J, ..e .|sc g|ver |r t|e t.b|e.
c||s pe. t.|p 1.00 0.90 0.80 0.0 0.60 0.50
|unbe. c| t.|ps pe. ye.. (|r n||||crs) 1 2 3 4 5 6
V.|rter.rce ccst pe. ye.. (|r n||||crs) 0. 1.2 1.8 2.9 4.2 6.0
1. |s|rg t|e n|Jpc|rt ccrvert|cr, ccnpute t|e p.c|tn.x|n|.|rg |eve| c| cutput.
2. |s|rg t|e n|Jpc|rt ccrvert|cr, .|.t p.|ce .||| t|e ccnp.ry c|..ge`
3. \|.t |s n..g|r.| .everue .t t|e p.c|tn.x|n|.|rg cutput |eve|` |c. Jces n..g|r.| .everue ccnp..e tc
p.|ce`
CHAP7R 10 MONOPOLY 263
Case in Point: Profit-Maximizing Hockey 7eams
2010 jupiterimages Corporation
|cve c| t|e g.ne` |cve c| t|e c|ty` A.e t|cse t|e |.ctc.s t|.t |r|uerce c.re.s c| p.c|ess|cr.| spc.ts te.ns |r
sett|rg .Jn|ss|crs p.|ces` |cu. eccrcn|sts .t t|e |r|ve.s|ty c| \.rccuve. |.ve .|.t t|ey t||rk |s t|e .rs.e.
|c. cre g.cup c| te.ns. p.c|ess|cr.| |cckey te.ns set .Jn|ss|cr p.|ces .t |eve|s t|.t n.x|n|.e t|e|. p.c|ts.
|ey .eg..J |cckey te.ns .s ncrcpc|y |.ns .rJ use t|e ncrcpc|y ncJe| tc ex.n|re t|e te.ns be|.v|c..
|e eccrcn|sts, |cr.|J C. |e.guscr, |erret| C. Ste...t, 'c|r c||r |. 'cres, .rJ ArJ.e |e |.ess.y, useJ
J.t. |.cn t|.ee se.scrs tc est|n.te Jen.rJ .rJ n..g|r.| .everue cu.ves |.c|rg e.c| te.n |r t|e |.t|cr.|
|cckey |e.gue. |ey |curJ t|.t Jen.rJ |c. . te.ns t|ckets |s .|ecteJ by pcpu|.t|cr .rJ |rccne |r t|e
te.ns |cne c|ty, t|e te.ns st.rJ|rg |r t|e |.t|cr.| |cckey |e.gue, .rJ t|e runbe. c| supe.st..s cr t|e
te.n.
bec.use . spc.ts te.ns ccsts Jc rct v..y s|gr||c.rt|y .|t| t|e runbe. c| |.rs .|c .tterJ . g|ver g.ne, t|e
eccrcn|sts .ssuneJ t|.t n..g|r.| ccst |s .e.c. |e p.c|tn.x|n|.|rg runbe. c| se.ts sc|J pe. g.ne |s t|us
t|e ,u.rt|ty .t .||c| n..g|r.| .everue |s .e.c, p.cv|JeJ . te.ns st.J|un |s |..ge ercug| tc |c|J t|.t ,u.rt
|ty c| |.rs. ||s urccrst..|reJ ,u.rt|ty |s |.be|eJ (
u
, .|t| . cc..espcrJ|rg p.|ce |
u
|r t|e g..p|.
St.J|un s|.e .rJ t|e Jen.rJ cu.ve |.c|rg . te.n n|g|t p.evert t|e te.n |.cn se|||rg t|e p.c|tn.x|n|.|rg
,u.rt|ty c| t|ckets. || |ts st.J|un |c|Js cr|y (
c
|.rs, |c. ex.np|e, t|e te.n .||| se|| t|.t n.ry t|ckets .t p.|ce |
c
,
|ts n..g|r.| .everue |s pcs|t|ve .t t|.t ,u.rt|ty. |ccrcn|c t|ec.y t|us p.eJ|cts t|.t t|e n..g|r.| .everue |c.
te.ns t|.t ccrs|stert|y se|| cut t|e|. g.nes .||| be pcs|t|ve, .rJ t|e n..g|r.| .everue |c. ct|e. te.ns .||| be
.e.c.
|e eccrcn|sts st.t|st|c.| .esu|ts .e.e ccrs|stert .|t| t|e t|ec.y. |ey |curJ t|.t te.ns t|.t Jcrt typ|c.||y
se|| cut t|e|. g.nes cpe..te .t . ,u.rt|ty .t .||c| n..g|r.| .everue |s .bcut .e.c, .rJ t|.t te.ns .|t| se|
|cuts |.ve pcs|t|ve n..g|r.| .everue. |ts c|e.. t|.t t|ese te.ns ..e ve.y scp||st|c.teJ |r t|e|. use c| p.|c|rg
tc n.x|n|.e p.c|ts, V.. |e.guscr s.|J.
264 PRINCIPLS OI CONOMICS
.ooce |oo|J O |eoo e| o|. |e ||c| o| .o| |.e| |o eo /o||e |o||.' .ooo| o| |Jo||o| |coo|c 39(3: (/oc| 99:
29.30 oJ eoo| ||e.|e.
A N S W R 7 O 7 R Y I 7 ! P R O 8 L M
V.|rter.rce ccsts ccrst|tute t|e v..|.b|e ccsts .sscc|.teJ .|t| bu||J|rg t|e .c.J. |r c.Je. tc .rs.e. t|e |.st
|cu. p..ts c| t|e ,uest|cr, ycu .||| reeJ tc ccnpute tct.| .everue, n..g|r.| .everue, .rJ n..g|r.| ccst, .s
s|c.r .t .|g|t.
1. |s|rg t|e n|Jpc|rt ccrvert|cr, t|e p.c|tn.x|n|.|rg |eve| c| cutput |s 2.5 n||||cr t.|ps pe. ye... \|t|
t|.t runbe. c| t.|ps, n..g|r.| .everue (0.60) e,u.|s n..g|r.| ccst (0.60).
2. Ag.|r, .e use t|e n|Jpc|rt ccrvert|cr. |e ccnp.ry .||| c|..ge . tc|| c| 0.85.
3. |e n..g|r.| .everue |s 0.60, .||c| |s |ess t|.r t|e 0.85 tc|| (p.|ce).
CHAP7R 10 MONOPOLY 265
3. ASSESSING MONOPOLY
L A R N I N G O 8 1 C 7 I V S
1. xpIain and iIIustrate that a monopoIy rm produces an output that is Iess than the emcient
IeveI and why this resuIts in a deadweight Ioss to society.
2. xpIain and iIIustrate how the higher price that a monopoIy charges, compared to an otherwise
identicaI perfectIy competitive rm, transfers part of consumer surpIus to the monopoIist and
raises questions of equity.
3. Considering both advantages and disadvantages, discuss the potentiaI eects that a monopoIy
may have on consumer choices, price, quaIity of products, and technoIogicaI innovations.
4. Discuss the pubIic poIicy responses to monopoIy.
We have seen that foi monopolies puisuing pioft maximization, the outcome difeis fiom the case of
peifect competition. Does this mattei to society: In this section, we will focus on the difeiences that
stem fiom maiket stiuctuie and assess theii implications.
3.1 Efficiency, Equity, and Concentiation of Powei
A monopoly fim deteimines its output by setting maiginal cost equal to maiginal ievenue. It then
chaiges the piice at which it can sell that output, a piice deteimined by the demand cuive. That piice
exceeds maiginal ievenue; it theiefoie exceeds maiginal cost as well. That contiasts with the case in
peifect competition, in which piice and maiginal cost aie equal. The highei piice chaiged by a mono-
poly fim may allow it a pioft-in laige pait at the expense of consumeis, whose ieduced options may
give them little say in the mattei. The monopoly solution thus iaises pioblems of emciency, equity, and
the concentiation of powei.
MonopoIy and fficiency
The fact that piice in monopoly exceeds maiginal cost suggests that the monopoly solution violates the
basic condition foi economic emciency, that the piice system must confiont decision makeis with all of
the costs and all of the benefts of theii choices. Emciency iequiies that consumeis confiont piices that
equal maiginal costs. Because a monopoly fim chaiges a piice gieatei than maiginal cost, consumeis
will consume less of the monopoly's good oi seivice than is economically emcient.
To contiast the emciency of the peifectly competitive outcome with the inemciency of the mono-
poly outcome, imagine a peifectly competitive industiy whose solution is depicted in Figuie 10.11. The
shoit-iun industiy supply cuive is the summation of individual maiginal cost cuives; it may be ie-
gaided as the maiginal cost cuive foi the industiy. A peifectly competitive industiy achieves equilibii-
um at point C, at piice P
c
and quantity Q
c
.
266 PRINCIPLS OI CONOMICS
II GUR 10. 11 Perfect Competition, MonopoIy, and fficiency
C|ver n..ket Jen.rJ .rJ n..g|r.| .everue, .e c.r ccnp..e t|e be|.v|c. c| . ncrcpc|y tc t|.t c| . pe.|ect|y
ccnpet|t|ve |rJust.y. |e n..g|r.| ccst cu.ve n.y be t|cug|t c| .s t|e supp|y cu.ve c| . pe.|ect|y ccnpet|t|ve
|rJust.y. |e pe.|ect|y ccnpet|t|ve |rJust.y p.cJuces ,u.rt|ty (
c
.rJ se||s t|e cutput .t p.|ce |
c
. |e ncrcpc||st
.est.|cts cutput tc (
n
.rJ ..|ses t|e p.|ce tc |
n
. |ec.g.r|.|rg . pe.|ect|y ccnpet|t|ve |rJust.y .s . ncrcpc|y
.esu|ts |r . Je.J.e|g|t |css tc scc|ety g|ver by t|e s|.JeJ ..e. C|. |t .|sc t..rs|e.s . pc.t|cr c| t|e ccrsune.
su.p|us e..reJ |r t|e ccnpet|t|ve c.se tc t|e ncrcpc|y |.n.
Now, suppose that all the fims in the industiy meige and a goveinment iestiiction piohibits entiy by
any new fims. Oui peifectly competitive industiy is now a monopoly. Assume the monopoly contin-
ues to have the same maiginal cost and demand cuives that the competitive industiy did. The mono-
poly fim faces the same maiket demand cuive, fiom which it deiives its maiginal ievenue cuive. It
maximizes pioft at output Q
m
and chaiges piice P
m
. Output is lowei and piice highei than in the
competitive solution.
Society would gain by moving fiom the monopoly solution at Q
m
to the competitive solution at
Q
c
. The beneft to consumeis would be given by the aiea undei the demand cuive between Q
m
and Q
c
;
it is the aiea Q
m
RCQ
c
. An inciease in output, of couise, has a cost. Because the maiginal cost cuive
measuies the cost of each additional unit, we can think of the aiea undei the maiginal cost cuive ovei
some iange of output as measuiing the total cost of that output. Thus, the total cost of incieasing out-
put fiom Q
m
to Q
c
is the aiea undei the maiginal cost cuive ovei that iange-the aiea Q
m
GCQ
c
. Sub-
tiacting this cost fiom the beneft gives us the net gain of moving fiom the monopoly to the competit-
ive solution; it is the shaded aiea GRC. That is the potential gain fiom moving to the emcient solution.
The aiea GRC is a deadweight loss.
MonopoIy and quity
The monopoly solution iaises issues not just of emciency but also of equity. Figuie 10.11 shows that the
monopolist chaiges piice P
m
iathei than the competitive piice P
c
; the highei piice chaiged by the
monopoly fim ieduces consumei suiplus. Consumei suiplus is the difeience between what con-
sumeis aie willing to pay foi a good and what they actually pay. It is measuied by the aiea undei the
demand cuive and above the piice of the good ovei the iange of output pioduced.
If the industiy weie competitive, consumei suiplus would be the aiea below the demand cuive and
above P
c
C. With monopoly, consumei suiplus would be the aiea below the demand cuive and above
P
m
R. Pait of the ieduction in consumei suiplus is the aiea undei the demand cuive between Q
c
and
CHAP7R 10 MONOPOLY 267
Q
m
; it is contained in the deadweight loss aiea GRC. But consumeis also lose the aiea of the iectangle
bounded by the competitive and monopoly piices and by the monopoly output; this lost consumei sui-
plus is tiansfeiied to the monopolist.
The fact that society sufeis a deadweight loss due to monopoly is an emciency pioblem. But the
tiansfei of a poition of consumei suiplus to the monopolist is an equity issue. Is such a tiansfei legit-
imate: Aftei all, the monopoly fim enjoys a piivileged position, piotected by baiiieis to entiy fiom
competition. Should it be allowed to extiact these gains fiom consumeis: We will see that public policy
suggests that the answei is no. Regulatoiy efoits imposed in monopoly cases often seek to ieduce the
degiee to which monopoly fims extiact consumei suiplus fiom consumeis by ieducing the piices
these fims chaige.
MonopoIy and the Concentration of Power
The objections to monopoly iun much deepei than woiiies ovei economic emciency and high piices.
Because it enjoys baiiieis that block potential iivals, a monopoly fim wields consideiable maiket
powei. Foi many people, that concentiation of powei is objectionable. A decentialized, competitive
maiket constantly tests the ability of fims to satisfy consumeis, pushes them to fnd new pioducts and
new and bettei pioduction methods, and whittles away economic piofts. Fiims that opeiate in the
sheltei of monopoly may be laigely immune to such piessuies. Consumeis aie likely to be left with
fewei choices, highei costs, and lowei quality.
Peihaps moie impoitant in the view of many economists is the fact that the existence of economic
piofts piovides both an incentive and the means foi monopolists to aggiessively piotect theii position
and extend it if possible. These economists point out that monopolists may be willing to spend theii
economic piofts in attempts to infuence political leadeis and public authoiities (including iegulatoiy
authoiities) who can help them maintain oi enhance theii monopoly position. Giaft and coiiuption
may be the iesult, claim these ciitics. Indeed, Miciosoft has been accused by its iivals of bullying com-
putei manufactuieis into installing its web biowsei, Inteinet Exploiei, exclusively on theii computeis.
Attitudes about Miciosoft iefect these conceins. Even among people who feel that its pioducts aie
good and faiily piiced, theie is uneasiness about oui seeming dependence on them. And once it has se-
cuied its dominant position, will it chaige moie foi its pioducts: Will it continue to innovate:
PubIic PoIicy 7oward MonopoIy
Pulling togethei what we have leained in this chaptei on monopoly and pieviously on peifect competi-
tion, Table 10.1 summaiizes the difeiences between the models of peifect competition and monopoly.
Most impoitantly we note that wheieas the peifectly competitive fim is a piice takei, the monopoly
fim is a piice settei. Because of this difeience, we can object to monopoly on giounds of economic
emciency; monopolies pioduce too little and chaige too much. Also, the high piice and peisistent
piofts stiike many as inequitable. Otheis may simply see monopoly as an unacceptable concentiation
of powei.
7A8L 10. 1 Characteristics of Perfect Competition and MonopoIy
Characteristic
or vent
Perfect Competition MonopoIy
V..ket |..ge runbe. c| se||e.s .rJ buye.s
p.cJuc|rg . |cncgerecus gccJ c.
se.v|ce, e.sy ert.y.
|..ge runbe. c| buye.s, cre se||e.. |rt.y |s b|cckeJ.
|en.rJ .rJ
n..g|r.|
.everue cu.ves
|e |.ns Jen.rJ .rJ n..g|r.|
.everue cu.ve |s . |c.|.crt.| ||re .t t|e
n..ket p.|ce.
|e |.n |.ces t|e n..ket Jen.rJ cu.ve, n..g|r.|
.everue |s be|c. n..ket Jen.rJ.
|.|ce |ete.n|reJ by Jen.rJ .rJ supp|y,
e.c| |.n |s . p.|ce t.ke.. |.|ce e,u.|s
n..g|r.| ccst.
|e ncrcpc|y |.n Jete.n|res p.|ce, |t |s . p.|ce
sette.. |.|ce |s g.e.te. t|.r n..g|r.| ccst.
|.c|t
n.x|n|..t|cr
||.ns p.cJuce .|e.e n..g|r.| ccst
e,u.|s n..g|r.| .everue
||.ns p.cJuce .|e.e n..g|r.| ccst e,u.|s n..g|r.|
.everue .rJ c|..ge t|e cc..espcrJ|rg p.|ce cr t|e
Jen.rJ cu.ve.
|.c|t |rt.y |c.ces eccrcn|c p.c|t tc .e.c |r
t|e |crg .ur.
bec.use ert.y |s b|cckeJ, . ncrcpc|y |.n c.r
sust.|r .r eccrcn|c p.c|t |r t|e |crg .ur.
||c|ercy |e e,u|||b.|un sc|ut|cr |s e|c|ert
bec.use p.|ce e,u.|s n..g|r.| ccst.
|e e,u|||b.|un sc|ut|cr |s |re|c|ert bec.use p.|ce |s
g.e.te. t|.r n..g|r.| ccst.
Public policy towaid monopoly geneially iecognizes two impoitant dimensions of the monopoly piob-
lem. On the one hand, the combining of competing fims into a monopoly cieates an inemcient and, to
268 PRINCIPLS OI CONOMICS
many, inequitable solution. On the othei hand, some industiies aie chaiacteiized as natuial monopol-
ies; pioduction by a single fim allows economies of scale that iesult in lowei costs.
The combining of competing fims into a monopoly fim oi unfaiily diiving competitois out of
business is geneially foibidden in the United States. Regulatoiy efoits to pievent monopoly fall undei
the puiview of the nation's antitiust laws, discussed in moie detail in a latei chaptei.
At the same time, we must be caieful to avoid the mistake of simply assuming that competition is
the alteinative to monopoly, that eveiy monopoly can and should be ieplaced by a competitive maiket.
One key souice of monopoly powei, aftei all, is economies of scale. In the case of natuial monopoly,
the alteinative to a single fim is many small, high-cost pioduceis. We may not like having only one
local piovidei of watei, but we might like even less having dozens of piovideis whose costs-and
piices-aie highei. Wheie monopolies exist because economies of scale pievail ovei the entiie iange of
maiket demand, they may seive a useful economic iole. We might want to iegulate theii pioduction
and piicing choices, but we may not want to give up theii cost advantages.
Wheie a natuial monopoly exists, the piice chaiged by the fim and othei aspects of its behavioi
may be subject to iegulation. Watei oi natuial gas, foi example, aie often distiibuted by a public util-
ity-a monopoly fim-at piices iegulated by a state oi local goveinment agency. Typically, such agen-
cies seek to foice the fim to chaige lowei piices, and to make less pioft, than it would otheiwise seek.
Although economists aie hesitant to levy blanket condemnations of monopoly, they aie geneially
shaiply ciitical of monopoly powei wheie no iationale foi it exists. When fims have substantial mono-
poly powei only as the iesult of goveinment policies that block entiy, theie may be little defense foi
theii monopoly positions.
Public policy towaid monopoly aims geneially to stiike the balance implied by economic analysis.
Wheie iationales exist, as in the case of natuial monopoly, monopolies aie peimitted-and theii piices
aie iegulated. In othei cases, monopoly is piohibited outiight. Societies aie likely to at least considei
taking action of some kind against monopolies unless they appeai to ofei cost oi othei technological
advantages.
3.2 The Fiagility of Monopoly Powei
An impoitant factoi in thinking about public policy towaid monopoly is to iecognize that monopoly
powei can be a feeting thing. Fiims constantly seek out the maiket powei that monopoly ofeis. When
conditions aie iight to achieve this powei, fims that succeed in caiving out monopoly positions enjoy
substantial piofts. But the potential foi high piofts invites continuing attempts to bieak down the bai-
iieis to entiy that insulate monopolies fiom competition.
Technological change and the puisuit of piofts chip away constantly at the entienched powei of
monopolies. Bieathtaking technological change has occuiied in the telecommunications industiy.
Catalog companies aie challenging the monopoly positions of some ietaileis; inteinet bookselleis and
online textbook companies such as Flatwoildknowledge.com aie challenging the monopoly powei of
youi univeisity's bookstoie; and Fedeial Expiess, UPS, and othei companies aie taking on the U.S.
Postal Seivice. The assaults on monopoly powei aie continuous. Thus, even the monopoly fim must
be on the lookout foi potential competitois.
Potential iivals aie always beating at the dooi and theieby making the monopoly's fiagile maiket
contestable-that is, open to entiy, at least in the sense of iival fims pioducing close enough," if not
peifect, substitutes-close enough that they might eliminate the fim's monopoly powei.
k Y 7 A k A W A Y S
< A ncrcpc|y |.n p.cJuces .r cutput t|.t |s |ess t|.r t|e e|c|ert |eve|. |e .esu|t |s . Je.J.e|g|t |css tc
scc|ety, g|ver by t|e ..e. bet.eer t|e Jen.rJ .rJ n..g|r.| ccst cu.ves cve. t|e ..rge c| cutput
bet.eer t|e cutput c|cser by t|e ncrcpc|y |.n .rJ t|e e|c|ert cutput.
< |e ||g|e. p.|ce c|..geJ by t|e ncrcpc|y |.n ccnp..eJ tc t|e pe.|ect|y ccnpet|t|ve |.n .eJuces
ccrsune. su.p|us, p..t c| .||c| |s t..rs|e..eJ tc t|e ncrcpc||st. ||s t..rs|e. gere..tes .r e,u|ty |ssue.
< |e ncrcpc|y |.ns n..ket pc.e. .eJuces ccrsune.s c|c|ces .rJ n.y .esu|t |r ||g|e. p.|ces, but t|e.e
n.y be .Jv.rt.ges tc ncrcpc|y .s .e||, suc| .s eccrcn|es c| sc.|e .rJ tec|rc|cg|c.| |rrcv.t|crs
erccu..geJ by t|e p.tert systen.
< |ub||c pc||cy tc...J ncrcpc|y ccrs|sts c| .rt|t.ust |..s .rJ .egu|.t|cr c| r.tu..| ncrcpc||es.
< |c.ces t|.t ||n|t t|e pc.e. c| ncrcpc|y |.ns ..e t|e ccrst.rt e|c.t by ct|e. |.ns tc c.ptu.e scne c|
t|e ncrcpc|y |.ns p.c|ts .rJ tec|rc|cg|c.| c|.rge t|.t e.cJes ncrcpc|y pc.e..
CHAP7R 10 MONOPOLY 269
7 R Y I 7 !
|ces t|e st.tenert be|c. bette. Jesc.|be . |.n cpe..t|rg |r . pe.|ect|y ccnpet|t|ve n..ket c. . |.n t|.t |s .
ncrcpc|y`
1. |e Jen.rJ cu.ve |.ceJ by t|e |.n |s Jc.r...Js|cp|rg.
2. |e Jen.rJ cu.ve .rJ t|e n..g|r.| .everue cu.ves ..e t|e s.ne.
3. |rt.y .rJ ex|t ..e .e|.t|ve|y J||cu|t.
4. |e |.n |s ||ke|y tc be ccrce.reJ .bcut .rt|t.ust |..s.
5. crsune. su.p|us .cu|J be |rc.e.seJ || t|e |.n p.cJuceJ nc.e cutput.
Case in Point: 7echnoIogicaI Change, PubIic PoIicy, and Competition in
7eIecommunications
2010 jupiterimages Corporation
b.ck |r t|e c|Jer J.ysbe|c.e 1984tc use . te|ep|cre |r t|e |r|teJ St.tes .|ncst ce.t.|r|y ne.rt be|rg .
custcne. c| A. V. be||, .s t|e ccnp.ry ..s krc.r, p.cv|JeJ |cc.| .rJ |crgJ|st.rce se.v|ce tc v|.tu.||y
eve.y |.S. |cuse|c|J. A ..s c|e..|y . ncrcpc|y.
|e 'ust|ce |ep..tnert beg.r |ts b.tt|e .|t| A |r t|e 190s, c|..g|rg |t .|t| ncrcpc||.|rg t|e |rJust.y.
|e c.se cu|n|r.teJ |r . |.rJn..k 1984 .u||rg t|.t b.cke t|e ccnp.ry up |rtc sever scc.||eJ b.by be||s
t|.t .cu|J p.cv|Je |cc.| te|ep|cre se.v|ce. A .cu|J ccrt|rue tc p.cv|Je |crgJ|st.rce se.v|ce.
|r e|ect, t|e .u||rg .ep|.ceJ . s|rg|e r.t|cr.| ncrcpc|y .|t| sever .eg|cr.| ncrcpc||es |r |cc.| te|ep|cre
se.v|ce. A n.|rt.|reJ |ts ncrcpc|y pcs|t|cr |r |crgJ|st.rce se.v|ce|c. . .|||e. |e tu.nc|| t|.t |.s |c|
|c.eJ |||ust..tes t|e |..g|||ty c| ncrcpc|y pc.e..
ec|rc|cg|c.| Jeve|cpnerts |r t|e |rJust.y |.ve b.cug|t J..n.t|c c|.rges. cnp.r|es ||ke V| |curJ ..ys
tc c|.||erge As ncrcpc|y pcs|t|cr |r |crgJ|st.rce te|ep|cre se.v|ce. .b|e cpe..tc.s sp..rg up, typ|c
.||y Jeve|cp|rg ncrcpc|y pc.e. cve. t|e p.cv|s|cr c| c.b|e te|ev|s|cr |r t|e|. .eg|cr.| n..kets. but, tec|rc
|cg|c.| c|.rge |.s e.cJeJ t|e ncrcpc|y pc.e. c| |cc.| te|ep|cre ccnp.r|es .rJ c| c.b|e cpe..tc.s. .b|e
ccnp.r|es |.ve begur p.cv|J|rg te|ep|cre se.v|ce |r ccnpet|t|cr .|t| |cc.| te|ep|cre ccnp.r|es, te|e
p|cre ccnp.r|es |.ve begur p.cv|J|rg c.b|e se.v|ces .s .e|| .s |rte.ret .ccess, .rJ t|e |rt.cJuct|cr c|
.|.e|ess ccnnur|c.t|crs |.s |u.t|e. b|u..eJ t|e J|st|rct|cr bet.eer J||e.ert types c| ccnp.r|es. |c., t|e
.e.Jy .v.||.b|||ty c| v|Jec se.v|ces cr t|e |rte.ret t|.e.ters tc n.ke c.b|e p.cv|Je.s cutncJeJ n|JJ|ener.
|e |||us|cr C.cup, . |.n t|.t p.cv|Jes .r.|ys|s c| t|e te|eccnnur|c.t|crs |rJust.y, p.eJ|cts t|.t by 2010
nc.e t|.r 300 n||||cr |cuse|c|Js .c.|J.|Je .||| |.ve ||g|speeJ |rte.ret .ccess .rJ .||| t|us be .b|e tc
t.ke .Jv.rt.ge c| |rc.e.s|rg .v.||.b|||ty c| v|Jec se.v|ces cr t|e |rte.ret.
270 PRINCIPLS OI CONOMICS
\|er ycu gc b.ck tc t|e e..|y 1980s, .e .e.e t.|k|rg .bcut . s|rg|e p.cJuct, |u.re Acke.n.r, |O c|
be||Scut| tc|J |e /o|| .|ee| .ooo|. S|rce t|er, t|e.e |.s beer . t.enerJcus exp|cs|cr c| tec|rc|cgy. |t |.s
c|.rgeJ eve.yt||rg.
.b|e ccnp.r|es crce |.J t.ct||.Js c| t|e n..ket |c. te|ev|s|cr se.v|ces. cJ.y, te|ep|cre ccnp.r|es ..e
n.k|rg |r.c.Js |rtc t|.t n..ket. |r 2005, te|ep|cre ccnp.r|es p|ckeJ up nc.e te|ev|s|cr custcne.s t|.r
c.b|e ccnp.r|es J|J. |e |e.ce ccnpet|t|cr |.s beer .e|ecteJ |r t|e |.|||rg stcck p.|ces c| c.b|e ccnp.r|es.
cnc.st c.p., t|e |..gest c.b|e ccnp.ry |r t|e |r|teJ St.tes .|t| 22 n||||cr subsc.|be.s, su|e.eJ . 22 .e
Juct|cr |r |ts stcck p.|ce |r 2005. VeJ|.ccn, t|e severt| |..gest c.b|e ccnp.ry, |.J . 33 .eJuct|cr |r |ts
stcck p.|ce bet.eer 2004 .rJ 2006. by 2008, t|e|. stcck p.|ces |.ve st||| rct .eccve.eJ.
A|.e.Jy, |ne \..re., . c.b|e ccnp.ry t|.t c|..ges |ts custcne.s 39.95 pe. ncrt|, |s c|e.|rg custcne.s
t|.t c.|| tc c.rce| t|e|. se.v|ce tc s.|tc| tc .rct|e. p.cv|Je. . J|sccurt tc 29.95. |ts . s.ve t.ct|c, .Jn|ts
ccnp.ry spckespe.scr V..k |....J. |t n.y be t|.t, but |t su.e|y .ppe..s tc be . |..b|rge. c| .|.t |s ccn|rg
tc te|eccnnur|c.t|crs.
.ooce |e|e Oo| oJ ^,. .c|o|. |o o|e O|o|. ^ |eo| | e /oe |eoo |o /o,.' |e /o|| .|ee| .ooo| ||e. /oc| .. 2006. ^
||oe .eoce,. ^, .c|o|. ^|o |ooo oJ |e| | |eo. ^ |eo ^ |o |o, |e||.oo||.' |e /o|| .|ee| .ooo| ||e. /oc| 6. 2006. ^
.oo .||.e. |e| | |eo. oJ |e||o c ^ooJ. | |oce| |eo|. .o ||.o| |oce |ooJ |o ||o.' |e /o|| .|ee| .ooo| ||e. /oc| 25.
2006. ^..oo .||.e. |e| | |eo. oJ |e||o c ^ooJ. | |oce| |eo|. .o ||.o| |oce ||o.' |e /o|| .|ee| .ooo| ||e. /oc| 25. 2006. ^
A N S W R S 7 O 7 R Y I 7 ! P R O 8 L M S
1. ncrcpc|y
2. pe.|ect ccnpet|t|cr
3. ncrcpc|y
4. ncrcpc|y
5. ncrcpc|y
4. REVIEW AND PRACTICE
Summary
||s c|.pte. |.s ex.n|reJ t|e p.c|tn.x|n|.|rg be|.v|c. c| ncrcpc|y |.ns. Vcrcpc|y cccu.s || .r |rJust.y
ccrs|sts c| . s|rg|e |.n .rJ ert.y |rtc t|.t |rJust.y |s b|cckeJ.
|ctert|.| scu.ces c| ncrcpc|y pc.e. |rc|uJe t|e ex|sterce c| eccrcn|es c| sc.|e cve. t|e ..rge c| n..ket
Jen.rJ, |cc.t|cr.| .Jv.rt.ges, ||g| surk ccsts .sscc|.teJ .|t| ert.y, .est.|cteJ c.re.s||p c| ... n.te.|.|s
.rJ |rputs, .rJ gcve.rnert .est.|ct|crs suc| .s ||cerses c. p.terts. |et.c.k e|ects |c. ce.t.|r p.cJucts |u.
t|e. |rc.e.se t|e n..ket pc.e. t|.t p.terts .|c.J.
bec.use t|e Jen.rJ cu.ve |.ceJ by t|e ncrcpc||st |s Jc.r...Js|cp|rg, t|e |.n |s . p.|ce sette.. |t .||| n.x
|n|.e p.c|ts by p.cJuc|rg t|e ,u.rt|ty c| cutput .t .||c| n..g|r.| ccst e,u.|s n..g|r.| .everue. |e p.c|t
n.x|n|.|rg p.|ce |s t|er |curJ cr t|e Jen.rJ cu.ve |c. t|.t ,u.rt|ty.
bec.use . typ|c.| ncrcpc||st |c|Js n..ket p.|ce .bcve n..g|r.| ccst, t|e n.c. |np.ct c| ncrcpc|y |s . .e
Juct|cr |r e|c|ercy. cnp..eJ tc . ccnpet|t|ve n..ket, t|e ncrcpc|y |s c|...cte.|.eJ by nc.e cert..||.eJ
pc.e., pctert|.| ||g|e. p.c|ts, .rJ |ess p.essu.e tc be .espcrs|ve tc ccrsune. p.e|e.erces. |ub||c pc||cy tc
...J ncrcpc|y |rc|uJes .rt|t.ust |..s .rJ, |r t|e c.se c| r.tu..| ncrcpc||es, .egu|.t|cr c| p.|ce .rJ ct|e.
.spects c| t|e |.ns be|.v|c..
CHAP7R 10 MONOPOLY 271
C O N C P 7 P R O 8 L M S
1. crs|Je. t|e |c||c.|rg |.ns. \cu|J ycu .eg..J .ry c| t|en .s . ncrcpc|y` \|y c. .|y rct` cu|J ycu
use t|e ncrcpc|y ncJe| |r .r.|y.|rg t|e c|c|ces c| .ry c| t|en` |xp|.|r.
.. t|e best .est.u..rt |r tc.r
b. ycu. b..be. c. be.ut|c|.r
c. ycu. |cc.| te|ep|cre ccnp.ry
J. ycu. c.npus bcckstc.e
e. V|c.csc|t
|. Ant..k
g. t|e |r|teJ St.tes |cst.| Se.v|ce
2. |xp|.|r t|e J||e.erce bet.eer t|e Jen.rJ cu.ve |.c|rg . ncrcpc|y |.n .rJ t|e Jen.rJ cu.ve |.c|rg .
pe.|ect|y ccnpet|t|ve |.n.
3. \|.t ..e t|e recess..y ccrJ|t|crs |c. . ncrcpc|y pcs|t|cr |r t|e n..ket tc be est.b||s|eJ`
4. A ncrcpc|y |.n |s |.ee tc c|..ge .ry p.|ce |t .|s|es. \|.t ccrst..|rs |ts c|c|ce c| . p.|ce`
5. Suppcse t|e gcve.rnert .e.e tc |npcse .r .rru.| ||cerse |ee cr . ncrcpc||st t|.t ust |.ppereJ tc be
e,u.| tc |ts eccrcn|c p.c|ts |c. . p..t|cu|.. ye... |c. .cu|J suc| . |ee .|ect p.|ce .rJ cutput` |c ycu
t||rk t|.t suc| . |ee .cu|J be .pp.cp.|.te` \|y c. .|y rct`
6. |.ne cre ncrcpc|y |.n ycu Je.| .|t|. \|.t |s t|e scu.ce c| |ts ncrcpc|y pc.e.` |c ycu t||rk |t seeks
tc n.x|n|.e |ts p.c|ts`
. A ncrcpc||st .||| reve. p.cJuce sc nuc| cutput .s tc cpe..te |r t|e |re|.st|c pc.t|cr c| t|e Jen.rJ
cu.ve. |xp|.|r.
8. A ncrcpc|y |s rct e|c|ert, .rJ |ts p.|c|rg be|.v|c. |e.Js tc |csses tc scc|ety. \|.t Jces t||s st.tenert
ne.r` S|cu|J scc|ety b.r ncrcpc||es`
9. A sn.|| tc.r |cc.teJ 30 n||es |.cn t|e re..est tc.r |.s cr|y cre se.v|ce st.t|cr. |s t|e se.v|ce st.t|cr .
ncrcpc|y` \|y c. .|y rct`
10. |xp|.|r .|y urJe. ncrcpc|y p.|ce |s g.e.te. t|.r n..g|r.| .everue, .|||e urJe. pe.|ect ccnpet|t|cr
p.|ce |s e,u.| tc n..g|r.| .everue.
11. |r .|.t serse c.r t|e ncrcpc|y e,u|||b.|un be ccrs|Je.eJ |re,u|t.b|e`
12. \|.t |s . r.tu..| ncrcpc|y` S|cu|J . r.tu..| ncrcpc|y be .||c.eJ tc ex|st`
13. C|ve scne ex.np|es c| |rJust.|es |r .||c| ycu t||rk r.tu..| ncrcpc|y ccrJ|t|crs ..e ||ke|y tc p.ev.||.
\|y Jc ycu t||rk sc`
14. |ecp|e c|ter b|.ne t|e ||g| p.|ces |c. everts suc| .s p.c|ess|cr.| |cctb.|| .rJ b.sketb.|| .rJ b.seb.||
g.nes cr t|e ||g| s.|..|es c| p.c|ess|cr.| .t||etes. Assun|rg cre c| t|ese te.ns |s . ncrcpc|y, use t|e
ncJe| tc .e|ute t||s ..gunert.
15. |c. Jc t|e |c||c.|rg everts .|ect . ncrcpc|y |.ns p.|ce .rJ cutput` |c. .||| |t .|ect t|e |.ns
p.c|ts` |||ust..te ycu. .rs.e.s g..p||c.||y.
.. .r |rc.e.se |r |.bc. ccsts |r t|e n..ket |r .||c| t|e |.n cpe..tes
b. . .eJuct|cr |r t|e p.|ce c| g.sc||re
c. t|e |.ns ||e| |xecut|ve O|ce. pe.su.Jes t|e bc..J tc |rc.e.se ||s c. |e. .rru.| s.|..y
J. Jen.rJ |c. t|e |.ns p.cJuct |.||s
e. Jen.rJ |c. t|e |.ns p.cJuct .|ses
|. t|e p.|ce c| . subst|tute |c. t|e |.ns p.cJuct .|ses
272 PRINCIPLS OI CONOMICS
N U M R I C A L P R O 8 L M S
1. A ur|ve.s|ty |cctb.|| te.n est|n.tes t|.t |t |.ces t|e Jen.rJ sc|eJu|e s|c.r |c. t|ckets |c. e.c| |cne
g.ne |t p|.ys. |e te.n p|.ys |r . st.J|un t|.t |c|Js 60,000 |.rs. |t est|n.tes t|.t |ts n..g|r.| ccst c|
.tterJ.rce, .rJ t|us |c. t|ckets sc|J, |s .e.c.
Price per ticket 7ickets per game
100 0
80 20,000
60 40,000
40 60,000
20 80,000
0 100,000
.. |... t|e Jen.rJ .rJ n..g|r.| .everue cu.ves. cnpute t|e te.ns p.c|tn.x|n|.|rg p.|ce
.rJ t|e runbe. c| t|ckets |t .||| se|| .t t|.t p.|ce.
b. |ete.n|re t|e p.|ce e|.st|c|ty c| Jen.rJ .t t|e p.|ce ycu Jete.n|reJ |r p..t (.).
c. |c. nuc| tct.| .everue .||| t|e te.n e..r`
J. |c. suppcse t|e c|ty |r .||c| t|e ur|ve.s|ty |s |cc.teJ |npcses . 10,000 .rru.| ||cerse |ee cr
.|| supp||e.s c| spc.t|rg everts, |rc|uJ|rg t|e |r|ve.s|ty. |c. Jces t||s .|ect t|e p.|ce c| t|ckets`
e. Suppcse t|e te.n |rc.e.ses |ts sperJ|rg |c. sc|c|..s||ps |c. |ts .t||etes. |c. .||| t||s .|ect
t|cket p.|ces, .ssun|rg t|.t |t ccrt|rues tc n.x|n|.e p.c|t`
|. |c. suppcse t|.t t|e c|ty |npcses . t.x c| 10 pe. t|cket sc|J. |c. .cu|J t||s .|ect t|e p.|ce
c|..geJ by t|e te.n`
2. A ncrcpc|y |.n |.ces . Jen.rJ cu.ve g|ver by t|e |c||c.|rg e,u.t|cr. | 500 10(, .|e.e ( e,u.|s
,u.rt|ty sc|J pe. J.y. |ts n..g|r.| ccst cu.ve |s MC = $100 pe. J.y. Assune t|.t t|e |.n |.ces rc |xeJ
ccst.
.. |c. nuc| .||| t|e |.n p.cJuce`
b. |c. nuc| .||| |t c|..ge`
c. .r ycu Jete.n|re |ts p.c|t pe. J.y` (||rt. ycu c.r, st.te |c. nuc| |t |s.)
J. Suppcse . t.x c| 1,000 pe. J.y |s |npcseJ cr t|e |.n. |c. .||| t||s .|ect |ts p.|ce`
e. |c. .cu|J t|e 1,000 pe. J.y t.x |ts cutput pe. J.y`
|. |c. .cu|J t|e 1,000 pe. J.y t.x .|ect |ts p.c|t pe. J.y`
g. |c. suppcse . t.x c| 100 pe. ur|t |s |npcseJ. |c. .||| t||s .|ect t|e |.ns p.|ce`
|. |c. .cu|J . 100 pe. ur|t t.x .|ect t|e |.ns p.c|t n.x|n|.|rg cutput pe. J.y`
|. |c. .cu|J t|e 100 pe. ur|t t.x .|ect t|e |.ns p.c|t pe. J.y`
CHAP7R 10 MONOPOLY 273
274 PRINCIPLS OI CONOMICS
| A | | | 1 1
The World of Imperfect
Competition
S7AR7 UP: 8AY NDS GOOGL, GOOGL NDS
8AY, AND NI7HR 7RUS7S 7H O7HR
|e |rte.ret .uct|cr s|te eb.y |.s |.J . c|cse .rJ cccpe..t|ve .e|.t|crs||p .|t| Cccg|e, t|e g|.rt se..c| erg|re.
eb.y |.s .e||eJ |e.v||y cr Cccg|e tc .Jve.t|se |ts p.cJucts. Cccg|e .e||es |e.v||y cr t|e .Jve.t|s|rg .everue |t gets
|.cn eb.y. |e g.e.te. t|e success c| eb.y, t|e g.e.te. t|e .everue Cccg|e .||| |.ve |.cn eb.ys .Jve.t|s|rg. |e
g.e.te. t|e success c| Cccg|e .s . se..c| erg|re, t|e g.e.te. .||| be t|e |np.ct c| eb.ys .Jve.t|s|rg. c p...p|..se
||cks ||re |.cn .s.b|.rc., ||s ccu|J be . be.ut||u| .e|.t|crs||p. |t |s rct. |e t.c |rte.ret g|.rts s|np|y Jc rct
get .|crg.
crs|Je. .|.t |.ppereJ |r 200. A Cccg|e spckesn.r s.|J t|e |.n ..s |cst|rg . |.eeJcn |..ty tc .r
rcurce t|e |r.ugu..t|cr c| . re. p.ynerts se.v|ce t|.t .cu|J ccnpete J|.ect|y .|t| |.y|.|, t|e cr||re p.ynert
se.v|ce c.reJ by eb.y. eb.y ..s ,u|ck tc .et.||.te. |t pu||eJ .|| c| |ts .Jve.t|s|rg |.cn Cccg|e |.te. cr t|e s.ne J.y
Cccg|e n.Je |ts .rrcurcenert. .c J.ys |.te., Cccg|e b.ckeJ Jc.r. |t c.rce|eJ |ts p..ty .rJ t|e p.ynert se.
v|ce t|e p..ty ..s tc k|ck c|.
1
|r 2003, eb.y |.J ccnn|ss|creJ .r .r.|ys|s c| .|et|e. Cccg|e .ep.eserteJ . t|.e.t tc |ts cpe..t|crs. |e
stuJy ccrc|uJeJ t|.t Cccg|e ..s ur||ke|y tc erte. |rtc eccnne.ce .rJ ..s rct . pctert|.| .|v.| tc eb.y. |.t s.r
gu|re ccrc|us|cr st..teJ tc ur..ve| |r 2005. Cccg|e beg.r .ec.u|t|rg eb.y erg|ree.s. |r Octcbe., Cccg|e st..teJ
test|rg Cccg|e b.se, . |.ee c|.ss||eJ .Jve.t|s|rg se.v|ce t|.t t|.e.tereJ eb.ys .uct|cr se.v|ce.
|xecut|ves .t eb.y tcck t|e t|.e.t se.|cus|y. |r p.|v.te neet|rgs, t|ey J|v|JeJ |rtc t.c te.ns. A g.eer te.n
.ep.eserteJ eb.ys |rte.ests, . .eJ te.n t.|eJ tc enu|.te Cccg|es st..tegy. |e .eJ te.n ccrc|uJeJ t|.t Cccg|e
.ep.eserteJ . se.|cus t|.e.t, .rJ eb.y execut|ves beg.r exp|c..tc.y t.|ks .|t| V|c.csc|t .rJ +.|cc tc see || scne
cc||.bc..t|ve e|c.t ccu|J ...J c| t|e Cccg|e t|.e.t.
eb.y spckesn.r |.|s |cr|.y Jesc.|bes t|e |.ns J||enn. c| Je.||rg .|t| . |.n t|.t |.s beer . v.|u.b|e .||y
but .t t|e s.ne t|ne ccu|J be . ccnpet|t|ve t|.e.t. C|ver |c. .e.||y |.st t|e |rte.ret c|.rges, |t ccnes .s rc su.
p.|se t|.t t|e ||re bet.eer ccnpet|t|cr .rJ cccpe..t|cr |s scnet|nes b|u..y.
by t|e |.te sp.|rg c| 2006, eb.ys n.r.genert ..s st||| |r . ,u.rJ..y .bcut .|.t tc Jc .bcut Cccg|e. Scne
execut|ves, |e..|u| c| |cs|rg t|e .Jv.rt.ges c| ccrt|ru|rg tc .c.k .|t| Cccg|e, ..rt tc n.|rt.|r eb.ys t|es tc t|e
|.n. Ot|e.s .c..|eJ t|.t ccrt|ru|rg . c|cse .e|.t|crs||p .|t| Cccg|e ..s .k|r tc putt|rg t|e |cx |r t|e p.cve.b|.|
|er|cuse. |ey ..rt tc ncve ,u|ck|y tc est.b||s| . .e|.t|crs||p .|t| +.|cc c. .|t| V|c.csc|t t|.t .cu|J ccnpete
.|t| Cccg|e.
2
|e ters|cr bet.eer eb.y .rJ Cccg|e |..J|y suggests t|e .|cc| .c.|J c| pe.|ect ccnpet|t|cr .|e.e ccr
sune.s ..e |rJ||e.ert .bcut .||c| |.n |.s p.cJuceJ . p..t|cu|.. p.cJuct, .|e.e e.c| |.n krc.s |t c.r se|| .|| |t
imperfect competition
A n..ket st.uctu.e .|t| nc.e
t|.r cre |.n |r .r |rJust.y
|r .||c| .t |e.st cre |.n |s .
p.|ce sette..
monopoIistic competition
A ncJe| c|...cte.|.eJ by
n.ry |.ns p.cJuc|rg s|n||..
but J||e.ert|.teJ p.cJucts |r
. n..ket .|t| e.sy ert.y .rJ
ex|t.
..rts .t t|e gc|rg n..ket p.|ce, .|e.e |.ns nust sett|e |c. .e.c eccrcn|c p.c|t |r t|e |crg .ur. |c. |s |t t|e .c.|J
c| ncrcpc|y, .|e.e . s|rg|e |.n n.x|n|.es |ts p.c|ts, be||ev|rg t|.t b...|e.s tc ert.y .||| keep cut .cu|Jbe ccn
pet|tc.s, .t |e.st |c. . .|||e. ||s |s t|e .c.|J c| |npe.|ect ccnpet|t|cr, cre t|.t ||es bet.eer t|e |Je.||.eJ ex
t.enes c| pe.|ect ccnpet|t|cr .rJ ncrcpc|y. |t |s . .c.|J |r .||c| |.ns b.tt|e cve. n..ket s|..es, |r .||c| ecc
rcn|c p.c|ts n.y pe.s|st, |r .||c| .|v.|s t.y tc cutguess e.c| ct|e. .|t| p.|c|rg, .Jve.t|s|rg, .rJ p.cJuctJeve|cp
nert st..teg|es.
|r||ke t|e c|.pte.s cr pe.|ect ccnpet|t|cr .rJ ncrcpc|y, t||s c|.pte. Jces rct p.cv|Je . s|rg|e ncJe| tc ex
p|.|r |.ns be|.v|c.. |e.e ..e tcc n.ry v..|.t|crs cr .r urce.t.|r t|ene |c. cre ncJe| tc exp|.|r t|e ccnp|ex|t
|es c| |npe.|ect ccnpet|t|cr. |.t|e., t|e c|.pte. p.cv|Jes .r cve.v|e. c| scne c| t|e n.ry J||e.ert ncJe|s .rJ
exp|.r.t|crs .Jv.rceJ by eccrcn|sts |c. t|e be|.v|c. c| |.ns |r t|e |npe.|ect|y ccnpet|t|ve n..kets. |e .r.|yt
|c.| tcc|s ycu |.ve .c,u|.eJ |r t|e ccu.se c| stuJy|rg t|e ncJe|s c| ccnpet|t|ve .rJ ncrcpc|y n..kets .||| be
ve.y nuc| |r ev|Jerce |r t||s J|scuss|cr.
|e spect.un c| bus|ress erte.p.|se ..rges |.cn pe.|ect|y ccnpet|t|ve |.ns tc ncrcpc|y. bet.eer t|ese ex
t.enes ||es t|e bus|ress |.rJsc.pe |r .||c| t|e v.st n.c.|ty c| |.nst|cse |r t|e .c.|J c| |npe.|ect ccnpet|
t|cr.ctu.||y cpe..te. Imperfect competition |s . n..ket st.uctu.e .|t| nc.e t|.r cre |.n |r .r |rJust.y |r
.||c| .t |e.st cre |.n |s . p.|ce sette.. Ar |npe.|ect|y ccnpet|t|ve |.n |.s . Jeg.ee c| ncrcpc|y pc.e., e|t|e.
b.seJ cr p.cJuct J||e.ert|.t|cr t|.t |e.Js tc . Jc.r...Js|cp|rg Jen.rJ cu.ve c. .esu|t|rg |.cn t|e |rte..ct|cr
c| .|v.| |.ns |r .r |rJust.y .|t| cr|y . |e. |.ns.
|e.e ..e t.c b.c.J c.tegc.|es c| |npe.|ect|y ccnpet|t|ve n..kets. |e |.st |s cre |r .||c| n.ry |.ns ccn
pete, e.c| c|e.|rg . s||g|t|y J||e.ert p.cJuct. |e seccrJ |s cre |r .||c| t|e |rJust.y |s Jcn|r.teJ by . |e. |.ns.
|npc.t.rt |e.tu.es c| bct| k|rJs c| n..kets ..e .Jve.t|s|rg .rJ p.|ce J|sc.|n|r.t|cr, .||c| .e ex.n|re |.te. |r t||s
c|.pte..
1. MONOPOLISTIC COMPETITION: COMPETITION
AMONG MANY
L A R N I N G O 8 1 C 7 I V S
1. xpIain the main characteristics of a monopoIisticaIIy competitive industry, describing both its
simiIarities and dierences from the modeIs of perfect competition and monopoIy.
2. xpIain and iIIustrate both short-run equiIibrium and Iong-run equiIibrium for a monopoIistic-
aIIy competitive rm.
3. xpIain what it means to say that a rm operating under monopoIistic competition has excess
capacity in the Iong run and discuss the impIications of this concIusion.
The fist model of an impeifectly competitive industiy that we shall investigate has conditions quite
similai to those of peifect competition. The model of monopolistic competition assumes a laige num-
bei of fims. It also assumes easy entiy and exit. This model difeis fiom the model of peifect competi-
tion in one key iespect: it assumes that the goods and seivices pioduced by fims aie difeientiated.
This difeientiation may occui by viitue of adveitising, convenience of location, pioduct quality, iepu-
tation of the sellei, oi othei factois. Pioduct difeientiation gives fims pioducing a paiticulai pioduct
some degiee of piice-setting oi monopoly powei. Howevei, because of the availability of close substi-
tutes, the piice-setting powei of monopolistically competitive fims is quite limited. Monopolistic
competition is a model chaiacteiized by many fims pioducing similai but difeientiated pioducts in
a maiket with easy entiy and exit.
276 PRINCIPLS OI CONOMICS
oIigopoIy
S|tu.t|cr |r .||c| . n..ket |s
Jcn|r.teJ by . |e. |.ns,
e.c| c| .||c| .eccgr|.es
t|.t |ts c.r .ct|crs .|||
p.cJuce . .espcrse |.cn |ts
.|v.|s .rJ t|.t t|cse
.espcrses .||| .|ect |t.
concentration ratio
|e pe.cert.ge c| cutput
.cccurteJ |c. by t|e |..gest
|.ns |r .r |rJust.y.
2. OLIGOPOLY: COMPETITION AMONG THE FEW
L A R N I N G O 8 1 C 7 I V S
1. xpIain the main characteristics of an oIigopoIy, dierentiating it from other types of market
structures.
2. xpIain the measures that are used to determine the degree of concentration in an industry.
3. xpIain and iIIustrate the coIIusion modeI of oIigopoIy.
4. Discuss how game theory can be used to understand the behavior of rms in an oIigopoIy.
In July, 2003, Geneial Motois Coipoiation (GMC) ofeied employee discount piicing" to viitually all
GMC customeis, not just employees and theii ielatives. This new maiketing stiategy intioduced by
GMC obviously afected Foid, Chiyslei, Toyota and othei automobile and tiuck manufactuieis; Foid
matched GMC's employee-discount plan by ofeiing up to $1,000 to its own employees who convinced
fiiends to puichase its cais and tiucks. Foid also ofeied its customeis the same piices paid by its em-
ployees. By mid-July, Chiyslei indicated that it was looking at many alteinatives, but was waiting foi
GMC to make its next move. Ultimately, Chiyslei also ofeied employee discount piicing.
Toyota had to iespond. It quickly developed a new maiketing stiategy of its own, which included
loweiing the piices of its cais and ofeiing new fnancing teims. The iesponses of Foid, Chiyslei, and
Toyota to GMC's piicing stiategy obviously afected the outcome of that stiategy. Similaily, a decision
by Pioctei & Gamble to lowei the piice of Ciest toothpaste may elicit a iesponse fiom Colgate-Pal-
molive, and that iesponse will afect the sales of Ciest. In an oligopoly, the fouith and fnal maiket
stiuctuie that we will study, the maiket is dominated by a few fims, each of which iecognizes that its
own actions will pioduce a iesponse fiom its iivals and that those iesponses will afect it.
The fims that dominate an oligopoly iecognize that they aie inteidependent: What one fim does
afects each of the otheis. This inteidependence stands in shaip contiast to the models of peifect com-
petition and monopolistic competition, wheie we assume that each fim is so small that it assumes the
iest of the maiket will, in efect, ignoie what it does. A peifectly competitive fim iesponds to the mai-
ket, not to the actions of any othei fim. A monopolistically competitive fim iesponds to its own de-
mand, not to the actions of specifc iivals. These piesumptions gieatly simplify the analysis of peifect
competition and monopolistic competition. We do not have that luxuiy in oligopoly, wheie the intei-
dependence of fims is the defning chaiacteiistic of the maiket.
Some oligopoly industiies make standaidized pioducts: steel, aluminum, wiie, and industiial tools.
Otheis make difeientiated pioducts: cigaiettes, automobiles, computeis, ieady-to-eat bieakfast ceieal,
and soft diinks.
2.1 Measuiing Concentiation in Oligopoly
Oligopoly means that a few fims dominate an industiy. But how many is a few," and how laige a
shaie of industiy output does it take to dominate" the industiy:
Compaie, foi example, the ieady-to-eat bieakfast ceieal industiy and the ice cieam industiy. The
ceieal maiket is dominated by two fims, Kellogg's and Geneial Mills, which togethei hold moie than
half the ceieal maiket. This oligopoly opeiates in a highly concentiated maiket. The maiket foi ice
cieam, wheie the foui laigest fims account foi just less than a thiid of output, is much less
concentiated.
One way to measuie the degiee to which output in an industiy is concentiated among a few fims
is to use a concentration ratio, which iepoits the peicentage of output accounted foi by the laigest
fims in an industiy. The highei the concentiation iatio, the moie the fims in the industiy take ac-
count of theii iivals' behavioi. The lowei the concentiation iatio, the moie the industiy iefects the
chaiacteiistics of monopolistic competition oi peifect competition.
The U.S. Census Buieau, based on suiveys it conducts of manufactuiing fims eveiy fve yeais, ie-
poits concentiation iatios. These suiveys show concentiation iatios foi the laigest 4, 8, 20, and 30 fims
in each industiy categoiy. Some concentiation iatios fiom the 2002 suivey, the latest available, aie ie-
poited in Table 11.1. Notice that the foui-fim concentiation iatio foi bieakfast ceieals is 78; foi ice
cieam it is 48.
CHAP7R 11 7H WORLD OI IMPRIC7 COMP7I7ION 283
HerndahI-Hirschman
Index
Ar .|te.r.t|ve ne.su.e c|
ccrcert..t|cr |curJ by
s,u..|rg t|e pe.cert.ge
s|..e (st.teJ .s . .|c|e
runbe.) c| e.c| |.n |r .r
|rJust.y, t|er sunn|rg
t|ese s,u..eJ n..ket s|..es.
duopoIy
Ar |rJust.y t|.t |.s cr|y t.c
|.ns.
7A8L 11. 1 Concentration Ratios and HerfindahI-Hirschman Indexes
.c ne.su.es c| |rJust.y ccrcert..t|cr ..e .epc.teJ by t|e ersus bu.e.u. ccrcert..t|cr ..t|cs .rJ t|e
|e.|rJ.||||.sc|n.r |rJex (|||).
Industry Largest 4
rms
Largest 8
rms
Largest 20
rms
Largest 50
rms
HHI
|ce c.e.n 48 64 82 93 36
b.e.k|.st ce.e.|s 8 91 99 100 2521
|g..ettes 95 99 100 |
Vers .rJ bcys s||.ts 38 53 3 89 481
\cners .rJ g|.|s b|cuses .rJ
s||.ts
21 32 49 0 186
Autcncb||es 6 94 99 100 1911
Spc.t|rg .rJ .t||et|c gccJs 23 32 46 62 182
|ert.| |.bc..tc.|es 13 18 23 30 54
"D, data withheId by the government to avoid reveaIing information about specic rms.
Source Selected statistics from Sector 31 Manufacturing Subject Series-Concentration Ratios Share of Value of Shipments Accounted for by the 4, 8,
20, and 50 Largest Companies for Industries 2002 at http//www.census.gov/epcd/www/concentration.html.
An alteinative measuie of concentiation is found by squaiing the peicentage shaie (stated as a whole
numbei) of each fim in an industiy, then summing these squaied maiket shaies to deiive a Her-
nndahl-Hirschman Index (HHI). The laigest HHI possible is the case of monopoly, wheie one
fim has 100 of the maiket; the index is 100
2
, oi 10,000. An industiy with two fims, each with 30 of
total output, has an HHI of 3,000 (30
2
+ 30
2
). In an industiy with 10,000 fims that have 0.01 of the
maiket each, the HHI is 1. Heifndahl-Hiischman Indexes iepoited by the Census Buieau aie also giv-
en in Table 11.1. Notice that the HHI is 2,321 foi bieakfast ceieals and only 736 foi ice cieam, suggest-
ing that the ice cieam industiy is moie competitive than the bieakfast ceieal industiy.
In some cases, the census data undeistate the degiee to which a few fims dominate the maiket.
One pioblem is that industiy categoiies may be too bioad to captuie signifcant cases of industiy dom-
inance. The spoiting goods industiy, foi example, appeais to be highly competitive if we look just at
measuies of concentiation, but maikets foi individual goods, such as golf clubs, iunning shoes, and
tennis iackets, tend to be dominated by a few fims. Fuithei, the data iefect shaies of the national mai-
ket. A tendency foi iegional domination does not show up. Foi example, the conciete industiy appeais
to be highly competitive. But conciete is pioduced in local maikets-it is too expensive to ship it veiy
fai-and many of these local maikets aie dominated by a handful of fims.
The census data can also oveistate the degiee of actual concentiation. The automobiles" categoiy,
foi example, has a foui-fim concentiation iatio that suggests the industiy is stiongly dominated by
foui laige fims (in fact, U.S. pioduction is dominated by thiee: Geneial Motois, Foid, and Chiyslei).
Those fims haidly account foi all cai sales in the United States, howevei, as othei foieign pioduceis
have captuied a laige poition of the domestic maiket. Including those foieign competitois suggests a
fai less concentiated industiy than the census data imply.
2.2 The Collusion Model
Theie is no single model of pioft-maximizing oligopoly behavioi that coiiesponds to economists'
models of peifect competition, monopoly, and monopolistic competition. Unceitainty about the intei-
action of iival fims makes specifcation of a single model of oligopoly impossible. Instead, economists
have devised a vaiiety of models that deal with the unceitain natuie of iivals' iesponses in difeient
ways. In this section we ieview one type of oligopoly model, the collusion model. Aftei examining this
tiaditional appioach to the analysis of oligopoly behavioi, we shall tuin to anothei method of examin-
ing oligopolistic inteiaction: game theoiy.
Fiims in any industiy could achieve the maximum pioft attainable if they all agieed to select the
monopoly piice and output and to shaie the piofts. One appioach to the analysis of oligopoly is to as-
sume that fims in the industiy collude, selecting the monopoly solution.
Suppose an industiy is a duopoly, an industiy with two fims. Figuie 11.3 shows a case in which
the two fims aie identical. They sell identical pioducts and face identical demand and cost conditions.
To simplify the analysis, we will assume that each has a hoiizontal maiginal cost cuive, MC. The de-
mand and maiginal ievenue cuives aie the same foi both fims. We fnd the combined demand cuive
foi the two fims, D
combined
, by adding the individual demand cuives togethei. Because one fim's de-
mand cuive, D
fim
, iepiesents one-half of maiket demand, it is the same as the combined maiginal
284 PRINCIPLS OI CONOMICS
overt coIIusion
\|er |.ns cper|y .g.ee cr
p.|ce, cutput, .rJ ct|e.
Jec|s|crs .|neJ .t .c||ev|rg
ncrcpc|y p.c|ts.
carteI
||.ns t|.t ccc.J|r.te t|e|.
.ct|v|t|es t|.cug| cve.t
cc||us|cr .rJ by |c.n|rg
cc||us|ve ccc.J|r.t|rg
nec|.r|sns.
ievenue cuive foi the two fims. If these two fims act as a monopoly, togethei they pioduce Q
m
and
chaige a piice P
m
. This iesult is achieved if each fim selects its pioft-maximizing output, which equals
1/2 Q
m
. This solution is inemcient; the emcient solution is piice P
c
and output Q
c
, found wheie the
combined maiket demand cuive D
combined
and the maiginal cost cuive MC inteisect.
II GUR 11. 5 MonopoIy 7hrough CoIIusion
.c |Jert|c.| |.ns |.ve t|e s.ne |c.|.crt.| n..g|r.| ccst cu.ve / |e|. Jen.rJ cu.ves |
|.n
.rJ n..g|r.|
.everue cu.ves /|
|.n
..e .|sc |Jert|c.|. |e ccnb|reJ Jen.rJ cu.ve |s |
ccnb|reJ
, t|e ccnb|reJ n..g|r.|
.everue cu.ve |s /|
ccnb|reJ
. |e p.c|ts c| t|e t.c |.ns ..e n.x|n|.eJ || e.c| p.cJuces 1/2 (
n
.t pc|rt A.
|rJust.y cutput .t pc|rt b |s t|us (
n
.rJ t|e p.|ce |s |
n
. At pc|rt , t|e e|c|ert sc|ut|cr cutput .cu|J be (
c
, .rJ
t|e p.|ce .cu|J e,u.| /.
In the simplest foim of collusion, overt collusion, fims openly agiee on piice, output, and othei de-
cisions aimed at achieving monopoly piofts. Fiims that cooidinate theii activities thiough oveit collu-
sion and by foiming collusive cooidinating mechanisms make up a cartel.
Fiims foim a caitel to gain monopoly powei. A successful caitel can eain laige piofts, but theie
aie seveial pioblems with foiming and maintaining one. Fiist, in many countiies, including the United
States, caitels aie geneially illegal.
[3]
They aie banned, because theii puipose is to iaise piices and ie-
stiict output. Second, the caitel may not succeed in inducing all fims in the industiy to join. Fiims that
iemain outside the caitel can compete by loweiing piice, and thus they pievent the caitel fiom achiev-
ing the monopoly solution. Thiid, theie is always an incentive foi individual membeis to cheat on cai-
tel agieements. Suppose the membeis of a caitel have agieed to impose the monopoly piice in theii
maiket and to limit theii output accoidingly. Any one fim might calculate that it could chaige slightly
less than the caitel piice and thus captuie a laigei shaie of the maiket foi itself. Cheating fims expand
output and diive piices down below the level oiiginally chosen.
The Oiganization of Petioleum Expoiting Countiies (OPEC), peihaps the best-known caitel, is
made up of 13 oil-pioducing countiies. In the 1970s, OPEC successfully acted like a monopoly by ie-
stiicting output and iaising piices. By the mid-1980s, howevei, the monopoly powei of the caitel had
been weakened by expansion of output by nonmembei pioduceis such as Mexico and Noiway and by
cheating among the caitel membeis.
CHAP7R 11 7H WORLD OI IMPRIC7 COMP7I7ION 285
tacit coIIusion
Ar ur..|tter, urspcker
urJe.st.rJ|rg t|.cug|
.||c| |.ns .g.ee tc ||n|t
t|e|. ccnpet|t|cr.
strategic choice
A c|c|ce b.seJ cr t|e
.eccgr|t|cr t|.t t|e .ct|crs
c| ct|e.s .||| .|ect t|e
cutccne c| t|e c|c|ce .rJ
t|.t t.kes t|ese pcss|b|e
.ct|crs |rtc .cccurt.
game theory
Ar .r.|yt|c.| .pp.c.c|
t|.cug| .||c| st..teg|c
c|c|ces c.r be .ssesseJ.
payo
|e cutccne c| . st..teg|c
Jec|s|cr.
An alteinative to oveit collusion is tacit collusion, an unwiitten, unspoken undeistanding
thiough which fims agiee to limit theii competition. Fiims may, foi example, begin following the
piice leadeiship of a paiticulai fim, iaising oi loweiing theii piices when the leadei makes such a
change. The piice leadei may be the laigest fim in the industiy, oi it may be a fim that has been pai-
ticulaily good at assessing changes in demand oi cost. At vaiious times, tacit collusion has been alleged
to occui in a wide iange of industiies, including steel, cais, and bieakfast ceieals.
It is dimcult to know how common tacit collusion is. The fact that one fim changes its piice
shoitly aftei anothei one does cannot piove that a tacit conspiiacy exists. Aftei all, we expect to see the
piices of all fims in a peifectly competitive industiy moving togethei in iesponse to changes in de-
mand oi pioduction costs.
2.3 Game Theoiy and Oligopoly Behavioi
Oligopoly piesents a pioblem in which decision makeis must select stiategies by taking into account
the iesponses of theii iivals, which they cannot know foi suie in advance. The Stait Up featuie at the
beginning of this chaptei suggested the unceitainty eBay faces as it consideis the possibility of competi-
tion fiom Google. A choice based on the iecognition that the actions of otheis will afect the outcome
of the choice and that takes these possible actions into account is called a strategic choice. Game
theory is an analytical appioach thiough which stiategic choices can be assessed.
Among the stiategic choices available to an oligopoly fim aie piicing choices, maiketing
stiategies, and pioduct-development efoits. An aiiline's decision to iaise oi lowei its faies-oi to leave
them unchanged-is a stiategic choice. The othei aiilines' decision to match oi ignoie theii iival's
piice decision is also a stiategic choice. IBM boosted its shaie in the highly competitive peisonal com-
putei maiket in laige pait because a stiategic pioduct-development stiategy acceleiated the fim's in-
tioduction of new pioducts.
Once a fim implements a stiategic decision, theie will be an outcome. The outcome of a stiategic
decision is called a payon. In geneial, the payof in an oligopoly game is the change in economic pioft
to each fim. The fim's payof depends paitly on the stiategic choice it makes and paitly on the stia-
tegic choices of its iivals. Some fims in the aiiline industiy, foi example, iaised theii faies in 2003, ex-
pecting to enjoy incieased piofts as a iesult. They changed theii stiategic choices when othei aiilines
chose to slash theii faies, and all fims ended up with a payof of lowei piofts-many went into
bankiuptcy.
We shall use two applications to examine the basic concepts of game theoiy. The fist examines a
classic game theoiy pioblem called the piisoneis' dilemma. The second deals with stiategic choices by
two fims in a duopoly.
7he Prisoners' DiIemma
Suppose a local distiict attoiney (DA) is ceitain that two individuals, Fiankie and Johnny, have com-
mitted a buiglaiy, but she has no evidence that would be admissible in couit.
The DA aiiests the two. On being seaiched, each is discoveied to have a small amount of cocaine.
The DA now has a suie conviction on a possession of cocaine chaige, but she will get a conviction on
the buiglaiy chaige only if at least one of the piisoneis confesses and implicates the othei.
The DA decides on a stiategy designed to elicit confessions. She sepaiates the two piisoneis and
then ofeis each the following deal: If you confess and youi paitnei doesn't, you will get the minimum
sentence of one yeai in jail on the possession and buiglaiy chaiges. If you both confess, youi sentence
will be thiee yeais in jail. If youi paitnei confesses and you do not, the plea baigain is of and you will
get six yeais in piison. If neithei of you confesses, you will each get two yeais in piison on the diug
chaige."
The two piisoneis each face a dilemma; they can choose to confess oi not confess. Because the
piisoneis aie sepaiated, they cannot plot a joint stiategy. Each must make a stiategic choice in
isolation.
The outcomes of these stiategic choices, as outlined by the DA, depend on the stiategic choice
made by the othei piisonei. The payof matiix foi this game is given in Figuie 11.6. The two iows iep-
iesent Fiankie's stiategic choices; she may confess oi not confess. The two columns iepiesent Johnny's
stiategic choices; he may confess oi not confess. Theie aie foui possible outcomes: Fiankie and Johnny
both confess (cell A), Fiankie confesses but Johnny does not (cell B), Fiankie does not confess but
Johnny does (cell C), and neithei Fiankie noi Johnny confesses (cell D). The poition at the lowei left in
each cell shows Fiankie's payof; the shaded poition at the uppei iight shows Johnny's payof.
286 PRINCIPLS OI CONOMICS
II GUR 11. 6 Payoff Matrix for the
Prisoners' DiIemma
|e |cu. ce||s .ep.esert e.c| c| t|e pcss|b|e
cutccnes c| t|e p.|scre.s g.ne.
dominant strategy
\|er . p|.ye.s best st..tegy
|s t|e s.ne .eg..J|ess c| t|e
.ct|cr c| t|e ct|e. p|.ye..
dominant strategy
equiIibrium
A g.ne |r .||c| t|e.e |s .
Jcn|r.rt st..tegy |c. e.c|
p|.ye..
If Johnny confesses, Fiankie's best choice is to confess-she will get a thiee-yeai
sentence iathei than the six-yeai sentence she would get if she did not confess. If
Johnny does not confess, Fiankie's best stiategy is still to confess-she will get a one-
yeai iathei than a two-yeai sentence. In this game, Fiankie's best stiategy is to confess,
iegaidless of what Johnny does. When a playei's best stiategy is the same iegaidless of
the action of the othei playei, that stiategy is said to be a dominant strategy.
Fiankie's dominant stiategy is to confess to the buiglaiy.
Foi Johnny, the best stiategy to follow, if Fiankie confesses, is to confess. The best
stiategy to follow if Fiankie does not confess is also to confess. Confessing is a domin-
ant stiategy foi Johnny as well. A game in which theie is a dominant stiategy foi each
playei is called a dominant strategy equilibrium. Heie, the dominant stiategy equi-
libiium is foi both piisoneis to confess; the payof will be given by cell A in the payof
matiix.
Fiom the point of view of the two piisoneis togethei, a payof in cell D would have
been piefeiable. Had they both denied paiticipation in the iobbeiy, theii combined
sentence would have been foui yeais in piison-two yeais each. Indeed, cell D ofeis
the lowest combined piison time of any of the outcomes in the payof matiix. But be-
cause the piisoneis cannot communicate, each is likely to make a stiategic choice that
iesults in a moie costly outcome. Of couise, the outcome of the game depends on the
way the payof matiix is stiuctuied.
Repeated OIigopoIy Games
The piisoneis' dilemma was played once, by two playeis. The playeis weie given a payof matiix; each
could make one choice, and the game ended aftei the fist iound of choices.
The ieal woild of oligopoly has as many playeis as theie aie fims in the industiy. They play iound
aftei iound: a fim iaises its piice, anothei fim intioduces a new pioduct, the fist fim cuts its piice, a
thiid fim intioduces a new maiketing stiategy, and so on. An oligopoly game is a bit like a baseball
game with an unlimited numbei of innings-one fim may come out ahead aftei one iound, but anoth-
ei will emeige on top anothei day. In the computei industiy game, the intioduction of peisonal com-
puteis changed the iules. IBM, which had won the mainfiame game quite handily, stiuggles to keep up
in a woild in which iivals continue to slash piices and impiove quality.
Oligopoly games may have moie than two playeis, so the games aie moie complex, but this does
not change theii basic stiuctuie. The fact that the games aie iepeated intioduces new stiategic consid-
eiations. A playei must considei not just the ways in which its choices will afect its iivals now, but how
its choices will afect them in the futuie as well.
We will keep the game simple, howevei, and considei a duopoly game. The two fims have col-
luded, eithei tacitly oi oveitly, to cieate a monopoly solution. As long as each playei upholds the agiee-
ment, the two fims will eain the maximum economic pioft possible in the enteipiise.
Theie will, howevei, be a poweiful incentive foi each fim to cheat. The monopoly solution may
geneiate the maximum economic pioft possible foi the two fims combined, but what if one fim cap-
tuies some of the othei fim's pioft: Suppose, foi example, that two equipment iental fims, Quick
Rent and Speedy Rent, opeiate in a community. Given the economies of scale in the business and the
size of the community, it is not likely that anothei fim will entei. Each fim has about half the maiket,
and they have agieed to chaige the piices that would be chosen if the two combined as a single fim.
Each eains economic piofts of $20,000 pei month.
Quick and Speedy could cheat on theii aiiangement in seveial ways. One of the fims could slash
piices, intioduce a new line of iental pioducts, oi launch an adveitising blitz. This appioach would not
be likely to inciease the total pioftability of the two fims, but if one fim could take the othei by sui-
piise, it might pioft at the expense of its iival, at least foi a while.
We will focus on the stiategy of cutting piices, which we will call a stiategy of cheating on the duo-
poly agieement. The alteinative is not to cheat on the agieement. Cheating incieases a fim's piofts if
its iival does not iespond. Figuie 11.7 shows the payof matiix facing the two fims at a paiticulai time.
As in the piisoneis' dilemma matiix, the foui cells list the payofs foi the two fims. If neithei fim
cheats (cell D), piofts iemain unchanged.
CHAP7R 11 7H WORLD OI IMPRIC7 COMP7I7ION 287
II GUR 11. 7 7o Cheat or Not to Cheat:
Game 7heory in OIigopoIy
.c .ert.| |.ns, u|ck |ert .rJ SpeeJy |ert,
cpe..te |r . Jucpc|y n..ket. |ey |.ve
cc||uJeJ |r t|e p.st, .c||ev|rg . ncrcpc|y
sc|ut|cr. utt|rg p.|ces ne.rs c|e.t|rg cr t|e
....rgenert, rct c|e.t|rg ne.rs n.|rt.|r|rg
cu..ert p.|ces. |e p.yc|s ..e c|.rges |r
ncrt||y p.c|ts, |r t|cus.rJs c| Jc||..s. ||
re|t|e. |.n c|e.ts, t|er re|t|e. |.ns p.c|ts
.||| c|.rge. |r t||s g.ne, c|e.t|rg |s .
Jcn|r.rt st..tegy e,u|||b.|un.
tit-for-tat strategy
S|tu.t|cr |r .||c| . |.n
.espcrJs tc c|e.t|rg by
c|e.t|rg, .rJ .espcrJs tc
cccpe..t|ve be|.v|c. by
cccpe..t|rg.
trigger strategy
S|tu.t|cr |r .||c| . |.n
n.kes c|e.. t|.t |t |s .||||rg
.rJ .b|e tc .espcrJ tc
c|e.t|rg by pe.n.rert|y
.evck|rg .r .g.eenert.
This game has a dominant stiategy equilibiium. Quick's piefeiied stiategy, iegaid-
less of what Speedy does, is to cheat. Speedy's best stiategy, iegaidless of what Quick
does, is to cheat. The iesult is that the two fims will select a stiategy that loweis theii
combined piofts!
Quick Rent and Speedy Rent face an unpleasant dilemma. They want to maximize
pioft, yet each is likely to choose a stiategy inconsistent with that goal. If they continue
the game as it now exists, each will continue to cut piices, eventually diiving piices
down to the point wheie piice equals aveiage total cost (piesumably, the piice-cutting
will stop theie). But that would leave the two fims with zeio economic piofts.
Both fims have an inteiest in maintaining the status quo of theii collusive agiee-
ment. Oveit collusion is one device thiough which the monopoly outcome may be
maintained, but that is illegal. One way foi the fims to encouiage each othei not to
cheat is to use a tit-foi-tat stiategy. In a tit-for-tat strategy a fim iesponds to cheat-
ing by cheating, and it iesponds to coopeiative behavioi by coopeiating. As each fim
leains that its iival will iespond to cheating by cheating, and to coopeiation by coopei-
ating, cheating on agieements becomes less and less likely.
Still anothei way fims may seek to foice iivals to behave coopeiatively iathei than
competitively is to use a trigger strategy, in which a fim makes cleai that it is willing
and able to iespond to cheating by peimanently ievoking an agieement. A fim might,
foi example, make a ciedible thieat to cut piices down to the level of aveiage total
cost-and leave them theie-in iesponse to any piice-cutting by a iival. A tiiggei
stiategy is calculated to impose huge costs on any fim that cheats-and on the fim
that thieatens to invoke the tiiggei. A fim might thieaten to invoke a tiiggei in hopes
that the thieat will foiestall any cheating by its iivals.
Game theoiy has pioved to be an enoimously fiuitful appioach to the analysis of a
wide iange of pioblems. Coipoiations use it to map out stiategies and to anticipate
iivals' iesponses. Goveinments use it in developing foieign-policy stiategies. Militaiy
leadeis play wai games on computeis using the basic ideas of game theoiy. Any situ-
ation in which iivals make stiategic choices to which competitois will iespond can be
assessed using game theoiy analysis.
One iathei chilly application of game theoiy analysis can be found in the peiiod of
the Cold Wai when the United States and the foimei Soviet Union maintained a nucleai weapons
policy that was desciibed by the acionym MAD, which stood foi mutually assuied destiuction. Both
countiies had enough nucleai weapons to destioy the othei seveial times ovei, and each thieatened to
launch sumcient nucleai weapons to destioy the othei countiy if the othei countiy launched a nucleai
attack against it oi any of its allies. On its face, the MAD doctiine seems, well, mad. It was, aftei all, a
commitment by each nation to iespond to any nucleai attack with a counteiattack that many scientists
expected would end human life on eaith. As ciazy as it seemed, howevei, it woiked. Foi 40 yeais, the
two nations did not go to wai. While the collapse of the Soviet Union in 1991 ended the need foi a
MAD doctiine, duiing the time that the two countiies weie iivals, MAD was a veiy efective tiiggei
indeed.
Of couise, the ending of the Cold Wai has not pioduced the ending of a nucleai thieat. Seveial na-
tions now have nucleai weapons. The thieat that Iian will intioduce nucleai weapons, given its stated
commitment to destioy the state of Isiael, suggests that the possibility of nucleai wai still haunts the
woild community.
k Y 7 A k A W A Y S
< |e key c|...cte.|st|cs c| c||gcpc|y ..e . .eccgr|t|cr t|.t t|e .ct|crs c| cre |.n .||| p.cJuce . .espcrse
|.cn .|v.|s .rJ t|.t t|ese .espcrses .||| .|ect |t. |.c| |.n |s urce.t.|r .|.t |ts .|v.|s .espcrses n|g|t be.
< |e Jeg.ee tc .||c| . |e. |.ns Jcn|r.te .r |rJust.y c.r be ne.su.eJ us|rg . ccrcert..t|cr ..t|c c. .
|e.|rJ.||||.sc|n.r |rJex.
< Ore ..y tc .vc|J t|e urce.t.|rty |.ns |.ce |r c||gcpc|y |s t|.cug| cc||us|cr. c||us|cr n.y be cve.t, .s |r
t|e c.se c| . c..te|, c. t.c|t, .s |r t|e c.se c| p.|ce |e.Je.s||p.
< C.ne t|ec.y |s . tcc| t|.t c.r be useJ tc urJe.st.rJ st..teg|c c|c|ces by |.ns.
< ||.ns c.r use t|t|c.t.t .rJ t.|gge. st..teg|es tc erccu..ge cccpe..t|ve be|.v|c. by .|v.|s.
288 PRINCIPLS OI CONOMICS
7 R Y I 7 !
\||c| ncJe| c| c||gcpc|y .cu|J seen tc be ncst .pp.cp.|.te |c. .r.|y.|rg |.ns be|.v|c. |r e.c| c| t|e
s|tu.t|crs g|ver be|c.`
1. \|er Scut| A|.||res |c.e.s |ts |..e bet.eer V|.n| .rJ |e. +c.k |ty, |c.t| A|.||res |c.e.s |ts |..e
bet.eer t|e t.c c|t|es. \|er Scut| A|.||res ..|ses |ts |..e, |c.t| A|.||res Jces tcc.
2. \|ereve. b.rk A ..|ses |rte.est ..tes cr c.. |c.rs, ct|e. b.rks |r t|e ..e. Jc tcc.
3. |r 1986, S.uJ| A..b|. |rtert|cr.||y |ccJeJ t|e n..ket .|t| c|| |r c.Je. tc pur|s| |e||c. O|| nenbe.s |c.
c|e.t|rg cr t|e|. p.cJuct|cr ,uct.s.
4. |r 'u|y 1998, S.uJ| A..b|. |c.teJ . p.cpcs.| |r .||c| . g.cup c| e|g|t c. r|re n.c. c||expc.t|rg
ccurt.|es (|rc|uJ|rg O|| nenbe.s .rJ scne rcrnenbe.s, suc| .s Vex|cc) .cu|J n.r.ge .c.|J c||
p.|ces by .Just|rg t|e|. p.cJuct|cr.
Case in Point: Memory Chip Makers Caught in GIobaI Price-Iixing Scheme
2010 jupiterimages Corporation
|t n.y |.ve beer t|e .en..k by .|. |.rg, v|ce p.es|Jert c| t|e .|..rb.seJ nenc.y c||p n.ru|.ctu.e.
Vcse|\|te||c t|.t sp..keJ t|e |rvest|g.t|cr by t|e |.S. |ep..tnert c| 'ust|ce Art|t.ust ||v|s|cr. V.. |.rg ..s
,ucteJ |r .|..rs oec|o| |e |r V.y 2002 .s .Jn|tt|rg tc p.|ce|x|rg neet|rgs |e|J |r As|. .ncrg
t|e n.c. p.cJuce.s c| ||AV, c. Jyr.n|c ..rJcn .ccess nenc.y. ||AV |s t|e ncst ccnncr sen|ccr
Juctc. n.|r nenc.y |c.n.t |c. stc..ge .rJ .et.|ev.| c| |r|c.n.t|cr t|.t |s useJ |r pe.scr.| ccnpute.s, nc
b||e p|cres, J|g|t.| c.ne..s, V|3 nus|c p|.ye.s, .rJ ct|e. e|ect.cr|cs p.cJucts. At t|cse neet|rgs, .s .e|| .s
t|.cug| en.||s .rJ te|ep|cre ccr|e.erces, t|e n.|r n.ru|.ctu.e.s c| ||AV Jec|JeJ rct cr|y .|.t p.|ces tc
c|..ge .rJ |c. nuc| tc n.ke .v.||.b|e, but .|sc exc|.rgeJ |r|c.n.t|cr cr ||AV s.|es |c. t|e pu.pcse c|
ncr|tc.|rg .rJ er|c.c|rg .J|e.erce tc t|e .g.eeJ p.|ces. |e cc||us|cr |.steJ |c. t|.ee ye..s|.cn 1999 tc
2002. |r |ecenbe. 2001, ||AV p.|ces .e.e |ess t|.r 1.00. by V.y c| 2002, p.|ce |.J .|ser tc t|e 4 tc 5
..rge.
|e ccnp.r|es t|.t .e.e J|.ect|y |ru.eJ by t|e ||g|e. c||p p.|ces |rc|uJeJ |e||, cnp.,, |e.|ett|.ck..J,
App|e, |bV, .rJ C.te..y. |r t|e erJ, t|cug|, t|e pu.c|.se.s c| t|e|. p.cJucts p.|J |r t|e |c.n c| ||g|e. p.|ces
c. |ess nenc.y.
|r |ecenbe. 2003, . V|c.cr ec|rc|cgy s.|es n.r.ge. p|eJ gu||ty tc cbst.uct|cr c| ust|ce .rJ se.veJ s|x
ncrt|s c| |cne Jetert|cr. |e |.st c||pn.ke. tc p|e.J gu||ty . ye.. |.te. ..s Ce.n.ryb.seJ |r|recr ec|
rc|cg|es, .||c| ..s |reJ 160 n||||cr. As c| Septenbe. 200, |ve ccnp.r|es, S.nsurg be|rg t|e |..gest,
|.J beer c|..geJ |res c| nc.e t|.r 32 n||||cr, .rJ cve. 3,000 J.ys c| .|| t|ne |.J beer neteJ cut tc
e|g|teer cc.pc..te execut|ves.
CHAP7R 11 7H WORLD OI IMPRIC7 COMP7I7ION 289
|e s|..p .eJuct|cr |r t|e runbe. c| ||AV n.ke.s |r t|e |.te 1990s urJcubteJ|y n.Je |t e.s|e. tc cc||uJe.
|e |rJust.y |s st||| ,u|te ccrcert..teJ .|t| S.nsurg |c|J|rg 2. c| t|e n..ket .rJ |yr|x 21.3. |e p.|ce,
|c.eve., |.s |.||er ,u|te s|..p|y |r .ecert ye..s.
.ooce |eo|e| o| .o||ce. .||| .oo |eco||.e ^ee |o ||eoJ Oo|||, |o |o||c|o|| | ||^/ ||ce||| o|e|.' |e |e|eoe ^|| 9.
200.. .|e|e |oo|o. ||eo o |o, o ||e | ||e ||| o| || ||ce.' |e |e. `o| |e. .e|ee 6. 2004. Oeoe |eoo|J oJ |o.|J
|oe. ||^/ |Je Oo | ^|||o| |oe'. ||ec|o|c ||ee| |e. 24 (.oe 24. 2002:. 02. |ee .o`oo. .oo ee| ||^/
|eoJe||.' |oeo |eo|J. o||e. /oc| 3. 200S
A N S W R S 7 O 7 R Y I 7 ! P R O 8 L M S
1. |c.t| A|.||res seens tc be p..ct|c|rg . p.|ce st..tegy krc.r |r g.ne t|ec.y .s t|t|c.t.t.
2. |e b.rks ccu|J be erg.geJ |r t.c|t cc||us|cr, .|t| b.rk A .s t|e p.|ce |e.Je..
3. S.uJ| A..b|. .ppe..s tc |.ve useJ . t.|gge. st..tegy, .rct|e. .spect c| g.ne t|ec.y. |r gere..|, c| ccu.se,
p..t|c|p.rts |cpe t|ey .||| reve. |.ve tc pu|| t|e t.|gge., bec.use Jc|rg sc |..ns .|| p..t|c|p.rts. A|te.
ye..s c| c|e.t|rg by ct|e. O|| nenbe.s, S.uJ| A..b|. J|J urJe.t.ke . pc||cy t|.t |u.t .|| nenbe.s c|
O||, |rc|uJ|rg |tse||, O|| |.s reve. s|rce .eg.|reJ t|e p.cn|rert .c|e |t p|.yeJ |r c|| n..kets.
4. S.uJ| A..b|. seens tc be t.y|rg tc c.e.te .rct|e. c|| c..te|, . |c.n c| cve.t cc||us|cr.
3. EXTENSIONS OF IMPERFECT COMPETITION:
ADVERTISING AND PRICE DISCRIMINATION
L A R N I N G O 8 1 C 7 I V S
1. Discuss the possibIe eects of advertising on competition, price, and output.
2. Dene price discrimination, Iist the conditions that make it possibIe, and expIain the reIation-
ship between the price charged and price eIasticity of demand.
The models of monopoly and of impeifectly competitive maikets allow us to explain two commonly
obseived featuies of many maikets: adveitising and piice disciimination. Fiims in maikets that aie not
peifectly competitive tiy to infuence the positions of the demand cuives they face, and hence piofts,
thiough adveitising. Piofts may also be enhanced by chaiging difeient customeis difeient piices. In
this section we will discuss these aspects of the behavioi of fims in maikets that aie not peifectly
competitive.
3.1 Adveitising
Fiims in monopoly, monopolistic competition, and oligopoly use adveitising when they expect it to in-
ciease theii piofts. We see the iesults of these expendituies in a daily baiiage of adveitising on televi-
sion, iadio, newspapeis, magazines, billboaids, passing buses, paik benches, the mail, home telephones,
and the ubiquitous pop-up adveitisements on oui computeis-in viitually eveiy medium imaginable.
Is all this adveitising good foi the economy:
We have alieady seen that a peifectly competitive economy with fully defned and easily tiansfei-
able piopeity iights will achieve an emcient allocation of iesouices. Theie is no iole foi adveitising in
such an economy, because eveiyone knows that fims in each industiy pioduce identical pioducts. Fui-
theimoie, buyeis alieady have complete infoimation about the alteinatives available to them in the
maiket.
But peifect competition contiasts shaiply with impeifect competition. Impeifect competition can
lead to a piice gieatei than maiginal cost and thus geneiate an inemcient allocation of iesouices. Fiims
in an impeifectly competitive maiket may adveitise heavily. Does adveitising cause inemciency, oi is it
pait of the solution: Does adveitising insulate impeifectly competitive fims fiom competition and al-
low them to iaise theii piices even highei, oi does it encouiage gieatei competition and push piices
down:
Theie aie two ways in which adveitising could lead to highei piices foi consumeis. Fiist, the ad-
veitising itself is costly; in 2007, fims in the United States spent about $149 billion on adveitising. By
290 PRINCIPLS OI CONOMICS
pushing up pioduction costs, adveitising may push up piices. If the adveitising seives no socially useful
puipose, these costs iepiesent a waste of iesouices in the economy. Second, fims may be able to use
adveitising to manipulate demand and cieate baiiieis to entiy. If a few fims in a paiticulai maiket
have developed intense biand loyalty, it may be dimcult foi new fims to entei-the adveitising cieates
a kind of baiiiei to entiy. By maintaining baiiieis to entiy, fims may be able to sustain high piices.
But adveitising has its defendeis. They aigue that adveitising piovides consumeis with useful in-
foimation and encouiages piice competition. Without adveitising, these defendeis aigue, it would be
impossible foi new fims to entei an industiy. Adveitising, they say, piomotes competition, loweis
piices, and encouiages a gieatei iange of choice foi consumeis.
Adveitising, like all othei economic phenomena, has benefts as well as costs. To assess those be-
nefts and costs, let us examine the impact of adveitising on the economy.
Advertising and Information
Adveitising does infoim us about pioducts and theii piices. Even ciitics of adveitising geneially agiee
that when adveitising advises consumeis about the availability of new pioducts, oi when it piovides
piice infoimation, it seives a useful function. But much of the infoimation piovided by adveitising ap-
peais to be of limited value. Heaiing that Pepsi is the iight one, baby" oi Tide gets youi clothes
whitei than white" may not be among the most edifying lessons consumeis could leain.
Some economists aigue, howevei, that even adveitising that seems to tell us nothing may piovide
useful infoimation. They note that a consumei is unlikely to make a iepeat puichase of a pioduct that
tuins out to be a dud. Adveitising an infeiioi pioduct is likely to have little payof; people who do tiy it
aie not likely to tiy it again. It is not likely a fim could pioft by going to gieat expense to launch a
pioduct that pioduced only unhappy consumeis. Thus, if a pioduct is heavily adveitised, its pioducei
is likely to be confdent that many consumeis will be satisfed with it and make iepeat puichases. If this
is the case, then the fact that the pioduct is adveitised, iegaidless of the content of that adveitising, sig-
nals consumeis that at least its pioducei is confdent that the pioduct will satisfy them.
Advertising and Competition
If adveitising cieates consumei loyalty to a paiticulai biand, then that loyalty may seive as a baiiiei to
entiy to othei fims. Some biands of household pioducts, such as laundiy deteigents, aie so well estab-
lished they may make it dimcult foi othei fims to entei the maiket.
In geneial, theie is a positive ielationship between the degiee of concentiation of maiket powei
and the fiaction of total costs devoted to adveitising. This ielationship, ciitics aigue, is a causal one; the
high expendituies on adveitising aie the cause of the concentiation. To the extent that adveitising in-
cieases industiy concentiation, it is likely to iesult in highei piices to consumeis and lowei levels of
output. The highei piices associated with adveitising aie not simply the iesult of passing on the cost of
the adveitising itself to consumeis; highei piices also deiive fiom the monopoly powei the adveitising
cieates.
But adveitising may encouiage competition as well. By pioviding infoimation to consumeis about
piices, foi example, it may encouiage piice competition. Suppose a fim in a woild of no adveitising
wants to inciease its sales. One way to do that is to lowei piice. But without adveitising, it is extiemely
dimcult to infoim potential customeis of this new policy. The likely iesult is that theie would be little
iesponse, and the piice expeiiment would piobably fail. Piice competition would thus be discouiaged
in a woild without adveitising.
Empiiical studies of maikets in which adveitising is not allowed have confimed that adveitising
encouiages piice competition. One of the most famous studies of the efects of adveitising looked at
piicing foi piesciiption eyeglasses. In the eaily 1970s, about half the states in the United States banned
adveitising by fims making piesciiption eyeglasses; the othei half allowed it. A compaiison of piices in
the two gioups of states by economist Lee Benham showed that the cost of piesciiption eyeglasses was
fai lowei in states that allowed adveitising than in states that banned it.
[4]
Mi. Benham's ieseaich
pioved quite infuential-viitually all states have since ievoked theii bans on such adveitising. Simil-
aily, a study of the cigaiette industiy ievealed that befoie the 1970 ban on iadio and television adveit-
ising maiket shaies of the leading cigaiette manufactuieis had been declining, while aftei the ban mai-
ket shaies and pioft maigins incieased.
[3]
Adveitising may also allow moie entiy by new fims. When Kia, a South Koiean automobile man-
ufactuiei, enteied the U.S. low-cost compact cai maiket in 1994, it fooded the aiiwaves with adveit-
ising. Suppose such adveitising had not been possible. Could Kia have enteied the maiket in the Un-
ited States: It seems highly unlikely that any new pioduct could be launched without adveitising. The
absence of adveitising would thus be a baiiiei to entiy that would inciease the degiee of monopoly
powei in the economy. A gieatei degiee of monopoly powei would, ovei time, tianslate into highei
piices and ieduced output.
CHAP7R 11 7H WORLD OI IMPRIC7 COMP7I7ION 291
price discrimination
S|tu.t|cr |r .||c| . |.n
c|..ges J||e.ert p.|ces |c.
t|e s.ne gccJ c. se.v|ce tc
J||e.ert ccrsune.s, ever
t|cug| t|e.e |s rc J||e.erce
|r t|e ccst tc t|e |.n c|
supp|y|rg t|ese ccrsune.s.
Adveitising is thus a two-edged swoid. On the one hand, the existence of established and heavily
adveitised iivals may make it dimcult foi a new fim to entei a maiket. On the othei hand, entiy into
most industiies would be viitually impossible without adveitising.
Economists do not agiee on whethei adveitising helps oi huits competition in paiticulai maikets,
but one geneial obseivation can safely be made-a woild with adveitising is moie competitive than a
woild without adveitising would be. The impoitant policy question is moie limited-and moie dim-
cult to answei: Would a woild with less adveitising be moie competitive than a woild with moie:
3.2 Piice Disciimination
Thioughout the text up to this point, we have assumed that fims sold all units of output at the same
piice. In some cases, howevei, fims can chaige difeient piices to difeient consumeis. If such an op-
poitunity exists, the fim can inciease piofts fuithei.
When a fim chaiges difeient piices foi the same good oi seivice to difeient consumeis, even
though theie is no difeience in the cost to the fim of supplying these consumeis, the fim is engaging
in price discrimination. Except foi a few situations of piice disciimination that have been declaied
illegal, such as manufactuieis selling theii goods to distiibutois at difeient piices when theie aie no
difeiences in cost, piice disciimination is geneially legal.
The potential foi piice disciimination exists in all maiket stiuctuies except peifect competition. As
long as a fim faces a downwaid-sloping demand cuive and thus has some degiee of monopoly powei,
it may be able to engage in piice disciimination. But monopoly powei alone is not enough to allow a
fim to piice disciiminate. Monopoly powei is one of thiee conditions that must be met:
1. A Price-Setting Firm The fim must have some degiee of monopoly powei-it must be a piice
settei. A piice-taking fim can only take the maiket piice as given-it is not in a position to make
piice choices of any kind. Thus, fims in peifectly competitive maikets will not engage in piice
disciimination. Fiims in monopoly, monopolistically competitive, oi oligopolistic maikets may
engage in piice disciimination.
2. Distinguishable Customers The maiket must be capable of being faiily easily
segmented-sepaiated so that customeis with difeient elasticities of demand can be identifed
and tieated difeiently.
3. Prevention of Resale The vaiious maiket segments must be isolated in some way fiom one
anothei to pievent customeis who aie ofeied a lowei piice fiom selling to customeis who aie
chaiged a highei piice. If consumeis can easily iesell a pioduct, then disciimination is unlikely to
be successful. Resale may be paiticulaily dimcult foi ceitain seivices, such as dental checkups.
Examples of piice disciimination abound. Senioi citizens and students aie often ofeied discount faies
on city buses. Childien ieceive discount piices foi movie theatei tickets and entiance fees at zoos and
theme paiks. Faculty and staf at colleges and univeisities might ieceive discounts at the campus book-
stoie. Aiilines give discount piices to customeis who aie willing to stay ovei a Satuiday night. Physi-
cians might chaige wealthy patients moie than pooi ones. People who save coupons aie able to get dis-
counts on many items. In all these cases a fim chaiges difeient piices to difeient customeis foi what
is essentially the same pioduct.
Not eveiy instance of fims chaiging difeient piices to difeient customeis constitutes piice dis-
ciimination. Difeiences in piices may iefect difeient costs associated with pioviding the pioduct.
One buyei might iequiie special billing piactices, anothei might iequiie deliveiy on a paiticulai day of
the week, and yet anothei might iequiie special packaging. Piice difeientials based on difeiences in
pioduction costs aie not examples of piice disciimination.
Why would a fim chaige difeient piices to difeient consumeis: The answei can be found in the
maiginal decision iule and in the ielationship between maiginal ievenue and elasticity.
Suppose an aiiline has found that its long-iun pioft-maximizing solution foi a iound-tiip fight
between Minneapolis and Cleveland, when it chaiges the same piice to all passengeis, is to caiiy 300
passengeis at $200 pei ticket. The aiiline has a degiee of monopoly powei, so it faces a downwaid-slop-
ing demand cuive. The aiiline has noticed that theie aie essentially two gioups of customeis on each
fight: people who aie tiaveling foi business ieasons and people who aie tiaveling foi peisonal ieasons
(visiting family oi fiiends oi taking a vacation). We will call this lattei gioup touiists." Of the 300 pas-
sengeis, 200 aie business tiaveleis and 100 aie touiists. The aiiline's ievenue fiom business tiaveleis is
theiefoie cuiiently $40,000 ($200 times 200 business tiaveleis) and fiom touiists is cuiiently $20,000
($200 times 100 touiists).
It seems likely that the piice elasticities of demand of these two gioups foi a paiticulai fight will
difei. Touiists may have a wide iange of substitutes: They could take theii tiips at a difeient time, they
could vacation in a difeient aiea, oi they could easily choose not to go at all. Business tiaveleis,
howevei, might be attending meetings oi confeiences at a paiticulai time and in a paiticulai city. They
have options, of couise, but the iange of options is likely to be moie limited than the iange of options
292 PRINCIPLS OI CONOMICS
facing touiists. Given all this, touiists aie likely to have ielatively moie piice elastic demand than busi-
ness tiaveleis foi a paiticulai fight.
The difeience in piice elasticities suggests the aiiline could inciease its pioft by adjusting its pii-
cing. To simplify, suppose that at a piice of about $200 pei ticket, demand by touiists is ielatively piice
elastic and by business tiaveleis is ielatively less piice elastic. It is plausible that the maiginal cost of ad-
ditional passengeis is likely to be quite low, since the numbei of ciewmembeis will not vaiy and no
food is seived on shoit fights. Thus, if the aiiline can inciease its ievenue, its piofts will inciease. Sup-
pose the aiiline loweis the piice foi touiists to $190. Suppose that the lowei piice encouiages 10 moie
touiists to take the fight. Of couise, the aiiline cannot chaige difeient piices to difeient touiists;
iathei it chaiges $190 to all, now 110, touiists. Still, the aiiline's ievenue fiom touiist passengeis in-
cieases fiom $20,000 to $20,900 ($190 times 110 touiists). Suppose it chaiges $230 to its business tiav-
eleis. As a iesult, only 193 business tiaveleis take the fight. The aiiline's ievenue fiom business tiavel-
eis still iises fiom $40,000 to $48,730 ($230 times 193 business tiaveleis). The aiiline will continue to
change the mix of passengeis, and inciease the numbei of passengeis, so long as doing so incieases its
pioft. Because touiist demand is ielatively piice elastic, ielatively small ieductions in piice will attiact
ielatively laige numbeis of additional touiists. Because business demand is ielatively less elastic, ielat-
ively laige incieases in piice will discouiage ielatively small numbeis of business tiaveleis fiom making
the tiip. The aiiline will continue to ieduce its piice to touiists and iaise its piice to business tiaveleis
as long as it gains pioft fiom doing so.
Of couise, the aiiline can impose a disciiminatoiy faie stiuctuie only if it can distinguish touiists
fiom business tiaveleis. Aiilines typically do this by looking at the tiavel plans of theii customeis. Tiips
that involve a stay ovei a weekend, foi example, aie moie likely to be touiist ielated, wheieas tiips that
begin and end duiing the woikweek aie likely to be business tiips. Thus, aiilines chaige much lowei
faies foi tiips that extend thiough a weekend than foi tiips that begin and end on weekdays.
In geneial, piice-disciimination stiategies aie based on difeiences in piice elasticity of demand
among gioups of customeis and the difeiences in maiginal ievenue that iesult. A fim will seek a piice
stiuctuie that ofeis customeis with moie elastic demand a lowei piice and ofeis customeis with ielat-
ively less elastic demand a highei piice.
It is always in the inteiest of a fim to disciiminate. Yet most of the goods and seivices that we buy
aie not ofeied on a disciiminatoiy basis. A gioceiy stoie does not chaige a highei piice foi vegetables
to vegetaiians, whose demand is likely to be less elastic than that of its omnivoious customeis. An au-
dio stoie does not chaige a difeient piice foi Peail Jam's compact disks to collectois seeking a com-
plete collection than it chaiges to casual fans who could easily substitute a disk fiom anothei pei-
foimei. In these cases, fims lack a mechanism foi knowing the difeient demands of theii customeis
and foi pieventing iesale.
k Y 7 A k A W A Y S
< || .Jve.t|s|rg .eJuces ccnpet|t|cr, |t terJs tc ..|se p.|ces .rJ .eJuce ,u.rt|t|es p.cJuceJ. || |t er|.rces
ccnpet|t|cr, |t terJs tc |c.e. p.|ces .rJ |rc.e.se ,u.rt|t|es p.cJuceJ.
< |r c.Je. tc erg.ge |r p.|ce J|sc.|n|r.t|cr, . |.n nust be . p.|ce sette., nust be .b|e tc |Jert||y
ccrsune.s .|cse e|.st|c|t|es J||e., .rJ nust be .b|e tc p.evert .es.|e c| t|e gccJ c. se.v|ce .ncrg
ccrsune.s.
< |e p.|ceJ|sc.|n|r.t|rg |.n .||| .Just |ts p.|ces sc t|.t custcne.s .|t| nc.e e|.st|c Jen.rJ p.y |c.e.
p.|ces t|.r custcne.s .|t| |ess e|.st|c Jen.rJ.
7 R Y I 7 !
|xp|.|r .|y p.|ce J|sc.|n|r.t|cr |s c|ter |curJ |r e.c| c| t|e |c||c.|rg sett|rgs. |ces |t n.ke serse |r te.ns
c| p.|ce e|.st|c|ty c| Jen.rJ`
1. Ser|c. c|t|.er J|sccurts |c. t..ve|
2. |ccJ sc|J c|e.pe. || t|e custcne. |.s . ccupcr |c. t|e |ten
3. c||ege sc|c|..s||ps tc stuJerts .|t| t|e best .c.Jen|c .ecc.Js c. tc stuJerts .|t| spec|.| .t||et|c,
nus|c.|, c. ct|e. sk|||s
CHAP7R 11 7H WORLD OI IMPRIC7 COMP7I7ION 293
Case in Point: Pricing Costa Rica's NationaI Parks
2010 jupiterimages Corporation
cst. ||c. bc.sts scne c| t|e ncst be.ut||u| r.t|cr.| p..ks |r t|e .c.|J. Ar .r.|ys|s by |..rc|scc A|p|..., .r
eccrcn|st .|t| Cct|erbu.g |r|ve.s|ty |r S.eJer .rJ A||, . t.cp|c.| .ese..c| |rst|tute |r cst. ||c., sug
gests t|.t cst. ||c. s|cu|J |rc.e.se t|e Jeg.ee tc .||c| |t erg.ges |r p.|ce J|sc.|n|r.t|cr |r p.|c|rg |ts r.
t|cr.| p..ks.
|e ccurt.y |.s expe.|nerteJ .|t| . .|Je ..rge c| p.|ces |c. |ts r.t|cr.| p..ks, .|t| t|e p.|ce v..y|rg
bet.eer .80 .rJ 15 |c. . J.||y v|s|t. \|t| J.t. cr t|e .esu|t.rt runbe. c| v|s|tc.s .t e.c| p.|ce, |.c|essc.
A|p|... ..s .b|e tc est|n.te t|e Jen.rJ cu.ve, ccnpute t|e p.|ce e|.st|c|ty c| Jen.rJ, .rJ Jeve|cp . .eccn
nerJ.t|cr |c. p.|c|rg t|e ccurt.ys r.t|cr.| p..ks.
|.esun.b|y, |c.e|gr v|s|tc.s |.ve . .e|.t|ve|y |ess e|.st|c Jen.rJ |c. v|s|t|rg t|e p..ks t|.r Jc |cc.| c|t|.ers.
|cc.| c|t|.ers |.ve bette. krc.|eJge c| subst|tutes |c. t|e p..ksr.ne|y ct|e. ..e.s |r cst. ||c.. ArJ, c|
ccu.se, crce |c.e|gr t..ve|e.s ..e |r t|e ccurt.y, t|ey |.ve .|.e.Jy ccnn|tteJ t|e experse c| gett|rg t|e.e,
.rJ ..e |ess ||ke|y tc be .||||rg tc p.ss up . v|s|t tc r.t|cr.| p..ks b.seJ cr p.|c|rg ccrs|Je..t|crs.
cst. ||c. .|.e.Jy J|sc.|n|r.tes tc . |..ge Jeg.ee. |c.e|gre.s ..e c|..geJ pe. J.y tc v|s|t t|e p..ks, |cc.|s
..e c|..geJ 2. |.c|essc. A|p|... p.cpcses |rc.e.s|rg t|e Jeg.ee c| J|sc.|n|r.t|cr.
|e est|n.tes t|.t t|e p.|ce e|.st|c|ty c| |c.e|gr Jen.rJ |c. v|s|ts tc cst. ||c.s r.t|cr.| p..ks |s 0.68. |.t, c|
ccu.se, suggests |re|.st|c Jen.rJ. cst. ||c. ccu|J |rc.e.se |ts .everue |.cn |c.e|gr v|s|tc.s by |rc.e.s|rg t|e
|ee. |.c|essc. A|p|... p.cpcses |rc.e.s|rg t|e |ee |c. |c.e|gre.s tc 10. |e p.cpcses t|.t t|e p.|ce c|..geJ tc
cst. ||c.rs .en.|r .t 2. p.|ce t|.t |e c.|cu|.tes e,u.|s t|e n..g|r.| ccst c| .r .JJ|t|cr.| v|s|t.
|.c|essc. A|p|... c.|cu|.tes . |ee c| 10 pe. v|s|t by . |c.e|gre. .cu|J nc.e t|.r p.y t|e ccurt.ys |xeJ ccst c|
n.|rt.|r|rg |ts exters|ve p..k systen, .||c| ut|||.es 24 c| t|e ccurt.ys |.rJ. |e ||g|e. p.|ce .cu|J t|us .|
|c. t|e gcve.rnert tc neet t|e n.c. ccsts c| cpe..t|rg t|e r.t|cr.| p..ks. |..g|rg . 2 |ee tc |cc.|s
.cu|J s.t|s|y t|e e|c|ercy .e,u|.enert t|.t p.|ce e,u.| n..g|r.| ccst |c. |cc.| v|s|tc.s, t|e 10 |ee tc |c.e|gr
e.s .cu|J pe.n|t t|e ccurt.y tc exp|c|t |ts ncrcpc|y pc.e. |r pe.n|tt|rg pecp|e tc v|s|t t|e p..ks. |e cst.
||c.r gcve.rnert |.s .skeJ |.c|essc. A|p|... tc Jes|gr t|.ee p||ct p.cects .|neJ .t |rcc.pc..t|rg ||s p.c
pcs.| tc ..|se p..k |ees tc |c.e|gr v|s|tc.s.
.ooce |oc|co ^||o. |e ||c| o| |o|ec|eJ ^eo | |o|oe|oeJ oo| ^ |oco| |oec||.e.' |co|o|co| |coo|c. 56(2: (|eoo, 2006:
29430. oJ eoo| coeoJece .||| |o|eo ^||o
294 PRINCIPLS OI CONOMICS
A N S W R S 7 O 7 R Y I 7 ! P R O 8 L M S
1. Ser|c. c|t|.ers ..e (usu.||y!) e.sy tc |Jert||y, .rJ |c. t..ve|, p.evert|rg .es.|e |s usu.||y ,u|te e.sy .s .e||.
|c. ex.np|e, . p|ctu.e || |s .e,u|.eJ tc bc..J .r .|.p|.re. A|.||res n|g|t be expecteJ tc cppcse
|np|enert|rg t|e .u|e s|rce |t |s ccst|y |c. t|en. |e |.ct t|.t t|ey suppc.t t|e .u|e c.r be exp|.|reJ by
|c. |t .|Js t|en |r p..ct|c|rg p.|ce J|sc.|n|r.t|cr, by p.evert|rg t|e .es.|e c| J|sccurt t|ckets, .||c|
rc. c.r e.s||y be n.tc|eJ tc t|e pu.c|.s|rg custcne.s. |e Jen.rJ |c. .|. t..ve| by ser|c. c|t|.ers |s
||ke|y tc be nc.e e|.st|c t|.r |t |s |c. ct|e. p.sserge.s, espec|.||y bus|ress t..ve|e.s, s|rce t|e pu.pcse c|
t|e|. t..ve| |s |..ge|y J|sc.et|cr..y (c|ter tcu.|st|c |r r.tu.e) .rJ s|rce t|e|. t|ne |s ||ke|y tc be |ess ccst|y,
n.k|rg t|en nc.e .||||rg tc seek cut |r|c.n.t|cr cr t..ve| .|te.r.t|ves t|.r t|e .est c| t|e pcpu|.t|cr.
2. S|rce t|e custcne. nust p.esert t|e ccupcr .t t|e pc|rt c| s.|e, |Jert||c.t|cr |s e.sy. \||||rgress tc
se..c| |c. .rJ cut cut ccupcrs suggests . ||g| Jeg.ee c| p.|ce ccrsc|cusress .rJ t|us . g.e.te. p.|ce
e|.st|c|ty c| Jen.rJ.
3. Suc| stuJerts ..e ||ke|y tc |.ve nc.e c|c|ces c| .|e.e tc .tterJ cc||ege. As .e |e..reJ |r .r e..||e.
c|.pte. cr e|.st|c|ty, Jen.rJ |s ||ke|y tc be nc.e e|.st|c .|er subst|tutes ..e .v.||.b|e |c. |t. |r.c||nert
p.cceJu.es n.ke |Jert||c.t|cr .rJ p.evert|cr c| .es.|e ve.y e.sy.
4. REVIEW AND PRACTICE
Summary
||s c|.pte. ex.n|reJ t|e .c.|J c| |npe.|ect ccnpet|t|cr t|.t ex|sts bet.eer t|e |Je.||.eJ ext.enes c| pe.
|ect ccnpet|t|cr .rJ ncrcpc|y. |npe.|ect|y ccnpet|t|ve n..kets ex|st .|ereve. t|e.e |s nc.e t|.r cre se||e.
|r . n..ket .rJ .t |e.st cre se||e. |.s scne Jeg.ee c| ccrt.c| cve. p.|ce.
\e J|scusseJ t.c gere..| types c| |npe.|ect|y ccnpet|t|ve n..kets. ncrcpc||st|c ccnpet|t|cr .rJ c||gcpc|y.
Vcrcpc||st|c ccnpet|t|cr |s c|...cte.|.eJ by n.ry |.ns p.cJuc|rg s|n||.. but J||e.ert|.teJ gccJs .rJ se.
v|ces |r . n..ket .|t| e.sy ert.y .rJ ex|t. O||gcpc|y |s c|...cte.|.eJ by .e|.t|ve|y |e. |.ns p.cJuc|rg e|t|e.
st.rJ..J|.eJ c. J||e.ert|.teJ p.cJucts. |e.e n.y be subst.rt|.| b...|e.s tc ert.y .rJ ex|t.
|r t|e s|c.t .ur, . ncrcpc||st|c.||y ccnpet|t|ve |.ns p.|c|rg .rJ cutput Jec|s|crs ..e t|e s.ne .s t|cse c| .
ncrcpc|y. |r t|e |crg .ur, eccrcn|c p.c|ts .||| be .||tt|eJ ...y by t|e ert.y c| re. |.ns .rJ re.
p.cJucts t|.t |rc.e.se t|e runbe. c| c|cse subst|tutes. Ar |rJust.y Jcn|r.teJ by . |e. |.ns |s .r c||gcpc|y.
|.c| c||gcpc||st |s ....e c| |ts |rte.JeperJerce .|t| ct|e. |.ns |r t|e |rJust.y .rJ |s ccrst.rt|y ....e c| t|e
be|.v|c. c| |ts .|v.|s. O||gcpc||sts erg.ge |r st..teg|c Jec|s|cr n.k|rg |r c.Je. tc Jete.n|re t|e|. best cutput
.rJ p.|c|rg st..teg|es .s .e|| .s t|e best |c.ns c| rcrp.|ce ccnpet|t|cr.
AJve.t|s|rg |r |npe.|ect|y ccnpet|t|ve n..kets c.r |rc.e.se t|e Jeg.ee c| ccnpet|t|veress by erccu..g|rg
p.|ce ccnpet|t|cr .rJ p.cnct|rg ert.y. |t c.r .|sc Jec.e.se ccnpet|t|cr by est.b||s||rg b..rJ |cy.|ty .rJ
t|us c.e.t|rg b...|e.s tc ert.y.
\|e.e ccrJ|t|crs pe.n|t, . |.n c.r |rc.e.se |ts p.c|ts by p.|ce J|sc.|n|r.t|cr, c|..g|rg J||e.ert p.|ces tc
custcne.s .|t| J||e.ert e|.st|c|t|es c| Jen.rJ. c p..ct|ce p.|ce J|sc.|n|r.t|cr, . p.|cesett|rg |.n nust be
.b|e tc segnert custcne.s t|.t |.ve J||e.ert e|.st|c|t|es c| Jen.rJ .rJ nust be .b|e tc p.evert .es.|e
.ncrg |ts custcne.s.
CHAP7R 11 7H WORLD OI IMPRIC7 COMP7I7ION 295
C O N C P 7 P R O 8 L M S
1. \|.t ..e t|e n.c. J|st|rct|crs bet.eer . ncrcpc||st|c.||y ccnpet|t|ve |rJust.y .rJ .r c||gcpc||st|c
|rJust.y`
2. \|.t |s t|e J||e.erce bet.eer . p.|ce t.ke. .rJ . p.|ce sette.` \||c| Jc ycu t||rk . |.n .cu|J p.e|e. tc
be` \|y`
3. |r t|e ncJe| c| ncrcpc||st|c ccnpet|t|cr, .e s.y t|.t t|e.e |s p.cJuct J||e.ert|.t|cr. \|.t Jces t||s
ne.r, .rJ |c. Jces |t J||e. |.cn t|e .ssunpt|cr c| |cncgerecus gccJs |r pe.|ect ccnpet|t|cr`
4. |r t|e |c||c.|rg ||st c| gccJs .rJ se.v|ces, Jete.n|re .|et|e. t|e |ten |s p.cJuceJ urJe. ccrJ|t|crs c|
ncrcpc||st|c ccnpet|t|cr c. c| c||gcpc|y.
.. sc|t J.|rks
b. exe.c|se J.|rks
c. c|ce supp|y stc.es
J. n.ss.ge t|e..p|sts
e. .cccurt.rts
|. cc||eges .rJ ur|ve.s|t|es
g. .st.c|cg|sts
5. Suppcse . c|ty expe.|erces subst.rt|.| pcpu|.t|cr g.c.t|. \|.t |s ||ke|y tc |.pper tc p.c|ts |r t|e s|c.t
.ur .rJ |r t|e |crg .ur |r t|e n..ket |c. |.|.cuts, . ncrcpc||st|c.||y ccnpet|t|ve n..ket`
6. Scne p.c|essc.s g..Je stuJerts cr t|e b.s|s c| .r .bsc|ute pe.cert.ge c| t|e ||g|est scc.e e..reJ cr
e.c| test g|ver Ju.|rg t|e seneste.. A|| stuJerts .|c get .|t||r . ce.t.|r pe.cert.ge c| t|e ||g|est scc.e
e..reJ get .r A. \|y Jc t|ese p.c|essc.s rct .c..y t|.t t|e stuJerts .||| get tcget|e. .rJ cc||uJe |r
suc| . ..y .s tc keep t|e ||g| scc.e |r t|e c|.ss e,u.| tc . ve.y |c. tct.|`
. +cu. p..erts p.cb.b|y tc|J ycu tc .vc|J t|t|c.t.t be|.v|c.. \|y Jces |t n.ke serse |c. |.ns tc Jc |t`
8. \|.t ncJe| c| c||gcpc|y be|.v|c. .e.e t|e ||AV p.cJuce.s J|scusseJ |r t|e .se |r |c|rt |c||c.|rg`
|c. n|g|t t|e ||AV p.cJuce.s |.ve .c||eveJ t|e|. gc.| .rJ st||| st.yeJ .|t||r t|e |..`
9. |xp|.|r .|y . p.|ce |rc.e.se |c. |c.e|gre.s .cu|J |rc.e.se cst. ||c.s tct.| .everue .rJ p.c|ts |.cn
cpe..t|rg |ts r.t|cr.| p..k systen.
10. |est.u..rts typ|c.||y c|..ge nuc| ||g|e. p.|ces |c. J|rre. t|.r |c. |urc|, Jesp|te t|e |.ct t|.t t|e ccst c|
se.v|rg t|ese ne.|s |s .bcut t|e s.ne. \|y Jc ycu t||rk t||s |s t|e c.se` (||| ||rk .bcut t|e p.|n..y
ccrsune.s c| t|ese ne.|s .rJ t|e|. .espect|ve e|.st|c|t|es.)
11. \|.t e|ect Jc ycu t||rk .Jve.t|s|rg tc J|sccu..ge c|g..ette snck|rg .||| |.ve cr teers` Or .Ju|ts` \|.t
c|.rges n|g|t cccu. |r t|e c|g..ette n..ket .s . .esu|t`
12. V.ry n.ru|.ctu.e.s c| c|ct||rg .rJ ct|e. ccrsune. gccJs cper stc.es |r cut|et n.||s .|e.e t|ey c|..ge
nuc| |c.e. p.|ces t|.r t|ey c|..ge |r t|e|. c.r stc.es |cc.teJ .|t||r c|t|es. Out|et n.||s ..e typ|c.||y
|cc.teJ . ccrs|Je..b|e J|st.rce |.cn n.c. net.cpc||t.r ..e.s, .rJ stc.es |r t|en typ|c.||y c|..ge nuc|
|c.e. p.|ces t|.r Jc stc.es |cc.teJ .|t||r c|t|es. C|ver t|.t bct| sets c| stc.es ..e c|ter c.reJ by t|e
s.ne |.n, exp|.|r t||s p.|ce J|sc.|n|r.t|cr b.seJ cr ||ke|y J||e.erces |r t|e p.|ce e|.st|c|ty c| Jen.rJ
bet.eer ccrsune.s |r t|e t.c types c| stc.es.
13. Suppcse . p..t|cu|.. st.te .e.e tc b.r t|e .Jve.t|s|rg c| p.|ces c|..geJ by |.ns t|.t p.cv|Je |.se. eye
su.ge.y. \|.t e|ect Jc ycu t||rk t|.t .cu|J |.ve cr t|e p.|ce c| t||s se.v|ce`
14. |e .se |r |c|rt cr n|c.cb.e.e.|es rcteJ t|.t . |..ge runbe. c| suc| b.e.e.|es cper eve.y ye... +et,
t|e ncJe| c| ncrcpc||st|c ccnpet|t|cr p.eJ|cts t|.t t|e |crg .ur e,u|||b.|un sc|ut|cr |r suc| n..kets |s
cre c| .e.c eccrcn|c p.c|ts. \|y Jc |.ns erte. suc| |rJust.|es`
15. V.ry |..ye.s .Jve.t|se t|e|. se.v|ces. |c ycu t||rk t||s ..|ses c. |c.e.s t|e p.|ce c| |eg.| se.v|ces` |xp|.|r
ycu. .rs.e. c..e|u||y.
296 PRINCIPLS OI CONOMICS
N U M R I C A L P R O 8 L M S
1. Suppcse t|e ncrcpc||st|c.||y ccnpet|t|ve b..be. s|cp |rJust.y |r . ccnnur|ty |s |r |crg.ur e,u|||b.|un,
.rJ t|.t t|e typ|c.| p.|ce |s 20 pe. |.|.cut. Vc.ecve., t|e pcpu|.t|cr |s .|s|rg.
.. |||ust..te t|e s|c.t.ur e|ects c| . c|.rge cr t|e p.|ce .rJ cutput c| . typ|c.| |.n |r t|e n..ket.
b. S|c. .|.t |.ppers |r t|e |crg .ur. \||| t|e |r.| p.|ce be ||g|e. t|.r 20` |,u.| 20` be |ess
t|.r 20` Assune t|.t rct||rg |.ppers tc t|e ccst c| p.cJuc|rg |.|.cuts.
c. Suppcse t|.t, |r|t|.||y, t|e p.|ce c| . typ|c.| c|||J.ers |.|.cut |s 10. |c ycu t||rk t||s .ep.eserts
p.|ce J|sc.|n|r.t|cr` \|y c. .|y rct`
2. crs|Je. t|e s.ne |rJust.y .s |r |.cb|en 1. Suppcse t|e n..ket |s |r |crg.ur e,u|||b.|un .rJ t|.t .r
.rru.| ||cerse |ee |s |npcseJ cr b..be. s|cps.
.. |||ust..te t|e s|c.t.ur e|ects c| t|e c|.rge cr t|e p.|ce .rJ cutput c| |.|.cuts |c. . typ|c.| |.n
|r t|e ccnnur|ty.
b. |c. s|c. .|.t |.ppers tc p.|ce .rJ cutput |c. . typ|c.| |.n |r t|e |crg .ur.
c. \|c p.ys t|e |ee |r t|e |crg .ur` |c. Jces t||s ccnp..e tc t|e ccrc|us|crs c| t|e ncJe| c|
pe.|ect ccnpet|t|cr`
3. |rJust.y A ccrs|sts c| |cu. |.ns, e.c| c| .||c| |.s .r e,u.| s|..e c| t|e n..ket.
.. cnpute t|e |e.|rJ.||||.sc|n.r |rJex |c. t|e |rJust.y.
b. |rJust.y b ccrs|sts c| 10 |.ns, e.c| c| .||c| |.s .r e,u.| s|..e c| t|e n..ket. cnp..e t|e
|e.|rJ.||||.sc|n.r |rJexes |c. t|e t.c |rJust.|es.
c. |c. suppcse t|.t t|e.e ..e 100 |.ns |r t|e |rJust.y, e.c| .|t| e,u.| s|..es. \|.t |s t|e
|e.|rJ.||||.sc|n.r |rJex |c. t||s |rJust.y`
J. St.te t|e gere..| .e|.t|crs||p bet.eer t|e ccnpet|t|veress c| .r |rJust.y .rJ |ts |e.|rJ.||
||.sc|n.r |rJex.
4. C|ver t|e p.yc| n.t.|x (s|c.r be|c.) |c. . Jucpc|y, ccrs|st|rg c| ||.n A .rJ ||.n b, |r .||c| e.c| |.n |s
ccrs|Je.|rg .r exp.rJeJ .Jve.t|s|rg c.np.|gr, .rs.e. t|e |c||c.|rg ,uest|crs (.|| |gu.es |r t|e p.yc|
n.t.|x g|ve c|.rges |r .rru.| p.c|ts |r n||||crs c| Jc||..s).
.. |ces ||.n A |.ve . Jcn|r.rt st..tegy`
b. |ces ||.n b |.ve . Jcn|r.rt st..tegy`
c. |s t|e.e . Jcn|r.rt st..tegy e,u|||b.|un` |xp|.|r.
5. Suppcse t|.t t.c |rJust.|es e.c| |.ve . |cu.|.n ccrcert..t|cr ..t|c c| 5.
.. |xp|.|r .|.t t||s ne.rs.
b. Suppcse t|.t t|e ||| c| t|e |.st |rJust.y |s 425, .rJ t|.t t|e ||| c| t|e seccrJ |s 260. \||c|
.cu|J ycu s.y |s t|e nc.e ccnpet|t|ve` \|y`
6. Suppcse t|.t . typ|c.| |.n |r . ncrcpc||st|c.||y ccnpet|t|ve |rJust.y |.ces . Jen.rJ cu.ve g|ver by.
q = 60 (1 / 2)p, wheie q is quantity sold pei week.
|e |.ns n..g|r.| ccst cu.ve |s g|ver by. MC = 60 .
.. |c. nuc| .||| t|e |.n p.cJuce |r t|e s|c.t .ur`
b. \|.t p.|ce .||| |t c|..ge`
c. |... t|e |.ns Jen.rJ, n..g|r.| .everue, .rJ n..g|r.| ccst cu.ves. |ces t||s sc|ut|cr
.ep.esert . |crg.ur e,u|||b.|un` \|y c. .|y rct`
CHAP7R 11 7H WORLD OI IMPRIC7 COMP7I7ION 297
1.
2.
3.
4.
5.
ENDNOTES
\|ctc.|. Vu.p|y b...ett, |e.J|rg +cu. V|rJ, |oe ||e, Octcbe. 29, 200, 180 |s
sue 9, p. 50.
Vy|ere V.rg.||rJ.r .rJ |cbe.t A. Cut|, eb.y .|ks tc V|c.csc|t, +.|cc Abcut .
cnncr |ce. Cccg|e, |e /o|| .|ee| .ooo| ||e, Ap.|| 21, 2006, p. A1.
Ore |eg.| c..te| |s t|e |AA, .||c| n.ry eccrcn|sts .eg..J .s . success|u| Jev|ce
t|.cug| .||c| nenbe. |.ns (cc||eges .rJ ur|ve.s|t|es) cc||uJe cr . .|Je ..rge c|
.u|es t|.cug| .||c| t|ey p.cJuce spc.ts.
|ee ber|.n, |e ||ect c| AJve.t|s|rg cr t|e |.|ce c| |yeg|.sses, .ooo| o| |o.oJ
|coo|c 15(2) (192). 33352.
\ccJ.c. |ck..J, cnpet|t|cr .rJ t|e |g..ette \ AJve.t|s|rg b.r, |coo|c |
o|, 29(1) ('.ru..y 1991), 119133.
298 PRINCIPLS OI CONOMICS
| A | | | 1 3
Interest Rates and the
Markets for Capital and
Natural Resources
S7AR7 UP: 8UILDING 7H IN7RN7 IN 7H SkY"
|e ..ce tc bu||J t|e |rte.ret |r t|e Sky st..teJ |r t|e e..|y 1990s. Ore p|.r ..s tc bu||J 840 |c. e..t|c.b|t|rg
(||O) s.te|||tes t|.t .cu|J .||c. |r|c.n.t|cr tc be sert .rJ .ece|veJ |rst.rt.recus|y .ry.|e.e cr t|e |.ce c| t|e
g|cbe. At |e.st t|.t ..s t|e p|.r.
A runbe. c| te|eccnnur|c.t|cr |rJust.y g|.rts, .s .e|| .s scne |..ge n.ru|.ctu.|rg ccnp.r|es, .e.e |n
p.esseJ .|t| t|e pcss|b|||t|es. |ey s.. .|.t t|ey t|cug|t ..s . p.c|t.b|e cppc.tur|ty .rJ Jec|JeJ tc put up
scne |r.rc|.| c.p|t.|. ..|g Vc.., .|c n.Je . |c.ture Jeve|cp|rg .rJ t|er se|||rg tc A, t|e .c.|Js |..gest
ce||u|.. p|cre ret.c.k, bec.ne c|.|. c| e|eJes|c, t|e ccnp.ry |e |c.neJ tc bu||J t|e ||O s.te|||te systen.
Vc.. put up n||||crs c| Jc||..s tc |urJ t|e p.cect, .s J|J V|c.csc|ts b||| C.tes .rJ |.|rce A|..|eeJ b|r .|.| b|r
AbJu|..|. c| S.uJ| A..b|.. bce|rg, Vctc.c|., .rJ V.t.. V..ccr| Sp.ce, |u.cpes |e.J|rg s.te|||te n.ru|.ctu.e., be
c.ne cc.pc..te p..tre.s. A|tcget|e., t|e ccnp.ry ..|seJ .|ncst . b||||cr Jc||..s. |e ert|.e p.cect ..s est|n.teJ
tc ccst 9 b||||cr.
but, .|.s, . Jec.Je |.te. t|e ccnp.ry |.J s|||teJ |rtc ve.y |c. ge... |.cn t|e |r|t|.| p|.r |c. 840 s.te|||tes, t|e
p.cect ..s sc.|eJ b.ck tc 300 s.te|||tes .rJ t|er tc . ne.e 30. |er, |r 2003 |r . |ette. tc t|e |.S. |eJe..| cnnu
r|c.t|crs ccnn|ss|cr, |t .rrcurceJ t|.t |t ..s g|v|rg up |ts ||cerse tc use . |..ge p..t c| t|e ..J|c spect.un.
1
\|.t |.ppereJ tc t||s J.e.n` |e Jeve|cpnert c| ce||u|.. ret.c.ks tc |.rJ|e J.t. .rJ v|Jec t..rsn|ss|crs
n.y |.ve n.Je t|e s.te|||te systen seen urrecess..y. |r ccrt..st tc . s.te|||te systen t|.t |.s tc be bu||t |r tct.|
|r c.Je. tc b.|rg |r . s|rg|e custcne., .|.e|ess ccnp.r|es .e.e .b|e tc bu||J t|e|. custcne. b.se c|ty by c|ty.
|ver || t|e p.cect |.J beccne success|u|, t|e .e...Js tc t|e ccnp.r|es .rJ tc t|e |rJ|v|Ju.|s t|.t put t|e|.
|r.rc|.| c.p|t.| |rtc t|e vertu.e .cu|J |.ve beer . |crg t|ne |r ccn|rg. Se.v|ce ..s |r|t|.||y sc|eJu|eJ tc beg|r
|r 2001, but e|eJes|c J|J rct ever s|gr . ccrt..ct tc bu||J |ts |.st t.c s.te|||tes urt|| |eb.u..y 2002, .rJ s|x ncrt|s
|.te. t|e ccnp.ry .rrcurceJ t|.t .c.k cr t|cse |.J beer susperJeJ.
e|eJes|cs p.cpcseJ vertu.e ..s b|gge. t|.r ncst c.p|t.| p.cects, but |t s|..es scne b.s|c c|...cte.|st|cs
.|t| .ry .c,u|s|t|cr c| c.p|t.| by |.ns. |e p.cJuct|cr c| c.p|t.|t|e gccJs useJ |r p.cJuc|rg ct|e. gccJs .rJ
se.v|ces.e,u|.es s.c.||c|rg ccrsunpt|cr. |e .etu.rs tc c.p|t.| .||| be sp.e.J cve. t|e pe.|cJ |r .||c| t|e c.p|t
.| |s useJ. |e c|c|ce tc .c,u|.e c.p|t.| |s t|us . c|c|ce tc g|ve up ccrsunpt|cr tcJ.y |r |cpes c| .etu.rs |r t|e |u
tu.e. bec.use t|cse .etu.rs ..e |.. |.cn ce.t.|r, t|e c|c|ce tc .c,u|.e c.p|t.| |s |rev|t.b|y . .|sky cre.
|c. .|| |ts spec|.| c|...cte.|st|cs, |c.eve., c.p|t.| |s . |.ctc. c| p.cJuct|cr. As .e |rvest|g.te t|e n..ket |c. c.p
|t.|, t|e ccrcepts c| n..g|r.| .everue p.cJuct, n..g|r.| |.ctc. ccst, .rJ t|e n..g|r.| Jec|s|cr .u|e t|.t .e |.ve
weaIth
|e sun c| .ssets |ess
||.b|||t|es.
assets
Aryt||rg c| v.|ue.
IiabiIities
Ob||g.t|crs tc n.ke |utu.e
p.ynerts.
Jeve|cpeJ .||| ccrt|rue tc se.ve us. |e b|g J||e.erce |s t|.t t|e bere|ts .rJ ccsts c| |c|J|rg c.p|t.| ..e J|st.|b
uteJ cve. t|ne.
\e .||| .|sc ex.n|re n..kets |c. r.tu..| .escu.ces |r t||s c|.pte.. ||ke Jec|s|crs |rvc|v|rg c.p|t.|, c|c|ces |r
t|e .||cc.t|cr c| r.tu..| .escu.ces |.ve |.st|rg e|ects. |c. pctert|.||y ex|.ust|b|e r.tu..| .escu.ces suc| .s c||, t|e
e|ects c| t|cse c|c|ces |.st |c.eve..
|c. t|e .r.|ys|s c| c.p|t.| .rJ r.tu..| .escu.ces, .e s|||t |.cn t|e ex.n|r.t|cr c| cutccnes |r t|e cu..ert pe.|
cJ tc t|e .r.|ys|s c| cutccnes J|st.|buteJ cve. n.ry pe.|cJs. |rte.est ..tes, .||c| ||rk t|e v.|ues c| p.ynerts t|.t
cccu. .t J||e.ert t|nes, .||| be cert..| tc cu. .r.|ys|s.
1. TIME AND INTEREST RATES
L A R N I N G O 8 1 C 7 I V S
1. Dene interest and the interest rate.
2. Describe and show aIgebraicaIIy how to compute present vaIue.
3. List and expIain the factors that aect what the present vaIue of some future payment wiII be.
Time, the saying goes, is natuie's way of keeping eveiything fiom happening all at once. And the fact
that eveiything does not happen at once intioduces an impoitant complication in economic analysis.
When a company decides to use funds to install capital that will not begin to pioduce income foi
seveial yeais, it needs a way to compaie the signifcance of funds spent now to income eained latei. It
must fnd a way to compensate fnancial investois who give up the use of theii funds foi seveial yeais,
until the pioject begins to pay of. How can payments that aie distiibuted acioss time be linked to one
anothei: Inteiest iates aie the linkage mechanism; we shall investigate how they achieve that linkage in
this section.
1.1 The Natuie of Inteiest Rates
Considei a delightful pioblem of choice. Youi Aunt Caimen ofeis to give you $10,000 now oi $10,000
in one yeai. Which would you pick:
Most people would choose to take the payment now. One ieason foi that choice is that the aveiage
level of piices is likely to iise ovei the next yeai. The puichasing powei of $10,000 today is thus gieatei
than the puichasing powei of $10,000 a yeai hence. Theie is also a question of whethei you can count
on ieceiving the payment. If you take it now, you have it. It is iisky to wait a yeai; who knows what will
happen:
Let us eliminate both of these pioblems. Suppose that you aie confdent that the aveiage level of
piices will not change duiing the yeai, and you aie absolutely ceitain that if you choose to wait foi the
payment, you and it will both be available. Will you take the payment now oi wait:
Chances aie you would still want to take the payment now. Peihaps theie aie some things you
would like to puichase with it, and you would like them soonei iathei than latei. Moieovei, if you wait
a yeai to get the payment, you will not be able to use it while you aie waiting. If you take it now, you
can choose to spend it now oi wait.
Now suppose Aunt Caimen wants to induce you to wait and changes the teims of hei gift. She
ofeis you $10,000 now oi $11,000 in one yeai. In efect, she is ofeiing you a $1,000 bonus if you will
wait a yeai. If you agiee to wait a yeai to ieceive Aunt Caimen's payment, you will be accepting hei
promise to piovide funds instead of the funds themselves. Eithei will inciease youi wealth, which is the
sum of all youi assets less all youi liabilities. Assets aie anything you have that is of value; liabilities
aie obligations to make futuie payments. Both a $10,000 payment fiom Aunt Caimen now and hei
piomise of $11,000 in a yeai aie examples of assets. The alteinative to holding wealth is to consume it.
You could, foi example, take Aunt Caimen's $10,000 and spend it foi a tiip to Euiope, thus ieducing
youi wealth. By making a bettei ofei-$11,000 instead of $10,000-Aunt Caimen is tiying to induce
you to accept an asset you will not be able to consume duiing the yeai.
328 PRINCIPLS OI CONOMICS
interest
A p.ynert n.Je tc pecp|e
.|c .g.ee tc pcstpcre t|e|.
use c| .e.|t|.
interest rate
|e cppc.tur|ty ccst c| us|rg
.e.|t| tcJ.y, exp.esseJ .s .
pe.cert.ge c| t|e .ncurt c|
.e.|t| .|cse use |s
pcstpcreJ.
present vaIue
Ar .ncurt t|.t .cu|J e,u.|
. p..t|cu|.. |utu.e v.|ue ||
Jepcs|teJ tcJ.y .t . spec||c
|rte.est ..te.
The $1,000 bonus Aunt Caimen is ofeiing if you will wait a yeai foi hei payment is inteiest. In
geneial, interest is a payment made to people who agiee to postpone theii use of wealth. The interest
rate iepiesents the oppoitunity cost of using wealth today, expiessed as a peicentage of the amount of
wealth whose use is postponed. Aunt Caimen is ofeiing you $1,000 if you will pass up the $10,000
today. She is thus ofeiing you an inteiest iate of 10 ($1,000 / $10,000 = 0.1 = 10 ).
Suppose you tell Aunt Caimen that, given the two options, you would still iathei have the $10,000
today. She now ofeis you $11,300 if you will wait a yeai foi the payment-an inteiest iate of 13 (
$1,300 / $10,000 = 0.13 = 13 ). The moie she pays foi waiting, the highei the inteiest iate.
You aie piobably familiai with the iole of inteiest iates in loans. In a loan, the boiiowei obtains a
payment now in exchange foi piomising to iepay the loan in the futuie. The lendei thus must postpone
his oi hei use of wealth until the time of iepayment. To induce lendeis to postpone theii use of theii
wealth, boiioweis ofei inteiest. Boiioweis aie willing to pay inteiest because it allows them to acquiie
the sum now iathei than having to wait foi it. And lendeis iequiie inteiest payments to compensate
them foi postponing theii own use of theii wealth.
1.2 Inteiest Rates and Piesent Value
We saw in the pievious section that people geneially piefei to ieceive a payment of some amount today
iathei than wait to ieceive that same amount latei. We may conclude that the value today of a payment
in the futuie is less than the dollai value of the futuie payment. An impoitant application of inteiest
iates is to show the ielationship between the cuiient and futuie values of a paiticulai payment.
To see how we can calculate the cuiient value of a futuie payment, let us considei an example sim-
ilai to Aunt Caimen's ofei. This time you have $1,000 and you deposit it in a bank, wheie it eains in-
teiest at the iate of 10 pei yeai.
How much will you have in youi bank account at the end of one yeai: You will have the oiiginal
$1,000 plus 10 of $1,000, oi $1,100:
$1,000 + (0.10)($1,000) = $1,100
Moie geneially, if we let P
0
equal the amount you deposit today, r the peicentage iate of inteiest, and
P
1
the balance of youi deposit at the end of 1 yeai, then we can wiite:
QUA7I ON 13. 1
P
0
+ rP
0
= P
1
Factoiing out the P
0
teim on the left-hand side of Equation 13.1, we have:
QUA7I ON 13. 2
P
0
(1 + r) = P
1
Equation 13.2 shows how to deteimine the futuie value of a payment oi deposit made today. Now
let us tuin the question aiound. We can ask what P
1
, an amount that will be available 1 yeai fiom now,
is woith today. We solve foi this by dividing both sides of Equation 13.2 by (1 + r) to obtain:
QUA7I ON 13. 3
P
0
=
P
1
(1 + r)
Equation 13.3 suggests how we can compute the value today, P
0
, of an amount P
1
that will be paid
a yeai hence. An amount that would equal a paiticulai futuie value if deposited today at a specifc in-
teiest iate is called the present value of that futuie value.
Moie geneially, the piesent value of any payment to be ieceived n peiiods fiom now =
QUA7I ON 13. 4
P
0
=
P
n
(1+i)
n
Suppose, foi example, that youi Aunt Caimen ofeis you the option of $1,000 now oi $13,000 in 30
yeais. We can use Equation 13.4 to help you decide which sum to take. The piesent value of $13,000 to
be ieceived in 30 yeais, assuming an inteiest iate of 10, is:
CHAP7R 13 IN7RS7 RA7S AND 7H MARk7S IOR CAPI7AL AND NA7URAL RSOURCS 329
P
0
=
P
30
(1 + r)
30
=
$13,000
(1 + 0.10)
30
= $839.63
Assuming that you could eain that 10 ietuin with ceitainty, you would be bettei of taking Aunt Cai-
men's $1,000 now; it is gieatei than the piesent value, at an inteiest iate of 10, of the $13,000 she
would give you in 30 yeais. The $1,000 she gives you now, assuming an inteiest iate of 10, in 30 yeais
will giow to:
$1,000(1 + 0.10)
30
= $17,449.40
The piesent value of some futuie payment depends on thiee things.
1. The Size of the Payment Itself. The biggei the futuie payment, the gieatei its piesent value.
2. The Iength of the Period Until Payment. The piesent value depends on how long a peiiod will
elapse befoie the payment is made. The piesent value of $13,000 in 30 yeais, at an inteiest iate of
10, is $839.63. But that same sum, if paid in 20 yeais, has a piesent value of $2,229.63. And if
paid in 10 yeais, its piesent value is moie than twice as gieat: $3,783.13. The longei the time
peiiod befoie a payment is to be made, the lowei its piesent value.
3. The Rate of Interest. The piesent value of a payment of $13,000 to be made in 20 yeais is
$2,229.63 if the inteiest iate is 10; it iises to $3,633.34 at an inteiest iate of 3. The lowei the
inteiest iate, the highei the piesent value of a futuie payment. Table 13.1 gives piesent values of a
payment of $13,000 at vaiious inteiest iates and foi vaiious time peiiods.
7A8L 13. 1 7ime, Interest Rates, and Present VaIue
|e ||g|e. t|e |rte.est ..te .rJ t|e |crge. t|e t|ne urt|| p.ynert |s n.Je, t|e |c.e. t|e p.esert v.|ue c| . |utu.e
p.ynert. |e t.b|e be|c. s|c.s t|e p.esert v.|ue c| . |utu.e p.ynert c| 15,000 urJe. J||e.ert ccrJ|t|crs. |e
p.esert v.|ue c| 15,000 tc be p.|J |r |ve ye..s |s 11,52.89 || t|e |rte.est ..te |s 5. |ts p.esert v.|ue |s ust
391.26 || |t |s tc be p.|J |r 20 ye..s .rJ t|e |rte.est ..te |s 20.
Present VaIue of $15,000
7ime untiI payment Interest rate (%)
5 years 10 years 15 years 20 years
5 11,52.89 9,208.0 ,215.26 5,653.34
10 9,313.82 5,83.15 3,590.88 2,229.65
15 ,45.65 3,0. 1,843.42 916.50
20 6,028.16 2,422.58 93.58 391.26
The concept of piesent value can also be applied to a seiies of futuie payments. Suppose you have been
piomised $1,000 at the end of each of the next 3 yeais. Because each payment will occui at a difeient
time, we calculate the piesent value of the seiies of payments by taking the value of each payment sep-
aiately and adding them togethei. At an inteiest iate of 10, the piesent value P
0
is:
P
0
=
$1,000
1.10
+
$1,000
(1.10)
2
+
$1,000
(1.10)
3
+
$1,000
(1.10)
4
+
$1,000
(1.10)
3
= $3,790.78
Inteiest iates can thus be used to compaie the values of payments that will occui at difeient times.
Choices conceining capital and natuial iesouices iequiie such compaiisons, so you will fnd applica-
tions of the concept of piesent value thioughout this chaptei, but the concept of piesent value applies
whenevei costs and benefts do not all take place in the cuiient peiiod.
State lotteiy winneis often have a choice between a single laige payment now oi smallei payments
paid out ovei a 23- oi 30-yeai peiiod. Compaiing the single payment now to the piesent value of the
futuie payments allows winneis to make infoimed decisions. Foi example, in June 2003 Biad Duke, of
Boise, Idaho, became the winnei of one of the laigest lotteiy piizes evei. Given the alteinative of claim-
ing the $220.3 million jackpot in 30 annual payments of $7.4 million oi taking $123.3 million in a lump
sum, he chose the lattei. Holding unchanged all othei consideiations that must have been going
thiough his mind, he must have thought his best iate of ietuin would be gieatei than 4.17. Why
4.17: Using an inteiest iate of 4.17, $123.3 million is equal to slightly less than the piesent value of
the 30-yeai stieam of payments. At all inteiest iates gieatei than 4.17, the piesent value of the stieam
of benefts would be less than $123.3 million. At all inteiest iates less than 4.17, the piesent value of
the stieam of payments would be moie than $123.3 million. Oui piesent value analysis suggests that if
he thought the inteiest iate he could eain was moie than 4.17, he should take the lump sum payment,
which he did.
330 PRINCIPLS OI CONOMICS
k Y 7 A k A W A Y S
< |ecp|e gere..||y p.e|e. tc .ece|ve . spec||c p.ynert rc. ..t|e. t|.r tc ..|t .rJ .ece|ve |t |.te..
< |rte.est |s . p.ynert n.Je tc pecp|e .|c .g.ee tc pcstpcre t|e|. use c| .e.|t|.
< \e ccnpute t|e p.esert v.|ue, |
0
, c| . sun tc be .ece|veJ |r ye..s, |
r
, .s.
P
0
=
P
n
(1 + r)
n
< |e p.esert v.|ue c| . |utu.e p.ynert .||| be g.e.te. t|e |..ge. t|e p.ynert, t|e sccre. |t |s Jue, .rJ t|e
|c.e. t|e ..te c| |rte.est.
7 R Y I 7 !
Suppcse ycu. |.|erJ S... .sks ycu tc |erJ |e. 5,000 sc s|e c.r buy . useJ c... S|e te||s ycu s|e c.r p.y ycu
b.ck 5,200 |r . ye... |e||.b|e S... .|..ys keeps |e. .c.J. Suppcse t|e |rte.est ..te ycu ccu|J e..r by put
t|rg t|e 5,000 |r . s.v|rgs .cccurt |s 5. \|.t |s t|e p.esert v.|ue c| |e. c|e.` |s |t . gccJ Je.| |c. ycu c.
rct` \|.t || t|e |rte.est ..te cr ycu. s.v|rgs .cccurt |s cr|y 3`
Case in Point: Waiting for Death and Life Insurance
2010 jupiterimages Corporation
|t |s . t.|e t|.t |.s beccne .|| tcc |.n|||...
.|| ||n |cge. 'c|rscr. |e |.s ust |e..reJ t|.t ||s c.rce. |s rct t.e.t.b|e .rJ t|.t |e |.s cr|y . ye.. c. t.c
tc ||ve. V.. 'c|rscr |s ur.b|e tc .c.k, .rJ ||s |r.rc|.| bu.Jers ccnpcurJ ||s t..g|c neJ|c.| s|tu.t|cr. |e |.s
nc.tg.geJ ||s |cuse .rJ sc|J ||s ct|e. .ssets |r . Jespe..te e|c.t tc get ||s |.rJs cr t|e c.s| |e reeJs |c.
c..e, |c. |ccJ, .rJ |c. s|e|te.. |e |.s . |||e |rsu..rce pc||cy, but |t .||| p.y c| cr|y .|er |e J|es. || cr|y |e
ccu|J get scne c| t|.t ncrey sccre..
|e p.cb|en |.c|rg V.. 'c|rscr |.s sp..reJ . n..ket sc|ut|crccnp.r|es .rJ |rJ|v|Ju.|s t|.t buy t|e |||e
|rsu..rce pc||c|es c| t|e te.n|r.||y |||. V.. 'c|rscr ccu|J se|| ||s pc||cy tc cre c| t|ese ccnp.r|es c. |rJ|v|Ju
.|s .rJ cc||ect t|e pu.c|.se p.|ce. |e buye. t.kes cve. ||s p.en|un p.ynerts. \|er |e J|es, t|e ccnp.ry
.||| cc||ect t|e p.cceeJs c| t|e pc||cy.
CHAP7R 13 IN7RS7 RA7S AND 7H MARk7S IOR CAPI7AL AND NA7URAL RSOURCS 331
|e |rJust.y |s c.||eJ t|e v|.t|c.| |rJust.y (t|e te.n .|o||co| ccnes |.cn .|o||co, . |.|st|.r s.c..nert g|ver
tc . Jy|rg pe.scr). |t p.cv|Jes t|e te.n|r.||y ||| .|t| .ccess tc ncrey .|||e t|ey ..e .||ve, |t p.cv|Jes |r.rc|.|
|rvestc.s . |e.|t|y |rte.est p.en|un cr t|e|. |urJs.
|t |s . c|||||rg bus|ress. |ctert|.| buye.s pc.e cve. p.t|erts neJ|c.| ||stc.|es, stuJy|rg ce|| ccurts .rJ ct|e.
|rJ|c.tc.s c| . p.t|erts |e.|t|. |.cn t|e buye.s pc|rt c| v|e., . speeJy Je.t| |s Jes|..b|e, bec.use |t ne.rs
t|e |rvestc. .||| cc||ect ,u|ck|y cr t|e pu.c|.se c| . p.t|erts pc||cy.
A p.t|ert .|t| . |||e expect.rcy c| |ess t|.r s|x ncrt|s n|g|t be .b|e tc se|| ||s c. |e. |||e |rsu..rce pc||cy |c.
80 c| t|e |.ce v.|ue. A 200,000 pc||cy .cu|J t|us se|| |c. 160,000. A pe.scr .|t| . bette. p.cgrcs|s .||| cc|
|ect |ess. |.t|erts expecteJ tc ||ve t.c ye..s, |c. ex.np|e, n|g|t get cr|y 60 c| t|e |.ce v.|ue c| t|e|. pc||c|es.
A.e |rvestc.s p.c|t|rg |.cn t|e n|se.y c| ct|e.s` O| ccu.se t|ey ..e. but, suppcse t|.t |rvestc.s .e|useJ tc
t.ke .Jv.rt.ge c| t|e n|s|c.ture c| t|e te.n|r.||y |||. |.t .cu|J Jery Jy|rg pecp|e t|e c|.rce tc .c,u|.e
|urJs t|.t t|ey Jespe..te|y reeJ. As |s t|e c.se .|t| .|| vc|urt..y exc|.rge, t|e v|.t|c.| n..ket c.e.tes .|r
.|r s|tu.t|crs. |rvestc.s .|r by e..r|rg ||g| ..tes c| .etu.r cr t|e|. |rvestnert. ArJ t|e Jy|rg p.t|ert` |e
c. s|e |s |r . te..|b|e s|tu.t|cr, but t|e cppc.tur|ty tc cbt.|r |urJs n.kes t|.t pe.scr . .|rre. .s .e||.
||n |. O.., . |c.ne. .gert .|t| |||e |..tre.s |rc. (....|||ep..tre.s|rc.ccn), cre c| t|e |e.J|rg |.ns |r t|e v|
.t|c.| |rJust.y, .ec.||eJ . c.se |r ||s c.r |.n||y. Scne ye..s .gc, | |.J . ccus|r .|c J|eJ c| A||S. |e ..s, .t
t|e erJ, Jest|tute .rJ |.J tc .e|y tct.||y cr ||s |.n||y |c. suppc.t. cJ.y, t|e.e |s . b.c.J n..ket .|t| |cts c|
p..t|c|p.rts, .rJ . p.t|ert c.r .e.||.e . ||g| |..ct|cr c| t|e |.ce v.|ue c| . pc||cy cr se|||rg |t. |e n..ket |e|ps
buye.s .rJ p.t|erts .||ke.
|r .ecert ye..s, t||s |rJust.y |.s beer .er.neJ t|e |||e sett|enerts |rJust.y, .|t| pc||cy t..rs|e.s be|rg
c|e.eJ tc |e.|t||e., c|ter e|Je.|y, pc||cy|c|Je.s. |ese |e.|t||e. |rJ|v|Ju.|s ..e scnet|nes tu.r|rg cve. t|e|.
pc||c|es |c. . p.ynert tc t||.J p..t|es .|c p.y t|e p.en|uns .rJ t|er cc||ect t|e bere|t .|er t|e pc||cy
|c|Je.s J|e. |xp.rs|cr c| t||s p..ct|ce |.s begur tc ..|se ccsts |c. |||e |rsu.e.s, .|c .ssuneJ t|.t |rJ|v|Ju.|s
.cu|J scnet|nes |et t|e|. pc||c|es |.pse, .|t| t|e .esu|t t|.t t|e |rsu..rce ccnp.ry Jces rct |.ve tc p.y
c|.|ns cr t|en. bus|resses buy|rg |||e |rsu..rce pc||c|es ..e rct ||ke|y tc |et t|en |.pse.
.ooce |eoo| ||e.|e. oJ ||o ||e.e oJ |oc|e| |o .||.eo. |.e|o .ee| |o|| | .|oe |eo||'. |e /o|| .|ee| .ooo| ||e. 2
/o, 2006.
A N S W R 7 O 7 R Y I 7 ! P R O 8 L M
|e p.esert v.|ue c| 5,200 p.y.b|e |r . ye.. .|t| .r |rte.est ..te c| 5 |s.
P
0
=
$3,200
(1 + 0.03)
1
= $4,932.38
S|rce t|e p.esert v.|ue c| 5,200 |s |ess t|.r t|e 5,000 S... |.s .skeJ ycu tc |erJ |e., ycu .cu|J be bette.
c| .e|us|rg tc n.ke t|e |c.r. Arct|e. ..y c| ev.|u.t|rg t|e |c.r |s t|.t S... |s c|e.|rg . .etu.r cr ycu.
5,000 c| 200 / 3,000 = 4, .|||e t|e b.rk |s c|e.|rg ycu . 5 .etu.r. Or t|e ct|e. |.rJ, || t|e |rte.est ..te
t|.t ycu. b.rk |s p.y|rg |s 3, t|er t|e p.esert v.|ue c| .|.t S... .||| p.y ycu |r . ye.. |s.
P
0
=
$3,200
(1 + 0.03)
1
= $3,048.34
\|t| ycu. b.rk cr|y p.y|rg . 3 .etu.r, S...s c|e. |ccks ||ke . gccJ Je.|.
332 PRINCIPLS OI CONOMICS
net present vaIue (||v)
|e v.|ue e,u.| tc t|e
p.esert v.|ue c| .|| t|e
.everues expecteJ |.cn .r
.sset n|rus t|e p.esert v.|ue
c| .|| t|e ccsts .sscc|.teJ
.|t| |t.
2. INTEREST RATES AND CAPITAL
L A R N I N G O 8 1 C 7 I V S
1. Dene investment, expIain how to determine the net present vaIue of an investment pro|ect,
and expIain how the net present vaIue caIcuIation aids the decision maker in determining
whether or not to pursue an investment pro|ect.
2. xpIain the demand curve for capitaI and the factors that can cause it to shift.
3. xpIain and iIIustrate the IoanabIe funds market and expIain how changes in the demand for
capitaI aect that market and vice versa.
The quantity of capital that fims employ in theii pioduction of goods and seivices has enoimously im-
poitant implications foi economic activity and foi the standaid of living people in the economy enjoy.
Incieases in capital inciease the maiginal pioduct of laboi and boost wages at the same time they boost
total output. An inciease in the stock of capital theiefoie tends to iaise incomes and impiove the stand-
aid of living in the economy.
Capital is often a fxed factoi of pioduction in the shoit iun. A fim cannot quickly ietool an as-
sembly line oi add a new omce building. Deteimining the quantity of capital a fim will use is likely to
involve long-iun choices.
2.1 The Demand foi Capital
A fim uses additional units of a factoi until maiginal ievenue pioduct equals maiginal factoi cost.
Capital is no difeient fiom othei factois of pioduction, save foi the fact that the ievenues and costs it
geneiates aie distiibuted ovei time. As the fist step in assessing a fim's demand foi capital, we de-
teimine the piesent value of maiginal ievenue pioducts and maiginal factoi costs.
CapitaI and Net Present VaIue
Suppose Caiol Stein is consideiing the puichase of a new $93,000 tiactoi foi hei faim. Ms. Stein ex-
pects to use the tiactoi foi fve yeais and then sell it; she expects that it will sell foi $22,000 at the end of
the fve-yeai peiiod. She has the $93,000 on hand now; hei alteinative to puichasing the tiactoi could
be to put $93,000 in a bond account eaining 7 annual inteiest.
Ms. Stein expects that the tiactoi will biing in additional annual ievenue of $30,000 but will cost
$30,000 pei yeai to opeiate, foi net ievenue of $20,000 annually. Foi simplicity, we shall suppose that
this net ievenue acciues at the end of each yeai.
Should she buy the tiactoi: We can answei this question by computing the tiactoi's net present
value (NPV), which is equal to the piesent value of all the ievenues expected fiom an asset minus the
piesent value of all the costs associated with it. We thus measuie the difeience between the piesent
value of maiginal ievenue pioducts and the piesent value of maiginal factoi costs. If ^PV is gieatei
than zeio, puichase of the asset will inciease the pioftability of the fim. A negative ^PV implies that
the funds foi the asset would yield a highei ietuin if used to puichase an inteiest-beaiing asset. A fim
will maximize piofts by acquiiing additional capital up to the point that the piesent value of capital's
maiginal ievenue pioduct equals the piesent value of maiginal factoi cost.
If the ievenues geneiated by an asset in peiiod n equal R
n
and the costs in peiiod n equal C
n
, then
the net piesent value ^PV
0
of an asset expected to last foi n yeais is:
QUA7I ON 13. 5
^PV
0
= R
0
C
0
+
R
1
C
1
1 + r
+ ... +
R
n
C
n
(1 + r)
n
To puichase the tiactoi, Ms. Stein pays $93,000. She will ieceive additional ievenues of $30,000 pei
yeai fiom incieased planting and moie emcient haivesting, less the opeiating cost pei yeai of $30,000,
plus the $22,000 she expects to get by selling the tiactoi at the end of fve yeais. The net piesent value of
the tiactoi, ^PV
0
is thus given by:
^PV
0
= $93,000 +
$20,000
1.07
+
$20,000
1.07
2
+
$20,000
1.07
3
+
$20,000
1.07
4
+
$42,000
1.07
3
= $2,690
CHAP7R 13 IN7RS7 RA7S AND 7H MARk7S IOR CAPI7AL AND NA7URAL RSOURCS 333
investment
Ar .JJ|t|cr tc c.p|t.| stcck.
demand curve for capitaI
S|c.s t|e ,u.rt|ty c| c.p|t.|
|.ns |rterJ tc |c|J .t e.c|
|rte.est ..te.
II GUR 13. 2 7he Demand Curve for
CapitaI
|e ,u.rt|ty c| c.p|t.| |.ns .||| ..rt tc |c|J
JeperJs cr t|e |rte.est ..te. |e ||g|e. t|e
|rte.est ..te, t|e |ess c.p|t.| |.ns .||| ..rt tc
|c|J.
Given the cost of the tiactoi, the net ietuins Ms. Stein piojects, and an inteiest iate of 7, Ms. Stein
will inciease hei piofts by puichasing the tiactoi. The tiactoi will yield a ietuin whose piesent value is
$2,690 gieatei than the ietuin that could be obtained by the alteinative of putting the $93,000 in a bond
account yielding 7.
Ms. Stein's acquisition of the tiactoi is called investment. Economists defne investment as an ad-
dition to capital stock. Any acquisition of new capital goods theiefoie qualifes as investment.
7he Demand Curve for CapitaI
Oui analysis of Caiol Stein's decision iegaiding the puichase of a new tiactoi suggests the foices at
woik in deteimining the economy's demand foi capital. In deciding to puichase the tiactoi, Ms. Stein
consideied the piice she would have to pay to obtain the tiactoi, the costs of opeiating it, the maiginal
ievenue pioduct she would ieceive by owning it, and the piice she could get by selling the tiactoi when
she expects to be done with it. Notice that with the exception of the puichase piice of the tiactoi, all
those fguies weie piojections. Hei decision to puichase the tiactoi depends almost entiiely on the
costs and benefts she expects will be associated with its use.
Finally, Ms. Stein conveited all those fguies to a net piesent value based on the inteiest iate pie-
vailing at the time she made hei choice. A positive ^PV means that hei piofts will be incieased by pui-
chasing the tiactoi. That iesult, of couise, depends on the pievailing inteiest iate. At an inteiest iate of
7, the ^PV is positive. At an inteiest iate of 8, the ^PV would be negative. At that inteiest iate, Ms.
Stein would do bettei to put hei funds elsewheie.
At any one time, millions of choices like that of Ms. Stein conceining the acquisition of capital will
be undei consideiation. Each decision will hinge on the piice of a paiticulai piece of capital, the expec-
ted cost of its use, its expected maiginal ievenue pioduct, its expected sciap value, and the inteiest iate.
Not only will fims be consideiing the acquisition of new capital, they will be consideiing ietaining ex-
isting capital as well. Ms. Stein, foi example, may have othei tiactois. Should she continue to use them,
oi should she sell them: If she keeps them, she will expeiience a stieam of ievenues and costs ovei the
next seveial peiiods; if she sells them, she will have funds now that she could use foi something else. To
decide whethei a fim should keep the capital it alieady has, we need an estimate of the ^PV of each
unit of capital. Such decisions aie always afected by the inteiest iate. At highei iates of inteiest, it
makes sense to sell some capital iathei than hold it. At lowei iates of inteiest, the ^PV of holding capit-
al will iise.
Because fims' choices to acquiie new capital and to hold existing capital depend on the inteiest
iate, the demand curve for capital in Figuie 13.2, which shows the quantity of capital fims intend to
hold at each inteiest iate, is downwaid-sloping. At point A, we see that at an inteiest iate of 10, $8
tiillion woith of capital is demanded in the economy. At point B, a ieduction in the inteiest iate to 7
incieases the quantity of capital demanded to $9 tiillion. At point C, at an inteiest iate of 4, the
quantity of capital demanded is $10 tiillion. A ieduction in the inteiest iate incieases the quantity of
capital demanded.
The demand cuive foi capital foi the economy is found by summing the demand
cuives of all holdeis of capital. Ms. Stein's demand cuive, foi example, might show that
at an inteiest iate of 8, she will demand the capital she alieady has-suppose it is
$600,000 woith of equipment. If the inteiest iate diops to 7, she will add the tiactoi;
the quantity of capital she demands iises to $693,000. At inteiest iates gieatei than 8,
she might decide to ieduce hei maintenance efoits foi some of the capital she alieady
has; the quantity of capital she demands would fall below $600,000. As with the de-
mand foi capital in the economy, we can expect individual fims to demand a smallei
quantity of capital when the inteiest iate is highei.
Shifts in the Demand for CapitaI
Why might the demand foi capital change: Because the demand foi capital iefects the
maiginal ievenue pioduct of capital, anything that changes the maiginal ievenue
pioduct of capital will shift the demand foi capital. Oui seaich foi demand shifteis
must thus focus on factois that change the maiginal pioduct of capital, the piices of the
goods capital pioduces, and the costs of acquiiing and holding capital. Let us discuss
some factois that could afect these vaiiables and thus shift the demand foi capital.
Changes in xpectations
Choices conceining capital aie always based on expectations. Net piesent value is com-
puted fiom the expected ievenues and costs ovei the expected life of an asset. If fims'
expectations change, theii demand foi capital will change. If something causes fims to ievise theii
sales expectations upwaid (such as stiongei than expected sales in the iecent past), it is likely to
334 PRINCIPLS OI CONOMICS
bond
A p.cn|se tc p.y b.ck .
ce.t.|r .ncurt .t . ce.t.|r
t|ne.
IoanabIe funds market
|e n..ket |r .||c|
bc..c.e.s (Jen.rJe.s c|
|urJs) .rJ |erJe.s (supp||e.s
c| |urJs) neet.
inciease theii demand foi capital. Similaily, an event that dampens fims' expectations (such as iecent
weak sales) is likely to ieduce theii demand foi capital.
7echnoIogicaI Change
Technological changes can inciease the maiginal pioduct of capital and thus boost the demand foi cap-
ital. The discoveiy of new ways to integiate computeis into pioduction piocesses, foi example, has dia-
matically incieased the demand foi capital in the last few yeais. Many univeisities aie adding new
classioom buildings oi ienovating old ones so they can bettei use computeis in instiuction, and busi-
nesses use computeis in neaily eveiy facet of opeiations.
Changing Demand for Goods and Services
Ultimately, the souice of demand foi factois of pioduction is the demand foi the goods and seivices
pioduced by those factois. Economists say that the demand foi a factoi is a deiived" de-
mand-deiived, that is, fiom the demand foi what the factoi pioduces. As population and incomes ex-
pand, we can expect gieatei demand foi goods and seivices, a change that will inciease the demand foi
capital.
Changes in ReIative Iactor Prices
Fiims achieve the gieatest possible output foi a given total cost by opeiating wheie the iatios of mai-
ginal pioduct to factoi piice aie equal foi all factois of pioduction. Foi a fim that uses laboi (L) and
capital (K), foi example, this iequiies that MP
L
/P
L
= MP
K
/P
K
, wheie MP
L
and MP
K
aie the maiginal
pioducts of laboi and capital, iespectively, and P
L
and P
K
aie the piices of laboi and capital, iespect-
ively. Suppose these equalities hold and the piice of laboi iises. The iatio of the maiginal pioduct of
laboi to its piice goes down, and the fim substitutes capital foi laboi. Similaily, an inciease in the piice
of capital, all othei things unchanged, would cause fims to substitute othei factois of pioduction foi
capital. The demand foi capital, theiefoie, would fall.
Changes in 7ax PoIicy
Goveinment can indiiectly afect the piice of capital thiough changes in tax policy. Foi example, sup-
pose the goveinment enacts an investment tax ciedit foi businesses, that is, a deduction of a ceitain
peicentage of theii spending on capital fiom theii piofts befoie paying taxes. Such a policy would
efectively lowei the piice of capital, causing fims to substitute capital foi othei factois of pioduction
and incieasing the demand foi capital. The iepeal of an investment tax ciedit would lead to a deciease
in the demand foi capital.
2.2 The Maiket foi Loanable Funds
When a fim decides to expand its capital stock, it can fnance its puichase of capital in seveial ways. It
might alieady have the funds on hand. It can also iaise funds by selling shaies of stock, as we discussed
in a pievious chaptei. When a fim sells stock, it is selling shaies of owneiship of the fim. It can boi-
iow the funds foi the capital fiom a bank. Anothei option is to issue and sell its own bonds. A bond is
a piomise to pay back a ceitain amount at a ceitain time. When a fim boiiows fiom a bank oi sells
bonds, of couise, it accepts a liability-it must make inteiest payments to the bank oi the owneis of its
bonds as they come due.
Regaidless of the method of fnancing chosen, a ciitical factoi in the fim's decision on whethei to
acquiie and hold capital and on how to fnance the capital is the inteiest iate. The iole of the inteiest
iate is obvious when the fim issues its own bonds oi boiiows fiom a bank. But even when the fim
uses its own funds to puichase the capital, it is foigoing the option of lending those funds diiectly to
othei fims by buying theii bonds oi indiiectly by putting the funds in bank accounts, theieby allowing
the banks to lend the funds. The inteiest iate gives the oppoitunity cost of using funds to acquiie capit-
al iathei than putting the funds to the best alteinative use available to the fim.
The inteiest iate is deteimined in a maiket in the same way that the piice of potatoes is deteim-
ined in a maiket: by the foices of demand and supply. The maiket in which boiioweis (demandeis of
funds) and lendeis (supplieis of funds) meet is the loanable funds market.
We will simplify oui model of the iole that the inteiest iate plays in the demand foi capital by ig-
noiing difeiences in actual inteiest iates that specifc consumeis and fims face in the economy. Foi
example, the inteiest iate on ciedit caids is highei than the moitgage iate of inteiest, and laige, estab-
lished companies can boiiow funds oi issue bonds at lowei inteiest iates than new, stait-up companies
can. Inteiest iates that fims face depend on a vaiiety of factois, such as iiskiness of the loan, the duia-
tion of the loan, and the costs of administeiing the loan. Howevei, since we will focus on geneial tend-
encies that cause inteiest iates to iise oi fall and since the vaiious inteiest iates in the economy tend to
move up and down togethei, the conclusions we ieach about the maiket foi loanable funds and how
fims and consumeis iespond to inteiest iate changes will still be valid.
CHAP7R 13 IN7RS7 RA7S AND 7H MARk7S IOR CAPI7AL AND NA7URAL RSOURCS 335
II GUR 13. 3 7he Demand and SuppIy
of LoanabIe Iunds
At |c.e. |rte.est ..tes, |.ns Jen.rJ nc.e
c.p|t.| .rJ t|e.e|c.e nc.e |c.r.b|e |urJs. |e
Jen.rJ |c. |c.r.b|e |urJs |s Jc.r...J
s|cp|rg. |e supp|y c| |c.r.b|e |urJs |s
gere..||y up...Js|cp|rg. |e e,u|||b.|un
|rte.est ..te,
|
, .||| be |curJ .|e.e t|e t.c
cu.ves |rte.sect.
saving
|rccne rct spert cr
ccrsunpt|cr.
dissaving
crsunpt|cr t|.t exceeJs
|rccne Ju.|rg . g|ver
pe.|cJ.
7he Demand for LoanabIe Iunds
In the pievious section we leained that a fim's decision to acquiie and keep capital depends on the net
piesent value of the capital in question, which in tuin depends on the inteiest iate. The lowei the in-
teiest iate, the gieatei the amount of capital that fims will want to acquiie and hold, since lowei in-
teiest iates tianslate into moie capital with positive net piesent values. The desiie foi moie capital
means, in tuin, a desiie foi moie loanable funds. Similaily, at highei inteiest iates, less capital will be
demanded, because moie of the capital in question will have negative net piesent values. Highei in-
teiest iates theiefoie mean less funding demanded.
Thus the demand foi loanable funds is downwaid-sloping, like the demand foi vii-
tually eveiything else, as shown in Figuie 13.3. The lowei the inteiest iate, the moie
capital fims will demand. The moie capital that fims demand, the gieatei the funding
that is iequiied to fnance it.
7he SuppIy of LoanabIe Iunds
Lendeis aie consumeis oi fims that decide that they aie willing to foigo some cuiient
use of theii funds in oidei to have moie available in the futuie. Lendeis supply funds to
the loanable funds maiket. In geneial, highei inteiest iates make the lending option
moie attiactive.
Foi consumeis, howevei, the decision is a bit moie complicated than it is foi fims.
In examining consumption choices acioss time, economists think of consumeis as hav-
ing an expected stieam of income ovei theii lifetimes. It is that expected income that
defnes theii consumption possibilities. The pioblem foi consumeis is to deteimine
when to consume this income. They can spend less of theii piojected income now and
thus have moie available in the futuie. Alteinatively, they can boost theii cuiient
spending by boiiowing against theii futuie income.
Saving is income not spent on consumption. (We shall ignoie taxes in this analys-
is.) Dissaving occuis when consumption exceeds income duiing a peiiod. Dissaving
means that the individual's saving is negative. Dissaving can be fnanced eithei by boi-
iowing oi by using past savings. Many people, foi example, save in piepaiation foi ie-
tiiement and then dissave duiing theii ietiiement yeais.
Saving adds to a household's wealth. Dissaving ieduces it. Indeed, a household's
wealth is the sum of the value of all past saving less all past dissaving.
We can think of saving as a choice to postpone consumption. Because inteiest
iates aie a payment paid to people who postpone theii use of wealth, inteiest iates aie a
kind of iewaid paid to saveis. Will highei inteiest iates encouiage the behavioi they ie-
waid: The answei is a iesounding maybe." Just as highei wages might not inciease the
quantity of laboi supplied, highei inteiest iates might not inciease the quantity of saving. The pioblem,
once again, lies in the fact that the income and substitution efects of a change in inteiest iates will pull
in opposite diiections.
Considei a hypothetical consumei, Tom Smith. Let us simplify the analysis of Mi. Smith's choices
conceining the timing of consumption by assuming that theie aie only two peiiods: the piesent peiiod
is peiiod 0, and the next is peiiod 1. Suppose the inteiest iate is 8 and his income in both peiiods is
expected to be $30,000.
Mi. Smith could, of couise, spend $30,000 in peiiod 0 and $30,000 in peiiod 1. In that case, his
saving equals zeio in both peiiods. But he has alteinatives. He could, foi example, spend moie than
$30,000 in peiiod 0 by boiiowing against his income foi peiiod 1. Alteinatively, he could spend less
than $30,000 in peiiod 0 and use his saving-and the inteiest he eains on that saving-to boost his
consumption in peiiod 1. If, foi example, he spends $20,000 in peiiod 0, his saving in peiiod 0 equals
$10,000. He will eain $800 inteiest on that saving, so he will have $40,800 to spend in the next peiiod.
Suppose the inteiest iate iises to 10. The inciease in the inteiest iate has boosted the piice of cui-
ient consumption. Now foi eveiy $1 he spends in peiiod 0 he gives up $1.10 in consumption in peiiod
1, instead of $1.08, which was the amount that would have been given up in consumption in peiiod 1
when the inteiest iate was 8. A highei piice pioduces a substitution efect that ieduces an activ-
ity-Mi. Smith will spend less in the cuiient peiiod due to the substitution efect. The substitution
efect of a highei inteiest iate thus boosts saving. But the highei inteiest iate also means that he eains
moie income on his saving. Consumption in the cuiient peiiod is a noimal good, so an inciease in in-
come can be expected to inciease cuiient consumption. But an inciease in cuiient consumption im-
plies a ieduction in saving. The income efect of a highei inteiest iate thus tends to ieduce saving.
Whethei Mi. Smith's saving will iise oi fall in iesponse to a highei inteiest iate depends on the ielative
stiengths of the substitution and income efects.
To see how an inciease in inteiest iates might ieduce saving, imagine that Mi. Smith has decided
that his goal is to have $40,800 to spend in peiiod 1. At an inteiest iate of 10, he can ieduce his saving
336 PRINCIPLS OI CONOMICS
below $10,000 and still achieve his goal of having $40,800 to spend in the next peiiod. The income
efect of the inciease in the inteiest iate has ieduced his saving, and consequently his desiie to supply
funds to the loanable funds maiket.
Because changes in inteiest iates pioduce substitution and income efects that pull saving in op-
posite diiections, we cannot be suie what will happen to saving if inteiest iates change. The combined
efect of all consumeis' and fims' decisions, howevei, geneially leads to an upwaid-sloping supply
cuive foi loanable funds, as shown in Figuie 13.3. That is, the substitution efect usually dominates the
income efect.
The equilibiium inteiest iate is deteimined by the inteisection of the demand and supply cuives in
the maiket foi loanable funds.
2.3 Capital and the Loanable Funds Maiket
If the quantity of capital demanded vaiies inveisely with the inteiest iate, and if the inteiest iate is de-
teimined in the loanable funds maiket, then it follows that the demand foi capital and the loanable
funds maiket aie inteiielated. Because the acquisition of new capital is geneially fnanced in the
loanable funds maiket, a change in the demand foi capital leads to a change in the demand foi loanable
funds-and that afects the inteiest iate. A change in the inteiest iate, in tuin, afects the quantity of
capital demanded on any demand cuive.
The ielationship between the demand foi capital and the loanable funds maiket thus goes both
ways. Changes in the demand foi capital afect the loanable funds maiket, and changes in the loanable
funds maiket can afect the quantity of capital demanded.
Changes in the Demand for CapitaI and the LoanabIe Iunds Market
Figuie 13.4 suggests how an incieased demand foi capital by fims will afect the loanable funds mai-
ket, and thus the quantity of capital fims will demand. In Panel (a) the initial inteiest iate is r
1
. At r
1
in
Panel (b) K
1
units of capital aie demanded (on cuive D
1
). Now suppose an impiovement in technology
incieases the maiginal pioduct of capital, shifting the demand cuive foi capital in Panel (b) to the iight
to D
2
. Fiims can be expected to fnance the incieased acquisition of capital by demanding moie
loanable funds, shifting the demand cuive foi loanable funds to D
2
in Panel (a). The inteiest iate thus
iises to r
2
. Consequently, in the maiket foi capital the demand foi capital is gieatei and the inteiest iate
is highei. The new quantity of capital demanded is K
2
on demand cuive D
2
.
II GUR 13. 4 LoanabIe Iunds and the Demand for CapitaI
|e |rte.est ..te |s Jete.n|reJ |r t|e |c.r.b|e |urJs n..ket, .rJ t|e ,u.rt|ty c| c.p|t.| Jen.rJeJ v..|es .|t| t|e
|rte.est ..te. |us, everts |r t|e |c.r.b|e |urJs n..ket .rJ t|e Jen.rJ |c. c.p|t.| ..e |rte..e|.teJ. || t|e Jen.rJ
|c. c.p|t.| |rc.e.ses tc |
2
|r |.re| (b), t|e Jen.rJ |c. |c.r.b|e |urJs |s ||ke|y tc |rc.e.se .s .e||. |.re| (.) s|c.s
t|e .esu|t |r t|e |c.r.b|e |urJs n..ket. s|||t |r t|e Jen.rJ cu.ve |c. |c.r.b|e |urJs |.cn |
1
tc |
2
.rJ .r
|rc.e.se |r t|e |rte.est ..te |.cn
1
tc
2
. At
2
, t|e ,u.rt|ty c| c.p|t.| Jen.rJeJ .||| be |
2
, .s s|c.r |r |.re| (b).
CHAP7R 13 IN7RS7 RA7S AND 7H MARk7S IOR CAPI7AL AND NA7URAL RSOURCS 337
Changes in the LoanabIe Iunds Market and the Demand for CapitaI
Events in the loanable funds maiket can also afect the quantity of capital fims will hold. Suppose, foi
example, that consumeis decide to inciease cuiient consumption and thus to supply fewei funds to the
loanable funds maiket at any inteiest iate. This change in consumei piefeiences shifts the supply cuive
foi loanable funds in Panel (a) of Figuie 13.3 fiom S
1
to S
2
and iaises the inteiest iate to r
2
. If theie is
no change in the demand foi capital D
1
, the quantity of capital fims demand falls to K
2
in Panel (b).
II GUR 13. 5 A Change in the LoanabIe Iunds Market and the Quantity of CapitaI Demanded
A c|.rge t|.t beg|rs |r t|e |c.r.b|e |urJs n..ket c.r .|ect t|e ,u.rt|ty c| c.p|t.| |.ns Jen.rJ. |e.e, . Jec.e.se
|r ccrsune. s.v|rg c.uses . s|||t |r t|e supp|y c| |c.r.b|e |urJs |.cn .
1
tc .
2
|r |.re| (.). Assun|rg t|e.e |s rc
c|.rge |r t|e Jen.rJ |c. c.p|t.|, t|e ,u.rt|ty c| c.p|t.| Jen.rJeJ |.||s |.cn |
1
tc |
2
|r |.re| (b).
Oui model of the ielationship between the demand foi capital and the loanable funds maiket thus as-
sumes that the inteiest iate is deteimined in the maiket foi loanable funds. Given the demand cuive foi
capital, that inteiest iate then deteimines the quantity of capital fims demand.
Table 13.2 shows that a change in the quantity of capital that fims demand can begin with a
change in the demand foi capital oi with a change in the demand foi oi supply of loanable funds. A
change in the demand foi capital afects the demand foi loanable funds and hence the inteiest iate in
the loanable funds maiket. The change in the inteiest iate leads to a change in the quantity of capital
demanded. Alteinatively, a change in the loanable funds maiket, which leads to a change in the inteiest
iate, causes a change in quantity of capital demanded.
7A8L 13. 2 7wo Routes to Changes in the Quantity of CapitaI Demanded
A c|.rge |r t|e ,u.rt|ty c| c.p|t.| t|.t |.ns Jen.rJ c.r beg|r .|t| . c|.rge |r t|e Jen.rJ |c. c.p|t.| c. .|t| .
c|.rge |r t|e Jen.rJ c. supp|y c| |c.r.b|e |urJs.
A change originating in the capitaI market A change originating in the IoanabIe funds market
1. A c|.rge |r t|e Jen.rJ |c. c.p|t.| |e.Js tc. 1. A c|.rge |r t|e Jen.rJ |c. c. supp|y c| |c.r.b|e
|urJs |e.Js tc .
2... c|.rge |r t|e Jen.rJ |c. |c.r.b|e |urJs,
.||c| |e.Js tc.
2... c|.rge |r t|e |rte.est ..te, .||c| |e.Js tc.
3... c|.rge |r t|e |rte.est ..te, .||c| |e.Js tc. 3... c|.rge |r t|e ,u.rt|ty c| c.p|t.| Jen.rJeJ.
4... c|.rge |r t|e ,u.rt|ty c| c.pt|.| Jen.rJeJ.
338 PRINCIPLS OI CONOMICS
k Y 7 A k A W A Y S
< |e ret p.esert v.|ue (||v) c| .r |rvestnert p.cect |s e,u.| tc t|e p.esert v.|ue c| |ts expecteJ .everues
n|rus t|e p.esert v.|ue c| |ts expecteJ ccsts. ||.ns .||| ..rt tc urJe.t.ke t|cse |rvestnerts |c. .||c|
t|e ||v |s g.e.te. t|.r c. e,u.| tc .e.c.
< |e Jen.rJ cu.ve |c. c.p|t.| s|c.s t|.t |.ns Jen.rJ . g.e.te. ,u.rt|ty c| c.p|t.| .t |c.e. |rte.est ..tes.
Ancrg t|e |c.ces t|.t c.r s|||t t|e Jen.rJ cu.ve |c. c.p|t.| ..e c|.rges |r expect.t|crs, c|.rges |r
tec|rc|cgy, c|.rges |r t|e Jen.rJs |c. gccJs .rJ se.v|ces, c|.rges |r .e|.t|ve |.ctc. p.|ces, .rJ c|.rges
|r t.x pc||cy.
< |e |rte.est ..te |s Jete.n|reJ |r t|e n..ket |c. |c.r.b|e |urJs. |e Jen.rJ cu.ve |c. |c.r.b|e |urJs |.s
. reg.t|ve s|cpe, t|e supp|y cu.ve |.s . pcs|t|ve s|cpe.
< |.rges |r t|e Jen.rJ |c. c.p|t.| .|ect t|e |c.r.b|e |urJs n..ket, .rJ c|.rges |r t|e |c.r.b|e |urJs
n..ket .|ect t|e ,u.rt|ty c| c.p|t.| Jen.rJeJ.
7 R Y I 7 !
Suppcse t|.t b.by bccne.s beccne |rc.e.s|rg|y ccrce.reJ .bcut .|et|e. c. rct t|e gcve.rnert .||| .e.||y
|.ve t|e |urJs tc n.ke Scc|.| Secu.|ty p.ynerts tc t|en cve. t|e|. .et|.enert ye..s. As . .esu|t, t|ey bccst
s.v|rg rc.. |c. .cu|J t|e|. Jec|s|crs .|ect t|e n..ket |c. |c.r.b|e |urJs .rJ t|e Jen.rJ cu.ve |c.
c.p|t.|`
Case in Point: 7he Net Present VaIue of an M8A
2010 jupiterimages Corporation
Ar |rvestnert |r |un.r c.p|t.| J||e.s ||tt|e |.cn .r |rvestnert |r c.p|t.|cre .c,u|.es .r .sset t|.t .|||
p.cJuce .JJ|t|cr.| |rccne cve. t|e |||e c| t|e .sset. Ores eJuc.t|cr p.cJucesc. |t c.r be expecteJ tc p.c
Juce.JJ|t|cr.| |rccne cve. cres .c.k|rg c..ee..
|cr.|J +e.p|e, . p.c|essc. .t t|e |r|ve.s|ty c| |cc|este. bus|ress sc|cc|, |.s est|n.teJ t|e ret p.esert v.|ue
(||v) c| .r VbA cbt.|reJ |.cn e.c| c| 20 tcp bus|ress sc|cc|s. |e ccsts c| .tterJ|rg e.c| sc|cc| |rc|uJeJ
tu|t|cr .rJ |c.gcre |rccne. c est|n.te t|e n..g|r.| .everue p.cJuct c| . Jeg.ee, V.. +e.p|e st..teJ .|t|
su.vey J.t. s|c.|rg .|.t g..Ju.tes c| e.c| sc|cc| .e.e e..r|rg |ve ye..s .|te. cbt.|r|rg t|e|. VbAs. |e
t|er est|n.teJ .|.t stuJerts .|t| t|e .b|||ty tc .tterJ t|cse sc|cc|s .cu|J |.ve beer e..r|rg .|t|cut .r
VbA. |e est|n.teJ n..g|r.| .everue p.cJuct |c. e.c| ye.. |s t|e J||e.erce bet.eer t|e s.|..|es stuJerts
e..reJ .|t| . Jeg.ee ve.sus .|.t t|ey .cu|J |.ve e..reJ .|t|cut |t. |e ||v |s t|er ccnputeJ us|rg
|,u.t|cr 13.5.
CHAP7R 13 IN7RS7 RA7S AND 7H MARk7S IOR CAPI7AL AND NA7URAL RSOURCS 339
|e est|n.tes g|ver |e.e s|c. t|e ||v c| .r VbA cve. t|e |.st sever ye..s c| .c.k .|te. .ece|v|rg t|e Jeg.ee.
|ey suggest t|.t .r VbA |.cn 15 c| t|e sc|cc|s ..rkeJ |s . gccJ |rvestnertbut t|.t . Jeg.ee .t t|e ct|e.
sc|cc|s n|g|t rct be. V.. +e.p|e s.ys t|.t exterJ|rg |rccne p.cect|crs beycrJ sever ye..s .cu|J rct s|g
r||c.rt|y .|ect t|e .r.|ys|s, bec.use p.esert v.|ues c| p.cecteJ |rccne J||e.ert|.|s .|t| .rJ .|t|cut .r
VbA beccne ve.y sn.||.
\|||e t|e +e.p|e stuJy |s scne.|.t J.teJ, . 2002 stuJy by St.r|c.J |r|ve.s|ty C..Ju.te Sc|cc| c| bus|ress
p.c|essc. 'e|.ey ||e|e. .rJ St.r|c.J ||.|. c.rJ|J.te |.|st|r. . |crg .ev|e.eJ 40 ye..s c| .ese..c| cr t||s
tcp|c .rJ .e.c|eJ t|e ccrc|us|cr t|.t, |c. t|e ncst p..t, t|e.e |s sc.rt ev|Jerce t|.t t|e VbA c.eJert|.|, p..
t|cu|..|y |.cn rcre||te sc|cc|s...e .e|.teJ tc e|t|e. s.|..y c. t|e .tt.|rnert c| ||g|e. |eve| pcs|t|crs |r
c.g.r|..t|crs.
O| ccu.se, t|ese stuJ|es cr|y |rc|uJe |r.rc|.| .spects c| t|e |rvestnert .rJ J|J rct ccve. .ry psyc||c be
re|ts t|.t VbA .ec|p|erts n.y |rcu. |.cn nc.e |rte.est|rg .c.k c. p.est|ge.
SchooI Net present vaIue, rst 7 years of
work
SchooI Net present vaIue, rst 7 years of
work
|..v..J 148,38 \|.g|r|. 30,046
||c.gc 106,84 |..tncut| 22,509
St.r|c.J 9,462 V|c||g.r 21,502
V| 85,36 ..reg|e
Ve||cr
18,69
+.|e 83,5 ex.s 1,459
\|..tcr 59,486 |cc|este. 30
||A 55,088 |rJ|.r. 3,315
be.ke|ey 54,101 |+| 3,49
|c.t|.este.r 53,562 Scut| ..c||r. 4,565
c.re|| 30,84 |uke 1,631
.ooce |e /|^ o||ee|| ^o|,|.' |e |coo||. ^oo| 6 994. 5S o|e e||eJ .||| e||o |o||e eoJoc||o o||||eJ (/e
eeJ |o o|o| e||o |o oe ||| |o|e oo|: .e|e, ||e|e oJ ||||o |o. |e |J o| |o|e .c|oo|. |e .occe |o /ee| ||e |,e.'
^coJe, o| /ooee| |eo| oJ |Joco||o (.e|ee 2002: .S95
A N S W R 7 O 7 R Y I 7 ! P R O 8 L M
Ar |rc.e.se |r s.v|rg .t e.c| |rte.est ..te |np||es . .|g|t...J s|||t |r t|e supp|y cu.ve c| |c.r.b|e |urJs. As .
.esu|t, t|e e,u|||b.|un |rte.est ..te |.||s. \|t| t|e |c.e. |rte.est ..te, t|e.e |s ncvenert Jc.r...J tc t|e
.|g|t .|crg t|e Jen.rJ|c.c.p|t.| cu.ve, .s s|c.r.
340 PRINCIPLS OI CONOMICS
4. REVIEW AND PRACTICE
Summary
|ne |s t|e ccnp||c.t|rg |.ctc. .|er .e .r.|y.e c.p|t.| .rJ r.tu..| .escu.ces. bec.use cu..ert c|c|ces .|ect
t|e |utu.e stccks c| bct| .escu.ces, .e nust t.ke t|cse |utu.e ccrse,uerces |rtc .cccurt. ArJ bec.use .
p.ynert |r t|e |utu.e |s .c.t| |ess t|.r .r e,u.| p.ynert tcJ.y, .e reeJ tc ccrve.t t|e Jc||.. v.|ue c| |u
tu.e ccrse,uerces tc p.esert v.|ue. \e Jete.n|re t|e p.esert v.|ue c| . |utu.e p.ynert by J|v|J|rg t|e
.ncurt c| t|.t p.ynert by (1 + )
r
, .|e.e |s t|e |rte.est ..te .rJ r |s t|e runbe. c| ye..s urt|| t|e p.ynert
.||| cccu.. |e p.esert v.|ue c| . g|ver |utu.e v.|ue |s sn.||e. .t ||g|e. v.|ues c| .rJ .t ||g|e. |rte.est ..tes.
|rte.est ..tes ..e Jete.n|reJ |r t|e n..ket |c. |c.r.b|e |urJs. |e Jen.rJ |c. |c.r.b|e |urJs |s Je.|veJ |.cn
t|e Jen.rJ |c. c.p|t.|. At |c.e. |rte.est ..tes, t|e ,u.rt|ty c| c.p|t.| Jen.rJeJ |rc.e.ses. ||s, |r tu.r, |e.Js
tc .r |rc.e.se |r t|e Jen.rJ |c. |c.r.b|e |urJs. |r t|e .gg.eg.te, t|e supp|y cu.ve c| |c.r.b|e |urJs |s ||ke|y
tc be up...Js|cp|rg.
\e .ssune t|.t |.ns Jete.n|re .|et|e. tc .c,u|.e .r .JJ|t|cr.| ur|t c| c.p|t.| by (||v: c| t|e .sset. \|er
||v e,u.|s .e.c, t|e p.esert v.|ue c| c.p|t.|s n..g|r.| .everue p.cJuct e,u.|s t|e p.esert v.|ue c| |ts n..
g|r.| |.ctc. ccst. |e Jen.rJ cu.ve |c. c.p|t.| s|c.s t|e ,u.rt|ty c| c.p|t.| Jen.rJeJ .t e.c| |rte.est ..te.
Ancrg t|e |.ctc.s t|.t s|||t t|e Jen.rJ cu.ve |c. c.p|t.| ..e c|.rges |r expect.t|crs, re. tec|rc|cgy,
c|.rge |r Jen.rJs |c. gccJs .rJ se.v|ces, .rJ c|.rge |r .e|.t|ve |.ctc. p.|ces.
V..kets |c. r.tu..| .escu.ces ..e J|st|rgu|s|eJ .ccc.J|rg tc .|et|e. t|e .escu.ces ..e ex|.ust|b|e c. .ere.
.b|e. O.re.s c| r.tu..| .escu.ces |.ve .r |rcert|ve tc ccrs|Je. |utu.e .s .e|| .s p.esert Jen.rJs |c. t|ese .e
scu.ces. |.rJ, .|er |t |.s . ve.t|c.| supp|y cu.ve, gere..tes . .etu.r t|.t ccrs|sts ert|.e|y c| .ert. |r gere..|,
eccrcn|c .ert |s .etu.r tc . .escu.ce |r excess c| t|e n|r|nun p.|ce recess..y tc n.ke t|.t .escu.ce
.v.||.b|e.
C O N C P 7 P R O 8 L M S
1. |e c|..g|rg c| |rte.est ..tes |s c|ter v|e.eJ .|t| ccrtenpt. |c |rte.est ..tes se.ve .ry use|u| pu.pcse`
2. |c. Jces .r |rc.e.se |r |rte.est ..tes .|ect t|e p.esert v.|ue c| . |utu.e p.ynert`
3. |c. Jces .r |rc.e.se |r t|e s|.e c| . |utu.e p.ynert .|ect t|e p.esert v.|ue c| t|e |utu.e p.ynert`
4. .c p.ynerts c| 1,000 ..e tc be n.Je. Ore c| t|en .||| be p.|J cre ye.. |.cn tcJ.y .rJ t|e ct|e. .|||
be p.|J t.c ye..s |.cn tcJ.y. \||c| |.s t|e g.e.te. p.esert v.|ue` \|y`
5. |e ess.y cr t|e v|.t|c.| sett|enerts |rJust.y suggests t|.t |rvestc.s p.y cr|y 80 c| t|e |.ce v.|ue c| .
|||e |rsu..rce pc||cy t|.t |s expecteJ tc be p.|J c| |r s|x ncrt|s. \|y` \cu|J |t rct be |.|.e. || |rvestc.s
p.|J t|e |u|| v.|ue`
6. |c. .cu|J e.c| c| t|e |c||c.|rg everts .|ect t|e Jen.rJ cu.ve |c. c.p|t.|`
.. A p.cspect|ve cut |r t.xes |npcseJ cr bus|ress |.ns
b. A .eJuct|cr |r t|e p.|ce c| |.bc.
c. Ar |np.cvenert |r tec|rc|cgy t|.t |rc.e.ses c.p|t.|s n..g|r.| p.cJuct
J. Ar |rc.e.se |r |rte.est ..tes
. || Jeve|cpeJ .rJ n.Je p..ct|c.|, |us|cr tec|rc|cgy .cu|J .||c. t|e p.cJuct|cr c| v|.tu.||y ur||n|teJ
,u.rt|t|es c| c|e.p, pc||ut|cr|.ee ere.gy. Scne sc|ert|sts p.eJ|ct t|.t t|e tec|rc|cgy |c. |us|cr .||| be
Jeve|cpeJ .|t||r t|e rext |e. Jec.Jes. |c. Jces .r expect.t|cr t|.t |us|cr .||| be Jeve|cpeJ .|ect t|e
n..ket |c. c|| tcJ.y`
8. |s t|e .ert p.|J |c. .r .p..tnert eccrcn|c .ert` |xp|.|r.
9. |||n J|.ectc. b.ett |.tre. (|us| |cu., A|te. t|e Surset, .rJ ct|e.s) ccnnerteJ tc . |e. `o| |e
(|cvenbe. 13, 2004, p. A19) .epc.te. t|.t, || |e .e.ert . J|.ectc., V.. |.tre. s.|J |e .cu|J su.e|y be
t.k|rg c.Je.s .t Vc|cr.|Js. |c. nuc| eccrcn|c .ert |s V.. |.tre. ||ke|y e..r|rg`
10. Suppcse ycu c.r . ..rc|, .rJ t|.t ccnne.c|.| .rJ .es|Jert|.| Jeve|cpnert st..t tc t.ke p|.ce ..curJ
ycu. ..rc|. |c. .||| t||s .|ect t|e v.|ue c| ycu. p.cpe.ty` \|.t .||| |.pper tc t|e ,u.rt|ty c| |.rJ` \|.t
k|rJ c| .etu.r .||| ycu e..r`
11. |xp|.|r .|y ||g|e. |rte.est ..tes terJ tc |rc.e.se t|e cu..ert use c| r.tu..| .escu.ces.
CHAP7R 13 IN7RS7 RA7S AND 7H MARk7S IOR CAPI7AL AND NA7URAL RSOURCS 347
N U M R I C A L P R O 8 L M S
|se t|e t.b|es be|c. tc .rs.e. |.cb|ens 15. |e |.st t.b|e g|ves t|e p.esert v.|ue c| 1 .t t|e erJ c| J||e.
ert t|ne pe.|cJs, g|ver J||e.ert |rte.est ..tes. |c. ex.np|e, .t .r |rte.est ..te c| 10, t|e p.esert v.|ue c| 1
tc be p.|J |r 20 ye..s |s 0.149. At 10 |rte.est, t|e p.esert v.|ue c| 1,000 tc be p.|J |r 20 ye..s e,u.|s
1,000 t|nes 0.149, c. 149. |e seccrJ t.b|e g|ves t|e p.esert v.|ue c| . st.e.n c| p.ynerts c| 1 tc be
n.Je .t t|e erJ c| e.c| pe.|cJ |c. . g|ver runbe. c| pe.|cJs. |c. ex.np|e, .t 10 |rte.est, t|e p.esert v.|ue
c| . se.|es c| 1 p.ynerts, n.Je .t t|e erJ c| e.c| ye.. |c. t|e rext 10 ye..s, |s 6.145. |s|rg t|.t s.ne |r
te.est ..te, t|e p.esert v.|ue c| . se.|es c| 10 p.ynerts c| 1,000 e.c| |s 1,000 t|nes 6.145, c. 6,145.
Present VaIue of $1 to 8e Received at the nd of a Given Number of Periods
Percent Interest
PerloJ 2 4 8 !0 !2 !4 ! !8 20
1 0.980 0.962 0.943 0.926 0.909 0.893 0.8 0.862 0.84 0.833
2 0.961 0.925 0.890 0.85 0.826 0.9 0.69 0.43 0.18 0.694
3 0.942 0.889 0.840 0.94 0.51 0.12 0.65 0.641 0.609 0.59
4 0.924 0.855 0.92 0.35 0.683 0.636 0.592 0.552 0.515 0.442
5 0.906 0.822 0.4 0.681 0.621 0.56 0.519 0.46 0.43 0.402
10 0.820 0.66 0.558 0.463 0.386 0.322 0.20 0.22 0.191 0.162
15 0.43 0.555 0.41 0.315 0.239 0.183 0.140 0.180 0.084 0.065
20 0.63 0.456 0.312 0.215 0.149 0.104 0.03 0.051 0.03 0.026
25 0.610 0.35 0.233 0.146 0.092 0.059 0.038 0.024 0.016 0.010
40 0.453 0.208 0.09 0.046 0.022 0.011 0.005 0.003 0.001 0.001
50 0.32 0.141 0.054 0.021 0.009 0.003 0.001 0.001 0 0
Present VaIue of $1 to 8e Received at the nd of ach Period for a Given Number of Periods
Percent Interest
PerloJ 2 4 8 !0 !2 !4 ! !8 20
1 0.980 0.962 0.943 0.926 0.909 0.893 0.8 0.862 0.84 0.833
2 1.942 1.886 1.833 1.83 1.36 1.690 1.64 1.605 1.566 1.528
3 2.884 2.5 2.63 2.5 2.48 2.402 2.322 2.246 2.14 2.106
4 3.808 3.630 3.465 3.312 3.10 3.03 2.910 2.98 2.690 2.589
5 4.13 4.452 4.212 3.993 3.91 3.605 3.433 3.24 3.12 2.991
10 8.983 8.111 .360 6.10 6.145 5.650 5.216 4.833 4.494 4.192
15 12.849 11.18 9.12 8.559 .606 6.811 6.142 5.55 5.092 4.65
20 16.351 13.590 11.40 9.818 8.514 .469 6.623 5.929 5.353 4.80
25 19.523 15.622 12.83 10.65 9.0 .843 6.83 6.09 5.46 4.948
30 22.396 1.292 13.65 11.258 9.42 8.055 .003 6.1 5.51 4.99
40 2.355 19.93 15.046 11.925 9.9 8.244 .105 6.233 5.548 4.99
50 31.424 21.482 15.62 12.233 9.915 8.304 .133 6.246 5.554 4.999
1. +cu. |rc|e A.t|u., rct tc be cutJcre by Aurt ..ner, c|e.s ycu . c|c|ce. +cu c.r |.ve 10,000 rc. c.
30,000 |r 15 ye..s. || ycu tcck t|e p.ynert rc., ycu ccu|J put |t |r . bcrJ |urJ c. b.rk .cccurt e..r|rg
8 |rte.est. |se p.esert v.|ue .r.|ys|s tc Jete.n|re .||c| .|te.r.t|ve |s bette..
2. |enenbe. ..c| Ste|rs t..ctc.` \e s.. t|.t .t .r |rte.est ..te c| , . Jec|s|cr tc pu.c|.se t|e t..ctc.
.cu|J p.y c|, |ts ret p.esert v.|ue |s pcs|t|ve. Suppcse t|e t..ctc. |s st||| expecteJ tc y|e|J 20,000 |r ret
.everue pe. ye.. |c. e.c| c| t|e rext 5 ye..s .rJ tc se|| .t t|e erJ c| 5 ye..s |c. 22,000, .rJ t|e pu.c|.se
p.|ce c| t|e t..ctc. st||| e,u.|s 95,000. |se .b|es (.) .rJ (b) tc ccnpute t|e ret p.esert v.|ue c| t|e
t..ctc. .t .r |rte.est ..te c| 8.
3. V..k 'cres |s t||rk|rg .bcut gc|rg tc cc||ege. || |e gces, |e .||| e..r rct||rg |c. t|e rext |cu. ye..s .rJ,
|r .JJ|t|cr, .||| |.ve tc p.y tu|t|cr .rJ |ees tct.||rg 10,000 pe. ye... |e .|sc .cu|J rct e..r t|e 25,000
pe. ye.. |e ccu|J n.ke by .c.k|rg |u|| t|ne Ju.|rg t|e rext |cu. ye..s. A|te. ||s |cu. ye..s c| cc||ege, |e
expects t|.t ||s |rccne, bct| .|||e .c.k|rg .rJ |r .et|.enert, .||| be 20,000 pe. ye.. nc.e, cve. t|e
348 PRINCIPLS OI CONOMICS
rext 50 ye..s, t|.r |t .cu|J |.ve beer |.J |e rct .tterJeJ cc||ege. S|cu|J |e gc tc cc||ege` Assune
t|.t e.c| p.ynert |c. cc||ege .rJ Jc||.. c| |rccne e..reJ cccu. .t t|e erJ c| t|e ye..s |r .||c| t|ey
cccu.. |grc.e pcss|b|e |rccne t.xes |r n.k|rg ycu. c.|cu|.t|crs. |ec|Je .|et|e. ycu s|cu|J .tterJ
cc||ege, .ssun|rg e.c| c| t|e |c||c.|rg |rte.est ..tes.
.. 2
b. 4
c. 6
J. 8
4. A re. |e.|t| c|ub |.s ust cpereJ up |r ycu. tc.r. St.ugg||rg tc b.|rg |r ncrey rc., t|e c|ub |s c|e.|rg
10ye.. nenbe.s||ps |c. . cret|ne p.ynert rc. c| 800. +cu c.rrct be su.e t|.t ycu .||| st||| be |r
tc.r |c. t|e rext 10 ye..s, but ycu expect t|.t ycu .||| be. +cu .rt|c|p.te t|.t ycu. bere|t c| be|crg|rg
tc t|e c|ub .||| be 10 pe. ncrt| (t||rk c| t||s .s .r .rru.| bere|t c| 120). |ec|Je .|et|e. ycu s|cu|J
c|r .t e.c| c| t|e |c||c.|rg |rte.est ..tes.
.. 2
b. 4
c. 6
J. 8
5. +cu |.ve ust pu.c|.seJ . re. |cne. |c ncrey ..s .e,u|.eJ .s . Jc.r p.ynert, ycu .||| be n.k|rg
p.ynerts c| 2,000 pe. ncrt| (t||rk c| t|ese .s .rru.| p.ynerts c| 24,000) |c. t|e rext 30 ye..s.
|ete.n|re t|e p.esert v.|ue c| ycu. |utu.e p.ynerts .t e.c| c| t|e |c||c.|rg |rte.est ..tes.
.. 2
b. 4
c. 6
J. 8
6. +cu c.r seve..| b...e|s c| .|re, cve. t|e ye..s, t|e v.|ue c| t||s .|re |.s .|ser .t .r .ve..ge ..te c| 10
pe. ye... |t |s expecteJ tc ccrt|rue tc .|se |r v.|ue, but .t . s|c.e. .rJ s|c.e. ..te. Assun|rg ycu. gc.| |s
tc n.x|n|.e ycu. .everue |.cn t|e .|re, .t .|.t pc|rt .||| ycu se|| |t`
. +cu |.ve beer g|ver . cc|r cc||ect|cr. +cu |.ve rc pe.scr.| |rte.est |r cc|rs, ycu. cr|y |rte.est |s tc n.ke
ncrey |.cn |t. +cu est|n.te t|.t t|e cu..ert v.|ue c| t|e cc||ect|cr |s 10,000. +cu ..e tc|J t|e cc|rs ..e
||ke|y tc .|se |r v.|ue cve. t|ne by 5 pe. ye... \|.t s|cu|J ycu Jc .|t| t|e cc||ect|cr` Or .|.t |.ctc.s
Jces ycu. .rs.e. JeperJ`
8. |e .se |r |c|rt cr t|e |rc.e.s|rg sc..c|ty c| c|| suggesteJ t|.t t|e ||u..|s ccnp|ex |s expecteJ tc .JJ
1.2 n||||cr b...e|s tc .c.|J c|| p.cJuct|cr by 2009. Suppcse t|.t .c.|J p.cJuct|cr t|.t ye.. .|.t
ct|e..|se be 8 n||||cr b...e|s pe. J.y. Assune t|.t t|e p.|ce e|.st|c|ty c| Jen.rJ |c. c|| |s 0.5. by |c.
nuc| .cu|J ycu expect t|e .JJ|t|cr c| c|| |.cn t|e ||u..|s ccnp|ex tc .eJuce t|e .c.|J c|| p.|ce`
CHAP7R 13 IN7RS7 RA7S AND 7H MARk7S IOR CAPI7AL AND NA7URAL RSOURCS 349
1.
ENDNOTES
|ete. b. |e Se|J|rg, e|eJes|c ||.ys |ts |.st ..J, |e.ves t|e C.ne, .oce |e. |o|
e |eo| cr||re, 'u|y , 2005.
350 PRINCIPLS OI CONOMICS
| A | | | 2 0
Macroeconomics: The Big
Picture
S7AR7 UP: IINANCIAL CRISIS 8A77RS CONOMY
|e |.S. eccrcny seeneJ tc be Jc|rg .e|| cve..|| .|te. t|e .ecess|cr c| 2001. C.c.t| |.J beer |.|.|y ..p|J, uren
p|cynert |.J st.yeJ |c., .rJ |r|.t|cr seeneJ tc be urJe. ccrt.c|. |e eccrcny beg.r tc ur..ve| .t t|e erJ c|
200tct.| cutput |e|| |r t|e |cu.t| ,u..te.. |t .eccve.eJb..e|y|r t|e |.st ,u..te., .rJ p|ckeJ up subst.rt|.||y
|r t|e seccrJ. |er t||rgs .ert scu..ve.y scu.. |e eccrcn|es c| t|e |r|teJ St.tes .rJ t|cse c| nuc| c| t|e
.c.|J .e.e .cckeJ by t|e .c.st |r.rc|.| c.|s|s |r re..|y 80 ye..s. |.t c.|s|s p|urgeJ t|e eccrcny |rtc . Jc.rtu.r
|r tct.| cutput .rJ enp|cynert t|.t seeneJ ||ke|y tc |.st . |crg t|ne.
A gccJ Je.| c| t|e eccrcnys ncnertun .|er t||rgs .e.e gc|rg .e|| |.J beer |ue|eJ by .|s|rg |cuse
p.|ces. bet.eer 1995 .rJ 200, |cus|rg p.|ces |r t|e |r|teJ St.tes nc.e t|.r Jcub|eJ. As |cuse p.|ces .cse, ccr
sune.s .|c c.reJ |cuses g.e. .|c|e. .rJ |rc.e.seJ t|e|. ccrsunpt|cr pu.c|.ses. |.t |e|peJ |ue| eccrcn|c
g.c.t|. |e bccn |r |cus|rg p.|ces |.J beer erccu..geJ by pc||c|es c| t|e r.t|crs ncret..y .ut|c.|ty, t|e
|eJe..| |ese.ve, .||c| |.J s|||teJ tc .r exp.rs|cr..y ncret..y pc||cy t|.t |e|J s|c.tte.n |rte.est ..tes be|c.
t|e |r|.t|cr ..te. Arct|e. Jeve|cpnert, subp.|ne nc.tg.gesnc.tg.ge |c.rs tc buye.s .|cse c.eJ|t c. |rccne
.cu|J rct c.J|r..||y ,u.|||y |c. nc.tg.ge |c.rs|e|peJ b.|rg cr t|e u|t|n.te cc||.pse. \|er t|ey .e.e |.st Je
ve|cpeJ, subp.|ne nc.tg.ge |c.rs seeneJ . |uge|y p.c|t.b|e |rvestnert |c. b.rks .rJ . gccJ Je.| |c. |cne
buye.s. ||r.rc|.| |rst|tut|crs Jeve|cpeJ . .|Je ..rge c| |rst.unerts b.seJ cr nc.tg.geb.ckeJ secu.|t|es. As
|crg .s |cuse p.|ces kept .|s|rg, t|e systen .c.keJ .rJ ..s p.c|t.b|e |c. v|.tu.||y .|| p|.ye.s |r t|e nc.tg.ge n..
ket. V.ry |.ns urJe.tcck |rvestnerts |r nc.tg.geb.ckeJ secu.|t|es t|.t .ssuneJ |cuse p.|ces .cu|J keep
.|s|rg. |..ge |rvestnert b.rks bet |e.v||y t|.t |cuse p.|ces .cu|J ccrt|rue .|s|rg. |c.e.|u| nenbe.s c| crg.ess
p.essu.eJ t.c gcve.rnertspcrsc.eJ erte.p.|ses, |.rr|e V.e (t|e |eJe..| |.t|cr.| Vc.tg.ge Asscc|.t|cr) .rJ
|.eJJ|e V.c (t|e |eJe..| |cne |c.r Vc.tg.ge c.pc..t|cr), tc be ever nc.e .gg.ess|ve |r erccu..g|rg b.rks tc
n.ke nc.tg.ge |c.rs tc |c.|rccne |.n|||es. |e p.essu.e c.ne |.cn t|e execut|ve b..rc| c| gcve.rnert .s
.e||urJe. bct| |encc..t|c .rJ |epub||c.r .Jn|r|st..t|crs. |r 1996, t|e |ep..tnert c| |cus|rg .rJ |.b.r
|eve|cpnert (urJe. b||| ||rtcr, . |encc..t) .e,u|.eJ t|.t 12 c| nc.tg.ges pu.c|.seJ by |.rr|e V.e .rJ |.eJ
J|e V.c be |c. |cuse|c|Js .|t| |rccnes |ess t|.r 60 c| t|e neJ|.r |rccne |r t|e|. .eg|cr. |.t t..get ..s |r
c.e.seJ tc 20 |r 2000, 22 |r 2005 (rc. urJe. Cec.ge \. bus|, . |epub||c.r), .rJ ..s tc |.ve |rc.e.seJ tc 28
|r 2008.
1
but t|.t |r.| t..get .cu|J rct be .e.c|eJ, .s bct| |.rr|e V.e .rJ |.eJJ|e V.c .e.e se|.eJ by t|e gcv
e.rnert |r 2008. c tcp t||rgs c|, . |ccser|rg |r b.rk .rJ |rvestnert b.rk .egu|.t|crs g.ve |r.rc|.| |rst|tut|crs
g.e.te. |ee..y |r gc|rg cve.bc..J .|t| pu.c|.ses c| nc.tg.geb.ckeJ secu.|t|es. As |cuse p.|ces beg.r |.|||rg |r
200, . systen b.seJ cr t|e .ssunpt|cr t|ey .cu|J ccrt|rue .|s|rg beg.r tc ur..ve| ve.y |.st. |e |rvestnert
b.rk be.. Ste..rs .rJ |rsu..rce ccnp.ry Ane.|c.r |rte.r.t|cr.| C.cup (A|C) .e,u|.eJ n.ss|ve |r|us|crs c| |eJe..|
ncrey tc keep t|en .|c.t. |r Septenbe. c| 2008, |.n .|te. |.n .|t| .ssets t|eJ tc nc.tg.geb.ckeJ secu.|t|es
beg.r tc |.||. |r scne c.ses, t|e gcve.rnert .escueJ t|en, |r ct|e. c.ses, suc| .s |e|n.r b.ct|e.s, t|ey .e.e .|
|c.eJ tc |.||.
|e |r.rc|.| c.|s|s |.J J..n.t|c .rJ |nneJ|.te e|ects cr t|e eccrcny. |e eccrcnys tct.| cutput, .||c|
|.J beer g.c.|rg t|.cug| t|e |.st |.|| c| 2008, |e|| .t .r .rru.| ..te c| 0.5 |r t|e t||.J ,u..te., .ccc.J|rg tc .J
v.rce est|n.tes by t|e bu.e.u c| |ccrcn|c Ar.|ys|s. crsune.s, |.v|rg .e.t|e.eJ ||g|e. g.sc||re p.|ces .rJ
||g|e. |ccJ p.|ces |c. ncst c| t|e ye.., .eJuceJ t|e|. ccrsunpt|cr experJ|tu.es .s t|e v.|ue c| t|e|. |cuses .rJ
t|e stccks t|ey |e|J p|urgeJccrsunpt|cr |e|| .t .r .rru.| ..te c| 3. |r t|e t||.J ,u..te.. As cc|J |e.. g.|ppeJ
|r.rc|.| n..kets .rJ expect.t|crs c| |u.t|e. s|c.Jc.r ersueJ, |.ns cut Jc.r cr |rvestnert sperJ|rg, .||c| |r
c|uJes sperJ|rg cr p|.rt .rJ e,u|pnert useJ |r p.cJuct|cr. \|||e t||s rcr.es|Jert|.| |rvestnert ccnpcrert c|
cutput |e|| .t .r .rru.| ..te c| 1.5, t|e .es|Jert|.| ccnpcrert |e|| ever |.ste. .s |cus|rg |rvestnert s.rk .t .r
.rru.| ..te c| 1.6. Ccve.rnert pu.c|.ses .rJ ret expc.ts .cse, but rct ercug| tc c|set .eJuct|crs |r ccr
sunpt|cr .rJ p.|v.te |rvestnert.
2
As cutput s|..rk, urenp|cynert .cse. |.cug| t|e |.st r|re ncrt|s c| 2008
t|e.e ..s ccrce.r t|.t p.|ce |eve|s |r t|e |r|teJ St.tes .rJ |r ncst c| t|e .c.|J eccrcn|es .e.e .|s|rg ..p|J|y, but
tc...J t|e erJ c| t|e ye.. t|e ccrce.r s|||teJ tc .|et|e. c. rct t|e p.|ce |eve| n|g|t |.||.
Output, enp|cynert, .rJ t|e p.|ce |eve| ..e t|e key v..|.b|es |r t|e stuJy c| n.c.ceccrcn|cs, .||c| |s t|e
.r.|ys|s c| .gg.eg.te v.|ues c| eccrcn|c v..|.b|es. \|.t Jete.n|res . ccurt.ys cutput, .rJ .|y Jces cutput |r
scne eccrcn|es exp.rJ .|||e |r ct|e.s |t ccrt..cts` \|y Jc scne eccrcn|es g.c. |.ste. t|.r ct|e.s` \|.t
c.uses p.|ces t|.cug|cut .r eccrcny tc |uctu.te, .rJ |c. Jc suc| |uctu.t|crs .|ect pecp|e` \|.t c.uses en
p|cynert .rJ urenp|cynert` \|y Jces . ccurt.ys urenp|cynert ..te |uctu.te` \|y Jc J||e.ert ccurt.|es
|.ve J||e.ert urenp|cynert ..tes`
\e .cu|J p.crcurce .r eccrcny |e.|t|y || |ts .rru.| cutput c| gccJs .rJ se.v|ces .e.e g.c.|rg .t . ..te
|t c.r sust.|r, |ts p.|ce |eve| st.b|e, .rJ |ts urenp|cynert ..te |c.. \|.t .cu|J ccrst|tute gccJ runbe.s |c.
e.c| c| t|ese v..|.b|es JeperJs cr t|ne .rJ p|.ce, but t|cse ..e t|e cutccnes t|.t ncst pecp|e .cu|J .g.ee ..e
Jes|..b|e |c. t|e .gg.eg.te eccrcny. \|er t|e eccrcny Jev|.tes |.cn .|.t |s ccrs|Je.eJ gccJ pe.|c.n.rce,
t|e.e ..e c|ter c.||s |c. t|e gcve.rnert tc Jc scnet||rg tc |np.cve pe.|c.n.rce. |c. gcve.rnert pc||c|es
.|ect eccrcn|c pe.|c.n.rce |s . n.c. tcp|c c| n.c.ceccrcn|cs. \|er t|e |r.rc|.| .rJ eccrcn|c c.|ses st.uck
t|.cug|cut t|e .c.|J |r 2008, t|e.e ..s n.ss|ve |rte.vert|cr |.cn .c.|J cert..| b.rks .rJ |.cn gcve.rnerts
t|.cug|cut t|e .c.|J |r .r e|c.t tc st|nu|.te t|e|. eccrcn|es.
||s c|.pte. p.cv|Jes . p.e||n|r..y sketc| c| t|e ncst |npc.t.rt n.c.ceccrcn|c |ssues. g.c.t| c| tct.| cut
put .rJ t|e bus|ress cyc|e, c|.rges |r t|e p.|ce |eve|, .rJ urenp|cynert. C..pp||rg .|t| t|ese |ssues .||| be |n
pc.t.rt tc ycu rct cr|y |r ycu. exp|c..t|cr c| n.c.ceccrcn|cs but t|.cug|cut ycu. |||e.
496 PRINCIPLS OI CONOMICS
reaI GDP
|e tct.| v.|ue c| .|| |r.|
gccJs .rJ se.v|ces p.cJuceJ
Ju.|rg . p..t|cu|.. ye.. c.
pe.|cJ, .JusteJ tc e||n|r.te
t|e e|ects c| c|.rges |r
p.|ces.
nominaI GDP
|e tct.| v.|ue c| |r.| gccJs
.rJ se.v|ces |c. . p..t|cu|..
pe.|cJ v.|ueJ |r te.ns c|
p.|ces |c. t|.t pe.|cJ.
business cycIe
|e eccrcnys p.tte.r c|
exp.rs|cr, t|er ccrt..ct|cr,
t|er exp.rs|cr .g.|r.
expansion
A sust.|reJ pe.|cJ |r .||c|
.e.| C|| |s .|s|rg.
recession
A sust.|reJ pe.|cJ |r .||c|
.e.| C|| |s |.|||rg.
1. GROWTH OF REAL GDP AND BUSINESS CYCLES
L A R N I N G O 8 1 C 7 I V S
1. Dene reaI gross domestic product and expIain how its caIcuIation avoids both doubIe-count-
ing and the eects of changes in the price IeveI.
2. Identify the phases of a business cycIe.
3. ReIate business cycIes to the overaII Iong-run trend in reaI GDP in the United States.
To deteimine whethei the economy of a nation is giowing oi shiinking in size, economists use a meas-
uie of total output called ieal GDP. Real GDP, shoit foi ieal gioss domestic pioduct, is the total value
of all fnal goods and seivices pioduced duiing a paiticulai yeai oi peiiod, adjusted to eliminate the
efects of changes in piices. Let us bieak that defnition up into paits.
Notice that only fnal" goods and seivices aie included in GDP. Many goods and seivices aie pui-
chased foi use as inputs in pioducing something else. Foi example, a pizza pailoi buys foui to make
pizzas. If we counted the value of the foui and the value of the pizza, we would end up counting the
foui twice and thus oveistating the value of total pioduction. Including only fnal goods avoids
double-counting. If the foui is pioduced duiing a paiticulai peiiod but has not been sold, then it is a
fnal good" foi that peiiod and is counted.
We want to deteimine whethei the economy's output is giowing oi shiinking. If each fnal good oi
seivice pioduced, fiom hammeis to haiicuts, weie valued at its cuiient maiket piice, and then we weie
to add the values of all such items pioduced, we would not know if the total had changed because out-
put changed oi because piices changed oi both. The maiket value of all fnal goods and seivices pio-
duced can iise even if total output falls. To isolate the behavioi of total output only, we must hold
piices constant at some level. Foi example, if we measuie the value of basketball output ovei time using
a fxed piice foi valuing the basketballs, then only an inciease in the numbei of basketballs pioduced
could inciease the value of the contiibution made by basketballs to total output. By making such an ad-
justment foi basketballs and all othei goods and seivices, we obtain a value foi ieal GDP. In contiast,
nominal GDP, usually just iefeiied to as gioss domestic pioduct (GDP), is the total value of fnal
goods and seivices foi a paiticulai peiiod valued in teims of piices foi that peiiod. Real GDP fell in the
thiid quaitei of 2008. But, because the piice level in the United States was iising, nominal GDP iose
3.6.
We will save a detailed discussion of the computation of GDP foi anothei chaptei. In this section,
oui goal is to use the concept of ieal GDP to look at the business cycle-the economy's pattein of ex-
pansion, then contiaction, then expansion again-and at giowth of ieal GDP.
1.1 Phases of the Business Cycle
Figuie 20.1 shows a stylized pictuie of a typical business cycle. It shows that economies go thiough
peiiods of incieasing and decieasing ieal GDP, but that ovei time they geneially move in the diiection
of incieasing levels of ieal GDP. A sustained peiiod in which ieal GDP is iising is an expansion; a sus-
tained peiiod in which ieal GDP is falling is a recession. Typically, an economy is said to be in a ieces-
sion when ieal GDP diops foi two consecutive quaiteis, but in the United States, the iesponsibility of
defning piecisely when the economy is in iecession is left to the Business Cycle Dating Committee of
the National Buieau of Economic Reseaich (NBER). The Committee defnes a iecession as a
signifcant decline in economic activity spiead acioss the economy, lasting moie than a few months,
noimally visible in ieal GDP, ieal income, employment, industiial pioduction, and wholesale-ietail
sales."
[3]
CHAP7R 20 MACROCONOMICS: 7H 8IG PIC7UR 497
II GUR 20. 1 Phases of the 8usiness
CycIe
|e bus|ress cyc|e |s . se.|es c| exp.rs|crs .rJ
ccrt..ct|crs |r .e.| C||. |e cyc|e beg|rs .t .
pe.k .rJ ccrt|rues t|.cug| . .ecess|cr, .
t.cug|, .rJ .r exp.rs|cr. A re. cyc|e beg|rs
.t t|e rext pe.k. |e.e, t|e |.st pe.k cccu.s .t
t|ne |
1
, t|e t.cug| .t t|ne |
2
, .rJ t|e rext
pe.k .t t|ne |
3
. |ct|ce t|.t t|e.e |s . terJercy
|c. .e.| C|| tc .|se cve. t|ne.
peak
|e pc|rt c| t|e bus|ress
cyc|e .t .||c| .r exp.rs|cr
erJs .rJ . .ecess|cr beg|rs.
trough
|e pc|rt c| t|e bus|ress
cyc|e .t .||c| . .ecess|cr
erJs .rJ .r exp.rs|cr
beg|rs.
At time t
1
in Figuie 20.1, an expansion ends and ieal GDP tuins downwaid. The
point at which an expansion ends and a iecession begins is called the peak of the busi-
ness cycle. Real GDP then falls duiing a peiiod of iecession. Eventually it staits upwaid
again (at time t
2
). The point at which a iecession ends and an expansion begins is called
the trough of the business cycle. The expansion continues until anothei peak is
ieached at time t
3
.
[4]
A complete business cycle is defned by the passage fiom one peak
to the next.
Because the Business Cycle Dating Committee dates peaks and tioughs by specifc
months, and because ieal GDP is estimated only on a quaiteily basis by the Buieau of
Economic Analysis, the committee ielies on a vaiiety of othei indicatois that aie pub-
lished monthly, including ieal peisonal income, employment, industiial pioduction,
and ieal wholesale and ietail sales. The committee typically deteimines that a iecession
has happened long aftei it has actually begun and sometimes ended! In laige pait, that
avoids pioblems when data ieleased about the economy aie ievised, and the committee
avoids having to ieveise itself on its deteimination of when a iecession begins oi ends,
something it has nevei done. In Decembei 2008, the Committee announced that a ie-
cession in the United States had begun in Decembei 2007. Inteiestingly, ieal GDP fell
in the fouith quaitei of 2007, giew in the fist and second quaiteis of 2008, and shiank
in the thiid quaitei of 2008, so cleaily the Committee was not using the two consecut-
ive quaiteis of declining GDP iule-of-thumb. Rathei, it was taking into account the be-
havioi of a vaiiety of othei vaiiables, such as employment and peisonal income.
1.2 Business Cycles and the Giowth of Real GDP in the
United States
Figuie 20.2 shows movements in ieal GDP in the United States fiom 1960 to 2008.
Ovei those yeais, the economy expeiienced eight iecessions (including the cuiient
one), shown by the shaded aieas in the chait. Although peiiods of expansion have been
moie piolonged than peiiods of iecession, we see the cycle of economic activity that chaiacteiizes eco-
nomic life.
II GUR 20. 2 xpansions and Recessions, 1960-2008
|e c|..t s|c.s ncvenerts |r .e.| C|| s|rce 1960. |ecess|crspe.|cJs c| |.|||rg .e.| C||..e s|c.r .s s|.JeJ
..e.s. Or .ve..ge, t|e .rru.| ..te c| g.c.t| c| .e.| C|| cve. t|e pe.|cJ ..s 3.3 pe. ye...
Source Bureau of Economic Analysis, ^IPA Table 1.1.. Real Gross Domestic Product, Chained Dollars jBillions of chained (2000) dollars]. Seasonally
adjusted at annual rates. Data for 2008 is through 3rd quarter.
Real GDP cleaily giew between 1960 and 2008. While the economy expeiienced expansions and ieces-
sions, its geneial tiend duiing the peiiod was one of iising ieal GDP. The aveiage annual iate of giowth
of ieal GDP was about 3.3.
Duiing the post-Woild Wai II peiiod, the aveiage expansion has lasted 37 months, and the avei-
age iecession has lasted about 10 months. The 2001 iecession, which lasted eight months, was thus
slightly shoitei than the aveiage. The cuiient iecession is alieady longei than the aveiage iecession.
498 PRINCIPLS OI CONOMICS
Economists have sought foi centuiies to explain the foices at woik in a business cycle. Not only aie
the cuiients that move the economy up oi down intellectually fascinating but also an undeistanding of
them is of tiemendous piactical impoitance. A business cycle is not just a movement along a cuive in a
textbook. It is new jobs foi people, oi the loss of them. It is new income, oi the loss of it. It is the funds
to build new schools oi to piovide bettei health caie-oi the lack of funds to do all those things. The
stoiy of the business cycle is the stoiy of piogiess and plenty, of failuie and saciifce.
If cuiient piojections that the economy is in foi a piolonged downtuin piove tiue, then the job
outlook foi new college giaduates will be giim. Alieady, students who giaduated in the class of 2008
expeiienced a veiy dimcult job maiket.
[3]
The efects of iecessions extend beyond the puiely economic iealm and infuence the social fabiic
of society as well. Suicide iates and piopeity ciimes-buiglaiy, laiceny, and motoi vehicle theft tend to
iise duiing iecessions. Even populai music appeais to be afected. Teiiy F. Pettohn II, a psychologist
at Coastal Caiolina Univeisity, has studied Billboaid No. 1 songs fiom 1933-2003. He fnds that duiing
iecessions, populai songs tend to be longei and slowei, and to have moie seiious lyiics. It's 'Biidge
ovei Tioubled Watei' oi 'That's What Fiiends Aie Foi'," he says. Duiing expansions, songs tend to be
fastei, shoitei, and somewhat silliei, such as At the Hop" oi My Shaiona."
[6]
In oui study of macioeconomics, we will gain an undeistanding of the foices at woik in the busi-
ness cycle. We will also exploie policies thiough which the public sectoi might act to make iecessions
less seveie and, peihaps, to piolong expansions. We tuin next to an examination of piice-level changes
and unemployment.
k Y 7 A k A W A Y S
< |e.| g.css Jcnest|c p.cJuct (.e.| C||) |s . ne.su.e c| t|e v.|ue c| .|| |r.| gccJs .rJ se.v|ces p.cJuceJ
Ju.|rg . p..t|cu|.. ye.. c. pe.|cJ, .JusteJ tc e||n|r.te t|e e|ects c| p.|ce c|.rges.
< |e eccrcny |c||c.s . p.t| c| exp.rs|cr, t|er ccrt..ct|cr, t|er exp.rs|cr .g.|r. |ese |uctu.t|crs
n.ke up t|e bus|ress cyc|e.
< |e pc|rt .t .||c| .r exp.rs|cr beccnes . .ecess|cr |s c.||eJ t|e pe.k c| . bus|ress cyc|e, t|e pc|rt .t
.||c| . .ecess|cr beccnes .r exp.rs|cr |s c.||eJ t|e t.cug|.
< Ove. t|ne, t|e gere..| t.erJ |c. ncst eccrcn|es |s cre c| .|s|rg .e.| C||. Or .ve..ge, .e.| C|| |r t|e
|r|teJ St.tes |.s g.c.r .t . ..te c| cve. 3 pe. ye.. s|rce 1960.
7 R Y I 7 !
|e J.t. be|c. s|c. t|e be|.v|c. c| .e.| C|| |r u.key |.cn t|e |.st ,u..te. c| 2001 t|.cug| t|e t||.J
,u..te. c| 2002. |se t|e J.t. tc p|ct .e.| C|| |r u.key .rJ |rJ|c.te t|e p|.ses c| t|e bus|ress cyc|e.
Period ReaI GDP (biIIions of New 7urkish Iira, 1987 prices)
||.st ,u..te., 2001 24.1
SeccrJ ,u..te., 2001 26.0
||.J ,u..te., 2001 33.1
|cu.t| ,u..te., 2001 2.1
||.st ,u..te., 2002 24.6
SeccrJ ,u..te., 2002 28.3
||.J ,u..te., 2002 35.
CHAP7R 20 MACROCONOMICS: 7H 8IG PIC7UR 499
Case in Point: 7he Art of Predicting Recessions
|ecp|e .|c n.ke . ||v|rg t..ck|rg t|e eccrcny .rJ t.y|rg tc p.eJ|ct |ts |utu.e Jc rct Jc . ve.y gccJ cb .t
p.eJ|ct|rg tu.r|rg pc|rts |r eccrcn|c .ct|v|ty. |e 52 eccrcn|sts su.veyeJ by t|e /o|| .|ee| .ooo| e.c|
ncrt| J|J p.eJ|ct t|.t t|e eccrcny .cu|J s||p |rtc . .ecess|cr |r t|e t||.J ,u..te. c| 2008. |ey n.Je t|.t
p.eJ|ct|cr, |c.eve., |r Octcbe..|te. t|e t||.J ,u..te. |.J erJeJ. |r Septenbe., t|e |.st ncrt| c| t|e t||.J
,u..te., t|e .ve..ge |c.ec.st .ncrg t|e 52 eccrcn|sts |.J t|e eccrcny ccrt|ru|rg tc g.c. t|.cug| t|e
t||.J .rJ |cu.t| ,u..te.s c| 2008. |.t Septenbe. su.vey ..s t.ker be|c.e t|e |r.rc|.| c.|s|s ||t, . c.|s|s t|.t
tcck v|.tu.||y eve.ycre by su.p.|se. O| ccu.se, .s .e |.ve .|.e.Jy rcteJ, t|e t||.J,u..te. Jc.rtu.r |.J rct
beer |Jert||eJ .s . .ecess|cr by t|e |b||s bus|ress yc|e |.t|rg cnn|ttee .s c| |cvenbe. c| 2008.
|.eJ|ct|rg bus|ress cyc|e tu.r|rg pc|rts |.s .|..ys beer . t.|cky bus|ress. |e expe.|erce c| t|e .ecess|cr c|
2001 |||ust..tes t||s. As t|e .cccnp.ry|rg t.b|e s|c.s, ever .s |.te .s Septenbe. 10, 2001, cr|y 13 cut c| t|e
100 b|ue ||p |c.ec.ste.s |.J .rs.e.eJ |r t|e .|.n.t|ve tc t|e ,uest|cr, |.s t|e |r|teJ St.tes s||ppeJ |rtc
. .ecess|cr` ever t|cug| .e rc. krc. t|e .ecess|cr |.J begur t|e p.ev|cus V..c|. cnp..|rg t|e J.t.
t|.t .e.e c.|g|r.||y .e|e.seJ by t|e |.S. bu.e.u c| |ccrcn|c Ar.|ys|s s|c.t|y .|te. t|e erJ c| e.c| ,u..te. .|t|
t|e .ev|seJ J.t. t|.t .e.e .e|e.seJ .|te. 'u|y 2002 p.cv|Jes .r |npc.t.rt |rs|g|t |rtc exp|.|r|rg .|y t|e |c.e
c.ste.s seen tc |.ve Jcre sc b.J|y. As t|e g..p| cr p.e.ev|s|cr .rJ pcst.ev|s|cr est|n.tes c| .e.| C||
g.c.t| s|c.s, t|e J.t. .e|e.seJ s|c.t|y .|te. t|e erJ c| e.c| ,u..te. s|c.eJ .r eccrcny exp.rJ|rg
t|.cug| t|e seccrJ ,u..te. c| 2001, .|e.e.s t|e .ev|seJ J.t. s|c. t|e eccrcny ccrt..ct|rg ncJest|y |r t|e
|.st ,u..te. c| 2001 .rJ t|er nc.e |c.ce|u||y |r t|e seccrJ ,u..te.. Or|y .|te. t|e .tt.cks cr t|e \c.|J ..Je
erte. |r |e. +c.k |ty .rJ t|e |ert.gcr |r \.s||rgtcr, |.., cr Septenbe. 11, 2001, J|J ncst c| t|e b|ue
||p |c.ec.ste.s .e.||.e t|e eccrcny ..s |r .ecess|cr.
|e |.t|cr.| bu.e.u c| |ccrcn|c |ese..c| (|b||) bus|ress yc|e |.t|rg cnn|ttee |r |cvenbe. 2001 .e
|e.seJ . p.ess .rrcurcenert J.t|rg t|e crset c| t|e .ecess|cr .s V..c| 2001. |e ccnn|ttee ..gueJ t|.t
be|c.e t|e .tt.cks c| Septenbe. 11, |t |s pcss|b|e t|.t t|e Jec||re |r t|e eccrcny .cu|J |.ve beer tcc n||J
tc ,u.|||y .s . .ecess|cr. |e .tt.cks c|e..|y JeepereJ t|e ccrt..ct|cr .rJ n.y |.ve beer .r |npc.t.rt |.ctc.
|r tu.r|rg t|e ep|scJe |rtc . .ecess|cr. \|||e su.p.|s|rg .t t|e t|ne, t|e .ev|seJ J.t. suggest t|.t t|e ccn
n|ttee n.Je . gccJ c.||.
||s ep|scJe |r eccrcn|c ||stc.y .|sc pc|rts cut t|e J||e.erce bet.eer t|e ccnncr Je|r|t|cr c| . .ecess|cr
.s t.c ccrsecut|ve ,u..te.s c| Jec||r|rg .e.| C|| .rJ t|e |b|| |.t|rg cnn|ttees ccrt|rueJ |rs|sterce t|.t
|t Jces rct Je|re . .ecess|cr |r t||s ..y. |.t|e. t|e ccnn|ttee |ccks rct cr|y .t .e.| C|| but .|sc .t enp|cy
nert, |rccne, .rJ ct|e. |.ctc.s. |e be|.v|c. c| enp|cynert Ju.|rg 2001 seens tc |.ve beer .r |npc.t.rt
|.ctc. |r t|e |cvenbe. 2001 Jec|s|cr tc p.cc|.|n V..c| 2001 .s t|e pe.k Jesp|te t|e n|s|e.J|rg |r|c.n.t|cr
cr .e.| C|| ccn|rg cut c| t|e bu.e.u c| |ccrcn|c Ar.|ys|s .t t|e t|ne. |e s|c. p|ckup |r enp|cynert n.y
.|sc, t|cug|, |.ve n.Je |t |es|t.te tc c.|| |cvenbe. 2001 t|e t.cug| urt|| 'u|y 2003.
500 PRINCIPLS OI CONOMICS
Question posed: Has the United States sIipped into a recession1"
Date Percent of 8Iue Chip responders
answering Yes"
Percent of 8Iue Chip responders
answering No"
|eb.u..y 2001 5 95
'ure 2001 93
'u|y 2001 13 8
August 2001 5 85
Septenbe. 10,
2001
13 8
Septenbe. 19,
2001
82 18
ReaI GDP Growth: Pre- and Post-1uIy, 2002
5ources |||| |o. |coo|| |ec| || |o |eee.' /o|| .|ee| .ooo| ||e. c|oe 0. 200S. |e.| | |||ee. |e 200 |ece|o |o. /o ||
|||ee| oJ /|o| |e.e|oe| /o, |o.e ooeJ ||.' |eJeo| |ee.e |o| o| .| |oo| |e.|e.. .e|eec|oe 2003 233..
|||...eoc,c|e. |e |e|eoe.' |o|e ,c|e |o|| o|||ee. |o||oo| |oeoo o| |coo|c |eeoc|. e e|eoe. o|Je.
/ooc|oe||. .o|, .. 2002
A N S W R 7 O 7 R Y I 7 ! P R O 8 L M
CHAP7R 20 MACROCONOMICS: 7H 8IG PIC7UR 501
in0ation
Ar |rc.e.se |r t|e .ve..ge
|eve| c| p.|ces.
de0ation
A Jec.e.se |r t|e .ve..ge
|eve| c| p.|ces.
2. PRICE-LEVEL CHANGES
L A R N I N G O 8 1 C 7 I V S
1. Dene in0ation and de0ation, expIain how their rates are determined, and articuIate why
price-IeveI changes matter.
2. xpIain what a price index is and outIine the generaI steps in computing a price index.
3. Describe and compare dierent price indexes.
4. xpIain how to convert nominaI vaIues to reaI vaIues and expIain why it is usefuI to make this
caIcuIation.
5. Discuss the biases that may arise from price indexes that empIoy xed market baskets of goods
and services.
Concein about changes in the piice level has always dominated economic discussion. With infation in
the United States geneially aveiaging only between 2 and 3 each yeai since 1990, it may seem sui-
piising how much attention the behavioi of the piice level still commands. Yet infation was a concein
in 2004 when theie was feai that the iising piice of oil could tiiggei highei piices in othei aieas. Just
the yeai befoie, when infation fell below 2, theie was talk about the iisk of defation. That did not
happen; piices continued iising. Infation iose substantially in the fist half of 2008, ienewing feais
about subsequent fuithei incieases. Just what aie infation and defation: How aie they measuied: And
most impoitant, why do we caie: These aie some of the questions we will exploie in this section.
Ination is an inciease in the aveiage level of piices, and deation is a deciease in the aveiage
level of piices. In an economy expeiiencing infation, most piices aie likely to be iising, wheieas in an
economy expeiiencing defation, most piices aie likely to be falling.
Theie aie two key points in these defnitions:
1. Infation and defation iefei to changes in the aveiage level of piices, not to changes in paiticulai
piices. An inciease in medical costs is not infation. A deciease in gasoline piices is not defation.
Infation means the aveiage level of piices is iising, and defation means the aveiage level of
piices is falling.
2. Infation and defation iefei to rising piices and falling piices, iespectively; theiefoie, they do not
have anything to do with the level of piices at any one time. High" piices do not imply the
piesence of infation, noi do low" piices imply defation. Infation means a positive rate of
change in aveiage piices, and defation means a negative rate of change in aveiage piices.
2.1 Why Do We Caie:
What difeience does it make if the aveiage level of piices changes: Fiist, considei the impact of
infation.
Infation is measuied as the annual iate of inciease in the aveiage level of piices. Figuie 20.6 shows
how volatile infation has been in the United States ovei the past foui decades. In the 1960s the infa-
tion iate iose, and it became diamatically woise in the 1970s. The infation iate plunged in the 1980s
and continued to ease downwaid in the 1990s. It iemained low in the eaily 2000s and began to accelei-
ate in 2007 and duiing most of 2008.
502 PRINCIPLS OI CONOMICS
II GUR 20. 6 InfIation, 1960-2008
|e |.S. |r|.t|cr ..te, ne.su.eJ .s t|e .rru.| ..te c| c|.rge |r t|e .ve..ge |eve| c| p.|ces p.|J by ccrsune.s,
v..|eJ ccrs|Je..b|y cve. t|e 19602008 pe.|cJ.
Source Bureau of Labor Statistics, All Urban Consumers CPI-U, 1982-84 = 100, Dec.-Dec. infation rate. Data for 2008 is through October.
Whethei one iegaids infation as a good" thing oi a bad" thing depends veiy much on one's econom-
ic situation. If you aie a boiiowei, unexpected infation is a good thing-it ieduces the value of money
that you must iepay. If you aie a lendei, it is a bad thing because it ieduces the value of futuie pay-
ments you will ieceive. Whatevei any paiticulai peison's situation may be, infation always pioduces
the following efects on the economy: it ieduces the value of money and it ieduces the value of futuie
monetaiy obligations. It can also cieate unceitainty about the futuie.
Suppose that you have just found a $10 bill you stashed away in 1990. Piices have incieased by
about 30 since then; youi money will buy less than what it would have puichased when you put it
away. Youi money has thus lost value.
Money loses value when its puichasing powei falls. Since infation is a iise in the level of piices, the
amount of goods and seivices a given amount of money can buy falls with infation.
Just as infation ieduces the value of money, it ieduces the value of futuie claims on money. Sup-
pose you have boiiowed $100 fiom a fiiend and have agieed to pay it back in one yeai. Duiing the
yeai, howevei, piices double. That means that when you pay the money back, it will buy only half as
much as it could have bought when you boiiowed it. That is good foi you but tough on the peison who
lent you the money. Of couise, if you and youi fiiend had anticipated such iapid infation, you might
have agieed to pay back a laigei sum to adjust foi it. When people anticipate infation, they can adjust
foi its consequences in deteimining futuie obligations. But unanticipated infation helps boiioweis and
huits lendeis.
Infation's impact on futuie claims can be paiticulaily haid on people who must live on a fxed in-
come, that is, on an income that is piedeteimined thiough some contiactual aiiangement and does not
change with economic conditions. An annuity, foi example, typically piovides a fxed stieam of money
payments. Retiiement pensions sometimes geneiate fxed income. Infation eiodes the value of such
payments.
Given the dangei posed by infation foi people on fxed incomes, many ietiiement plans piovide
foi indexed payments. An indexed payment is one whose dollai amount changes with the iate of
change in the piice level. If a payment changes at the same iate as the iate of change in the piice level,
the puichasing powei of the payment iemains constant. Social Secuiity payments, foi example, aie in-
dexed to maintain theii puichasing powei.
Because infation ieduces the puichasing powei of money, the thieat of futuie infation can make
people ieluctant to lend foi long peiiods. Fiom a lendei's point of view, the dangei of a long-teim com-
mitment of funds is that futuie infation will wipe out the value of the amount that will eventually be
paid back. Lendeis aie ieluctant to make such commitments.
CHAP7R 20 MACROCONOMICS: 7H 8IG PIC7UR 503
hyperin0ation
Ar |r|.t|cr ..te |r excess c|
200 pe. ye...
price index
A runbe. .|cse ncvenert
.e|ects ncvenert |r t|e
.ve..ge |eve| c| p.|ces.
base period
A t|ne pe.|cJ .g.|rst .||c|
ccsts c| t|e n..ket b.sket |r
ct|e. pe.|cJs .||| be
ccnp..eJ |r ccnput|rg .
p.|ce |rJex.
Unceitainty can be paiticulaily pionounced in countiies wheie extiemely high infation is a thieat.
Hyperination is geneially defned as an infation iate in excess of 200 pei yeai. Infation of that
magnitude eiodes the value of money veiy quickly. Hypeiinfations occuiied in Geimany in the 1920s
and in Yugoslavia in the eaily 1990s. Theie aie stoiies about how people in Geimany duiing the hypei-
infation biought wheelbaiiows full of money to stoies to pay foi oidinaiy items. In Yugoslavia in 1993
theie was a iepoit of a shop ownei baiiing the entiance to his stoie with a mop while he changed his
piices.
The infation iate iose to an astionomical iate in 2008 in Zimbabwe. As the goveinment piinted
moie money and put it in ciiculation, piices iose. When infation began to acceleiate, the goveinment
found it necessaiy" to piint moie and moie money, causing piices to iise veiy fast. The infation iate
in Zimbabwe ieached an astonishing 11.2 million peicent in July of 2008, accoiding to Zimbabwe's
Cential Statistics Omce. A loaf of biead cost 200,000 Zimbabwe dollais in Febiuaiy 2008. That same
loaf cost 1.6 tiillion Zimbabwe dollais by August.
[7]
Do the pioblems associated with infation imply that defation would be a good thing: The answei
is simple: no. Like infation, defation changes the value of money and the value of futuie obligations. It
also cieates unceitainty about the futuie.
If theie is defation, the ieal value of a given amount of money iises. In othei woids, if theie had
been defation since 2000, a $10 bill you had stashed away in 2000 would buy moie goods and seivices
today. That sounds good, but should you buy $10 woith of goods and seivices now when you would be
able to buy even moie foi youi $10 in the futuie if the defation continues: When Japan expeiienced
defation in the late 1990s and eaily 2000s, Japanese consumeis seemed to be doing just that-waiting
to see if piices would fall fuithei. They weie spending less pei peison and, as we will see thioughout
oui study of macioeconomics, less consumption often meant less output, fewei jobs, and the piospect
of a iecuiiing iecessions.
And, if you had to use the $10 to pay back a debt you owed, the puichasing powei of youi money
would be highei than when you boiiowed the money. The lendei would feel good about being able to
buy moie with the $10 than you weie able to, but you would feel like you had gotten a iaw deal.
Unanticipated defation huits boiioweis and helps lendeis. If the paities anticipate the defation, a
loan agieement can be wiitten to iefect expected changes in the piice level.
The thieat of defation can make people ieluctant to boiiow foi long peiiods. Boiioweis become
ieluctant to entei into long-teim contiacts because they feai that defation will iaise the value of the
money they must pay back in the futuie. In such an enviionment, fims may be ieluctant to boiiow to
build new factoiies, foi example. This is because they feai that the piices at which they can sell theii
output will diop, making it dimcult foi them to iepay theii loans.
Defation was common in the United States in the lattei thiid of the 19th centuiy. In the 20th cen-
tuiy, theie was a peiiod of defation aftei Woild Wai I and again duiing the Gieat Depiession in the
1930s.
2.2 Piice Indexes
How do we actually measuie infation and defation (that is, changes in the piice level): Piice-level
change is measuied as the peicentage iate of change in the level of piices. But how do we fnd a piice
level:
Economists measuie the piice level with a piice index. A price index is a numbei whose move-
ment iefects movement in the aveiage level of piices. If a piice index iises 10, it means the aveiage
level of piices has iisen 10.
Theie aie foui steps one must take in computing a piice index:
1. Select the kinds and quantities of goods and seivices to be included in the index. A list of these
goods and seivices, and the quantities of each, is the maiket basket" foi the index.
2. Deteimine what it would cost to buy the goods and seivices in the maiket basket in some peiiod
that is the base peiiod foi the index. A base period is a time peiiod against which costs of the
maiket basket in othei peiiods will be compaied in computing a piice index. Most often, the base
peiiod foi an index is a single yeai. If, foi example, a piice index had a base peiiod of 1990, costs
of the basket in othei peiiods would be compaied to the cost of the basket in 1990. We will
encountei one index, howevei, whose base peiiod stietches ovei thiee yeais.
3. Compute the cost of the maiket basket in the cuiient peiiod.
4. Compute the piice index. It equals the cuiient cost divided by the base-peiiod cost of the maiket
basket.
QUA7I ON 20. 1
Piice index = cuiient cost of basket / base-peiiod cost of basket
504 PRINCIPLS OI CONOMICS
consumer price index (CPI)
A p.|ce |rJex .|cse
ncvenert .e|ects c|.rges
|r t|e p.|ces c| gccJs .rJ
se.v|ces typ|c.||y pu.c|.seJ
by ccrsune.s.
(While published piice indexes aie typically iepoited with this numbei multiplied by 100, oui woik
with indexes will be simplifed by omitting this step.)
Suppose that we want to compute a piice index foi movie fans, and a suivey of movie watcheis
tells us that a typical fan ients 4 movies on DVD and sees 3 movies in theateis each month. At the
theatei, this viewei consumes a medium-sized soft diink and a medium-sized box of popcoin. Oui
maiket basket thus might include 4 DVD ientals, 3 movie admissions, 3 medium soft diinks, and 3 me-
dium seivings of popcoin.
Oui next step in computing the movie piice index is to deteimine the cost of the maiket basket.
Suppose we suiveyed movie theateis and DVD-iental stoies in 2007 to deteimine the aveiage piices of
these items, fnding the values given in Table 20.1. At those piices, the total monthly cost of oui movie
maiket basket in 2007 was $48. Now suppose that in 2008 the piices of movie admissions and DVD
ientals iise, soft-diink piices at movies fall, and popcoin piices iemain unchanged. The combined
efect of these changes pushes the 2008 cost of the basket to $30.88.
7A8L 20. 1 Pricing a Market 8asket
c ccnpute . p.|ce |rJex, .e reeJ tc Je|re . n..ket b.sket .rJ Jete.n|re |ts p.|ce. |e t.b|e g|ves t|e
ccnpcs|t|cr c| t|e ncv|e n..ket b.sket .rJ p.|ces |c. 200 .rJ 2008. |e ccst c| t|e ert|.e b.sket .|ses |.cn 48
|r 200 tc 50.88 |r 2008.
Item Quantity in 8asket 2007 Price Cost in 2007 8asket 2008 Price Cost in 2008 8asket
|\| .ert.| 4 2.25 9.00 2.9 11.88
Vcv|e .Jn|ss|cr 3 .5 23.25 8.00 24.00
|cpcc.r 3 2.25 6.5 2.25 6.5
Sc|t J.|rk 3 3.00 9.00 2.5 8.25
ct.| ccst c| b.sket 200 48.00 2008 50.88
Using the data in Table 20.1, we could compute piice indexes foi each yeai. Recall that a piice index is
the iatio of the cuiient cost of the basket to the base-peiiod cost. We can select any yeai we wish as the
base yeai; take 2007. The 2008 movie piice index (MPI) is thus
MPI
2008
= $30.88 / $48 = 1.06
The value of any piice index in the base peiiod is always 1. In the case of oui movie piice index, the
2007 index would be the cuiient (2007) cost of the basket, $48, divided by the base-peiiod cost, which
is the same thing: $48/$48 = 1.
7he Consumer Price Index (CPI)
One widely used piice index in the United States is the consumer price index (CPI), a piice index
whose movement iefects changes in the piices of goods and seivices typically puichased by con-
sumeis. When the media iepoit the U.S. infation iate, the numbei cited is usually a iate computed us-
ing the CPI. The CPI is also used to deteimine whethei people's incomes aie keeping up with the costs
of the things they buy. The CPI is often used to measuie changes in the cost of living, though as we
shall see, theie aie pioblems in using it foi this puipose.
The maiket basket foi the CPI contains thousands of goods and seivices. The composition of the
basket is deteimined by the Buieau of Laboi Statistics (BLS), an agency of the Depaitment of Laboi,
based on Census Buieau suiveys of household buying behavioi. Suiveyois tally the piices of the goods
and seivices in the basket each month in cities all ovei the United States to deteimine the cuiient cost
of the basket. The majoi categoiies of items in the CPI aie food and beveiages, housing, appaiel, tians-
poitation, medical caie, iecieation, education and communication, and othei goods and seivices.
The cuiient cost of the basket of consumei goods and seivices is then compaied to the base-peiiod
cost of that same basket. The base peiiod foi the CPI is 1982-1984; the base-peiiod cost of the basket is
its aveiage cost ovei this peiiod. Each month's CPI thus iefects the iatio of the cuiient cost of the bas-
ket divided by its base-peiiod cost.
QUA7I ON 20. 2
CPI = cuiient cost of basket / 1982-1984 cost of basket
Like many othei piice indexes, the CPI is computed with a fxed maiket basket. The composition
of the basket geneially iemains unchanged fiom one peiiod to the next. Because buying patteins
change, howevei, the basket is ievised accoidingly. The data in Table 20.1, foi example, aie based on
2003-2006 expendituie weights. The base peiiod, though, was still 1982-1984.
CHAP7R 20 MACROCONOMICS: 7H 8IG PIC7UR 505
impIicit price de0ator
A p.|ce |rJex |c. .|| |r.|
gccJs .rJ se.v|ces
p.cJuceJ, |t |s t|e ..t|c c|
rcn|r.| C|| tc .e.| C||.
personaI consumption
expenditures price index
A p.|ce |rJex t|.t |rc|uJes
Ju..b|e gccJs, rcrJu..b|e
gccJs, .rJ se.v|ces .rJ |s
p.cv|JeJ .|crg .|t|
est|n.tes |c. p.|ces c| e.c|
ccnpcrert c| ccrsunpt|cr
sperJ|rg.
7he ImpIicit Price DefIator
Values foi nominal and ieal GDP, desciibed eailiei in this chaptei, piovide us with the infoimation to
calculate the most bioad-based piice index available. The implicit price deator, a piice index foi all
fnal goods and seivices pioduced, is the iatio of nominal GDP to ieal GDP.
In computing the implicit piice defatoi foi a paiticulai peiiod, economists defne the maiket bas-
ket quite simply: it includes all the fnal goods and seivices pioduced duiing that peiiod. The nominal
GDP gives the cuiient cost of that basket; the ieal GDP adjusts the nominal GDP foi changes in piices.
The implicit piice defatoi is thus given by
QUA7I ON 20. 3
Implicit piice deflatoi = nominal GDP/ ieal GDP
Foi example, in 2007, nominal GDP in the United States was $13,807.3 billion, and ieal GDP was
$11,323.9 billion. Thus, the implicit piice defatoi was 1.198. Following the convention of multiplying
piice indexes by 100, the published numbei foi the implicit piice defatoi was 119.8.
In oui analysis of the deteimination of output and the piice level in subsequent chapteis, we will
use the implicit piice defatoi as the measuie of the piice level in the economy.
7he PC Price Index
The Buieau of Economic Analysis also pioduces piice index infoimation foi each of the components of
GDP (that is, a sepaiate piice index foi consumei piices, piices foi difeient components of gioss
piivate domestic investment, and goveinment spending). The personal consumption expenditures
price index, oi PCE piice index, includes duiable goods, nonduiable goods, and seivices and is
piovided along with estimates foi piices of each component of consumption spending. Because piices
foi food and eneigy can be volatile, the piice measuie that excludes food and eneigy is often used as a
measuie of undeilying, oi coie," infation. Note that the PCE piice index difeis substantially fiom the
consumei piice index, piimaiily because it is not a fxed basket" index.
[8]
The PCE piice index has be-
come a politically impoitant measuie of infation since the Fedeial Reseive (discussed in detail in latei
chapteis) uses it as its piimaiy measuie of piice levels in the United States.
2.3 Computing the Rate of Inflation oi Deflation
The iate of infation oi defation is the peicentage iate of change in a piice index between two peiiods.
Given piice-index values foi two peiiods, we can calculate the iate of infation oi defation as the
change in the index divided by the initial value of the index, stated as a peicentage:
QUA7I ON 20. 4
Rate of inflation oi deflation = peicentage change in index / initial value of index
To calculate infation in movie piices ovei the 2007-2008 peiiod, foi example, we could apply
Equation 20.4 to the piice indexes we computed foi those two yeais as follows:
Movie inflation iate in 2008 = (1.06 1.00) / 1.00 = 0.06 = 6
The CPI is often used foi calculating piice-level change foi the economy. Foi example, the iate of infa-
tion in 2007 can be computed fiom the Decembei 2006 piice level (2.016) and the Decembei 2007 level
(2.073):
Inflation iate = (2.073 2.016) / 2.016 = 0.028 = 2.8
2.4 Computing Real Values Using Piice Indexes
Suppose youi uncle staited college in 1998 and had a job busing dishes that paid $3 pei houi. In 2008
you had the same job; it paid $6 pei houi. Which job paid moie:
506 PRINCIPLS OI CONOMICS
reaI vaIue
A v.|ue exp.esseJ |r ur|ts c|
ccrst.rt pu.c|.s|rg pc.e..
nominaI vaIue
A v.|ue exp.esseJ |r Jc||..s
c| t|e cu..ert pe.|cJ.
At fist glance, the answei is stiaightfoiwaid: $6 is a highei wage than $3. But $1 had gieatei pui-
chasing powei in 1998 than in 2008 because piices weie lowei in 1998 than in 2008. To obtain a valid
compaiison of the two wages, we must use dollais of equivalent puichasing powei. A value expiessed
in units of constant puichasing powei is a real value. A value expiessed in dollais of the cuiient peii-
od is called a nominal value. The $3 wage in 1998 and the $6 wage in 2008 aie nominal wages.
To conveit nominal values to ieal values, we divide by a piice index. The ieal value foi a given
peiiod is the nominal value foi that peiiod divided by the piice index foi that peiiod. This pioceduie
gives us a value in dollais that have the puichasing powei of the base peiiod foi the piice index used.
Using the CPI, foi example, yields values expiessed in dollais of 1982-1984 puichasing powei, the base
peiiod foi the CPI. The ieal value of a nominal amount X at time t, X
t
, is found using the piice index
foi time t:
QUA7I ON 20. 5
Real value of X
t
= X
t
/ piice index at timet
Let us compute the ieal value of the $6 wage foi busing dishes in 2008 veisus the $3 wage paid to
youi uncle in 1998. The CPI in 1998 was 163.0; in 2008 it was 216.3. Real wages foi the two yeais weie
thus
Real wage in 1998 = $3 / 1.630 = $3.07
Real wage in 2008 = $6 / 2.163 = $2.77
Given the nominal wages in oui example, you eained about 10 less in ieal teims in 2008 than youi
uncle did in 1998.
Piice indexes aie useful. They allow us to see how the geneial level of piices has changed. They al-
low us to estimate the iate of change in piices, which we iepoit as the iate of infation oi defation. And
they give us a tool foi conveiting nominal values to ieal values so we can make bettei compaiisons of
economic peifoimance acioss time.
Are Price Indexes Accurate Measures of Price-LeveI Changes1
Piice indexes that employ fxed maiket baskets aie likely to oveistate infation (and undeistate defa-
tion) foi foui ieasons:
1. Because the components of the maiket basket aie fxed, the index does not incoipoiate consumei
iesponses to changing ielative piices.
2. A fxed basket excludes new goods and seivices.
3. Quality changes may not be completely accounted foi in computing piice-level changes.
4. The type of stoie in which consumeis choose to shop can afect the piices they pay, and the piice
indexes do not iefect changes consumeis have made in wheie they shop.
To see how these factois can lead to inaccuiate measuies of piice-level changes, suppose the piice of
chicken iises and the piice of beef falls. The law of demand tells us that people will iespond by consum-
ing less chicken and moie beef. But if we use a fxed maiket basket of goods and seivices in computing
a piice index, we will not be able to make these adjustments. The maiket basket holds constant the
quantities of chicken and beef consumed. The impoitance in consumei budgets of the highei chicken
piice is thus oveistated, while the impoitance of the lowei beef piice is undeistated. Moie geneially, a
fxed maiket basket will oveistate the impoitance of items that iise in piice and undeistate the impoit-
ance of items that fall in piice. This souice of bias is iefeiied to as the substitution bias.
The new-pioduct bias, a second souice of bias in piice indexes, occuis because it takes time foi
new pioducts to be incoipoiated into the maiket basket that makes up the CPI. A good intioduced to
the maiket aftei the basket has been defned will not, of couise, be included in it. But a new good, once
successfully intioduced, is likely to fall in piice. When VCRs weie fist intioduced, foi example, they
geneially cost moie than $1,000. Within a few yeais, an equivalent machine cost less than $200. But
when VCRs weie intioduced, the CPI was based on a maiket basket that had been defned in the eaily
1970s. Theie was no VCR in the basket, so the impact of this falling piice was not iefected in the index.
The DVD playei was intioduced into the CPI within a yeai of its availability.
A thiid piice index bias, the quality-change bias, comes fiom impiovements in the quality of
goods and seivices. Suppose, foi example, that Foid intioduces a new cai with bettei safety featuies
and a smoothei iide than its pievious model. Suppose the old model cost $20,000 and the new model
costs $24,000, a 20 inciease in piice. Should economists at the Buieau of Laboi Statistics (BLS) simply
iecoid the new model as being 20 moie expensive than the old one: Cleaily, the new model is not the
same pioduct as the old model. BLS economists faced with such changes tiy to adjust foi quality. To
CHAP7R 20 MACROCONOMICS: 7H 8IG PIC7UR 507
the extent that such adjustments undeistate quality change, they oveistate any inciease in the piice
level.
The fouith souice of bias is called the outlet bias. Households can ieduce some of the impact of
iising piices by shopping at supeistoies oi outlet stoies (such as T.J. Maxx, Wal-Mait, oi factoiy outlet
stoies), though this often means they get less customei seivice than at tiaditional depaitment stoies oi
at smallei ietail stoies. Howevei, since such shopping has incieased in iecent yeais, it must be that foi
theii customeis, the ieduction in piices has been moie valuable to them than loss of seivice. Piioi to
1998, the CPI did not account foi a change in the numbei of households shopping at these newei kinds
of stoies in a timely mannei, but the BLS now does quaiteily suiveys and updates its sample of stoies
much moie fiequently. Anothei foim of this bias aiises because the goveinment data collectois do not
collect piice data on weekends and holidays, when many stoies iun sales.
Economists difei on the degiee to which these biases iesult in inaccuiacies in iecoiding piice-level
changes. In late 1996, Michael Boskin, an economist at Stanfoid Univeisity, chaiied a panel of econom-
ists appointed by the Senate Finance Committee to deteimine the magnitude of the pioblem in the Un-
ited States. The panel iepoited that the CPI was oveistating infation in the United States by 0.8 to 1.6
peicentage points pei yeai. Theii best estimate was 1.1 peicentage points, as shown in Table 20.2. Since
then, the Buieau of Laboi Statistics has made a numbei of changes to coiiect foi these souices of bias
and since August 2002 has iepoited a new consumei piice index called the Chained Consumei Piice
Index foi all Uiban Consumeis (C-CPU-U) that attempts to piovide a closei appioximation to a cost-
of-living" index by utilizing expendituie data that iefect the substitutions that consumeis make acioss
item categoiies in iesponse to changes in ielative piices.
[9]
Howevei, a 2006 study by Robeit Goidon, a
piofessoi at Noithwestein Univeisity and a membei of the oiiginal 1996 Boskin Commission, estim-
ates that the total bias is still about 0.8 peicentage points pei yeai, as also shown in Table 20.2.
7A8L 20. 2 stimates of 8ias in the Consumer Price Index
|e bcsk|r cnn|ss|cr .epc.teJ t|.t t|e || cve.st.tes t|e ..te c| |r|.t|cr by 0.8 tc 1.6 pe.cert.ge pc|rts Jue tc
t|e b|.ses s|c.r, .|t| . bestguess est|n.te c| 1.1. A 2006 stuJy by |cbe.t Cc.Jcr est|n.tes t|.t t|e b|.s |e|| but
|s st||| .bcut 0.8 pe.cert.ge pc|rts.
Sources of 8ias 1997 stimate 2006 stimate
Subst|tut|cr 0.4 0.4
|e. p.cJucts .rJ ,u.||ty c|.rge 0.6 0.3
S.|tc||rg tc re. cut|ets 0.1 0.1
ct.| 1.1 0.8
||.us|b|e ..rge 0.81.6
Source Robert j. Gordon, The Boskin Commission Report A Retrospective One Decade Later (^ational Bureau of Economic Research Vorking
Paper 12311, june 200), available at http//www.nber.org/papers/w12311.
These fndings of upwaid bias have enoimous piactical signifcance. With annual infation iunning be-
low 2 in thiee out of the last 10 yeais and aveiaging 2.7 ovei the 10 yeais, it means that the United
States has come close to achieving piice stability foi almost a decade.
To the extent that the computation of piice indexes oveistates the iate of infation, then the use of
piice indexes to coiiect nominal values iesults in an undeistatement of gains in ieal incomes. Foi ex-
ample, aveiage nominal houily eainings of U.S. pioduction woikeis weie $13.01 in 1998 and $17.42 in
2007. Adjusting foi CPI-measuied infation, the aveiage ieal houily eainings was $7.98 in 1998 and
$8.40 in 2007, suggesting that ieal wages iose about 3.3 ovei the peiiod. If infation was oveistated by
0.8 pei yeai ovei that entiie peiiod, as suggested by Goidon's updating of the Boskin Commission's
best estimate, then, adjusting foi this oveistatement, ieal wages should have been iepoited as $7.98 foi
1998 and $9.01 foi 2007, a gain of neaily 13.
Also, because the CPI is used as the basis foi calculating U.S. goveinment payments foi piogiams
such as Social Secuiity and foi adjusting tax biackets, this piice index afects the goveinment's budget
balance, the difeience between goveinment ievenues and goveinment expendituies. The Congiession-
al Budget Omce has estimated that coiiecting the biases in the index would have incieased ievenue by
$2 billion and ieduced outlays by $4 billion in 1997. By 2007, the U.S. goveinment's budget would have
had an additional $140 billion if the bias weie iemoved.
508 PRINCIPLS OI CONOMICS
k Y 7 A k A W A Y S
< |r|.t|cr |s .r |rc.e.se |r t|e .ve..ge |eve| c| p.|ces, .rJ Je|.t|cr |s . Jec.e.se |r t|e .ve..ge |eve| c|
p.|ces. |e ..te c| |r|.t|cr c. Je|.t|cr |s t|e pe.cert.ge ..te c| c|.rge |r . p.|ce |rJex.
< |e ccrsune. p.|ce |rJex (||) |s t|e ncst .|Je|y useJ p.|ce |rJex |r t|e |r|teJ St.tes.
< |cn|r.| v.|ues c.r be ccrve.teJ tc .e.| v.|ues by J|v|J|rg by . p.|ce |rJex.
< |r|.t|cr .rJ Je|.t|cr .|ect t|e .e.| v.|ue c| ncrey, c| |utu.e cb||g.t|crs ne.su.eJ |r ncrey, .rJ c|
|xeJ |rccnes. |r.rt|c|p.teJ |r|.t|cr .rJ Je|.t|cr c.e.te urce.t.|rty .bcut t|e |utu.e.
< |ccrcn|sts gere..||y .g.ee t|.t t|e || .rJ ct|e. p.|ce |rJexes t|.t enp|cy |xeJ n..ket b.skets c|
gccJs .rJ se.v|ces Jc rct .ccu..te|y ne.su.e p.|ce|eve| c|.rges. b|.ses |rc|uJe t|e subst|tut|cr b|.s, t|e
re.p.cJuct b|.s, t|e ,u.||tyc|.rge b|.s, .rJ t|e cut|et b|.s.
7 R Y I 7 !
Suppcse t|.t rcn|r.| C|| |s 10 t.||||cr |r 2003 .rJ 11 t.||||cr |r 2004, .rJ t|.t t|e |np||c|t p.|ce Je|.tc.
|.s gcre |.cn 1.063 |r 2003 tc 1.091 |r 2004. cnpute .e.| C|| |r 2003 .rJ 2004. |s|rg t|e pe.cert.ge
c|.rge |r t|e |np||c|t p.|ce Je|.tc. .s t|e g.uge, .|.t ..s t|e |r|.t|cr ..te cve. t|e pe.|cJ`
Case in Point: 7ake Me Out to the 8aII Game .
|e ccst c| . t.|p tc t|e c|J b.|| g.ne unpeJ .9 |r 2008, .ccc.J|rg tc eo /o|e|| |eo|, . ||c.gc
b.seJ re.s|ette.. |e .epc.t b.ses |ts est|n.te cr |ts |.r p.|ce |rJex, .|cse n..ket b.sket |rc|uJes t.c .Ju|t
.ve..gep.|ceJ t|ckets, t.c c|||J .ve..gep.|ceJ t|ckets, t.c sn.|| J..|t bee.s, |cu. sn.|| sc|t J.|rks, |cu.
.egu|..s|.eJ |ct Jcgs, p..k|rg |c. cre c.., t.c g.ne p.cg..ns, .rJ t.c |e.st expers|ve, .Ju|ts|.eJ .J
ust.b|e b.seb.|| c.ps. |e .ve..ge p.|ce c| t|e n..ket b.sket ..s 191.92 |r 2008.
eo/o|e|| ccnp||es t|e ccst c| t|e b.sket |c. e.c| c| n.c. |e.gue b.seb.||s 30 te.ns. Accc.J|rg tc t||s
ccnp||.t|cr, t|e bcstcr |eJ Scx ..s t|e ncst expers|ve te.n tc ..tc| |r 2008, t|e .np. b.y |.ys ..s t|e
c|e.pest. |e |.ys n.Je |t tc t|e \c.|J Se.|es |r 2008, t|e |eJ Scx J|J rct. by t|.t ne.su.e, t|e |.ys .e.e
scnet||rg c| . b..g.|r. |e t.b|e s|c.s t|e ccst c| t|e |.r p.|ce |rJex n..ket b.sket |c. 2008.
CHAP7R 20 MACROCONOMICS: 7H 8IG PIC7UR 509
7eam 8asket Cost 7eam 8asket Cost
bcstcr |eJ Scx 320.1 S.r |..rc|scc C|.rts 183.4
|e. +c.k +.rkees 25.10 |rc|rr.t| |eJs 16.14
||c.gc ubs 251.96 V|rresct. .|rs 165.1
|e. +c.k Vets 251.19 b.|t|nc.e O.|c|es 165.40
c.crtc b|ue '.ys 230.46 ||c.|J. V..||rs 164.26
|cs Arge|es |cJge.s 229.14 A ||.ncrJb.cks 162.84
St. |cu|s ..J|r.|s 21.28 c|c..Jc |cck|es 160.00
|custcr Ast.cs 215.45 At|.rt. b..ves 15.15
||c.gc \||te Scx 214.51 |.rs.s |ty |cy.|s 151.16
O.k|.rJ At||et|cs 206.80 ex.s |.rge.s 148.04
S.r ||egc |.J.es 201.2 ||ttsbu.g| ||..tes 146.32
||||.Je|p||. ||||||es 199.56 V||..ukee b.e.e.s 141.52
\.s||rgtcr |.t|cr.|s 195.50 |cs Arge|es Arge|s 140.42
|eve|.rJ |rJ|.rs 192.38 .np. b.y |.ys 136.31
Se.tt|e V..|re.s 191.16 ML8 Averoge $!9!.92
|et.c|t |ge.s 190.13
Sources Team Marketing Report, TMRs Fan Cost Index Major League Baseball 2008 at http//www.teammarketing.com and personal
interview.
A N S W R 7 O 7 R Y I 7 ! P R O 8 L M
|e....rg|rg |,u.t|cr 20.3, .e.| C|| rcn|r.| C||/|np||c|t p.|ce Je|.tc.. |e.e|c.e,
Real GDP in 2003 = $10 tiillion. / 1.063 = $9.4 tiillion.
Real GDP in 2004 = $11 tiillion / 1.091 = $10.1 tiillion.
|us, |r t||s eccrcny |r .e.| te.ns, C|| |.s g.c.r by 0. t.||||cr.
c |rJ t|e ..te c| |r|.t|cr, .e .e|e. tc |,u.t|cr 20.4, .rJ .e c.|cu|.te.
Inflation iate in 2004 = (1.091 1.063) / 1.063 = 0.026 = 2.6
|us, t|e p.|ce |eve| .cse 2.6 bet.eer 2003 .rJ 2004.
3. UNEMPLOYMENT
L A R N I N G O 8 1 C 7 I V S
1. xpIain how unempIoyment is measured in the United States.
2. Dene three dierent types of unempIoyment.
3. Dene and iIIustrate graphicaIIy what is meant by the naturaI IeveI of empIoyment. ReIate the
naturaI IeveI of empIoyment to the naturaI rate of unempIoyment.
Foi an economy to pioduce all it can and achieve a solution on its pioduction possibilities cuive, the
factois of pioduction in the economy must be fully employed. Failuie to fully employ these factois
leads to a solution inside the pioduction possibilities cuive in which society is not achieving the output
it is capable of pioducing.
510 PRINCIPLS OI CONOMICS
Iabor force
|e tct.| runbe. c| pecp|e
.c.k|rg c. urenp|cyeJ.
unempIoyment rate
|e pe.cert.ge c| t|e |.bc.
|c.ce t|.t |s urenp|cyeJ.
In thinking about the employment of society's factois of pioduction, we place special emphasis on
laboi. The loss of a job can wipe out a household's entiie income; it is a moie compelling human piob-
lem than, say, unemployed capital, such as a vacant apaitment. In measuiing unemployment, we thus
focus on laboi iathei than on capital and natuial iesouices.
3.1 Measuiing Unemployment
The Buieau of Laboi Statistics defnes a peison as unemployed if he oi she is not woiking but is look-
ing foi and available foi woik. The labor force is the total numbei of people woiking oi unemployed.
The unemployment rate is the peicentage of the laboi foice that is unemployed.
To estimate the unemployment iate, goveinment suiveyois fan out acioss the countiy each month
to visit ioughly 60,000 households. At each of these iandomly selected households, the suiveyoi asks
about the employment status of each adult (eveiyone age 16 oi ovei) who lives theie. Many households
include moie than one adult; the suivey gatheis infoimation on about ioughly 100,000 adults. The sui-
veyoi asks if each adult is woiking. If the answei is yes, the peison is counted as employed. If the an-
swei is no, the suiveyoi asks if that peison has looked foi woik at some time duiing the pievious foui
weeks and is available foi woik at the time of the suivey. If the answei to that question is yes, the pei-
son is counted as unemployed. If the answei is no, that peison is not counted as a membei of the laboi
foice. Figuie 20.8 shows the suivey's iesults foi the civilian (nonmilitaiy) population foi Octobei 2008.
The unemployment iate is then computed as the numbei of people unemployed divided by the laboi
foice-the sum of the numbei of people not woiking but available and looking foi woik plus the num-
bei of people woiking. In Octobei 2008, the unemployment iate was 6.3.
II GUR 20. 8 Computing the UnempIoyment Rate
A ncrt||y su.vey c| |cuse|c|Js J|v|Jes t|e c|v|||.r .Ju|t pcpu|.t|cr |rtc t|.ee g.cups. |cse .|c |.ve cbs ..e
ccurteJ .s enp|cyeJ, t|cse .|c Jc rct |.ve cbs but ..e |cck|rg |c. t|en .rJ ..e .v.||.b|e |c. .c.k ..e ccurteJ
.s urenp|cyeJ, .rJ t|cse .|c ..e rct .c.k|rg .rJ ..e rct |cck|rg |c. .c.k ..e rct ccurteJ .s nenbe.s c| t|e
|.bc. |c.ce. |e urenp|cynert ..te e,u.|s t|e runbe. c| pecp|e |cck|rg |c. .c.k J|v|JeJ by t|e sun c| t|e
runbe. c| pecp|e |cck|rg |c. .c.k .rJ t|e runbe. c| pecp|e enp|cyeJ. \.|ues ..e |c. Octcbe. 2008. A|| runbe.s
..e |r t|cus.rJs.
Theie aie seveial dimculties with the suivey. The old suivey, designed duiing the 1930s, put the Aie
you woiking:" question difeiently depending on whethei the iespondent was a man oi woman. A man
was asked, Last week, did you do any woik foi pay oi pioft:" A woman was asked, What weie you
doing foi woik last week, keeping house oi something else:" Consequently, many women who weie
looking foi paid woik stated that they weie keeping house"; those women weie not counted as unem-
ployed. The BLS did not get aiound to fxing the suivey-asking women the same question it asked
men-until 1994. The fist time the new suivey question was used, the unemployment iate among wo-
men iose by 0.3 peicentage point. Moie than 30 million women aie in the laboi foice; the change ad-
ded moie than a quaitei of a million woikeis to the omcial count of the unemployed.
[10]
CHAP7R 20 MACROCONOMICS: 7H 8IG PIC7UR 511
naturaI IeveI of
empIoyment
|e enp|cynert |eve| .t
.||c| t|e ,u.rt|ty c| |.bc.
Jen.rJeJ e,u.|s t|e
,u.rt|ty supp||eJ.
The pioblem of undeistating unemployment among women has been fxed, but otheis iemain. A
woikei who has been cut back to pait-time woik still counts as employed, even if that woikei would
piefei to woik full time. A peison who is out of woik, would like to woik, has looked foi woik in the
past yeai, and is available foi woik, but who has given up looking, is consideied a discouiaged woikei.
Discouiaged woikeis aie not counted as unemployed, but a tally is kept each month of the numbei of
discouiaged woikeis.
The omcial measuies of employment and unemployment can yield unexpected iesults. Foi ex-
ample, when fims expand output, they may be ieluctant to hiie additional woikeis until they can be
suie the demand foi incieased output will be sustained. They may iespond fist by extending the houis
of employees pieviously ieduced to pait-time woik oi by asking full-time peisonnel to woik oveitime.
None of that will inciease employment, because people aie simply counted as employed" if they aie
woiking, iegaidless of how much oi how little they aie woiking. In addition, an economic expansion
may make discouiaged woikeis moie optimistic about job piospects, and they may iesume theii job
seaiches. Engaging in a seaich makes them unemployed again-and incieases unemployment. Thus,
an economic expansion may have little efect initially on employment and may even inciease
unemployment.
3.2 Types of Unemployment
Woikeis may fnd themselves unemployed foi difeient ieasons. Each souice of unemployment has
quite difeient implications, not only foi the woikeis it afects but also foi public policy.
Figuie 20.9 applies the demand and supply model to the laboi maiket. The piice of laboi is taken
as the ieal wage, which is the nominal wage divided by the piice level; the symbol used to iepiesent the
ieal wage is the Gieek lettei omega, a. The supply cuive is diawn as upwaid sloping, though steep, to
iefect studies showing that the quantity of laboi supplied at any one time is neaily fxed. Thus, an in-
ciease in the ieal wage induces a ielatively small inciease in the quantity of laboi supplied. The demand
cuive shows the quantity of laboi demanded at each ieal wage. The lowei the ieal wage, the gieatei the
quantity of laboi fims will demand. In the case shown heie, the ieal wage, a
e
, equals the equilibiium
solution defned by the inteisection of the demand cuive D
1
and the supply cuive S
1
. The quantity of
laboi demanded, L
e
, equals the quantity supplied. The employment level at which the quantity of laboi
demanded equals the quantity supplied is called the natural level of employment. It is sometimes
iefeiied to as full employment.
II GUR 20. 9 7he NaturaI LeveI of mpIoyment
|e enp|cynert |eve| .t .||c| t|e ,u.rt|ty c| |.bc. Jen.rJeJ e,u.|s t|e ,u.rt|ty supp||eJ |s c.||eJ t|e r.tu..|
|eve| c| enp|cynert. |e.e, t|e r.tu..| |eve| c| enp|cynert |s |
e
, .||c| |s .c||eveJ .t . .e.| ..ge .
e
.
512 PRINCIPLS OI CONOMICS
naturaI rate of
unempIoyment
|e ..te c| urenp|cynert
ccrs|stert .|t| t|e r.tu..|
|eve| c| enp|cynert.
frictionaI unempIoyment
|renp|cynert t|.t cccu.s
bec.use |t t.kes t|ne |c.
enp|cye.s .rJ .c.ke.s tc
|rJ e.c| ct|e..
structuraI unempIoyment
|renp|cynert t|.t .esu|ts
|.cn . n|sn.tc| bet.eer
.c.ke. ,u.|||c.t|crs .rJ t|e
c|...cte.|st|cs enp|cye.s
.e,u|.e.
Even if the economy is opeiating at its natuial level of employment, theie will still be some unemploy-
ment. The iate of unemployment consistent with the natuial level of employment is called the natural
rate of unemployment. Business cycles may geneiate additional unemployment. We discuss these
vaiious souices of unemployment below.
IrictionaI UnempIoyment
Even when the quantity of laboi demanded equals the quantity of laboi supplied, not all employeis and
potential woikeis have found each othei. Some woikeis aie looking foi jobs, and some employeis aie
looking foi woikeis. Duiing the time it takes to match them up, the woikeis aie unemployed. Unem-
ployment that occuis because it takes time foi employeis and woikeis to fnd each othei is called fric-
tional unemployment.
The case of college giaduates engaged in job seaiches is a good example of fiictional unemploy-
ment. Those who did not land a job while still in school will seek woik. Most of them will fnd jobs, but
it will take time. Duiing that time, these new giaduates will be unemployed. If infoimation about the
laboi maiket weie costless, fims and potential woikeis would instantly know eveiything they needed
to know about each othei and theie would be no need foi seaiches on the pait of woikeis and fims.
Theie would be no fiictional unemployment. But infoimation is costly. Job seaiches aie needed to pio-
duce this infoimation, and fiictional unemployment exists while the seaiches continue.
The goveinment may attempt to ieduce fiictional unemployment by focusing on its souice: in-
foimation costs. Many state agencies, foi example, seive as cleaiinghouses foi job maiket infoimation.
They encouiage fims seeking woikeis and woikeis seeking jobs to iegistei with them. To the extent
that such efoits make laboi-maiket infoimation moie ieadily available, they ieduce fiictional
unemployment.
StructuraI UnempIoyment
Anothei ieason theie can be unemployment even if employment equals its natuial level stems fiom po-
tential mismatches between the skills employeis seek and the skills potential woikeis ofei. Eveiy woik-
ei is difeient; eveiy job has its special chaiacteiistics and iequiiements. The qualifcations of job
seekeis may not match those that fims iequiie. Even if the numbei of employees fims demand equals
the numbei of woikeis available, people whose qualifcations do not satisfy what fims aie seeking will
fnd themselves without woik. Unemployment that iesults fiom a mismatch between woikei qualifca-
tions and the chaiacteiistics employeis iequiie is called structural unemployment.
Stiuctuial unemployment emeiges foi seveial ieasons. Technological change may make some
skills obsolete oi iequiie new ones. The widespiead intioduction of peisonal computeis since the
1980s, foi example, has loweied demand foi typists who lacked computei skills.
Stiuctuial unemployment can occui if too many oi too few woikeis seek tiaining oi education that
matches job iequiiements. Students cannot piedict piecisely how many jobs theie will be in a paiticu-
lai categoiy when they giaduate, and they aie not likely to know how many of theii fellow students aie
tiaining foi these jobs. Stiuctuial unemployment can easily occui if students guess wiong about how
many woikeis will be needed oi how many will be supplied.
Stiuctuial unemployment can also iesult fiom geogiaphical mismatches. Economic activity may
be booming in one iegion and slumping in anothei. It will take time foi unemployed woikeis to ielo-
cate and fnd new jobs. And pooi oi costly tianspoitation may block some uiban iesidents fiom ob-
taining jobs only a few miles away.
Public policy iesponses to stiuctuial unemployment geneially focus on job tiaining and education
to equip woikeis with the skills fims demand. The goveinment publishes iegional laboi-maiket in-
foimation, helping to infoim unemployed woikeis of wheie jobs can be found. The Noith Ameiican
Fiee Tiade Agieement (NAFTA), which cieated a fiee tiade iegion encompassing Mexico, the United
States, and Canada, has cieated some stiuctuial unemployment in the thiee countiies. In the United
States, the legislation authoiizing the pact also piovided foi job tiaining piogiams foi displaced U.S.
woikeis.
Although goveinment piogiams may ieduce fiictional and stiuctuial unemployment, they cannot
eliminate it. Infoimation in the laboi maiket will always have a cost, and that cost cieates fiictional un-
employment. An economy with changing demands foi goods and seivices, changing technology, and
changing pioduction costs will always have some sectois expanding and otheis contiacting-stiuctuial
unemployment is inevitable. An economy at its natuial level of employment will theiefoie have fiic-
tional and stiuctuial unemployment.
CHAP7R 20 MACROCONOMICS: 7H 8IG PIC7UR 513
cycIicaI unempIoyment
|renp|cynert |r excess c|
t|e urenp|cynert t|.t
ex|sts .t t|e r.tu..| |eve| c|
enp|cynert.
CycIicaI UnempIoyment
Of couise, the economy may not be opeiating at its natuial level of employment, so unemployment
may be above oi below its natuial level. In a latei chaptei we will exploie what happens when the eco-
nomy geneiates employment gieatei oi less than the natuial level. Cyclical unemployment is unem-
ployment in excess of the unemployment that exists at the natuial level of employment.
Figuie 20.10 shows the unemployment iate in the United States foi the peiiod fiom 1960 thiough
Octobei 2008. We see that it has fuctuated consideiably. How much of it coiiesponds to the natuial
iate of unemployment vaiies ovei time with changing ciicumstances. Foi example, in a countiy with a
demogiaphic bulge" of new entiants into the laboi foice, fiictional unemployment is likely to be high,
because it takes the new entiants some time to fnd theii fist jobs. This factoi alone would iaise the
natuial iate of unemployment. A demogiaphic shift towaid moie matuie woikeis would lowei the nat-
uial iate. Duiing iecessions, highlighted in Figuie 20.10, the pait of unemployment that is cyclical un-
employment giows. The analysis of fuctuations in the unemployment iate, and the goveinment's ie-
sponses to them, will occupy centei stage in much of the iemaindei of this book.
II GUR 20. 10 UnempIoyment Rate, 1960-2008
|e c|..t s|c.s t|e urenp|cynert ..te |c. e.c| ye.. |.cn 1960 tc 2008. |ecess|crs ..e s|c.r .s s|.JeJ ..e.s.
Source Economic Report of the President, 2008, Table B-42. Date for 2008 is average of frst ten months from the Bureau of Labor Statistics home
page.
k Y 7 A k A W A Y S
< |ecp|e .|c ..e rct .c.k|rg but ..e |cck|rg .rJ .v.||.b|e |c. .c.k .t .ry cre t|ne ..e ccrs|Je.eJ
urenp|cyeJ. |e urenp|cynert ..te |s t|e pe.cert.ge c| t|e |.bc. |c.ce t|.t |s urenp|cyeJ.
< \|er t|e |.bc. n..ket |s |r e,u|||b.|un, enp|cynert |s .t t|e r.tu..| |eve| .rJ t|e urenp|cynert ..te
e,u.|s t|e r.tu..| ..te c| urenp|cynert.
< |ver || enp|cynert |s .t t|e r.tu..| |eve|, t|e eccrcny .||| expe.|erce |.|ct|cr.| .rJ st.uctu..|
urenp|cynert. yc||c.| urenp|cynert |s urenp|cynert |r excess c| t|.t .sscc|.teJ .|t| t|e r.tu..|
|eve| c| enp|cynert.
7 R Y I 7 !
C|ver t|e J.t. |r t|e t.b|e, ccnpute t|e urenp|cynert ..te |r +e.. 1 .rJ |r +e.. 2. |xp|.|r .|y, |r t||s ex
.np|e, bct| t|e runbe. c| pecp|e enp|cyeJ .rJ t|e urenp|cynert ..te |rc.e.seJ.
Year Number empIoyed (in miIIions) Number unempIoyed (in miIIions)
1 20 2
2 21 2.4
514 PRINCIPLS OI CONOMICS
Case in Point: Might Increased StructuraI UnempIoyment xpIain the 1obIess
Recovery" IoIIowing the 2001 Recession1
|e |.S. 2001 .ecess|cr ..s n||J by ||stc.|c.| st.rJ..Js, but .eccve.y |r te.ns c| |rc.e.seJ enp|cynert
seeneJ p.|r|u||y s|c. |r ccn|rg. |ccrcn|sts |.|c. Ccs|er .rJ S|ncr |ctte. .t t|e |eJe..| |ese.ve b.rk c|
|e. +c.k t||rk t|e .e.scr |c. t|e s|c. .eccve.y |r cbs n.y |.ve .ctu.||y .e|ecteJ st.uctu..| c|.rges |r t|e
|.S. eccrcny. |ey ..gue t|.t Ju.|rg t|e .ecess|cr pe.n.rert ..t|e. t|.r tenpc...y |.yc|s p.eJcn|r.teJ
.rJ t|.t |t t.kes |crge. |c. |.ns tc ||.e .c.ke.s |rtc re. pcs|t|crs t|.r tc ||.e t|en b.ck |rtc |c.ne. cbs.
\|.t |s t|e|. ev|Jerce` \|er t|e |.yc| |s tenpc...y, t|e enp|cye. susperJs t|e cb, Jue tc s|.ck Jen.rJ,
.rJ t|e enp|cyee expects tc be .ec.||eJ crce Jen.rJ p|cks up. \|t| . pe.n.rert |.yc|, t|e enp|cye. e||n
|r.tes t|e cb. Sc, t|ey |cckeJ .t t|e ccrt.|but|cr c| tenpc...y |.yc|s tc t|e urenp|cynert ..te Ju.|rg t|e
.ecert .ecess|cr ccnp..eJ tc t|e s|tu.t|cr |r t|e |cu. .ecess|crs be|c.e 1990. |r t|e e..||e. .ecess|crs, uren
p|cynert |.cn tenpc...y |.yc|s .cse .|er t|e eccrcny ..s s|.|rk|rg .rJ |e|| .|te. t|e eccrcny beg.r tc
.eccve.. |r bct| t|e 1991 .rJ 2001 .ecess|crs, tenpc...y |.yc|s .e.e n|rc.. |er, t|e .ut|c.s ex.n|reJ cb
|c.s |r 0 |rJust.|es. |ey c|.ss||eJ |.yc|s |r .r |rJust.y .s be|rg cyc||c.| |r r.tu.e || t|e cb |csses Ju.|rg
t|e .ecess|cr .e.e .eve.seJ Ju.|rg t|e .eccve.y but st.uctu..| || cb |csses |c. t|e |rJust.y ccrt|rueJ Ju.|rg
t|e .eccve.y. |e|. .r.|ys|s .eve.|eJ t|.t Ju.|rg t|e .ecess|cr c| t|e e..|y 1980s, cb |csses .e.e .bcut ever|y
sp||t bet.eer cyc||c.| .rJ st.uctu..| c|.rges. |r t|e 1991 .ecess|cr .rJ t|er nc.e st.crg|y |r t|e 2001 .eces
s|cr, st.uctu..| c|.rges Jcn|r.teJ. Vcst c| t|e |rJust.|es t|.t |cst cbs Ju.|rg t|e 2001 .ecess|cr|c. ex
.np|e, ccnnur|c.t|crs, e|ect.cr|c e,u|pnert, .rJ secu.|t|es .rJ ccnncJ|t|es b.cke.s.e.e st||| |cs|rg
cbs |r 2003. |e t.erJ .eve.|eJ . . . |s cre |r .||c| cbs ..e .e|cc.teJ |.cn scne |rJust.|es tc ct|e.s, rct .e
c|.|neJ by t|e s.ne |rJust.|es t|.t |cst t|en e..||e..
|e .ut|c.s suggest t|.ee pcss|b|e .e.scrs |c. t|e .ecert |rc.e.seJ .c|e c| st.uctu..| c|.rge. (1) |e st.uctu..|
Jec||re |r scne |rJust.|es ccu|J be t|e .esu|t c| cve.exp.rs|cr |r t|cse |rJust.|es Ju.|rg t|e 1990s. |e ||g|
tec| .rJ te|eccnnur|c.t|crs |rJust.|es |r p..t|cu|.. ccu|J be ex.np|es c| |rJust.|es t|.t .e.e cve.bu||t be
|c.e t|e 2001 .ecess|cr. (2) |np.cveJ gcve.rnert pc||c|es n.y |.ve .eJuceJ cyc||c.| urenp|cynert. |x.n
|r.t|cr c| n.c.ceccrcn|c pc||cy |r |utu.e c|.pte.s .||| .etu.r tc t||s |ssue. (3) |e. n.r.genert st..teg|es tc
.eJuce ccsts n.y be p.cnct|rg |e.re. st.|rg. |c. |.ns .Jcpt|rg suc| st..teg|es, . .ecess|cr n.y p.cv|Je .r
cppc.tur|ty tc .ec.g.r|.e t|e p.cJuct|cr p.ccess pe.n.rert|y .rJ .eJuce p.y.c||s |r t|e p.ccess.
Ccs|er .rJ |ctte. pc|rt cut t|.t, |c. .c.ke.s, |rJ|rg re. cbs |s |..Je. t|.r s|np|y .etu.r|rg tc c|J cres.
|c. |.ns, n.k|rg Jec|s|crs .bcut t|e r.tu.e c| re. cbs |s t|ne ccrsun|rg .t best. |e urce.t.|rty c.e.teJ
by t|e ... |r |.., .rJ t|e |npcs|t|cr c| re. .cccurt|rg st.rJ..Js |c||c.|rg t|e |r.cr||ke sc.rJ.|s n.y
|.ve |u.t|e. p.c|crgeJ t|e c.e.t|cr c| re. cbs.
5ource ||co | Oo|e oJ .|o |o||e. |o .|oc|oo| |oe o||o|eJ |o o .o|e |eco.e,.' |eJeo| |ee.e |o| o| |e. `o| oe| |oe
| |coo|c oJ ||oce 9. o S (^oo| 2003: .
CHAP7R 20 MACROCONOMICS: 7H 8IG PIC7UR 515
A N S W R 7 O 7 R Y I 7 ! P R O 8 L M
|r +e.. 1 t|e tct.| |.bc. |c.ce |rc|uJes 22 n||||cr .c.ke.s, .rJ sc t|e urenp|cynert ..te |s 2/22 9.1. |r
+e.. 2 t|e tct.| |.bc. |c.ce runbe.s 23.4 n||||cr .c.ke.s, t|e.e|c.e t|e urenp|cynert ..te |s 2.4/23.4
10.3. |r t||s ex.np|e, bct| t|e runbe. c| pecp|e enp|cyeJ .rJ t|e urenp|cynert ..te .cse, bec.use
nc.e pecp|e (23.4 22 1.4 n||||cr) erte.eJ t|e |.bc. |c.ce, c| .|cn 1 n||||cr |curJ cbs .rJ 0.4 n||||cr
.e.e st||| |cck|rg |c. cbs.
4. REVIEW AND PRACTICE
Summary
|r t||s c|.pte. .e ex.n|reJ g.c.t| |r .e.| C|| .rJ bus|ress cyc|es, p.|ce|eve| c|.rges, .rJ urenp|cynert.
\e s.. |c. t|ese p|ercner. ..e Je|reJ .rJ |cckeJ .t t|e|. ccrse,uerces.
|x.n|r|rg .e.| C||, ..t|e. t|.r rcn|r.| C||, cve. t|ne te||s us .|et|e. t|e eccrcny |s exp.rJ|rg c. ccr
t..ct|rg. |e.| C|| |r t|e |r|teJ St.tes s|c.s . |crg up...J t.erJ, but .|t| t|e eccrcny gc|rg t|.cug|
p|.ses c| exp.rs|cr .rJ .ecess|cr ..curJ t|.t t.erJ. |ese p|.ses n.ke up t|e bus|ress cyc|e. Ar exp.rs|cr
.e.c|es . pe.k, .rJ t|e eccrcny |.||s |rtc . .ecess|cr. |e .ecess|cr .e.c|es . t.cug| .rJ beg|rs .r exp.r
s|cr .g.|r.
|r|.t|cr |s .r |rc.e.se |r t|e p.|ce |eve| .rJ Je|.t|cr |s . Jec.e.se |r t|e p.|ce |eve|. |e ..te c| |r|.t|cr c.
Je|.t|cr |s t|e pe.cert.ge ..te c| c|.rge |r . p.|ce |rJex. \e |cckeJ .t t|e c.|cu|.t|cr c| t|e ccrsune. p.|ce
|rJex (||) .rJ t|e |np||c|t p.|ce Je|.tc.. |e || |s .|Je|y useJ |r t|e c.|cu|.t|cr c| p.|ce|eve| c|.rges.
|e.e ..e, |c.eve., b|.ses |r |ts c.|cu|.t|cr. t|e subst|tut|cr b|.s, t|e re.p.cJuct b|.s, t|e ,u.||tyc|.rge b|
.s, .rJ t|e cut|et b|.s.
|r|.t|cr .rJ Je|.t|cr .|ect eccrcn|c .ct|v|ty |r seve..| ..ys. |ey c|.rge t|e v.|ue c| ncrey .rJ c| c|.|ns
cr ncrey. |rexpecteJ |r|.t|cr bere|ts bc..c.e.s .rJ |u.ts |erJe.s. |rexpecteJ Je|.t|cr bere|ts |erJe.s
.rJ |u.ts bc..c.e.s. bct| |r|.t|cr .rJ Je|.t|cr c.e.te urce.t.|rty .rJ n.ke |t J||cu|t |c. |rJ|v|Ju.|s .rJ
|.ns tc erte. |rtc |crgte.n |r.rc|.| ccnn|tnerts.
|e urenp|cynert ..te |s ne.su.eJ .s t|e pe.cert.ge c| t|e |.bc. |c.ce rct .c.k|rg but seek|rg .c.k. |.|c
t|cr.| urenp|cynert cccu.s bec.use |r|c.n.t|cr .bcut t|e |.bc. n..ket |s ccst|y, |t t.kes t|ne |c. |.ns seek
|rg .c.ke.s .rJ .c.ke.s seek|rg |.ns tc |rJ e.c| ct|e.. St.uctu..| urenp|cynert cccu.s .|er t|e.e |s .
n|sn.tc| bet.eer t|e sk|||s c|e.eJ by pctert|.| .c.ke.s .rJ t|e sk|||s scug|t by |.ns. bct| |.|ct|cr.| .rJ
st.uctu..| urenp|cynert cccu. ever || enp|cynert .rJ t|e urenp|cynert ..te ..e .t t|e|. r.tu..| |eve|s.
yc||c.| urenp|cynert |s urenp|cynert t|.t |s |r excess c| t|.t .sscc|.teJ .|t| t|e r.tu..| |eve| c|
enp|cynert.
516 PRINCIPLS OI CONOMICS
C O N C P 7 P R O 8 L M S
1. |esc.|be t|e p|.ses c| . bus|ress cyc|e.
2. Or t|e b.s|s c| .ecert re.s .epc.ts, .|.t p|.se c| t|e bus|ress cyc|e Jc ycu t||rk t|e eccrcny |s |r
rc.` \|.t |s t|e |r|.t|cr c. Je|.t|cr ..te` |e urenp|cynert ..te`
3. Suppcse ycu ccnp..e ycu. |rccne t||s ye.. .rJ |.st ye.. .rJ |rJ t|.t ycu. rcn|r.| |rccne |e|| but ycu.
.e.| |rccne .cse. |c. ccu|J t||s |.ve |.ppereJ`
4. Suppcse ycu c.|cu|.te . g.cce.y p.|ce |r|.t|cr ..te. |s|rg t|e ..gunerts p.eserteJ |r t|e c|.pte.,
exp|.|r pcss|b|e scu.ces c| up...J b|.s |r t|e ..te ycu c.|cu|.te, .e|.t|ve tc t|e .ctu.| t.erJ c| |ccJ p.|ces.
5. |.ne t|.ee |tens ycu |.ve pu.c|.seJ Ju.|rg t|e p.st ye.. t|.t |.ve |rc.e.seJ |r ,u.||ty Ju.|rg t|e ye...
\|.t k|rJ c| .Justnert .cu|J ycu n.ke |r .ssess|rg t|e|. p.|ces |c. t|e ||`
6. \|y Jc scne pecp|e g.|r .rJ ct|e. pecp|e |cse |.cn |r|.t|cr .rJ Je|.t|cr`
. Suppcse urenp|cyeJ pecp|e |e.ve . st.te tc cbt.|r cbs |r ct|e. st.tes. \|.t Jc ycu p.eJ|ct .||| |.pper
tc t|e urenp|cynert ..te |r t|e st.te expe.|erc|rg t|e cutn|g..t|cr` \|.t n|g|t |.pper tc t|e
urenp|cynert ..tes |r t|e st.tes expe.|erc|rg |rn|g..t|cr`
8. V|rc.|ty teer.ge.s |.ve t|e ||g|est urenp|cynert ..tes c| .ry g.cup. Ore .e.scr |c. t||s p|ercnercr
|s ||g| t..rspc.t.t|cr ccsts |c. n.ry n|rc.|ty teers. \|.t |c.n c| urenp|cynert (cyc||c.|, |.|ct|cr.|, c.
st.uctu..|) Jc ||g| t..rspc.t.t|cr ccsts suggest`
9. \e||..e .e|c.ns er.cteJ |r 1996 put nc.e p.essu.e cr .e||..e .ec|p|erts tc |cck |c. .c.k. |e re. |..
n.rJ.teJ cutt|rg c| bere|ts .|te. . ce.t.|r |ergt| c| t|ne. |c. Jc ycu t||rk t||s p.cv|s|cr n|g|t .|ect
t|e urenp|cynert ..te`
10. Ane.|c.r .c.ke.s .c.k nc.e |cu.s t|.r t|e|. |u.cpe.r ccurte.p..ts. S|cu|J crg.ess |eg|s|.te . s|c.te.
.c.k.eek`
CHAP7R 20 MACROCONOMICS: 7H 8IG PIC7UR 517
N U M R I C A L P R O 8 L M S
1. ||ct t|e ,u..te.|y J.t. |c. .e.| C|| |c. t|e |.st t.c ye..s. (+cu c.r |rJ t|e J.t. |r t|e .o.e, o| oe|
|o|e c. |r oe| |coo|c |J|co|o |r t|e cu..ert pe.|cJ|c.|s sect|cr c| ycu. ||b...y. A|te.r.t|ve|y, gc
tc t|e \||te |cuse, |ccrcn|c St.t|st|cs b.|e|rg |ccn .t |ttp.//.....||te|cuse.gcv/|sb./esb..|tn|.
|e|.te .ecert c|.rges |r .e.| C|| tc t|e ccrcept c| t|e p|.ses c| t|e bus|ress cyc|e.)
2. Suppcse t|.t |r 2009, t|e |tens |r t|e n..ket b.sket |c. cu. ncv|e p.|ce |rJex ccst 53.40. |se t|e
|r|c.n.t|cr |r t|e c|.pte. tc ccnpute t|e p.|ce |rJex |c. t|.t ye... |c. Jces t|e ..te c| ncv|e p.|ce
|r|.t|cr |.cn 2008 tc 2009 ccnp..e .|t| t|e ..te |.cn 200 tc 2008`
3. |eccnpute t|e ncv|e p.|ce |rJexes |c. 200 .rJ 2008 us|rg 2008 .s t|e b.se ye... |c. ccnpute t|e
..te c| |r|.t|cr |c. t|e 2002008 pe.|cJ. cnp..e ycu. .esu|t tc t|e |r|.t|cr ..te c.|cu|.teJ |c. t|.t
s.ne pe.|cJ us|rg 200 .s t|e b.se ye...
4. |e.e ..e scne st.t|st|cs |c. August 2006. cnpute t|e urenp|cynert ..te |c. t|.t ncrt| (.|| |gu.es ..e
|r t|cus.rJs).
|cpu|.t|cr (|v|||.r, rcr|rst|tut|cr.|) 229,16
|v|||.r |.bc. |c.ce 151,698
|..t|c|p.t|cr |.te 66.2
|ct |r |.bc. |c.ce ,469
|np|cyeJ 144,59
|renp|cyeJ ,119
5. Suppcse .r eccrcny |.s 10,000 pecp|e .|c ..e rct .c.k|rg but |cck|rg .rJ .v.||.b|e |c. .c.k .rJ
90,000 pecp|e .|c ..e .c.k|rg. \|.t |s |ts urenp|cynert ..te` |c. suppcse 4,000 c| t|e pecp|e
|cck|rg |c. .c.k get J|sccu..geJ .rJ g|ve up t|e|. se..c|es. \|.t |.ppers tc t|e urenp|cynert ..te`
\cu|J ycu |rte.p.et t||s .s gccJ re.s |c. t|e eccrcny c. b.J re.s` |xp|.|r.
6. |e .ve..ge p.|ce c| gc|rg tc . b.seb.|| g.ne |r 2008, b.seJ cr t|e cbse.v.t|crs |r t|e .se |r |c|rt, ..s
191.92. |s|rg t||s .ve..ge .s t|e e,u|v.|ert c| . b.se ye.., ccnpute |.r p.|ce |rJexes |c..
.. |e |e. +c.k +.rkees.
b. |e ||c.gc ubs.
c. |e bcstcr |eJ Scx.
J. |e .np. b.y |.ys.
e. |e te.n c| ycu. c|c|ce.
. Suppcse ycu ..e g|ver t|e |c||c.|rg J.t. |c. . sn.|| eccrcny.
|unbe. c| urenp|cyeJ .c.ke.s. 1,000,000.
|.bc. |c.ce. 10,000,000.
b.seJ cr t||s J.t., .rs.e. |c||c.|rg.
.. \|.t |s t|e urenp|cynert ..te`
b. .r ycu Jete.n|re .|et|e. t|e eccrcny |s cpe..t|rg .t |ts |u|| enp|cynert |eve|`
c. |c. suppcse pecp|e .|c |.J beer seek|rg cbs beccne J|sccu..geJ, .rJ g|ve up t|e|. cb
se..c|es. |e |.bc. |c.ce s|.|rks tc 900,500 .c.ke.s, .rJ urenp|cynert |.||s tc 500,000 .c.ke.s.
\|.t |s t|e urenp|cynert ..te rc.` |.s t|e eccrcny |np.cveJ`
8. |cn|r.| C|| |c. .r eccrcny |s 10 t.||||cr. |e.| C|| |s 9 t.||||cr. \|.t |s t|e v.|ue c| t|e |np||c|t p.|ce
Je|.tc.`
9. Suppcse ycu ..e g|ver t|e |c||c.|rg J.t. |c. .r eccrcny.
518 PRINCIPLS OI CONOMICS
Month ReaI GDP mpIoyment
1 10.0 t.||||cr 100 n||||cr
2 10.4 t.||||cr 104 n||||cr
3 10.5 t.||||cr 105 n||||cr
4 10.3 t.||||cr 103 n||||cr
5 10.2 t.||||cr 102 n||||cr
6 10.3 t.||||cr 103 n||||cr
10.6 t.||||cr 106 n||||cr
8 10. t.||||cr 10 n||||cr
9 10.6 t.||||cr 106 n||||cr
.. ||ct t|e J.t. |c. .e.| C||, .|t| t|e t|ne pe.|cJ cr t|e |c.|.crt.| .x|s .rJ .e.| C|| cr t|e ve.t|c.|
.x|s.
b. |e.e ..e t.c pe.ks. \|er Jc t|ey cccu.`
c. \|er Jces t|e t.cug| cccu.`
10. |e crsune. |.|ce |rJex |r |e.|cJ 1 |s 10.5. |t |s 103.8 |r |e.|cJ 2. |s t||s . s|tu.t|cr c| |r|.t|cr |c.
Je|.t|cr` \|.t |s t|e ..te`
CHAP7R 20 MACROCONOMICS: 7H 8IG PIC7UR 519
1.
2.
3.
4.
5.
6.
.
8.
9.
10.
ENDNOTES
|usse|| |cbe.ts, |c. Ccve.rnert StckeJ t|e V.r|., /o|| .|ee| .ooo|, Octcbe. 3,
2008, p. A21.
bu.e.u c| |ccrcn|c Ar.|ys|s, p.ess .e|e.se, |cvenbe. 25, 2008.
|e |b||s |ecess|cr |.t|rg |.cceJu.e, |.t|cr.| bu.e.u c| |ccrcn|c |ese..c|,
'.ru..y , 2008.
Scne eccrcn|sts p.e|e. tc b.e.k t|e exp.rs|cr p|.se |rtc t.c p..ts. |e .eccve.y
p|.se |s s.|J tc be t|e pe.|cJ bet.eer t|e p.ev|cus t.cug| .rJ t|e t|ne .|er t|e
eccrcny .c||eves |ts p.ev|cus pe.k |eve| c| .e.| C||. |e exp.rs|cr p|.se |s |.cn
t|.t pc|rt urt|| t|e |c||c.|rg pe.k.
Ar.|| At|.v.|ey, |c. |.ss c| 08, . Sc..nb|e |c. 'cbs, /o|| .|ee| .ooo|, Ap.|| 8,
2008, |1.
.n.. |e.|r, A |en||re |rJex, |pJ.teJ, |e. `o| |e, Octcbe. 19, 2008, Sect|cr
\|, 1.
|nb.b.e |r|.t|cr ||ts 11,200,000, ||.ccn, August 19, 2008.
|c. . ccnp..|scr c| p.|ce ne.su.es, |rc|uJ|rg . ccnp..|scr c| t|e || p.|ce |rJex
.rJ t|e crsune. |.|ce |rJex, see b..|r . Vcye., cnp..|rg |.|ce Ve.su.es|e
|| .rJ || |.|ce |rJex (|ectu.e, |.t|cr.| Asscc|.t|cr |c. bus|ress |ccrcn|cs, 2006
\.s||rgtcr |ccrcn|c |c||cy cr|e.erce, V..c| 1314, 2006), .v.||.b|e .t
|ttp.//....be..gcv/be./p.pe.s.|tn.
|cbe.t .ge, 'c|r C.eer|ees, .rJ |.t.|ck '.ckn.r, |rt.cJuc|rg t|e |.|reJ cr
sune. |.|ce |rJex (p.pe., Severt| Veet|rg c| t|e |rte.r.t|cr.| \c.k|rg C.cup cr
|.|ce |rJ|ces, |..|s, |..rce, V.y 2003), .v.||.b|e .t |ttp.//st.ts.b|s.gcv/cp|/
supe.||rk.|tn.
|c. . Jesc.|pt|cr c| t|e re. su.vey .rJ ct|e. c|.rges |rt.cJuceJ |r t|e net|cJ c|
ccurt|rg urenp|cynert, see '.ret |. |c..ccJ .rJ 'uJ|t| V. .ru.,
|renp|cynert Ve.su.es |c. t|e ||ret|es, |o||c ||o (oo|e|, 58, rc. 2
(Sunne. 1994). 294.
520 PRINCIPLS OI CONOMICS
| A | | | 2 1
Measuring Total Output and
Income
S7AR7 UP: 7H LOCkUP
|t |s e..|y nc.r|rg .|er . |.||Jc.er ser|c. c|c|.|s erte. t|e .ccn .t t|e cnne.ce |ep..tnert |r \.s||rgtcr.
Orce |rs|Je, t|ey .||| |.ve rc ccnnur|c.t|cr .|t| t|e cuts|Je .c.|J urt|| t|ey |.ve ccnp|eteJ t|e|. .c.k |.te.
t|.t J.y. |ey .||| |.ve rc te|ep|cre, rc ccnpute. ||rks. |ey .||| be .b|e tc s||p cut tc .r .Jc|r|rg .est.ccn, but
cr|y |r p.|.s. |t |s rc .crJe. t|e .ccn |s c.||eJ t|e |cckup. |e |cckup p.cJuces cre c| t|e ncst |npc.t.rt |r
J|c.tc.s c| eccrcn|c .ct|v|ty .e |.ve. t|e c|c|.| est|n.te c| t|e v.|ue c| t|e eccrcnys tct.| cutput, krc.r .s |ts
g.css Jcnest|c p.cJuct (C||).
\|er t|e te.n |.s |r|s|eJ |ts ccnput.t|crs, t|e .esu|ts .||| be p|.ceJ |r . se.|eJ erve|cpe. A gcve.rnert
nesserge. .||| |.rJ c...y t|e erve|cpe tc t|e |xecut|ve O|ce bu||J|rg .rJ Je||ve. |t tc . ser|c. .Jv|se. tc t|e
p.es|Jert c| t|e |r|teJ St.tes. |e .Jv|se. .||| ex.n|re |ts ccrterts, t|er c...y |t .c.css t|e st.eet tc t|e \||te
|cuse .rJ g|ve |t tc t|e p.es|Jert.
|e ser|c. c|c|.|s .|c neet |r sec.et tc ccnpute C|| ..e rct sp|es, t|ey ..e eccrcn|sts. |e .Jv|se. .|c
Je||ve.s t|e est|n.te tc t|e p.es|Jert |s t|e c|.|.n.r c| t|e curc|| c| |ccrcn|c AJv|se.s.
|e e|.bc..te p.ec.ut|crs |c. sec.ecy Jc rct erJ t|e.e. At .30 t|e rext nc.r|rg, cu.r.||sts |.cn .|| cve. t|e
.c.|J .||| g.t|e. |r .r e|ect.cr|c.||y se.|eJ .uJ|tc.|un .t t|e cnne.ce |ep..tnert. |e.e t|ey .||| be g|ver t|e
C|| |gu.e .rJ .e|.teJ eccrcn|c |rJ|c.tc.s, .|crg .|t| .r exp|.r.t|cr. |e .epc.te.s .||| |.ve .r |cu. tc p.ep..e
t|e|. .epc.ts, but t|ey .||| rct be .b|e tc ccnnur|c.te .|t| .rycre e|se urt|| .r c|c|.| t|.c.s t|e s.|tc| .t 8.30.
At t|.t |rst.rt t|e|. ccnpute.s .||| ccrrect tc t|e|. re.s se.v|ces, .rJ t|ey .||| be .b|e tc ||e t|e|. .epc.ts. |ese
.||| be n.c. stc.|es cr t|e |rte.ret .rJ |r t|e rext eJ|t|crs c| t|e r.t|crs re.sp.pe.s, t|e est|n.te c| t|e p.ev|
cus ,u..te.s C|| .||| be cre c| t|e |e.J |tens cr te|ev|s|cr .rJ ..J|c re.s b.c.Jc.sts t|.t J.y.
|e c|.rJest|re p.ep...t|crs |c. t|e .e|e.se c| ,u..te.|y C|| |gu.es .e|ect t|e |npc.t.rce c| t||s |rJ|c.tc..
|e est|n.te c| C|| p.cv|Jes t|e best .v.||.b|e .e.J|rg c| n.c.ceccrcn|c pe.|c.n.rce. |t .||| .|ect gcve.rnert
pc||cy, .rJ |t .||| |r|uerce n||||crs c| Jec|s|crs |r t|e p.|v.te sectc.. |.|c. krc.|eJge c| t|e C|| est|n.te ccu|J be
useJ tc .rt|c|p.te t|e .espcrse |r t|e stcck .rJ bcrJ n..kets, sc g.e.t c..e |s t.ker t|.t cr|y . |.rJ|u| c| t.usteJ
c|c|.|s |.ve .ccess tc t|e |r|c.n.t|cr urt|| |t |s c|c|.||y .e|e.seJ.
|e C|| est|n.te tcck cr |uge s|gr||c.rce |r t|e |.|| c| 2008 .s t|e |r|teJ St.tes .rJ nuc| c| t|e .est c| t|e
.c.|J .ert t|.cug| t|e ..erc||rg expe.|erce c| t|e .c.st |r.rc|.| c.|s|s s|rce t|e C.e.t |ep.ess|cr c| t|e 1930s.
|e expect.t|cr t|.t t|e |r.rc|.| c.|s|s .cu|J |e.J tc .r eccrcn|c cc||.pse ..s .|Jesp.e.J, .rJ t|e ,u..te.|y .r
rcurcenerts c| C|| |gu.es .e.e nc.e .rx|cus|y ...|teJ t|.r eve..
|e p.|n..y ne.su.e c| t|e ups .rJ Jc.rs c| eccrcn|c .ct|v|tyt|e bus|ress cyc|e|s .e.| C||. \|er .r
eccrcnys cutput |s .|s|rg, t|e eccrcny c.e.tes nc.e cbs, nc.e |rccne, .rJ nc.e cppc.tur|t|es |c. pecp|e. |r
0ow variabIe
A v..|.b|e t|.t |s ne.su.eJ
cve. . spec||c pe.|cJ c| t|ne.
stock variabIe
A v..|.b|e t|.t |s
|rJeperJert c| t|ne.
t|e |crg .ur, .r eccrcnys cutput .rJ |rccne, .e|.t|ve tc |ts pcpu|.t|cr, Jete.n|re t|e n.te.|.| st.rJ..J c| ||v|rg
c| |ts pecp|e.
|e..|y C|| |s .r |npc.t.rt |rJ|c.tc. c| n.c.ceccrcn|c pe.|c.n.rce. |t |s t|e tcp|c .e .||| ccrs|Je. |r t||s
c|.pte.. \e .||| |e..r t|.t C|| c.r be ne.su.eJ e|t|e. |r te.ns c| t|e tct.| v.|ue c| cutput p.cJuceJ c. .s t|e
tct.| v.|ue c| |rccne gere..teJ |r p.cJuc|rg t|.t cutput. \e .||| beg|r .|t| .r ex.n|r.t|cr c| ne.su.es c| C||
|r te.ns c| cutput. Ou. |r|t|.| |ccus .||| be cr rcn|r.| C||. t|e v.|ue c| tct.| cutput ne.su.eJ |r cu..ert p.|ces.
\e .||| tu.r tc .e.| C||. ne.su.e c| cutput t|.t |.s beer .JusteJ |c. p.|ce |eve| c|.rges|.te. |r t|e c|.pte..
\e .||| .e|e. tc rcn|r.| C|| s|np|y .s C||. \|er .e J|scuss t|e .e.| v.|ue c| t|e ne.su.e, .e .||| c.|| |t .e.| C||.
1. MEASURING TOTAL OUTPUT
L A R N I N G O 8 1 C 7 I V S
1. Dene gross domestic product and its four ma|or spending components and iIIustrate the vari-
ous 0ows using the circuIar 0ow modeI.
2. Distinguish between measuring GDP as the sum of the vaIues of naI goods and services and
as the sum of vaIues added at each stage of production.
3. Distinguish between gross domestic product and gross nationaI product.
An economy pioduces a mind-boggling aiiay of goods and seivices. In 2007, foi example, Domino's
Pizza pioduced 400 million pizzas. The United States Steel Coipoiation, the nation's laigest steel com-
pany, pioduced 23.6 million tons of steel. Stiong Biotheis Lumbei Co., a Coloiado fim, pioduced 2.1
million boaid feet of lumbei. The Louisiana State Univeisity football team diew 722,166 fans to its
home games-and won the national championship. Leonoi Montenegio, a pediatiic nuise in Los
Angeles, deliveied 387 babies and took caie of 233 additional patients. A list of all the goods and sei-
vices pioduced in any yeai would be viitually endless.
So-what kind of yeai was 2007: We would not get veiy fai tiying to wade thiough a list of all the
goods and seivices pioduced that yeai. It is helpful to have instead a single numbei that measuies total
output in the economy; that numbei is GDP.
1.1 The Components of GDP
We can divide the goods and seivices pioduced duiing any peiiod into foui bioad components, based
on who buys them. These components of GDP aie peisonal consumption (C), gioss piivate domestic
investment (I), goveinment puichases (G), and net expoits (X
n
). Thus
QUA7I ON 21. 1
GDP
GDP
=
=
consumption (C)
C
+
+
piivate investment (I)
oi
I
+
+
goveinment puichases (G)
G
+
+
net expoits (X
n
),
X
n
We will examine each of these components, and we will see how each fts into the pattein of mac-
ioeconomic activity. Befoie we begin, it will be helpful to distinguish between two types of vaiiables:
stocks and fows. A ow variable is a vaiiable that is measuied ovei a specifc peiiod of time. A stock
variable is a vaiiable that is independent of time. Income is an example of a fow vaiiable. To say one's
income is, foi example, $1,000 is meaningless without a time dimension. Is it $1,000 pei houi: Pei day:
Pei week: Pei month: Until we know the time peiiod, we have no idea what the income fguie means.
The balance in a checking account is an example of a stock vaiiable. When we leain that the balance in
a checking account is $1,000, we know piecisely what that means; we do not need a time dimension.
We will see that stock and fow vaiiables play veiy difeient ioles in macioeconomic analysis.
522 PRINCIPLS OI CONOMICS
personaI consumption
A |c. v..|.b|e t|.t ne.su.es
t|e v.|ue c| gccJs .rJ
se.v|ces pu.c|.seJ by
|cuse|c|Js Ju.|rg . t|ne
pe.|cJ.
II GUR 21. 1 PersonaI Consumption in
the CircuIar IIow
|e.scr.| ccrsunpt|cr sperJ|rg |c.s |.cn
|cuse|c|Js tc |.ns. |r .etu.r, ccrsune.
gccJs .rJ se.v|ces |c. |.cn |.ns tc
|cuse|c|Js. c p.cJuce t|e gccJs .rJ
se.v|ces |cuse|c|Js Jen.rJ, |.ns enp|cy
|.ctc.s c| p.cJuct|cr c.reJ by |cuse|c|Js.
|e.e |s t|us . |c. c| |.ctc. se.v|ces |.cn
|cuse|c|Js tc |.ns, .rJ . |c. c| p.ynerts c|
|.ctc. |rccnes |.cn |.ns tc |cuse|c|Js.
gross private domestic
investment
|e v.|ue c| .|| gccJs
p.cJuceJ Ju.|rg . pe.|cJ |c.
use |r t|e p.cJuct|cr c|
ct|e. gccJs .rJ se.v|ces.
PersonaI Consumption
Personal consumption is a fow vaiiable that measuies the value of goods and seivices puichased by
households duiing a time peiiod. Puichases by households of gioceiies, health-caie seivices, clothing,
and automobiles-all aie counted as consumption.
The pioduction of consumei goods and seivices accounts foi about 70 of total output. Because
consumption is such a laige pait of GDP, economists seeking to undeistand the deteiminants of GDP
must pay special attention to the deteiminants of consumption. In a latei chaptei we will exploie these
deteiminants and the impact of consumption on economic activity.
Peisonal consumption iepiesents a demand foi goods and seivices placed on fims by households.
In the chaptei on demand and supply, we saw how this demand could be piesented in a ciiculai fow
model of the economy. Figuie 21.1 piesents a ciiculai fow model foi an economy that pioduces only
peisonal consumption goods and seivices. (We will add the othei components of GDP to the ciiculai
fow as we discuss them.) Spending foi these goods fows fiom households to fims; it is the aiiow
labeled Peisonal consumption." Fiims pioduce these goods and seivices using factois of pioduction:
laboi, capital, and natuial iesouices. These factois aie ultimately owned by households. The pioduc-
tion of goods and seivices thus geneiates income to households; we see this income as the fow fiom
fims to households labeled Factoi incomes" in the exhibit.
In exchange foi payments that fow fiom households to fims, theie is a fow of
consumei goods and seivices fiom fims to households. This fow is shown in Figuie
21.1 as an aiiow going fiom fims to households. When you buy a soda, foi example,
youi payment to the stoie is pait of the fow of peisonal consumption; the soda is pait
of the fow of consumei goods and seivices that goes fiom the stoie to a house-
hold-youis.
Similaily, the lowei aiiow in Figuie 21.1 shows the fow of factois of pioduc-
tion-laboi, capital, and natuial iesouices-fiom households to fims. If you woik foi a
fim, youi laboi is pait of this fow. The wages you ieceive aie pait of the factoi in-
comes that fow fiom fims to households.
Theie is a key difeience in oui inteipietation of the ciiculai fow pictuie in Figuie
21.1 fiom oui analysis of the same model in the demand and supply chaptei. Theie, oui
focus was micioeconomics, which examines individual units of the economy. In think-
ing about the fow of consumption spending fiom households to fims, we emphasized
demand and supply in paiticulai maikets-maikets foi such things as blue jeans, haii-
cuts, and apaitments. In thinking about the fow of income payments fiom fims to
households, we focused on the demand and supply foi paiticulai factois of pioduction,
such as textile woikeis, baibeis, and apaitment buildings. Because oui focus now is
macioeconomics, the study of aggiegates of economic activity, we will think in teims of
the total of peisonal consumption and the total of payments to households.
Private Investment
Gross private domestic investment is the value of all goods pioduced duiing a peii-
od foi use in the pioduction of othei goods and seivices. Like peisonal consumption,
gioss piivate domestic investment is a fow vaiiable. It is often simply iefeiied to as
piivate investment." A hammei pioduced foi a caipentei is piivate investment. A
piinting piess pioduced foi a magazine publishei is piivate investment, as is a
conveyoi-belt system pioduced foi a manufactuiing fim. Capital includes all the goods
that have been pioduced foi use in pioducing othei goods; it is a stock vaiiable. Piivate
investment is a fow vaiiable that adds to the stock of capital duiing a peiiod.
Heads Up!
|e te.n |.e|e| c.r gere..te ccr|us|cr. |r eve.yJ.y ccrve.s.t|cr, .e use t|e te.n |.e|e| tc .e|e.
tc uses c| ncrey tc e..r |rccne. \e s.y .e |.ve |rvesteJ |r . stcck c. |rvesteJ |r . bcrJ. |ccrcn|sts,
|c.eve., .est.|ct |.e|e| tc .ct|v|t|es t|.t |rc.e.se t|e eccrcnys stcck c| c.p|t.|. |e pu.c|.se c| . s|..e
c| stcck Jces rct .JJ tc t|e c.p|t.| stcck, |t |s rct |rvestnert |r t|e eccrcn|c ne.r|rg c| t|e .c.J. \e .e|e.
tc t|e exc|.rge c| |r.rc|.| .ssets, suc| .s stccks c. bcrJs, .s |r.rc|.| |rvestnert tc J|st|rgu|s| |t |.cn t|e
c.e.t|cr c| c.p|t.| t|.t cccu.s .s t|e .esu|t c| |rvestnert. Or|y .|er re. c.p|t.| |s p.cJuceJ Jces |rvestnert
cccu.. cr|us|rg t|e eccrcn|c ccrcept c| p.|v.te |rvestnert .|t| t|e ccrcept c| |r.rc|.| |rvestnert c.r
c.use n|surJe.st.rJ|rg c| t|e ..y |r .||c| key ccnpcrerts c| t|e eccrcny .e|.te tc cre .rct|e..
Gioss piivate domestic investment includes thiee fows that add to oi maintain the nation's capital
stock: expendituies by business fims on new buildings, plants, tools, equipment, and softwaie that will
CHAP7R 21 MASURING 7O7AL OU7PU7 AND INCOM 523
II GUR 21. 2 Private Investment in the
CircuIar IIow
|.|v.te |rvestnert ccrst|tutes . Jen.rJ
p|.ceJ cr |.ns by ct|e. |.ns. |t .|sc
gere..tes |.ctc. |rccnes |c. |cuse|c|Js. c
s|np|||y t|e J|.g..n, cr|y t|e sperJ|rg |c.s
..e s|c.rt|e cc..espcrJ|rg |c.s c| gccJs
.rJ se.v|ces |.ve beer cn|tteJ.
government purchases
|e sun c| pu.c|.ses c|
gccJs .rJ se.v|ces |.cn
|.ns by gcve.rnert
.gerc|es p|us t|e tct.| v.|ue
c| cutput p.cJuceJ by
gcve.rnert .gerc|es
t|ense|ves Ju.|rg . t|ne
pe.|cJ.
be used in the pioduction of goods and seivices; expendituies on new iesidential housing; and changes
in business inventoiies. Any addition to a fim's inventoiies iepiesents an addition to investment; a ie-
duction subtiacts fiom investment. Foi example, if a clothing stoie stocks 1,000 paiis of jeans, the jeans
iepiesent an addition to inventoiy and aie pait of gioss piivate domestic investment. As the jeans aie
sold, they aie subtiacted fiom inventoiy and thus subtiacted fiom investment.
By iecoiding additions to inventoiies as investment and ieductions fiom inventoiies as subtiac-
tions fiom investment, the accounting foi GDP iecoids pioduction in the peiiod in which it occuis.
Suppose, foi example, that Levi Stiauss manufactuies 1 million paiis of jeans late in 2007 and distiib-
utes them to stoies at the end of Decembei. The jeans will be added to inventoiy; they thus count as in-
vestment in 2007 and entei GDP foi that yeai. Suppose they aie sold in Januaiy 2008. They will be
counted as consumption in GDP foi 2008 but subtiacted fiom inventoiy, and fiom investment. Thus,
the pioduction of the jeans will add to GDP in 2007, when they weie pioduced. They will not count in
2008, save foi any inciease in the value of the jeans iesulting fiom the seivices piovided by the ietail
stoies that sold them.
Piivate investment accounts foi about 16 of GDP. Despite its ielatively small shaie of total eco-
nomic activity, piivate investment plays a ciucial iole in the macioeconomy foi two ieasons:
1. Piivate investment iepiesents a choice to foigo cuiient consumption in oidei to add to the
capital stock of the economy. Piivate investment theiefoie adds to the economy's capacity to
pioduce and shifts its pioduction possibilities cuive outwaid. Investment is thus one deteiminant
of economic giowth, which is exploied in anothei chaptei.
2. Piivate investment is a ielatively volatile component of GDP; it can change diamatically fiom one
yeai to the next. Fluctuations in GDP aie often diiven by fuctuations in piivate investment. We
will examine the deteiminants of piivate investment in a chaptei devoted to the study of
investment.
Piivate investment iepiesents a demand placed on fims foi the pioduction of capital goods. While it is
a demand placed on fims, it fows fiom fims. In the ciiculai fow model in Figuie 21.2, we see a fow
of investment going fiom fims to fims. The pioduction of goods and seivices foi consumption genei-
ates factoi incomes to households; the pioduction of capital goods foi investment geneiates income to
households as well.
Figuie 21.2 shows only spending fows and omits the physical fows iepiesented by
the aiiows in Figuie 21.1. This simplifcation will make oui analysis of the ciiculai fow
model easiei. It will also focus oui attention on spending fows, which aie the fows we
will be studying.
Government Purchases
Goveinment agencies at all levels puichase goods and seivices fiom fims. They pui-
chase omce equipment, vehicles, buildings, janitoiial seivices, and so on. Many govein-
ment agencies also pioduce goods and seivices. Police depaitments pioduce police pio-
tection. Public schools pioduce education. The National Aeionautics and Space Ad-
ministiation (NASA) pioduces space exploiation.
Government purchases aie the sum of puichases of goods and seivices fiom
fims by goveinment agencies plus the total value of output pioduced by goveinment
agencies themselves duiing a time peiiod. Goveinment puichases make up about 20
of GDP.
524 PRINCIPLS OI CONOMICS
transfer payments
|.ynerts t|.t Jc rct .e,u|.e
t|e .ec|p|ert tc p.cJuce .
gccJ c. se.v|ce |r c.Je. tc
.ece|ve t|en.
II GUR 21. 3 Government Purchases in
the CircuIar IIow
|u.c|.ses c| gccJs .rJ se.v|ces by
gcve.rnert .gerc|es c.e.te Jen.rJs cr
|.ns. As |.ns p.cJuce t|ese gccJs .rJ
se.v|ces, t|ey c.e.te |.ctc. |rccnes |c.
|cuse|c|Js.
exports
S.|es c| . ccurt.ys gccJs
.rJ se.v|ces tc buye.s |r t|e
.est c| t|e .c.|J Ju.|rg .
p..t|cu|.. t|ne pe.|cJ.
imports
|u.c|.ses c|
|c.e|grp.cJuceJ gccJs .rJ
se.v|ces by . ccurt.ys
.es|Jerts Ju.|rg . pe.|cJ.
net exports
|xpc.ts n|rus |npc.ts.
trade decit
|eg.t|ve ret expc.ts.
trade surpIus
|cs|t|ve ret expc.ts.
Goveinment puichases aie not the same thing as goveinment spending. Much goveinment spend-
ing takes the foim of transfer payments, which aie payments that do not iequiie the iecipient to pio-
duce a good oi seivice in oidei to ieceive them. Tiansfei payments include Social Secuiity and othei
types of assistance to ietiied people, welfaie payments to pooi people, and unemployment compensa-
tion to people who have lost theii jobs. Tiansfei payments aie ceitainly signifcant-they account foi
ioughly half of all fedeial goveinment spending in the United States. They do not count in a nation's
GDP, because they do not iefect the pioduction of a good oi seivice.
Goveinment puichases iepiesent a demand placed on fims, iepiesented by the fow shown in Fig-
uie 21.3. Like all the components of GDP, the pioduction of goods and seivices foi goveinment agen-
cies cieates factoi incomes foi households.
Net xports
Sales of a countiy's goods and seivices to buyeis in the iest of the woild duiing a paitic-
ulai time peiiod iepiesent its exports. A puichase by a Japanese buyei of a Foid Taui-
us pioduced in the United States is a U.S. expoit. Expoits also include such tiansactions
as the puichase of accounting seivices fiom a New Yoik accounting fim by a shipping
line based in Hong Kong oi the puichase of a ticket to Disney Woild by a touiist fiom
Aigentina. Imports aie puichases of foieign-pioduced goods and seivices by a coun-
tiy's iesidents duiing a peiiod. United States impoits include such tiansactions as the
puichase by Ameiicans of cais pioduced in Japan oi tomatoes giown in Mexico oi a
stay in a Fiench hotel by a touiist fiom the United States. Subtiacting impoits fiom ex-
poits yields net exports.
QUA7I ON 21. 2
Expoits (X) impoits (M) = net expoits (X
n
)
In the thiid quaitei of 2008, foieign buyeis puichased $1,971.3 billion woith of
goods and seivices fiom the United States. In the same yeai, U.S. iesidents, fims, and
goveinment agencies puichased $2,677.9 billion woith of goods and seivices fiom foi-
eign countiies. The difeience between these two fguies, $706.6 billion, iepiesented
the net expoits of the U.S. economy in the thiid quaitei of 2008. Net expoits weie neg-
ative because impoits exceeded expoits. Negative net expoits constitute a trade den-
cit. The amount of the defcit is the amount by which impoits exceed expoits. When expoits exceed
impoits theie is a trade surplus. The magnitude of the suiplus is the amount by which expoits exceed
impoits.
The United States has iecoided moie defcits than suipluses since Woild Wai II, but the amounts
have typically been ielatively small, only a few billion dollais. The tiade defcit began to soai, howevei,
in the 1980s and again in the 2000s. We will examine the ieasons foi peisistent tiade defcits in anothei
chaptei. The iest of the woild plays a key iole in the domestic economy and, as we will see latei in the
book, theie is nothing paiticulaily good oi bad about tiade suipluses oi defcits. Goods and seivices
pioduced foi expoit iepiesent ioughly 14 of GDP, and the goods and seivices the United States im-
poits add signifcantly to oui standaid of living.
In the ciiculai fow diagiam in Figuie 21.4, net expoits aie shown with an aiiow connecting fims
to the iest of the woild. The balance between the fows of expoits and impoits is net expoits. When
theie is a tiade suiplus, net expoits aie positive and add spending to the ciiculai fow. A tiade defcit
implies negative net expoits; spending fows fiom fims to the iest of the woild.
CHAP7R 21 MASURING 7O7AL OU7PU7 AND INCOM 525
II GUR 21. 4 Net xports in the
CircuIar IIow
|et expc.ts .ep.esert t|e b.|.rce bet.eer
expc.ts .rJ |npc.ts. |et expc.ts c.r be
pcs|t|ve c. reg.t|ve. || t|ey ..e pcs|t|ve, ret
expc.t sperJ|rg |c.s |.cn t|e .est c| t|e
.c.|J tc |.ns. || t|ey ..e reg.t|ve, sperJ|rg
|c.s |.cn |.ns tc t|e .est c| t|e .c.|J.
II GUR 21. 5 Spending in the CircuIar
IIow ModeI
C|| e,u.|s t|e sun c| p.cJuct|cr by |.ns c|
gccJs .rJ se.v|ces |c. pe.scr.| ccrsunpt|cr
(1), p.|v.te |rvestnert (2), gcve.rnert
pu.c|.ses (3), .rJ ret expc.ts (4). |e c|.cu|..
|c. ncJe| s|c.s t|ese |c.s .rJ s|c.s t|.t
t|e p.cJuct|cr c| gccJs .rJ se.v|ces
gere..tes |.ctc. |rccnes (5) tc |cuse|c|Js.
The pioduction of goods and seivices foi peisonal consumption, piivate invest-
ment, goveinment puichases, and net expoits makes up a nation's GDP. Fiims pioduce
these goods and seivices in iesponse to demands fiom households (peisonal consump-
tion), fiom othei fims (piivate investment), fiom goveinment agencies (goveinment
puichases), and fiom the iest of the woild (net expoits). All of this pioduction cieates
factoi income foi households. Figuie 21.3 shows the ciiculai fow model foi all the
spending fows we have discussed. Each fow is numbeied foi use in the exeicise at the
end of this section.
The ciiculai fow model identifes some of the foices at woik in the economy,
foices that we will be studying in latei chapteis. Foi example, an inciease in any of the
fows that place demands on fims (peisonal consumption, piivate investment, govein-
ment puichases, and expoits) will induce fims to expand theii pioduction. This efect
is chaiacteiistic of the expansion phase of the business cycle. An inciease in pioduction
will iequiie fims to employ moie factois of pioduction, which will cieate moie income
foi households. Households aie likely to iespond with moie consumption, which will
induce still moie pioduction, moie income, and still moie consumption. Similaily, a
ieduction in any of the demands placed on fims will lead to a ieduction in output, a ie-
duction in fims' use of factois of pioduction, a ieduction in household incomes, a ie-
duction in income, and so on. This sequence of events is chaiacteiistic of the contiac-
tion phase of the business cycle. Much of oui woik in macioeconomics will involve an
analysis of the foices that piompt such changes in demand and an examination of the
economy's iesponse to them.
Figuie 21.6 shows the size of the components of GDP in 2008. We see that the pio-
duction of goods and seivices foi peisonal consumption accounted foi about 70 of
GDP. Impoits exceeded expoits, so net expoits weie negative.
II GUR 21. 6 Components of GDP, 2008 (Q3) in 8iIIions of DoIIars
crsunpt|cr n.kes up t|e |..gest s|..e c| C||. |et expc.ts .e.e reg.t|ve |r 2008. ct.|
C||t|e sun c| pe.scr.| ccrsunpt|cr, p.|v.te |rvestnert, gcve.rnert pu.c|.ses, .rJ ret
expc.tse,u.|eJ 14,420.5 b||||cr |r 2008.
1.2 Final Goods and Value Added
GDP is the total value of all fnal goods and seivices pioduced duiing a paiticulai peiiod valued at
piices in that peiiod. That is not the same as the total value of all goods and seivices pioduced duiing a
peiiod. This distinction gives us anothei method of estimating GDP in teims of output.
Suppose, foi example, that a loggei cuts some tiees and sells the logs to a sawmill. The mill makes
lumbei and sells it to a constiuction fim, which builds a house. The maiket piice foi the lumbei in-
cludes the value of the logs; the piice of the house includes the value of the lumbei. If we tiy to estimate
GDP by adding the value of the logs, the lumbei, and the house, we would be counting the lumbei
twice and the logs thiee times. This pioblem is called double counting," and the economists who com-
pute GDP seek to avoid it.
526 PRINCIPLS OI CONOMICS
vaIue added
|e .ncurt by .||c| t|e
v.|ue c| . |.ns cutput
exceeJs t|e v.|ue c| t|e
gccJs .rJ se.v|ces t|e |.n
pu.c|.ses |.cn ct|e. |.ns.
gross nationaI product
(GNP)
|e tct.| v.|ue c| |r.| gccJs
.rJ se.v|ces p.cJuceJ Ju.|rg
. p..t|cu|.. pe.|cJ .|t|
|.ctc.s c| p.cJuct|cr c.reJ
by t|e .es|Jerts c| .
p..t|cu|.. ccurt.y.
In the case of logs used foi lumbei and lumbei pioduced foi a house, GDP would include the value
of the house. The lumbei and the logs would not be counted as additional pioduction because they aie
inteimediate goods that weie pioduced foi use in building the house.
Anothei appioach to estimating the value of fnal pioduction is to estimate foi each stage of pio-
duction the value added, the amount by which the value of a fim's output exceeds the value of the
goods and seivices the fim puichases fiom othei fims. Table 21.1 illustiates the use of value added in
the pioduction of a house.
7A8L 21. 1 IinaI VaIue and VaIue Added
|| .e sun t|e v.|ue .JJeJ .t e.c| st.ge c| t|e p.cJuct|cr c| . gccJ c. se.v|ce, .e get t|e |r.| v.|ue c| t|e |ten.
|e ex.np|e s|c.r |e.e |rvc|ves t|e ccrst.uct|cr c| . |cuse, .||c| |s p.cJuceJ |.cn |unbe. t|.t |s, |r tu.r,
p.cJuceJ |.cn |cgs.
Good Produced by Purchased by Price VaIue Added
|cgs |cgge. S..n||| 12,000 12,000
|unbe. S..n||| crst.uct|cr |.n 25,000 13,000
|cuse crst.uct|cr |.n |cuse|c|J 125,000 100,000
IinaI VaIue $125,000
Sum of VaIues Added $125,000
Suppose the logs pioduced by the loggei aie sold foi $12,000 to a mill, and that the mill sells the lumbei
it pioduces fiom these logs foi $23,000 to a constiuction fim. The constiuction fim uses the lumbei
to build a house, which it sells to a household foi $123,000. (To simplify the example, we will ignoie in-
puts othei than lumbei that aie used to build the house.) The value of the fnal pioduct, the house, is
$123,000. The value added at each stage of pioduction is estimated as follows:
a. The loggei adds $12,000 by cutting the logs.
b. The mill adds $13,000 ($23,000 $12,000) by cutting the logs into lumbei.
c. The constiuction fim adds $100,000 ($123,000 $23,000) by using the lumbei to build a house.
The sum of values added at each stage ($12,000 + $13,000 + $100,000) equals the fnal value of the
house, $123,000.
The value of an economy's output in any peiiod can thus be estimated in eithei of two ways. The
values of fnal goods and seivices pioduced can be added diiectly, oi the values added at each stage in
the pioduction piocess can be added. The Commeice Depaitment uses both appioaches in its estimate
of the nation's GDP.
1.3 GNP: An Alteinative Measuie of Output
While GDP iepiesents the most commonly used measuie of an economy's output, economists some-
times use an alteinative measuie. Gross national product (GNP) is the total value of fnal goods and
seivices pioduced duiing a paiticulai peiiod with factois of pioduction owned by the iesidents of a
paiticulai countiy.
The difeience between GDP and GNP is a subtle one. The GDP of a countiy equals the value of
fnal output pioduced within the boideis of that countiy; the GNP of a countiy equals the value of fnal
output pioduced using factois owned by iesidents of the countiy. Most pioduction in a countiy em-
ploys factois of pioduction owned by iesidents of that countiy, so the two measuies oveilap. Difei-
ences between the two measuies emeige when pioduction in one countiy employs factois of pioduc-
tion owned by iesidents of othei countiies.
Suppose, foi example, that a iesident of Bellingham, Washington, owns and opeiates a watch ie-
paii shop acioss the Canadian-U.S. boidei in Victoiia, Biitish Columbia. The value of watch iepaii
seivices pioduced at the shop would be counted as pait of Canada's GDP because they aie pioduced in
Canada. That value would not, howevei, be pait of U.S. GDP. But, because the watch iepaii seivices
weie pioduced using capital and laboi piovided by a iesident of the United States, they would be coun-
ted as pait of GNP in the United States and not as pait of GNP in Canada.
Because most pioduction fts in both a countiy's GDP as well as its GNP, theie is seldom much
difeience between the two measuies. The ielationship between GDP and GNP is given by
QUA7I ON 21. 3
GDP+net income ieceived fiom abioad by iesidents of a nation = GNP
CHAP7R 21 MASURING 7O7AL OU7PU7 AND INCOM 527
In the thiid quaitei of 2008, foi example, GDP equaled $14,220.3 billion. We add income ieceipts
eained by iesidents of the United States fiom the iest of the woild of $803.8 billion and then subtiact
income payments that went fiom the United States to the iest of the woild of $688.4 billion to get GNP
of $14,338.0 billion foi the thiid quaitei of 2008. GNP is often used in inteinational compaiisons of in-
come; we shall examine those latei in this chaptei.
k Y 7 A k A W A Y S
< C|| |s t|e sun c| |r.| gccJs .rJ se.v|ces p.cJuceJ |c. ccrsunpt|cr (), p.|v.te |rvestnert (|),
gcve.rnert pu.c|.ses (O), .rJ ret expc.ts (\
r
). |us C|| + | + O + \
r
.
< C|| c.r be v|e.eJ |r t|e ccrtext c| t|e c|.cu|.. |c. ncJe|. crsunpt|cr gccJs .rJ se.v|ces ..e
p.cJuceJ |r .espcrse tc Jen.rJs |.cn |cuse|c|Js, |rvestnert gccJs ..e p.cJuceJ |r .espcrse tc
Jen.rJs |c. re. c.p|t.| by |.ns, gcve.rnert pu.c|.ses |rc|uJe gccJs .rJ se.v|ces pu.c|.seJ by
gcve.rnert .gerc|es, .rJ ret expc.ts e,u.| expc.ts |ess |npc.ts.
< ct.| cutput c.r be ne.su.eJ t.c ..ys. .s t|e sun c| t|e v.|ues c| |r.| gccJs .rJ se.v|ces p.cJuceJ
.rJ .s t|e sun c| v.|ues .JJeJ .t e.c| st.ge c| p.cJuct|cr.
< C|| p|us ret |rccne .ece|veJ |.cn ct|e. ccurt.|es e,u.|s C||. C|| |s t|e ne.su.e c| cutput typ|c.||y
useJ tc ccnp..e |rccnes gere..teJ by J||e.ert eccrcn|es.
7 R Y I 7 !
|e.e |s . t.cp..t exe.c|se.
.. Suppcse ycu ..e g|ver t|e |c||c.|rg J.t. |c. .r eccrcny.
|e.scr.| ccrsunpt|cr 1,000
|cne ccrst.uct|cr 100
|rc.e.se |r |rvertc.|es 40
|,u|pnert pu.c|.ses by |.ns 60
Ccve.rnert pu.c|.ses 100
Scc|.| Secu.|ty p.ynerts tc |cuse|c|Js 40
Ccve.rnert .e||..e p.ynerts 100
|xpc.ts 50
|npc.ts 150
|Jert||y t|e runbe. c| t|e |c. |r ||gu.e 21.5 tc .||c| e.c| c| t|ese |tens cc..espcrJs. \|.t |s t|e
eccrcnys C||`
b. Suppcse . J.|.y |..n p.cJuces ... n||k, .||c| |t se||s |c. 1,000 tc . J.|.y. |e J.|.y p.cJuces c.e.n,
.||c| |t se||s |c. 3,000 tc .r |ce c.e.n n.ru|.ctu.e.. |e |ce c.e.n n.ru|.ctu.e. uses t|e c.e.n tc
n.ke |ce c.e.n, .||c| |t se||s |c. ,000 tc . g.cce.y stc.e. |e g.cce.y stc.e se||s t|e |ce c.e.n tc
ccrsune.s |c. 10,000. cnpute t|e v.|ue .JJeJ .t e.c| st.ge c| p.cJuct|cr, .rJ ccnp..e t||s |gu.e tc
t|e |r.| v.|ue c| t|e p.cJuct p.cJuceJ. |epc.t ycu. .esu|ts |r . t.b|e s|n||.. tc t|.t g|ver |r .b|e 21.1.
528 PRINCIPLS OI CONOMICS
Case in Point: 7he Spread of the VaIue Added 7ax
Outs|Je t|e |r|teJ St.tes, t|e v.|ue .JJeJ t.x (\A) |.s beccne ccnncrp|.ce. Ccve.rnerts c| nc.e t|.r
120 ccurt.|es use |t .s t|e|. p.|n..y ne.rs c| ..|s|rg .everue. \|||e t|e ccrcept c| t|e \A c.|g|r.teJ |r
|..rce |r t|e 1920s, rc ccurt.y .JcpteJ |t urt|| .|te. \c.|J \.. ||. |r 1948, |..rce bec.ne t|e |.st ccurt.y |r
t|e .c.|J tc use t|e \A. |r 196, b...|| bec.ne t|e |.st ccurt.y |r t|e \este.r |en|sp|e.e tc Jc sc. |e \A
sp.e.J tc ct|e. .este.r |u.cpe.r .rJ |.t|r Ane.|c.r ccurt.|es |r t|e 190s .rJ 1980s .rJ t|er tc ccurt.|es
|r t|e As|./|.c||c .eg|cr, cert..| |u.cpe.r .rJ |c.ne. Scv|et |r|cr ..e., .rJ A|.|c. |r t|e 1990s .rJ e..|y
2000s.
\|.t |s t|e \A` |t |s e,u|v.|ert tc . s.|es t.x cr |r.| gccJs .rJ se.v|ces but |s cc||ecteJ .t e.c| st.ge c|
p.cJuct|cr.
.ke t|e ex.np|e g|ver |r .b|e 21.1, .||c| |s . s|np|||eJ |||ust..t|cr c| . |cuse bu||t |r t|.ee st.ges. || t|e.e
.e.e . s.|es t.x c| 10 cr t|e |cuse, t|e |cuse|c|J buy|rg |t .cu|J p.y 13,500, c| .||c| t|e ccrst.uct|cr
|.n .cu|J keep 125,000 c| t|e tct.| .rJ tu.r 12,500 cve. tc t|e gcve.rnert.
\|t| . 10 \A, t|e s..n||| .cu|J p.y t|e |cgge. 13,200, c| .||c| t|e |cgge. .cu|J keep 12,000 .rJ tu.r
1,200 cve. tc t|e gcve.rnert. |e s..n||| .cu|J se|| t|e |unbe. tc t|e ccrst.uct|cr |.n |c.
2,500keep|rg 26,200, .||c| |s t|e 25,000 |c. t|e |unbe. |tse|| .rJ 1,200 |t .|.e.Jy p.|J |r t.x. |e gcv
e.rnert .t t||s st.ge .cu|J get 1,300, t|e J||e.erce bet.eer t|e 2,500 t|e ccrst.uct|cr |.n cc||ecteJ .s
t.x .rJ t|e 1,200 t|e s..n||| .|.e.Jy p.|J |r t.x tc t|e |cgge. .t t|e p.ev|cus st.ge. |e |cuse|c|J .cu|J
p.y t|e ccrst.uct|cr |.n 13,500. O| t|.t tct.|, t|e ccrst.uct|cr |.n .cu|J tu.r cve. tc t|e gcve.rnert
10,000, .||c| |s t|e J||e.erce bet.eer t|e 12,500 |t cc||ecteJ |c. t|e gcve.rnert |r t.x |.cn t|e |cuse
|c|J .rJ t|e 2,500 |r t.x t|.t |t .|.e.Jy p.|J .|er |t bcug|t t|e |unbe. |.cn t|e s..n|||. |e t.b|e be|c.
s|c.s t|.t |r t|e erJ, t|e t.x .everue gere..teJ by . 10 \A |s t|e s.ne .s t|.t gere..teJ by . 10 t.x cr
|r.| s.|es.
\|y bct|e. tc t.x |r st.ges |rste.J c| ust cr |r.| s.|es` Ore .e.scr |s s|np|y .ecc.J keep|rg, s|rce |t n.y be
J||cu|t tc Jete.n|re |r p..ct|ce || .ry p..t|cu|.. s.|e |s t|e |r.| cre. |r t|e ex.np|e, t|e ccrst.uct|cr |.n
Jces rct reeJ tc krc. || |t |s se|||rg t|e |cuse tc . |cuse|c|J c. tc scne |rte.neJ|..y bus|ress.
A|sc, t|e \A n.y |e.J tc ||g|e. .everue cc||ecteJ. |c. ex.np|e, ever || scne|c. t|e |cuse|c|J buy|rg t|e
|cuse .vc|JeJ p.y|rg t|e t.x, t|e gcve.rnert .cu|J st||| |.ve cc||ecteJ scne t.x .everue .t e..||e. st.ges c|
p.cJuct|cr. \|t| . t.x cr .et.|| s.|es, |t .cu|J |.ve cc||ecteJ rct||rg. |e \A |.s .rct|e. .Jv.rt.ge |.cn
t|e pc|rt c| v|e. c| gcve.rnert .gerc|es. |t |.s t|e .ppe...rce .t e.c| st.ge c| t.k|rg . sn.||e. s|..e. |e
|rJ|v|Ju.| .ncurts cc||ecteJ ..e rct .s cbv|cus tc t.xp.ye.s .s . s.|es t.x n|g|t be.
Good Price VaIue Added 7ax CoIIected - 7ax AIready Paid = VaIue Added 7ax
|cgs 12,000 12,000 1,200 0 1,200
|unbe. 25,000 13,000 2,500 1,200 1,300
|cuse 125,000 100,000 12,500 2,500 10,000
7oto/ $!,200 -$3,700 = $!2,500
CHAP7R 21 MASURING 7O7AL OU7PU7 AND INCOM 529
gross domestic income
(GDI)
|e tct.| |rccne gere..teJ
|r .r eccrcny by t|e
p.cJuct|cr c| |r.| gccJs .rJ
se.v|ces Ju.|rg . p..t|cu|..
pe.|cJ.
A N S W R 7 O 7 R Y I 7 ! P R O 8 L M
.. C|| e,u.|s 1,200 .rJ |s ccnputeJ .s |c||c.s (t|e runbe.s |r p..ert|eses cc..espcrJ tc t|e |c.s |r
||gu.e 21.5).
|e.scr.| ccrsunpt|cr (1) 1,000
|.|v.te |rvestnert (2) 200
|cus|rg 100
|,u|pnert .rJ sc|t...e 60
|rvertc.y c|.rge 40
Ccve.rnert pu.c|.ses (3) 100
|et expc.ts (4) 00
C|| 1,200
|ct|ce t|.t re|t|e. .e||..e p.ynerts rc. Scc|.| Secu.|ty p.ynerts tc |cuse|c|Js ..e |rc|uJeJ. |ese ..e
t..rs|e. p.ynerts, .||c| ..e rct p..t c| t|e gcve.rnert pu.c|.ses ccnpcrert c| C||.
b. |e.e |s t|e t.b|e c| v.|ue .JJeJ.
Good Produced by Purchased by Price VaIue Added
|.. n||k |.|.y |..n |.|.y 1,000 1,000
.e.n |.|.y |ce c.e.n n.ke. 3,000 2,000
|ce c.e.n |ce c.e.n n.ru|.ctu.e. C.cce.y stc.e ,000 4,000
|et.|| |ce c.e.n C.cce.y stc.e crsune. 10,000 3,000
llno/ vo/ue $!0,000
5um of vo/ues AJJeJ $!0,000
2. MEASURING TOTAL INCOME
L A R N I N G O 8 1 C 7 I V S
1. Dene gross domestic income and expIain its reIationship to gross domestic product.
2. Discuss the components of gross domestic income.
3. Dene disposabIe personaI income and expIain how to caIcuIate it from GDP.
We saw in the last section that the pioduction of goods and seivices geneiates factoi incomes to house-
holds. The pioduction of a given value of goods and seivices geneiates an equal value of total income.
Gross domestic income (GDI) equals the total income geneiated in an economy by the pioduction
of fnal goods and seivices duiing a paiticulai peiiod. It is a fow vaiiable. Because an economy's total
output equals the total income geneiated in pioducing that output, GDP = GDI. We can estimate GDP
eithei by measuiing total output oi by measuiing total income.
Considei a $4 box of Cheeiios. It is pait of total output and thus is pait of GDP. Who gets the $4:
Pait of the answei to that question can be found by looking at the ceieal box. Cheeiios aie made fiom
oat foui, wheat staich, sugai, salt, and a vaiiety of vitamins and mineials. Theiefoie, pait of the $4 goes
to the faimeis who giew the oats, the wheat, and the beets oi cane fiom which the sugai was extiacted.
Woikeis and machines at Geneial Mills combined the ingiedients, ciafted all those little O's, toasted
them, and put them in a box. The woikeis weie paid pait of the $4 as wages. The owneis of Geneial
Mills and the capital it used ieceived pait of the $4 as pioft. The box containing the Cheeiios was made
fiom a tiee, so a lumbei company somewheie ieceived pait of the $4. The tiuck diivei who biought the
box of ceieal to the gioceiy stoie got pait of the $4, as did the ownei of the tiuck itself and the ownei of
the oil that fueled the tiuck. The cleik who iang up the sale at the gioceiy stoie was paid pait of the $4.
And so on.
How much of the $4 was income geneiated in the pioduction of the Cheeiios: The answei is
simple: all of it. Some of the money went to woikeis as wages. Some went to owneis of the capital and
530 PRINCIPLS OI CONOMICS
depreciation
A ne.su.e c| t|e .ncurt c|
c.p|t.| t|.t .e..s cut c.
beccnes cbsc|ete Ju.|rg .
pe.|cJ.
natuial iesouices used to pioduce it. Piofts geneiated along the way went to the owneis of the fims
involved. All these items iepiesent costs of pioducing the Cheeiios and also iepiesent income to
households.
Pait of the $4 cost of the Cheeiios, while it makes up a poition of GDI, does not iepiesent oidinaiy
income actually eained by households. That pait iesults fiom two othei pioduction costs: depieciation
and taxes ielated to the pioduction of the Cheeiios. Neveitheless, they aie tieated as a kind of income;
we will examine theii iole in GDI below.
As it is with Cheeiios, so it is with eveiything else. The value of output equals the income genei-
ated as the output is pioduced.
2.1 The Components of GDI
Employee compensation is the laigest among the components of factoi income. Factoi income also in-
cludes pioft, ient, and inteiest. In addition, GDI includes chaiges foi depieciation and taxes associated
with pioduction. Depieciation and pioduction-ielated taxes, such as sales taxes, make up pait of the
cost of pioducing goods and seivices and must be accounted foi in estimating GDI. We will discuss
each of these components of GDI next.
mpIoyee Compensation
Compensation of employees in the foim of wages, salaiies, and benefts makes up the laigest single
component of income geneiated in the pioduction of GDP. In the second quaitei of 2008, employee
compensation iepiesented 37 of GDI.
The stiuctuie of employee compensation has changed diamatically in the last seveial decades. In
1930, viitually all employee compensation-93 of it-came in the foim of wages and salaiies. The ie-
maindei, about 3, came in the foim of additional benefts such as employei contiibutions to ietiie-
ment piogiams and health insuiance. In 2008, the shaie of benefts was ioughly 19 of total employee
compensation.
Profits
The pioft component of income eained by households equals total ievenues of fims less costs as
measuied by conventional accounting. Piofts amounted to 13.6 of GDI, oi $2,226.7 billion in 2008,
down shaiply fiom fve decades eailiei, when piofts iepiesented about 23 of the income geneiated in
GDI.
[1]
Piofts aie the iewaid the owneis of fims ieceive foi being in business. The oppoitunity to eain
piofts is the diiving foice behind pioduction in a maiket economy.
RentaI Income
Rental income, such as the income eained by owneis of iental housing oi payments foi the ient of nat-
uial iesouices, is the smallest component of GDI (less than 0.3); it is the smallest of the income fows
to households. The meaning of ient in the computation of GDI is the same as its meaning in conven-
tional usage; it is a chaige foi the tempoiaiy use of some capital asset oi natuial iesouice.
[2]
Net Interest
Businesses both ieceive and pay inteiest. GDI includes net inteiest, which equals inteiest paid less in-
teiest ieceived by domestic businesses, plus inteiest ieceived fiom foieigneis less inteiest paid to foi-
eigneis. Inteiest payments on moitgage and home impiovement loans aie counted as inteiest paid by
business, because homeowneis aie tieated as businesses in the income accounts. In 2008 net inteiest
accounted foi 6.3 of GDI.
Depreciation
Ovei time the machineiy and buildings that aie used to pioduce goods and seivices weai out oi be-
come obsolete. A faimei's tiactoi, foi example, weais out as it is used. A technological change may
make some equipment obsolete. The intioduction of peisonal computeis, foi example, made the elec-
tiic typewiiteis used by many fims obsolete. Depreciation is a measuie of the amount of capital that
weais out oi becomes obsolete duiing a peiiod. Depieciation is iefeiied to in omcial iepoits as the con-
sumption of fxed capital.
Depieciation is a cost of pioduction, so it iepiesents pait of the piice chaiged foi goods and sei-
vices. It is theiefoie counted as pait of the income geneiated in the pioduction of those goods and sei-
vices. Depieciation iepiesented about 13 of GDI in 2008.
CHAP7R 21 MASURING 7O7AL OU7PU7 AND INCOM 531
indirect taxes
.xes |npcseJ cr t|e
p.cJuct|cr c. s.|e c| gccJs
.rJ se.v|ces c. cr ct|e.
bus|ress .ct|v|ty.
direct taxes
.xes |npcseJ J|.ect|y cr
|rccne.
disposabIe personaI
income
|e |rccne |cuse|c|Js |.ve
.v.||.b|e tc sperJ cr gccJs
.rJ se.v|ces.
Indirect 7axes
The fnal component of the income measuie of GDI is indiiect business taxes.
[3]
Indirect taxes aie
taxes imposed on the pioduction oi sale of goods and seivices oi on othei business activity. (By con-
tiast, a direct tax is a tax imposed diiectly on income; the peisonal income and coipoiate income
taxes aie diiect taxes.) Indiiect taxes, which include sales and excise taxes and piopeity taxes, make up
pait of the cost to fims of pioducing goods and seivices. Like depieciation, they aie pait of the piice of
those goods and seivices and aie theiefoie tieated as pait of the income geneiated in theii pioduction.
Indiiect business taxes amounted to 7.6 of GDI in 2008.
Table 21.2 shows the components of GDI in 2008. Employee compensation iepiesented the laigest
shaie of GDI. The exhibit also shows the components of GDP foi the same yeai.
In piinciple, GDP and GDI should be equal, but theii estimated values nevei aie, because the data
come fiom difeient souices. Output data fiom a sample of fims aie used to estimate GDP, while in-
come data fiom a sample of households aie used to estimate GDI. The difeience is the statistical dis-
ciepancy shown in the iight-hand column of Table 21.2. Some of the dimculties with these data aie ex-
amined in the Case in Point featuie on disciepancies between GDP and GDI.
7A8L 21. 2 GDP and GDI, 2008
|e t.b|e s|c.s t|e ccnpcs|t|cr c| C|| .rJ C|| |r t|e t||.J ,u..te. c| 2008 (|r b||||crs c| Jc||..s .t .r .rru.|
..te). |ct|ce t|e .cug| e,u.||ty c| t|e t.c ne.su.es. (|ey ..e rct ,u|te e,u.| bec.use c| ne.su.enert e..c.s, t|e
J||e.erce |s Jue tc . st.t|st|c.| J|sc.ep.rcy .rJ |s .eJuceJ s|gr||c.rt|y cve. t|ne .s t|e J.t. ..e .ev|seJ.)
C.css Jcnest|c p.cJuct 14,420.5 C.css |cnest|c |rccne 14,260.0
|e.scr.| crsunpt|cr |xperJ|tu.es 10,169.5 cnpers.t|cr c| |np|cyees 8,089.8
C.css |.|v.te |cnest|c |rvestnert 2,013.6 |.c|ts
4
2,226.
Ccve.rnert ccrsunpt|cr experJ|tu.es .rJ g.css
|rvestnert
2,943.9 |ert.| |rccne c| pe.scrs 63.1
|et expc.ts c| gccJs .rJ se.v|ces 06.5 |et |rte.est 903.8
.xes cr p.cJuct|cr .rJ |npc.ts
5
1,06.9
crsunpt|cr c| |xeJ c.p|t.|
(Jep.ec|.t|cr)
1,899.
St.t|st|c.| J|sc.ep.rcy 160.5
Sources Bureau of Economic Analysis ^ational Income and Product Accounts, Tables 1.10 and 1.1.5 (^ovember, 2008). See Brent R. Moulton and
Eugene P. Seskin, Preview of the 2003 Comprehensive Revision of the ^ational Income and Product Accounts, Bureau of Economic Analysis, Survey
of Current Business, june 2003, pp. 17-34.
2.2 Tiacing Income fiom the Economy to Households
We have seen that the pioduction of goods and seivices geneiates income foi households. Thus, the
value of total output equals the value of total income in an economy. But we have also seen that oui
measuie of total income, GDI, includes such things as depieciation and indiiect business taxes that aie
not actually ieceived by households. Households also ieceive some income, such as tiansfei payments,
that does not count as pait of GDP oi GDI. Because the income households actually ieceive plays an
impoitant iole in deteimining theii consumption, it is useful to examine the ielationship between a na-
tion's total output and the income households actually ieceive.
Table 21.3 tiaces the path we take in going fiom GDP to disposable personal income, which
equals the income households have available to spend on goods and seivices. We fist conveit GDP to
GNP and then subtiact elements of GNP that do not iepiesent income ieceived by households and add
payments such as tiansfei payments that households ieceive but do not eain in the pioduction of GNP.
Disposable peisonal income is eithei spent foi peisonal consumption oi saved by households.
532 PRINCIPLS OI CONOMICS
7A8L 21. 3 Irom GDP to DisposabIe PersonaI Income
C||, . ne.su.e c| tct.| cutput, e,u.|s C||, t|e tct.| |rccne gere..teJ |r t|e p.cJuct|cr c| gccJs .rJ se.v|ces |r
.r eccrcny. |e c|..t t..ces t|e p.t| |.cn C|| tc J|spcs.b|e pe.scr.| |rccne, .||c| e,u.|s t|e |rccne
|cuse|c|Js .ctu.||y .ece|ve. \e |.st ccrve.t C|| tc C||. |er, .e subt..ct Jep.ec|.t|cr tc cbt.|r ret r.t|cr.|
p.cJuct .rJ subt..ct t|e st.t|st|c.| J|sc.ep.rcy tc ...|ve .t r.t|cr.| |rccne (|.e., g.css r.t|cr.| |rccne C|| ret c|
Jep.ec|.t|cr .rJ t|e st.t|st|c.| J|sc.ep.rcy). |ext, .e subt..ct ccnpcrerts c| C|| .rJ C|| t|.t Jc rct .ep.esert
|rccne .ctu.||y .ece|veJ by |cuse|c|Js, suc| .s t.xes cr p.cJuct|cr .rJ |npc.ts, cc.pc..te p.c|t .rJ p.y.c||
t.xes (ccrt.|but|crs tc scc|.| |rsu..rce), .rJ cc.pc..te .et.|reJ e..r|rgs. \e .JJ |tens suc| .s t..rs|e. p.ynerts
t|.t ..e |rccne tc |cuse|c|Js but ..e rct p..t c| C|| .rJ C||. |e .Justnerts s|c.r ..e t|e ncst |npc.t.rt
.Justnerts |r gc|rg |.cn C|| .rJ C|| tc J|spcs.b|e pe.scr.| |rccne, seve..| sn.||e. .Justnerts (e.g., subs|J|es,
bus|ress cu..ert t..rs|e. p.ynerts ret, .rJ cu..ert su.p|us c| gcve.rnert erte.p.|ses) |.ve beer cn|tteJ.
C|| + ret |.ctc. e..r|rgs |.cn .b.c.J g.css r.t|cr.| p.cJuct (C||)
C|| Jep.ec|.t|cr (ccrsunpt|cr c| |xeJ c.p|t.|) ret r.t|cr.| p.cJuct (|||)
||| st.t|st|c.| J|sc.ep.rcy r.t|cr.| |rccne (||)
|| |rccne e..reJ but rct .ece|veJ e.g., t.xes cr p.cJuct|cr .rJ
|npc.ts, scc|.| secu.|ty p.y.c|| t.xes, cc.pc..te p.c|t t.xes, .rJ .et.|reJ
e..r|rgs + t..rs|e. p.ynerts .rJ ct|e. |rccne .ece|veJ but rct e..reJ |r
t|e p.cJuct|cr c| C||
pe.scr.| |rccne (||)
|| pe.scr.| |rccne t.xes J|spcs.b|e pe.scr.| |rccne (|||)
k Y 7 A k A W A Y S
< C.css Jcnest|c p.cJuct, C||, e,u.|s g.css Jcnest|c |rccne, C||, .||c| |rc|uJes ccnpers.t|cr, p.c|ts,
.ert.| |rccne, |rJ|.ect t.xes, .rJ Jep.ec|.t|cr.
< \e c.r use C||, . ne.su.e c| tct.| cutput, tc ccnpute J|spcs.b|e pe.scr.| |rccne, . ne.su.e c| |rccne
.ece|veJ by |cuse|c|Js .rJ .v.||.b|e |c. t|en tc sperJ.
7 R Y I 7 !
|e |c||c.|rg |rccne J.t. .e|e. tc t|e s.ne eccrcny |c. .||c| ycu |.J cutput J.t. |r t|e |.st p..t c| t|e
p.ev|cus .y |t! cnpute C|| |.cn t|e J.t. be|c. .rJ ccr|.n t|.t ycu. .esu|t e,u.|s t|e C|| |gu.e ycu
ccnputeJ |r t|e p.ev|cus .y |t! Assune t|.t C|| C|| |c. t||s p.cb|en (t|.t |s, .ssune .|| |.ctc. |rccnes
..e e..reJ .rJ p.|J |r t|e Jcnest|c eccrcny).
|np|cyee ccnpers.t|cr 00
Scc|.| Secu.|ty p.ynerts tc |cuse|c|Js 40
\e||..e p.ynerts 100
|.c|ts 200
|ert.| |rccne 50
|et |rte.est 25
|ep.ec|.t|cr 50
|rJ|.ect t.xes 15
CHAP7R 21 MASURING 7O7AL OU7PU7 AND INCOM 533
Case in Point: 7he GDP-GDI Gap
C|| e,u.|s C||, .t |e.st, t|.t |s t|e ..y |t |s suppcseJ tc .c.k. but |r .r erc.ncus|y ccnp|ex eccrcny, t|e
ne.su.enert c| t|ese t.c v..|.b|es |rev|t.b|y gces ...y. |st|n.tes c| t|e t.c ..e reve. ,u|te e,u.|. |r .ecert
ye..s, t|e .bsc|ute v.|ue c| t|e g.p |.s beer ,u|te s|..b|e. |c. 200 .rJ 2008, |c. ex.np|e, C|| |.s J||e.eJ
|.cn C|| by 81.4 .rJ 160.5 b||||cr pe. ye.., .espect|ve|y.
A|t|cug| t|e g.p seens |..ge, |t .ep.eserts . .en..k.b|y sn.|| |..ct|cr c| ne.su.eJ .ct|v|ty..curJ 1 c.
|ess. O| ccu.se, 1 c| . b|g runbe. |s st||| . b|g runbe.. but |t |s |npc.t.rt tc .enenbe. t|.t, .e|.t|ve tc t|e
s|.e c| t|e eccrcny, t|e g.p bet.eer C|| .rJ C|| |s rct |..ge. |e g.p |s ||steJ .s . st.t|st|c.| J|sc.ep.rcy
|r t|e |ep..tnert c| cnne.ce .epc.t|rg c| t|e t.c runbe.s.
\|y Jces t|e g.p ex|st` |.cn .r .cccurt|rg pc|rt c| v|e., |t s|cu|J rct. |e tct.| v.|ue c| |r.| gccJs .rJ
se.v|ces p.cJuceJ nust be e,u.| tc t|e tct.| v.|ue c| |rccne gere..teJ |r t|.t p.cJuct|cr. but cutput |s
ne.su.eJ |.cn s.|es .rJ |rvertc.y |gu.es cc||ecteJ |.cn ust 10 c| ccnne.c|.| est.b||s|nerts. |.e||n|r..y
|rccne |gu.es ..e cbt.|reJ |.cn |cuse|c|J su.veys, but t|ese .ep.esert . t|ry |..ct|cr c| |cuse|c|Js. Vc.e
ccnp|ete |rccne J.t. ..e p.cv|JeJ by |rccne t.x .etu.rs, but t|ese ..e .v.||.b|e tc t|e eccrcn|sts .|c es
t|n.te C|| cr|y .|te. . t.c tc |cu.ye.. Je|.y.
|e |ep..tnert c| cnne.ce |ssues .ev|s|crs c| |ts C|| .rJ C|| est|n.tes .s nc.e ccnp|ete J.t. beccne
.v.||.b|e. \|t| e.c| .ev|s|cr, t|e g.p bet.eer C|| .rJ C|| est|n.tes |s s|gr||c.rt|y .eJuceJ.
\|||e C|| .rJ C|| |gu.es c.rrct p.cv|Je p.ec|se ne.su.es c| eccrcn|c .ct|v|ty, t|ey ccne .en..k.b|y
c|cse. |rJeeJ, g|ver t|.t t|e runbe.s ccne |.cn ert|.e|y J||e.ert scu.ces, t|e |.ct t|.t t|ey ccne .s c|cse .s
t|ey Jc p.cv|Jes .r |np.ess|ve c|eck c| t|e .ccu..cy c| t|e Jep..tnerts est|n.tes c| C|| .rJ C||.
A N S W R 7 O 7 R Y I 7 ! P R O 8 L M
C|| e,u.|s 1,200. |cte t|.t t||s v.|ue e,u.|s t|e v.|ue |c. C|| cbt.|reJ |.cn t|e est|n.te c| cutput |r t|e
|.st p..t c| t|e p.ev|cus .y |t! |e.e |s t|e ccnput.t|cr.
|np|cyee ccnpers.t|cr 00
|.c|ts 200
|ert.| |rccne 50
|et |rte.est 25
|ep.ec|.t|cr 50
|rJ|.ect t.xes 15
C|| 1,200
Orce .g.|r, rcte t|.t Scc|.| Secu.|ty .rJ .e||..e p.ynerts tc |cuse|c|Js ..e t..rs|e. p.ynerts. |ey Jc rct
.ep.esert p.ynerts tc |cuse|c|J |.ctc.s c| p.cJuct|cr |c. cu..ert cutput c| gccJs .rJ se.v|ces, .rJ t|e.e
|c.e ..e rct |rc|uJeJ |r C||.
534 PRINCIPLS OI CONOMICS
3. GDP AND ECONOMIC WELL-BEING
L A R N I N G O 8 1 C 7 I V S
1. Discuss and give exampIes of measurement and conceptuaI probIems in using reaI GDP as a
measure of economic performance and of economic weII-being.
2. xpIain the use of per capita reaI GNP or GDP to compare economic performance across coun-
tries and discuss its Iimitations.
GDP is the measuie most often used to assess the economic well-being of a countiy. Besides measuiing
the pulse of a countiy, it is the fguie used to compaie living standaids in difeient countiies.
Of couise, to use GDP as an indicatoi of oveiall economic peifoimance, we must conveit nominal
GDP to ieal GDP, since nominal values can iise oi fall simply as a iesult of changes in the piice level.
Foi example, the movie Titanic, ieleased in 1997, biought in $601 million-the highest amount evei in
gioss box omce ieceipts, while Gone with the Vind, ieleased in 1939, eained only $199 million and
ianks 49th in teims of nominal ieceipts. But does that mean that Titanic actually did bettei than Gone
with the Vind: Aftei all, the aveiage piice of a movie ticket in 1939 was 23 cents. At the time of Titanic,
the aveiage ticket piice was about $3. A bettei way to compaie these two movies in teims of populaiity
is to contiol foi the piice of movie tickets-the same stiategy that economists use with ieal GDP in oi-
dei to deteimine whethei output is iising oi falling. Adjusting the nominal box-omce ieceipts using
1998 movie piices to obtain ieal ievenue ieveals that in ieal teims Gone with the Vind continues to be
the top ieal giossei of all time with ieal box-omce ieceipts of about $1.3 billion. As illustiated by this
example on ievenues fiom populai movies, we might diaw eiioneous conclusions about peifoimance
if we base them on nominal values instead of on ieal values. In contiast, ieal GDP, despite the piob-
lems with piice indexes that weie explained in anothei chaptei, piovides a ieasonable measuie of the
total output of an economy, and changes in ieal GDP piovide an indication of the diiection of move-
ment in total output.
We begin this section by noting some of the diawbacks of using ieal GDP as a measuie of the eco-
nomic welfaie of a countiy. Despite these shoitcomings, we will see that it piobably iemains oui best
single indicatoi of macioeconomic peifoimance.
3.1 Measuiement Pioblems in Real GDP
Theie aie two measuiement pioblems, othei than those associated with adjusting foi piice level
changes, in using ieal GDP to assess domestic economic peifoimance.
Revisions
The fist estimate of ieal GDP foi a calendai quaitei is called the advance estimate. It is issued about a
month aftei the quaitei ends. To pioduce a measuie so quickly, omcials at the Depaitment of Com-
meice must iely on infoimation fiom ielatively few fims and households. One month latei, it issues a
ievised estimate, and a month aftei that it issues its fnal estimate. Often the advance estimate of GDP
and the fnal estimate do not coiiespond. The iecession of 2001, foi example, began in Maich of that
yeai. But the fist estimates of ieal GDP foi the second and thiid quaiteis of 2001 showed output con-
tinuing to iise. It was not until latei ievisions that it became cleai that a iecession had been undei way.
But the ievision stoiy does not end theie. Eveiy summei, the Commeice Depaitment issues ie-
vised fguies foi the pievious two oi thiee yeais. Once eveiy fve yeais, the depaitment conducts an ex-
tensive analysis that tiaces fows of inputs and outputs thioughout the economy. It focuses on the out-
puts of some fims that aie inputs to othei fims. In the piocess of conducting this analysis, the depait-
ment ievises ieal GDP estimates foi the pievious fve yeais. Sometimes the ievisions can paint a pictuie
of economic activity that is quite difeient fiom the one given even by the ievised estimates of GDP.
Foi example, ievisions of the data foi the 1990-1991 iecession issued seveial yeais latei showed that
the iecession had been much moie seiious than had pieviously been appaient, and the iecoveiy was
moie pionounced.
7he Service Sector
Anothei pioblem lies in estimating pioduction in the seivice sectoi. The output of goods in an eco-
nomy is ielatively easy to compute. Theie aie so many bushels of coin, so many pounds of beef. But
what is the output of a bank: Of a hospital: It is easy to iecoid the dollai value of output to entei in
nominal GDP, but estimating the quantity of output to use in ieal GDP is quite anothei mattei. In
CHAP7R 21 MASURING 7O7AL OU7PU7 AND INCOM 535
some cases, the Depaitment of Commeice estimates seivice sectoi output based on the quantity of
laboi used. Foi example, if this technique weie used in the banking industiy and banking used 10
moie laboi, the depaitment would iepoit that pioduction has iisen 10. If the numbei of employees
iemains unchanged, iepoited output iemains unchanged. In efect, this appioach assumes that output
pei woikei-pioductivity-in those sectois iemains constant when studies have indicated that pio-
ductivity has incieased gieatly in the seivice sectoi. Since 1990 piogiess has been made in measuie-
ment in this aiea, which allows in paiticulai foi bettei estimation of pioductivity changes and piice in-
dexes foi difeient seivice sectoi industiies, but moie iemains to be done in this laige sectoi of the U.S.
economy.
[6]
3.2 Conceptual Pioblems with Real GDP
A second set of limitations of ieal GDP stems fiom pioblems inheient in the indicatoi itself. Real GDP
measuies maiket activity. Goods and seivices that aie pioduced and exchanged in a maiket aie coun-
ted; goods and seivices that aie pioduced but that aie not exchanged in maikets aie not.
[7]
HousehoId Production
Suppose you aie consideiing whethei to eat at home foi dinnei tonight oi to eat out. You could cook
dinnei foi youiself at a cost of $3 foi the ingiedients plus an houi oi so of youi time. Alteinatively, you
could buy an equivalent meal at a iestauiant foi peihaps $13. Youi decision to eat out iathei than cook
would add $10 to the GDP.
But that $10 addition would be misleading. Aftei all, if you had stayed home you might have pio-
duced an equivalent meal. The only difeience is that the value of youi time would not have been coun-
ted. But suiely youi time is not woithless; it is just not counted. Similaily, GDP does not count the
value of youi efoits to clean youi own house, to wash youi own cai, oi to giow youi own vegetables.
In geneial, GDP omits the entiie value added by membeis of a household who do household woik
themselves.
Theie is ieason to believe this omission is seiious. Economists J. Steven Landefeld and Stephanie
H. McCulla of the U.S. Buieau of Economic Analysis estimated in a 2000 papei the value of household
output fiom 1946 to 1997. Theii estimate of household output in 1946 was 30 of iepoited GDP. Since
then, that peicentage has fallen, because moie women have enteied the woikfoice, so that moie pio-
duction that once took place in households now occuis in the maiket. Households now eat out moie,
puichase moie piepaied foods at the gioceiy stoie, hiie out child-caie seivices they once peifoimed
themselves, and so on. Theii estimate foi 1997, foi example, suggests that household pioduction
amounted to 36 of iepoited GDP.
[8]
This pioblem is especially signifcant when GDP is used to make compaiisons acioss countiies. In
low-income countiies, a much gieatei shaie of goods and seivices is not exchanged in a maiket. Estim-
ates of GDP in such countiies aie adjusted to iefect nonmaiket pioduction, but these adjustments aie
inevitably impiecise.
Underground and IIIegaI Production
Some pioduction goes uniepoited in oidei to evade taxes oi the law. It is not likely to be counted in
GDP. Legal pioduction foi which income is uniepoited in oidei to evade taxes geneially takes place in
what is known as the undeigiound economy." Foi example, a caipentei might build a small addition
to a dentist's house in exchange foi oithodontic woik foi the caipentei's childien. Although income
has been eained and output geneiated in this example of baiteiing, the tiansaction is unlikely to be ie-
poited foi income tax oi othei puiposes and thus is not counted in GDP. Illegal activities aie not ie-
poited foi income taxes foi obvious ieasons and aie thus dimcult to include in GDP.
Leisure
Leisuie is an economic good. All othei things being equal, moie leisuie is bettei than less leisuie.
But all othei things aie not likely to be equal when it comes to consuming leisuie. Consuming
moie leisuie means supplying less woik efoit. And that means pioducing less GDP. If eveiyone de-
cided to woik 10 fewei houis, GDP would fall. But that would not mean that people weie woise of.
In fact, theii choice of moie leisuie would suggest they piefei the extia leisuie to the goods and seivices
they give up by consuming it. Consequently, a ieduction in GDP would be accompanied by an inciease
in satisfaction, not a ieduction.
536 PRINCIPLS OI CONOMICS
per capita reaI GNP or GDP
A ccurt.ys .e.| C|| c. C||
J|v|JeJ by |ts pcpu|.t|cr.
7he GDP Accounts Ignore 8ads"
Suppose a wave of buiglaiies weie to bieak out acioss the United States. One would expect people to
iespond by buying moie and loudei buiglai alaims, bettei locks, feicei Geiman shepheids, and moie
guaid seivices than they had befoie. To the extent that they pay foi these by dipping into savings iathei
than ieplacing othei consumption, GDP incieases. An epidemic might have much the same efect on
GDP by diiving up health-caie spending. But that does not mean that ciime and disease aie good
things; it means only that ciime and disease may foice an inciease in the pioduction of goods and sei-
vices counted in the GDP.
Similaily, the GDP accounts ignoie the impact of pollution on the enviionment. We might pio-
duce an additional $200 billion in goods and seivices but cieate pollution that makes us feel woise of
by, say, $300 billion. The GDP accounts simply iepoit the $200 billion in incieased pioduction. Indeed,
some of the enviionmental degiadation might itself boost GDP. Diitiei aii may foice us to wash
clothes moie often, to paint buildings moie often, and to see the doctoi moie often, all of which will
tend to inciease GDP!
ConcIusion: GDP and Human Happiness
Moie GDP cannot necessaiily be equated with moie human happiness. But moie GDP does mean
moie of the goods and seivices we measuie. It means moie jobs. It means moie income. And most
people seem to place a high value on these things. Foi all its faults, GDP does measuie the pioduction
of most goods and seivices. And goods and seivices get pioduced, foi the most pait, because we want
them. We might thus be safe in giving two cheeis foi GDP-and holding back the thiid in iecognition
of the conceptual dimculties that aie inheient in using a single numbei to summaiize the output of an
entiie economy.
3.3 Inteinational Compaiisons of Real GDP and GNP
Real GDP oi GNP estimates aie often used in compaiing economic peifoimance among countiies. In
making such compaiisons, it is impoitant to keep in mind the geneial limitations to these measuies of
economic peifoimance that we noted eailiei. Fuithei, countiies use difeient methodologies foi col-
lecting and compiling data.
Thiee othei issues aie impoitant in compaiing ieal GDP oi GNP foi difeient countiies: the ad-
justment of these fguies foi population, adjusting to a common cuiiency, and the incoipoiation of
nonmaiket pioduction.
In inteinational compaiisons of ieal GNP oi ieal GDP, economists geneially make compaiisons
not of ieal GNP oi GDP but of per capita real GNP or GDP, which equals a countiy's ieal GNP oi
GDP divided by its population. In 2007, foi example, Japan had a ieal GDP of about $3,000 billion and
Luxembouig had a ieal GDP of about $23 billion, both fguies in U.S. $2,000. We can conclude that
Japan's economy pioduced fai moie goods and seivices than did Luxembouig's. But Japan had almost
300 times as many people as did Luxembouig. Japan's pei capita ieal GDP in 2007 was $40,636; Lux-
embouig's was $34,482, the highest in the woild that yeai.
Figuie 21.9 compaies pei capita ieal GNP foi 11 countiies in 2007. It is based on data that uses a
measuie called inteinational dollais" in oidei to coiiect foi difeiences in the puichasing powei of $1
acioss countiies. The data also attempt to adjust foi nonmaiket pioduction (such as that of iuial famil-
ies that giow theii own food, make theii own clothing, and pioduce othei household goods and sei-
vices themselves).
CHAP7R 21 MASURING 7O7AL OU7PU7 AND INCOM 537
II GUR 21. 9 Comparing Per Capita ReaI GNP, 2007
|e.e |s . |uge g.p bet.eer pe. c.p|t. |rccne |r cre c| t|e pcc.est ccurt.|es |r t|e .c.|J, ||be.|., .rJ .e.|t||e.
r.t|crs suc| .s t|e |r|teJ St.tes .rJ |uxenbcu.g.
Source Vorld Bank, Vorld Development Indicators Online
The dispaiities in income aie stiiking; Luxembouig, the countiy with the highest pei capita ieal GNP,
had an income level moie than 200 times gieatei than Libeiia, the countiy with the lowest pei capita
ieal GNP.
What can we conclude about inteinational compaiisons in levels of GDP and GNP: Ceitainly we
must be cautious. Theie aie enoimous dimculties in estimating any countiy's total output. Compaiing
one countiy's output to anothei piesents additional challenges. But the fact that a task is dimcult does
not mean it is impossible. When the data suggest huge dispaiities in levels of GNP pei capita, foi ex-
ample, we obseive ieal difeiences in living standaids.
k Y 7 A k A W A Y S
< |e.| C|| c. .e.| C|| |s c|ter useJ .s .r |rJ|c.tc. c| t|e eccrcn|c .e||be|rg c| . ccurt.y.
< |.cb|ens |r t|e ne.su.enert c| .e.| C||, |r .JJ|t|cr tc p.cb|ens erccurte.eJ |r ccrve.t|rg |.cn
rcn|r.| tc .e.| C||, sten |.cn .ev|s|crs |r t|e J.t. .rJ t|e J||cu|ty c| ne.su.|rg cutput |r scne
sectc.s, p..t|cu|..|y t|e se.v|ce sectc..
< crceptu.| p.cb|ens |r t|e use c| .e.| C|| .s . ne.su.e c| eccrcn|c .e||be|rg |rc|uJe t|e |.cts t|.t |t
Jces rct |rc|uJe rcrn..ket p.cJuct|cr .rJ t|.t |t Jces rct p.cpe.|y .Just |c. b.Js p.cJuceJ |r t|e
eccrcny.
< |e. c.p|t. .e.| C|| c. C|| c.r be useJ tc ccnp..e eccrcn|c pe.|c.n.rce |r J||e.ert ccurt.|es.
7 R Y I 7 !
\|.t |np.ct .cu|J e.c| c| t|e |c||c.|rg |.ve cr .e.| C||` \cu|J eccrcn|c .e||be|rg |rc.e.se c. Je
c.e.se .s . .esu|t`
.. Or .ve..ge, pecp|e |r . ccurt.y Jec|Je tc |rc.e.se t|e runbe. c| |cu.s t|ey .c.k by 5.
b. SperJ|rg cr |cne|.rJ secu.|ty |rc.e.ses |r .espcrse tc . te..c.|st .tt.ck.
c. |e p.|ce |eve| .rJ rcn|r.| C|| |rc.e.se by 10.
538 PRINCIPLS OI CONOMICS
Case in Point: Per Capita ReaI GDP and OIympic MedaI Counts
|r t|e pcpu|.. |c.e, t|e O|ynp|cs p.cv|Je .r cppc.tur|ty |c. t|e |rest .t||etes |r t|e .c.|J tc ccnpete .|t|
e.c| ct|e. |e.Jtc|e.J cr t|e b.s|s c| ... t.|ert .rJ |..J .c.k. ArJ yet, ccrterJe.s |.cn |.cs terJ tc |r
|s| |.st c. c|cse tc |t |r .|ncst .ry evert |r .||c| t|ey ccnpete. Ore |.ct|.r .t||ete g..re.eJ t|e urerv|.b|e
.ecc.J c| |.v|rg beer t|e s|c.est ert..rt |r t|e re..|y |.||certu.y |crg ||stc.y c| t|e 20k||cnete. ..|k. |r
ccrt..st, |.S. .t||etes .cr 103 neJ.|s .t t|e 2004 At|ers O|ynp|cs .rJ 110 neJ.|s .t t|e 2008 be|rg
O|ynp|cs. \|y Jc |.ct|.rs |..e sc pcc.|y .rJ Ane.|c.rs sc .e||, .|t| .t||etes |.cn ct|e. ccurt.|es |.|||rg |r
bet.eer`
|ccrcn|sts |.r|e| |. |. 'c|rscr .rJ Ay|e. A|| |.ve beer .b|e tc p.eJ|ct .|t| .stcr|s||rg .ccu..cy t|e run
be. c| neJ.|s J||e.ert ccurt.|es .||| .|r cr t|e b.s|s c| . |.rJ|u| c| |.ctc.s, |rc|uJ|rg pcpu|.t|cr, c||n.te,
pc||t|c.| st.uctu.e, .rJ .e.| pe. c.p|t. C||. |c. ex.np|e, t|ey p.eJ|cteJ t|.t t|e |r|teJ St.tes .cu|J .|r 103
neJ.|s |r At|ers .rJ t|.t |s p.ec|se|y |c. n.ry t|e |r|teJ St.tes .cr. |ey p.eJ|cteJ 103 neJ.|s |c. t|e
|r|teJ St.tes |r be|rg, 110 .e.e .cr. |ey J|J rct expect t|e |.ct|.rs tc .|r .ry neJ.|s |r e|t|e. At|ers c.
be|rg, .rJ t|.t ..s |rJeeJ t|e cutccne.
'c|rscr .rJ A|| est|n.teJ t|.t sunne. g.ne p..t|c|p.rt r.t|crs .ve..ge cre nc.e neJ.| pe. .JJ|t|cr.|
1,000 c| pe. c.p|t. .e.| C||. \|t| pe. c.p|t. .e.| C|| |r |.cs |ess t|.r t|e e,u|v.|ert c| 500 ccnp..eJ tc
pe. c.p|t. .e.| C|| |r t|e |r|teJ St.tes c| .bcut 38,000, t|e .esu|ts |c. t|ese t.c r.t|crs ccu|J be ccr
s|Je.eJ . |c.egcre ccrc|us|cr. Accc.J|rg tc 'c|rscr .rJ A||, ||g| p.cJuct|ve c.p.c|ty c. |rccne pe. pe.scr
J|sp|.ys .r .b|||ty tc p.y t|e ccsts recess..y tc serJ .t||etes tc t|e C.nes, .rJ n.y .|sc be .sscc|.teJ .|t| .
||g|e. ,u.||ty c| t..|r|rg .rJ bette. e,u|pnert. |c. ex.np|e, . |.ct|.r s.|nne. .t At|ers, \||.yp|cre
\crgp|.c|.r|, |.J reve. p..ct|ceJ |r .r O|ynp|cs|.e pcc|, .rJ . .urre., S|.|v.r| |et.vcrg, |.J .c.r t|e
s.ne .urr|rg s|ces |c. |cu. ye..s.
|e gccJ re.s |s t|.t .s t|e pe. c.p|t. .e.| C|| |r scne .e|.t|ve|y pcc. ccurt.|es |.s .|ser, t|e |np.cveJ ||v
|rg st.rJ..Js |.ve |eJ tc |rc.e.seJ O|ynp|c neJ.| ccurts. ||r., |c. |rst.rce, .cr 28 neJ.|s |r 1988 .rJ 63
|r 2004. As t|e |cst |c. t|e 2008 g.nes, |t .cr .r |np.ess|ve tct.| c| 100 neJ.|s.
\|||e rct . pe.|ect ne.su.e c| t|e .e||be|rg c| pecp|e |r . ccurt.y, pe. c.p|t. .e.| C|| Jces te|| us .bcut
t|e cppc.tur|t|es .v.||.b|e tc t|e .ve..ge c|t|.er |r . ccurt.y. Ane.|c.rs .cu|J su.e|y |rJ |t |..J tc |n.g|re
||v|rg .t t|e |eve| c| ccrsunpt|cr c| t|e .ve..ge |.ct|.r. |r |e |oe |ooJo |o. |||e Oe| |e||e /|||e
|eo|e |ee| /oe, ess.y|st C.egg |.ste.b.cck rctes t|.t . ||g|e. n.te.|.| st.rJ..J c| ||v|rg |s rct .sscc|.teJ
.|t| ||g|e. .epc.teJ |.pp|ress. but, |e ccrc|uJes, t|e p.cb|ens c| p.cspe.|ty seen |ess se.|cus t|.r t|cse c|
pcve.ty, .rJ p.cspe.|ty g|ves pecp|e .rJ r.t|crs t|e ne.rs tc .JJ.ess p.cb|ens. |e O|ynp|c neJ.| ccurt
|c. e.c| r.t|cr st.crg|y .e|ects |ts .ve..ge st.rJ..J c| ||v|rg .rJ |erce t|e cppc.tur|t|es .v.||.b|e tc |ts
c|t|.ers.
5ources Oe |o|eoo|. |e |oe |ooJo |o. |||e Oe| |e||e /|||e |eo|e |ee| /oe (|e. `o| |oJo |ooe. 2003:. |o|e| | |
.o|o oJ ^,|e ^||. ^ o|e o| .o .eoo |o||c|o||o oJ /eJo| oo| o| ||e .oe oJ /||e |,|c Ooe.' .oc|o| .c|ece (oo|e|,
S4. o 4 (|ecee 2004: 9.493. |o.|J /o||ec|||,. /|, ||| |ee |o |oo.' |ooJe /oo|e. ^oo| S. 2004. S
A N S W R 7 O 7 R Y I 7 ! P R O 8 L M
.. |e.| C|| .cu|J |rc.e.se. Assun|rg t|e pecp|e c|cse tc |rc.e.se t|e|. .c.k e|c.t .rJ |c.gc t|e ext..
|e|su.e, eccrcn|c .e||be|rg .cu|J |rc.e.se .s .e||.
b. |e.| C|| .cu|J |rc.e.se, but t|e ext.. experJ|tu.e |r t|e eccrcny ..s Jue tc .r |rc.e.se |r scnet||rg
b.J, sc eccrcn|c .e||be|rg .cu|J ||ke|y be |c.e..
c. |c c|.rge |r .e.| C||. |c. scne pecp|e, eccrcn|c .e||be|rg n|g|t |rc.e.se .rJ |c. ct|e.s |t n|g|t
Jec.e.se, s|rce |r|.t|cr Jces rct .|ect e.c| pe.scr |r t|e s.ne ..y.
CHAP7R 21 MASURING 7O7AL OU7PU7 AND INCOM 539
4. REVIEW AND PRACTICE
Summary
||s c|.pte. |ccuseJ cr t|e ne.su.enert c| C||. |e tct.| v.|ue c| cutput (C||) e,u.|s t|e tct.| v.|ue c| |r
ccne gere..teJ |r p.cJuc|rg t|.t cutput (C||). \e c.r |||ust..te t|e |c.s c| sperJ|rg .rJ |rccne t|.cug|
t|e eccrcny .|t| t|e c|.cu|.. |c. ncJe|. ||.ns p.cJuce gccJs .rJ se.v|ces |r .espcrse tc Jen.rJs |.cn
|cuse|c|Js (pe.scr.| ccrsunpt|cr), ct|e. |.ns (p.|v.te |rvestnert), gcve.rnert .gerc|es (gcve.rnert pu.
c|.ses), .rJ t|e .est c| t|e .c.|J (ret expc.ts). ||s p.cJuct|cr, |r tu.r, c.e.tes . |c. c| |.ctc. |rccnes tc
|cuse|c|Js. |us, C|| c.r be est|n.teJ us|rg t.c types c| J.t.. (1) J.t. t|.t s|c. t|e tct.| v.|ue c| cutput
.rJ (2) J.t. t|.t s|c. t|e tct.| v.|ue c| |rccne gere..teJ by t|.t cutput.
|r |cck|rg .t C|| .s . ne.su.e c| cutput, .e J|v|Je |t |rtc |cu. ccnpcrerts. ccrsunpt|cr, |rvestnert, gcv
e.rnert pu.c|.ses, .rJ ret expc.ts. C|| e,u.|s t|e sun c| |r.| v.|ues p.cJuceJ |r e.c| c| t|ese ..e.s. |t
c.r .|sc be ne.su.eJ .s t|e sun c| v.|ues .JJeJ .t e.c| st.ge c| p.cJuct|cr. |e ccnpcrerts c| C|| ne.s
u.eJ |r te.ns c| |rccne (C||) ..e enp|cyee ccnpers.t|cr, p.c|ts, .ert.| |rccne, ret |rte.est, Jep.ec|.t|cr,
.rJ |rJ|.ect t.xes.
\e .|sc exp|.|reJ ct|e. ne.su.es c| |rccne suc| .s C|| .rJ J|spcs.b|e pe.scr.| |rccne. ||spcs.b|e pe.
scr.| |rccne |s .r |npc.t.rt eccrcn|c |rJ|c.tc., bec.use |t |s c|cse|y .e|.teJ tc pe.scr.| ccrsunpt|cr, t|e
|..gest ccnpcrert c| C||.
C|| |s c|ter useJ .s .r |rJ|c.tc. c| |c. .e|| c| . ccurt.y |s. O| ccu.se, tc use |t |c. t||s pu.pcse, .e nust be
c..e|u| tc use .e.| C|| ..t|e. t|.r rcn|r.| C||. St|||, t|e.e ..e p.cb|ens .|t| cu. est|n.te c| .e.| C||. |.cb
|ens erccurte.eJ |r ccrve.t|rg rcn|r.| C|| tc .e.| C|| .e.e J|scusseJ |r t|e p.ev|cus c|.pte.. |r t||s
c|.pte. .e |cckeJ .t .JJ|t|cr.| ne.su.enert p.cb|ens .rJ ccrceptu.| p.cb|ens.
|.e,uert .ev|s|crs |r t|e J.t. scnet|nes c|.rge cu. p|ctu.e c| t|e eccrcny ccrs|Je..b|y. Acccurt|rg |c.
t|e se.v|ce sectc. |s ,u|te J||cu|t. crceptu.| p.cb|ens |rc|uJe t|e cn|ss|cr c| rcrn..ket p.cJuct|cr .rJ c|
urJe.g.curJ .rJ |||eg.| p.cJuct|cr. C|| |grc.es t|e v.|ue c| |e|su.e .rJ |rc|uJes ce.t.|r b.Js.
\e c.rrct .sse.t .|t| ccr|Jerce t|.t nc.e C|| |s . gccJ t||rg .rJ t|.t |ess |s b.J. |c.eve., .e.| C|| .e
n.|rs cu. best s|rg|e |rJ|c.tc. c| eccrcn|c pe.|c.n.rce. |t |s useJ rct cr|y tc |rJ|c.te |c. .ry cre ecc
rcny |s pe.|c.n|rg cve. t|ne but .|sc tc ccnp..e t|e eccrcn|c pe.|c.n.rce c| J||e.ert ccurt.|es.
540 PRINCIPLS OI CONOMICS
C O N C P 7 P R O 8 L M S
1. C|| |s useJ .s . ne.su.e c| n.c.ceccrcn|c pe.|c.n.rce. \|.t, p.ec|se|y, Jces |t ne.su.e`
2. V.ry eccrcn|sts |.ve .ttenpteJ tc c.e.te . set c| scc|.| .cccurts t|.t .cu|J ccne c|cse. tc ne.su.|rg
t|e eccrcn|c .e||be|rg c| t|e scc|ety t|.r Jces C||. \|.t ncJ||c.t|crs c| t|e cu..ert .pp.c.c|
.cu|J ycu .eccnnerJ tc t|en`
3. |ve.y gccJ p.cJuceJ c.e.tes |rccne |c. t|e c.re.s c| t|e |.ctc.s c| p.cJuct|cr t|.t c.e.teJ t|e p.cJuct
c. se.v|ce. |c. . .ecert pu.c|.se ycu n.Je, t.y tc ||st .|| t|e types c| |.ctc.s c| p.cJuct|cr |rvc|veJ |r
n.k|rg t|e p.cJuct .v.||.b|e, .rJ t.y tc Jete.n|re .|c .ece|veJ |rccne .s . .esu|t c| ycu. pu.c|.se.
4. |xp|.|r |c. t|e s.|e c| useJ textbccks |r ycu. c.npus bcckstc.e .|ects t|e C|| c.|cu|.t|cr.
5. |cck .g.|r .t t|e c|.cu|.. |c. J|.g..n |r ||gu.e 21.5 .rJ .ssune |t |s J...r |c. t|e |r|teJ St.tes. St.te
t|e |c.s |r .||c| e.c| c| t|e |c||c.|rg t..rs.ct|crs .cu|J be erte.eJ.
.. A ccrsune. pu.c|.ses |.es| |s| .t . |cc.| |s| n..ket.
b. A g.cce.y stc.e .c,u|.es 1,000 .c||s c| p.pe. tc.e|s |c. |.te. .es.|e.
c. |ASA pu.c|.ses . re. S.tu.r .ccket.
J. |ecp|e |r |..rce |cck tc see t|e |.test b..J ||tt ncv|e.
e. A ccrst.uct|cr |.n bu||Js . re. |cuse.
|. A ccup|e |.cn Se.tt|e v|s|ts Cu.J.|.... .rJ st.ys |r . |cte| t|e.e.
g. |e c|ty c| |.||.s pu.c|.ses ccnpute. p.pe. |.cn . |cc.| |.n.
6. Suggest .r ..gunert |c. .rJ .r ..gunert .g.|rst ccurt|rg |r C|| .|| |cuse|c|Jp.cJuceJ gccJs .rJ
se.v|ces t|.t ..e rct sc|J, suc| .s t|e v.|ue c| c|||J c..e c. |cneccckeJ ne.|s.
. Suppcse . r.t|crs |.ns n.ke |e.vy use c| |.ctc.s c| p.cJuct|cr c.reJ by .es|Jerts c| |c.e|gr ccurt.|es,
.|||e |c.e|gr |.ns n.ke .e|.t|ve|y ||tt|e use c| |.ctc.s c.reJ by .es|Jerts c| t|.t r.t|cr. |c. Jces t|e
r.t|crs C|| ccnp..e tc |ts C||`
8. Suppcse curt.y A |.s t|e s.ne C|| .s curt.y b, .rJ t|.t re|t|e. r.t|crs .es|Jerts c.r |.ctc.s c|
p.cJuct|cr useJ by |c.e|gr |.ns, rc. Jc e|t|e. r.t|crs |.ns use |.ctc.s c| p.cJuct|cr c.reJ by |c.e|gr
.es|Jerts. Suppcse t|.t, .e|.t|ve tc curt.y b, Jep.ec|.t|cr, |rJ|.ect bus|ress t.xes, .rJ pe.scr.| |rccne
t.xes |r curt.y A ..e ||g|, .|||e .e||..e .rJ Scc|.| Secu.|ty p.ynerts tc |cuse|c|Js |r curt.y A ..e
.e|.t|ve|y |c.. \||c| ccurt.y |.s t|e ||g|e. J|spcs.b|e pe.scr.| |rccne` \|y`
9. Suppcse t|.t v|.tu.||y eve.ycre |r t|e |r|teJ St.tes Jec|Jes tc t.ke |||e . ||tt|e e.s|e., .rJ t|e |ergt| c| t|e
.ve..ge .c.k.eek |.||s by 25. |c. .||| t|.t .|ect C||` |e. c.p|t. C||` |c. .||| |t .|ect eccrcn|c
.e||..e`
10. cnnert cr t|e |c||c.|rg st.tenert. |t Jces rct n.tte. t|.t t|e v.|ue c| t|e |.bc. pecp|e Jevcte tc
p.cJuc|rg t||rgs |c. t|ense|ves |s rct ccurteJ |r C||, bec.use .e n.ke t|e s.ne n|st.ke eve.y ye..,
.e|.t|ve v.|ues ..e ur.|ecteJ.
11. |.ne scne c| t|e se.v|ces, || .ry, ycu p.cJuceJ .t |cne t|.t Jc get ccurteJ |r C||. A.e t|e.e .ry
gccJs ycu p.cJuce t|.t ..e rct ccurteJ`
12. V..u.r. |s scnet|nes est|n.teJ tc be .|||c.r|.s |..gest c.s| c.cp. |t |s rct |rc|uJeJ |r est|n.tes c|
C||. S|cu|J |t be`
CHAP7R 21 MASURING 7O7AL OU7PU7 AND INCOM 541
N U M R I C A L P R O 8 L M S
1. C|ver t|e |c||c.|rg rcn|r.| J.t., ccnpute C||. Assune ret |.ctc. |rccnes |.cn .b.c.J 0 (t|.t |s,
C|| C||).
NominaI Data for GDP and NNP $ 8iIIions
crsunpt|cr 2,99.8
|ep.ec|.t|cr 481.6
|xpc.ts 36.2
C.css p.|v.te Jcnest|c |rvestnert 61.0
|rJ|.ect t.xes 331.4
Ccve.rnert pu.c|.ses 869.
Ccve.rnert t..rs|e. p.ynerts 94.8
|npc.ts 481.
2. ||rJ J.t. |c. e.c| c| t|e |c||c.|rg ccurt.|es cr .e.| C|| .rJ pcpu|.t|cr. |se t|e J.t. tc c.|cu|.te t|e
C|| pe. c.p|t. |c. e.c| c| t|e |c||c.|rg ccurt.|es.
.. Vc..nb|,ue
b. |rJ|.
c. |.k|st.r
J. |r|teJ St.tes
e. .r.J.
|. |uss|.
g. b...||
|. |..r
|. c|cnb|.
3. |c. ccrst.uct . b.. g..p| s|c.|rg ycu. .esu|ts |r t|e p.ev|cus p.cb|en, c.g.r|.|rg t|e ccurt.|es |.cn
t|e ||g|est tc t|e |c.est C|| pe. c.p|t., .|t| ccurt.|es cr t|e |c.|.crt.| .x|s .rJ C|| pe. c.p|t. cr t|e
ve.t|c.| .x|s.
4. Suppcse curt.y A |.s . C|| c| 4 t.||||cr. |es|Jerts c| t||s ccurt.y e..r 500 n||||cr |.cn .ssets t|ey
c.r |r |c.e|gr ccurt.|es. |es|Jerts c| |c.e|gr ccurt.|es e..r 300 n||||cr |.cn .ssets t|ey c.r |r curt.y
A. cnpute.
.. curt.y As ret |c.e|gr |rccne.
b. curt.y As C||.
5. Suppcse . ccurt.ys C|| e,u.|s 500 b||||cr |c. . p..t|cu|.. ye... |ccrcn|sts |r t|e ccurt.y est|n.te t|.t
|cuse|c|J p.cJuct|cr e,u.|s 40 c| C||.
.. \|.t |s t|e v.|ue c| t|e ccurt.ys |cuse|c|J p.cJuct|cr |c. t|.t ye..`
b. curt|rg bct| C|| .rJ |cuse|c|J p.cJuct|cr, .|.t |s t|e ccurt.ys tct.| cutput |c. t|e ye..`
6. A n|re. ext..cts |.cr |.cn t|e e..t|. A stee| n||| ccrve.ts t|e |.cr tc stee| be.ns |c. use |r ccrst.uct|cr. A
ccrst.uct|cr ccnp.ry uses t|e stee| be.ns tc n.ke . bu||J|rg. Assune t|.t t|e tct.| p.cJuct c| t|ese
|.ns .ep.eserts t|e cr|y ccnpcrerts c| t|e bu||J|rg .rJ t|.t t|ey .||| |.ve rc ct|e. uses. cnp|ete t|e
|c||c.|rg t.b|e.
Company Product 7otaI SaIes VaIue Added
Acne V|r|rg |.cr c.e 100,000 `
|u||e. V||| stee| be.ns 15,000 `
..re crst.uct|cr bu||J|rg 1,100,000 `
7oto/ vo/ue AJJeJ `
. +cu ..e g|ver t|e J.t. be|c. |c. 2008 |c. t|e |n.g|r..y ccurt.y c| An.g.e, .|cse cu..ercy |s t|e C.
542 PRINCIPLS OI CONOMICS
crsunpt|cr 350 b||||cr C
..rs|e. p.ynerts 100 b||||cr C
|rvestnert 100 b||||cr C
Ccve.rnert pu.c|.ses 200 b||||cr C
|xpc.ts 50 b||||cr C
|npc.ts 150 b||||cr C
bcrJ pu.c|.ses 200 b||||cr C
|..r|rgs cr |c.e|gr |rvestnerts 5 b||||cr C
|c.e|gr e..r|rgs cr An.g.e |rvestnert 25 b||||cr C
.. cnpute ret |c.e|gr |rvestnert.
b. cnpute ret expc.ts.
c. cnpute C||.
J. cnpute C||.
CHAP7R 21 MASURING 7O7AL OU7PU7 AND INCOM 543
1.
2.
3.
4.
5.
6.
.
8.
ENDNOTES
A|t|cug| .epc.teJ sep...te|y by t|e |ep..tnert c| cnne.ce, .e |.ve ccnb|reJ
p.cp.|etc.s |rccne (typ|c.||y |rJeperJert bus|ress c.re.s .rJ |..ne.s) .|t| cc.
pc..te p.c|ts tc s|np|||y t|e J|scuss|cr.
|| ycu |.ve stuJ|eJ n|c.ceccrcn|cs, ycu krc. t|.t t|e te.n e| |r eccrcn|cs |.s
. ,u|te J||e.ert ne.r|rg. |e r.t|cr.| |rccne .rJ p.cJuct .cccurts use t|e .c
ccurt|rg, rct t|e eccrcn|c, ne.r|rg c| e|.
|e .Justnert |c. |rJ|.ect bus|ress t.xes |rc|uJes t.c ct|e. n|rc. e|enerts. t..rs
|e. p.ynerts n.Je by bus|ress |.ns .rJ su.p|uses c. Je|c|ts c| gcve.rnert
erte.p.|ses.
|.c|t |s cc.pc..te p.c|t (1,146.1) p|us p.cp.|etc.s |rccne (1,080.6), bct| .|t| |r
vertc.y v.|u.t|cr .rJ c.p|t.| ccrsunpt|cr .Justnert.
|rJ|.ect t.xes |rc|uJe t.xes cr p.cJuct|cr .rJ |npc.ts c| 1,042.5 p|us bus|ress
t..rs|e. p.ynerts (92.8) |ess subs|J|es (50.4) .rJ cu..ert su.p|us c| gcve.rnert
erte.p.|se (8). |.|c. tc t|e 2003 |.t|cr.| |rccne .rJ |.cJuct Acccurts (|||A) .ev|
s|crs, t|e c.tegc.y t.xes cr p.cJuct|cr .rJ |npc.ts ..s, .|t| scne tec|r|c.| .rJ
ct|e. n|rc. .Justnerts, .e|e..eJ tc .s |rJ|.ect bus|ress t.xes.
'.ck |. .|p|ett .rJ b...y |. bcs.c.t|, |e St.te c| |.t. |c. Se.v|ces |.cJuct|v|ty
Ve.su.enert |r t|e |r|teJ St.tes, ||eo||oo| |oJoc||.||, /o||o 16 (Sp.|rg
2008). 530.
|e.e ..e t.c except|crs tc t||s .u|e. |e v.|ue c| |ccJ p.cJuceJ .rJ ccrsuneJ by
|..n |cuse|c|Js |s ccurteJ |r C||. Vc.e |npc.t.rt, .r est|n.te c| t|e .ert.| v.|ues
c| c.re.cccup|eJ |cnes |s |rc|uJeJ. || . |.n||y .erts . |cuse, t|e .ert.| p.ynerts
..e |rc|uJeJ |r C||. || . |.n||y ||ves |r . |cuse |t c.rs, t|e |ep..tnert c| cn
ne.ce est|n.tes .|.t t|e |cuse .cu|J .ert |c. .rJ t|er |rc|uJes t|.t .ert |r t|e
C|| est|n.te, ever t|cug| t|e |cuses se.v|ces .e.e rct exc|.rgeJ |r t|e
n..ketp|.ce.
'. Stever |.rJe|e|J .rJ Step|.r|e |. Vcu||., Acccurt|rg |c. |crn..ket |cuse
|c|J |.cJuct|cr .|t||r . |.t|cr.| Acccurts |..ne.c.k, |e.|e. o| |coe /eo|||
46, rc. 3 (Septenbe. 2000). 28930.
544 PRINCIPLS OI CONOMICS
potentiaI output
|e |eve| c| cutput .r
eccrcny c.r .c||eve .|er
|.bc. |s enp|cyeJ .t |ts
r.tu..| |eve|.
| A | | | 2 2
Aggregate Demand and
Aggregate Supply
S7AR7 UP: 7H GRA7 WARNING
|e |.st ...r|rg c.ne |.cn t|e |..v..J |ccrcn|c Scc|ety, .r .sscc|.t|cr c| |..v..J eccrcn|cs p.c|essc.s, e..|y |r
1929. |e scc|ety p.eJ|cteJ |r |ts .eek|y re.s|ette. t|.t t|e severye..c|J exp.rs|cr ..s ccn|rg tc .r erJ. |e
cess|cr ..s .|e.J. A|ncst rc cre tcck t|e ...r|rg se.|cus|y. |e eccrcny, |ue|eJ by sc..|rg |rvestnert, |.J ex
pe.|erceJ sturr|rg g.c.t|. |e 1920s |.J seer t|e ene.gerce c| n.ry ert|.e|y re. |rJust.|es.utcncb||es,
pub||c pc.e., |cne .pp||.rces, syrt|et|c |.b.|cs, ..J|c, .rJ nct|cr p|ctu.es. |e Jec.Je seeneJ tc |.ve .c,u|.eJ
. ncnertun .|| |ts c.r. |.cspe.|ty ..s rct .bcut tc erJ, rc n.tte. .|.t . |e. eccrcn|sts n|g|t s.y.
Sunne. c.ne, .rJ rc .ecess|cr ..s .pp..ert. |e |..v..J eccrcn|sts .|t|J.e. t|e|. |c.ec.st. As |t tu.reJ
cut, t|ey |cst t|e|. re.ve tcc sccr. |rJeeJ, |rJust.|.| p.cJuct|cr |.J .|.e.Jy begur tc |.||. |e .c.st Jc.rtu.r |r
cu. ||stc.y, t|e C.e.t |ep.ess|cr, |.J begur.
|e cc||.pse ..s s.||t. |e stcck n..ket c..s|eJ |r Octcbe. 1929. |e.| C|| p|urgeJ re..|y 10 by 1930. by
t|e t|ne t|e eccrcny ||t bcttcn |r 1933, .e.| C|| |.J |.||er 30, urenp|cynert |.J |rc.e.seJ |.cn 3.2 |r
1929 tc 25 |r 1933, .rJ p.|ces, ne.su.eJ by t|e |np||c|t p.|ce Je|.tc., |.J p|urgeJ 23 |.cn t|e|. 1929 |eve|.
|e Jep.ess|cr |e|J t|e eccrcny |r |ts c.ue| g.|p |c. nc.e t|.r . Jec.Je, |t ..s rct urt|| \c.|J \.. || t|.t |u|| en
p|cynert ..s .estc.eJ.
|r t||s c|.pte. .e gc beycrJ exp|.r.t|crs c| t|e n.|r n.c.ceccrcn|c v..|.b|es tc |rt.cJuce . ncJe| c| n.c
.ceccrcn|c .ct|v|ty t|.t .e c.r use tc .r.|y.e p.cb|ens suc| .s |uctu.t|crs |r g.css Jcnest|c p.cJuct (.e.| C||),
t|e p.|ce |eve|, .rJ enp|cynert. t|e ncJe| c| .gg.eg.te Jen.rJ .rJ .gg.eg.te supp|y. \e .||| use t||s ncJe|
t|.cug|cut cu. exp|c..t|cr c| n.c.ceccrcn|cs. |r t||s c|.pte. .e .||| p.esert t|e b.c.J cut||res c| t|e ncJe|,
g.e.te. Jet.||, nc.e ex.np|es, .rJ nc.e t|c.cug| exp|.r.t|crs .||| |c||c. |r subse,uert c|.pte.s.
\e .||| ex.n|re t|e ccrcepts c| t|e .gg.eg.te Jen.rJ cu.ve .rJ t|e s|c.t .rJ |crg.ur .gg.eg.te supp|y
cu.ves. \e .||| |Jert||y ccrJ|t|crs urJe. .||c| .r eccrcny .c||eves .r e,u|||b.|un |eve| c| .e.| C|| t|.t |s ccr
s|stert .|t| |u|| enp|cynert c| |.bc.. PotentiaI output |s t|e |eve| c| cutput .r eccrcny c.r .c||eve .|er
|.bc. |s enp|cyeJ .t |ts r.tu..| |eve|. |ctert|.| cutput |s .|sc c.||eJ t|e r.tu..| |eve| c| .e.| C||. \|er .r eccrcny
|.||s tc p.cJuce .t |ts pctert|.|, t|e.e n.y be .ct|crs t|.t t|e gcve.rnert c. t|e cert..| b.rk c.r t.ke tc pus| t|e
eccrcny tc...J |t, .rJ |r t||s c|.pte. .e .||| beg|r tc ccrs|Je. t|e p.cs .rJ ccrs c| Jc|rg sc.
aggregate demand
|e .e|.t|crs||p bet.eer t|e
tct.| ,u.rt|ty c| gccJs .rJ
se.v|ces Jen.rJeJ (|.cn .||
t|e |cu. scu.ces c| Jen.rJ)
.rJ t|e p.|ce |eve|, .|| ct|e.
Jete.n|r.rts c| sperJ|rg
urc|.rgeJ.
aggregate demand curve
A g..p||c.| .ep.esert.t|cr c|
.gg.eg.te Jen.rJ.
1. AGGREGATE DEMAND
L A R N I N G O 8 1 C 7 I V S
1. Dene potentiaI output, aIso caIIed the naturaI IeveI of GDP.
2. Dene aggregate demand, represent it using a hypotheticaI aggregate demand curve, and
identify and expIain the three eects that cause this curve to sIope downward.
3. Distinguish between a change in the aggregate quantity of goods and services demanded and
a change in aggregate demand.
4. Use exampIes to expIain how each component of aggregate demand can be a possibIe aggreg-
ate demand shifter.
5. xpIain what a muItipIier is and teII how to caIcuIate it.
Fiims face foui souices of demand: households (peisonal consumption), othei fims (investment), gov-
einment agencies (goveinment puichases), and foieign maikets (net expoits). Aggregate demand is
the ielationship between the total quantity of goods and seivices demanded (fiom all the foui souices
of demand) and the piice level, all othei deteiminants of spending unchanged. The aggregate de-
mand curve is a giaphical iepiesentation of aggiegate demand.
1.1 The Slope of the Aggiegate Demand Cuive
We will use the implicit piice defatoi as oui measuie of the piice level; the aggiegate quantity of goods
and seivices demanded is measuied as ieal GDP. The table in Figuie 22.1 gives values foi each com-
ponent of aggiegate demand at each piice level foi a hypothetical economy. Vaiious points on the ag-
giegate demand cuive aie found by adding the values of these components at difeient piice levels. The
aggiegate demand cuive foi the data given in the table is plotted on the giaph in Figuie 22.1. At point
A, at a piice level of 1.18, $11,800 billion woith of goods and seivices will be demanded; at point C, a
ieduction in the piice level to 1.14 incieases the quantity of goods and seivices demanded to $12,000
billion; and at point E, at a piice level of 1.10, $12,200 billion will be demanded.
II GUR 22. 1 Aggregate Demand
Ar .gg.eg.te Jen.rJ cu.ve (^|) s|c.s t|e .e|.t|crs||p bet.eer t|e tct.| ,u.rt|ty c| cutput Jen.rJeJ
(ne.su.eJ .s .e.| C||) .rJ t|e p.|ce |eve| (ne.su.eJ .s t|e |np||c|t p.|ce Je|.tc.). At e.c| p.|ce |eve|, t|e tct.|
,u.rt|ty c| gccJs .rJ se.v|ces Jen.rJeJ |s t|e sun c| t|e ccnpcrerts c| .e.| C||, .s s|c.r |r t|e t.b|e. |e.e
|s . reg.t|ve .e|.t|crs||p bet.eer t|e p.|ce |eve| .rJ t|e tct.| ,u.rt|ty c| gccJs .rJ se.v|ces Jen.rJeJ, .|| ct|e.
t||rgs urc|.rgeJ.
546 PRINCIPLS OI CONOMICS
weaIth eect
|e terJercy |c. . c|.rge |r
t|e p.|ce |eve| tc .|ect .e.|
.e.|t| .rJ t|us .|te.
ccrsunpt|cr.
interest rate eect
|e terJercy |c. . c|.rge |r
t|e p.|ce |eve| tc .|ect t|e
|rte.est ..te .rJ t|us tc
.|ect t|e ,u.rt|ty c|
|rvestnert Jen.rJeJ.
internationaI trade eect
|e terJercy |c. . c|.rge |r
t|e p.|ce |eve| tc .|ect ret
expc.ts.
change in the aggregate
quantity of goods and
services demanded
Vcvenert .|crg .r
.gg.eg.te Jen.rJ cu.ve.
The negative slope of the aggiegate demand cuive suggests that it behaves in the same mannei as an oi-
dinaiy demand cuive. But we cannot apply the ieasoning we use to explain downwaid-sloping demand
cuives in individual maikets to explain the downwaid-sloping aggiegate demand cuive. Theie aie two
ieasons foi a negative ielationship between piice and quantity demanded in individual maikets. Fiist, a
lowei piice induces people to substitute moie of the good whose piice has fallen foi othei goods, in-
cieasing the quantity demanded. Second, the lowei piice cieates a highei ieal income. This noimally
incieases quantity demanded fuithei.
Neithei of these efects is ielevant to a change in piices in the aggiegate. When we aie dealing with
the aveiage of all piices-the piice level-we can no longei say that a fall in piices will induce a change
in ielative piices that will lead consumeis to buy moie of the goods and seivices whose piices have
fallen and less of the goods and seivices whose piices have not fallen. The piice of coin may have fallen,
but the piices of wheat, sugai, tiactois, steel, and most othei goods oi seivices pioduced in the eco-
nomy aie likely to have fallen as well.
Fuitheimoie, a ieduction in the piice level means that it is not just the piices consumeis pay that
aie falling. It means the piices people ieceive-theii wages, the ients they may chaige as landloids, the
inteiest iates they eain-aie likely to be falling as well. A falling piice level means that goods and sei-
vices aie cheapei, but incomes aie lowei, too. Theie is no ieason to expect that a change in ieal income
will boost the quantity of goods and seivices demanded-indeed, no change in ieal income would oc-
cui. If nominal incomes and piices all fall by 10, foi example, ieal incomes do not change.
Why, then, does the aggiegate demand cuive slope downwaid: One ieason foi the downwaid
slope of the aggiegate demand cuive lies in the ielationship between ieal wealth (the stocks, bonds, and
othei assets that people have accumulated) and consumption (one of the foui components of aggiegate
demand). When the piice level falls, the ieal value of wealth incieases-it packs moie puichasing
powei. Foi example, if the piice level falls by 23, then $10,000 of wealth could puichase moie goods
and seivices than it would have if the piice level had not fallen. An inciease in wealth will induce
people to inciease theii consumption. The consumption component of aggiegate demand will thus be
gieatei at lowei piice levels than at highei piice levels. The tendency foi a change in the piice level to
afect ieal wealth and thus altei consumption is called the wealth enect; it suggests a negative ielation-
ship between the piice level and the ieal value of consumption spending.
A second ieason the aggiegate demand cuive slopes downwaid lies in the ielationship between in-
teiest iates and investment. A lowei piice level loweis the demand foi money, because less money is ie-
quiied to buy a given quantity of goods. What economists mean by money demand will be explained in
moie detail in a latei chaptei. But, as we leained in studying demand and supply, a ieduction in the de-
mand foi something, all othei things unchanged, loweis its piice. In this case, the something" is
money and its piice is the inteiest iate. A lowei piice level thus ieduces inteiest iates. Lowei inteiest
iates make boiiowing by fims to build factoiies oi buy equipment and othei capital moie attiactive. A
lowei inteiest iate means lowei moitgage payments, which tends to inciease investment in iesidential
houses. Investment thus iises when the piice level falls. The tendency foi a change in the piice level to
afect the inteiest iate and thus to afect the quantity of investment demanded is called the interest
rate enect. John Maynaid Keynes, a Biitish economist whose analysis of the Gieat Depiession and
what to do about it led to the biith of modein macioeconomics, emphasized this efect. Foi this ieason,
the inteiest iate efect is sometimes called the Keynes efect.
A thiid ieason foi the iise in the total quantity of goods and seivices demanded as the piice level
falls can be found in changes in the net expoit component of aggiegate demand. All othei things un-
changed, a lowei piice level in an economy ieduces the piices of its goods and seivices ielative to
foieign-pioduced goods and seivices. A lowei piice level makes that economy's goods moie attiactive
to foieign buyeis, incieasing expoits. It will also make foieign-pioduced goods and seivices less attiact-
ive to the economy's buyeis, ieducing impoits. The iesult is an inciease in net expoits. The interna-
tional trade enect is the tendency foi a change in the piice level to afect net expoits.
Taken togethei, then, a fall in the piice level means that the quantities of consumption, invest-
ment, and net expoit components of aggiegate demand may all iise. Since goveinment puichases aie
deteimined thiough a political piocess, we assume theie is no causal link between the piice level and
the ieal volume of goveinment puichases. Theiefoie, this component of GDP does not contiibute to
the downwaid slope of the cuive.
In geneial, a change in the piice level, with all othei deteiminants of aggiegate demand un-
changed, causes a movement along the aggiegate demand cuive. A movement along an aggiegate de-
mand cuive is a change in the aggregate quantity of goods and services demanded. A move-
ment fiom point A to point B on the aggiegate demand cuive in Figuie 22.1 is an example. Such a
change is a iesponse to a change in the piice level.
Notice that the axes of the aggiegate demand cuive giaph aie diawn with a bieak neai the oiigin to
iemind us that the plotted values iefect a ielatively naiiow iange of changes in ieal GDP and the piice
level. We do not know what might happen if the piice level oi output foi an entiie economy ap-
pioached zeio. Such a phenomenon has nevei been obseived.
CHAP7R 22 AGGRGA7 DMAND AND AGGRGA7 SUPPLY 547
change in aggregate
demand
|.rge |r t|e .gg.eg.te
,u.rt|ty c| gccJs .rJ
se.v|ces Jen.rJeJ .t eve.y
p.|ce |eve|.
1.2 Changes in Aggiegate Demand
Aggiegate demand changes in iesponse to a change in any of its components. An inciease in the total
quantity of consumei goods and seivices demanded at eveiy piice level, foi example, would shift the
aggiegate demand cuive to the iight. A change in the aggiegate quantity of goods and seivices deman-
ded at eveiy piice level is a change in aggregate demand, which shifts the aggiegate demand cuive.
Incieases and decieases in aggiegate demand aie shown in Figuie 22.2.
II GUR 22. 2 Changes in Aggregate Demand
Ar |rc.e.se |r ccrsunpt|cr, |rvestnert, gcve.rnert pu.c|.ses, c. ret expc.ts s|||ts t|e .gg.eg.te Jen.rJ cu.ve
^|
1
tc t|e .|g|t .s s|c.r |r |.re| (.). A .eJuct|cr |r cre c| t|e ccnpcrerts c| .gg.eg.te Jen.rJ s|||ts t|e cu.ve
tc t|e |e|t, .s s|c.r |r |.re| (b).
What factois might cause the aggiegate demand cuive to shift: Each of the components of aggiegate
demand is a possible aggiegate demand shiftei. We shall look at some of the events that can tiiggei
changes in the components of aggiegate demand and thus shift the aggiegate demand cuive.
Changes in Consumption
Seveial events could change the quantity of consumption at each piice level and thus shift aggiegate
demand. One deteiminant of consumption is consumei confdence. If consumeis expect good eco-
nomic conditions and aie optimistic about theii economic piospects, they aie moie likely to buy majoi
items such as cais oi fuinituie. The iesult would be an inciease in the ieal value of consumption at
each piice level and an inciease in aggiegate demand. In the second half of the 1990s, sustained eco-
nomic giowth and low unemployment fueled high expectations and consumei optimism. Suiveys ie-
vealed consumei confdence to be veiy high. That consumei confdence tianslated into incieased con-
sumption and incieased aggiegate demand. In contiast, a deciease in consumption would accompany
diminished consumei expectations and a deciease in consumei confdence, as happened aftei the stock
maiket ciash of 1929. The same pioblem has plagued the economies of most Westein nations in 2008
as declining consumei confdence has tended to ieduce consumption. A suivey by the Confeience
Boaid in Septembei of 2008 showed that just 13.3 of consumeis suiveyed expected economic condi-
tions in the United States to impiove in the next six months. Similaily pessimistic views pievailed in
the pievious two months. That contiibuted to the decline in consumption that occuiied in the thiid
quaitei of the yeai.
Anothei factoi that can change consumption and shift aggiegate demand is tax policy. A cut in
peisonal income taxes leaves people with moie aftei-tax income, which may induce them to inciease
theii consumption. The fedeial goveinment in the United States cut taxes in 1964, 1981, 1986, 1997,
and 2003; each of those tax cuts tended to inciease consumption and aggiegate demand at each piice
level.
In the United States, anothei goveinment policy aimed at incieasing consumption and thus ag-
giegate demand has been the use of iebates in which taxpayeis aie simply sent checks in hopes that
548 PRINCIPLS OI CONOMICS
exchange rate
|e p.|ce c| . cu..ercy |r
te.ns c| .rct|e. cu..ercy c.
cu..erc|es.
those checks will be used foi consumption. Rebates have been used in 1973, 2001, and 2008. In each
case the iebate was a one-time payment. Caieful studies by economists of the 1973 and 2001 iebates
showed little impact on consumption. Final evidence on the impact of the 2008 iebates is not yet in, but
eaily iesults suggest a similai outcome. In a subsequent chaptei, we will investigate aiguments about
whethei tempoiaiy incieases in income pioduced by iebates aie likely to have a signifcant impact on
consumption.
Tiansfei payments such as welfaie and Social Secuiity also afect the income people have available
to spend. At any given piice level, an inciease in tiansfei payments iaises consumption and aggiegate
demand, and a ieduction loweis consumption and aggiegate demand.
Changes in Investment
Investment is the pioduction of new capital that will be used foi futuie pioduction of goods and sei-
vices. Fiims make investment choices based on what they think they will be pioducing in the futuie.
The expectations of fims thus play a ciitical iole in deteimining investment. If fims expect theii sales
to go up, they aie likely to inciease theii investment so that they can inciease pioduction and meet
consumei demand. Such an inciease in investment iaises the aggiegate quantity of goods and seivices
demanded at each piice level; it incieases aggiegate demand.
Changes in inteiest iates also afect investment and thus afect aggiegate demand. We must be
caieful to distinguish such changes fiom the inteiest iate efect, which causes a movement along the ag-
giegate demand cuive. A change in inteiest iates that iesults fiom a change in the piice level afects in-
vestment in a way that is alieady captuied in the downwaid slope of the aggiegate demand cuive; it
causes a movement along the cuive. A change in inteiest iates foi some othei ieason shifts the cuive.
We examine ieasons inteiest iates might change in anothei chaptei.
Investment can also be afected by tax policy. One piovision of the Job and Giowth Tax Relief Re-
conciliation Act of 2003 was a ieduction in the tax iate on ceitain capital gains. Capital gains iesult
when the ownei of an asset, such as a house oi a factoiy, sells the asset foi moie than its puichase piice
(less any depieciation claimed in eailiei yeais). The lowei capital gains tax could stimulate investment,
because the owneis of such assets know that they will lose less to taxes when they sell those assets, thus
making assets subject to the tax moie attiactive.
Changes in Government Purchases
Any change in goveinment puichases, all othei things unchanged, will afect aggiegate demand. An in-
ciease in goveinment puichases incieases aggiegate demand; a deciease in goveinment puichases de-
cieases aggiegate demand.
Many economists aigued that ieductions in defense spending in the wake of the collapse of the
Soviet Union in 1991 tended to ieduce aggiegate demand. Similaily, incieased defense spending foi the
wais in Afghanistan and Iiaq incieased aggiegate demand. Diamatic incieases in defense spending to
fght Woild Wai II accounted in laige pait foi the iapid iecoveiy fiom the Gieat Depiession.
Changes in Net xports
A change in the value of net expoits at each piice level shifts the aggiegate demand cuive. A majoi de-
teiminant of net expoits is foieign demand foi a countiy's goods and seivices; that demand will vaiy
with foieign incomes. An inciease in foieign incomes incieases a countiy's net expoits and aggiegate
demand; a slump in foieign incomes ieduces net expoits and aggiegate demand. Foi example, seveial
majoi U.S. tiading paitneis in Asia sufeied iecessions in 1997 and 1998. Lowei ieal incomes in those
countiies ieduced U.S. expoits and tended to ieduce aggiegate demand.
Exchange iates also infuence net expoits, all othei things unchanged. A countiy's exchange rate
is the piice of its cuiiency in teims of anothei cuiiency oi cuiiencies. A iise in the U.S. exchange iate
means that it takes moie Japanese yen, foi example, to puichase one dollai. That also means that U.S.
tiadeis get moie yen pei dollai. Since piices of goods pioduced in Japan aie given in yen and piices of
goods pioduced in the United States aie given in dollais, a iise in the U.S. exchange iate incieases the
piice to foieigneis foi goods and seivices pioduced in the United States, thus ieducing U.S. expoits; it
ieduces the piice of foieign-pioduced goods and seivices foi U.S. consumeis, thus incieasing impoits
to the United States. A highei exchange iate tends to ieduce net expoits, ieducing aggiegate demand. A
lowei exchange iate tends to inciease net expoits, incieasing aggiegate demand.
Foieign piice levels can afect aggiegate demand in the same way as exchange iates. Foi example,
when foieign piice levels fall ielative to the piice level in the United States, U.S. goods and seivices be-
come ielatively moie expensive, ieducing expoits and boosting impoits in the United States. Such a ie-
duction in net expoits ieduces aggiegate demand. An inciease in foieign piices ielative to U.S. piices
has the opposite efect.
The tiade policies of vaiious countiies can also afect net expoits. A policy by Japan to inciease its
impoits of goods and seivices fiom India, foi example, would inciease net expoits in India.
CHAP7R 22 AGGRGA7 DMAND AND AGGRGA7 SUPPLY 549
muItipIier
|e ..t|c c| t|e c|.rge |r t|e
,u.rt|ty c| .e.| C||
Jen.rJeJ .t e.c| p.|ce |eve|
tc t|e |r|t|.| c|.rge |r cre c.
nc.e ccnpcrerts c|
.gg.eg.te Jen.rJ t|.t
p.cJuceJ |t.
7he MuItipIier
A change in any component of aggiegate demand shifts the aggiegate demand cuive. Geneially, the ag-
giegate demand cuive shifts by moie than the amount by which the component initially causing it to
shift changes.
Suppose that net expoits inciease due to an inciease in foieign incomes. As foieign demand foi
domestically made pioducts iises, a countiy's fims will hiie additional woikeis oi peihaps inciease the
aveiage numbei of houis that theii employees woik. In eithei case, incomes will iise, and highei in-
comes will lead to an inciease in consumption. Taking into account these othei incieases in the com-
ponents of aggiegate demand, the aggiegate demand cuive will shift by moie than the initial shift
caused by the initial inciease in net expoits.
The multiplier is the iatio of the change in the quantity of ieal GDP demanded at each piice level
to the initial change in one oi moie components of aggiegate demand that pioduced it:
QUA7I ON 22. 1
Multipliei =
A (ieal GDP demanded at each piice level)
initial A (component of AD)
We use the capital Gieek lettei delta (A) to mean change in." In the aggiegate demand-aggiegate
supply model piesented in this chaptei, it is the numbei by which we multiply an initial change in ag-
giegate demand to obtain the amount by which the aggiegate demand cuive shifts as a iesult of the ini-
tial change. In othei woids, we can use Equation 22.1 to solve foi the change in ieal GDP demanded at
each piice level:
QUA7I ON 22. 2
A (ieal GDP demanded at each piice level) = multipliei initial A (component of AD)
Suppose that the initial inciease in net expoits is $100 billion and that the initial $100-billion in-
ciease geneiates additional consumption of $100 billion at each piice level. In Panel (a) of Figuie 22.3,
the aggiegate demand cuive shifts to the iight by $200 billion-the amount of the initial inciease in net
expoits times the multipliei of 2. We obtained the value foi the multipliei in this example by plugging
$200 billion (the initial $100-billion inciease in net expoits plus the $100-billion inciease that it genei-
ated in consumption) into the numeiatoi of Equation 22.1 and $100 billion into the denominatoi. Sim-
ilaily, a deciease in net expoits of $100 billion leads to a deciease in aggiegate demand of $200 billion
at each piice level, as shown in Panel (b).
II GUR 22. 3 7he MuItipIier
A c|.rge |r cre ccnpcrert c| .gg.eg.te Jen.rJ s|||ts t|e .gg.eg.te Jen.rJ cu.ve by nc.e t|.r t|e |r|t|.|
c|.rge. |r |.re| (.), .r |r|t|.| |rc.e.se c| 100 b||||cr c| ret expc.ts s|||ts t|e .gg.eg.te Jen.rJ cu.ve tc t|e .|g|t
by 200 b||||cr .t e.c| p.|ce |eve|. |r |.re| (b), . Jec.e.se c| ret expc.ts c| 100 b||||cr s|||ts t|e .gg.eg.te
Jen.rJ cu.ve tc t|e |e|t by 200 b||||cr. |r t||s ex.np|e, t|e nu|t|p||e. |s 2.
550 PRINCIPLS OI CONOMICS
k Y 7 A k A W A Y S
< |ctert|.| cutput |s t|e |eve| c| cutput .r eccrcny c.r .c||eve .|er |.bc. |s enp|cyeJ .t |ts r.tu..| |eve|.
\|er .r eccrcny |.||s tc p.cJuce .t |ts pctert|.|, t|e gcve.rnert c. t|e cert..| b.rk n.y t.y tc pus|
t|e eccrcny tc...J |ts pctert|.|.
< |e .gg.eg.te Jen.rJ cu.ve .ep.eserts t|e tct.| c| ccrsunpt|cr, |rvestnert, gcve.rnert pu.c|.ses,
.rJ ret expc.ts .t e.c| p.|ce |eve| |r .ry pe.|cJ. |t s|cpes Jc.r...J bec.use c| t|e .e.|t| e|ect cr
ccrsunpt|cr, t|e |rte.est ..te e|ect cr |rvestnert, .rJ t|e |rte.r.t|cr.| t..Je e|ect cr ret expc.ts.
< |e .gg.eg.te Jen.rJ cu.ve s|||ts .|er t|e ,u.rt|ty c| .e.| C|| Jen.rJeJ .t e.c| p.|ce |eve| c|.rges.
< |e nu|t|p||e. |s t|e runbe. by .||c| .e nu|t|p|y .r |r|t|.| c|.rge |r .gg.eg.te Jen.rJ tc cbt.|r t|e
.ncurt by .||c| t|e .gg.eg.te Jen.rJ cu.ve s|||ts .t e.c| p.|ce |eve| .s . .esu|t c| t|e |r|t|.| c|.rge.
7 R Y I 7 !
|xp|.|r t|e e|ect c| e.c| c| t|e |c||c.|rg cr t|e .gg.eg.te Jen.rJ cu.ve |c. t|e |r|teJ St.tes.
.. A Jec.e.se |r ccrsune. cpt|n|sn
b. Ar |rc.e.se |r .e.| C|| |r t|e ccurt.|es t|.t buy |.S. expc.ts
c. Ar |rc.e.se |r t|e p.|ce |eve|
J. Ar |rc.e.se |r gcve.rnert sperJ|rg cr ||g|..ys
Case in Point: 7he MuItipIied conomic Impact of SARS on China's conomy
Seve.e Acute |esp|..tc.y SyrJ.cne (SA|S), .r .typ|c.| preuncr|.||ke J|se.se, b.cke crtc t|e .c.|J scere |r
|.te 2002. |r V..c| 2003, t|e \c.|J |e.|t| O.g.r|..t|cr (\|O) |ssueJ |ts |.st .c.|J.|Je .|e.t .rJ . ncrt|
|.te. |ts |.st t..ve| .Jv|sc.y, .||c| .eccnnerJeJ t|.t t..ve|e.s .vc|J |crg |crg .rJ t|e scut|e.r p.cv|rce
c| ||r., Cu.rgJcrg. Ove. t|e rext |e. ncrt|s, .JJ|t|cr.| t..ve| .Jv|sc.|es .e.e |ssueJ |c. ct|e. p..ts c| |
|r., .|..r, .rJ b.|e|y |c. c.crtc, .r.J.. by t|e erJ c| 'ure, .|| \|O t..ve| .Jv|sc.|es |.J beer .encveJ.
c est|n.te t|e cve..|| |np.ct c| SA|S cr t|e ||rese eccrcny |r 2003, eccrcn|sts \er |.|, |crg |.c,
.rJ '|.r \.rt c| |ek|rg |r|ve.s|tys ||r. erte. |c. |ccrcn|c |ese..c| ccrJucteJ . su.vey c| be|rgs tcu.
|sn |rJust.y |r Ap.|| 2003. b.seJ cr |rJ|rgs |.cn t|e be|rg ..e., t|ey p.cecteJ t|e tcu.|sn sectc. c| ||r.
.s . .|c|e .cu|J |cse 16.8 b||||crc| .||c| 10.8 b||||cr c.ne |.cn .r .pp.cx|n.te 50 .eJuct|cr |r |c.
e|gr tcu.|st .everue .rJ 6 b||||cr |.cn cu.t.||eJ Jcnest|c tcu.|sn, .s |c||J.y ce|eb..t|crs .e.e c.rce||eJ
.rJ Jcnest|c t..ve| .est.|ct|crs |npcseJ.
c |gu.e cut t|e tct.| |np.ct c| SA|S cr ||r.s eccrcny, t|ey ..gueJ t|.t t|e nu|t|p||e. |c. tcu.|sn .ever
ue |r ||r. |s bet.eer 2 .rJ 3. S|rce t|e SA|S cutb.e.k cr|y beg.r tc |.ve . n.c. eccrcn|c |np.ct .|te.
V..c|, t|ey .ssuneJ . sn.||e. nu|t|p||e. c| 1.5 |c. .|| c| 2003. |ey t|us p.eJ|cteJ t|.t t|e ||rese eccrcny
.cu|J be 25.3 b||||cr sn.||e. |r 2003 .s . .esu|t c| SA|S.
5ource /e |o|. |o |oo. oJ .|o /o. |e .|o|e |oc| o| .^|. o ||e ||ee |coo,.' ^|o |coo|c |oe 3. o (/||e
2004: 5.6
CHAP7R 22 AGGRGA7 DMAND AND AGGRGA7 SUPPLY 551
short run
|r n.c.ceccrcn|c .r.|ys|s, .
pe.|cJ |r .||c| ..ges .rJ
scne ct|e. p.|ces ..e st|cky
.rJ Jc rct .espcrJ tc
c|.rges |r eccrcn|c
ccrJ|t|crs.
sticky price
A p.|ce t|.t |s s|c. tc .Just
tc |ts e,u|||b.|un |eve|,
c.e.t|rg sust.|reJ pe.|cJs c|
s|c.t.ge c. su.p|us.
Iong run
|r n.c.ceccrcn|c .r.|ys|s, .
pe.|cJ |r .||c| ..ges .rJ
p.|ces ..e |ex|b|e.
A N S W R 7 O 7 R Y I 7 ! P R O 8 L M
.. A Jec||re |r ccrsune. cpt|n|sn .cu|J c.use t|e .gg.eg.te Jen.rJ cu.ve tc s|||t tc t|e |e|t. ||
ccrsune.s ..e nc.e pess|n|st|c .bcut t|e |utu.e, t|ey ..e ||ke|y tc cut pu.c|.ses, espec|.||y c| n.c.
|tens.
b. Ar |rc.e.se |r t|e .e.| C|| c| ct|e. ccurt.|es .cu|J |rc.e.se t|e Jen.rJ |c. |.S. expc.ts .rJ c.use t|e
.gg.eg.te Jen.rJ cu.ve tc s|||t tc t|e .|g|t. ||g|e. |rccnes |r ct|e. ccurt.|es .||| n.ke ccrsune.s |r
t|cse ccurt.|es nc.e .||||rg .rJ .b|e tc buy |.S. gccJs.
c. Ar |rc.e.se |r t|e p.|ce |eve| cc..espcrJs tc . ncvenert up .|crg t|e urc|.rgeJ .gg.eg.te Jen.rJ
cu.ve. At t|e ||g|e. p.|ce |eve|, t|e ccrsunpt|cr, |rvestnert, .rJ ret expc.t ccnpcrerts c| .gg.eg.te
Jen.rJ .||| .|| |.||, t|.t |s, t|e.e .||| be . .eJuct|cr |r t|e tct.| ,u.rt|ty c| gccJs .rJ se.v|ces Jen.rJeJ,
but rct . s|||t c| t|e .gg.eg.te Jen.rJ cu.ve |tse||.
J. Ar |rc.e.se |r gcve.rnert sperJ|rg cr ||g|..ys ne.rs .r |rc.e.se |r gcve.rnert pu.c|.ses. |e
.gg.eg.te Jen.rJ cu.ve .cu|J s|||t tc t|e .|g|t.
2. AGGREGATE DEMAND AND AGGREGATE SUPPLY:
THE LONG RUN AND THE SHORT RUN
L A R N I N G O 8 1 C 7 I V S
1. Distinguish between the short run and the Iong run, as these terms are used in
macroeconomics.
2. Draw a hypotheticaI Iong-run aggregate suppIy curve and expIain what it shows about the nat-
uraI IeveIs of empIoyment and output at various price IeveIs, given changes in aggregate
demand.
3. Draw a hypotheticaI short-run aggregate suppIy curve, expIain why it sIopes upward, and ex-
pIain why it may shift, that is, distinguish between a change in the aggregate quantity of goods
and services suppIied and a change in short-run aggregate suppIy.
4. Discuss various expIanations for wage and price stickiness.
5. xpIain and iIIustrate what is meant by equiIibrium in the short run and reIate the equiIibrium
to potentiaI output.
In macioeconomics, we seek to undeistand two types of equilibiia, one coiiesponding to the shoit iun
and the othei coiiesponding to the long iun. The short run in macioeconomic analysis is a peiiod in
which wages and some othei piices do not iespond to changes in economic conditions. In ceitain mai-
kets, as economic conditions change, piices (including wages) may not adjust quickly enough to main-
tain equilibiium in these maikets. A sticky price is a piice that is slow to adjust to its equilibiium
level, cieating sustained peiiods of shoitage oi suiplus. Wage and piice stickiness pievent the economy
fiom achieving its natuial level of employment and its potential output. In contiast, the long run in
macioeconomic analysis is a peiiod in which wages and piices aie fexible. In the long iun, employ-
ment will move to its natuial level and ieal GDP to potential.
We begin with a discussion of long-iun macioeconomic equilibiium, because this type of equilib-
iium allows us to see the macioeconomy aftei full maiket adjustment has been achieved. In contiast, in
the shoit iun, piice oi wage stickiness is an obstacle to full adjustment. Why these deviations fiom the
potential level of output occui and what the implications aie foi the macioeconomy will be discussed
in the section on shoit-iun macioeconomic equilibiium.
2.1 The Long Run
As explained in a pievious chaptei, the natuial level of employment occuis wheie the ieal wage adjusts
so that the quantity of laboi demanded equals the quantity of laboi supplied. When the economy
achieves its natuial level of employment, it achieves its potential level of output. We will see that ieal
GDP eventually moves to potential, because all wages and piices aie assumed to be fexible in the long
iun.
552 PRINCIPLS OI CONOMICS
Iong-run aggregate suppIy
(LRAS) curve
A g..p||c.| .ep.esert.t|cr
t|.t .e|.tes t|e |eve| c|
cutput p.cJuceJ by |.ns tc
t|e p.|ce |eve| |r t|e |crg .ur.
Long-Run Aggregate SuppIy
The long-run aggregate supply (IRAS) curve ielates the level of output pioduced by fims to the
piice level in the long iun. In Panel (b) of Figuie 22.3, the long-iun aggiegate supply cuive is a veitical
line at the economy's potential level of output. Theie is a single ieal wage at which employment ieaches
its natuial level. In Panel (a) of Figuie 22.3, only a ieal wage of a
e
geneiates natuial employment L
e
.
The economy could, howevei, achieve this ieal wage with any of an infnitely laige set of nominal wage
and piice-level combinations. Suppose, foi example, that the equilibiium ieal wage (the iatio of wages
to the piice level) is 1.3. We could have that with a nominal wage level of 1.3 and a piice level of 1.0, a
nominal wage level of 1.63 and a piice level of 1.1, a nominal wage level of 3.0 and a piice level of 2.0,
and so on.
II GUR 22. 5 NaturaI mpIoyment and Long-Run Aggregate SuppIy
\|er t|e eccrcny .c||eves |ts r.tu..| |eve| c| enp|cynert, .s s|c.r |r |.re| (.) .t t|e |rte.sect|cr c| t|e
Jen.rJ .rJ supp|y cu.ves |c. |.bc., |t .c||eves |ts pctert|.| cutput, .s s|c.r |r |.re| (b) by t|e ve.t|c.| |crg.ur
.gg.eg.te supp|y cu.ve ||^. .t `
|
.
In Panel (b) we see piice levels ianging fiom P
1
to P
4
. Highei piice levels would iequiie highei nominal
wages to cieate a ieal wage of a
e
, and fexible nominal wages would achieve that in the long iun.
In the long iun, then, the economy can achieve its natuial level of employment and potential out-
put at any piice level. This conclusion gives us oui long-iun aggiegate supply cuive. With only one
level of output at any piice level, the long-iun aggiegate supply cuive is a veitical line at the economy's
potential level of output of Y
P
.
quiIibrium LeveIs of Price and Output in the Long Run
The inteisection of the economy's aggiegate demand cuive and the long-iun aggiegate supply cuive
deteimines its equilibiium ieal GDP and piice level in the long iun. Figuie 22.6 depicts an economy in
long-iun equilibiium. With aggiegate demand at AD
1
and the long-iun aggiegate supply cuive as
shown, ieal GDP is $12,000 billion pei yeai and the piice level is 1.14. If aggiegate demand incieases to
AD
2
, long-iun equilibiium will be ieestablished at ieal GDP of $12,000 billion pei yeai, but at a highei
piice level of 1.18. If aggiegate demand decieases to AD
3
, long-iun equilibiium will still be at ieal GDP
of $12,000 billion pei yeai, but with the now lowei piice level of 1.10.
CHAP7R 22 AGGRGA7 DMAND AND AGGRGA7 SUPPLY 553
II GUR 22. 6 Long-Run quiIibrium
|crg.ur e,u|||b.|un cccu.s .t t|e |rte.sect|cr c| t|e .gg.eg.te Jen.rJ cu.ve .rJ t|e |crg.ur .gg.eg.te supp|y
cu.ve. |c. t|e t|.ee .gg.eg.te Jen.rJ cu.ves s|c.r, |crg.ur e,u|||b.|un cccu.s .t t|.ee J||e.ert p.|ce |eve|s,
but .|..ys .t .r cutput |eve| c| 12,000 b||||cr pe. ye.., .||c| cc..espcrJs tc pctert|.| cutput.
2.2 The Shoit Run
Analysis of the macioeconomy in the shoit iun-a peiiod in which stickiness of wages and piices may
pievent the economy fiom opeiating at potential output-helps explain how deviations of ieal GDP
fiom potential output can and do occui. We will exploie the efects of changes in aggiegate demand
and in shoit-iun aggiegate supply in this section.
554 PRINCIPLS OI CONOMICS
II GUR 22. 7 Deriving the Short-Run
Aggregate SuppIy Curve
|e eccrcny s|c.r |e.e |s |r |crg.ur
e,u|||b.|un .t t|e |rte.sect|cr c| ^|
1
.|t| t|e
|crg.ur .gg.eg.te supp|y cu.ve. || .gg.eg.te
Jen.rJ |rc.e.ses tc ^|
2
, |r t|e s|c.t .ur,
bct| .e.| C|| .rJ t|e p.|ce |eve| .|se. ||
.gg.eg.te Jen.rJ Jec.e.ses tc ^|
3
, |r t|e
s|c.t .ur, bct| .e.| C|| .rJ t|e p.|ce |eve| |.||.
A ||re J...r t|.cug| pc|rts A, b, .rJ t..ces
cut t|e s|c.t.ur .gg.eg.te supp|y cu.ve .|^..
short-run aggregate suppIy
(SRAS) curve
A g..p||c.| .ep.esert.t|cr c|
t|e .e|.t|crs||p bet.eer
p.cJuct|cr .rJ t|e p.|ce
|eve| |r t|e s|c.t .ur.
change in the aggregate
quantity of goods and
services suppIied
Vcvenert .|crg t|e
s|c.t.ur .gg.eg.te supp|y
cu.ve.
change in short-run
aggregate suppIy
A c|.rge |r t|e .gg.eg.te
,u.rt|ty c| gccJs .rJ
se.v|ces supp||eJ .t eve.y
p.|ce |eve| |r t|e s|c.t .ur.
Short-Run Aggregate SuppIy
The model of aggiegate demand and long-iun aggiegate supply piedicts that the eco-
nomy will eventually move towaid its potential output. To see how nominal wage and
piice stickiness can cause ieal GDP to be eithei above oi below potential in the shoit
iun, considei the iesponse of the economy to a change in aggiegate demand. Figuie
22.7 shows an economy that has been opeiating at potential output of $12,000 billion
and a piice level of 1.14. This occuis at the inteisection of AD
1
with the long-iun ag-
giegate supply cuive at point B. Now suppose that the aggiegate demand cuive shifts to
the iight (to AD
2
). This could occui as a iesult of an inciease in expoits. (The shift
fiom AD
1
to AD
2
includes the multiplied efect of the inciease in expoits.) At the piice
level of 1.14, theie is now excess demand and piessuie on piices to iise. If all piices in
the economy adjusted quickly, the economy would quickly settle at potential output of
$12,000 billion, but at a highei piice level (1.18 in this case).
Is it possible to expand output above potential: Yes. It may be the case, foi ex-
ample, that some people who weie in the laboi foice but weie fiictionally oi stiuctui-
ally unemployed fnd woik because of the ease of getting jobs at the going nominal
wage in such an enviionment. The iesult is an economy opeiating at point A in Figuie
22.7 at a highei piice level and with output tempoiaiily above potential.
Considei next the efect of a ieduction in aggiegate demand (to AD
3
), possibly due
to a ieduction in investment. As the piice level staits to fall, output also falls. The eco-
nomy fnds itself at a piice level-output combination at which ieal GDP is below po-
tential, at point C. Again, piice stickiness is to blame. The piices fims ieceive aie fall-
ing with the ieduction in demand. Without coiiesponding ieductions in nominal
wages, theie will be an inciease in the ieal wage. Fiims will employ less laboi and pio-
duce less output.
By examining what happens as aggiegate demand shifts ovei a peiiod when piice
adjustment is incomplete, we can tiace out the shoit-iun aggiegate supply cuive by
diawing a line thiough points A, B, and C. The short-run aggregate supply (SRAS)
curve is a giaphical iepiesentation of the ielationship between pioduction and the
piice level in the shoit iun. Among the factois held constant in diawing a shoit-iun ag-
giegate supply cuive aie the capital stock, the stock of natuial iesouices, the level of
technology, and the piices of factois of pioduction.
A change in the piice level pioduces a change in the aggregate quantity of goods and ser-
vices supplied and is illustiated by the movement along the shoit-iun aggiegate supply cuive. This
occuis between points A, B, and C in Figuie 22.7.
A change in the quantity of goods and seivices supplied at eveiy piice level in the shoit iun is a
change in short-run aggregate supply. Changes in the factois held constant in diawing the shoit-
iun aggiegate supply cuive shift the cuive. (These factois may also shift the long-iun aggiegate supply
cuive; we will discuss them along with othei deteiminants of long-iun aggiegate supply in the next
chaptei.)
One type of event that would shift the shoit-iun aggiegate supply cuive is an inciease in the piice
of a natuial iesouice such as oil. An inciease in the piice of natuial iesouices oi any othei factoi of
pioduction, all othei things unchanged, iaises the cost of pioduction and leads to a ieduction in shoit-
iun aggiegate supply. In Panel (a) of Figuie 22.8, SRAS
1
shifts leftwaid to SRAS
2
. A deciease in the
piice of a natuial iesouice would lowei the cost of pioduction and, othei things unchanged, would al-
low gieatei pioduction fiom the economy's stock of iesouices and would shift the shoit-iun aggiegate
supply cuive to the iight; such a shift is shown in Panel (b) by a shift fiom SRAS
1
to SRAS
3
.
CHAP7R 22 AGGRGA7 DMAND AND AGGRGA7 SUPPLY 555
II GUR 22. 8 Changes in Short-Run Aggregate SuppIy
A .eJuct|cr |r s|c.t.ur .gg.eg.te supp|y s|||ts t|e cu.ve |.cn .|^.
1
tc .|^.
2
|r |.re| (.). Ar |rc.e.se s|||ts |t tc
t|e .|g|t tc .|^.
3
, .s s|c.r |r |.re| (b).
Reasons for Wage and Price Stickiness
Wage oi piice stickiness means that the economy may not always be opeiating at potential. Rathei, the
economy may opeiate eithei above oi below potential output in the shoit iun. Coiiespondingly, the
oveiall unemployment iate will be below oi above the natuial level.
Many piices obseived thioughout the economy do adjust quickly to changes in maiket conditions
so that equilibiium, once lost, is quickly iegained. Piices foi fiesh food and shaies of common stock aie
two such examples.
Othei piices, though, adjust moie slowly. Nominal wages, the piice of laboi, adjust veiy slowly.
We will fist look at why nominal wages aie sticky, due to theii association with the unemployment
iate, a vaiiable of gieat inteiest in macioeconomics, and then at othei piices that may be sticky.
Wage Stickiness
Wage contiacts fx nominal wages foi the life of the contiact. The length of wage contiacts vaiies fiom
one week oi one month foi tempoiaiy employees, to one yeai (teacheis and piofessois often have such
contiacts), to thiee yeais (foi most union woikeis employed undei majoi collective baigaining agiee-
ments). The existence of such explicit contiacts means that both woikeis and fims accept some wage
at the time of negotiating, even though economic conditions could change while the agieement is still
in foice.
Think about youi own job oi a job you once had. Chances aie you go to woik each day knowing
what youi wage will be. Youi wage does not fuctuate fiom one day to the next with changes in demand
oi supply. You may have a foimal contiact with youi employei that specifes what youi wage will be
ovei some peiiod. Oi you may have an infoimal undeistanding that sets youi wage. Whatevei the
natuie of youi agieement, youi wage is stuck" ovei the peiiod of the agieement. Youi wage is an ex-
ample of a sticky piice.
One ieason woikeis and fims may be willing to accept long-teim nominal wage contiacts is that
negotiating a contiact is a costly piocess. Both paities must keep themselves adequately infoimed about
maiket conditions. Wheie unions aie involved, wage negotiations iaise the possibility of a laboi stiike,
an eventuality that fims may piepaie foi by accumulating additional inventoiies, also a costly piocess.
Even when unions aie not involved, time and eneigy spent discussing wages takes away fiom time and
eneigy spent pioducing goods and seivices. In addition, woikeis may simply piefei knowing that theii
nominal wage will be fxed foi some peiiod of time.
Some contiacts do attempt to take into account changing economic conditions, such as infation,
thiough cost-of-living adjustments, but even these ielatively simple contingencies aie not as wide-
spiead as one might think. One ieason might be that a fim is conceined that while the aggiegate piice
level is iising, the piices foi the goods and seivices it sells might not be moving at the same iate. Also,
cost-of-living oi othei contingencies add complexity to contiacts that both sides may want to avoid.
Even maikets wheie woikeis aie not employed undei explicit contiacts seem to behave as if such
contiacts existed. In these cases, wage stickiness may stem fiom a desiie to avoid the same unceitainty
and adjustment costs that explicit contiacts aveit.
556 PRINCIPLS OI CONOMICS
Finally, minimum wage laws pievent wages fiom falling below a legal minimum, even if unem-
ployment is iising. Unskilled woikeis aie paiticulaily vulneiable to shifts in aggiegate demand.
Price Stickiness
Rigidity of othei piices becomes easiei to explain in light of the aiguments about nominal wage sticki-
ness. Since wages aie a majoi component of the oveiall cost of doing business, wage stickiness may lead
to output piice stickiness. With nominal wages stable, at least some fims can adopt a wait and see" at-
titude befoie adjusting theii piices. Duiing this time, they can evaluate infoimation about why sales aie
iising oi falling (Is the change in demand tempoiaiy oi peimanent:) and tiy to assess likely ieactions
by consumeis oi competing fims in the industiy to any piice changes they might make (Will con-
sumeis be angeied by a piice inciease, foi example: Will competing fims match piice changes:).
In the meantime, fims may piefei to adjust output and employment in iesponse to changing mai-
ket conditions, leaving pioduct piice alone. Quantity adjustments have costs, but fims may assume
that the associated iisks aie smallei than those associated with piice adjustments.
Anothei possible explanation foi piice stickiness is the notion that theie aie adjustment costs asso-
ciated with changing piices. In some cases, fims must piint new piice lists and catalogs, and notify
customeis of piice changes. Doing this too often could jeopaidize customei ielations.
Yet anothei explanation of piice stickiness is that fims may have explicit long-teim contiacts to
sell theii pioducts to othei fims at specifed piices. Foi example, electiic utilities often buy theii inputs
of coal oi oil undei long-teim contiacts.
Taken togethei, these ieasons foi wage and piice stickiness explain why aggiegate piice adjust-
ment may be incomplete in the sense that the change in the piice level is insumcient to maintain ieal
GDP at its potential level. These ieasons do not lead to the conclusion that no piice adjustments occui.
But the adjustments iequiie some time. Duiing this time, the economy may iemain above oi below its
potential level of output.
quiIibrium LeveIs of Price and Output in the Short Run
To illustiate how we will use the model of aggiegate demand and aggiegate supply, let us examine the
impact of two events: an inciease in the cost of health caie and an inciease in goveinment puichases.
The fist ieduces shoit-iun aggiegate supply; the second incieases aggiegate demand. Both events
change equilibiium ieal GDP and the piice level in the shoit iun.
A Change in the Cost of HeaIth Care
In the United States, most people ieceive health insuiance foi themselves and theii families thiough
theii employeis. In fact, it is quite common foi employeis to pay a laige peicentage of employees'
health insuiance piemiums, and this beneft is often wiitten into laboi contiacts. As the cost of health
caie has gone up ovei time, fims have had to pay highei and highei health insuiance piemiums. With
nominal wages fxed in the shoit iun, an inciease in health insuiance piemiums paid by fims iaises the
cost of employing each woikei. It afects the cost of pioduction in the same way that highei wages
would. The iesult of highei health insuiance piemiums is that fims will choose to employ fewei
woikeis.
Suppose the economy is opeiating initially at the shoit-iun equilibiium at the inteisection of AD
1
and SRAS
1
, with a ieal GDP of Y
1
and a piice level of P
1
, as shown in Figuie 22.9. This is the initial
equilibiium piice and output in the shoit iun. The inciease in laboi cost shifts the shoit-iun aggiegate
supply cuive to SRAS
2
. The piice level iises to P
2
and ieal GDP falls to Y
2
.
CHAP7R 22 AGGRGA7 DMAND AND AGGRGA7 SUPPLY 557
II GUR 22. 9 An Increase in HeaIth Insurance Premiums Paid by Iirms
Ar |rc.e.se |r |e.|t| |rsu..rce p.en|uns p.|J by |.ns |rc.e.ses |.bc. ccsts, .eJuc|rg s|c.t.ur .gg.eg.te supp|y
|.cn .|^.
1
tc .|^.
2
. |e p.|ce |eve| .|ses |.cn |
1
tc |
2
.rJ cutput |.||s |.cn `
1
tc `
2
.
A ieduction in health insuiance piemiums would have the opposite efect. Theie would be a shift to the
iight in the shoit-iun aggiegate supply cuive with piessuie on the piice level to fall and ieal GDP to
iise.
A Change in Government Purchases
Suppose the fedeial goveinment incieases its spending foi highway constiuction. This ciicumstance
leads to an inciease in U.S. goveinment puichases and an inciease in aggiegate demand.
Assuming no othei changes afect aggiegate demand, the inciease in goveinment puichases shifts
the aggiegate demand cuive by a multiplied amount of the initial inciease in goveinment puichases to
AD
2
in Figuie 22.10. Real GDP iises fiom Y
1
to Y
2
, while the piice level iises fiom P
1
to P
2
. Notice that
the inciease in ieal GDP is less than it would have been if the piice level had not iisen.
II GUR 22. 10 An Increase in Government Purchases
Ar |rc.e.se |r gcve.rnert pu.c|.ses bccsts .gg.eg.te Jen.rJ |.cn ^|
1
tc ^|
2
. S|c.t.ur e,u|||b.|un |s .t t|e
|rte.sect|cr c| ^|
2
.rJ t|e s|c.t.ur .gg.eg.te supp|y cu.ve .|^.
1
. |e p.|ce |eve| .|ses tc |
2
.rJ .e.| C|| .|ses tc
`
2
.
558 PRINCIPLS OI CONOMICS
In contiast, a ieduction in goveinment puichases would ieduce aggiegate demand. The aggiegate de-
mand cuive shifts to the left, putting piessuie on both the piice level and ieal GDP to fall.
In the shoit iun, ieal GDP and the piice level aie deteimined by the inteisection of the aggiegate
demand and shoit-iun aggiegate supply cuives. Recall, howevei, that the shoit iun is a peiiod in which
sticky piices may pievent the economy fiom ieaching its natuial level of employment and potential
output. In the next section, we will see how the model adjusts to move the economy to long-iun equi-
libiium and what, if anything, can be done to steei the economy towaid the natuial level of employ-
ment and potential output.
k Y 7 A k A W A Y S
< |e s|c.t .ur |r n.c.ceccrcn|cs |s . pe.|cJ |r .||c| ..ges .rJ scne ct|e. p.|ces ..e st|cky. |e |crg
.ur |s . pe.|cJ |r .||c| |u|| ..ge .rJ p.|ce |ex|b|||ty, .rJ n..ket .Justnert, |.s beer .c||eveJ, sc t|.t
t|e eccrcny |s .t t|e r.tu..| |eve| c| enp|cynert .rJ pctert|.| cutput.
< |e |crg.ur .gg.eg.te supp|y cu.ve |s . ve.t|c.| ||re .t t|e pctert|.| |eve| c| cutput. |e |rte.sect|cr c|
t|e eccrcnys .gg.eg.te Jen.rJ .rJ |crg.ur .gg.eg.te supp|y cu.ves Jete.n|res |ts e,u|||b.|un .e.|
C|| .rJ p.|ce |eve| |r t|e |crg .ur.
< |e s|c.t.ur .gg.eg.te supp|y cu.ve |s .r up...Js|cp|rg cu.ve t|.t s|c.s t|e ,u.rt|ty c| tct.| cutput
t|.t .||| be p.cJuceJ .t e.c| p.|ce |eve| |r t|e s|c.t .ur. \.ge .rJ p.|ce st|ck|ress .cccurt |c. t|e s|c.t
.ur .gg.eg.te supp|y cu.ves up...J s|cpe.
< |.rges |r p.|ces c| |.ctc.s c| p.cJuct|cr s|||t t|e s|c.t.ur .gg.eg.te supp|y cu.ve. |r .JJ|t|cr, c|.rges
|r t|e c.p|t.| stcck, t|e stcck c| r.tu..| .escu.ces, .rJ t|e |eve| c| tec|rc|cgy c.r .|sc c.use t|e s|c.t.ur
.gg.eg.te supp|y cu.ve tc s|||t.
< |r t|e s|c.t .ur, t|e e,u|||b.|un p.|ce |eve| .rJ t|e e,u|||b.|un |eve| c| tct.| cutput ..e Jete.n|reJ by t|e
|rte.sect|cr c| t|e .gg.eg.te Jen.rJ .rJ t|e s|c.t.ur .gg.eg.te supp|y cu.ves. |r t|e s|c.t .ur, cutput
c.r be e|t|e. be|c. c. .bcve pctert|.| cutput.
7 R Y I 7 !
|e tcc|s .e |.ve ccve.eJ |r t||s sect|cr c.r be useJ tc urJe.st.rJ t|e C.e.t |ep.ess|cr c| t|e 1930s. \e
krc. t|.t |rvestnert .rJ ccrsunpt|cr beg.r |.|||rg |r |.te 1929. |e .eJuct|crs .e.e .e|r|c.ceJ by
p|urges |r ret expc.ts .rJ gcve.rnert pu.c|.ses cve. t|e rext |cu. ye..s. |r .JJ|t|cr, rcn|r.| ..ges
p|urgeJ 26 bet.eer 1929 .rJ 1933. \e .|sc krc. t|.t .e.| C|| |r 1933 ..s 30 be|c. .e.| C|| |r 1929.
|se t|e tcc|s c| .gg.eg.te Jen.rJ .rJ s|c.t.ur .gg.eg.te supp|y tc g..p| .rJ exp|.|r .|.t |.ppereJ tc
t|e eccrcny bet.eer 1929 .rJ 1933.
Case in Point: 7he U.S. Recession of 2001
\|.t .e.e t|e c.uses c| t|e |.S. .ecess|cr c| 2001` |ccrcn|st |ev|r |||eser c| t|e |eJe..| |ese.ve b.rk c| St.
|cu|s pc|rts tc |cu. |.ctc.s t|.t, t.ker tcget|e., s|||teJ t|e .gg.eg.te Jen.rJ cu.ve tc t|e |e|t .rJ kept |t
t|e.e |c. . |crg ercug| pe.|cJ tc keep .e.| C|| |.|||rg |c. .bcut r|re ncrt|s. |ey .e.e t|e |.|| |r stcck n..
ket p.|ces, t|e Jec.e.se |r bus|ress |rvestnert bct| |c. ccnpute.s .rJ sc|t...e .rJ |r st.uctu.es, t|e Jec||re
|r t|e .e.| v.|ue c| expc.ts, .rJ t|e .|te.n.t| c| 9/11. |ct.b|e except|crs tc t||s ||st c| cu|p.|ts .e.e t|e be|.
v|c. c| ccrsune. sperJ|rg Ju.|rg t|e pe.|cJ .rJ re. .es|Jert|.| |cus|rg, .||c| |.||s |rtc t|e |rvestnert
c.tegc.y.
CHAP7R 22 AGGRGA7 DMAND AND AGGRGA7 SUPPLY 559
|u.|rg t|e exp.rs|cr |r t|e |.te 1990s, . su.g|rg stcck n..ket p.cb.b|y n.Je |t e.s|e. |c. |.ns tc ..|se |urJ
|rg |c. |rvestnert |r bct| st.uctu.es .rJ |r|c.n.t|cr tec|rc|cgy. |ver t|cug| t|e stcck n..ket bubb|e bu.st
.e|| be|c.e t|e .ctu.| .ecess|cr, t|e ccrt|ru.t|cr c| p.cects .|.e.Jy urJe...y Je|.yeJ t|e Jec||re |r t|e |r
vestnert ccnpcrert c| C||. A|sc, sperJ|rg |c. |r|c.n.t|cr tec|rc|cgy ..s p.cb.b|y p.c|crgeJ .s |.ns
Je.|t .|t| +2| ccnput|rg |ssues, t|.t |s, ccnpute. p.cb|ens .sscc|.teJ .|t| t|e c|.rge |r t|e J.te |.cn
1999 tc 2000. Vcst ccnpute.s useJ cr|y t.c J|g|ts tc |rJ|c.te t|e ye.., .rJ .|er t|e ye.. c|.rgeJ |.cn 99
tc 00, ccnpute.s J|J rct krc. |c. tc |rte.p.et t|e c|.rge, .rJ exters|ve .ep.cg..nn|rg c| ccnpute.s ..s
.e,u|.eJ.
|e.| expc.ts |e|| Ju.|rg t|e .ecess|cr bec.use (1) t|e Jc||.. ..s st.crg Ju.|rg t|e pe.|cJ .rJ (2) .e.| C||
g.c.t| |r t|e .est c| t|e .c.|J |e|| .|ncst 5 |.cn 2000 tc 2001.
|er, t|e te..c.|st .tt.cks c| 9/11, .||c| ||te..||y s|ut Jc.r t..rspc.t.t|cr .rJ |r.rc|.| n..kets |c. seve..|
J.ys, n.y |.ve p.c|crgeJ t|ese reg.t|ve terJerc|es ust |crg ercug| tc tu.r .|.t n|g|t ct|e..|se |.ve
beer . n||J Jec||re |rtc ercug| c| . Jc.rtc.r tc ,u.|||y t|e pe.|cJ .s . .ecess|cr.
|u.|rg t||s pe.|cJ t|e ne.su.eJ p.|ce |eve| ..s essert|.||y st.b|e.|t| t|e |np||c|t p.|ce Je|.tc. .|s|rg by
|ess t|.r 1. |us, .|||e t|e .gg.eg.te Jen.rJ cu.ve s|||teJ |e|t .s . .esu|t c| .|| t|e .e.scrs g|ver .bcve,
t|e.e ..s .|sc . |e|t...J s|||t |r t|e s|c.t.ur .gg.eg.te supp|y cu.ve.
5ource |e.| | |||ee. |e 200 |ece|o |o./o || |||ee| oJ /|o| |e.e|oe| /o, |o.e ooeJ ||.' |e |eJeo| |ee.e |o| o| .| |oo|
|e.|e.. .e|eec|oe 2003 233.
A N S W R 7 O 7 R Y I 7 ! P R O 8 L M
A|| ccnpcrerts c| .gg.eg.te Jen.rJ (ccrsunpt|cr, |rvestnert, gcve.rnert pu.c|.ses, .rJ ret expc.ts)
Jec||reJ bet.eer 1929 .rJ 1933. |us t|e .gg.eg.te Jen.rJ cu.ve s|||teJ n..keJ|y tc t|e |e|t, ncv|rg
|.cn ^|
1929
tc ^|
1933
. |e .eJuct|cr |r rcn|r.| ..ges cc..espcrJs tc .r |rc.e.se |r s|c.t.ur .gg.eg.te
supp|y |.cn .|^.
1929
tc .|^.
1933
. S|rce .e.| C|| |r 1933 ..s |ess t|.r .e.| C|| |r 1929, .e krc. t|.t t|e
ncvenert |r t|e .gg.eg.te Jen.rJ cu.ve ..s g.e.te. t|.r t|.t c| t|e s|c.t.ur .gg.eg.te supp|y cu.ve.
560 PRINCIPLS OI CONOMICS
recessionary gap
|e g.p bet.eer t|e |eve| c|
.e.| C|| .rJ pctert|.|
cutput, .|er .e.| C|| |s |ess
t|.r pctert|.|.
3. RECESSIONARY AND INFLATIONARY GAPS AND
LONG-RUN MACROECONOMIC EQUILIBRIUM
L A R N I N G O 8 1 C 7 I V S
1. xpIain and iIIustrate graphicaIIy recessionary and in0ationary gaps and reIate these gaps to
what is happening in the Iabor market.
2. Identify the various poIicy choices avaiIabIe when an economy experiences an in0ationary or
recessionary gap and discuss some of the pros and cons that make these choices controversiaI.
The inteisection of the economy's aggiegate demand and shoit-iun aggiegate supply cuives deteim-
ines equilibiium ieal GDP and piice level in the shoit iun. The inteisection of aggiegate demand and
long-iun aggiegate supply deteimines its long-iun equilibiium. In this section we will examine the
piocess thiough which an economy moves fiom equilibiium in the shoit iun to equilibiium in the long
iun.
The long iun puts a nation's macioeconomic house in oidei: only fiictional and stiuctuial unem-
ployment iemain, and the piice level is stabilized. In the shoit iun, stickiness of nominal wages and
othei piices can pievent the economy fiom achieving its potential output. Actual output may exceed oi
fall shoit of potential output. In such a situation the economy opeiates with a gap. When output is
above potential, employment is above the natuial level of employment. When output is below poten-
tial, employment is below the natuial level.
3.1 Recessionaiy and Inflationaiy Gaps
At any time, ieal GDP and the piice level aie deteimined by the inteisection of the aggiegate demand
and shoit-iun aggiegate supply cuives. If employment is below the natuial level of employment, ieal
GDP will be below potential. The aggiegate demand and shoit-iun aggiegate supply cuives will intei-
sect to the left of the long-iun aggiegate supply cuive.
Suppose an economy's natuial level of employment is L
e
, shown in Panel (a) of Figuie 22.13. This
level of employment is achieved at a ieal wage of a
e
. Suppose, howevei, that the initial ieal wage a
1
ex-
ceeds this equilibiium value. Employment at L
1
falls shoit of the natuial level. A lowei level of employ-
ment pioduces a lowei level of output; the aggiegate demand and shoit-iun aggiegate supply cuives,
AD and SRAS, inteisect to the left of the long-iun aggiegate supply cuive LRAS in Panel (b). The gap
between the level of ieal GDP and potential output, when ieal GDP is less than potential, is called a re-
cessionary gap.
CHAP7R 22 AGGRGA7 DMAND AND AGGRGA7 SUPPLY 561
in0ationary gap
|e g.p bet.eer t|e |eve| c|
.e.| C|| .rJ pctert|.|
cutput, .|er .e.| C|| |s
g.e.te. t|.r pctert|.|.
II GUR 22. 13 A Recessionary Gap
|| enp|cynert |s be|c. t|e r.tu..| |eve|, .s s|c.r |r |.re| (.), t|er cutput nust be be|c. pctert|.|. |.re| (b)
s|c.s t|e .ecess|cr..y g.p `
|
`
1
, .||c| cccu.s .|er t|e .gg.eg.te Jen.rJ cu.ve ^| .rJ t|e s|c.t.ur
.gg.eg.te supp|y cu.ve .|^. |rte.sect tc t|e |e|t c| t|e |crg.ur .gg.eg.te supp|y cu.ve ||^..
Just as employment can fall shoit of its natuial level, it can also exceed it. If employment is gieatei than
its natuial level, ieal GDP will also be gieatei than its potential level. Figuie 22.14 shows an economy
with a natuial level of employment of L
e
in Panel (a) and potential output of Y
P
in Panel (b). If the ieal
wage a
1
is less than the equilibiium ieal wage a
e
, then employment L
1
will exceed the natuial level. As
a iesult, ieal GDP, Y
1
, exceeds potential. The gap between the level of ieal GDP and potential output,
when ieal GDP is gieatei than potential, is called an inationary gap. In Panel (b), the infationaiy
gap equals Y
1
Y
P
.
II GUR 22. 14 An InfIationary Gap
|.re| (.) s|c.s t|.t || enp|cynert |s .bcve t|e r.tu..| |eve|, t|er cutput nust be .bcve pctert|.|. |e
|r|.t|cr..y g.p, s|c.r |r |.re| (b), e,u.|s `
1
`
|
. |e .gg.eg.te Jen.rJ cu.ve ^| .rJ t|e s|c.t.ur .gg.eg.te
supp|y cu.ve .|^. |rte.sect tc t|e .|g|t c| t|e |crg.ur .gg.eg.te supp|y cu.ve ||^..
3.2 Restoiing Long-Run Macioeconomic Equilibiium
We have alieady seen that the aggiegate demand cuive shifts in iesponse to a change in consumption,
investment, goveinment puichases, oi net expoits. The shoit-iun aggiegate supply cuive shifts in
562 PRINCIPLS OI CONOMICS
II GUR 22. 15 Long-Run Ad|ustment
to an InfIationary Gap
Ar |rc.e.se |r .gg.eg.te Jen.rJ tc ^|
2
bccsts .e.| C|| tc `
2
.rJ t|e p.|ce |eve| tc |
2
,
c.e.t|rg .r |r|.t|cr..y g.p c| `
2
`
|
. |r t|e
|crg .ur, .s p.|ce .rJ rcn|r.| ..ges |rc.e.se,
t|e s|c.t.ur .gg.eg.te supp|y cu.ve ncves
tc .|^.
2
. |e.| C|| .etu.rs tc pctert|.|.
iesponse to changes in the piices of factois of pioduction, the quantities of factois of pioduction avail-
able, oi technology. Now we will see how the economy iesponds to a shift in aggiegate demand oi
shoit-iun aggiegate supply using two examples piesented eailiei: a change in goveinment puichases
and a change in health-caie costs. By ietuining to these examples, we will be able to distinguish the
long-iun iesponse fiom the shoit-iun iesponse.
A Shift in Aggregate Demand: An Increase in Government Purchases
Suppose an economy is initially in equilibiium at potential output YP as in Figuie 22.13. Because the
economy is opeiating at its potential, the laboi maiket must be in equilibiium; the quantities of laboi
demanded and supplied aie equal.
Now suppose aggiegate demand incieases because one oi moie of its components
(consumption, investment, goveinment puichases, and net expoits) has incieased at
each piice level. Foi example, suppose goveinment puichases inciease. The aggiegate
demand cuive shifts fiom AD
1
to AD
2
in Figuie 22.13. That will inciease ieal GDP to
Y
2
and foice the piice level up to P
2
in the shoit iun. The highei piice level, combined
with a fxed nominal wage, iesults in a lowei ieal wage. Fiims employ moie woikeis to
supply the incieased output.
The economy's new pioduction level Y
2
exceeds potential output. Employment ex-
ceeds its natuial level. The economy with output of Y
2
and piice level of P
2
is only in
shoit-iun equilibiium; theie is an infationaiy gap equal to the difeience between Y
2
and Y
P
. Because ieal GDP is above potential, theie will be piessuie on piices to iise
fuithei.
Ultimately, the nominal wage will iise as woikeis seek to iestoie theii lost puichas-
ing powei. As the nominal wage iises, the shoit-iun aggiegate supply cuive will begin
shifting to the left. It will continue to shift as long as the nominal wage iises, and the
nominal wage will iise as long as theie is an infationaiy gap. These shifts in shoit-iun
aggiegate supply, howevei, will ieduce ieal GDP and thus begin to close this gap. When
the shoit-iun aggiegate supply cuive ieaches SRAS
2
, the economy will have ietuined to
its potential output, and employment will have ietuined to its natuial level. These ad-
justments will close the infationaiy gap.
A Shift in Short-Run Aggregate SuppIy: An Increase in the Cost of HeaIth
Care
Again suppose, with an aggiegate demand cuive at AD
1
and a shoit-iun aggiegate sup-
ply at SRAS
1
, an economy is initially in equilibiium at its potential output Y
P
, at a piice
level of P
1
, as shown in Figuie 22.16. Now suppose that the shoit-iun aggiegate supply
cuive shifts owing to a iise in the cost of health caie. As we explained eailiei, because
health insuiance piemiums aie paid piimaiily by fims foi theii woikeis, an inciease in piemiums
iaises the cost of pioduction and causes a ieduction in the shoit-iun aggiegate supply cuive fiom
SRAS
1
to SRAS
2
.
CHAP7R 22 AGGRGA7 DMAND AND AGGRGA7 SUPPLY 563
II GUR 22. 16 Long-Run Ad|ustment
to a Recessionary Gap
A Jec.e.se |r .gg.eg.te supp|y |.cn .|^.
1
tc
.|^.
2
.eJuces .e.| C|| tc `
2
.rJ ..|ses t|e
p.|ce |eve| tc |
2
, c.e.t|rg . .ecess|cr..y g.p c|
`
|
`
2
. |r t|e |crg .ur, .s p.|ces .rJ rcn|r.|
..ges Jec.e.se, t|e s|c.t.ur .gg.eg.te
supp|y cu.ve ncves b.ck tc .|^.
1
.rJ .e.|
C|| .etu.rs tc pctert|.|.
nonintervention poIicy
A pc||cy c|c|ce tc t.ke rc
.ct|cr tc t.y tc c|cse .
.ecess|cr..y c. .r |r|.t|cr..y
g.p, but tc .||c. t|e
eccrcny tc .Just cr |ts c.r
tc |ts pctert|.| cutput.
stabiIization poIicy
A pc||cy |r .||c| t|e
gcve.rnert c. cert..| b.rk
.cts tc ncve t|e eccrcny tc
|ts pctert|.| cutput.
As a iesult, the piice level iises to P
2
and ieal GDP falls to Y
2
. The economy now
has a iecessionaiy gap equal to the difeience between Y
P
and Y
2
. Notice that this situ-
ation is paiticulaily disagieeable, because both unemployment and the piice level iose.
With ieal GDP below potential, though, theie will eventually be piessuie on the
piice level to fall. Incieased unemployment also puts piessuie on nominal wages to fall.
In the long iun, the shoit-iun aggiegate supply cuive shifts back to SRAS
1
. In this case,
ieal GDP ietuins to potential at Y
P
, the piice level falls back to P
1
, and employment ie-
tuins to its natuial level. These adjustments will close the iecessionaiy gap.
How sticky piices and nominal wages aie will deteimine the time it takes foi the
economy to ietuin to potential. People often expect the goveinment oi the cential bank
to iespond in some way to tiy to close gaps. This issue is addiessed next.
3.3 Gaps and Public Policy
If the economy faces a gap, how do we get fiom that situation to potential output:
Gaps piesent us with two alteinatives. Fiist, we can do nothing. In the long iun,
ieal wages will adjust to the equilibiium level, employment will move to its natuial
level, and ieal GDP will move to its potential. Second, we can do something. Faced with
a iecessionaiy oi an infationaiy gap, policy makeis can undeitake policies aimed at
shifting the aggiegate demand oi shoit-iun aggiegate supply cuives in a way that
moves the economy to its potential. A policy choice to take no action to tiy to close a
iecessionaiy oi an infationaiy gap, but to allow the economy to adjust on its own to its
potential output, is a nonintervention policy. A policy in which the goveinment oi
cential bank acts to move the economy to its potential output is called a stabilization
policy.
Nonintervention or xpansionary PoIicy1
Figuie 22.17 illustiates the alteinatives foi closing a iecessionaiy gap. In both panels,
the economy staits with a ieal GDP of Y
1
and a piice level of P
1
. Theie is a iecessionaiy gap equal to Y
P
Y
1
. In Panel (a), the economy closes the gap thiough a piocess of self-coiiection. Real and nominal
wages will fall as long as employment iemains below the natuial level. Lowei nominal wages shift the
shoit-iun aggiegate supply cuive. The piocess is a giadual one, howevei, given the stickiness of nomin-
al wages, but aftei a seiies of shifts in the shoit-iun aggiegate supply cuive, the economy moves towaid
equilibiium at a piice level of P
2
and its potential output of Y
P
.
II GUR 22. 17 AIternatives in CIosing a Recessionary Gap
|.re| (.) |||ust..tes . g..Ju.| c|cs|rg c| . .ecess|cr..y g.p. |rJe. . rcr|rte.vert|cr pc||cy, s|c.t.ur .gg.eg.te
supp|y s|||ts |.cn .|^.
1
tc .|^.
2
. |.re| (b) s|c.s t|e e|ects c| exp.rs|cr..y pc||cy .ct|rg cr .gg.eg.te Jen.rJ
tc c|cse t|e g.p.
564 PRINCIPLS OI CONOMICS
expansionary poIicy
A st.b|||..t|cr pc||cy
Jes|greJ tc |rc.e.se .e.|
C||.
contractionary poIicy
A st.b|||..t|cr pc||cy
Jes|greJ tc .eJuce .e.| C||.
scaI poIicy
|e use c| gcve.rnert
pu.c|.ses, t..rs|e. p.ynerts,
.rJ t.xes tc |r|uerce t|e
|eve| c| eccrcn|c .ct|v|ty.
monetary poIicy
|e use c| cert..| b.rk
pc||c|es tc |r|uerce t|e |eve|
c| eccrcn|c .ct|v|ty.
Panel (b) illustiates the stabilization alteinative. Faced with an economy opeiating below its potential,
public omcials act to stimulate aggiegate demand. Foi example, the goveinment can inciease govein-
ment puichases of goods and seivices oi cut taxes. Tax cuts leave people with moie aftei-tax income to
spend, boost theii consumption, and inciease aggiegate demand. As AD
1
shifts to AD
2
in Panel (b) of
Figuie 22.17, the economy achieves output of YP, but at a highei piice level, P
3
. A stabilization policy
designed to inciease ieal GDP is known as an expansionary policy.
Nonintervention or Contractionary PoIicy1
Figuie 22.18 illustiates the alteinatives foi closing an infationaiy gap. Employment in an economy
with an infationaiy gap exceeds its natuial level-the quantity of laboi demanded exceeds the long-iun
supply of laboi. A noninteivention policy would iely on nominal wages to iise in iesponse to the shoit-
age of laboi. As nominal wages iise, the shoit-iun aggiegate supply cuive begins to shift, as shown in
Panel (a), biinging the economy to its potential output when it ieaches SRAS
2
and P
2
.
II GUR 22. 18 AIternatives in CIosing an InfIationary Gap
|.re| (.) |||ust..tes . g..Ju.| c|cs|rg c| .r |r|.t|cr..y g.p. |rJe. . rcr|rte.vert|cr pc||cy, s|c.t.ur .gg.eg.te
supp|y s|||ts |.cn .|^.
1
tc .|^.
2
. |.re| (b) s|c.s t|e e|ects c| ccrt..ct|cr..y pc||cy tc .eJuce .gg.eg.te Jen.rJ
|.cn ^|
1
tc ^|
2
|r c.Je. tc c|cse t|e g.p.
A stabilization policy that ieduces the level of GDP is a contractionary policy. Such a policy would
aim at shifting the aggiegate demand cuive fiom AD
1
to AD
2
to close the gap, as shown in Panel (b). A
policy to shift the aggiegate demand cuive to the left would ietuin ieal GDP to its potential at a piice
level of P
3
.
Foi both kinds of gaps, a combination of letting maiket foices in the economy close pait of the gap
and of using stabilization policy to close the iest of the gap is also an option. Latei chapteis will explain
stabilization policies in moie detail, but theie aie essentially two types of stabilization policy: fscal
policy and monetaiy policy. Fiscal policy is the use of goveinment puichases, tiansfei payments, and
taxes to infuence the level of economic activity. Monetary policy is the use of cential bank policies to
infuence the level of economic activity.
7o Intervene or Not to Intervene: An Introduction to the Controversy
How laige aie infationaiy and iecessionaiy gaps: Panel (a) of Figuie 22.19 shows potential output
veisus the actual level of ieal GDP in the United States since 1960. Real GDP appeais to follow poten-
tial output quite closely, although you see some peiiods wheie theie have been infationaiy oi ieces-
sionaiy gaps. Panel (b) shows the sizes of these gaps expiessed as peicentages of potential output. The
peicentage gap is positive duiing peiiods of infationaiy gaps and negative duiing peiiods of iecession-
aiy gaps. The economy seldom depaits by moie than 3 fiom its potential output.
CHAP7R 22 AGGRGA7 DMAND AND AGGRGA7 SUPPLY 565
II GUR 22. 19 ReaI GDP and PotentiaI Output
|.re| (.) s|c.s pctert|.| cutput (t|e b|ue ||re) .rJ .ctu.| .e.| C|| (t|e pu.p|e ||re) s|rce 1960. |.re| (b) s|c.s t|e
g.p bet.eer pctert|.| .rJ .ctu.| .e.| C|| exp.esseJ .s . pe.cert.ge c| pctert|.| cutput. |r|.t|cr..y g.ps ..e
s|c.r |r g.eer .rJ .ecess|cr..y g.ps ..e s|c.r |r ye||c..
Source Bureau of Economic Analysis, ^IPA Table 1.1.. Real Gross Domestic Product, Chained Dollars jBillions of chained (2000) dollars]. Seasonally
adjusted at annual rates 2008 is through 3rd quarter, Congressional Budget Opce, The Budget and Economic Outlook, September 9, 2008.
Panel (a) gives a long-iun peispective on the economy. It suggests that the economy geneially opeiates
at about potential output. In Panel (a), the gaps seem minoi. Panel (b) gives a shoit-iun peispective;
the view it gives emphasizes the gaps. Both of these peispectives aie impoitant. While it is ieassuiing to
see that the economy is often close to potential, the yeais in which theie aie substantial gaps have ieal
efects: Infation oi unemployment can haim people.
Some economists aigue that stabilization policy can and should be used when iecessionaiy oi in-
fationaiy gaps exist. Otheis uige ieliance on the economy's own ability to coiiect itself. They some-
times aigue that the tools available to the public sectoi to infuence aggiegate demand aie not likely to
shift the cuive, oi they aigue that the tools would shift the cuive in a way that could do moie haim
than good.
Economists who advocate stabilization policies aigue that piices aie sumciently sticky that the eco-
nomy's own adjustment to its potential will be a slow piocess-and a painful one. Foi an economy with
a iecessionaiy gap, unacceptably high levels of unemployment will peisist foi too long a time. Foi an
566 PRINCIPLS OI CONOMICS
economy with an infationaiy gap, the incieased piices that occui as the shoit-iun aggiegate supply
cuive shifts upwaid impose too high an infation iate in the shoit iun. These economists believe it is fai
piefeiable to use stabilization policy to shift the aggiegate demand cuive in an efoit to shoiten the
time the economy is subject to a gap.
Economists who favoi a noninteivention appioach accept the notion that stabilization policy can
shift the aggiegate demand cuive. They aigue, howevei, that such efoits aie not neaily as simple in the
ieal woild as they may appeai on papei. Foi example, policies to change ieal GDP may not afect the
economy foi months oi even yeais. By the time the impact of the stabilization policy occuis, the state
of the economy might have changed. Policy makeis might choose an expansionaiy policy when a con-
tiactionaiy one is needed oi vice veisa. Othei economists who favoi noninteivention also question
how sticky piices ieally aie and if gaps even exist.
The debate ovei how policy makeis should iespond to iecessionaiy and infationaiy gaps is an on-
going one. These issues of noninteivention veisus stabilization policies lie at the heait of the macioeco-
nomic policy debate. We will ietuin to them as we continue oui analysis of the deteimination of output
and the piice level.
k Y 7 A k A W A Y S
< \|er t|e .gg.eg.te Jen.rJ .rJ s|c.t.ur .gg.eg.te supp|y cu.ves |rte.sect be|c. pctert|.| cutput, t|e
eccrcny |.s . .ecess|cr..y g.p. \|er t|ey |rte.sect .bcve pctert|.| cutput, t|e eccrcny |.s .r
|r|.t|cr..y g.p.
< |r|.t|cr..y .rJ .ecess|cr..y g.ps ..e c|cseJ .s t|e .e.| ..ge .etu.rs tc e,u|||b.|un, .|e.e t|e ,u.rt|ty c|
|.bc. Jen.rJeJ e,u.|s t|e ,u.rt|ty supp||eJ. bec.use c| rcn|r.| ..ge .rJ p.|ce st|ck|ress, |c.eve.,
suc| .r .Justnert t.kes t|ne.
< \|er t|e eccrcny |.s . g.p, pc||cy n.ke.s c.r c|ccse tc Jc rct||rg .rJ |et t|e eccrcny .etu.r tc
pctert|.| cutput .rJ t|e r.tu..| |eve| c| enp|cynert cr |ts c.r. A pc||cy tc t.ke rc .ct|cr tc t.y tc c|cse
. g.p |s . rcr|rte.vert|cr pc||cy.
< A|te.r.t|ve|y, pc||cy n.ke.s c.r c|ccse tc t.y tc c|cse . g.p by us|rg st.b|||..t|cr pc||cy. St.b|||..t|cr
pc||cy Jes|greJ tc |rc.e.se .e.| C|| |s c.||eJ exp.rs|cr..y pc||cy. St.b|||..t|cr pc||cy Jes|greJ tc
Jec.e.se .e.| C|| |s c.||eJ ccrt..ct|cr..y pc||cy.
7 R Y I 7 !
|s|rg t|e scer..|c c| t|e C.e.t |ep.ess|cr c| t|e 1930s, .s .r.|y.eJ |r t|e p.ev|cus .y |t!, te|| .|.t k|rJ c|
g.p t|e |.S. eccrcny |.ceJ |r 1933, .ssun|rg t|e eccrcny |.J beer .t pctert|.| cutput |r 1929. |c ycu
t||rk t|e urenp|cynert ..te ..s .bcve c. be|c. t|e r.tu..| ..te c| urenp|cynert` |c. ccu|J t|e ecc
rcny |.ve beer b.cug|t b.ck tc |ts pctert|.| cutput`
Case in Point: Survey of conomists ReveaIs LittIe Consensus on Macroeconomic
PoIicy Issues
Ar eccrcny |r s|c.t.ur e,u|||b.|un .t . .e.| C|| be|c. pctert|.| C|| |.s . se||cc..ect|rg nec|.r|sn t|.t
.||| evertu.||y .etu.r |t tc pctert|.| .e.| C||.
O| eccrcn|sts su.veyeJ, 36 J|s.g.eeJ, 33 .g.eeJ .|t| p.cv|scs, 25 .g.eeJ, .rJ 5 J|J rct .espcrJ. Sc,
cr|y .bcut 60 c| eccrcn|sts .espcrJ|rg tc t|e su.vey .g.eeJ t|.t t|e eccrcny .cu|J .Just cr |ts c.r.
|.rges |r .gg.eg.te Jen.rJ .|ect .e.| C|| |r t|e s|c.t .ur but rct |r t|e |crg .ur.
CHAP7R 22 AGGRGA7 DMAND AND AGGRGA7 SUPPLY 567
Or t||s st.tenert, 36 J|s.g.eeJ, 31 .g.eeJ .|t| p.cv|scs, 29 .g.eeJ, .rJ 4 J|J rct .espcrJ. Orce
.g.|r, .bcut 60 c| eccrcn|sts .ccepteJ t|e ccrc|us|cr c| t|e .gg.eg.te Jen.rJ.gg.eg.te supp|y ncJe|.
||s |eve| c| J|s.g.eenert cr n.c.ceccrcn|c pc||cy |ssues .ncrg eccrcn|sts, b.seJ cr . |.|| 2000 su.vey c|
nenbe.s c| t|e Ane.|c.r |ccrcn|c Asscc|.t|cr, st.rJs |r s|..p ccrt..st tc t|e|. nc.e |..ncr|cus .espcrses
tc ,uest|crs cr |rte.r.t|cr.| eccrcn|cs .rJ n|c.ceccrcn|cs. |c. ex.np|e,
..||s .rJ |npc.t ,uct.s usu.||y .eJuce t|e gere..| .e||..e c| scc|ety.
Severtyt.c pe.cert c| t|cse su.veyeJ .g.eeJ .|t| t||s st.tenert cut.|g|t .rJ .rct|e. 21 .g.eeJ .|t|
p.cv|scs. Sc, 93 c| eccrcn|sts gere..||y .g.eeJ .|t| t|e st.tenert.
V|r|nun ..ges |rc.e.se urenp|cynert .ncrg ycurg .rJ ursk|||eJ .c.ke.s.
Or t||s, 45 .g.eeJ .rJ 29 .g.eeJ .|t| p.cv|scs.
|c||ut|cr t.xes c. n..ket.b|e pc||ut|cr pe.n|ts ..e . nc.e eccrcn|c.||y e|c|ert .pp.c.c| tc pc||ut|cr ccr
t.c| t|.r en|ss|cr st.rJ..Js.
Or t||s erv|.crnert.| ,uest|cr, cr|y 6 J|s.g.eeJ .rJ 63 .|c|e|e..teJ|y .g.eeJ.
|e .e|.t|ve|y |c. Jeg.ee c| ccrsersus cr n.c.ceccrcn|c pc||cy |ssues .rJ t|e ||g|e. Jeg.ees c| ccrsersus
cr ct|e. eccrcn|c |ssues |curJ |r t||s su.vey ccrcu. .|t| .esu|ts c| ct|e. pe.|cJ|c su.veys s|rce 196.
Sc, .s textbcck .ut|c.s, .e .||| rct ||Je t|e J|.ty |.urJ.y |.cn ycu. |c.tur.te|y, t|cug|, t|e ncJe| c| .gg.eg
.te Jen.rJ.gg.eg.te supp|y .e p.esert t|.cug|cut t|e n.c.ceccrcn|c c|.pte.s c.r |.rJ|e ncst c|
t|ese J|s.g.eenerts. |c. ex.np|e, eccrcn|sts .|c .g.ee .|t| t|e |.st p.cpcs|t|cr ,ucteJ .bcve, t|.t .r
eccrcny cpe..t|rg be|c. pctert|.| |.s se||cc..ect|rg nec|.r|sns tc b.|rg |t b.ck tc pctert|.|, ..e p.cb.b|y
.ssun|rg t|.t ..ges .rJ p.|ces ..e rct ve.y st|cky .rJ |erce t|.t t|e s|c.t.ur .gg.eg.te supp|y cu.ve .|||
s|||t ..t|e. e.s||y tc t|e .|g|t, .s s|c.r |r |.re| (.) c| ||gu.e 22.1. |r ccrt..st, eccrcn|sts .|c J|s.g.ee .|t|
t|e st.tenert ..e s.y|rg t|.t t|e ncvenert c| t|e s|c.t.ur .gg.eg.te supp|y cu.ve |s ||ke|y tc be s|c.. ||s
|.tte. g.cup c| eccrcn|sts p.cb.b|y .Jvcc.tes exp.rs|cr..y pc||cy .s s|c.r |r |.re| (b) c| ||gu.e 22.1. bct|
g.cups c| eccrcn|sts c.r use t|e s.ne ncJe| .rJ |ts ccrst.ucts tc .r.|y.e t|e n.c.ceccrcny, but t|ey n.y
J|s.g.ee cr suc| t||rgs .s t|e s|cpes c| t|e v..|cus cu.ves, cr |c. |.st t|ese v..|cus cu.ves s|||t, .rJ cr t|e
s|.e c| t|e urJe.|y|rg nu|t|p||e.. |e ncJe| .||c.s eccrcn|sts tc spe.k t|e s.ne |.rgu.ge c| .r.|ys|s ever
t|cug| t|ey J|s.g.ee cr scne spec||cs.
5ource |o |o||e oJ |o| Oe|Je.|e.eo. oeo o |coo|c |oe ^ .o.e, o| |eo||co. |eoco| oJ |coo||.' |o|e
|coo|c .ooo| 33. o (/||e 200.: S94
568 PRINCIPLS OI CONOMICS
A N S W R 7 O 7 R Y I 7 ! P R O 8 L M
c t|e g..p| |r t|e p.ev|cus .y |t! p.cb|en .e .JJ t|e |crg.ur .gg.eg.te supp|y cu.ve tc s|c. t|.t, .|t|
cutput be|c. pctert|.|, t|e |.S. eccrcny |r 1933 ..s |r . .ecess|cr..y g.p. |e urenp|cynert ..te ..s
.bcve t|e r.tu..| ..te c| urenp|cynert. |rJeeJ, .e.| C|| |r 1933 ..s .bcut 30 be|c. .|.t |t |.J beer |r
1929, .rJ t|e urenp|cynert ..te |.J |rc.e.seJ |.cn 3 tc 25. |cte t|.t Ju.|rg t|e pe.|cJ c| t|e C.e.t
|ep.ess|cr, ..ges J|J |.||. |e rct|cr c| rcn|r.| ..ge .rJ ct|e. p.|ce st|ck|ress J|scusseJ |r t||s sect|cr
s|cu|J rct be ccrst.ueJ tc ne.r ccnp|ete ..ge .rJ p.|ce |r|ex|b|||ty. |.t|e., Ju.|rg t||s pe.|cJ, rcn|r.|
..ges .rJ ct|e. p.|ces .e.e rct |ex|b|e ercug| tc .estc.e t|e eccrcny tc t|e pctert|.| |eve| c| cutput.
|e.e ..e t.c b.s|c c|c|ces cr |c. tc c|cse .ecess|cr..y g.ps. |cr|rte.vert|cr .cu|J ne.r ..|t|rg |c.
..ges tc |.|| |u.t|e.. As ..ges |.||, t|e s|c.t.ur .gg.eg.te supp|y cu.ve .cu|J ccrt|rue tc s|||t tc t|e .|g|t.
|e .|te.r.t|ve .cu|J be tc use scne type c| exp.rs|cr..y pc||cy. ||s .cu|J s|||t t|e .gg.eg.te Jen.rJ
cu.ve tc t|e .|g|t. |ese t.c cpt|crs .e.e |||ust..teJ |r ||gu.e 22.18.
4. REVIEW AND PRACTICE
Summary
|r t||s c|.pte., .e cut||reJ t|e ncJe| c| .gg.eg.te Jen.rJ .rJ .gg.eg.te supp|y. \e s.. t|.t t|e .gg.eg
.te Jen.rJ cu.ve s|cpes Jc.r...J, .e|ect|rg t|e terJercy |c. t|e .gg.eg.te ,u.rt|ty c| gccJs .rJ se.v|ces
Jen.rJeJ tc .|se .s t|e p.|ce |eve| |.||s .rJ tc |.|| .s t|e p.|ce |eve| .|ses. |e reg.t|ve .e|.t|crs||p bet.eer
t|e p.|ce |eve| .rJ t|e ,u.rt|ty c| gccJs .rJ se.v|ces Jen.rJeJ .esu|ts |.cn t|e .e.|t| e|ect |c. ccrsunp
t|cr, t|e |rte.est ..te e|ect |c. |rvestnert, .rJ t|e |rte.r.t|cr.| t..Je e|ect |c. ret expc.ts. \e ex.n|reJ t|e
|.ctc.s t|.t c.r s|||t t|e .gg.eg.te Jen.rJ cu.ve .s .e||. Cere..||y, t|e .gg.eg.te Jen.rJ cu.ve s|||ts by .
nu|t|p|e c| t|e |r|t|.| .ncurt by .||c| t|e ccnpcrert c.us|rg |t tc s|||t c|.rges.
\e J|st|rgu|s|eJ bet.eer t.c types c| e,u|||b.|. |r n.c.ceccrcn|cscre cc..espcrJ|rg tc t|e s|c.t .ur, .
pe.|cJ c| .r.|ys|s |r .||c| rcn|r.| ..ges .rJ scne p.|ces ..e st|cky, .rJ t|e ct|e. cc..espcrJ|rg tc t|e
|crg .ur, . pe.|cJ |r .||c| |u|| ..ge .rJ p.|ce |ex|b|||ty, .rJ |erce n..ket .Justnert, |.ve beer .c||eveJ.
|crg.ur e,u|||b.|un cccu.s .t t|e |rte.sect|cr c| t|e .gg.eg.te Jen.rJ cu.ve .|t| t|e |crg.ur .gg.eg.te
supp|y cu.ve. |e |crg.ur .gg.eg.te supp|y cu.ve |s . ve.t|c.| ||re .t t|e eccrcnys pctert|.| |eve| c| cutput.
S|c.t.ur e,u|||b.|un cccu.s .t t|e |rte.sect|cr c| t|e .gg.eg.te Jen.rJ cu.ve .|t| t|e s|c.t.ur .gg.eg.te
supp|y cu.ve. |e s|c.t.ur .gg.eg.te supp|y cu.ve .e|.tes t|e ,u.rt|ty c| tct.| cutput p.cJuceJ tc t|e p.|ce
|eve| |r t|e s|c.t .ur. |t |s up...J s|cp|rg bec.use c| ..ge .rJ p.|ce st|ck|ress. |r s|c.t.ur e,u|||b.|un, cut
put c.r be be|c. c. .bcve pctert|.|.
|| .r eccrcny |s |r|t|.||y cpe..t|rg .t |ts pctert|.| cutput, t|er . c|.rge |r .gg.eg.te Jen.rJ c. s|c.t.ur
.gg.eg.te supp|y .||| |rJuce . .ecess|cr..y c. |r|.t|cr..y g.p. Suc| . g.p .||| be c|cseJ |r t|e |crg .ur by
c|.rges |r t|e rcn|r.| ..ge, .||c| .||| s|||t t|e s|c.t.ur .gg.eg.te supp|y cu.ve tc t|e |e|t (tc c|cse .r |r
|.t|cr..y g.p) c. tc t|e .|g|t (tc c|cse . .ecess|cr..y g.p). |c||cy n.ke.s n|g|t .espcrJ tc . .ecess|cr..y c.
|r|.t|cr..y g.p .|t| . rcr|rte.vert|cr pc||cy, c. t|ey ccu|J use st.b|||..t|cr pc||cy.
CHAP7R 22 AGGRGA7 DMAND AND AGGRGA7 SUPPLY 569
C O N C P 7 P R O 8 L M S
1. |xp|.|r |c. t|e |c||c.|rg c|.rges |r .gg.eg.te Jen.rJ c. s|c.t.ur .gg.eg.te supp|y, ct|e. t||rgs |e|J
urc|.rgeJ, ..e ||ke|y tc .|ect t|e |eve| c| tct.| cutput .rJ t|e p.|ce |eve| |r t|e s|c.t .ur.
.. Ar |rc.e.se |r .gg.eg.te Jen.rJ
b. A Jec.e.se |r .gg.eg.te Jen.rJ
c. Ar |rc.e.se |r s|c.t.ur .gg.eg.te supp|y
J. A .eJuct|cr |r s|c.t.ur .gg.eg.te supp|y
2. |xp|.|r .|y . c|.rge |r cre ccnpcrert c| .gg.eg.te Jen.rJ .||| c.use t|e .gg.eg.te Jen.rJ cu.ve tc
s|||t by . nu|t|p|e c| t|e |r|t|.| c|.rge.
3. |se t|e ncJe| c| .gg.eg.te Jen.rJ .rJ s|c.t.ur .gg.eg.te supp|y tc exp|.|r |c. e.c| c| t|e
|c||c.|rg .cu|J .|ect .e.| C|| .rJ t|e p.|ce |eve| |r t|e s|c.t .ur.
.. Ar |rc.e.se |r gcve.rnert pu.c|.ses
b. A .eJuct|cr |r rcn|r.| ..ges
c. A n.c. |np.cvenert |r tec|rc|cgy
J. A .eJuct|cr |r ret expc.ts
4. |c. .cu|J .r |rc.e.se |r t|e supp|y c| |.bc. .|ect t|e r.tu..| |eve| c| enp|cynert .rJ pctert|.|
cutput` |c. .cu|J |t .|ect t|e .e.| ..ge, t|e |eve| c| .e.| C||, .rJ t|e p.|ce |eve| |r t|e s|c.t .ur` |c.
.cu|J |t .|ect |crg.ur .gg.eg.te supp|y` \|.t k|rJ c| g.ps .cu|J be c.e.teJ`
5. C|ve t|.ee .e.scrs |c. t|e Jc.r...J s|cpe c| t|e .gg.eg.te Jen.rJ cu.ve.
6. \|er t|e p.|ce |eve| |.||s, pecp|es .e.|t| |rc.e.ses. \|er .e.|t| |rc.e.ses, t|e .e.| vc|une c|
ccrsunpt|cr |rc.e.ses. |e.e|c.e, . Jec.e.se |r t|e p.|ce |eve| .||| c.use t|e .gg.eg.te Jen.rJ cu.ve tc
s|||t tc t|e .|g|t. |c ycu .g.ee` |xp|.|r.
. Suppcse t|e eccrcny |.s . .ecess|cr..y g.p. \e krc. t|.t || .e Jc rct||rg, t|e eccrcny .||| c|cse t|e
g.p cr |ts c.r. A|te.r.t|ve|y, .e ccu|J ....rge |c. .r |rc.e.se |r .gg.eg.te Jen.rJ (s.y, by |rc.e.s|rg
gcve.rnert sperJ|rg) tc c|cse t|e g.p. |c. .cu|J ycu. v|e.s .bcut t|e Jeg.ee c| p.|ce st|ck|ress |r t|e
eccrcny |r|uerce ycu. v|e.s cr .|et|e. suc| . pc||cy .cu|J be Jes|..b|e`
8. |e ccst c| ||.|rg .c.ke.s |rc|uJes rct cr|y p.ynerts n.Je J|.ect|y tc .c.ke.s, t|.t |s, ..ges, but
p.ynerts n.Je cr be|.|| c| .c.ke.s .s .e||, suc| .s ccrt.|but|crs by enp|cye.s tc pers|cr p|.rs .rJ tc
|e.|t|c..e |rsu..rce |c. enp|cyees. |c. .cu|J . Jec.e.se |r t|e ccst c| enp|cye.p.cv|JeJ |e.|t|
|rsu..rce .|ect t|e eccrcny` |s|rg ||gu.e 22.9 .s . gu|Je, J... . g..p| tc |||ust..te ycu. .rs.e..
9. Suppcse rcn|r.| ..ges reve. c|.rgeJ. \|.t .cu|J be t|e s|gr||c.rce c| suc| . c|...cte.|st|c`
10. Suppcse t|e n|r|nun ..ge .e.e |rc.e.seJ s|..p|y. |c. .cu|J t||s .|ect t|e e,u|||b.|un p.|ce |eve|
.rJ cutput |eve| |r t|e ncJe| c| .gg.eg.te Jen.rJ .rJ .gg.eg.te supp|y |r t|e s|c.t .ur` |r t|e |crg
.ur`
11. |xp|.|r t|e s|c.t.ur |np.ct c| e.c| c| t|e |c||c.|rg.
.. A J|sccve.y t|.t n.kes cc|J |us|cr . .e.||ty, g.e.t|y .eJuc|rg t|e ccst c| p.cJuc|rg ere.gy
b. Ar |rc.e.se |r t|e p.y.c|| t.x
570 PRINCIPLS OI CONOMICS
N U M R I C A L P R O 8 L M S
1. Suppcse t|e .gg.eg.te Jen.rJ .rJ s|c.t.ur .gg.eg.te supp|y sc|eJu|es |c. .r eccrcny .|cse
pctert|.| cutput e,u.|s 2,00 ..e g|ver by t|e t.b|e.
Aggregate Quantity of Goods and Services
Prlce Leve/ emonJeJ 5u/leJ
0.50 3,500 1,000
0.5 3,000 2,000
1.00 2,500 2,500
1.25 2,000 2,00
1.50 1,500 2,800
.. |... t|e .gg.eg.te Jen.rJ, s|c.t.ur .gg.eg.te supp|y, .rJ |crg.ur .gg.eg.te supp|y cu.ves.
b. St.te t|e s|c.t.ur e,u|||b.|un |eve| c| .e.| C|| .rJ t|e p.|ce |eve|.
c. |...cte.|.e t|e cu..ert eccrcn|c s|tu.t|cr. |s t|e.e .r |r|.t|cr..y c. . .ecess|cr..y g.p` || sc,
|c. |..ge |s |t`
J. |c. suppcse .gg.eg.te Jen.rJ |rc.e.ses by 00 .t e.c| p.|ce |eve|, |c. ex.np|e, t|e
.gg.eg.te ,u.rt|ty c| gccJs .rJ se.v|ces Jen.rJeJ .t . p.|ce |eve| c| 0.50 rc. e,u.|s 4,200.
S|c. t|e re. .gg.eg.te Jen.rJ cu.ve, st.te t|e re. s|c.t.ur e,u|||b.|un p.|ce |eve| .rJ .e.|
C||, .rJ st.te .|et|e. t|e.e |s .r |r|.t|cr..y c. . .ecess|cr..y g.p .rJ g|ve |ts s|.e.
2. Ar eccrcny |s c|...cte.|.eJ by t|e v.|ues |r t|e t.b|e |c. .gg.eg.te Jen.rJ .rJ s|c.t.ur .gg.eg.te
supp|y. |ts pctert|.| cutput |s 1,500.
Aggregate Quantity of Goods and Services
Prlce Leve/ emonJeJ 5u/leJ
0.50 2,500 1,500
0.5 2,000 2,000
1.00 1,500 2,300
1.25 1,000 2,500
1.50 500 2,600
.. |... t|e .gg.eg.te Jen.rJ, s|c.t.ur .gg.eg.te supp|y, .rJ |crg.ur .gg.eg.te supp|y cu.ves.
b. St.te t|e e,u|||b.|un |eve| c| .e.| C|| .rJ t|e p.|ce |eve|.
c. |...cte.|.e t|e cu..ert eccrcn|c s|tu.t|cr. |s t|e.e .r |r|.t|cr..y c. . .ecess|cr..y g.p` || sc,
|c. |..ge |s |t`
J. |c. suppcse t|.t rcn|r.| ..ges .|se .rJ t|.t t|e p.|ce |eve| .e,u|.eJ tc |rJuce . p..t|cu|..
|eve| c| tct.| cutput .|ses by 0.50. |c. ex.np|e, . p.|ce |eve| c| 1.00 |s rc. .e,u|.eJ tc |rJuce
p.cJuce.s tc p.cJuce . .e.| C|| c| 1,500. S|c. t|e re. s|c.t.ur .gg.eg.te supp|y cu.ve,
st.te t|e re. e,u|||b.|un p.|ce |eve| .rJ .e.| C||, .rJ st.te .|et|e. t|e.e |s .r |r|.t|cr..y c. .
.ecess|cr..y g.p .rJ g|ve |ts s|.e. \|y n|g|t suc| . c|.rge cccu.`
3. Suppcse t|e p.|ce |eve| |r . p..t|cu|.. eccrcny e,u.|s 1.3 .rJ t|.t t|e ,u.rt|ty c| .e.| C|| Jen.rJeJ .t
t|.t p.|ce |eve| |s 1,200. Ar |rc.e.se c| 0.1 pc|rt |r t|e p.|ce |eve| .eJuces t|e ,u.rt|ty c| .e.| C||
Jen.rJeJ by 220, .rJ . .eJuct|cr c| 0.1 pc|rt .cu|J p.cJuce .r |rc.e.se |r t|e ,u.rt|ty c| .e.| C||
Jen.rJeJ c| 220. |... t|e .gg.eg.te Jen.rJ cu.ve .rJ s|c. t|e p.|ce |eve| .rJ ,u.rt|ty c| .e.| C||
Jen.rJeJ .t t|.ee pc|rts.
4. Suppcse .r eccrcny |s Jesc.|beJ by t|e |c||c.|rg .gg.eg.te Jen.rJ .rJ s|c.t.ur .gg.eg.te supp|y
cu.ves. |e pctert|.| |eve| c| cutput |s 10 t.||||cr.
CHAP7R 22 AGGRGA7 DMAND AND AGGRGA7 SUPPLY 571
Aggregate Quantity of Goods and Services
Prlce Leve/ emonJeJ 5u/leJ
3.0 11.0 t.||||cr 9.0 t.||||cr
3.4 10.8 t.||||cr 9.2 t.||||cr
3.8 10.6 t.||||cr 9.4 t.||||cr
4.2 10.4 t.||||cr 9.6 t.||||cr
4.6 10.2 t.||||cr 9.8 t.||||cr
5.0 10.0 t.||||cr 10.0 t.||||cr
5.4 9.8 t.||||cr 10.2 t.||||cr
5.8 9.6 t.||||cr 10.4 t.||||cr
6.2 9.4 t.||||cr 10.6 t.||||cr
6.6 9.2 t.||||cr 10.8 t.||||cr
.0 9.0 t.||||cr 11.0 t.||||cr
.. |... t|e .gg.eg.te Jen.rJ .rJ s|c.t.ur .gg.eg.te supp|y cu.ves.
b. \|.t |s t|e |r|t|.| .e.| C||`
c. \|.t |s t|e |r|t|.| p.|ce |eve|`
J. \|.t k|rJ c| g.p, || .ry, ex|sts`
e. A|te. t|e |rc.e.se |r |e.|t|c..e ccsts, e.c| |eve| c| .e.| C|| .e,u|.es .r |rc.e.se |r t|e p.|ce
|eve| c| 0.8. |c. ex.np|e, p.cJuc|rg 9.0 t.||||cr .c.t| c| gccJs .rJ se.v|ces rc. .e,u|.es . p.|ce
|eve| c| 3.8. \|.t |s t|e s|c.t.ur e,u|||b.|un |eve| c| .e.| C||`
|. A|te. t|e |e.|t|c..e ccst |rc.e.se, .|.t |s t|e re. e,u|||b.|un p.|ce |eve| |r t|e s|c.t .ur`
g. \|.t sc.t c| g.p, || .ry, rc. ex|sts`
5. Accc.J|rg tc A|.sk.r st.te eccrcn|st V..k |J...Js, t|e nu|t|p||e. e|ect c| A|.sk.s t..Je .|t| '.p.r |s
suc| t|.t |c. eve.y 1 b||||cr expc.teJ |.cn A|.sk. tc '.p.r .rct|e. 600 n||||cr |s .JJeJ tc t|e st.tes
eccrcny.
1
.|cu|.te t|e s|.e c| t|e expc.t nu|t|p||e..
6. |e |ctt|rg|.ns||.e |ese..c| Obse.v.tc.y |r |rg|.rJ c.|cu|.teJ t|.t stuJerts .|c .tterJ |ctt|rg|.n
ec|r|c.| |r|ve.s|ty sperJ .bcut :2,60 e.c| |r t|e |cc.| eccrcny |c. . tct.| c| :50.45 n||||cr. |r tct.|,
t|e |np.ct c| t|e|. sperJ|rg cr t|e |cc.| eccrcny |s :63 n||||cr.
2
.|cu|.te t|e s|.e c| t|e stuJert
sperJ|rg nu|t|p||e..
. |r Cc., |rJ|., t|e nu|t|p||e. e|ect c| |.cr c.e expc.ts |s c.|cu|.teJ tc be 1.62.
3
.|cu|.te t|e |np.ct c| .r
.JJ|t|cr.| 1,000 .upees c| |.cr c.e expc.ts cr t|e eccrcny c| Cc..
572 PRINCIPLS OI CONOMICS
1.
2.
3.
ENDNOTES
V.tt \c|., ..Je O|c|.|s |cpe|u| |c. '.p.rese |eccve.y, ^oc|o|eJ |e oJ |oco|
/|e, 'ure 22, 2004, b cyc|e.
|r|ve.s|ty b.|rgs |r :250n tc |ccrcny, |o||||o |.e| |o|, |cvenbe. 4,
2004, p. 3.
\|Jyut |un.. ., |.cr O.e V|r|rg C|ves |npetus tc Cc.s |ccrcny, |e o| |J|o,
Ap.|| 30, 2003.
CHAP7R 22 AGGRGA7 DMAND AND AGGRGA7 SUPPLY 573
574 PRINCIPLS OI CONOMICS
| A | | | 2 3
Economic Growth
S7AR7 UP: HOW IMPOR7AN7 IS CONOMIC
GROW7H1
|c. |npc.t.rt |s eccrcn|c g.c.t|` |e best ..y tc .rs.e. t|.t ,uest|cr |s tc |n.g|re |||e .|t|cut g.c.t|tc
|n.g|re t|.t .e J|J rct |.ve t|e g.|rs g.c.t| b.|rgs.
|c. st..te.s, J|v|Je ycu. |.n||ys cu..ert |rccne by s|x .rJ |n.g|re .|.t ycu. |||e .cu|J be ||ke. ||rk .bcut
t|e k|rJ c| |cus|rg ycu. |.n||y ccu|J .|c.J, t|e s|.e c| ycu. erte.t.|rnert buJget, .|et|e. ycu ccu|J st||| .tterJ
sc|cc|. |.t .||| g|ve ycu .r |Je. c| |||e . certu.y .gc |r t|e |r|teJ St.tes, .|er .ve..ge |cuse|c|J |rccnes, .J
usteJ |c. |r|.t|cr, .e.e .bcut cres|xt| .|.t t|ey ..e tcJ.y. |ecp|e |.J |.. sn.||e. |cnes, t|ey ...e|y |.J e|ect.|
c|ty |r t|e|. |cnes, .rJ cr|y . t|ry pe.cert.ge c| t|e pcpu|.t|cr ccu|J ever ccrs|Je. . cc||ege eJuc.t|cr.
c get . nc.e .ecert pe.spect|ve, ccrs|Je. |c. g.c.t| |.s c|.rgeJ ||v|rg st.rJ..Js cve. t|e p.st |.||cer
tu.y c. sc. |r 1950, t|e |r|teJ St.tes ..s t|e .c.|Js .|c|est r.t|cr. but || |cuse|c|Js .e.e .|c| t|er, subse,uert
eccrcn|c g.c.t| |.s n.Je t|en |.. .|c|e.. Ave..ge pe. c.p|t. .e.| J|spcs.b|e pe.scr.| |rccne |.s t.|p|eJ s|rce
t|er. |rJeeJ, t|e .ve..ge |cuse|c|J |rccne |r 1950, .||c| nust |.ve seeneJ |c|ty t|er, ..s be|c. .|.t .e
rc. Je|re .s t|e pcve.ty ||re |c. . |cuse|c|J c| |cu., ever .|te. .Just|rg |c. |r|.t|cr. |ccrcn|c g.c.t| Ju.|rg
t|e |.st |.||certu.y |.s J..n.t|c.||y bccsteJ cu. st.rJ..J c| ||v|rg.rJ cu. st.rJ..J c| .|.t |t t.kes tc get by.
Ore g.uge c| .|s|rg ||v|rg st.rJ..Js |s |cus|rg. A |.||certu.y .gc, ncst |.n|||es J|J rct c.r |cnes. cJ.y,
.bcut t.ct||.Js Jc. |cse |cnes |.ve gctter . |ct b|gge.. re. |cnes bu||t tcJ.y ..e nc.e t|.r t.|ce t|e s|.e c|
re. |cnes bu||t 50 ye..s .gc. Scne |cuse|c|J .pp||.rces, suc| .s te|ep|cres c. ..s||rg n.c||res, t|.t .e rc.
ccrs|Je. b.s|c, .e.e |uxu.|es . |.||certu.y .gc. |r 1950, |ess t|.r t.ct||.Js c| |cus|rg ur|ts |.J ccnp|ete p|unb
|rg |.c|||t|es. cJ.y, cve. 99 Jc.
|ccrcn|c g.c.t| |.s b.cug|t g.|rs |r ct|e. ..e.s .s .e||. |c. cre t||rg, .e ..e .b|e tc .|c.J nc.e sc|cc||rg.
|r 1950, t|e neJ|.r runbe. c| ye..s c| sc|cc| ccnp|eteJ by .Ju|ts .ge 25 c. cve. ..s 6.8. cJ.y, .bcut 85 |.ve
ccnp|eteJ 12 ye..s c| sc|cc||rg .rJ .bcut 28 |.ve ccnp|eteJ |cu. ye..s c| cc||ege. \e .|sc ||ve |crge.. A b.by
bc.r |r 1950 |.J . |||e expect.rcy c| 68 ye..s. A b.by bc.r |r 2004 |.J .r expecteJ |||e c| re..|y 10 ye..s |crge..
O| ccu.se, .|||e eccrcn|c g.c.t| c.r |np.cve cu. n.te.|.| .e||be|rg, |t |s rc p.r.ce. |c. .|| t|e |||s c| scc|
ety. Ane.|c.rs tcJ.y .c..y .bcut t|e |eve| c| v|c|erce |r scc|ety, erv|.crnert.| Jeg..J.t|cr, .rJ .|.t seens tc
be . |css c| b.s|c v.|ues. but .|||e |t |s e.sy tc be J|sn.yeJ .bcut n.ry c|.||erges c| ncJe.r |||e, .e c.r su.e|y be
g..te|u| |c. cu. n.te.|.| .e.|t|. Ou. .|uerce g|ves us t|e cppc.tur|ty tc g..pp|e .|t| scne c| cu. ncst J||cu|t
p.cb|ens .rJ tc ercy . ..rge c| c|c|ces t|.t pecp|e cr|y . |e. Jec.Jes .gc ccu|J rct |.ve |n.g|reJ.
\e |e..reJ . g.e.t Je.| .bcut eccrcn|c g.c.t| |r t|e ccrtext c| t|e p.cJuct|cr pcss|b|||t|es cu.ve. Ou. pu.
pcse |r t||s c|.pte. |s tc .e|.te t|e ccrcept c| eccrcn|c g.c.t| tc t|e ncJe| c| .gg.eg.te Jen.rJ .rJ .gg.eg
.te supp|y t|.t .e Jeve|cpeJ |r t|e p.ev|cus c|.pte. .rJ .||| use t|.cug|cut cu. exp|c..t|cr c| n.c.ceccrcn|cs.
\e .||| .ev|e. t|e |c.ces t|.t Jete.n|re . r.t|crs eccrcn|c g.c.t| ..te .rJ ex.n|re t|e p.cspects |c. g.c.t| |r
t|e |utu.e. \e beg|r by |cck|rg .t t|e s|gr||c.rce c| g.c.t| tc t|e cve..|| .e||be|rg c| scc|ety.
1. THE SIGNIFICANCE OF ECONOMIC GROWTH
L A R N I N G O 8 1 C 7 I V S
1. Dene economic growth and expIain it using the production possibiIities modeI and the
concept of potentiaI output.
2. State the ruIe of 72 and use it to show how even smaII dierences in growth rates can have ma-
|or eects on a country's potentiaI output over time.
3. CaIcuIate the percentage rate of growth of output per capita.
To demonstiate the impact of economic giowth on living standaids of a nation, we must stait with a
cleai defnition of economic giowth and then study its impact ovei time. We will also see how popula-
tion giowth afects the ielationship between economic giowth and the standaid of living an economy is
able to achieve.
1.1 Defining Economic Giowth
Economic giowth is a long-iun piocess that occuis as an economy's potential output incieases.
Changes in ieal GDP fiom quaitei to quaitei oi even fiom yeai to yeai aie shoit-iun fuctuations that
occui as aggiegate demand and shoit-iun aggiegate supply change. Regaidless of media iepoits stating
that the economy giew at a ceitain iate in the last quaitei oi that it is expected to giow at a paiticulai
iate duiing the next yeai, shoit-iun changes in ieal GDP say little about economic giowth. In the long
iun, economic activity moves towaid its level of potential output. Incieases in potential constitute eco-
nomic giowth.
Eailiei we defned economic giowth as the piocess thiough which an economy achieves an out-
waid shift in its pioduction possibilities cuive. How does a shift in the pioduction possibilities cuive
ielate to a change in potential output: To pioduce its potential level of output, an economy must opei-
ate on its pioduction possibilities cuive. An inciease in potential output thus implies an outwaid shift
in the pioduction possibilities cuive. In the fiamewoik of the macioeconomic model of aggiegate de-
mand and aggiegate supply, we show economic giowth as a shift to the iight in the long-iun aggiegate
supply cuive.
Theie aie thiee key points about economic giowth to keep in mind:
1. Giowth is a piocess. It is not a single event; iathei, it is an unfolding seiies of events.
2. We defne giowth in teims of the economy's ability to pioduce goods and seivices, as indicated
by its level of potential output.
3. Giowth suggests that the economy's ability to pioduce goods and seivices is iising. A discussion
of economic giowth is thus a discussion of the seiies of events that inciease the economy's ability
to pioduce goods and seivices.
Figuie 23.1 shows the iecoid of economic giowth foi the U.S. economy ovei the past centuiy. The
giaph shows annual levels of actual ieal GDP and of potential output. We see that the economy has ex-
peiienced diamatic giowth ovei the past centuiy; potential output has soaied moie than 30-fold. The
fguie also ieminds us of a cential theme of oui analysis of macioeconomics: ieal GDP fuctuates about
potential output. Real GDP sagged well below its potential duiing the Gieat Depiession of the 1930s
and iose well above its potential as the nation mobilized its iesouices to fght Woild Wai II. With the
exception of these two peiiods, ieal GDP has iemained close to the economy's potential output. Since
1930, the actual level of ieal GDP has deviated fiom potential output by an aveiage of less than 2.
576 PRINCIPLS OI CONOMICS
II GUR 23. 1 A Century of conomic Growth
At t|e st..t c| t|e 21
st
certu.y, t|e |eve| c| pctert|.| cutput .e.c|eJ . |eve| re..|y 30 t|nes |ts |eve| . certu.y e..||e..
Ove. t|e ye..s, .ctu.| .e.| C|| |uctu.teJ .bcut . .|s|rg |eve| c| pctert|.| cutput.
Source 1900-1949 data from Robert Gordon, Macroeconomics, th ed. (^ew York HarperCollins, 1993), Table A-1, pp. A1-A3, data for 1950-2008
from Congressional Budget Opce, The Budget and Economic Outlook, September 2008.
We uige you to take some time with Figuie 23.1. Ovei the couise of the last centuiy, it is economic
giowth that has taken centei stage. Ceitainly, the fuctuations about potential output have been impoit-
ant. The iecessionaiy gaps-peiiods when ieal GDP slipped below its potential-weie often wienching
expeiiences in which millions of people enduied gieat haidship. The infationaiy gaps-peiiods when
ieal GDP iose above its potential level-often pioduced diamatic incieases in piice levels. Those fuc-
tuations matteied. It was the unemployment and/oi the infation that came with them that made head-
lines. But it was the quiet piocess of economic giowth that pushed living standaids evei highei. We
must undeistand giowth if we aie to undeistand how we got wheie we aie, and wheie we aie likely to
be going duiing the 21st centuiy.
Figuie 23.2 tells us why we use changes in potential output, iathei than actual ieal GDP, as oui
measuie of economic giowth. Actual values of ieal GDP aie afected not just by changes in the poten-
tial level of output, but also by the cyclical fuctuations about that level of output.
Given oui defnition of economic giowth, we would say that the hypothetical economy depicted in
Figuie 23.2 giew at a 2.3 annual iate thioughout the peiiod. If we used actual values of ieal GDP,
howevei, we would obtain quite difeient inteipietations. Considei, foi example, the fist decade of this
peiiod: it began with a ieal GDP of $900 billion and a iecessionaiy gap, and it ended in yeai 10 with a
CHAP7R 23 CONOMIC GROW7H 577
II GUR 23. 2 CycIicaI Change Versus
Growth
|e use c| .ctu.| v.|ues c| .e.| C|| tc
ne.su.e g.c.t| c.r g|ve n|s|e.J|rg .esu|ts.
|e.e, .r eccrcnys pctert|.| cutput (s|c.r |r
g.eer) g.c.s .t . ste.Jy ..te c| 2.5 pe. ye..,
.|t| .ctu.| v.|ues c| .e.| C|| |uctu.t|rg
.bcut t|.t t.erJ. || .e ne.su.e g.c.t| |r t|e
|.st 10 ye..s .s t|e .rru.| ..te c| c|.rge
bet.eer beg|rr|rg .rJ erJ|rg v.|ues c| .e.|
C||, .e get . g.c.t| ..te c| 3.5. |e ..te |c.
t|e seccrJ Jec.Je |s 0.5. C.c.t| est|n.tes
b.seJ cr c|.rges |r .e.| C|| ..e .|ecteJ by
cyc||c.| c|.rges t|.t Jc rct .ep.esert
eccrcn|c g.c.t|.
exponentiaI growth
\|er . ,u.rt|ty g.c.s .t .
g|ver pe.cert.ge ..te.
ruIe of 72
A v..|.b|es .pp.cx|n.te
Jcub||rg t|ne e,u.|s 2
J|v|JeJ by t|e g.c.t| ..te,
st.teJ .s . .|c|e runbe..
ieal GDP of $1,408 billion and an infationaiy gap. If we iecoid giowth as the annual iate of change
between these levels, we fnd an annual iate of giowth of 4.6-a iathei impiessive peifoimance.
Now considei the second decade shown in Figuie 23.2. It began in yeai 10, and it
ended in yeai 20 with a iecessionaiy gap. If we measuie the giowth iate ovei that peii-
od by looking at beginning and ending values of actual ieal GDP, we compute an annu-
al giowth iate of 0.3. Viewed in this way, peifoimance in the fist decade is spectacu-
lai while peifoimance in the second is iathei lacklustei. But these fguies depend on the
staiting and ending points we select; the giowth iate of potential output was 2.3
thioughout the peiiod.
By measuiing economic giowth as the iate of inciease in potential output, we
avoid such pioblems. One way to do this is to select yeais in which the economy was
opeiating at the natuial level of employment and then to compute the annual iate of
change between those yeais. The iesult is an estimate of the iate at which potential out-
put incieased ovei the peiiod in question. Foi the economy shown in Figuie 23.2, foi
example, we see that ieal GDP equaled its potential in yeais 3 and 13. Real GDP in yeai
3 was $1,131, and ieal GDP in yeai 13 was $1,448. The annual iate of change between
these two yeais was 2.3. If we have estimates of potential output, of couise, we can
simply compute annual iates of change between any two yeais.
1.2 The Rule of 72 and Diffeiences in Giowth Rates
The Case in Point on piesidents and giowth at the end of this section suggests a stait-
ling fact: the U.S. giowth iate began slowing in the 1970s, did not iecovei until the
mid-1990s, only to slow down again in the 2000s. The question we addiess heie is: does
it mattei: Does a peicentage point diop in the giowth iate make much difeience: It
does. To see why, let us investigate what happens when a vaiiable giows at a paiticulai
peicentage iate.
Suppose two economies with equal populations stait out at the same level of ieal
GDP but giow at difeient iates. Economy A giows at a iate of 3.3, and Economy B
giows at a iate of 2.4. Aftei a yeai, the difeience in ieal GDP will haidly be notice-
able. Aftei a decade, howevei, ieal GDP in Economy A will be 11 gieatei than in
Economy B. Ovei longei peiiods, the difeience will be moie diamatic. Aftei 100 yeais,
foi example, income in Economy A will be neaily thiee times as gieat as in Economy B.
If population giowth in the two countiies has been the same, the people of Economy A
will have a fai highei standaid of living than those in Economy B. The difeience in ieal
GDP pei peison will be ioughly equivalent to the difeience that exists today between
Gieat Biitain and Mexico.
Ovei time, small difeiences in giowth iates cieate laige difeiences in incomes. An
economy giowing at a 3.3 iate incieases by 3.3 of its initial value in the fist yeai. In
the second yeai, the economy incieases by 3.3 of that new, highei value. In the thiid
yeai, it incieases by 3.3 of a still highei value. When a quantity giows at a given pei-
centage iate, it expeiiences exponential growth. A vaiiable that giows exponentially
follows a path such as those shown foi potential output in Figuie 23.1 and Figuie 23.2.
These cuives become steepei ovei time because the giowth iate is applied to an evei-
laigei base.
A vaiiable giowing at some exponential iate doubles ovei fxed inteivals of time.
The doubling time is given by the rule of 72, which states that a vaiiable's appioximate doubling time
equals 72 divided by the giowth iate, stated as a whole numbei. If the level of income weie incieasing
at a 9 iate, foi example, its doubling time would be ioughly 72/9, oi 8 yeais.
[1]
Let us apply this concept of a doubling time to the ieduction in the U.S. giowth iate. Had the U.S.
economy continued to giow at a 3.3 iate aftei 1970, then its potential output would have doubled
ioughly eveiy 20 yeais (72/3.3 = 20). That means potential output would have doubled by 1990, would
double again by 2010, and would double again by 2030. Real GDP in 2030 would thus be eight times as
gieat as its 1970 level. Giowing at a 2.4 iate, howevei, potential output doubles only eveiy 30 yeais
(72/2.4 = 30). It would take until 2000 to double once fiom its 1970 level, and it would double once
moie by 2030. Potential output in 2030 would thus be foui times its 1970 level if the economy giew at a
2.4 iate (veisus eight times its 1970 level if it giew at a 3.3 iate). The 1.1 difeience in giowth iates
pioduces a 100 difeience in potential output by 2030. The difeient giowth paths implied by these
giowth iates aie illustiated in Figuie 23.3.
578 PRINCIPLS OI CONOMICS
output per capita
|e.| C|| pe. pe.scr.
II GUR 23. 3 Differences in Growth Rates
|e c|..t suggests t|e s|gr||c.rce |r t|e |crg .ur c| . sn.|| J||e.erce |r t|e g.c.t| ..te c| .e.| C||. \e beg|r |r
190, .|er .e.| C|| e,u.|eJ 2,83.9 b||||cr. || .e.| C|| g.e. .t .r .rru.| ..te c| 3.5 |.cn t|.t ye.., |t .cu|J
Jcub|e .cug||y eve.y 20 ye..s. |r 1990, 2010, .rJ 2030. C.c.t| .t . 2.4 ..te, |c.eve., |np||es Jcub||rg eve.y 30
ye..s. |r 2000 .rJ 2030. by 2030, t|e 3.5 g.c.t| ..te |e.ves .e.| C|| .t t.|ce t|e |eve| t|.t .cu|J be .c||eveJ by
2.4 g.c.t|.
1.3 Giowth in Output pei Capita
Of couise, it is not just how fast potential output giows that deteimines how fast the aveiage peison's
mateiial standaid of living iises. Foi that puipose, we examine economic giowth on a pei capita basis.
An economy's output per capita equals ieal GDP pei peison. If we let ^ equal population, then
QUA7I ON 23. 1
Output pei capita =
ieal GDP
^
In the United States in the thiid quaitei of 2008, foi example, ieal GDP was $11,720 billion
(annual iate). The U.S. population was 303.7 million. Real U.S. output pei capita thus equaled $38,338.
We use output pei capita as a gauge of an economy's mateiial standaid of living. If the economy's
population is giowing, then output must iise as iapidly as the population if output pei capita is to ie-
main unchanged. If, foi example, population incieases by 2, then ieal GDP would have to iise by 2
to maintain the cuiient level of output pei capita. If ieal GDP iises by less than 2, output pei capita
will fall. If ieal GDP iises by moie than 2, output pei capita will iise. Moie geneially, we can wiite:
QUA7I ON 23. 2
iate of giowth of output pei capita iate of giowth of output iate of giowth of population
Foi economic giowth to tianslate into a highei standaid of living on aveiage, economic giowth
must exceed population giowth. Fiom 1970 to 2004, foi example, Sieiia Leone's population giew at an
annual iate of 2.1 pei yeai, while its ieal GDP giew at an annual iate of 1.4; its output pei capita
thus fell at a iate of 0.7 pei yeai. Ovei the same peiiod, Singapoie's population giew at an annual iate
of 2.1 pei yeai, while its ieal GDP giew 7.4 pei yeai. The iesultant 3.3 annual giowth in output
pei capita tiansfoimed Singapoie fiom a ielatively pooi countiy to a countiy with the one of the
highest pei capita incomes in the woild.
CHAP7R 23 CONOMIC GROW7H 579
k Y 7 A k A W A Y S
< |ccrcn|c g.c.t| |s t|e p.ccess t|.cug| .||c| .r eccrcnys p.cJuct|cr pcss|b|||t|es cu.ve s|||ts
cut...J. \e ne.su.e |t .s t|e ..te .t .||c| t|e eccrcnys pctert|.| |eve| c| cutput |rc.e.ses.
< Ve.su.|rg eccrcn|c g.c.t| .s t|e ..te c| |rc.e.se c| t|e .ctu.| |eve| c| .e.| C|| c.r |e.J tc n|s|e.J|rg
.esu|ts Jue tc t|e bus|ress cyc|e.
< C.c.t| c| . ,u.rt|ty .t . p..t|cu|.. pe.cert.ge ..te |np||es expcrert|.| g.c.t|. \|er scnet||rg g.c.s
expcrert|.||y, |t Jcub|es cve. |xeJ |rte.v.|s c| t|ne, t|ese |rte.v.|s n.y be ccnputeJ us|rg t|e .u|e c| 2.
< Sn.|| J||e.erces |r ..tes c| eccrcn|c g.c.t| c.r |e.J tc |..ge J||e.erces |r |eve|s c| pctert|.| cutput
cve. |crg pe.|cJs c| t|ne.
< c .ssess c|.rges |r .ve..ge st.rJ..Js c| ||v|rg, .e subt..ct t|e pe.cert.ge ..te c| g.c.t| c| pcpu|.t|cr
|.cn t|e pe.cert.ge ..te c| g.c.t| c| cutput tc get t|e pe.cert.ge ..te c| g.c.t| c| cutput pe. c.p|t..
7 R Y I 7 !
Suppcse .r eccrcnys pctert|.| cutput .rJ .e.| C|| |s 5 n||||cr |r 2000 .rJ |ts ..te c| eccrcn|c g.c.t| |s
3 pe. ye... A|sc suppcse t|.t |ts pcpu|.t|cr |s 5,000 |r 2000, .rJ t|.t |ts pcpu|.t|cr g.c.s .t . ..te c| 1 pe.
ye... cnpute C|| pe. c.p|t. |r 2000. |c. est|n.te C|| .rJ C|| pe. c.p|t. |r 202, us|rg t|e .u|e c| 2. At
.|.t ..te Jces C|| pe. c.p|t. g.c.` \|.t |s |ts Jcub||rg t|ne` |s t||s .esu|t ccrs|stert .|t| ycu. |rJ|rgs |c.
C|| pe. c.p|t. |r 2000 .rJ |r 202`
Case in Point: Presidents and conomic Growth
President AnnuaI Increase in ReaI GDP (%) Growth Rate (%)
.un.r 19491952 5.4 4.4
||ser|c.e. 19531960 2.4 3.4
|erreJy'c|rscr 19611968 5.1 4.3
||xcr|c.J 1969196 2. 3.4
..te. 191980 3.2 3.1
|e.g.r 19811988 3.5 3.1
C. |. \. bus| 19891992 2.4 2.
||rtcr 19922000 3.6 3.2
C. \. bus| 20012008 (3) 2.1 2.
|.es|Jerts ..e c|ter uJgeJ by t|e ..te .t .||c| t|e eccrcny g.e. .|||e t|ey .e.e |r c|ce. ||s test |s ur|.|.
cr t.c ccurts. ||.st, . p.es|Jert |.s ||tt|e tc Jc .|t| t|e |c.ces t|.t Jete.n|re g.c.t|. ArJ seccrJ, suc| tests
s|np|y ccnpute t|e .rru.| ..te c| g.c.t| |r .e.| C|| cve. t|e ccu.se c| . p.es|Jert|.| te.n, .||c| .e krc.
c.r be .|ecteJ by cyc||c.| |.ctc.s. A p.es|Jert .|c t.kes c|ce .|er t|e eccrcny |s Jc.r .rJ gces cut .|t|
t|e eccrcny up .||| |cck ||ke .r eccrcn|c st.., . p.es|Jert .|t| t|e b.J |uck tc |.ve .eve.se c|.cunst.rces
.||| seen ||ke . JuJ. |e.e ..e .rru.| ..tes c| c|.rge |r .e.| C|| |c. e.c| c| t|e pcst... p.es|Jerts, tcget|e.
.|t| ..tes c| eccrcn|c g.c.t|, ne.su.eJ .s t|e .rru.| ..te c| c|.rge |r pctert|.| cutput.
580 PRINCIPLS OI CONOMICS
|e p.es|Jerts eccrcn|c .ecc.Js ..e c|e..|y .|ecteJ by |uck. |.es|Jerts .un.r, |erreJy, |e.g.r, .rJ ||r
tcr, |c. ex.np|e, beg.r t|e|. te.ns .|er t|e eccrcny |.J . .ecess|cr..y g.p .rJ erJeJ t|en .|t| .r |r|.
t|cr..y g.p c. .t .bcut pctert|.| cutput. |e.| C|| t|us .cse |.ste. t|.r pctert|.| cutput Ju.|rg t|e|. p.es|J
erc|es. |e ||ser|c.e., ||xcr|c.J, |. \. bus|, .rJ C. \. bus| .Jn|r|st..t|crs e.c| st..teJ .|t| .r |r|.t|cr
..y g.p c. .t .bcut pctert|.| .rJ erJeJ .|t| . .ecess|cr..y g.p, t|us .ecc.J|rg ..tes c| .e.| C|| |rc.e.se be
|c. t|e ..te c| g.|r |r pctert|.|. Or|y '|nny ..te., .|c c.ne tc c|ce .rJ |e|t |t .|t| .ecess|cr..y g.ps,
p.es|JeJ cve. . .e|.t|ve|y e,u|v.|ert ..te c| |rc.e.se |r .ctu.| C|| ve.sus pctert|.| cutput.
A N S W R 7 O 7 R Y I 7 ! P R O 8 L M
C|| pe. c.p|t. |r 2000 e,u.|s 1,000 (5,000,000/5,000). || C|| .|ses 3 pe. ye.., |t Jcub|es eve.y 24 ye..s (
2/3). |us, C|| .||| be 10,000,000 |r 2024, 20,000,000 |r 2048, .rJ 40,000,000 |r 202. C.c.|rg .t . ..te
c| 1 pe. ye.., pcpu|.t|cr .||| |.ve Jcub|eJ crce by 202 tc 10,000. C|| pe. c.p|t. .||| t|us be 4,000 (
40,000,000/10,000). |ct|ce t|.t C|| .|ses by e|g|t t|nes |ts c.|g|r.| |eve|, .|||e t|e |rc.e.se |r C|| pe. c.p|t.
|s |cu.|c|J. |e |.tte. v.|ue .ep.eserts . g.c.t| ..te |r cutput pe. c.p|t. c| 2 pe. ye.., .||c| |np||es . Jcub
||rg t|ne c| 36 ye..s. |.t g|ves t.c Jcub||rgs |r C|| pe. c.p|t. bet.eer 2000 .rJ 202 .rJ ccr|.ns .
|cu.|c|J |rc.e.se.
2. GROWTH AND THE LONG-RUN AGGREGATE SUPPLY
CURVE
L A R N I N G O 8 1 C 7 I V S
1. xpIain and iIIustrate graphicaIIy the concept of the aggregate production function. xpIain
how its shape reIates to the concept of diminishing marginaI returns.
2. Derive the Iong-run aggregate suppIy curve from the modeI of the Iabor market and the ag-
gregate production function.
3. xpIain how the Iong-run aggregate suppIy curve shifts in responses to shifts in the aggregate
production function or to shifts in the demand for or suppIy of Iabor.
Economic giowth means the economy's potential output is iising. Because the long-iun aggiegate sup-
ply cuive is a veitical line at the economy's potential, we can depict the piocess of economic giowth as
one in which the long-iun aggiegate supply cuive shifts to the iight.
CHAP7R 23 CONOMIC GROW7H 581
II GUR 23. 5 conomic Growth and
the Long-Run Aggregate SuppIy Curve
bec.use eccrcn|c g.c.t| |s t|e p.ccess
t|.cug| .||c| t|e eccrcnys pctert|.|
cutput |s |rc.e.seJ, .e c.r Jep|ct |t .s . se.|es
c| .|g|t...J s|||ts |r t|e |crg.ur .gg.eg.te
supp|y cu.ve. |ct|ce t|.t .|t| expcrert|.|
g.c.t|, e.c| success|ve s|||t |r ||^. |s |..ge.
.rJ |..ge..
aggregate production
function
|urct|cr t|.t .e|.tes t|e tct.|
cutput c| .r eccrcny tc t|e
tct.| .ncurt c| |.bc.
enp|cyeJ |r t|e eccrcny, .||
ct|e. Jete.n|r.rts c|
p.cJuct|cr (c.p|t.|, r.tu..|
.escu.ces, .rJ tec|rc|cgy)
be|rg urc|.rgeJ.
diminishing marginaI
returns
S|tu.t|cr t|.t cccu.s .|er
.JJ|t|cr.| ur|ts c| . v..|.b|e
|.ctc. .JJ |ess .rJ |ess tc
tct.| cutput, g|ver ccrst.rt
,u.rt|t|es c| ct|e. |.ctc.s.
Figuie 23.3 illustiates the piocess of economic giowth. If the economy begins at
potential output of Y
1
, giowth incieases this potential. The fguie shows a succession of
incieases in potential to Y
2
, then Y
3
, and Y
4
. If the economy is giowing at a paiticulai
peicentage iate, and if the levels shown iepiesent successive yeais, then the size of the
incieases will become laigei and laigei, as indicated in the fguie.
Because economic giowth can be consideied as a piocess in which the long-iun ag-
giegate supply cuive shifts to the iight, and because output tends to iemain close to this
cuive, it is impoitant to gain a deepei undeistanding of what deteimines long-iun ag-
giegate supply (LRAS). We shall examine the deiivation of LRAS and then see what
factois shift the cuive. We shall begin oui woik by defning an aggiegate pioduction
function.
2.1 The Aggiegate Pioduction Function
An aggregate production function ielates the total output of an economy to the
total amount of laboi employed in the economy, all othei deteiminants of pioduction
(that is, capital, natuial iesouices, and technology) being unchanged. An economy op-
eiating on its aggiegate pioduction function is pioducing its potential level of output.
Figuie 23.6 shows an aggiegate pioduction function (PF). It shows output levels
foi a iange of employment between 120 million and 140 million woikeis. When the
level of employment is 120 million, the economy pioduces a ieal GDP of $11,300 bil-
lion (point A). A level of employment of 130 million pioduces a ieal GDP of $12,000
billion (point B), and when 140 million woikeis aie employed, a ieal GDP of $12,300
billion is pioduced (point C). In diawing the aggiegate pioduction function, the
amount of laboi vaiies, but eveiything else that could afect output, specifcally the
quantities of othei factois of pioduction and technology, is fxed.
The shape of the aggiegate pioduction function shows that as employment incieases, output in-
cieases, but at a decieasing iate. Incieasing employment fiom 120 million to 130 million, foi example,
incieases output by $300 billion to $12,000 billion at point B. The next 10 million woikeis inciease pio-
duction by $300 billion to $12,300 billion at point C. This example illustiates diminishing maiginal ie-
tuins. Diminishing marginal returns occui when additional units of a vaiiable factoi add less and
less to total output, given constant quantities of othei factois.
II GUR 23. 6 7he Aggregate Production Iunction
Ar .gg.eg.te p.cJuct|cr |urct|cr (||) .e|.tes tct.| cutput tc tct.| enp|cynert, .ssun|rg .|| ct|e. |.ctc.s c|
p.cJuct|cr .rJ tec|rc|cgy ..e |xeJ. |t s|c.s t|.t |rc.e.ses |r enp|cynert |e.J tc |rc.e.ses |r cutput but .t .
Jec.e.s|rg ..te.
It is easy to pictuie the pioblem of diminishing maiginal ietuins in the context of a single fim. The
fim is able to inciease output by adding woikeis. But because the fim's plant size and stock of equip-
ment aie fxed, the fim's capital pei woikei falls as it takes on moie woikeis. Each additional woikei
582 PRINCIPLS OI CONOMICS
adds less to output than the woikei befoie. The fim, like the economy, expeiiences diminishing mai-
ginal ietuins.
2.2 The Aggiegate Pioduction Function, the Maiket foi Laboi, and
Long-Run Aggiegate Supply
To deiive the long-iun aggiegate supply cuive, we biing togethei the model of the laboi maiket, intio-
duced in the fist macio chaptei and the aggiegate pioduction function.
As we leained, the laboi maiket is in equilibiium at the natuial level of employment. The demand
and supply cuives foi laboi inteisect at the ieal wage at which the economy achieves its natuial level of
employment. We see in Panel (a) of Figuie 23.7 that the equilibiium ieal wage is a
1
and the natuial
level of employment is L
1
. Panel (b) shows that with employment of L
1
, the economy can pioduce a
ieal GDP of Y
P
. That output equals the economy's potential output. It is that level of potential output
that deteimines the position of the long-iun aggiegate supply cuive in Panel (c).
II GUR 23. 7 Deriving the Long-Run Aggregate SuppIy Curve
|.re| (.) s|c.s t|.t t|e e,u|||b.|un .e.| ..ge |s .
1
, .rJ t|e r.tu..| |eve| c| enp|cynert |s |
1
. |.re| (b) s|c.s t|.t
.|t| enp|cynert c| |
1
, t|e eccrcny c.r p.cJuce . .e.| C|| c| `
|
. |.t cutput e,u.|s t|e eccrcnys pctert|.|
cutput. |t |s .t t|.t |eve| c| pctert|.| cutput t|.t .e J... t|e |crg.ur .gg.eg.te supp|y cu.ve |r |.re| (c).
Changes in Long-Run Aggregate SuppIy
The position of the long-iun aggiegate supply cuive is deteimined by the aggiegate pioduction func-
tion and the demand and supply cuives foi laboi. A change in any of these will shift the long-iun ag-
giegate supply cuive.
Figuie 23.8 shows one possible shiftei of long-iun aggiegate supply: a change in the pioduction
function. Suppose, foi example, that an impiovement in technology shifts the aggiegate pioduction
function in Panel (b) fiom PF
1
to PF
2
. Othei developments that could pioduce an upwaid shift in the
cuive include an inciease in the capital stock oi in the availability of natuial iesouices.
CHAP7R 23 CONOMIC GROW7H 583
productivity
|e .ncurt c| cutput pe.
.c.ke..
II GUR 23. 8 Shift in the Aggregate Production Iunction and the Long-Run Aggregate SuppIy Curve
Ar |np.cvenert |r tec|rc|cgy s|||ts t|e .gg.eg.te p.cJuct|cr |urct|cr up...J |r |.re| (b). bec.use |.bc. |s
nc.e p.cJuct|ve, t|e Jen.rJ |c. |.bc. s|||ts tc t|e .|g|t |r |.re| (.), .rJ t|e r.tu..| |eve| c| enp|cynert
|rc.e.ses tc |
2
. |r |.re| (c) t|e |crg.ur .gg.eg.te supp|y cu.ve s|||ts tc t|e .|g|t tc `
2
.
The shift in the pioduction function to PF
2
means that laboi is now moie pioductive than befoie. This
will afect the demand foi laboi in Panel (a). Befoie the technological change, fims employed L
1
woik-
eis at a ieal wage a
1
. If woikeis aie moie pioductive, fims will fnd it pioftable to hiie moie of them
at a
1
. The demand cuive foi laboi thus shifts to D
2
in Panel (a). The ieal wage iises to a
2
, and the nat-
uial level of employment iises to L
2
. The inciease in the ieal wage iefects laboi's enhanced productiv-
ity, the amount of output pei woikei. To see how potential output changes, we see in Panel (b) how
much output can be pioduced given the new natuial level of employment and the new aggiegate pio-
duction function. The ieal GDP that the economy is capable of pioducing iises fiom Y
1
to Y
2
. The
highei output is a iefection of a highei natuial level of employment, along with the fact that laboi has
become moie pioductive as a iesult of the technological advance. In Panel (c) the long-iun aggiegate
supply cuive shifts to the iight to the veitical line at Y
2
.
This analysis dispels a common misconception about the impact of impiovements in technology
oi incieases in the capital stock on employment. Some people believe that technological gains oi in-
cieases in the stock of capital ieduce the demand foi laboi, ieduce employment, and ieduce ieal wages.
Ceitainly the expeiience of the United States and most othei countiies belies that notion. Between 1990
and 2007, foi example, the U.S. capital stock and the level of technology incieased diamatically. Duiing
the same peiiod, employment and ieal wages iose, suggesting that the demand foi laboi incieased by
moie than the supply of laboi. As some fims add capital oi incoipoiate new technologies, some woik-
eis at those fims may lose theii jobs. But foi the economy as a whole, new jobs become available and
they geneially ofei highei wages. The demand foi laboi iises.
Anothei event that can shift the long-iun aggiegate supply cuive is an inciease in the supply of
laboi, as shown in Figuie 23.9. An incieased supply of laboi could iesult fiom immigiation, an inciease
in the population, oi incieased paiticipation in the laboi foice by the adult population. Incieased paiti-
cipation by women in the laboi foice, foi example, has tended to inciease the supply cuive foi laboi
duiing the past seveial decades.
584 PRINCIPLS OI CONOMICS
II GUR 23. 9 Increase in the SuppIy of Labor and the Long-Run Aggregate SuppIy Curve
Ar |rc.e.se |r t|e supp|y c| |.bc. s|||ts t|e supp|y cu.ve |r |.re| (.) tc .
2
, .rJ t|e r.tu..| |eve| c| enp|cynert
.|ses tc |
2
. |e .e.| ..ge |.||s tc .
2
. \|t| |rc.e.seJ |.bc., t|e .gg.eg.te p.cJuct|cr |urct|cr |r |.re| (b) s|c.s
t|.t t|e eccrcny |s rc. c.p.b|e c| p.cJuc|rg .e.| C|| .t `
2
. |e |crg.ur .gg.eg.te supp|y cu.ve |r |.re| (c)
s|||ts tc ||^.
2
.
In Panel (a), an inciease in the laboi supply shifts the supply cuive to S
2
. The inciease in the supply of
laboi does not change the stock of capital oi natuial iesouices, noi does it change technology-it theie-
foie does not shift the aggiegate pioduction function. Because theie is no change in the pioduction
function, theie is no shift in the demand foi laboi. The ieal wage falls fiom a
1
to a
2
in Panel (a), and
the natuial level of employment iises fiom L
1
to L
2
. To see the impact on potential output, Panel (b)
shows that employment of L
2
can pioduce ieal GDP of Y
2
. The long-iun aggiegate supply cuive in
Panel (c) thus shifts to LRAS
2
. Notice, howevei, that this shift in the long-iun aggiegate supply cuive to
the iight is associated with a ieduction in the ieal wage to a
2
.
Of couise, the aggiegate pioduction function and the supply cuive of laboi can shift togethei, pio-
ducing highei ieal wages at the same time population iises. That has been the expeiience of most in-
dustiialized nations. The inciease in ieal wages in the United States between 1990 and 2007, foi ex-
ample, came duiing a peiiod in which an incieasing population incieased the supply of laboi. The de-
mand foi laboi incieased by moie than the supply, pushing the ieal wage up. The accompanying Case
in Point looks at gains in ieal wages in the face of technological change, an inciease in the stock of cap-
ital, and iapid population giowth in the United States duiing the 19th centuiy.
Oui model of long-iun aggiegate supply tells us that in the long iun, ieal GDP, the natuial level of
employment, and the ieal wage aie deteimined by the economy's pioduction function and by the de-
mand and supply cuives foi laboi. Unless an event shifts the aggiegate pioduction function, the de-
mand cuive foi laboi, oi the supply cuive foi laboi, it afects neithei the natuial level of employment
noi potential output. Economic giowth occuis only if an event shifts the economy's pioduction func-
tion oi if theie is an inciease in the demand foi oi the supply of laboi.
k Y 7 A k A W A Y S
< |e .gg.eg.te p.cJuct|cr |urct|cr .e|.tes t|e |eve| c| enp|cynert tc t|e |eve| c| .e.| C|| p.cJuceJ pe.
pe.|cJ.
< |e .e.| ..ge .rJ t|e r.tu..| |eve| c| enp|cynert ..e Jete.n|reJ by t|e |rte.sect|cr c| t|e Jen.rJ .rJ
supp|y cu.ves |c. |.bc.. |ctert|.| cutput |s g|ver by t|e pc|rt cr t|e .gg.eg.te p.cJuct|cr |urct|cr
cc..espcrJ|rg tc t|e r.tu..| |eve| c| enp|cynert. ||s cutput |eve| |s t|e s.ne .s t|.t s|c.r by t|e
|crg.ur .gg.eg.te supp|y cu.ve.
< |ccrcn|c g.c.t| c.r be s|c.r .s . se.|es c| s|||ts tc t|e .|g|t |r ||^.. Suc| s|||ts .e,u|.e e|t|e. up...J
s|||ts |r t|e p.cJuct|cr |urct|cr c. |rc.e.ses |r Jen.rJ |c. c. supp|y c| |.bc..
CHAP7R 23 CONOMIC GROW7H 585
7 R Y I 7 !
Suppcse t|.t t|e ,u.rt|ty c| |.bc. supp||eJ |s 50 n||||cr .c.ke.s .|er t|e .e.| ..ge |s 20,000 pe. ye.. .rJ
t|.t pctert|.| cutput |s 2,000 b||||cr pe. ye... |... . t|.eep.re| g..p| s|n||.. tc t|e cre p.eserteJ |r ||g
u.e 23.9 tc s|c. t|e eccrcnys |crg.ur e,u|||b.|un. |.re| (.) c| ycu. g..p| s|cu|J s|c. t|e Jen.rJ .rJ
supp|y cu.ves |c. |.bc., |.re| (b) s|cu|J s|c. t|e .gg.eg.te p.cJuct|cr |urct|cr, .rJ |.re| (c) s|cu|J s|c.
t|e |crg.ur .gg.eg.te supp|y cu.ve. |c. suppcse . tec|rc|cg|c.| c|.rge |rc.e.ses t|e eccrcnys cutput
.|t| t|e s.ne ,u.rt|ty c| |.bc. .s be|c.e tc 2,200 b||||cr, .rJ t|e .e.| ..ge .|ses tc 21,500. |r .espcrse, t|e
,u.rt|ty c| |.bc. supp||eJ |rc.e.ses tc 51 n||||cr .c.ke.s. |r t|e s.ne t|.ee p.re|s ycu |.ve .|.e.Jy J...r,
sketc| t|e re. cu.ves t|.t .esu|t |.cn t||s c|.rge. |xp|.|r .|.t |.ppers tc t|e |eve| c| enp|cynert, t|e
|eve| c| pctert|.| cutput, .rJ t|e |crg.ur .gg.eg.te supp|y cu.ve. (||rt. ycu |.ve |r|c.n.t|cr |c. cr|y cre
pc|rt cr e.c| c| t|e cu.ves ycu J...t.c |c. t|e supp|y c| |.bc., s|np|y J... cu.ves c| t|e .pp.cp.|.te
s|.pe. |c rct .c..y .bcut gett|rg t|e sc.|e cc..ect.)
Case in Point: 7echnoIogicaI Change, mpIoyment, and ReaI Wages During the
IndustriaI RevoIution
ec|rc|cg|c.| c|.rge .rJ t|e c.p|t.| |rvestnert t|.t typ|c.||y ccnes .|t| |t ..e c|ter c.|t|c|.eJ bec.use t|ey
.ep|.ce |.bc. .|t| n.c||res, .eJuc|rg enp|cynert. Suc| c|.rges, c.|t|cs ..gue, |u.t .c.ke.s. |s|rg t|e ncJ
e| c| .gg.eg.te Jen.rJ .rJ .gg.eg.te supp|y, |c.eve., .e ...|ve .t . ,u|te J||e.ert ccrc|us|cr. |e ncJe|
p.eJ|cts t|.t |np.cveJ tec|rc|cgy .||| |rc.e.se t|e Jen.rJ |c. |.bc. .rJ bccst .e.| ..ges.
|e pe.|cJ c| |rJust.|.||..t|cr, gere..||y t.ker tc be t|e t|ne bet.eer t|e |v|| \.. .rJ \c.|J \.. |, ..s .
gccJ test c| t|ese ccnpet|rg |Je.s. ec|rc|cg|c.| c|.rges .e.e J..n.t|c .s |.ns s|||teJ tc...J n.ss p.c
Juct|cr .rJ .utcn.t|cr. .p|t.| |rvestnert sc..eJ. |nn|g..t|cr |rc.e.seJ t|e supp|y c| |.bc.. \|.t
|.ppereJ tc .c.ke.s`
|np|cynert nc.e t|.r Jcub|eJ Ju.|rg t||s pe.|cJ, ccrs|stert .|t| t|e p.eJ|ct|cr c| cu. ncJe|. |t |s |..Je. tc
p.eJ|ct, |.cn . t|ec.et|c.| pc|rt c| v|e., t|e ccrse,uerces |c. .e.| ..ges. |e |.tte. t||.J c| t|e 19t| certu.y
..s . pe.|cJ c| n.ss|ve |nn|g..t|cr tc t|e |r|teJ St.tes. bet.eer 1865 .rJ 1880, nc.e t|.r 5 n||||cr
pecp|e c.ne tc t|e |r|teJ St.tes |.cn .b.c.J, ncst .e.e c| .c.k|rg .ge. |e p.ce .cce|e..teJ bet.eer
1880 .rJ 1923, .|er nc.e t|.r 23 n||||cr pecp|e ncveJ tc t|e |r|teJ St.tes |.cn ct|e. ccurt.|es. |nn|g..
t|cr |rc.e.seJ t|e supp|y c| |.bc., .||c| s|cu|J .eJuce t|e .e.| ..ge. |e.e .e.e t|us t.c ccnpet|rg |c.ces
.t .c.k. ec|rc|cg|c.| c|.rge .rJ c.p|t.| |rvestnert terJeJ tc |rc.e.se .e.| ..ges, .|||e |nn|g..t|cr ter
JeJ tc .eJuce t|en by |rc.e.s|rg t|e supp|y c| |.bc..
|e ev|Jerce suggests t|.t t|e |c.ces c| tec|rc|cg|c.| c|.rge .rJ c.p|t.| |rvestnert p.cveJ |.. nc.e
pc.e.|u| t|.r |rc.e.ses |r |.bc. supp|y. |e.| ..ges sc..eJ 60 bet.eer 1860 .rJ 1890. |ey ccrt|rueJ tc
|rc.e.se .|te. t|.t. |e.| ..ges |r n.ru|.ctu.|rg, |c. ex.np|e, .cse 3 |.cn 1890 tc 1914.
ec|rc|cg|c.| c|.rge .rJ c.p|t.| |rvestnert J|sp|.ce .c.ke.s |r scne |rJust.|es. but |c. t|e eccrcny .s .
.|c|e, t|ey |rc.e.se .c.ke. p.cJuct|v|ty, |rc.e.se t|e Jen.rJ |c. |.bc., .rJ |rc.e.se .e.| ..ges.
!385(*6 /oe Jo|o |o|e |o |oece | |o. /oe oJ |o| | ||e |||eJ .|o|e. S60990 (||ce|o. |. ||ce|o ||.e||, |e.
960:. 09. oJ |o ^|e| |ee. /oe | /oo|oc|o|. S9094 (||ce|o. |. ||ce|o ||.e||, |e. 96:. 35 ||o||o |oe
|o|e |o Oo, / /o||o oJ |o| |oc|o|. |||o, o| ||e ^e|co |coo,. 6|| eJ (|e. `o| |ocoo| |oce .o.oo.|c|. 990:. 3.
586 PRINCIPLS OI CONOMICS
A N S W R 7 O 7 R Y I 7 ! P R O 8 L M
|e p.cJuct|cr |urct|cr |r |.re| (b) s|||ts up tc ||
2
. bec.use |t .e|ects g.e.te. p.cJuct|v|ty c| |.bc., |.ns .|||
|rc.e.se t|e|. Jen.rJ |c. |.bc., .rJ t|e Jen.rJ cu.ve |c. |.bc. s|||ts tc |
2
|r |.re| (.). ||^.
1
s|||ts tc ||^.
2
|r |.re| (c). |np|cynert .rJ pctert|.| cutput .|se. |ctert|.| cutput .||| be g.e.te. t|.r 2,200 b||||cr.
CHAP7R 23 CONOMIC GROW7H 587
3. DETERMINANTS OF ECONOMIC GROWTH
L A R N I N G O 8 1 C 7 I V S
1. Discuss the sources of economic growth.
2. Discuss possibIe reasons why countries grow at dierent rates.
In this section, we ieview the main deteiminants of economic giowth. We also examine the ieasons foi
the widening dispaiities in economic giowth iates among countiies in iecent yeais.
3.1 The Souices of Economic Giowth
As we have leained, theie aie two ways to model economic giowth: (1) as an outwaid shift in an eco-
nomy's pioduction possibilities cuive, and (2) as a shift to the iight in its long-iun aggiegate supply
cuive. In diawing eithei one at a point in time, we assume that the economy's factois of pioduction
and its technology aie unchanged. Changing these will shift both cuives. Theiefoie, anything that in-
cieases the quantity oi quality of factois of pioduction oi that impioves the technology available to the
economy contiibutes to economic giowth.
The souices of giowth foi the U.S. economy in the 20th centuiy weie piesented in the chaptei on
souices of pioduction. Theie we leained that the main souices of giowth foi the United States fiom
1948 to 2002 weie divided between incieases in the quantities of laboi and of physical capital (about
60) and in impiovements in the qualities of the factois of pioduction and technology (about 40).
Since 1993, howevei, impiovements in factoi quality and technology have been the main diiveis of
economic giowth in the United States.
In oidei to devote iesouices to incieasing physical and human capital and to impioving techno-
logy-activities that will enhance futuie pioduction-society must foigo using them now to pioduce
consumei goods. Even though the people in the economy would enjoy a highei standaid of living today
without this saciifce, they aie willing to ieduce piesent consumption in oidei to have moie goods and
seivices available foi the futuie.
As a college student, you peisonally made such a choice. You decided to devote time to study that
you could have spent eaining income. With the highei income, you could enjoy gieatei consumption
today. You made this choice because you expect to eain highei income in the futuie and thus to enjoy
gieatei consumption in the futuie. Because many othei people in the society also choose to acquiie
moie education, society allocates iesouices to pioduce education. The education pioduced today will
enhance the society's human capital and thus its economic giowth.
All othei things equal, highei saving allows moie iesouices to be devoted to incieases in physical
and human capital and technological impiovement. In othei woids, saving, which is income not spent
on consumption, piomotes economic giowth by making available iesouices that can be channeled into
giowth-enhancing uses.
3.2 Explaining Recent Dispaiities in Giowth Rates
Towaid the end of the 20th centuiy, it appeaied that some of the woild's moie amuent countiies weie
giowing iobustly while otheis weie giowing moie slowly oi even stagnating. This obseivation was con-
fimed in a majoi study by the Oiganization foi Economic Co-opeiation and Development (OECD),
[2]
whose membeis aie listed in Table 23.1. The table shows that foi the OECD countiies as a whole, eco-
nomic giowth pei capita fell fiom an aveiage of 2.2 pei yeai in the 1980s to an aveiage of 1.9 pei
yeai in the 1990s. The highei standaid deviation in the lattei peiiod confims an incieased dispaiity of
giowth iates in the moie iecent peiiod. Moieovei, the data on individual countiies show that pei cap-
ita giowth in some countiies (specifcally, the United States, Canada, Iieland, Netheilands, Noiway,
and Spain) picked up, especially in the lattei half of the 1990s, while it deceleiated in most of the coun-
tiies of continental Euiope and Japan.
588 PRINCIPLS OI CONOMICS
7A8L 23. 1 Growing Disparities in Rates of conomic Growth
\..|.t|cr |r t|e g.c.t| |r .e.| C|| pe. c.p|t. |.s .|JereJ .ncrg t|e .c.|Js |e.J|rg |rJust.|.||.eJ eccrcn|es.
7rend Growth of GDP per Capita
Country 1980-1990 1990-2000 1996-2000
|r|teJ St.tes 2.1 2.3 2.8
'.p.r 3.3 1.4 0.9
Ce.n.ry 1.9 1.2 1.
|..rce 1.6 1.5 1.9
|t.|y 2.3 1.5 1.
|r|teJ ||rgJcn 2.2 2.1 2.3
.r.J. 1.4 1. 2.6
Aust.|. 2.1 1.9 2.3
be|g|un 2.0 1.9 2.3
|ern..k 1.9 1.9 2.3
||r|.rJ 2.2 2.1 3.9
C.eece 0.5 1.8 2.
|ce|.rJ 1. 1.5 2.6
|.e|.rJ 3.0 6.4 .9
|uxenbcu.g 4.0 4.5 4.6
|et|e.|.rJs 1.6 2.4 2.
|c.tug.| 3.1 2.8 2.
Sp.|r 2.3 2. 3.2
S.eJer 1. 1.5 2.6
S.|t.e.|.rJ 1.4 0.4 1.1
u.key 2.1 2.1 1.9
Aust..||. 1.6 2.4 2.8
|e. e.|.rJ 1.4 1.2 1.8
Vex|cc 0.0 1.6 2.
|c.e. .2 5.1 4.2
|urg..y 2.3 3.5
|c|.rJ 4.2 4.8
.ec| |epub||c 1. 1.4
O||24
3
2.2 1.9 2.2
Standard Deviation of OCD24 0.4 1.1 1.3
Source Excerpted from Table 1.1 Organization for Economic Co-operation and Development, Sources of Economic Growth in OECD Countries, 2003
p. 32-33.
The study goes on to tiy to explain the ieasons foi the diveigent giowth tiends. The main fndings
weie:
< In geneial, countiies with acceleiating pei capita giowth iates also expeiienced signifcant
incieases in employment, while those with stagnant oi declining employment geneially
expeiienced ieductions in pei capita giowth iates.
< Enhancements in human capital contiibuted to laboi pioductivity and economic giowth, but in
slowei giowing countiies such impiovements weie not enough to ofset the impact of ieduced oi
stagnant laboi utilization.
< Infoimation and communication technology has contiibuted to economic giowth both thiough
iapid technological piogiess within the infoimation and communication technology industiy
itself as well as, moie iecently, thiough the use of infoimation and communication technology
equipment in othei industiies. This has made an impoitant contiibution to giowth in seveial of
the fastei giowing countiies.
CHAP7R 23 CONOMIC GROW7H 589
< Othei factois associated with moie giowth include: investments in physical and human capital,
sound macioeconomic policies (especially low infation), piivate sectoi ieseaich and
development, tiade exposuie, and bettei developed fnancial maikets. Results conceining the
impact of the size of the goveinment and of public sectoi ieseaich and development on giowth
weie moie dimcult to inteipiet.
< With qualifcations, the study found that stiict iegulation of pioduct maikets (foi example,
iegulations that ieduce competition) and stiict employment piotection legislation (foi example,
laws that make hiiing and fiing of woikeis moie dimcult) had negative efects on giowth.
< All countiies show a laige numbei of fims enteiing and exiting maikets. But, a key difeience
between the United States and Euiope is that new fims in the United States stait out smallei and
less pioductive than those of Euiope but giow fastei when they aie successful. The iepoit
hypothesizes that lowei stait-up costs and less stiict laboi maiket iegulations may encouiage U.S.
entiepieneuis to entei a maiket and then to expand, if waiianted. Euiopean entiepieneuis may
be less willing to expeiiment in a maiket in the fist place.
The geneial concein in the second half of the 1970s and the 1980s was that economic giowth was slow-
ing down and that it might not be possible to ieveise this pattein. The 1990s and eaily 2000s, in which
giowth picked up in some countiies but not in otheis, suggested that the pioblem was not univeisal
and led to a seaich foi the ieasons foi the dispaiities in giowth iates that emeiged. The OECD study
desciibed above gives some possible explanations. The fndings of that study piactically beg countiies
to examine closely theii economic policies at a vaiiety of levels and to considei changes that may add
fexibility to theii economies.
In closing, it is woith ieiteiating that economic fieedom and highei incomes tend to go togethei.
Countiies could not have attained high levels of income if they had not maintained the economic fiee-
dom that contiibuted to high incomes in the fist place. Thus, it is also likely that iates of economic
giowth in the futuie will be ielated to the amount of economic fieedom countiies choose. We shall see
in latei chapteis that monetaiy and fscal policies that aie used to stabilize the economy in the shoit
iun can also have an impact on long-iun economic giowth.
k Y 7 A k A W A Y S
< |e n.|r scu.ces c| g.c.t| |c. t|e |r|teJ St.tes |.cn 1948 tc 2002 .e.e J|v|JeJ bet.eer |rc.e.ses |r
t|e ,u.rt|t|es c| |.bc. .rJ c| p|ys|c.| c.p|t.| (.bcut 60) .rJ |r |np.cvenerts |r t|e ,u.||t|es c| t|e
|.ctc.s c| p.cJuct|cr .rJ tec|rc|cgy (.bcut 40). S|rce 1995, |c.eve., |np.cvenerts |r |.ctc. ,u.||ty
.rJ tec|rc|cgy |.ve beer t|e n.|r J.|ve.s c| eccrcn|c g.c.t| |r t|e |r|teJ St.tes.
< |e.e |.s beer . g.c.|rg J|sp..|ty |r t|e ..tes c| eccrcn|c g.c.t| |r |rJust.|.||.eJ ccurt.|es |r t|e |.st
Jec.Je, .||c| n.y .e|ect v..|cus J||e.erces |r eccrcn|c st.uctu.es .rJ pc||c|es.
7 R Y I 7 !
A|| ct|e. t||rgs urc|.rgeJ, ccnp..e t|e pcs|t|cr c| . ccurt.ys expecteJ p.cJuct|cr pcss|b|||ty cu.ve .rJ
t|e expecteJ pcs|t|cr c| |ts |crg.ur .gg.eg.te supp|y cu.ve ||.
1. |ts |.bc. |c.ce |rc.e.ses |r s|.e by 3 pe. ye.. ccnp..eJ tc 2 pe. ye...
2. |ts s.v|rg ..te |.||s |.cn 15 tc 10.
3. |t p.sses . |.. n.k|rg |t nc.e J||cu|t tc |.e .c.ke.s.
4. |ts |eve| c| eJuc.t|cr .|ses nc.e ,u|ck|y t|.r |t |.s |r t|e p.st.
590 PRINCIPLS OI CONOMICS
Case in Point: conomic Growth in Poor Countries . or Lack 7hereof
|ccrcn|st \||||.n |.ste.|y |r ||s .pt|y r.neJ bcck |e ||o|.e (oe| |o Oo.|| |coo|| ^J.e|oe oJ
/|oJ.e|oe | ||e o|c .Jn|ts t|.t .|te. 50 ye..s c| se..c||rg |c. t|e n.g|c |c.nu|. |c. tu.r|rg pcc.
ccurt.|es |rtc .|c| cres, t|e ,uest .en.|rs e|us|ve.
|cc. ccurt.|es ust reeJ nc.e p|ys|c.| c.p|t.|, ycu s.y` |.ste.|y pc|rts cut t|.t bet.eer 1960 .rJ 1985, t|e
c.p|t.| stcck pe. .c.k |r bct| C.nb|. .rJ '.p.r .cse by cve. 500. |e .esu|t` |r C.nb|., cutput pe. .c.ke.
cve. t|e 25ye.. pe.|cJ .cse 2, |r '.p.r, cutput pe. .c.ke. .cse 260.
Sc, |t nust be t|.t pcc. ccurt.|es reeJ nc.e |un.r c.p|t.|` Ag.|r, |e |rJs st..t||rg ccnp..|scrs. |c. ex
.np|e, |un.r c.p|t.| exp.rJeJ |.ste. |r .nb|. t|.r |r |c.e., but .nb|.s .rru.| g.c.t| ..te |s pe.cert
.ge pc|rts be|c. |c.e.s.
cc nuc| pcpu|.t|cr g.c.t|` cc ||tt|e` Vc.e |c.e|gr .|J` cc nuc|` As |.ste.|y p.cceeJs, ..|t|rg . p.e
sc.|pt|cr |c. g.c.t| seens eve. nc.e J||cu|t. |cre |.s Je||ve.eJ .s p.cn|seJ, |e ccrc|uJes (p. x|).
\|||e |.ste.|y Jces rct c|e. ||s c.r re. p.r.ce. |c. |c. tc ncve ccurt.|es tc . ||g|e. |eve| c| pe. c.p|t.
C||, .|e.e t|e ncJe| p.eserteJ |r t||s c|.pte. Jces seen tc p.cv|Je scne exp|.r.t|crs c| .|y . ccurt.ys
g.c.t| ..te n.y v..y cve. t|ne c. J||e. |.cn .rct|e. ccurt.ys, |e Jces ..gue t|.t c.e.t|rg |rcert|ves |c.
g.c.t| |r pcc. ccurt.|es |s c.uc|.|. Ackrc.|eJg|rg . .c|e |c. p|.|r |uck, |.ste.|y ..gues t|.t gccJ gcve.rnert
pc||c|escres t|.t keep |c. suc| reg.t|ves .s |r|.t|cr, cc..upt|cr, .rJ .eJ t.pe.rJ ,u.||ty |rst|tut|crs
.rJ |..scres t|.t, |c. ex.np|e, |crc. ccrt..cts .rJ .e...J ne.|t.||| |e|p.
|c. tc .ctu.||y |np.cve suc| |rcert|ves n|g|t ccrst|tute t|e rext g.e.t ,uest.
/e |o.e |eoeJ oce oJ |o o|| ||o| ||ee oe o o|co| e||| |o | o |o, eJ| |o
oo oe| |o o.|| |oe||, |oe .|e o|| ||e |o,e | ||e Je.e|oe| oe |o.e
||e ||| |ce||.e || |oe .|e o.ee| |ce||.e |Joce |ec|o|o|co|
oJo|o||o. |||oo|||, |.e|e| | oc||e. oJ |||oo|||, c|oo|| || |oe
.|e Joo |oce |ce||.e ||o| |Joce ||e |o |.e o|J |o coo||e .||| ooJ o||c|e
.|ee o|J .||| |o.e ||| o,o|. o| |o coo||e .||| oo o||c|e .|ee o|J | .o|eJ ||
|oe .|e ||e oo e| ooJ oo|o|||e oJ |ce||.e. .||c| eo|e o.ee|
.e||oe oo ||o| e.oJ o||e ||o eo||e eo| |coe || |oe .|e o||||c
| o| o|o|eJ e|.ee o|oo|||c ||ee| oo |e o|o||o oe o |o| oe J||co||
|o Jec|e ||o ||e o|e |e .o, |o.oJ o| e |o ceo|e |ce||.e |o o.|| |o
||e ||||, o| o.ee|. Joo. oJ |J|.|Joo|' ( 2S990:
CHAP7R 23 CONOMIC GROW7H 591
!385(* /||||o |o|e|,. |e ||o|.e (oe| |o Oo.|| |coo|| ^J.e|oe oJ /|oJ.e|oe | ||e o|c (o|Je /| |e. 2002:
A N S W R 7 O 7 R Y I 7 ! P R O 8 L M
S|tu.t|crs 1 .rJ 4 s|cu|J |e.J tc . s|||t |u.t|e. cut...J |r t|e ccurt.ys p.cJuct|cr pcss|b|||ty cu.ve .rJ |u.
t|e. tc t|e .|g|t |r |ts |crg.ur .gg.eg.te supp|y cu.ve. S|tu.t|crs 2 .rJ 3 s|cu|J |e.J tc sn.||e. cut...J
s|||ts |r t|e ccurt.ys p.cJuct|cr pcss|b|||ty cu.ve .rJ sn.||e. .|g|t...J s|||ts |r |ts |crg.ur .gg.eg.te sup
p|y cu.ve.
4. REVIEW AND PRACTICE
Summary
\e s.. t|.t eccrcn|c g.c.t| c.r be ne.su.eJ by t|e ..te c| |rc.e.se |r pctert|.| cutput. Ve.su.|rg t|e
..te c| |rc.e.se |r .ctu.| .e.| C|| c.r ccr|use g.c.t| st.t|st|cs by |rt.cJuc|rg e|enerts c| cyc||c.| v..|.t|cr.
C.c.t| |s .r expcrert|.| p.ccess. A v..|.b|e |rc.e.s|rg .t . |xeJ pe.cert.ge ..te Jcub|es cve. |xeJ |rte.v.|s.
|e Jcub||rg t|ne |s .pp.cx|n.teJ by t|e .u|e c| 2. |e expcrert|.| r.tu.e c| g.c.t| ne.rs t|.t sn.||
J||e.erces |r g.c.t| ..tes |.ve |..ge e|ects cve. |crg pe.|cJs c| t|ne. |e. c.p|t. ..tes c| |rc.e.se |r .e.| C||
..e |curJ by subt..ct|rg t|e g.c.t| ..te c| t|e pcpu|.t|cr |.cn t|e g.c.t| ..te c| C||.
C.c.t| c.r be s|c.r |r t|e ncJe| c| .gg.eg.te Jen.rJ .rJ .gg.eg.te supp|y .s . se.|es c| .|g|t...J s|||ts
|r t|e |crg.ur .gg.eg.te supp|y cu.ve. |e pcs|t|cr c| t|e ||^. |s Jete.n|reJ by t|e .gg.eg.te p.cJuct|cr
|urct|cr .rJ by t|e Jen.rJ .rJ supp|y cu.ves |c. |.bc.. A .|g|t...J s|||t |r ||^. .esu|ts e|t|e. |.cn .r up
...J s|||t |r t|e p.cJuct|cr |urct|cr, Jue tc |rc.e.ses |r |.ctc.s c| p.cJuct|cr ct|e. t|.r |.bc. c. tc |np.cve
nerts |r tec|rc|cgy, c. |.cn .r |rc.e.se |r t|e Jen.rJ |c. c. t|e supp|y c| |.bc..
S.v|rg p|.ys .r |npc.t.rt .c|e |r eccrcn|c g.c.t|, bec.use |t .||c.s |c. nc.e c.p|t.| tc be .v.||.b|e |c. |u
tu.e p.cJuct|cr, sc t|e ..te c| eccrcn|c g.c.t| c.r .|se. S.v|rg t|us p.cnctes g.c.t|.
|r .ecert ye..s, ..tes c| g.c.t| .ncrg t|e .c.|Js |rJust.|.||.eJ ccurt.|es |.ve g.c.r nc.e J|sp...te. |ecert
.ese..c| suggests t||s n.y be .e|.teJ tc J||e.|rg |.bc. .rJ p.cJuct n..ket ccrJ|t|crs, J||e.erces |r t|e
J||us|cr c| |r|c.n.t|cr .rJ ccnnur|c.t|crs tec|rc|cg|es, .s .e|| .s J||e.erces |r n.c.ceccrcn|c .rJ
t..Je pc||c|es. |v|Jerce cr t|e .c|e t|.t gcve.rnert p|.ys |r eccrcn|c g.c.t| ..s |ess ccrc|us|ve.
592 PRINCIPLS OI CONOMICS
C O N C P 7 P R O 8 L M S
1. Suppcse t|e pecp|e |r . ce.t.|r eccrcny Jec|Je tc stcp s.v|rg .rJ |rste.J use .|| t|e|. |rccne |c.
ccrsunpt|cr. |ey Jc rct||rg tc .JJ tc t|e|. stcck c| |un.r c. p|ys|c.| c.p|t.|. ||scuss t|e p.cspects
|c. g.c.t| c| suc| .r eccrcny.
2. S|rg.pc.e |.s . s.v|rg ..te t|.t |s .cug||y t|.ee t|nes g.e.te. t|.r t|.t c| t|e |r|teJ St.tes. |ts g.e.te.
s.v|rg ..te |.s beer cre .e.scr .|y t|e S|rg.pc.e eccrcny |.s g.c.r |.ste. t|.r t|e |.S. eccrcny.
Suppcse t|.t || t|e |r|teJ St.tes |rc.e.seJ |ts s.v|rg ..te tc, s.y, t.|ce t|e S|rg.pc.e |eve|, |.S. g.c.t|
.cu|J su.p.ss t|e S|rg.pc.e ..te. \cu|J t|.t be . gccJ |Je.`
3. Suppcse .r |rc.e.se |r .|. pc||ut|cr c.uses c.p|t.| tc .e.. cut nc.e ..p|J|y, Jcub||rg t|e ..te c|
Jep.ec|.t|cr. |c. .cu|J t||s .|ect eccrcn|c g.c.t|`
4. Scne pecp|e .c..y t|.t |rc.e.ses |r t|e c.p|t.| stcck .||| b.|rg .bcut .r eccrcny |r .||c| eve.yt||rg |s
Jcre by n.c||res, .|t| rc cbs |e|t |c. pecp|e. \|.t Jces t|e ncJe| c| eccrcn|c g.c.t| p.eserteJ |r
t||s c|.pte. p.eJ|ct`
5. ||r.s .rru.| ..te c| pcpu|.t|cr g.c.t| ..s 1.2 |.cn 195 tc 2003 .rJ |s expecteJ tc be 0.6 |.cn
2003 t|.cug| 2015. |c. Jc ycu t||rk t||s .||| .|ect t|e ..te c| |rc.e.se |r .e.| C||` |c. .||| t||s .|ect
t|e ..te c| |rc.e.se |r pe. c.p|t. .e.| C||`
6. Suppcse tec|rc|cgy stcps c|.rg|rg. |xp|.|r t|e |np.ct cr eccrcn|c g.c.t|.
. Suppcse . se.|es c| te..c.|st .tt.cks Jest.cys |.|| t|e c.p|t.| |r t|e |r|teJ St.tes but Jces rct .|ect t|e
pcpu|.t|cr. \|.t .||| |.pper tc pctert|.| cutput .rJ tc t|e .e.| ..ge`
8. C|ver t|e ..te .t .||c| sc|ert|sts ..e n.k|rg re. J|sccve.|es, .e .||| sccr .e.c| t|e pc|rt t|.t rc
|u.t|e. J|sccve.|es c.r be n.Je. |ccrcn|c g.c.t| .||| ccne tc . stcp. ||scuss.
9. Suppcse .e.| C|| |rc.e.ses Ju.|rg |.es|Jert Ob.n.s te.n |r c|ce .t . 5 ..te. \cu|J t|.t |np|y t|.t
||s pc||c|es .e.e success|u| |r g.c.|rg t|e eccrcny`
10. Suppcse t|.t |c. scne ccurt.y |t ..s |curJ t|.t |ts eccrcn|c g.c.t| ..s b.seJ .|ncst ert|.e|y cr
|rc.e.ses |r ,u.rt|t|es c| |.ctc.s c| p.cJuct|cr. \|y n|g|t suc| g.c.t| be J||cu|t tc sust.|r`
CHAP7R 23 CONOMIC GROW7H 593
N U M R I C A L P R O 8 L M S
1. |e pcpu|.t|cr c| t|e .c.|J |r 2003 ..s 6.314 b||||cr. |t g.e. bet.eer 195 .rJ 2003 .t .r .rru.| ..te c|
1.6. Assune t|.t |t ccrt|rues tc g.c. .t t||s ..te.
.. cnpute t|e Jcub||rg t|ne.
b. |st|n.te t|e .c.|J pcpu|.t|cr |r 2048 .rJ 2093 (.ssun|rg .|| ct|e. t||rgs .en.|r urc|.rgeJ).
2. \|t| . .c.|J pcpu|.t|cr |r 2003 c| 6.314 b||||cr .rJ . p.cecteJ pcpu|.t|cr g.c.t| ..te c| 1.1 |rste.J
(.||c| |s t|e |r|teJ |.t|crs p.cect|cr |c. t|e pe.|cJ 2003 tc 2015).
.. cnpute t|e Jcub||rg t|ne.
b. St.te t|e ye.. |r .||c| t|e .c.|Js pcpu|.t|cr .cu|J be 12.628 b||||cr.
3. Suppcse . ccurt.ys pcpu|.t|cr g.c.s .t t|e ..te c| 2 pe. ye.. .rJ |ts cutput g.c.s .t t|e ..te c| 3 pe.
ye...
.. .|cu|.te |ts ..te c| g.c.t| c| pe. c.p|t. cutput.
b. || |rste.J |ts pcpu|.t|cr g.c.s .t 3 pe. ye.. .rJ |ts cutput g.c.s .t 2 pe. ye.., c.|cu|.te |ts
..te c| g.c.t| c| pe. c.p|t. cutput.
4. |e ..te c| eccrcn|c g.c.t| pe. c.p|t. |r |..rce |.cn 1996 tc 2000 ..s 1.9 pe. ye.., .|||e |r |c.e.
cve. t|e s.ne pe.|cJ |t ..s 4.2. |e. c.p|t. .e.| C|| ..s 28,900 |r |..rce |r 2003, .rJ 12,00 |r |c.e..
Assune t|e g.c.t| ..tes |c. e.c| ccurt.y .en.|r t|e s.ne.
.. cnpute t|e Jcub||rg t|ne |c. |..rces pe. c.p|t. .e.| C||.
b. cnpute t|e Jcub||rg t|ne |c. |c.e.s pe. c.p|t. .e.| C||.
c. \|.t .||| |..rces pe. c.p|t. .e.| C|| be |r 2045`
J. \|.t .||| |c.e.s pe. c.p|t. .e.| C|| be |r 2045`
5. Suppcse .e.| C||s |r ccurt.y A .rJ ccurt.y b ..e |Jert|c.| .t 10 t.||||cr Jc||..s |r 2005. Suppcse ccurt.y
As eccrcn|c g.c.t| ..te |s 2 .rJ ccurt.y bs |s 4 .rJ bct| g.c.t| ..tes .en.|r ccrst.rt cve. t|ne.
.. Or . g..p|, s|c. ccurt.y As pctert|.| cutput urt|| 2025.
b. Or t|e s.ne g..p|, s|c. ccurt.y bs pctert|.| cutput.
c. .|cu|.te t|e pe.cert.ge J||e.erce |r t|e|. |eve|s c| pctert|.| cutput |r 2025.
Suppcse ccurt.y As pcpu|.t|cr g.c.s 1 pe. ye.. .rJ ccurt.y bs pcpu|.t|cr g.c.s 3 pe. ye...
J. Or . g..p|, s|c. ccurt.y As pctert|.| cutput pe. c.p|t. |r 2025.
e. Or t|e s.ne g..p|, s|c. ccurt.y bs pctert|.| cutput pe. c.p|t. |r 2025.
|. .|cu|.te t|e pe.cert.ge J||e.erce |r t|e|. |eve|s c| pctert|.| cutput pe. c.p|t. |r 2025.
6. .c ccurt.|es, A .rJ b, |.ve |Jert|c.| |eve|s c| .e.| C|| pe. c.p|t.. |r curt.y A, .r |rc.e.se |r t|e c.p|t.|
stcck |rc.e.ses t|e pctert|.| cutput by 10. curt.y b .|sc expe.|erces . 10 |rc.e.se |r |ts pctert|.|
cutput, but t||s |rc.e.se |s t|e .esu|t c| .r |rc.e.se |r |ts |.bc. |c.ce. |s|rg .gg.eg.te p.cJuct|cr
|urct|crs .rJ |.bc.n..ket .r.|yses |c. t|e t.c ccurt.|es, |||ust..te .rJ exp|.|r |c. t|ese everts ..e
||ke|y tc .|ect ||v|rg st.rJ..Js |r t|e t.c eccrcn|es.
. Suppcse t|e |r|c.n.t|cr be|c. c|...cte.|.es .r eccrcny.
mpIoyment (in miIIions) ReaI GDP (in biIIions)
1 200
2 00
3 1,100
4 1,400
5 1,650
6 1,850
2,000
8 2,100
9 2,10
10 2,200
.. crst.uct t|e .gg.eg.te p.cJuct|cr |urct|cr |c. t||s eccrcny.
b. \|.t k|rJ c| .etu.rs Jces t||s eccrcny expe.|erce` |c. Jc ycu krc.`
c. Assun|rg t|.t tct.| .v.||.b|e enp|cynert |s n||||cr, J... t|e eccrcnys |crg.ur .gg.eg.te
supp|y cu.ve.
594 PRINCIPLS OI CONOMICS
Suppcse t|.t |np.cvenert |r tec|rc|cgy ne.rs t|.t .e.| C|| .t e.c| |eve| c| enp|cynert .|ses by 200
b||||cr.
J. crst.uct t|e re. .gg.eg.te p.cJuct|cr |urct|cr |c. t||s eccrcny.
e. crst.uct t|e re. |crg.ur .gg.eg.te supp|y cu.ve |c. t|e eccrcny.
8. |r .b|e 23.1, .e c.r see t|.t '.p.rs g.c.t| ..te c| pe. c.p|t. .e.| C|| |e|| |.cn 3.3 pe. ye.. |r t|e
1980s tc 1.4 pe. ye.. |r t|e 1990s.
.. cnp..e t|e pe.cert |rc.e.se |r |ts pe. c.p|t. .e.| C|| cve. t|e 20ye.. pe.|cJ tc .|.t |t .cu|J
|.ve beer || |t |.J n.|rt.|reJ t|e 3.3 pe. c.p|t. g.c.t| ..te c| t|e 1980s.
b. '.p.rs pe. c.p|t. C|| |r 1980 ..s .bcut 24,000 (|r |.S. 2000 Jc||..s) |r 1980. .|cu|.te .|.t |t
.cu|J |.ve beer || t|e g.c.t| ..te c| t|e 1980s |.J beer n.|rt.|reJ .|cu|.te .bcut |c.
nuc| |t |s, g|ver t|e .ctu.| g.c.t| ..tes cve. t|e t.c Jec.Jes.
c. |r .b|e 23.1, .e c.r see t|.t |.e|.rJs g.c.t| ..te c| pe. c.p|t. .e.| C|| g.e. |.cn 3.0 pe.
ye.. |r t|e 1980s tc 6.4 pe. ye.. |r t|e 1990s.
J. cnp..e t|e pe.cert |rc.e.se |r |ts pe. c.p|t. .e.| C|| cve. t|e 20ye.. pe.|cJ tc .|.t |t .cu|J
|.ve beer || |t |.J n.|rt.|reJ t|e 3.0 pe. c.p|t. g.c.t| ..te c| t|e 1980s.
e. |.e|.rJs pe. c.p|t. C|| |r 1980 ..s .bcut 10,000 (|r |.S. 2000 Jc||..s). .|cu|.te .|.t |t .cu|J
|.ve beer || t|e g.c.t| ..te c| t|e 1980s |.J beer n.|rt.|reJ. .|cu|.te .bcut |c. nuc| |t |s,
g|ver t|e .ctu.| g.c.t| ..tes cve. t|e t.c Jec.Jes.
CHAP7R 23 CONOMIC GROW7H 595
1.
2.
3.
ENDNOTES
|ct|ce t|e use c| t|e .c.Js oo||, .rJ oo|o|e|,. |e .ctu.| v.|ue c| .r |r
ccne c| 1,000 g.c.|rg .t ..te |c. . pe.|cJ c| ye..s |s 1,000 (1 + )
r
. A|te. 8
ye..s c| g.c.t| .t . 9 ..te, |rccne .cu|J t|us be 1,000 (1 + 0.09)
8
1,992.56.
|e .u|e c| 2 p.eJ|cts t|.t |ts v.|ue .||| be 2,000. |e .u|e c| 2 g|ves .r .pp.cx|n
.t|cr, rct .r ex.ct ne.su.e, c| t|e |np.ct c| expcrert|.| g.c.t|.
|e n.te.|.| |r t||s sect|cr |s b.seJ cr O.g.r|..t|cr |c. |ccrcn|c ccpe..t|cr .rJ
|eve|cpnert, |e .ooce o| |coo|c Oo.|| | || oo||e, 2003.
|xc|uJes .ec| |epub||c, |urg..y, |c.e.r, Vex|cc, |c|.rJ, .rJ S|cv.k |epub||c
596 PRINCIPLS OI CONOMICS
| A | | | 2 4
The Nature and Creation of
Money
S7AR7 UP: HOW MANY MACkS DOS I7 COS71
|...y |ev|re |e|peJ . c||ert p.ep..e J|vc.ce p.pe.s . |e. ye..s .gc. |e ..s p.|J |r n.cke.e|.
|ts t|e cc|r c| t|e .e.|n, ||s c||ert, V..k b.||ey tc|J t|e /o|| .|ee| .ooo|. |e t.c ner .e.e p.|scre.s .t t|e
t|ne .t t|e |eJe..| per|tert|..y .t |cnpcc, .|||c.r|..
by t|e t|ne ||s .c.k cr t|e c.se ..s ccnp|eteJ, |e |.J .ccunu|.teJ . st.ck c| n.cks, V.. |ev|re s.|J. |e
useJ ||s |s|y |c..J tc buy |tens suc| .s |.|.cuts .t t|e p.|scr b..be. s|cp, tc |.ve ||s |.urJ.y p.esseJ, c. tc |.ve
||s ce|| c|e.reJ.
|e scne.|.t urp|e.s.rt |s| ene.geJ .s t|e cu..ercy c| c|c|ce |r n.ry |eJe..| p.|scrs |r 1994 .|er c|g..
ettes, t|e p.ev|cus ccnncJ|ty useJ .s cu..ercy, .e.e b.rreJ. ||.st|c b.gs c| n.cke.e| sc|J |c. .bcut 1 |r p.|scr
ccnn|ss..|es. A|ncst rc cre ||kes t|en, sc t|e p.|scr ncrey supp|y J|J rct get e.ter. |.|scre.s kre. ct|e. p.|s
cre.s .cu|J .e.J||y .ccept n.cks, sc t|ey .e.e .ccepteJ |r exc|.rge |c. gccJs .rJ se.v|ces. |e|. 1 p.|ce n.Je
t|en ccrver|ert .s . ur|t c| .cccurt. ArJ, .s V.. |ev|res expe.|erce suggests, t|ey .cteJ .s . stc.e c| v.|ue. As
.e s|.|| see, n.cks se.veJ .|| t|.ee |urct|crs c| ncreyt|ey .e.e . neJ|un c| exc|.rge, . ur|t c| .cccurt, .rJ
. stc.e c| v.|ue.
1
|r t||s c|.pte. .rJ t|e rext .e ex.n|re ncrey .rJ t|e ..y |t .|ects t|e |eve| c| .e.| C|| .rJ t|e p.|ce |eve|.
|r t||s c|.pte., .e .||| |ccus cr t|e r.tu.e c| ncrey .rJ t|e p.ccess t|.cug| .||c| |t |s c.e.teJ.
As t|e expe.|erce c| t|e p.|scre.s |r |cnpcc suggests, v|.tu.||y .ryt||rg c.r se.ve .s ncrey. ||stc.|c.||y, s.|t,
|c.ses, tcb.ccc, c|g..ettes, gc|J, .rJ s||ve. |.ve .|| se.veJ .s ncrey. \e s|.|| ex.n|re t|e c|...cte.|st|cs t|.t
Je|re . gccJ .s ncrey.
\e .||| .|sc |rt.cJuce t|e |..gest |r.rc|.| |rst|tut|cr |r t|e .c.|J, t|e |eJe..| |ese.ve Systen c| t|e |r|teJ
St.tes. |e |eJ, .s |t |s ccnncr|y c.||eJ, p|.ys . key .c|e |r Jete.n|r|rg t|e ,u.rt|ty c| ncrey |r t|e |r|teJ St.tes.
\e .||| see |c. t|e |eJ cpe..tes .rJ |c. |t .ttenpts tc ccrt.c| t|e supp|y c| ncrey.
money
Aryt||rg t|.t se.ves .s .
neJ|un c| exc|.rge.
medium of exchange
Aryt||rg t|.t |s .|Je|y
.ccepteJ .s . ne.rs c|
p.ynert.
barter
\|er gccJs ..e exc|.rgeJ
J|.ect|y |c. ct|e. gccJs.
unit of account
A ccrs|stert ne.rs c|
ne.su.|rg t|e v.|ue c|
t||rgs.
1. WHAT IS MONEY:
L A R N I N G O 8 1 C 7 I V S
1. Dene money and discuss its three basic functions.
2. Distinguish between commodity money and at money, giving exampIes of each.
3. Dene what is meant by the money suppIy and teII what is incIuded in the IederaI Reserve Sys-
tem's two denitions of it (M1 and M2).
If cigaiettes and mackeiel can be used as money, then just what is money: Money is anything that
seives as a medium of exchange. A medium of exchange is anything that is widely accepted as a
means of payment. In Romania undei Communist Paity iule in the 1980s, foi example, Kent cigaiettes
seived as a medium of exchange; the fact that they could be exchanged foi othei goods and seivices
made them money.
Money, ultimately, is defned by people and what they do. When people use something as a medi-
um of exchange, it becomes money. If people weie to begin accepting basketballs as payment foi most
goods and seivices, basketballs would be money. We will leain in this chaptei that changes in the way
people use money have cieated new types of money and changed the way money is measuied in iecent
decades.
1.1 The Functions of Money
Money seives thiee basic functions. By defnition, it is a medium of exchange. It also seives as a unit of
account and as a stoie of value-as the mack" did in Lompoc.
A Medium of xchange
The exchange of goods and seivices in maikets is among the most univeisal activities of human life. To
facilitate these exchanges, people settle on something that will seive as a medium of exchange-they se-
lect something to be money.
We can undeistand the signifcance of a medium of exchange by consideiing its absence. Barter
occuis when goods aie exchanged diiectly foi othei goods. Because no one item seives as a medium of
exchange in a baitei economy, potential buyeis must fnd things that individual selleis will accept. A
buyei might fnd a sellei who will tiade a paii of shoes foi two chickens. Anothei sellei might be willing
to piovide a haiicut in exchange foi a gaiden hose. Suppose you weie visiting a gioceiy stoie in a baitei
economy. You would need to load up a tiuckful of items the giocei might accept in exchange foi gio-
ceiies. That would be an unceitain afaii; you could not know when you headed foi the stoie which
items the giocei might agiee to tiade. Indeed, the complexity-and cost-of a visit to a gioceiy stoie in
a baitei economy would be so gieat that theie piobably would not be any gioceiy stoies! A moment's
contemplation of the dimculty of life in a baitei economy will demonstiate why human societies in-
vaiiably select something-sometimes moie than one thing-to seive as a medium of exchange, just as
piisoneis in fedeial penitentiaiies accepted mackeiel.
A Unit of Account
Ask someone in the United States what he oi she paid foi something, and that peison will iespond by
quoting a piice stated in dollais: I paid $73 foi this iadio," oi I paid $13 foi this pizza." People do not
say, I paid fve pizzas foi this iadio." That statement might, of couise, be liteially tiue in the sense of
the oppoitunity cost of the tiansaction, but we do not iepoit piices that way foi two ieasons. One is
that people do not aiiive at places like Radio Shack with fve pizzas and expect to puichase a iadio. The
othei is that the infoimation would not be veiy useful. Othei people may not think of values in pizza
teims, so they might not know what we meant. Instead, we iepoit the value of things in teims of
money.
Money seives as a unit of account, which is a consistent means of measuiing the value of things.
We use money in this fashion because it is also a medium of exchange. When we iepoit the value of a
good oi seivice in units of money, we aie iepoiting what anothei peison is likely to have to pay to ob-
tain that good oi seivice.
598 PRINCIPLS OI CONOMICS
store of vaIue
Ar |ten t|.t |c|Js v.|ue cve.
t|ne.
commodity money
Vcrey t|.t |.s v.|ue .p..t
|.cn |ts use .s ncrey.
at money
Vcrey t|.t scne .ut|c.|ty,
gere..||y . gcve.rnert, |.s
c.Je.eJ tc be .ccepteJ .s .
neJ|un c| exc|.rge.
currency
|.pe. ncrey .rJ cc|rs.
A Store of VaIue
The thiid function of money is to seive as a store of value, that is, an item that holds value ovei time.
Considei a $20 bill that you accidentally left in a coat pocket a yeai ago. When you fnd it, you will be
pleased. That is because you know the bill still has value. Value has, in efect, been stoied" in that little
piece of papei.
Money, of couise, is not the only thing that stoies value. Houses, omce buildings, land, woiks of
ait, and many othei commodities seive as a means of stoiing wealth and value. Money difeis fiom
these othei stoies of value by being ieadily exchangeable foi othei commodities. Its iole as a medium
of exchange makes it a convenient stoie of value.
Because money acts as a stoie of value, it can be used as a standaid foi futuie payments. When you
boiiow money, foi example, you typically sign a contiact pledging to make a seiies of futuie payments
to settle the debt. These payments will be made using money, because money acts as a stoie of value.
Money is not a iisk-fiee stoie of value, howevei. We saw in the chaptei that intioduced the
concept of infation that infation ieduces the value of money. In peiiods of iapid infation, people may
not want to iely on money as a stoie of value, and they may tuin to commodities such as land oi gold
instead.
1.2 Types of Money
Although money can take an extiaoidinaiy vaiiety of foims, theie aie ieally only two types of money:
money that has intiinsic value and money that does not have intiinsic value.
Commodity money is money that has value apait fiom its use as money. Mackeiel in fedeial
piisons is an example of commodity money. Mackeiel could be used to buy seivices fiom othei piison-
eis; they could also be eaten.
Gold and silvei aie the most widely used foims of commodity money. Gold and silvei can be used
as jeweliy and foi some industiial and medicinal puiposes, so they have value apait fiom theii use as
money. The fist known use of gold and silvei coins was in the Gieek city-state of Lydia in the begin-
ning of the seventh centuiy B.C. The coins weie fashioned fiom electium, a natuial mixtuie of gold
and silvei.
One disadvantage of commodity money is that its quantity can fuctuate eiiatically. Gold, foi ex-
ample, was one foim of money in the United States in the 19th centuiy. Gold discoveiies in Califoinia
and latei in Alaska sent the quantity of money soaiing. Some of this nation's woist bouts of infation
weie set of by incieases in the quantity of gold in ciiculation duiing the 19th centuiy. A much gieatei
pioblem exists with commodity money that can be pioduced. In the southein pait of colonial Ameiica,
foi example, tobacco seived as money. Theie was a continuing pioblem of faimeis incieasing the
quantity of money by giowing moie tobacco. The pioblem was sumciently seiious that vigilante squads
weie oiganized. They ioamed the countiyside buining tobacco felds in an efoit to keep the quantity
of tobacco, hence money, undei contiol. (Remaikably, these squads sought to contiol the money sup-
ply by buining tobacco giown by othei faimeis.)
Anothei pioblem is that commodity money may vaiy in quality. Given that vaiiability, theie is a
tendency foi lowei-quality commodities to diive highei-quality commodities out of ciiculation.
Hoises, foi example, seived as money in colonial New England. It was common foi loan obligations to
be stated in teims of a quantity of hoises to be paid back. Given such obligations, theie was a tendency
to use lowei-quality hoises to pay back debts; highei-quality hoises weie kept out of ciiculation foi
othei uses. Laws weie passed foibidding the use of lame hoises in the payment of debts. This is an ex-
ample of Giesham's law: the tendency foi a lowei-quality commodity (bad money) to diive a highei-
quality commodity (good money) out of ciiculation. Unless a means can be found to contiol the qual-
ity of commodity money, the tendency foi that quality to decline can thieaten its acceptability as a me-
dium of exchange.
But something need not have intiinsic value to seive as money. Fiat money is money that some
authoiity, geneially a goveinment, has oideied to be accepted as a medium of exchange. The cur-
rency-papei money and coins-used in the United States today is fat money; it has no value othei
than its use as money. You will notice that statement piinted on each bill: This note is legal tendei foi
all debts, public and piivate."
CHAP7R 24 7H NA7UR AND CRA7ION OI MONY 599
checkabIe deposits
b.|.rces |r c|eck|rg
.cccurts.
check
A ..|tter c.Je. tc . b.rk tc
t..rs|e. c.re.s||p c| .
c|eck.b|e Jepcs|t.
money suppIy
|e tct.| ,u.rt|ty c| ncrey
|r t|e eccrcny .t .ry cre
t|ne.
Iiquidity
|e e.se .|t| .||c| .r .sset
c.r be ccrve.teJ |rtc
cu..ercy.
Checkable deposits, which aie balances in checking accounts, and tiavelei's checks aie othei
foims of money that have no intiinsic value. They can be conveited to cuiiency, but geneially they aie
not; they simply seive as a medium of exchange. If you want to buy something, you can often pay with
a check oi a debit caid. A check is a wiitten oidei to a bank to tiansfei owneiship of a checkable de-
posit. A debit caid is the electionic equivalent of a check. Suppose, foi example, that you have $100 in
youi checking account and you wiite a check to youi campus bookstoie foi $30 oi instiuct the cleik to
swipe youi debit caid and chaige" it $30. In eithei case, $30 will be tiansfeiied fiom youi checking ac-
count to the bookstoie's checking account. ^otice that it is the checkable deposit, not the check or debit
card, that is money. The check oi debit caid just tells a bank to tiansfei money, in this case checkable
deposits, fiom one account to anothei.
What makes something money is ieally found in its acceptability, not in whethei oi not it has in-
tiinsic value oi whethei oi not a goveinment has declaied it as such. Foi example, fat money tends to
be accepted so long as too much of it is not piinted too quickly. When that happens, as it did in Russia
in the 1990s, people tend to look foi othei items to seive as money. In the case of Russia, the U.S. dollai
became a populai foim of money, even though the Russian goveinment still declaied the iuble to be its
fat money.
Heads Up!
|e te.n oe,, .s useJ by eccrcn|sts .rJ t|.cug|cut t||s bcck, |.s t|e ve.y spec||c Je|r|t|cr g|ver |r t|e
text. |ecp|e c.r |c|J .ssets |r . v..|ety c| |c.ns, |.cn .c.ks c| ..t tc stcck ce.t||c.tes tc cu..ercy c. c|eck|rg
.cccurt b.|.rces. |ver t|cug| |rJ|v|Ju.|s n.y be ve.y .e.|t|y, cr|y .|er t|ey ..e |c|J|rg t|e|. .ssets |r .
|c.n t|.t se.ves .s . neJ|un c| exc|.rge Jc t|ey, .ccc.J|rg tc t|e p.ec|se ne.r|rg c| t|e te.n, |.ve
ncrey. c ,u.|||y .s ncrey, scnet||rg nust be .|Je|y .ccepteJ .s . neJ|un c| exc|.rge.
1.3 Measuiing Money
The total quantity of money in the economy at any one time is called the money supply. Economists
measuie the money supply because it afects economic activity. What should be included in the money
supply: We want to include as pait of the money supply those things that seive as media of exchange.
Howevei, the items that piovide this function have vaiied ovei time.
Befoie 1980, the basic money supply was measuied as the sum of cuiiency in ciiculation, tiavelei's
checks, and checkable deposits. Cuiiency seives the medium-of-exchange function veiy nicely but
denies people any inteiest eainings. (Checking accounts did not eain inteiest befoie 1980.)
Ovei the last few decades, especially as a iesult of high inteiest iates and high infation in the late
1970s, people sought and found ways of holding theii fnancial assets in ways that eain inteiest and
that can easily be conveited to money. Foi example, it is now possible to tiansfei money fiom youi sav-
ings account to youi checking account using an automated tellei machine (ATM), and then to with-
diaw cash fiom youi checking account. Thus, many types of savings accounts aie easily conveited into
cuiiency.
Economists iefei to the ease with which an asset can be conveited into cuiiency as the asset's li-
quidity. Cuiiency itself is peifectly liquid; you can always change two $3 bills foi a $10 bill. Checkable
deposits aie almost peifectly liquid; you can easily cash a check oi visit an ATM. An omce building,
howevei, is highly illiquid. It can be conveited to money only by selling it, a time-consuming and costly
piocess.
As fnancial assets othei than checkable deposits have become moie liquid, economists have had to
develop bioadei measuies of money that would coiiespond to economic activity. In the United States,
the fnal aibitei of what is and what is not measuied as money is the Fedeial Reseive System. Because it
is dimcult to deteimine what (and what not) to measuie as money, the Fed iepoits seveial difeient
measuies of money, including M1 and M2.
600 PRINCIPLS OI CONOMICS
M1
|e r...c.est c| t|e |eJs
ncrey supp|y Je|r|t|crs
t|.t |rc|uJes cu..ercy |r
c|.cu|.t|cr, c|eck.b|e
Jepcs|ts, .rJ t..ve|e.s
c|ecks.
M2
A b.c.Je. ne.su.e c| t|e
ncrey supp|y t|.r V1 t|.t
|rc|uJes V1 .rJ ct|e.
Jepcs|ts.
M1 is the naiiowest of the Fed's money supply defnitions. It includes cuiiency in ciiculation,
checkable deposits, and tiavelei's checks. M2 is a bioadei measuie of the money supply than M1. It in-
cludes M1 and othei deposits such as small savings accounts (less than $100,000), as well as accounts
such as money maiket mutual funds (MMMFs) that place limits on the numbei oi the amounts of the
checks that can be wiitten in a ceitain peiiod.
M2 is sometimes called the bioadly defned money supply, while M1 is the naiiowly defned
money supply. The assets in M1 may be iegaided as peifectly liquid; the assets in M2 aie highly liquid,
but somewhat less liquid than the assets in M1. Even bioadei measuies of the money supply include
laige time-deposits, money maiket mutual funds held by institutions, and othei assets that aie some-
what less liquid than those in M2. Figuie 24.1 shows the composition of M1 and M2 in Decembei 2008.
II GUR 24. 1 7he 7wo Ms: December 2008
V1, t|e r...c.est Je|r|t|cr c| t|e ncrey supp|y, |rc|uJes .ssets t|.t ..e pe.|ect|y ||,u|J. V2 p.cv|Jes . b.c.Je.
ne.su.e c| t|e ncrey supp|y .rJ |rc|uJes scne.|.t |ess ||,u|J .ssets. Ancurts .ep.esert ncrey supp|y J.t. |r
b||||crs c| Jc||..s |c. |ecenbe. 2008.
Source Federal Reserve Statistical Release H., Tables 3 and 4 (january 29, 2009).
Heads Up!
.eJ|t c..Js ..e rct ncrey. A c.eJ|t c..J |Jert||es ycu .s . pe.scr .|c |.s . spec|.| ....rgenert .|t| t|e
c..J |ssue. |r .||c| t|e |ssue. .||| |erJ ycu ncrey .rJ t..rs|e. t|e p.cceeJs tc .rct|e. p..ty .|ereve. ycu
..rt. |us, || ycu p.esert . V.ste...J tc . e.e|e. .s p.ynert |c. . 500 .|rg, t|e |.n t|.t |ssueJ ycu t|e
c..J .||| |erJ ycu t|e 500 .rJ serJ t|.t ncrey, |ess . se.v|ce c|..ge, tc t|e e.e|e.. +cu, c| ccu.se, .||| be
.e,u|.eJ tc .ep.y t|e |c.r |.te.. but . c..J t|.t s.ys ycu |.ve suc| . .e|.t|crs||p |s rct ncrey, ust .s ycu.
Jeb|t c..J |s rct ncrey.
With all the opeiational defnitions of money available, which one should we use: Economists genei-
ally answei that question by asking anothei: Which measuie of money is most closely ielated to ieal
GDP and the piice level: As that changes, so must the defnition of money.
In 1980, the Fed decided that changes in the ways people weie managing theii money made M1
useless foi policy choices. Indeed, the Fed now pays little attention to M2 eithei. It has laigely given up
tiacking a paiticulai measuie of the money supply. The choice of what to measuie as money iemains
the subject of continuing ieseaich and consideiable debate.
CHAP7R 24 7H NA7UR AND CRA7ION OI MONY 601
k Y 7 A k A W A Y S
< Vcrey |s .ryt||rg t|.t se.ves .s . neJ|un c| exc|.rge. Ot|e. |urct|crs c| ncrey ..e tc se.ve .s . ur|t c|
.cccurt .rJ .s . stc.e c| v.|ue.
< Vcrey n.y c. n.y rct |.ve |rt.|rs|c v.|ue. cnncJ|ty ncrey |.s |rt.|rs|c v.|ue bec.use |t |.s ct|e.
uses bes|Jes be|rg . neJ|un c| exc|.rge. ||.t ncrey se.ves cr|y .s . neJ|un c| exc|.rge, bec.use |ts
use .s suc| |s .ut|c.|.eJ by t|e gcve.rnert, |t |.s rc |rt.|rs|c v.|ue.
< |e |eJ .epc.ts seve..| J||e.ert ne.su.es c| ncrey, |rc|uJ|rg V1 .rJ V2.
7 R Y I 7 !
\||c| c| t|e |c||c.|rg ..e ncrey |r t|e |r|teJ St.tes tcJ.y .rJ .||c| ..e rct` |xp|.|r ycu. .e.scr|rg |r
te.ns c| t|e |urct|crs c| ncrey.
.. Cc|J
b. A \.r Ccg| p.|rt|rg
c. A J|ne
Case in Point: Iiat-Iess Money
\e Jcrt |.ve . cu..ercy c| cu. c.r, p.cc|.|neJ |e.c||v.r b....r|, t|e |u.J|s| .eg|cr.| gcve.rnerts
p.|ne n|r|ste. |r . re.s |rte.v|e. |r 2003. but, ever .|t|cut c|c|.| .eccgr|t|cr by t|e gcve.rnert, t|e sc
c.||eJ S.|ss J|r.. ce.t.|r|y seeneJ tc |urct|cr .s . |.t ncrey. |e.e |s |c. t|e |u.J|s| ..e. c| rc.t|e.r |..,,
Ju.|rg t|e pe.|cJ bet.eer t|e Cu|| \.. |r 1991 .rJ t|e |.|| c| S.JJ.n |usse|r |r 2003, c.ne tc |.ve |ts c.r
cu..ercy, Jesp|te t|e p.crcurcenert c| |ts p.|ne n|r|ste. tc t|e ccrt...y.
A|te. t|e Cu|| \.., t|e rc.t|e.r, ncst|y |u.J|s| ..e. c| |.., ..s sep...teJ |.cn t|e .est c| |.., t|cug| t|e er
|c.cenert c| t|e rc|y.cre. bec.use c| |r|teJ |.t|crs s.rct|crs t|.t b...eJ t|e S.JJ.n |usse|r .eg|ne |r
t|e scut| |.cn ccrt|ru|rg tc |npc.t cu..ercy |.cn S.|t.e.|.rJ, t|e cert..| b.rk c| |.., .rrcurceJ |t .cu|J
.ep|.ce t|e S.|ss J|r..s, sc r.neJ bec.use t|ey |.J beer p.|rteJ |r S.|t.e.|.rJ, .|t| |cc.||y p.|rteJ cu.
.ercy, .||c| bec.ne krc.r .s S.JJ.n J|r..s. |..,| c|t|.ers |r scut|e.r |.., .e.e g|ver t|.ee .eeks tc ex
c|.rge t|e|. c|J J|r..s |c. t|e re. cres. |r t|e rc.t|e.r p..t c| |..,, c|t|.ers ccu|J rct exc|.rge t|e|. rctes
.rJ sc t|ey s|np|y ccrt|rueJ tc use t|e c|J cres.
ArJ sc |t ..s t|.t t|e S.|ss J|r.. |c. . pe.|cJ c| .bcut 10 ye..s, ever .|t|cut gcve.rnert b.ck|rg c. .ry
|.. est.b||s||rg |t .s |eg.| terJe., se.veJ .s rc.t|e.r |..,s |.t ncrey. |ccrcn|sts use t|e .c.J |o|, .||c|
|r |.t|r ne.rs |et |t be Jcre, tc Jesc.|be ncrey t|.t |.s rc |rt.|rs|c v.|ue. Suc| |c.ns c| ncrey ooo||, get
t|e|. v.|ue bec.use . gcve.rnert c. .ut|c.|ty |.s Jec|..eJ t|en tc be |eg.| terJe., but, .s t||s stc.y s|c.s, |t
Jces rct .e.||y .e,u|.e nuc| |.t |c. . ccrver|ert, |r.rJc||tse|| .c.t||ess, neJ|un c| exc|.rge tc evc|ve.
\|.t |.ppereJ tc bct| t|e S.|ss .rJ S.JJ.n J|r..s` A|te. t|e c.||t|cr |.cv|s|cr.| Aut|c.|ty (|A) .s
suneJ ccrt.c| c| .|| c| |..,, |.u| b.ene., t|er |e.J c| t|e |A, .rrcurceJ t|.t . re. |..,| J|r.. .cu|J be ex
c|.rgeJ |c. bct| c| t|e ex|st|rg cu..erc|es cve. . t|.eencrt| pe.|cJ erJ|rg |r '.ru..y 2004 .t . ..te t|.t
|np||eJ t|.t cre S.|ss J|r.. ..s v.|ueJ .t 150 S.JJ.n J|r..s. bec.use S.JJ.n |usse|rs .eg|ne |.J p.|r
teJ n.ry nc.e S.JJ.n J|r..s cve. t|e 10ye.. pe.|cJ, .|||e rc S.|ss J|r..s |.J beer p.|rteJ, .rJ be
c.use t|e c|e.p p.|rt|rg c| t|e S.JJ.n J|r..s n.Je t|en e.sy tc ccurte.|e|t, cve. t|e Jec.Je t|e S.|ss
602 PRINCIPLS OI CONOMICS
nanciaI intermediary
Ar |rst|tut|cr t|.t .n.sses
|urJs |.cn cre g.cup .rJ
n.kes t|en .v.||.b|e tc
.rct|e..
bank
A |r.rc|.| |rte.neJ|..y t|.t
.ccepts Jepcs|ts, n.kes
|c.rs, .rJ c|e.s c|eck|rg
.cccurts.
J|r..s bec.ne .e|.t|ve|y nc.e v.|u.b|e .rJ t|e exc|.rge ..te t|.t b.ene. c|e.eJ .bcut e,u.||.eJ t|e pu.
c|.s|rg pc.e. c| t|e t.c cu..erc|es. |c. ex.np|e, |t tcck .bcut 133 t|nes .s n.ry S.JJ.n J|r..s .s S.|ss
J|r..s tc buy . n.rs su|t |r |.., .t t|e t|ne. |e re. rctes, scnet|nes c.||eJ b.ene. J|r..s, .e.e p.|rteJ |r
b.|t.|r .rJ e|se.|e.e .rJ |c.r |rtc |.., cr 22 ||g|ts us|rg bce|rg 4s .rJ ct|e. |..ge .|.c..|t. |r bct| t|e
rc.t|e.r .rJ scut|e.r p..ts c| |..,, c|t|.ers tu.reJ |r t|e|. c|J J|r..s |c. t|e re. cres, suggest|rg .t |e.st
nc.e ccr|Jerce .t t|.t ncnert |r t|e b.ene. J|r.. t|.r |r e|t|e. t|e S.JJ.n c. S.|ss J|r..s.
!385(*6 /e., ^ ||. |e ||||o||o o| /oe|o, |o||c,' (|ec|oe. ^e|co |coo|c ^oc|o||o ^oo| /ee||. .o ||eo. .ooo, 4. 2004:.
o.o||o|e o| |||...o|o|e|oJcoo|eec|eeec|20SJ| |o| | vo|o. |oe oec, o |o.e vo|oe .|||oo| Oo.ee| |oc||.
o| .oc| |oc|| ^JJ .o|o||o||, |o || vo|oe.' |e. `o| |e. .ooo, 5. 2004. 2
A N S W R 7 O 7 R Y I 7 ! P R O 8 L M
.. Cc|J |s rct ncrey bec.use |t |s rct useJ .s . neJ|un c| exc|.rge. |r .JJ|t|cr, |t Jces rct se.ve .s . ur|t
c| .cccurt. |t n.y, |c.eve., se.ve .s . stc.e c| v.|ue.
b. A \.r Ccg| p.|rt|rg |s rct ncrey. |t se.ves .s . stc.e c| v.|ue. |t |s ||g||y ||||,u|J but ccu|J evertu.||y be
ccrve.teJ tc ncrey. |t |s re|t|e. . neJ|un c| exc|.rge rc. . ur|t c| .cccurt.
c. A J|ne |s ncrey .rJ se.ves .|| t|.ee |urct|crs c| ncrey. |t |s, c| ccu.se, pe.|ect|y ||,u|J.
2. THE BANKING SYSTEM AND MONEY CREATION
L A R N I N G O 8 1 C 7 I V S
1. xpIain what banks are, what their baIance sheets Iook Iike, and what is meant by a fractionaI
reserve banking system.
2. Describe the process of money creation (destruction), using the concept of the deposit
muItipIier.
3. Describe how and why banks are reguIated and insured.
Wheie does money come fiom: How is its quantity incieased oi decieased: The answei to these ques-
tions suggests that money has an almost magical quality: money is created by banks when they issue
loans. In efect, money is cieated by the stioke of a pen oi the click of a computei key.
We will begin by examining the opeiation of banks and the banking system. We will fnd that, like
money itself, the natuie of banking is expeiiencing iapid change.
2.1 Banks and Othei Financial Inteimediaiies
An institution that amasses funds fiom one gioup and makes them available to anothei is called a nn-
ancial intermediary. A pension fund is an example of a fnancial inteimediaiy. Woikeis and fims
place eainings in the fund foi theii ietiiement; the fund eains income by lending money to fims oi by
puichasing theii stock. The fund thus makes ietiiement saving available foi othei spending. Insuiance
companies aie also fnancial inteimediaiies, because they lend some of the piemiums paid by theii cus-
tomeis to fims foi investment. Mutual funds make money available to fims and othei institutions by
puichasing theii initial ofeiings of stocks oi bonds.
Banks play a paiticulaily impoitant iole as fnancial inteimediaiies. Banks accept depositois'
money and lend it to boiioweis. With the inteiest they eain on theii loans, banks aie able to pay in-
teiest to theii depositois, covei theii own opeiating costs, and eain a pioft, all the while maintaining
the ability of the oiiginal depositois to spend the funds when they desiie to do so. One key chaiacteiist-
ic of banks is that they ofei theii customeis the oppoitunity to open checking accounts, thus cieating
checkable deposits. These functions defne a bank, which is a fnancial inteimediaiy that accepts de-
posits, makes loans, and ofeis checking accounts.
Ovei time, some nonbank fnancial inteimediaiies have become moie and moie like banks. Foi
example, some biokeiage fims ofei customeis inteiest-eaining accounts and make loans. They now
allow theii customeis to wiite checks on theii accounts.
CHAP7R 24 7H NA7UR AND CRA7ION OI MONY 603
baIance sheet
A |r.rc|.| st.tenert
s|c.|rg .ssets, ||.b|||t|es, .rJ
ret .c.t|.
assets
Aryt||rg c| v.|ue.
IiabiIities
Ob||g.t|crs tc ct|e. p..t|es.
net worth
Assets |ess ||.b|||t|es.
reserves
b.rk .ssets |e|J .s c.s| |r
v.u|ts .rJ |r Jepcs|ts .|t|
t|e |eJe..| |ese.ve.
fractionaI reserve banking
system
Systen |r .||c| b.rks |c|J
.ese.ves .|cse v.|ue |s |ess
t|.r t|e sun c| c|.|ns
cutst.rJ|rg cr t|cse
.ese.ves.
As nonbank fnancial inteimediaiies have giown, banks' shaie of the nation's ciedit maiket fnan-
cial assets has diminished. In 1972, banks accounted foi neaily 30 of U.S. ciedit maiket fnancial as-
sets. In 2007, that shaie had diopped to about 13.
The fact that banks account foi a declining shaie of U.S. fnancial assets alaims some obseiveis.
We will see that banks aie moie tightly iegulated than aie othei fnancial institutions; one ieason foi
that iegulation is to maintain contiol ovei the money supply. Othei fnancial inteimediaiies do not
face the same iegulatoiy iestiictions as banks. Indeed, theii fieedom fiom iegulation is one ieason they
have giown so iapidly. As othei fnancial inteimediaiies become moie impoitant, cential authoiities
begin to lose contiol ovei the money supply.
The declining shaie of fnancial assets contiolled by banks" began to change in 2008. Many of the
nation's laigest investment banks-fnancial institutions that piovided seivices to fims but weie not
iegulated as commeicial banks-began having seiious fnancial dimculties as a iesult of theii invest-
ments tied to home moitgage loans. As home piices in the United States began falling, many of those
moitgage loans went into default. Investment banks that had made substantial puichases of secuiities
whose value was ultimately based on those moitgage loans themselves began failing. Beai Steains, one
of the laigest investment banks in the United States, iequiied fedeial funds to iemain solvent. Anothei
laige investment bank, Lehman Biotheis, failed. In an efoit to avoid a similai fate, seveial othei invest-
ment banks applied foi status as oidinaiy commeicial banks subject to the stiingent iegulation those
institutions face. One iesult of the teiiible fnancial ciisis that ciippled the U.S. and othei economies in
2008 may be gieatei contiol of the money supply by the Fed.
2.2 Bank Finance and a Fiactional Reseive System
Bank fnance lies at the heait of the piocess thiough which money is cieated. To undeistand money
cieation, we need to undeistand some of the basics of bank fnance.
Banks accept deposits and issue checks to the owneis of those deposits. Banks use the money col-
lected fiom depositois to make loans. The bank's fnancial pictuie at a given time can be depicted using
a simplifed balance sheet, which is a fnancial statement showing assets, liabilities, and net woith.
Assets aie anything of value. Iiabilities aie obligations to othei paities. Net worth equals assets less
liabilities. All these aie given dollai values in a fim's balance sheet. The sum of liabilities plus net woith
theiefoie must equal the sum of all assets. On a balance sheet, assets aie listed on the left, liabilities and
net woith on the iight.
The main way that banks eain piofts is thiough issuing loans. Because theii depositois do not typ-
ically all ask foi the entiie amount of theii deposits back at the same time, banks lend out most of the
deposits they have collected-to companies seeking to expand theii opeiations, to people buying cais
oi homes, and so on. Banks keep only a fiaction of theii deposits as cash in theii vaults and in deposits
with the Fed. These assets aie called reserves. Banks lend out the iest of theii deposits. A system in
which banks hold ieseives whose value is less than the sum of claims outstanding on those ieseives is
called a fractional reserve banking system.
Table 24.1 shows a consolidated balance sheet foi commeicial banks in the United States foi June
2008. Banks hold ieseives against the liabilities iepiesented by theii checkable deposits. Notice that
these ieseives weie a small fiaction of total deposit liabilities of that month. Most bank assets aie in the
foim of loans.
7A8L 24. 1 7he ConsoIidated 8aIance Sheet for U.S. CommerciaI 8anks, 1une 2008
||s b.|.rce s|eet |c. .|| ccnne.c|.| b.rks |r t|e |r|teJ St.tes s|c.s t|e|. |r.rc|.| s|tu.t|cr |r b||||crs c| Jc||..s,
se.scr.||y .JusteJ, cr 'ure 2008.
Assets LiabiIities and Net Worth
|ese.ves 300.0 |eck.b|e Jepcs|ts 604.5
Ot|e. .ssets 1,35.8 Ot|e. Jepcs|ts 6,306.
|c.rs 6,903.4 bc..c.|rgs 2,322.1
Secu.|t|es 2,466.9 Ot|e. ||.b|||t|es 6,56.6
ct.| .ssets 11,928.1 ct.| ||.b|||t|es 9,890.9
|et .c.t| 1,13.2
Source Federal Reserve Statistical Release H.8, August 1, 2008.
In the next section, we will leain that money is cieated when banks issue loans.
604 PRINCIPLS OI CONOMICS
required reserves
|e ,u.rt|ty c| .ese.ves
b.rks ..e .e,u|.eJ tc |c|J.
required reserve ratio
|e ..t|c c| .ese.ves tc
c|eck.b|e Jepcs|ts . b.rk
nust n.|rt.|r.
excess reserves
|ese.ves |r excess c| t|e
.e,u|.eJ |eve|.
Ioaned up
\|er . b.rks excess
.ese.ves e,u.| .e.c.
2.3 Money Cieation
To undeistand the piocess of money cieation today, let us cieate a hypothetical system of banks. We
will focus on thiee banks in this system: Acme Bank, Bellville Bank, and Claikston Bank. Assume that
all banks aie iequiied to hold ieseives equal to 10 of theii checkable deposits. The quantity of ie-
seives banks aie iequiied to hold is called required reserves. The ieseive iequiiement is expiessed as
a required reserve ratio; it specifes the iatio of ieseives to checkable deposits a bank must maintain.
Banks may hold ieseives in excess of the iequiied level; such ieseives aie called excess reserves. Ex-
cess ieseives plus iequiied ieseives equal total ieseives.
Because banks eain ielatively little inteiest on theii ieseives held on deposit with the Fedeial
Reseive, we shall assume that they seek to hold no excess ieseives. When a bank's excess ieseives equal
zeio, it is loaned up. Finally, we shall ignoie assets othei than ieseives and loans and deposits othei
than checkable deposits. To simplify the analysis fuithei, we shall suppose that banks have no net
woith; theii assets aie equal to theii liabilities.
Let us suppose that eveiy bank in oui imaginaiy system begins with $1,000 in ieseives, $9,000 in
loans outstanding, and $10,000 in checkable deposit balances held by customeis. The balance sheet foi
one of these banks, Acme Bank, is shown in Table 24.2. The iequiied ieseive iatio is 0.1: Each bank
must have ieseives equal to 10 of its checkable deposits. Because ieseives equal iequiied ieseives, ex-
cess ieseives equal zeio. Each bank is loaned up.
7A8L 24. 2 A 8aIance Sheet for Acme 8ank
\e .ssune t|.t .|| b.rks |r . |ypct|et|c.| systen c| b.rks |.ve 1,000 |r .ese.ves, 10,000 |r c|eck.b|e Jepcs|ts,
.rJ 9,000 |r |c.rs. \|t| . 10 .ese.ve .e,u|.enert, e.c| b.rk |s |c.reJ up, |t |.s .e.c excess .ese.ves.
Acme 8ank
Assets LiabiIities
|ese.ves 1,000 |epcs|ts 10,000
|c.rs 9,000
Acme Bank, like eveiy othei bank in oui hypothetical system, initially holds ieseives equal to the level
of iequiied ieseives. Now suppose one of Acme Bank's customeis deposits $1,000 in cash in a checking
account. The money goes into the bank's vault and thus adds to ieseives. The customei now has an ad-
ditional $1,000 in his oi hei account. Two veisions of Acme's balance sheet aie given heie. The fist
shows the changes biought by the customei's deposit: ieseives and checkable deposits iise by $1,000.
The second shows how these changes afect Acme's balances. Reseives now equal $2,000 and checkable
deposits equal $11,000. With checkable deposits of $11,000 and a 10 ieseive iequiiement, Acme is ie-
quiied to hold ieseives of $1,100. With ieseives equaling $2,000, Acme has $900 in excess ieseives.
At this stage, theie has been no change in the money supply. When the customei biought in the
$1,000 and Acme put the money in the vault, cuiiency in ciiculation fell by $1,000. At the same time,
the $1,000 was added to the customei's checking account balance, so the money supply did not change.
II GUR 24. 3
Because Acme eains only a low inteiest iate on its excess ieseives, we assume it will tiy to loan them
out. Suppose Acme lends the $900 to one of its customeis. It will make the loan by ciediting the cus-
tomei's checking account with $900. Acme's outstanding loans and checkable deposits iise by $900.
The $900 in checkable deposits is new money; Acme cieated it when it issued the $900 loan. Now you
know wheie money comes fiom-it is cieated when a bank issues a loan.
CHAP7R 24 7H NA7UR AND CRA7ION OI MONY 605
II GUR 24. 4
Piesumably, the customei who boiiowed the $900 did so in oidei to spend it. That customei will wiite
a check to someone else, who is likely to bank at some othei bank. Suppose that Acme's boiiowei
wiites a check to a fim with an account at Bellville Bank. In this set of tiansactions, Acme's checkable
deposits fall by $900. The fim that ieceives the check deposits it in its account at Bellville Bank, in-
cieasing that bank's checkable deposits by $900. Bellville Bank now has a check wiitten on an Acme ac-
count. Bellville will submit the check to the Fed, which will ieduce Acme's deposits with the Fed-its
ieseives-by $900 and inciease Bellville's ieseives by $900.
II GUR 24. 5
Notice that Acme Bank emeiges fiom this iound of tiansactions with $11,000 in checkable deposits
and $1,100 in ieseives. It has eliminated its excess ieseives by issuing the loan foi $900; Acme is now
loaned up. Notice also that fiom Acme's point of view, it has not cieated any money! It meiely took in
a $1,000 deposit and emeiged fiom the piocess with $1,000 in additional checkable deposits.
The $900 in new money Acme cieated when it issued a loan has not vanished-it is now in an ac-
count in Bellville Bank. Like the magician who shows the audience that the hat fiom which the iabbit
appeaied was empty, Acme can iepoit that it has not cieated any money. Theie is a wondeiful iiony in
the magic of money cieation: banks cieate money when they issue loans, but no one bank evei seems to
keep the money it cieates. That is because money is cieated within the banking system, not by a single
bank.
The piocess of money cieation will not end theie. Let us go back to Bellville Bank. Its deposits and
ieseives iose by $900 when the Acme check was deposited in a Bellville account. The $900 deposit ie-
quiied an inciease in iequiied ieseives of $90. Because Bellville's ieseives iose by $900, it now has $810
in excess ieseives. Just as Acme lent the amount of its excess ieseives, we can expect Bellville to lend
this $810. The next set of balance sheets shows this tiansaction. Bellville's loans and checkable deposits
iise by $810.
606 PRINCIPLS OI CONOMICS
II GUR 24. 6
The $810 that Bellville lent will be spent. Let us suppose it ends up with a customei who banks at
Claikston Bank. Bellville's checkable deposits fall by $810; Claikston's iise by the same amount. Claik-
ston submits the check to the Fed, which tiansfeis the money fiom Bellville's ieseive account to Claik-
ston's. Notice that Claikston's deposits iise by $810; Claikston must inciease its ieseives by $81. But its
ieseives have iisen by $810, so it has excess ieseives of $729.
II GUR 24. 7
Notice that Bellville is now loaned up. And notice that it can iepoit that it has not cieated any money
eithei! It took in a $900 deposit, and its checkable deposits have iisen by that same $900. The $810 it
cieated when it issued a loan is now at Claikston Bank.
The piocess will not end theie. Claikston will lend the $729 it now has in excess ieseives, and the
money that has been cieated will end up at some othei bank, which will then have excess ieseives-and
cieate still moie money. And that piocess will just keep going as long as theie aie excess ieseives to
pass thiough the banking system in the foim of loans. How much will ultimately be cieated by the sys-
tem as a whole: With a 10 ieseive iequiiement, each dollai in ieseives backs up $10 in checkable de-
posits. The $1,000 in cash that Acme's customei biought in adds $1,000 in ieseives to the banking sys-
tem. It can theiefoie back up an additional $10,000! In just the thiee banks we have shown, checkable
deposits have iisen by $2,710 ($1,000 at Acme, $900 at Bellville, and $810 at Claikston). Additional
banks in the system will continue to cieate money, up to a maximum of $7,290 among them. Subtiact-
ing the oiiginal $1,000 that had been a pait of cuiiency in ciiculation, we see that the money supply
could iise by as much as $9,000.
CHAP7R 24 7H NA7UR AND CRA7ION OI MONY 607
deposit muItipIier
|e ..t|c c| t|e n.x|nun
pcss|b|e c|.rge |r c|eck.b|e
Jepcs|ts (/|) tc t|e c|.rge
|r .ese.ves (/|).
Heads Up!
|ct|ce t|.t .|er t|e b.rks .ece|veJ re. Jepcs|ts, t|ey ccu|J n.ke re. |c.rs cr|y up tc t|e .ncurt c| t|e|.
excess .ese.ves, rct up tc t|e .ncurt c| t|e|. Jepcs|ts .rJ tct.| .ese.ve |rc.e.ses. |c. ex.np|e, .|t| t|e re.
Jepcs|t c| 1,000, Acne b.rk ..s .b|e tc n.ke .JJ|t|cr.| |c.rs c| 900. || |rste.J |t n.Je re. |c.rs e,u.| tc
|ts |rc.e.se |r tct.| .ese.ves, t|er .|te. t|e custcne.s .|c .ece|veJ re. |c.rs ..cte c|ecks tc ct|e.s, |ts .e
se.ves .cu|J be |ess t|.r t|e .e,u|.eJ .ncurt. |r t|e c.se c| Acne, |.J |t |ert cut .r .JJ|t|cr.| 1,000, .|te.
c|ecks .e.e ..|tter .g.|rst t|e re. |c.rs, |t .cu|J |.ve beer |e|t .|t| cr|y 1,000 |r .ese.ves .g.|rst
11,000 |r Jepcs|ts, |c. . .ese.ve ..t|c c| cr|y 0.09, .||c| |s |ess t|.r t|e .e,u|.eJ .ese.ve ..t|c c| 0.1 |r t|e
ex.np|e.
2.4 The Deposit Multipliei
We can ielate the potential inciease in the money supply to the change in ieseives that cieated it using
the deposit multiplier (m
d
), which equals the iatio of the maximum possible change in checkable de-
posits (AD) to the change in ieseives (AR). In oui example, the deposit multipliei was 10:
QUA7I ON 24. 1
m
d
=
AD
AR
=
$10,000
$1,000
= 10
To see how the deposit multipliei m
d
is ielated to the iequiied ieseive iatio, we use the fact that if
banks in the economy aie loaned up, then ieseives, R, equal the iequiied ieseive iatio (rrr) times
checkable deposits, D:
QUA7I ON 24. 2
R = rrrD
A change in ieseives pioduces a change in loans and a change in checkable deposits. Once banks
aie fully loaned up, the change in ieseives, AR, will equal the iequiied ieseive iatio times the change in
deposits, AD:
QUA7I ON 24. 3
AR = rrrAD
Solving foi AD, we have
QUA7I ON 24. 4
1
rrr
AR = AD
Dividing both sides by AR, we see that the deposit multipliei, m
d
, is 1/rrr:
QUA7I ON 24. 5
1
rrr
=
AD
AR
= m
d
The deposit multiplier is thus given by the reciprocal of the required reserve ratio. With a iequiied
ieseive iatio of 0.1, the deposit multipliei is 10. A iequiied ieseive iatio of 0.2 would pioduce a deposit
multipliei of 3. The highei the iequiied ieseive iatio, the lowei the deposit multipliei.
Actual incieases in checkable deposits will not be neaily as gieat as suggested by the deposit multi-
pliei. That is because the aitifcial conditions of oui example aie not met in the ieal woild. Some banks
hold excess ieseives, customeis withdiaw cash, and some loan pioceeds aie not spent. Each of these
factois ieduces the degiee to which checkable deposits aie afected by an inciease in ieseives. The basic
mechanism, howevei, is the one desciibed in oui example, and it iemains the case that checkable de-
posits inciease by a multiple of an inciease in ieseives.
The entiie piocess of money cieation can woik in ieveise. When you withdiaw cash fiom youi
bank, you ieduce the bank's ieseives. Just as a deposit at Acme Bank incieases the money supply by a
multiple of the oiiginal deposit, youi withdiawal ieduces the money supply by a multiple of the
amount you withdiaw. And just as money is cieated when banks issue loans, it is destioyed as the loans
aie iepaid. A loan payment ieduces checkable deposits; it thus ieduces the money supply.
608 PRINCIPLS OI CONOMICS
II GUR 24. 8
Suppose, foi example, that the Acme Bank customei who boiiowed the $900 makes a $100 pay-
ment on the loan. Only pait of the payment will ieduce the loan balance; pait will be inteiest. Suppose
$30 of the payment is foi inteiest, while the iemaining $70 ieduces the loan balance. The efect of the
payment on Acme's balance sheet is shown below. Checkable deposits fall by $100, loans fall by $70,
and net woith iises by the amount of the inteiest payment, $30.
Similai to the piocess of money cieation, the money ieduction piocess decieases checkable depos-
its by, at most, the amount of the ieduction in deposits times the deposit multipliei.
2.3 The Regulation of Banks
Banks aie among the most heavily iegulated of fnancial institutions. They aie iegu-
lated in pait to piotect individual depositois against coiiupt business piactices. Banks
aie also susceptible to ciises of confdence. Because theii ieseives equal only a fiaction
of theii deposit liabilities, an efoit by customeis to get all theii cash out of a bank could
foice it to fail. A few pooily managed banks could cieate such a ciisis, leading people to
tiy to withdiaw theii funds fiom well-managed banks. Anothei ieason foi the high de-
giee of iegulation is that vaiiations in the quantity of money have impoitant efects on
the economy as a whole, and banks aie the institutions thiough which money is
cieated.
Deposit Insurance
Fiom a customei's point of view, the most impoitant foim of iegulation comes in the foim of deposit
insuiance. Foi commeicial banks, this insuiance is piovided by the Fedeial Deposit Insuiance Coipoi-
ation (FDIC). Insuiance funds aie maintained thiough a piemium assessed on banks foi eveiy $100 of
bank deposits.
If a commeicial bank fails, the FDIC guaiantees to ieimbuise depositois up to at least $100,000 pei
account, tempoiaiily iaised to $230,000 thiough 2009. In piactice, the FDIC has made good on all de-
posits in failed banks, iegaidless of the size of the account. Fiom a depositoi's point of view, theiefoie,
it is not necessaiy to woiiy about a bank's safety.
One dimculty this insuiance cieates, howevei, is that it may induce the omceis of a bank to take
moie iisks. With a fedeial agency on hand to bail them out if they fail, the costs of failuie aie ieduced.
Bank omceis can thus be expected to take moie iisks than they would otheiwise, which, in tuin, makes
failuie moie likely. In addition, depositois, knowing that theii deposits aie insuied, may not sciutinize
the banks' lending activities as caiefully as they would if they felt that unwise loans could iesult in the
loss of theii deposits.
Thus, banks piesent us with a fundamental dilemma. A fiactional ieseive system means that banks
can opeiate only if theii customeis maintain theii confdence in them. If bank customeis lose con-
fdence, they aie likely to tiy to withdiaw theii funds. But with a fiactional ieseive system, a bank actu-
ally holds funds in ieseive equal to only a small fiaction of its deposit liabilities. If its customeis think a
bank will fail and tiy to withdiaw theii cash, the bank is likely to fail. Bank panics, in which fiightened
customeis iush to withdiaw theii deposits, contiibuted to the failuie of one-thiid of the nation's banks
between 1929 and 1933. Deposit insuiance was intioduced in laige pait to give people confdence in
theii banks and to pievent failuie. But the deposit insuiance that seeks to pievent bank failuies may
lead to less caieful management-and thus encouiage bank failuie.
ReguIation to Prevent 8ank IaiIure
To ieduce the numbei of bank failuies, banks aie seveiely limited in what they can do. They aie baiied
fiom ceitain types of fnancial investments and fiom activities viewed as too iisky. Banks aie iequiied
to maintain a minimum level of net woith as a fiaction of total assets. Regulatois fiom the FDIC iegu-
laily peifoim audits and othei checks of individual banks to ensuie they aie opeiating safely.
The FDIC has the powei to close a bank whose net woith has fallen below the iequiied level. In
piactice, it typically acts to close a bank when it becomes insolvent, that is, when its net woith becomes
negative. Negative net woith implies that the bank's liabilities exceed its assets.
When the FDIC closes a bank, it aiianges foi depositois to ieceive theii funds. When the bank's
funds aie insumcient to ietuin customeis' deposits, the FDIC uses money fiom the insuiance fund foi
this puipose. Alteinatively, the FDIC may aiiange foi anothei bank to puichase the failed bank. The
FDIC, howevei, continues to guaiantee that depositois will not lose any money.
CHAP7R 24 7H NA7UR AND CRA7ION OI MONY 609
k Y 7 A k A W A Y S
< b.rks ..e |r.rc|.| |rte.neJ|..|es t|.t .ccept Jepcs|ts, n.ke |c.rs, .rJ p.cv|Je c|eck|rg .cccurts |c.
t|e|. custcne.s.
< Vcrey |s c.e.teJ .|t||r t|e b.rk|rg systen .|er b.rks |ssue |c.rs, |t |s Jest.cyeJ .|er t|e |c.rs ..e
.ep.|J.
< Ar |rc.e.se (Jec.e.se) |r .ese.ves |r t|e b.rk|rg systen c.r |rc.e.se (Jec.e.se) t|e ncrey supp|y. |e
n.x|nun .ncurt c| t|e |rc.e.se (Jec.e.se) |s e,u.| tc t|e Jepcs|t nu|t|p||e. t|nes t|e c|.rge |r
.ese.ves, t|e Jepcs|t nu|t|p||e. e,u.|s t|e .ec|p.cc.| c| t|e .e,u|.eJ .ese.ve ..t|c.
< b.rk Jepcs|ts ..e |rsu.eJ .rJ b.rks ..e |e.v||y .egu|.teJ.
7 R Y I 7 !
.. Suppcse Acne b.rk |r|t|.||y |.s 10,000 |r Jepcs|ts, .ese.ves c| 2,000, .rJ |c.rs c| 8,000. At . .e,u|.eJ
.ese.ve ..t|c c| 0.2, |s Acne |c.reJ up` S|c. t|e b.|.rce s|eet c| Acne b.rk .t p.esert.
b. |c. suppcse t|.t .r Acne b.rk custcne., p|.rr|rg tc t.ke c.s| cr .r exterJeJ cc||ege g..Ju.t|cr t.|p
tc |rJ|., .|t|J...s 1,000 |.cn |e. .cccurt. S|c. t|e c|.rges tc Acne b.rks b.|.rce s|eet .rJ Acnes
b.|.rce s|eet .|te. t|e .|t|J....|. by |c. nuc| ..e |ts .ese.ves rc. Je|c|ert`
c. Acne .cu|J p.cb.b|y .ep|er|s| |ts .ese.ves by .eJuc|rg |c.rs. ||s .ct|cr .cu|J c.use . nu|t|p||eJ
ccrt..ct|cr c| c|eck.b|e Jepcs|ts .s ct|e. b.rks |cse Jepcs|ts bec.use t|e|. custcne.s .cu|J be p.y|rg
c| |c.rs tc Acne. |c. |..ge .cu|J t|e ccrt..ct|cr be`
Case in Point: A 8ig 8ank Goes Under
|t ..s t|e J..||rg c| \.|| St.eet|t s|c.eJ ..p|J g.c.t| .rJ n.Je b|g p.c|ts. \.s||rgtcr Vutu.|, . s.v|rgs
.rJ |c.r b.seJ |r t|e st.te c| \.s||rgtcr, ..s . .e|.t|ve|y sn.|| |rst|tut|cr .|cse |O, |e..y |. |||||rge., |.J
b|g p|.rs. |e ..rteJ tc t..rs|c.n ||s ||tt|e Se.tt|e S| |rtc t|e \.|V..t c| b.rks.
V.. |||||rge. beg.r pu.su|rg . .e|.t|ve|y st..|g|t|c....J st..tegy. |e .c,u|.eJ b.rks |r |..ge c|t|es suc| .s |
|c.gc .rJ |cs Arge|es. |e .c,u|.eJ b.rks up .rJ Jc.r t|e e.st .rJ .est cc.sts. |e .gg.ess|ve|y exterJeJ
c.eJ|t tc |c.|rccne |rJ|v|Ju.|s .rJ |.n|||esc.eJ|t c..Js, c.. |c.rs, .rJ nc.tg.ges. |r n.k|rg nc.tg.ge
|c.rs tc |c.|rccne |.n|||es, \.Vu, .s t|e b.rk ..s krc.r, ,u|ck|y bec.ne ve.y p.c|t.b|e. but |t ..s expcs
|rg |tse|| tc g.e.te. .rJ g.e.te. .|sk, .ccc.J|rg tc t|e |e. `o| |e.
610 PRINCIPLS OI CONOMICS
|cus|rg p.|ces |r t|e |r|teJ St.tes nc.e t|.r Jcub|eJ bet.eer 199 .rJ 200. |u.|rg t|.t t|ne, |c.rs tc
ever |c.|rccne |cuse|c|Js .e.e p.c|t.b|e. but, .s |cus|rg p.|ces beg.r |.|||rg |r 200, b.rks suc| .s
\.Vu beg.r tc expe.|erce |csses .s |cnec.re.s beg.r tc ..|k ...y |.cn |cuses .|cse v.|ues suJJer|y
|e|| be|c. t|e|. cutst.rJ|rg nc.tg.ges. \.Vu beg.r |cs|rg ncrey |r 200 .s |cus|rg p.|ces beg.r |.|||rg.
|e ccnp.ry |.J e..reJ 3.6 b||||cr |r 2006, .rJ s.urg tc . |css c| 6 n||||cr |r 200, .ccc.J|rg tc t|e |o
e| .ooJ |o|e .ooo|. V.. |||||rge. ..s custeJ by t|e bc..J e..|y |r Septenbe. c| 2008. |e b.rk |.||eJ
|.te. t|.t ncrt|. |t ..s t|e b|ggest b.rk |.||u.e |r t|e ||stc.y c| t|e |r|teJ St.tes.
|e |eJe..| |epcs|t |rsu..rce c.pc..t|cr (|||) |.J ust .escueJ .rct|e. b.rk, |rJyV.c, .||c| ..s cr|y .
tert| t|e s|.e c| \.Vu, .rJ .cu|J |.ve Jcre t|e s.ne |c. \.Vu || |t |.J rct beer .b|e tc |rJ . ccnp.ry tc
pu.c|.se |t. but |r t||s c.se, '|Vc.g.r |.se .g.eeJ tc t.ke |t cve.|ts Jepcs|ts, b.rk b..rc|es, .rJ |ts
t.cub|eJ .sset pc.t|c||c. |e gcve.rnert .rJ t|e |eJ ever regct|.teJ t|e Je.| be||rJ \.Vus b.ck! |e
t|er c||e| execut|ve c|ce. c| t|e ccnp.ry, A|.r |. ||s|n.r, ..s .epc.teJ|y |y|rg |.cn |e. +c.k tc Se.tt|e
.|er t|e Je.| ..s |r.||.eJ.
|e gcve.rnert ..s .rx|cus tc b.cke. . Je.| t|.t J|J rct .e,u|.e use c| t|e |||s Jep|eteJ |urJs |c||c.|rg
|rJyV.cs cc||.pse. but |t .cu|J |.ve Jcre sc || . buye. |.J rct beer |curJ. As t|e ||| .epc.ts cr |ts \eb
s|te. S|rce t|e |||s c.e.t|cr |r 1933, rc Jepcs|tc. |.s eve. |cst ever cre perry c| ||||rsu.eJ |urJs.
!385(*6 ||c |o| oJ ^Je. |o .o||. Oo.ee| .e|e /o/o oJ .e|| .oe ^e|.' |e |e. `o| |e. .e|ee 25. 200S. ^. |||e
O|J. ||Je |e|o|| |eoo |o /o/o |o||oe.' |oe| .ooJ |o|e .ooo|. .ooo, 23. 2009. oJ ||| /e ||e o| |||...|J|co.eJ|e
|J|c_||o|||
A N S W R 7 O 7 R Y I 7 ! P R O 8 L M
.. Acne b.rk |s |c.reJ up, s|rce 2,000/10,000 0.2, .||c| |s t|e .e,u|.eJ .ese.ve ..t|c. Acnes b.|.rce
s|eet |s.
b. Acne b.rks b.|.rce s|eet .|te. |cs|rg 1,000 |r Jepcs|ts.
|e,u|.eJ .ese.ves ..e Je|c|ert by 800. Acne nust |c|J 20 c| |ts Jepcs|ts, |r t||s c.se
1,800 (0.2 9,000 1,800)
, .s .ese.ves, but |t |.s cr|y 1,000 |r .ese.ves .t t|e ncnert.
c. |e ccrt..ct|cr |r c|eck.b|e Jepcs|ts .cu|J be
AD = (1 / 0.2) ( $1,000) = $3,000
CHAP7R 24 7H NA7UR AND CRA7ION OI MONY 611
centraI bank
A b.rk t|.t .cts .s . b.rke.
tc t|e cert..| gcve.rnert,
.cts .s . b.rke. tc b.rks, .cts
.s . .egu|.tc. c| b.rks,
ccrJucts ncret..y pc||cy,
.rJ suppc.ts t|e st.b|||ty c|
t|e |r.rc|.| systen.
3. THE FEDERAL RESERVE SYSTEM
L A R N I N G O 8 1 C 7 I V S
1. xpIain the primary functions of centraI banks.
2. Describe how the IederaI Reserve System is structured and governed.
3. Identify and expIain the tooIs of monetary poIicy.
4. Describe how the Ied creates and destroys money when it buys and seIIs federaI government
bonds.
The Fedeial Reseive System of the United States, oi Fed, is the U.S. cential bank. Japan's cential bank is
the Bank of Japan; the Euiopean Union has established the Euiopean Cential Bank. Most countiies
have a cential bank. A central bank peifoims fve piimaiy functions: (1) it acts as a bankei to the
cential goveinment, (2) it acts as a bankei to banks, (3) it acts as a iegulatoi of banks, (4) it conducts
monetaiy policy, and (3) it suppoits the stability of the fnancial system.
Foi the fist 137 yeais of its histoiy, the United States did not have a tiue cential bank. While a
cential bank was often pioposed, theie was iesistance to cieating an institution with such enoimous
powei. A seiies of bank panics slowly incieased suppoit foi the cieation of a cential bank. The bank
panic of 1907 pioved to be the fnal stiaw. Bank failuies weie so widespiead, and depositoi losses so
heavy, that conceins about centialization of powei gave way to a desiie foi an institution that would
piovide a stabilizing foice in the banking industiy. Congiess passed the Fedeial Reseive Act in 1913,
cieating the Fed and giving it all the poweis of a cential bank.
3.1 Stiuctuie of the Fed
In cieating the Fed, Congiess deteimined that a cential bank should be as independent of the govein-
ment as possible. It also sought to avoid too much centialization of powei in a single institution. These
potentially contiadictoiy goals of independence and decentialized powei aie evident in the Fed's stiuc-
tuie and in the continuing stiuggles between Congiess and the Fed ovei possible changes in that
stiuctuie.
In an efoit to decentialize powei, Congiess designed the Fed as a system of 12 iegional banks, as
shown in Figuie 24.12. Each of these banks opeiates as a kind of bankeis' coopeiative; the iegional
banks aie owned by the commeicial banks in theii distiicts that have chosen to be membeis of the Fed.
The owneis of each Fedeial Reseive bank select the boaid of diiectois of that bank; the boaid selects
the bank's piesident.
612 PRINCIPLS OI CONOMICS
II GUR 24. 12 7he 12 IederaI Reserve Districts and the Cities Where ach 8ank Is Located
Seveial piovisions of the Fedeial Reseive Act seek to maintain the Fed's independence. The boaid of
diiectois foi the entiie Fedeial Reseive System is called the Boaid of Goveinois. The seven membeis of
the boaid aie appointed by the piesident of the United States and confimed by the Senate. To ensuie a
laige measuie of independence fiom any one piesident, the membeis of the Boaid of Goveinois have
14-yeai teims. One membei of the boaid is selected by the piesident of the United States to seive as
chaiiman foi a foui-yeai teim.
As a fuithei means of ensuiing the independence of the Fed, Congiess authoiized it to buy and sell
fedeial goveinment bonds. This activity is a pioftable one that allows the Fed to pay its own bills. The
Fed is thus not dependent on a Congiess that might otheiwise be tempted to foice a paiticulai set of
policies on it. The Fed is limited in the piofts it is allowed to eain; its excess" piofts aie ietuined to
the Tieasuiy.
It is impoitant to iecognize that the Fed is technically not pait of the fedeial goveinment. Mem-
beis of the Boaid of Goveinois do not legally have to answei to Congiess, the piesident, oi anyone else.
The piesident and membeis of Congiess can ceitainly tiy to infuence the Fed, but they cannot oidei it
to do anything. Congiess, howevei, cieated the Fed. It could, by passing anothei law, abolish the Fed's
independence. The Fed can maintain its independence only by keeping the suppoit of Congiess-and
that sometimes iequiies being iesponsive to the wishes of Congiess.
In iecent yeais, Congiess has sought to inciease its oveisight of the Fed. The chaiiman of the
Fedeial Reseive Boaid is iequiied to iepoit to Congiess twice each yeai on its monetaiy policy, the set
of policies that the cential bank can use to infuence economic activity.
3.2 Poweis of the Fed
The Fed's piincipal poweis stem fiom its authoiity to conduct monetaiy policy. It has thiee main
policy tools: setting ieseive iequiiements, opeiating the discount window and othei ciedit facilities,
and conducting open-maiket opeiations.
Reserve Requirements
The Fed sets the iequiied iatio of ieseives that banks must hold ielative to theii deposit liabilities. In
theoiy, the Fed could use this powei as an instiument of monetaiy policy. It could lowei ieseive ie-
quiiements when it wanted to inciease the money supply and iaise them when it wanted to ieduce the
money supply. In piactice, howevei, the Fed does not use its powei to set ieseive iequiiements in this
way. The ieason is that fiequent manipulation of ieseive iequiiements would make life dimcult foi
bankeis, who would have to adjust theii lending policies to changing iequiiements.
The Fed's powei to set ieseive iequiiements was expanded by the Monetaiy Contiol Act of 1980.
Befoie that, the Fed set ieseive iequiiements only foi commeicial banks that weie membeis of the
Fedeial Reseive System. Most banks aie not membeis of the Fed; the Fed's contiol of ieseive
CHAP7R 24 7H NA7UR AND CRA7ION OI MONY 613
discount rate
|e |rte.est ..te c|..geJ by
t|e |eJ .|er |t |erJs
.ese.ves tc b.rks.
federaI funds market
A n..ket |r .||c| b.rks |erJ
.ese.ves tc cre .rct|e..
federaI funds rate
|e |rte.est ..te c|..geJ
.|er cre b.rk |erJs
.ese.ves tc .rct|e..
bond
A p.cn|se by t|e |ssue. c| t|e
bcrJ tc p.y t|e c.re. c| t|e
bcrJ . p.ynert c. . se.|es c|
p.ynerts cr . spec||c J.te
c. J.tes.
open-market operations
|e buy|rg .rJ se|||rg c|
|eJe..| gcve.rnert bcrJs by
t|e |eJ.
iequiiements thus extended to only a minoiity of banks. The 1980 act iequiied viitually all banks to
satisfy the Fed's ieseive iequiiements.
7he Discount Window and Other Credit IaciIities
A majoi iesponsibility of the Fed is to act as a lendei of last iesoit to banks. When banks fall shoit on
ieseives, they can boiiow ieseives fiom the Fed thiough its discount window. The discount rate is
the inteiest iate chaiged by the Fed when it lends ieseives to banks. The Boaid of Goveinois sets the
discount iate.
Loweiing the discount iate makes funds cheapei to banks. A lowei discount iate could place
downwaid piessuie on inteiest iates in the economy. Howevei, when fnancial maikets aie opeiating
noimally, banks iaiely boiiow fiom the Fed, ieseiving use of the discount window foi emeigencies. A
typical bank boiiows fiom the Fed only about once oi twice pei yeai.
Instead of boiiowing fiom the Fed when they need ieseives, banks typically iely on the fedeial
funds maiket to obtain ieseives. The federal funds market is a maiket in which banks lend ieseives
to one anothei. The federal funds rate is the inteiest iate chaiged foi such loans; it is deteimined by
banks' demand foi and supply of these ieseives. The ability to set the discount iate is no longei an im-
poitant tool of Fedeial Reseive policy.
To deal with the iecent fnancial and economic conditions, the Fed gieatly expanded its lending
beyond its tiaditional discount window lending. As falling house piices led to foieclosuies, piivate in-
vestment banks and othei fnancial institutions came undei incieasing piessuie. The Fed made ciedit
available to a wide iange of institutions in an efoit to stem the ciisis. In 2008, the Fed bailed out two
majoi housing fnance fims that had been established by the goveinment to piop up the housing in-
dustiy-Fannie Mae (the Fedeial National Moitgage Association) and Fieddie Mac (the Fedeial Home
Moitgage Coipoiation). Togethei, the two institutions backed the moitgages of half of the nation's
moitgage loans.
[2]
It also agieed to piovide $83 billion to AIG, the huge insuiance fim. AIG had a sub-
sidiaiy that was heavily exposed to moitgage loan losses, and that ciippled the fim. The Fed deteim-
ined that AIG was simply too big to be allowed to fail. Many banks had ties to the giant institution, and
its failuie would have been a blow to those banks. As the United States faced the woist fnancial ciisis
since the Gieat Depiession, the Fed took centei stage. Whatevei its iole in the fnancial ciisis of
2007-2008, the Fed iemains an impoitant backstop foi banks and othei fnancial institutions needing
liquidity. And foi that, it uses the tiaditional discount window, supplemented with a wide iange of oth-
ei ciedit facilities. The Case in Point in this section discusses these new ciedit facilities.
Open-Market Operations
The Fed's ability to buy and sell fedeial goveinment bonds has pioved to be its most potent policy tool.
A bond is a piomise by the issuei of the bond (in this case the fedeial goveinment) to pay the ownei of
the bond a payment oi a seiies of payments on a specifc date oi dates. The buying and selling of fedei-
al goveinment bonds by the Fed aie called open-market operations. When the Fed buys oi sells gov-
einment bonds, it adds oi subtiacts ieseives fiom the banking system. Such changes afect the money
supply.
Suppose the Fed buys a goveinment bond in the open maiket. It wiites a check on its own account
to the sellei of the bond. When the sellei deposits the check at a bank, the bank submits the check to
the Fed foi payment. The Fed pays" the check by ciediting the bank's account at the Fed, so the bank
has moie ieseives.
The Fed's puichase of a bond can be illustiated using a balance sheet. Suppose the Fed buys a bond
foi $1,000 fiom one of Acme Bank's customeis. When that customei deposits the check at Acme,
checkable deposits will iise by $1,000. The check is wiitten on the Fedeial Reseive System; the Fed will
ciedit Acme's account. Acme's ieseives thus iise by $1,000. With a 10 ieseive iequiiement, that will
cieate $900 in excess ieseives and set of the same piocess of money expansion as did the cash deposit
we have alieady examined. The difeience is that the Fed's puichase of a bond cieated new ieseives
with the stioke of a pen, wheie the cash deposit cieated them by iemoving $1,000 fiom cuiiency in cii-
culation. The puichase of the $1,000 bond by the Fed could thus inciease the money supply by as much
as $10,000, the maximum expansion suggested by the deposit multipliei.
614 PRINCIPLS OI CONOMICS
II GUR 24. 13
II GUR 24. 14 7he Ied and the IIow of
Money in the conomy
|rJ|v|Ju.|s .rJ |.ns (t|e pub||c) n.ke
Jepcs|ts |r b.rks, b.rks n.ke |c.rs tc
|rJ|v|Ju.|s .rJ |.ns. |e |eJ c.r buy bcrJs
tc |rect re. .ese.ves |rtc t|e systen, t|us
|rc.e.s|rg b.rk |erJ|rg, .||c| c.e.tes re.
Jepcs|ts, c.e.t|rg st||| nc.e |erJ|rg .s t|e
Jepcs|t nu|t|p||e. gces tc .c.k. A|te.r.t|ve|y,
t|e |eJ c.r se|| bcrJs, .|t|J...|rg .ese.ves
|.cn t|e systen, t|us .eJuc|rg b.rk |erJ|rg
.rJ .eJuc|rg tct.| Jepcs|ts.
Wheie does the Fed get $1,000 to puichase the bond: It simply cieates the money
when it wiites the check to puichase the bond. On the Fed's balance sheet, assets in-
ciease by $1,000 because the Fed now has the bond; bank deposits with the Fed, which
iepiesent a liability to the Fed, iise by $1,000 as well.
When the Fed sells a bond, it gives the buyei a fedeial goveinment bond that it had
pieviously puichased and accepts a check in exchange. The bank on which the check
was wiitten will fnd its deposit with the Fed ieduced by the amount of the check. That
bank's ieseives and checkable deposits will fall by equal amounts; the ieseives, in efect,
disappeai. The iesult is a ieduction in the money supply. The Fed thus incieases the
money supply by buying bonds; it ieduces the money supply by selling them.
Figuie 24.14 shows how the Fed infuences the fow of money in the economy.
Funds fow fiom the public-individuals and fims-to banks as deposits. Banks use
those funds to make loans to the public-to individuals and fims. The Fed can in-
fuence the volume of bank lending by buying bonds and thus injecting ieseives into the system. With
new ieseives, banks will inciease theii lending, which cieates still moie deposits and still moie lending
as the deposit multipliei goes to woik. Alteinatively, the Fed can sell bonds. When it does, ieseives fow
out of the system, ieducing bank lending and ieducing deposits.
The Fed's puichase oi sale of bonds is conducted by the Open Maiket Desk at the
Fedeial Reseive Bank of New Yoik, one of the 12 distiict banks. Tiadeis at the Open
Maiket Desk aie guided by policy diiectives issued by the Fedeial Open Maiket Com-
mittee (FOMC). The FOMC consists of the seven membeis of the Boaid of Goveinois
plus fve iegional bank piesidents. The piesident of the New Yoik Fedeial Reseive Bank
seives as a membei of the FOMC; the othei 11 bank piesidents take tuins flling the ie-
maining foui seats.
The FOMC meets eight times pei yeai to chait the Fed's monetaiy policies. In the
past, FOMC meetings weie closed, with no iepoit of the committee's action until the
ielease of the minutes six weeks aftei the meeting. Faced with piessuie to open its pio-
ceedings, the Fed began in 1994 issuing a iepoit of the decisions of the FOMC immedi-
ately aftei each meeting.
In piactice, the Fed sets taigets foi the fedeial funds iate. To achieve a lowei fedei-
al funds iate, the Fed goes into the open maiket buying secuiities and thus incieasing
the money supply. When the Fed iaises its taiget iate foi the fedeial funds iate, it sells
secuiities and thus ieduces the money supply.
Tiaditionally, the Fed has bought and sold shoit-teim goveinment secuiities;
howevei, in dealing with the condition of the economy in 2009, wheiein the Fed has
alieady set the taiget foi the fedeial funds iate at neai zeio, the Fed has announced that
it will also be buying longei teim goveinment secuiities. In so doing, it hopes to in-
fuence longei teim inteiest iates, such as those ielated to moitgages.
k Y 7 A k A W A Y S
< |e |eJ, t|e cert..| b.rk c| t|e |r|teJ St.tes, .cts .s . b.rk |c. ct|e. b.rks .rJ |c. t|e |eJe..|
gcve.rnert. |t .|sc .egu|.tes b.rks, sets ncret..y pc||cy, .rJ n.|rt.|rs t|e st.b|||ty c| t|e |r.rc|.|
systen.
< |e |eJ sets .ese.ve .e,u|.enerts .rJ t|e J|sccurt ..te .rJ ccrJucts cpern..ket cpe..t|crs. O| t|ese
tcc|s c| ncret..y pc||cy, cpern..ket cpe..t|crs ..e t|e ncst |npc.t.rt.
< St..t|rg |r 200, t|e |eJ beg.r c.e.t|rg .JJ|t|cr.| c.eJ|t |.c|||t|es tc |e|p tc st.b|||.e t|e |r.rc|.| systen.
< |e |eJ c.e.tes re. .ese.ves .rJ re. ncrey .|er |t pu.c|.ses bcrJs. |t Jest.cys .ese.ves .rJ t|us
.eJuces t|e ncrey supp|y .|er |t se||s bcrJs.
CHAP7R 24 7H NA7UR AND CRA7ION OI MONY 615
7 R Y I 7 !
Suppcse t|e |eJ se||s 8 n||||cr .c.t| c| bcrJs.
.. |c. Jc b.rk .ese.ves c|.rge`
b. \||| t|e ncrey supp|y |rc.e.se c. Jec.e.se`
c. \|.t |s t|e n.x|nun pcss|b|e c|.rge |r t|e ncrey supp|y || t|e .e,u|.eJ .ese.ve ..t|c |s 0.2`
Case in Point: Ied Supports the IinanciaI System by Creating New Credit IaciIities
\e|| be|c.e ncst c| t|e pub||c bec.ne ....e c| t|e p.ec..|cus st.te c| t|e |.S. |r.rc|.| systen, t|e |eJ
beg.r tc see s|grs c| g.c.|rg |r.rc|.| st..|rs .rJ tc .ct cr .eJuc|rg t|en. |r p..t|cu|.., t|e |eJ s.. t|.t
s|c.tte.n |rte.est ..tes t|.t ..e c|ter ,u|te c|cse tc t|e |eJe..| |urJs ..te beg.r tc .|se n..keJ|y .bcve |t.
|e .|Jer|rg sp.e.J ..s .|..n|rg, bec.use |t suggesteJ t|.t |erJe. ccr|Jerce ..s Jec||r|rg, ever |c. .|.t
..e gere..||y ccrs|Je.eJ |c..|sk |c.rs. cnne.c|.| p.pe., |r .||c| |..ge ccnp.r|es bc..c. |urJs |c. . pe.|
cJ c| .bcut . ncrt| tc n.r.ge t|e|. c.s| |c., |s .r ex.np|e. |ver ccnp.r|es .|t| ||g| c.eJ|t ..t|rgs .e.e
|.v|rg tc p.y urusu.||y ||g| |rte.est ..te p.en|uns |r c.Je. tc get |urJ|rg, c. |r scne c.ses ccu|J rct get
|urJ|rg .t .||.
c Je.| .|t| t|e J.y|rg up c| c.eJ|t n..kets, beg|rr|rg |r |.te 200 .rJ .cce|e..t|rg eve. s|rce, t|e |eJ |.s
c.e.teJ .r .|p|.bet scup c| re. c.eJ|t |.c|||t|es. Scne c| t|ese ..e c|e.eJ |r ccrurct|cr .|t| t|e |ep..t
nert c| t|e .e.su.y, .||c| |.s nc.e |.t|tuJe |r te.ns c| .ccept|rg scne c.eJ|t .|sk. |e |.c|||t|es J||e. |r
te.ns c| cc||.te..| useJ, t|e Ju..t|cr c| t|e |c.r, .||c| |rst|tut|crs ..e e||g|b|e tc bc..c., .rJ t|e ccst tc t|e
bc..c.e.. |c. ex.np|e, t|e |.|n..y |e.|e. .eJ|t |.c|||ty (|||) .||c.s p.|n..y Je.|e.s (|.e., t|cse |r.rc|.| |r
st|tut|crs t|.t rc.n.||y |.rJ|e t|e |eJs cper n..ket cpe..t|crs) tc cbt.|r cve.r|g|t |c.rs. |e e.n Asset
b.ckeJ Secu.|t|es |c.r |.c|||ty (A||) .||c.s . .|Je ..rge c| ccnp.r|es tc bc..c., us|rg t|e p.|n..y Je.|e.s
.s ccrJu|ts, b.seJ cr ,u.|||eJ .ssetb.ckeJ secu.|t|es .e|.teJ tc stuJert, .utc, c.eJ|t c..J, .rJ sn.|| bus|ress
Jebt, |c. . t|.eeye.. pe.|cJ. Vcst c| t|ese re. |.c|||t|es ..e Jes|greJ tc be tenpc...y, .|t| exp|..t|crs scne
t|ne |r 2009, but t|ey c.r be exterJeJ.
\|.t t|ey |.ve |r ccnncr, t|cug|, |s |rc.e.s|rg ||,u|J|ty t|.t .||| |cpe|u||y st|nu|.te p.|v.te sperJ|rg. |c.
ex.np|e, t|ese c.eJ|t |.c|||t|es n.y erccu..ge b.rks tc p..e Jc.r t|e|. excess .ese.ves (.||c| g.e. erc.n
cus|y .s t|e |r.rc|.| c.|s|s ur|c|JeJ .rJ t|e eccrcny Jete.|c..teJ) .rJ tc n.ke nc.e |c.rs. |r t|e .c.Js c|
|eJ |.|.n.r ber be.r.rke.
616 PRINCIPLS OI CONOMICS
||o|J||, o.||o , ||e ce|o| o| eJoce ,|e|c || , oo| o|e| o||c|o|
||o|. |oo|J |o||e |.e|o e| |o |oe co|Jece. |oc|o| ||||o||o .||| e o|e |o
ee| ||e eo||| JeoJ |o co| .|||oo| eo|| |o o|e||o||, Je|o|||| |e o|e o|
oe| /oeo.e. oc||o| ||e ||o|J||, eeJ o| |oc|o| ||||o||o eJoce |oJ|
|ee oJ. o|| e|e eoo|. |oo|J |ceoe ||e .||||e o| ||oe ||||o||o |o |eJ oJ
o|e o|e|'
|e |eg.| .ut|c.|ty |c. ncst c| t|ese re. c.eJ|t |.c|||t|es ccnes |.cn . p..t|cu|.. sect|cr c| t|e |eJe..|
|ese.ve Act t|.t .||c.s t|e bc..J c| Ccve.rc.s |r urusu.| .rJ ex|gert c|.cunst.rces tc exterJ c.eJ|t tc .
.|Je ..rge c| n..ket p|.ye.s.
!385(*6 |e . |eo|e. |e || oJ ||e |o||c, |eoe' (.|e |ec|oe. |oJo .c|oo| o| |coo|c. |oJo. ||oJ. .ooo, 3. 2009:.
||c|oJ ||ec|o oJ |o|e . Ooco. |e. /oe|o, |o||c, oo|.' |eJeo| |ee.e |o| o| .| |oo| /oe|o, eJ. /o, 200S. |eJeo| |ee.e
|ooJ o| Oo.eo /e ||e o| |||...|eJeo|ee.eo.oe|o,o||c,Je|oo||||
A N S W R 7 O 7 R Y I 7 ! P R O 8 L M
.. b.rk .ese.ves |.|| by 8 n||||cr.
b. |e ncrey supp|y Jec.e.ses.
c. |e n.x|nun pcss|b|e Jec.e.se |s 40 n||||cr, s|rce /| (1/0.2) (8 n||||cr) 40 n||||cr.
4. REVIEW AND PRACTICE
Summary
|r t||s c|.pte. .e |rvest|g.teJ t|e ncrey supp|y .rJ |cckeJ .t |c. |t |s Jete.n|reJ. Vcrey |s .ryt||rg t|.t
se.ves .s . neJ|un c| exc|.rge. \|.teve. se.ves .s ncrey .|sc |urct|crs .s . ur|t c| .cccurt .rJ .s . stc.e
c| v.|ue. Vcrey n.y c. n.y rct |.ve |rt.|rs|c v.|ue. |r t|e |r|teJ St.tes, t|e tct.| c| cu..ercy |r c|.cu|.t|cr,
t..ve|e.s c|ecks, .rJ c|eck.b|e Jepcs|ts e,u.|s V1. A b.c.Je. ne.su.e c| t|e ncrey supp|y |s V2, .||c| |r
c|uJes V1 p|us .ssets t|.t ..e ||g||y ||,u|J, but |ess ||,u|J t|.r t|cse |r V1.
b.rks c.e.te ncrey .|er t|ey |ssue |c.rs. |e .b|||ty c| b.rks tc |ssue |c.rs |s ccrt.c||eJ by t|e|. .ese.ves.
|ese.ves ccrs|st c| c.s| |r b.rk v.u|ts .rJ b.rk Jepcs|ts .|t| t|e |eJ. b.rks cpe..te |r . |..ct|cr.| .ese.ve
systen, t|.t |s, t|ey n.|rt.|r .ese.ves e,u.| tc cr|y . sn.|| |..ct|cr c| t|e|. Jepcs|t ||.b|||t|es. b.rks ..e |e.v||y
.egu|.teJ tc p.ctect |rJ|v|Ju.| Jepcs|tc.s .rJ tc p.evert c.|ses c| ccr|Jerce. |epcs|t |rsu..rce p.ctects |rJ|
v|Ju.| Jepcs|tc.s.
A cert..| b.rk se.ves .s . b.rk |c. b.rks, . .egu|.tc. c| b.rks, . n.r.ge. c| t|e ncrey supp|y, . b.rk |c. .
r.t|crs gcve.rnert, .rJ . suppc.te. c| |r.rc|.| n..kets gere..||y. |r t|e |r.rc|.| c.|s|s t|.t .cckeJ t|e |r
|teJ St.tes .rJ nuc| c| t|e .c.|J |r 2008, t|e |eJ p|.yeJ . cert..| .c|e |r keep|rg b.rk .rJ rcrb.rk |rst|tu
t|crs .|c.t .rJ |r keep|rg c.eJ|t .v.||.b|e. |e |eJe..| |ese.ve Systen (|eJ) |s t|e cert..| b.rk |c. t|e |r|teJ
St.tes. |e |eJ |s gcve.reJ by . bc..J c| Ccve.rc.s .|cse nenbe.s ..e .ppc|rteJ by t|e p.es|Jert c| t|e
|r|teJ St.tes, subect tc ccr|.n.t|cr by t|e Ser.te.
|e |eJ c.r |erJ tc b.rks .rJ ct|e. |rst|tut|crs t|.cug| t|e J|sccurt .|rJc. .rJ ct|e. c.eJ|t |.c|||t|es,
c|.rge .ese.ve .e,u|.enerts, .rJ erg.ge |r pu.c|.ses .rJ s.|es c| |eJe..| gcve.rnert bcrJs |r t|e cper
n..ket. |ec|s|crs tc buy c. se|| bcrJs ..e n.Je by t|e |eJe..| Oper V..ket cnn|ttee (|OV), t|e |eJs
cpern..ket cpe..t|crs .ep.esert |ts p.|n..y tcc| |c. |r|uerc|rg t|e ncrey supp|y. |u.c|.ses c| bcrJs by
t|e |eJ |r|t|.||y |rc.e.se t|e .ese.ves c| b.rks. \|t| excess .ese.ves cr |.rJ, b.rks .||| .ttenpt tc |rc.e.se
t|e|. |c.rs, .rJ |r t|e p.ccess t|e ncrey supp|y .||| c|.rge by .r .ncurt |ess t|.r c. e,u.| tc t|e Jepcs|t
nu|t|p||e. t|nes t|e c|.rge |r .ese.ves. S|n||..|y, t|e |eJ c.r .eJuce t|e ncrey supp|y by se|||rg bcrJs.
CHAP7R 24 7H NA7UR AND CRA7ION OI MONY 617
C O N C P 7 P R O 8 L M S
1. A|.||res |.ve |.e,uert ||e. c|ubs |r .||c| custcne.s .ccunu|.te n||es .ccc.J|rg tc t|e runbe. c| n||es
t|ey |.ve |c.r .|t| t|e .|.||re. |.e,uert ||e. n||es c.r t|er be useJ tc pu.c|.se ct|e. ||g|ts, tc .ert
c..s, c. tc st.y |r scne |cte|s. A.e |.e,uert ||e. n||es ncrey`
2. |eb|t c..Js .||c. .r |rJ|v|Ju.| tc t..rs|e. |urJs J|.ect|y |r . c|eck.b|e .cccurt tc . ne.c|.rt .|t|cut
..|t|rg . c|eck. |c. |s t||s J||e.ert |.cn t|e ..y c.eJ|t c..Js .c.k` A.e e|t|e. c.eJ|t c..Js c. Jeb|t c..Js
ncrey` |xp|.|r.
3. V.ry cc||eges se|| spec|.| c..Js t|.t stuJerts c.r use tc pu.c|.se eve.yt||rg |.cn textbccks c. ne.|s |r
t|e c.|ete.|. tc use c| ..s||rg n.c||res |r t|e Jc.n. StuJerts Jepcs|t ncrey |r t|e|. c..Js, .s t|ey use
t|e|. c..Js |c. pu.c|.ses, e|ect.cr|c sc.rre.s .encve ncrey |.cn t|e c..Js. c .ep|er|s| . c..Js ncrey,
. stuJert n.kes . c.s| Jepcs|t t|.t |s c.eJ|teJ tc t|e c..J. \cu|J t|ese c..Js ccurt .s p..t c| t|e ncrey
supp|y`
4. A sn..t c..J, .|sc krc.r .s .r e|ect.cr|c pu.se, |s . p|.st|c c..J t|.t c.r be |c.JeJ .|t| . ncret..y v.|ue.
|ts Jeve|cpe.s ..gue t|.t, crce .|Je|y .ccepteJ, |t ccu|J .ep|.ce t|e use c| cu..ercy |r verJ|rg n.c||res,
p..k|rg nete.s, .rJ e|se.|e.e. Suppcse sn..t c..Js c.ne |rtc .|Jesp.e.J use. |.esert ycu. v|e.s cr t|e
|c||c.|rg |ssues.
.. \cu|J ycu ccurt b.|.rces |r t|e pu.ses .s p..t c| t|e ncrey supp|y` || sc, .cu|J t|ey be p..t c|
V1` V2`
b. S|cu|J .ry |rst|tut|cr be pe.n|tteJ tc |ssue t|en, c. s|cu|J t|ey be .est.|cteJ tc b.rks`
c. S|cu|J t|e |ssue.s be subect tc .ese.ve .e,u|.enerts`
J. Suppcse t|ey .e.e |ssueJ by b.rks. |c. Jc ycu t||rk t|e use c| suc| pu.ses .cu|J .|ect t|e
ncrey supp|y` |xp|.|r ycu. .rs.e. c..e|u||y.
5. \||c| c| t|e |c||c.|rg |tens |s p..t c| V1` V2`
.. 0.2 certs t|.t |.s .ccunu|.teJ urJe. . ccuc| cus||cr.
b. +cu. 2,000 ||re c| c.eJ|t .|t| ycu. \|s. .cccurt.
c. |e 210 b.|.rce |r ycu. c|eck|rg .cccurt.
J. 41 |r ycu. s.v|rgs .cccurt.
e. 10 s|..es c| stcck ycu. urc|e g.ve ycu cr ycu. 18t| b|.t|J.y, .||c| ..e rc. .c.t| 520.
|. 200 |r t..ve|e.s c|ecks ycu |.ve pu.c|.seJ |c. ycu. sp.|rgb.e.k t.|p.
6. |r t|e V|JJ|e Ages, gc|Jsn|t|s tcck |r custcne.s Jepcs|ts (gc|J cc|rs) .rJ |ssueJ .ece|pts t|.t
|urct|creJ nuc| ||ke c|ecks Jc tcJ.y. |ecp|e useJ t|e .ece|pts .s . neJ|un c| exc|.rge. Cc|Jsn|t|s
.|sc |ssueJ |c.rs by ..|t|rg .JJ|t|cr.| .ece|pts .g.|rst .||c| t|ey .e.e |c|J|rg rc gc|J tc bc..c.e.s.
\e.e gc|Jsn|t|s erg.g|rg |r |..ct|cr.| .ese.ve b.rk|rg` \|y Jc ycu t||rk t|.t custcne.s tu.reJ t|e|.
gc|J cve. tc gc|Jsn|t|s` \|c bere|teJ |.cn t|e gc|Jsn|t|s .ct|cr` \|y J|J suc| . systen gere..||y
.c.k` \|er .cu|J |t |.ve beer ||ke|y tc |.||`
. A 1,000 Jepcs|t |r Acne b.rk |.s |rc.e.seJ .ese.ves by 1,000. A |c.r c|ce. .t Acne .e.scrs .s |c||c.s.
|e .ese.ve .e,u|.enert |s 10. |.t ne.rs t|.t t|e 1,000 |r re. .ese.ves c.r b.ck 10,000 |r
c|eck.b|e Jepcs|ts. |e.e|c.e ||| |c.r .r .JJ|t|cr.| 10,000. |s t|e.e .ry p.cb|en .|t| t|e |c.r c|ce.s
.e.scr|rg` |xp|.|r.
8. \|er t|e |eJ buys .rJ se||s bcrJs t|.cug| cpern..ket cpe..t|crs, t|e ncrey supp|y c|.rges, but
t|e.e |s rc e|ect cr t|e ncrey supp|y .|er |rJ|v|Ju.|s buy .rJ se|| bcrJs. |xp|.|r.
618 PRINCIPLS OI CONOMICS
N U M R I C A L P R O 8 L M S
1. crs|Je. t|e |c||c.|rg ex.np|e c| b..te.|rg.
1 10curce bcre ste.k c.r be t..JeJ |c. 5 sc|t J.|rks.
1 sc|t J.|rk c.r be t..JeJ |c. 10 .pp|es.
100 .pp|es c.r be t..JeJ |c. . s||.t.
5 s||.ts c.r be exc|.rgeJ |c. 1 textbcck.
|t t.kes 4 textbccks tc get 1 \|.
.. |c. n.ry 10curce bcre ste.ks ccu|J ycu exc|.rge |c. 1 textbcck` |c. n.ry sc|t J.|rks`
|c. n.ry .pp|es`
b. St.te t|e p.|ce c| s||.ts |r te.ns c| .pp|es, textbccks, .rJ sc|t J.|rks.
c. \|y Jc ycu t||rk .e use ncrey .s . ur|t c| .cccurt`
2. Assune t|.t t|e b.rk|rg systen |s |c.reJ up .rJ t|.t .ry cpern..ket pu.c|.se by t|e |eJ J|.ect|y
|rc.e.ses .ese.ves |r t|e b.rks. || t|e .e,u|.eJ .ese.ve ..t|c |s 0.2, by |c. nuc| ccu|J t|e ncrey supp|y
exp.rJ || t|e |eJ pu.c|.seJ 2 b||||cr .c.t| c| bcrJs`
3. Suppcse t|e |eJ se||s 5 n||||cr .c.t| c| bcrJs tc |ccrcb.rk.
.. \|.t |.ppers tc t|e .ese.ves c| t|e b.rk`
b. \|.t |.ppers tc t|e ncrey supp|y |r t|e eccrcny .s . .|c|e || t|e .ese.ve .e,u|.enert |s 10,
.|| p.ynerts ..e n.Je by c|eck, .rJ t|e.e |s rc ret J..|r |rtc cu..ercy`
c. |c. .cu|J ycu. .rs.e. |r p..t b be .|ecteJ || ycu kre. t|.t scne pecp|e |rvc|veJ |r t|e
ncrey c.e.t|cr p.ccess kept scne c| t|e|. |urJs .s c.s|`
4. || |.|| t|e b.rks |r t|e r.t|cr bc..c. .JJ|t|cr.| .ese.ves tct.||rg 10 n||||cr .t t|e |eJ J|sccurt .|rJc.,
.rJ .t t|e s.ne t|ne t|e ct|e. |.|| c| t|e b.rks .eJuce t|e|. excess .ese.ves by . tct.| c| 10 n||||cr,
.|.t |s ||ke|y tc |.pper tc t|e ncrey supp|y` |xp|.|r.
5. Suppcse . b.rk .|t| . 10 .ese.ve .e,u|.enert |.s 10 n||||cr |r .ese.ves .rJ 100 n||||cr |r c|eck.b|e
Jepcs|ts, .rJ . n.c. cc.pc..t|cr n.kes . Jepcs|t c| 1 n||||cr.
.. |xp|.|r |c. t|e Jepcs|t .|ects t|e b.rks .ese.ves .rJ c|eck.b|e Jepcs|ts.
b. by |c. nuc| c.r t|e b.rk |rc.e.se |ts |erJ|rg`
6. Suppcse . b.rk .|t| . 25 .ese.ve .e,u|.enert |.s 50 n||||cr |r .ese.ves .rJ 200 n||||cr |r c|eck.b|e
Jepcs|ts, .rJ cre c| t|e b.rks Jepcs|tc.s, . n.c. cc.pc..t|cr, ..|tes . c|eck tc .rct|e. cc.pc..t|cr |c.
5 n||||cr. |e c|eck |s Jepcs|teJ |r .rct|e. b.rk.
.. |xp|.|r |c. t|e .|t|J....| .|ects t|e b.rks .ese.ves .rJ c|eck.b|e Jepcs|ts.
b. by |c. nuc| .||| t|e b.rk |.ve tc .eJuce |ts |erJ|rg`
. Suppcse t|e b.rk |r p.cb|en 6 |.ces . 20 .ese.ve .e,u|.enert. |e custcne. ..|tes t|e s.ne c|eck.
|c. .||| t||s .|ect ycu. .rs.e.s`
8. |c. ccrs|Je. .r eccrcny |r .||c| t|e cert..| b.rk |.s ust pu.c|.seJ 8 b||||cr .c.t| c| gcve.rnert
bcrJs |.cn b.rks |r t|e eccrcny. \|.t .cu|J be t|e e|ect c| t||s pu.c|.se cr t|e ncrey supp|y |r t|e
ccurt.y, .ssun|rg .ese.ve .e,u|.enerts c|.
.. 10.
b. 15.
c. 20.
J. 25.
9. |c. ccrs|Je. t|e s.ne eccrcny, .rJ t|e cert..| b.rk se||s 8 b||||cr .c.t| c| gcve.rnert bcrJs tc
|cc.| b.rks. St.te t|e ||ke|y e|ects cr t|e ncrey supp|y urJe. .ese.ve .e,u|.enerts c|.
.. 10.
b. 15.
c. 20.
J. 25.
10. |c. .cu|J t|e pu.c|.se c| 8 b||||cr c| bcrJs by t|e cert..| b.rk |.cn |cc.| b.rks be ||ke|y tc .|ect
|rte.est ..tes` |c. .bcut t|e e|ect cr |rte.est ..tes c| t|e s.|e c| 8 b||||cr .c.t| c| bcrJs` |xp|.|r ycu.
.rs.e.s c..e|u||y.
CHAP7R 24 7H NA7UR AND CRA7ION OI MONY 619
1.
2.
ENDNOTES
'ust|r Sc|eck, V.cke.e| |ccrcn|cs |r |.|scr |e.Js tc App.ec|.t|cr |c. O||y ||s|,
/o|| .|ee| .ooo|, Octcbe. 2, 2008, p. A1.
S.n ucke.n.r, |eJs .ke crt.c| c| |.rr|e V.e, |.eJJ|e V.c, |e .o |oc|co
|o|c|e, Septenbe. 8, 2008, p. A1.
620 PRINCIPLS OI CONOMICS
nanciaI markets
V..kets |r .||c| |urJs
.ccunu|.teJ by cre g.cup
..e n.Je .v.||.b|e tc .rct|e.
g.cup.
| A | | | 2 5
Financial Markets and the
Economy
S7AR7 UP: CLAMPING DOWN ON MONY GROW7H
|c. re..|y t|.ee Jec.Jes, Ane.|c.rs |.ve ccne tc expect ve.y |c. |r|.t|cr, cr t|e c.Je. c| 2 tc 3 . ye... |c.
J|J t||s expect.t|cr ccne tc be` \.s |t .|..ys sc` Absc|ute|y rct.
|r 'u|y 199, .|t| |r|.t|cr .pp.c.c||rg 14 .rJ |rte.est ..tes cr t|.eencrt| .e.su.y b|||s sc..|rg p.st 10,
. Jespe..te |.es|Jert '|nny ..te. tcck .ct|cr. |e .ppc|rteJ |.u| \c|cke., t|e p.es|Jert c| t|e |e. +c.k |eJe..|
|ese.ve b.rk, .s c|.|.n.r c| t|e |eJs bc..J c| Ccve.rc.s. V.. \c|cke. n.Je c|e.. t|.t ||s cbect|ve .s c|.|.n.r
..s tc b.|rg Jc.r t|e |r|.t|cr ..terc n.tte. .|.t t|e ccrse,uerces |c. t|e eccrcny. V.. ..te. g.ve t||s
e|c.t ||s |u|| suppc.t.
V.. \c|cke. ..steJ rc t|ne |r putt|rg ||s pc||c|es tc .c.k. |e s|c.eJ t|e ..te c| ncrey g.c.t| |nneJ|.te|y.
|e eccrcnys .espcrse ..s s.||t, t|e |r|teJ St.tes s||ppeJ |rtc . b.|e| .ecess|cr |r 1980, |c||c.eJ by . c.us||rg
.ecess|cr |r 19811982. |r te.ns c| t|e gc.| c| .eJuc|rg |r|.t|cr, V.. \c|cke.s ncret..y pc||c|es .e.e . J...||rg
success. |r|.t|cr p|urgeJ be|c. . 4 ..te .|t||r t|.ee ye..s, by 1986 t|e |r|.t|cr ..te |.J |.||er tc 1.1. |e t.||,
b.|J, c|g..snck|rg V.. \c|cke. ene.geJ .s . |c|k |e.c |r t|e |g|t .g.|rst |r|.t|cr. |rJeeJ |e |.s .etu.reJ 20
ye..s |.te. .s p..t c| |.es|Jert Ob.n.s eccrcn|c te.n tc pe.|.ps crce .g.|r .escue t|e |.S. eccrcny.
|e |eJs severye.. |g|t .g.|rst |r|.t|cr |.cn 199 tc 1986 n.Je t|e cb |c. A|.r C.eersp.r, V.. \c|cke.s
successc., t|.t nuc| e.s|e.. c see |c. t|e Jec|s|crs c| t|e |eJe..| |ese.ve .|ect key n.c.ceccrcn|c v..|
.b|es.e.| C||, t|e p.|ce |eve|, .rJ urenp|cynert|r t||s c|.pte. .e .||| exp|c.e |c. nanciaI markets,
n..kets |r .||c| |urJs .ccunu|.teJ by cre g.cup ..e n.Je .v.||.b|e tc .rct|e. g.cup, ..e ||rkeJ tc t|e
eccrcny.
||s c|.pte. p.cv|Jes t|e bu||J|rg b|ccks |c. urJe.st.rJ|rg |r.rc|.| n..kets. beg|rr|rg .|t| .r cve.v|e. c|
bcrJ .rJ |c.e|gr exc|.rge n..kets, .e .||| ex.n|re |c. t|ey ..e .e|.teJ tc t|e |eve| c| .e.| C|| .rJ t|e p.|ce
|eve|. |e seccrJ sect|cr ccnp|etes t|e ncJe| c| t|e ncrey n..ket. \e |.ve |e..reJ t|.t t|e |eJ c.r c|.rge t|e
.ncurt c| .ese.ves |r t|e b.rk|rg systen, .rJ t|.t .|er |t Jces t|e ncrey supp|y c|.rges. |e.e .e exp|.|r
ncrey Jen.rJt|e ,u.rt|ty c| ncrey pecp|e .rJ |.ns ..rt tc |c|J.||c|, tcget|e. .|t| ncrey supp|y,
|e.Js tc .r e,u|||b.|un ..te c| |rte.est.
|e ncJe| c| .gg.eg.te Jen.rJ .rJ supp|y s|c.s |c. c|.rges |r t|e ccnpcrerts c| .gg.eg.te Jen.rJ
.|ect C|| .rJ t|e p.|ce |eve|. |r t||s c|.pte., .e .||| |e..r t|.t c|.rges |r t|e |r.rc|.| n..kets c.r .|ect .gg.eg
.te Jen.rJ.rJ |r tu.r c.r |e.J tc c|.rges |r .e.| C|| .rJ t|e p.|ce |eve|. S|c.|rg |c. t|e |r.rc|.| n..kets
|t |rtc t|e ncJe| c| .gg.eg.te Jen.rJ .rJ .gg.eg.te supp|y .e Jeve|cpeJ e..||e. p.cv|Jes . nc.e ccnp|ete p|c
tu.e c| |c. t|e n.c.ceccrcny .c.ks.
face vaIue of a bond
|e .ncurt t|e |ssue. c| .
bcrJ .||| |.ve tc p.y cr t|e
n.tu.|ty J.te.
maturity date
|e J.te .|er . bcrJ
n.tu.es, c. ccnes Jue.
interest rate
|.ynert n.Je |c. t|e use c|
ncrey, exp.esseJ .s .
pe.cert.ge c| t|e .ncurt
bc..c.eJ.
1. THE BOND AND FOREIGN EXCHANGE MARKETS
L A R N I N G O 8 1 C 7 I V S
1. xpIain and iIIustrate how the bond market works and discuss the reIationship between the
price of a bond and that bond's interest rate.
2. xpIain and iIIustrate the reIationship between a change in demand for or suppIy of bonds and
macroeconomic activity.
3. xpIain and iIIustrate how the foreign exchange market works and how a change in demand
for a country's currency or a change in its suppIy aects macroeconomic activity.
In this section, we will look at the bond maiket and at the maiket foi foieign exchange. Events in these
maikets can afect the piice level and output foi the entiie economy.
1.1 The Bond Maiket
In theii daily opeiations and in puisuit of new piojects, institutions such as fims and goveinments of-
ten boiiow. They may seek funds fiom a bank. Many institutions, howevei, obtain ciedit by selling
bonds. The fedeial goveinment is one institution that issues bonds. A local school distiict might sell
bonds to fnance the constiuction of a new school. Youi college oi univeisity has piobably sold bonds
to fnance new buildings on campus. Fiims often sell bonds to fnance expansion. The maiket foi
bonds is an enoimously impoitant one.
When an institution sells a bond, it obtains the piice paid foi the bond as a kind of loan. The insti-
tution that issues the bond is obligated to make payments on the bond in the futuie. The inteiest iate is
deteimined by the piice of the bond. To undeistand these ielationships, let us look moie closely at
bond piices and inteiest iates.
8ond Prices and Interest Rates
Suppose the managei of a manufactuiing company needs to boiiow some money to expand the fact-
oiy. The managei could do so in the following way: he oi she piints, say, 300 pieces of papei, each
beaiing the company's piomise to pay the beaiei $1,000 in a yeai. These pieces of papei aie bonds, and
the company, as the issuei, piomises to make a single payment. The managei then ofeis these bonds
foi sale, announcing that they will be sold to the buyeis who ofei the highest piices. Suppose the
highest piice ofeied is $930, and all the bonds aie sold at that piice. Each bond is, in efect, an obliga-
tion to iepay buyeis $1,000. The buyeis of the bonds aie being paid $30 foi the seivice of lending $930
foi a yeai.
The $1,000 piinted on each bond is the face value of the bond; it is the amount the issuei will
have to pay on the maturity date of the bond-the date when the loan matuies, oi comes due. The
$930 at which they weie sold is theii piice. The difeience between the face value and the piice is the
amount paid foi the use of the money obtained fiom selling the bond.
An interest rate is the payment made foi the use of money, expiessed as a peicentage of the
amount boiiowed. Bonds you sold command an inteiest iate equal to the difeience between the face
value and the bond piice, divided by the bond piice, and then multiplied by 100 to foim a peicentage:
QUA7I ON 25. 1
Face valuebond piice
Bond piice
100 = inteiest iate
At a piice of $930, the inteiest iate is 3.3
$1,000 $930
$930
100 = 3.3
The inteiest iate on any bond is deteimined by its piice. As the piice falls, the inteiest iate iises. Sup-
pose, foi example, that the best piice the managei can get foi the bonds is $900. Now the inteiest iate is
11.1. A piice of $800 would mean an inteiest iate of 23; $730 would mean an inteiest iate of 33.3;
a piice of $300 tianslates into an inteiest iate of 100. The lowei the piice of a bond ielative to its face
value, the highei the inteiest iate.
Bonds in the ieal woild aie moie complicated than the piece of papei in oui example, but theii
stiuctuie is basically the same. They have a face value (usually an amount between $1,000 and
622 PRINCIPLS OI CONOMICS
II GUR 25. 1 7he 8ond Market
|e e,u|||b.|un p.|ce |c. bcrJs |s Jete.n|reJ
.|e.e t|e Jen.rJ .rJ supp|y cu.ves
|rte.sect. |e |r|t|.| sc|ut|cr |e.e |s . p.|ce c|
950, |np|y|rg .r |rte.est ..te c| 5.3. Ar
|rc.e.se |r bc..c.|rg, .|| ct|e. t||rgs e,u.|,
|rc.e.ses t|e supp|y c| bcrJs tc .
2
.rJ |c.ces
t|e p.|ce c| bcrJs Jc.r tc 900. |e |rte.est
..te .|ses tc 11.1.
$100,000) and a matuiity date. The matuiity date might be thiee months fiom the date of issue; it
might be 30 yeais.
Whatevei the peiiod until it matuies, and whatevei the face value of the bond may be, its issuei
will attempt to sell the bond at the highest possible piice. Buyeis of bonds will seek the lowest piices
they can obtain. Newly issued bonds aie geneially sold in auctions. Potential buyeis bid foi the bonds,
which aie sold to the highest biddeis. The lowei the piice of the bond ielative to its face value, the high-
ei the inteiest iate.
Both piivate fims and goveinment entities issue bonds as a way of iaising funds. The oiiginal buy-
ei need not hold the bond until matuiity. Bonds can be iesold at any time, but the piice the bond will
fetch at the time of iesale will vaiy depending on conditions in the economy and the fnancial maikets.
Figuie 23.1 illustiates the maiket foi bonds. Theii piice is deteimined by demand and supply.
Buyeis of newly issued bonds aie, in efect, lendeis. Selleis of newly issued bonds aie boiioweis-iecall
that coipoiations, the fedeial goveinment, and othei institutions sell bonds when they want to boiiow
money. Once a newly issued bond has been sold, its ownei can iesell it; a bond may change hands sev-
eial times befoie it matuies.
Bonds aie not exactly the same soit of pioduct as, say, bioccoli oi some othei good
oi seivice. Can we expect bonds to have the same kind of downwaid-sloping demand
cuives and upwaid-sloping supply cuives we encountei foi oidinaiy goods and sei-
vices: Yes. Considei demand. At lowei piices, bonds pay highei inteiest. That makes
them moie attiactive to buyeis of bonds and thus incieases the quantity demanded. On
the othei hand, lowei piices mean highei costs to boiioweis-supplieis of bonds-and
should ieduce the quantity supplied. Thus, the negative ielationship between piice and
quantity demanded and the positive ielationship between piice and quantity supplied
suggested by conventional demand and supply cuives holds tiue in the maiket foi
bonds.
If the quantity of bonds demanded is not equal to the quantity of bonds supplied,
the piice will adjust almost instantaneously to balance the two. Bond piices aie pei-
fectly fexible in that they change immediately to balance demand and supply. Suppose,
foi example, that the initial piice of bonds is $930, as shown by the inteisection of the
demand and supply cuives in Figuie 23.1. We will assume that all bonds have equal iisk
and a face value of $1,000 and that they matuie in one yeai. Now suppose that boiiow-
eis inciease theii boiiowing by ofeiing to sell moie bonds at eveiy inteiest iate. This
incieases the supply of bonds: the supply cuive shifts to the iight fiom+
1
to +
2
. That, in
tuin, loweis the equilibiium piice of bonds-to $900 in Figuie 23.1. The lowei piice foi
bonds means a highei inteiest iate.
7he 8ond Market and Macroeconomic Performance
The connection between the bond maiket and the economy deiives fiom the way in-
teiest iates afect aggiegate demand. Foi example, investment is one component of ag-
giegate demand, and inteiest iates afect investment. Fiims aie less likely to acquiie
new capital (that is, plant and equipment) if inteiest iates aie high; they'ie moie likely
to add capital if inteiest iates aie low.
[1]
If bond piices fall, inteiest iates go up. Highei inteiest iates tend to discouiage investment, so ag-
giegate demand will fall. A fall in aggiegate demand, othei things unchanged, will mean fewei jobs and
less total output than would have been the case with lowei iates of inteiest. In contiast, an inciease in
the piice of bonds loweis inteiest iates and makes investment in new capital moie attiactive. That
change may boost investment and thus boost aggiegate demand.
Figuie 23.2 shows how an event in the bond maiket can stimulate changes in the economy's out-
put and piice level. In Panel (a), an inciease in demand foi bonds iaises bond piices. Inteiest iates thus
fall. Lowei inteiest iates inciease the quantity of investment demanded, shifting the aggiegate demand
cuive to the iight, fiom
1
to
2
in Panel (b). Real GDP iises fiom 1
1
to 1
2
; the piice level iises
fiom (
1
to (
2
. In Panel (c), an inciease in the supply of bonds pushes bond piices down. Inteiest iates
iise. The quantity of investment is likely to fall, shifting aggiegate demand to the left, fiom
1
to
2
in Panel (d). Output and the piice level fall fiom 1
1
to 1
2
and fiom (
1
to (
2
, iespectively. Assuming
othei deteiminants of aggiegate demand iemain unchanged, highei inteiest iates will tend to ieduce
aggiegate demand and lowei inteiest iates will tend to inciease aggiegate demand.
CHAP7R 25 IINANCIAL MARk7S AND 7H CONOMY 623
foreign exchange market
A n..ket |r .||c| cu..erc|es
c| J||e.ert ccurt.|es ..e
t..JeJ |c. cre .rct|e..
II GUR 25. 2 8ond Prices and Macroeconomic Activity
Ar |rc.e.se |r t|e Jen.rJ |c. bcrJs tc |
2
|r |.re| (.) ..|ses t|e p.|ce c| bcrJs tc |
b
2
, .||c| |c.e.s |rte.est ..tes
.rJ bccsts |rvestnert. |.t |rc.e.ses .gg.eg.te Jen.rJ tc ^|
2
|r |.re| (b), .e.| C|| .|ses tc `
2
.rJ t|e p.|ce
|eve| .|ses tc |
2
.Ar |rc.e.se |r t|e supp|y c| bcrJs tc .
2
|c.e.s bcrJ p.|ces tc |
b
2
|r |.re| (c) .rJ ..|ses |rte.est
..tes. |e ||g|e. |rte.est ..te, t.ker by |tse||, |s ||ke|y tc c.use . .eJuct|cr |r |rvestnert .rJ .gg.eg.te Jen.rJ.
^|
1
|.||s tc ^|
2
, .e.| C|| |.||s tc `
2
, .rJ t|e p.|ce |eve| |.||s tc |
2
|r |.re| (J).
In thinking about the impact of changes in inteiest iates on aggiegate demand, we must iemembei that
some events that change aggiegate demand can afect inteiest iates. We will examine those events in
subsequent chapteis. Oui focus in this chaptei is on the way in which events that oiiginate in fnancial
maikets afect aggiegate demand.
1.2 Foieign Exchange Maikets
Anothei fnancial maiket that infuences macioeconomic vaiiables is the foreign exchange market,
a maiket in which cuiiencies of difeient countiies aie tiaded foi one anothei. Since changes in expoits
and impoits afect aggiegate demand and thus ieal GDP and the piice level, the maiket in which cui-
iencies aie tiaded has tiemendous impoitance in the economy.
Foieigneis who want to puichase goods and seivices oi assets in the United States must typically
pay foi them with dollais. United States puichaseis of foieign goods must geneially make the puichase
in a foieign cuiiency. An Egyptian family, foi example, exchanges Egyptian pounds foi dollais in oidei
to pay foi admission to Disney Woild. A Geiman fnancial investoi puichases dollais to buy U.S.
624 PRINCIPLS OI CONOMICS
trade-weighted exchange
rate
Ar |rJex c| exc|.rge ..tes.
II GUR 25. 3 Determining an
xchange Rate
|e e,u|||b.|un exc|.rge ..te |s t|e ..te .t
.||c| t|e ,u.rt|ty c| Jc||..s Jen.rJeJ
e,u.|s t|e ,u.rt|ty supp||eJ. |e.e, e,u|||b.|un
cccu.s .t exc|.rge ..te |, .t .||c| ( Jc||..s
..e exc|.rgeJ pe. pe.|cJ.
goveinment bonds. A family fiom the United States visiting India, on the othei hand, needs to obtain
Indian iupees in oidei to make puichases theie. A U.S. bank wanting to puichase assets in Mexico City
fist puichases pesos. These tiansactions aie accomplished in the foieign exchange maiket.
The foieign exchange maiket is not a single location in which cuiiencies aie tiaded. The teim
iefeis instead to the entiie aiiay of institutions thiough which people buy and sell cuiiencies. It in-
cludes a hotel desk cleik who piovides cuiiency exchange as a seivice to hotel guests, biokeis who ai-
iange cuiiency exchanges woith billions of dollais, and goveinments and cential banks that exchange
cuiiencies. Majoi cuiiency dealeis aie linked by computeis so that they can tiack cuiiency exchanges
all ovei the woild.
7he xchange Rate
A countiy's exchange iate is the piice of its cuiiency in teims of anothei cuiiency oi cuiiencies. On
Decembei 12, 2008, foi example, the dollai tiaded foi 91.13 Japanese yen, 0.73 euios, 10.11 South
Afiican iands, and 13.31 Mexican pesos. Theie aie as many exchange iates foi the dollai as theie aie
countiies whose cuiiencies exchange foi the dollai-ioughly 200 of them.
Economists summaiize the movement of exchange iates with a trade-weighted exchange rate,
which is an index of exchange iates. To calculate a tiade-weighted exchange iate index foi the U.S. dol-
lai, we select a gioup of countiies, weight the piice of the dollai in each countiy's cuiiency by the
amount of tiade between that countiy and the United States, and then iepoit the piice of the dollai
based on that tiade-weighted aveiage. Because tiade-weighted exchange iates aie so widely used in ie-
poiting cuiiency values, they aie often iefeiied to as exchange iates themselves. We will follow that
convention in this text.
Determining xchange Rates
The iates at which most cuiiencies exchange foi one anothei aie deteimined by demand and supply.
How does the model of demand and supply opeiate in the foieign exchange maiket:
The demand cuive foi dollais ielates the numbei of dollais buyeis want to buy in any peiiod to the
exchange iate. An inciease in the exchange iate means it takes moie foieign cuiiency to buy a dollai. A
highei exchange iate, in tuin, makes U.S. goods and seivices moie expensive foi foieign buyeis and ie-
duces the quantity they will demand. That is likely to ieduce the quantity of dollais they demand. Foi-
eigneis thus will demand fewei dollais as the piice of the dollai-the exchange iate-iises. Con-
sequently, the demand cuive foi dollais is downwaid sloping, as in Figuie 23.3.
The supply cuive foi dollais emeiges fiom a similai piocess. When people and
fims in the United States puichase goods, seivices, oi assets in foieign countiies, they
must puichase the cuiiency of those countiies fist. They supply dollais in exchange foi
foieign cuiiency. The supply of dollais on the foieign exchange maiket thus iefects the
degiee to which people in the United States aie buying foieign money at vaiious ex-
change iates. A highei exchange iate means that a dollai tiades foi moie foieign cui-
iency. In efect, the highei iate makes foieign goods and seivices cheapei to U.S. buy-
eis, so U.S. consumeis will puichase moie foieign goods and seivices. People will thus
supply moie dollais at a highei exchange iate; we expect the supply cuive foi dollais to
be upwaid sloping, as suggested in Figuie 23.3.
In addition to piivate individuals and fims that paiticipate in the foieign exchange
maiket, most goveinments paiticipate as well. A goveinment might seek to lowei its
exchange iate by selling its cuiiency; it might seek to iaise the iate by buying its cui-
iency. Although goveinments often paiticipate in foieign exchange maikets, they gen-
eially iepiesent a veiy small shaie of these maikets. The most impoitant tiadeis aie
piivate buyeis and selleis of cuiiencies.
xchange Rates and Macroeconomic Performance
People puichase a countiy's cuiiency foi two quite difeient ieasons: to puichase goods
oi seivices in that countiy, oi to puichase the assets of that countiy-its money, its cap-
ital, its stocks, its bonds, oi its ieal estate. Both of these motives must be consideied to
undeistand why demand and supply in the foieign exchange maiket may change.
One thing that can cause the piice of the dollai to iise, foi example, is a ieduction
in bond piices in Ameiican maikets. Figuie 23.4 illustiates the efect of this change.
Suppose the supply of bonds in the U.S. bond maiket incieases fiom +
1
to +
2
in Panel
(a). Bond piices will diop. Lowei bond piices mean highei inteiest iates. Foieign fnan-
cial investois, attiacted by the oppoitunity to eain highei ietuins in the United States,
will inciease theii demand foi dollais on the foieign exchange maiket in oidei to puichase U.S. bonds.
Panel (b) shows that the demand cuive foi dollais shifts fiom
1
to
2
. Simultaneously, U.S. fnancial
investois, attiacted by the highei inteiest iates at home, become less likely to make fnancial
CHAP7R 25 IINANCIAL MARk7S AND 7H CONOMY 625
investments abioad and thus supply fewei dollais to exchange maikets. The fall in the piice of U.S.
bonds shifts the supply cuive foi dollais on the foieign exchange maiket fiom +
1
to +
2
, and the ex-
change iate iises fiom
1
to
2
.
II GUR 25. 4 Shifts in Demand and SuppIy for DoIIars on the Ioreign xchange Market
|r |.re| (.), .r |rc.e.se |r t|e supp|y c| bcrJs |c.e.s bcrJ p.|ces tc |
b
2
(.rJ t|us ..|ses |rte.est ..tes). ||g|e.
|rte.est ..tes bccst t|e Jen.rJ .rJ .eJuce t|e supp|y |c. Jc||..s, |rc.e.s|rg t|e exc|.rge ..te |r |.re| (b) tc |
2
.
|ese Jeve|cpnerts |r t|e bcrJ .rJ |c.e|gr exc|.rge n..kets ..e ||ke|y tc |e.J tc . .eJuct|cr |r ret expc.ts .rJ
|r |rvestnert, .eJuc|rg .gg.eg.te Jen.rJ |.cn ^|
1
tc ^|
2
|r |.re| (c). |e p.|ce |eve| |r t|e eccrcny |.||s tc |
2
,
.rJ .e.| C|| |.||s |.cn `
1
tc `
2
.
The highei exchange iate makes U.S. goods and seivices moie expensive to foieigneis, so it ieduces ex-
poits. It makes foieign goods cheapei foi U.S. buyeis, so it incieases impoits. Net expoits thus fall, ie-
ducing aggiegate demand. Panel (c) shows that output falls fiom 1
1
to 1
2
; the piice level falls fiom (
1
to (
2
. This development in the foieign exchange maiket ieinfoices the impact of highei inteiest iates
we obseived in Figuie 23.2, Panels (c) and (d). They not only ieduce investment-they ieduce net ex-
poits as well.
k Y 7 A k A W A Y S
< A bcrJ .ep.eserts . bc..c.e.s Jebt, bcrJ p.|ces ..e Jete.n|reJ by Jen.rJ .rJ supp|y.
< |e |rte.est ..te cr . bcrJ |s reg.t|ve|y .e|.teJ tc t|e p.|ce c| t|e bcrJ. As t|e p.|ce c| . bcrJ |rc.e.ses,
t|e |rte.est ..te |.||s.
< Ar |rc.e.se |r t|e |rte.est ..te terJs tc Jec.e.se t|e ,u.rt|ty c| |rvestnert Jen.rJeJ .rJ, |erce, tc
Jec.e.se .gg.eg.te Jen.rJ. A Jec.e.se |r t|e |rte.est ..te |rc.e.ses t|e ,u.rt|ty c| |rvestnert
Jen.rJeJ .rJ .gg.eg.te Jen.rJ.
< |e Jen.rJ |c. Jc||..s cr |c.e|gr exc|.rge n..kets .ep.eserts |c.e|gr Jen.rJ |c. |.S. gccJs, se.v|ces,
.rJ .ssets. |e supp|y c| Jc||..s cr |c.e|gr exc|.rge n..kets .ep.eserts |.S. Jen.rJ |c. |c.e|gr gccJs,
se.v|ces, .rJ .ssets. |e Jen.rJ |c. .rJ t|e supp|y c| Jc||..s Jete.n|re t|e exc|.rge ..te.
< A .|se |r |.S. |rte.est ..tes .||| |rc.e.se t|e Jen.rJ |c. Jc||..s .rJ Jec.e.se t|e supp|y c| Jc||..s cr
|c.e|gr exc|.rge n..kets. As . .esu|t, t|e exc|.rge ..te .||| |rc.e.se .rJ .gg.eg.te Jen.rJ .|||
Jec.e.se. A |.|| |r |.S. |rte.est ..tes .||| |.ve t|e cppcs|te e|ect.
7 R Y I 7 !
Suppcse t|e supp|y c| bcrJs |r t|e |.S. n..ket Jec.e.ses. S|c. .rJ exp|.|r t|e e|ects cr t|e bcrJ .rJ |c.
e|gr exc|.rge n..kets. |se t|e .gg.eg.te Jen.rJ/.gg.eg.te supp|y |..ne.c.k tc s|c. .rJ exp|.|r t|e
e|ects cr |rvestnert, ret expc.ts, .e.| C||, .rJ t|e p.|ce |eve|.
626 PRINCIPLS OI CONOMICS
Case in Point: 8etting on a PIunge
|r 2004, . ce.t.|r |cn.s '. |.cn ||c.|J. |.J . p|.r. |e urJe.stccJ c|e..|y t|e |rve.se .e|.t|crs||p bet.eer
bcrJ p.|ces .rJ |rte.est ..tes. \|.t |e J|J rct urJe.st.rJ ..s |c. expers|ve guess|rg |rcc..ect|y t|e J|.ec
t|cr c| |rte.est ..tes .cu|J be .|er |e Jec|JeJ tc buy |rtc .r |rve.se bcrJ |urJ.
Ar |rve.se bcrJ |urJ |s cre t|.t pe.|c.ns .e|| .|er bcrJ p.|ces |.||. |e |urJ |cn.s bcug|t |rtc
|.ppereJ tc t..Je |r 30ye.. |.S. .e.su.y bcrJs, .rJ |cn.s guesseJ t|.t |rte.est ..tes cr t|en .cu|J .|se.
|e cr|y p.cb|en .|t| t|e p|.r ..s t|.t t|e |rte.est ..te cr 30ye.. bcrJs .ctu.||y |e|| cve. t|e rext ye... Sc,
t|e |urJ |cn.s bcug|t |rtc |cst v.|ue .|er t|e p.|ces c| t|ese bcrJs .cse. |xperses .sscc|.teJ .|t| t||s
type c| |urJ ex.ce.b.teJ |cn.ss |css. || cr|y |cn.s |.J krc.r bct| t|e .e|.t|crs||p bet.eer bcrJ p.|ces
.rJ |rte.est ..tes oJ t|e J|.ect|cr c| |rte.est ..tes!
|e.|.ps .rct|e. t||rg |e J|J rct urJe.st.rJ ..s t|.t .|er |e |e..J t|.t t|e |eJe..| |ese.ve ..s ..|s|rg
..tes |r 2004 t|.t t||s .e|e..eJ tc t|e |eJe..| |urJs ..tes, . ve.y s|c.tte.n |rte.est ..te. \|||e ct|e. s|c.tte.n
|rte.est ..tes ncveJ .|t| t|e |eJe..| |urJs ..te |r 2004, |crgte.n ..tes J|J rct ever b||rk.
!385(* |oc| .o|ee. |o| |e .|o|J |e |oJ |||||e |o .||| |.ee |oJ |oJ.' |o|o |eo|J. .oe 4. 2005. 34
A N S W R 7 O 7 R Y I 7 ! P R O 8 L M
|| t|e supp|y c| bcrJs Jec.e.ses |.cn .
1
tc .
2
, bcrJ p.|ces .||| .|se |.cn |
b
1
tc |
b
2
, .s s|c.r |r |.re| (.).
||g|e. bcrJ p.|ces ne.r |c.e. |rte.est ..tes. |c.e. |rte.est ..tes |r t|e |r|teJ St.tes .||| n.ke |r.rc|.| |r
vestnerts |r t|e |r|teJ St.tes |ess .tt..ct|ve tc |c.e|gre.s. As . .esu|t, t|e|. Jen.rJ |c. Jc||..s .||| Jec.e.se
|.cn |
1
tc |
2
, .s s|c.r |r |.re| (b). S|n||..|y, |.S. |r.rc|.| |rvestc.s .||| |cck .b.c.J |c. ||g|e. .etu.rs .rJ
t|us supp|y nc.e Jc||..s tc |c.e|gr exc|.rge n..kets, s|||t|rg t|e supp|y cu.ve |.cn .
1
tc .
2
. |us, t|e ex
c|.rge ..te .||| Jec.e.se. |e ,u.rt|ty c| |rvestnert .|ses Jue tc t|e |c.e. |rte.est ..tes. |et expc.ts .|se be
c.use t|e |c.e. exc|.rge ..te n.kes |.S. gccJs .rJ se.v|ces nc.e .tt..ct|ve tc |c.e|gre.s, t|us |rc.e.s|rg
expc.ts, .rJ n.kes |c.e|gr gccJs |ess .tt..ct|ve tc |.S. buye.s, t|us .eJuc|rg |npc.ts. |rc.e.ses |r |rvestnert
.rJ ret expc.ts |np|y . .|g|t...J s|||t |r t|e .gg.eg.te Jen.rJ cu.ve |.cn ^|
1
tc ^|
2
. |e.| C|| .rJ t|e
p.|ce |eve| |rc.e.se.
CHAP7R 25 IINANCIAL MARk7S AND 7H CONOMY 627
demand for money
|e .e|.t|crs||p bet.eer t|e
,u.rt|ty c| ncrey pecp|e
..rt tc |c|J .rJ t|e |.ctc.s
t|.t Jete.n|re t|.t ,u.rt|ty.
transactions demand for
money
Vcrey pecp|e |c|J tc p.y |c.
gccJs .rJ se.v|ces t|ey
.rt|c|p.te buy|rg.
precautionary demand for
money
|e ncrey pecp|e |c|J |c.
ccrt|rgerc|es.
2. DEMAND, SUPPLY, AND EQUILIBRIUM IN THE
MONEY MARKET
L A R N I N G O 8 1 C 7 I V S
1. xpIain the motives for hoIding money and reIate them to the interest rate that couId be
earned from hoIding aIternative assets, such as bonds.
2. Draw a money demand curve and expIain how changes in other variabIes may Iead to shifts in
the money demand curve.
3. IIIustrate and expIain the notion of equiIibrium in the money market.
4. Use graphs to expIain how changes in money demand or money suppIy are reIated to changes
in the bond market, in interest rates, in aggregate demand, and in reaI GDP and the price IeveI.
In this section we will exploie the link between money maikets, bond maikets, and inteiest iates. We
fist look at the demand foi money. The demand cuive foi money is deiived like any othei demand
cuive, by examining the ielationship between the piice" of money (which, we will see, is the inteiest
iate) and the quantity demanded, holding all othei deteiminants unchanged. We then link the demand
foi money to the concept of money supply developed in the last chaptei, to deteimine the equilibiium
iate of inteiest. In tuin, we show how changes in inteiest iates afect the macioeconomy.
2.1 The Demand foi Money
In deciding how much money to hold, people make a choice about how to hold theii wealth. How
much wealth shall be held as money and how much as othei assets: Foi a given amount of wealth, the
answei to this question will depend on the ielative costs and benefts of holding money veisus othei as-
sets. The demand for money is the ielationship between the quantity of money people want to hold
and the factois that deteimine that quantity.
To simplify oui analysis, we will assume theie aie only two ways to hold wealth: as money in a
checking account, oi as funds in a bond maiket mutual fund that puichases long-teim bonds on behalf
of its subsciibeis. A bond fund is not money. Some money deposits eain inteiest, but the ietuin on
these accounts is geneially lowei than what could be obtained in a bond fund. The advantage of check-
ing accounts is that they aie highly liquid and can thus be spent easily. We will think of the demand foi
money as a cuive that iepiesents the outcomes of choices between the gieatei liquidity of money de-
posits and the highei inteiest iates that can be eained by holding a bond fund. The difeience between
the inteiest iates paid on money deposits and the inteiest ietuin available fiom bonds is the cost of
holding money.
Motives for HoIding Money
One ieason people hold theii assets as money is so that they can puichase goods and seivices. The
money held foi the puichase of goods and seivices may be foi eveiyday tiansactions such as buying
gioceiies oi paying the ient, oi it may be kept on hand foi contingencies such as having the funds
available to pay to have the cai fxed oi to pay foi a tiip to the doctoi.
The transactions demand for money is money people hold to pay foi goods and seivices they
anticipate buying. When you caiiy money in youi puise oi wallet to buy a movie ticket oi maintain a
checking account balance so you can puichase gioceiies latei in the month, you aie holding the money
as pait of youi tiansactions demand foi money.
The money people hold foi contingencies iepiesents theii precautionary demand for money.
Money held foi piecautionaiy puiposes may include checking account balances kept foi possible home
iepaiis oi health-caie needs. People do not know piecisely when the need foi such expendituies will
occui, but they can piepaie foi them by holding money so that they'll have it available when the need
aiises.
People also hold money foi speculative puiposes. Bond piices fuctuate constantly. As a iesult,
holdeis of bonds not only eain inteiest but expeiience gains oi losses in the value of theii assets. Bond-
holdeis enjoy gains when bond piices iise and sufei losses when bond piices fall. Because of this, ex-
pectations play an impoitant iole as a deteiminant of the demand foi bonds. Holding bonds is one al-
teinative to holding money, so these same expectations can afect the demand foi money.
628 PRINCIPLS OI CONOMICS
specuIative demand for
money
|e ncrey |e|J |r .espcrse
tc ccrce.r t|.t bcrJ p.|ces
.rJ t|e p.|ces c| ct|e.
|r.rc|.| .ssets n|g|t
c|.rge.
John Maynaid Keynes, who was an enoimously successful speculatoi in bond maikets himself,
suggested that bondholdeis who anticipate a diop in bond piices will tiy to sell theii bonds ahead of
the piice diop in oidei to avoid this loss in asset value. Selling a bond means conveiting it to money.
Keynes iefeiied to the speculative demand for money as the money held in iesponse to concein
that bond piices and the piices of othei fnancial assets might change.
Of couise, money is money. One cannot soit thiough someone's checking account and locate
which funds aie held foi tiansactions and which funds aie theie because the ownei of the account is
woiiied about a diop in bond piices oi is taking a piecaution. We distinguish money held foi difeient
motives in oidei to undeistand how the quantity of money demanded will be afected by a key deteim-
inant of the demand foi money: the inteiest iate.
Interest Rates and the Demand for Money
The quantity of money people hold to pay foi tiansactions and to satisfy piecautionaiy and speculative
demand is likely to vaiy with the inteiest iates they can eain fiom alteinative assets such as bonds.
When inteiest iates iise ielative to the iates that can be eained on money deposits, people hold less
money. When inteiest iates fall, people hold moie money. The logic of these conclusions about the
money people hold and inteiest iates depends on the people's motives foi holding money.
The quantity of money households want to hold vaiies accoiding to theii income and the inteiest
iate; difeient aveiage quantities of money held can satisfy theii tiansactions and piecautionaiy de-
mands foi money. To see why, suppose a household eains and spends $3,000 pei month. It spends an
equal amount of money each day. Foi a month with 30 days, that is $100 pei day. One way the house-
hold could manage this spending would be to leave the money in a checking account, which we will as-
sume pays zeio inteiest. The household would thus have $3,000 in the checking account when the
month begins, $2,900 at the end of the fist day, $1,300 halfway thiough the month, and zeio at the end
of the last day of the month. Aveiaging the daily balances, we fnd that the quantity of money the
household demands equals $1,300. This appioach to money management, which we will call the cash
appioach," has the viitue of simplicity, but the household will eain no inteiest on its funds.
Considei an alteinative money management appioach that peimits the same pattein of spending.
At the beginning of the month, the household deposits $1,000 in its checking account and the othei
$2,000 in a bond fund. Assume the bond fund pays 1 inteiest pei month, oi an annual inteiest iate of
12.7. Aftei 10 days, the money in the checking account is exhausted, and the household withdiaws
anothei $1,000 fiom the bond fund foi the next 10 days. On the 20th day, the fnal $1,000 fiom the
bond fund goes into the checking account. With this stiategy, the household has an aveiage daily bal-
ance of $300, which is the quantity of money it demands. Let us call this money management stiategy
the bond fund appioach."
Remembei that both appioaches allow the household to spend $3,000 pei month, $100 pei day.
The cash appioach iequiies a quantity of money demanded of $1,300, while the bond fund appioach
loweis this quantity to $300.
The bond fund appioach geneiates some inteiest income. The household has $1,000 in the fund
foi 10 days (1/3 of a month) and $1,000 foi 20 days (2/3 of a month). With an inteiest iate of 1 pei
month, the household eains $10 in inteiest each month ([$1,000 0.01 1/3] + [$1,000 0.01 2/3]).
The disadvantage of the bond fund, of couise, is that it iequiies moie attention-$1,000 must be tians-
feiied fiom the fund twice each month. Theie may also be fees associated with the tiansfeis.
Of couise, the bond fund stiategy we have examined heie is just one of many. The household
could begin each month with $1,300 in the checking account and $1,300 in the bond fund, tiansfeiiing
$1,300 to the checking account midway thiough the month. This stiategy iequiies one less tiansfei, but
it also geneiates less inteiest-$7.30 (= $1,300 0.01 1/2). With this stiategy, the household demands
a quantity of money of $730. The household could also maintain a much smallei aveiage quantity of
money in its checking account and keep moie in its bond fund. Foi simplicity, we can think of any
stiategy that involves tiansfeiiing money in and out of a bond fund oi anothei inteiest-eaining asset as
a bond fund stiategy.
Which appioach should the household use: That is a choice each household must make-it is a
question of weighing the inteiest a bond fund stiategy cieates against the hassle and possible fees asso-
ciated with the tiansfeis it iequiies. Oui example does not yield a cleai-cut choice foi any one house-
hold, but we can make some geneializations about its implications.
Fiist, a household is moie likely to adopt a bond fund stiategy when the inteiest iate is highei. At
low inteiest iates, a household does not saciifce much income by puisuing the simplei cash stiategy.
As the inteiest iate iises, a bond fund stiategy becomes moie attiactive. That means that the highei the
inteiest iate, the lowei the quantity of money demanded.
Second, people aie moie likely to use a bond fund stiategy when the cost of tiansfeiiing funds is
lowei. The cieation of savings plans, which began in the 1970s and 1980s, that allowed easy tiansfei of
funds between inteiest-eaining assets and checkable deposits tended to ieduce the demand foi money.
CHAP7R 25 IINANCIAL MARk7S AND 7H CONOMY 629
demand curve for money
u.ve t|.t s|c.s t|e
,u.rt|ty c| ncrey
Jen.rJeJ .t e.c| |rte.est
..te, .|| ct|e. t||rgs
urc|.rgeJ.
II GUR 25. 7 7he Demand Curve for
Money
|e Jen.rJ cu.ve |c. ncrey s|c.s t|e
,u.rt|ty c| ncrey Jen.rJeJ .t e.c| |rte.est
..te. |ts Jc.r...J s|cpe exp.esses t|e
reg.t|ve .e|.t|crs||p bet.eer t|e ,u.rt|ty c|
ncrey Jen.rJeJ .rJ t|e |rte.est ..te.
Some money deposits, such as savings accounts and money maiket deposit accounts, pay inteiest.
In evaluating the choice between holding assets as some foim of money oi in othei foims such as
bonds, households will look at the difeiential between what those funds pay and what they could eain
in the bond maiket. A highei inteiest iate in the bond maiket is likely to inciease this difeiential; a
lowei inteiest iate will ieduce it. An inciease in the spiead between iates on money deposits and the in-
teiest iate in the bond maiket ieduces the quantity of money demanded; a ieduction in the spiead in-
cieases the quantity of money demanded.
Fiims, too, must deteimine how to manage theii eainings and expendituies. Howevei, instead of
woiiying about $3,000 pei month, even a ielatively small fim may be conceined about $3,000,000 pei
month. Rathei than facing the difeience of $10 veisus $7.30 in inteiest eainings used in oui household
example, this small fim would face a difeience of $2,300 pei month ($10,000 veisus $7,300). Foi veiy
laige fims such as Toyota oi AT&T, inteiest iate difeientials among vaiious foims of holding theii
fnancial assets tianslate into millions of dollais pei day.
How is the speculative demand foi money ielated to inteiest iates: When fnancial investois be-
lieve that the piices of bonds and othei assets will fall, theii speculative demand foi money goes up.
The speculative demand foi money thus depends on expectations about futuie changes in asset piices.
Will this demand also be afected by piesent inteiest iates:
If inteiest iates aie low, bond piices aie high. It seems likely that if bond piices aie high, fnancial
investois will become conceined that bond piices might fall. That suggests that high bond piices-low
inteiest iates-would inciease the quantity of money held foi speculative puiposes. Conveisely, if bond
piices aie alieady ielatively low, it is likely that fewei fnancial investois will expect them to fall still fui-
thei. They will hold smallei speculative balances. Economists thus expect that the quantity of money
demanded foi speculative ieasons will vaiy negatively with the inteiest iate.
7he Demand Curve for Money
We have seen that the tiansactions, piecautionaiy, and speculative demands foi money vaiy negatively
with the inteiest iate. Putting those thiee souices of demand togethei, we can diaw a demand cuive foi
money to show how the inteiest iate afects the total quantity of money people hold. The demand
curve for money shows the quantity of money demanded at each inteiest iate, all othei things un-
changed. Such a cuive is shown in Figuie 23.7. An inciease in the inteiest iate ieduces the quantity of
money demanded. A ieduction in the inteiest iate incieases the quantity of money demanded.
The ielationship between inteiest iates and the quantity of money demanded is an
application of the law of demand. If we think of the alteinative to holding money as
holding bonds, then the inteiest iate-oi the difeiential between the inteiest iate in the
bond maiket and the inteiest paid on money deposits-iepiesents the piice of holding
money. As is the case with all goods and seivices, an inciease in piice ieduces the
quantity demanded.
Other Determinants of the Demand for Money
We diaw the demand cuive foi money to show the quantity of money people will hold
at each inteiest iate, all othei deteiminants of money demand unchanged. A change in
those othei deteiminants" will shift the demand foi money. Among the most impoit-
ant vaiiables that can shift the demand foi money aie the level of income and ieal GDP,
the piice level, expectations, tiansfei costs, and piefeiences.
ReaI GDP
A household with an income of $10,000 pei month is likely to demand a laigei quantity
of money than a household with an income of $1,000 pei month. That ielationship sug-
gests that money is a noimal good: as income incieases, people demand moie money at
each inteiest iate, and as income falls, they demand less.
An inciease in ieal GDP incieases incomes thioughout the economy. The demand
foi money in the economy is theiefoie likely to be gieatei when ieal GDP is gieatei.
7he Price LeveI
The highei the piice level, the moie money is iequiied to puichase a given quantity of
goods and seivices. All othei things unchanged, the highei the piice level, the gieatei
the demand foi money.
xpectations
The speculative demand foi money is based on expectations about bond piices. All othei things un-
changed, if people expect bond piices to fall, they will inciease theii demand foi money. If they expect
bond piices to iise, they will ieduce theii demand foi money.
630 PRINCIPLS OI CONOMICS
The expectation that bond piices aie about to change actually causes bond piices to change. If
people expect bond piices to fall, foi example, they will sell theii bonds, exchanging them foi money.
That will shift the supply cuive foi bonds to the iight, thus loweiing theii piice. The impoitance of ex-
pectations in moving maikets can lead to a self-fulflling piophecy.
Expectations about futuie piice levels also afect the demand foi money. The expectation of a high-
ei piice level means that people expect the money they aie holding to fall in value. Given that expecta-
tion, they aie likely to hold less of it in anticipation of a jump in piices.
Expectations about futuie piice levels play a paiticulaily impoitant iole duiing peiiods of hypei-
infation. If piices iise veiy iapidly and people expect them to continue iising, people aie likely to tiy to
ieduce the amount of money they hold, knowing that it will fall in value as it sits in theii wallets oi
theii bank accounts. Towaid the end of the gieat Geiman hypeiinfation of the eaily 1920s, piices weie
doubling as often as thiee times a day. Undei those ciicumstances, people tiied not to hold money even
foi a few minutes-within the space of eight houis money would lose half its value!
7ransfer Costs
Foi a given level of expendituies, ieducing the quantity of money demanded iequiies moie fiequent
tiansfeis between nonmoney and money deposits. As the cost of such tiansfeis iises, some consumeis
will choose to make fewei of them. They will theiefoie inciease the quantity of money they demand. In
geneial, the demand foi money will inciease as it becomes moie expensive to tiansfei between money
and nonmoney accounts. The demand foi money will fall if tiansfei costs decline. In iecent yeais,
tiansfei costs have fallen, leading to a deciease in money demand.
Preferences
Piefeiences also play a iole in deteimining the demand foi money. Some people place a high value on
having a consideiable amount of money on hand. Foi otheis, this may not be impoitant.
Household attitudes towaid iisk aie anothei aspect of piefeiences that afect money demand. As
we have seen, bonds pay highei inteiest iates than money deposits, but holding bonds entails a iisk that
bond piices might fall. Theie is also a chance that the issuei of a bond will default, that is, will not pay
the amount specifed on the bond to bondholdeis; indeed, bond issueis may end up paying nothing at
all. A money deposit, such as a savings deposit, might eain a lowei yield, but it is a safe yield. People's
attitudes about the tiade-of between iisk and yields afect the degiee to which they hold theii wealth as
money. Heightened conceins about iisk in the last half of 2008 led many households to inciease theii
demand foi money.
Figuie 23.8 shows an inciease in the demand foi money. Such an inciease could iesult fiom a
highei ieal GDP, a highei piice level, a change in expectations, an inciease in tiansfei costs, oi a change
in piefeiences.
II GUR 25. 8 An Increase in Money Demand
Ar |rc.e.se |r .e.| C||, t|e p.|ce |eve|, c. t..rs|e. ccsts, |c. ex.np|e, .||| |rc.e.se t|e ,u.rt|ty c| ncrey
Jen.rJeJ .t .ry |rte.est ..te , |rc.e.s|rg t|e Jen.rJ |c. ncrey |.cn |
1
tc |
2
. |e ,u.rt|ty c| ncrey
Jen.rJeJ .t |rte.est ..te .|ses |.cn / tc /. |e .eve.se c| .ry suc| everts .cu|J .eJuce t|e ,u.rt|ty c| ncrey
Jen.rJeJ .t eve.y |rte.est ..te, s|||t|rg t|e Jen.rJ cu.ve tc t|e |e|t.
CHAP7R 25 IINANCIAL MARk7S AND 7H CONOMY 631
suppIy curve of money
u.ve t|.t s|c.s t|e
.e|.t|crs||p bet.eer t|e
,u.rt|ty c| ncrey supp||eJ
.rJ t|e n..ket |rte.est ..te,
.|| ct|e. Jete.n|r.rts c|
supp|y urc|.rgeJ.
money market
|e |rte..ct|cr .ncrg
|rst|tut|crs t|.cug| .||c|
ncrey |s supp||eJ tc
|rJ|v|Ju.|s, |.ns, .rJ ct|e.
|rst|tut|crs t|.t Jen.rJ
ncrey.
money market equiIibrium
|e |rte.est ..te .t .||c| t|e
,u.rt|ty c| ncrey
Jen.rJeJ |s e,u.| tc t|e
,u.rt|ty c| ncrey supp||eJ.
2.2 The Supply of Money
The supply curve of money shows the ielationship between the quantity of money supplied and the
maiket inteiest iate, all othei deteiminants of supply unchanged. We have leained that the Fed,
thiough its open-maiket opeiations, deteimines the total quantity of ieseives in the banking system.
We shall assume that banks inciease the money supply in fxed piopoition to theii ieseives. Because
the quantity of ieseives is deteimined by Fedeial Reseive policy, we diaw the supply cuive of money in
Figuie 23.9 as a veitical line, deteimined by the Fed's monetaiy policies. In diawing the supply cuive of
money as a veitical line, we aie assuming the money supply does not depend on the inteiest iate.
Changing the quantity of ieseives and hence the money supply is an example of monetaiy policy.
II GUR 25. 9 7he SuppIy Curve of Money
\e .ssune t|.t t|e ,u.rt|ty c| ncrey supp||eJ |r t|e eccrcny |s Jete.n|reJ .s . |xeJ nu|t|p|e c| t|e ,u.rt|ty
c| b.rk .ese.ves, .||c| |s Jete.n|reJ by t|e |eJ. |e supp|y cu.ve c| ncrey |s . ve.t|c.| ||re .t t|.t ,u.rt|ty.
2.3 Equilibiium in the Maiket foi Money
The money market is the inteiaction among institutions thiough which money is supplied to indi-
viduals, fims, and othei institutions that demand money. Money market equilibrium occuis at the
inteiest iate at which the quantity of money demanded is equal to the quantity of money supplied. Fig-
uie 23.10 combines demand and supply cuives foi money to illustiate equilibiium in the maiket foi
money. With a stock of money (%), the equilibiium inteiest iate is G.
632 PRINCIPLS OI CONOMICS
II GUR 25. 10 Money Market quiIibrium
|e n..ket |c. ncrey |s |r e,u|||b.|un || t|e ,u.rt|ty c| ncrey Jen.rJeJ |s e,u.| tc t|e ,u.rt|ty c| ncrey
supp||eJ. |e.e, e,u|||b.|un cccu.s .t |rte.est ..te .
2.4 Effects of Changes in the Money Maiket
A shift in money demand oi supply will lead to a change in the equilibiium inteiest iate. Let's look at
the efects of such changes on the economy.
Changes in Money Demand
Suppose that the money maiket is initially in equilibiium at G
1
with supply cuive + and a demand cuive

1
as shown in Panel (a) of Figuie 23.11. Now suppose that theie is a deciease in money demand, all
othei things unchanged. A deciease in money demand could iesult fiom a deciease in the cost of tians-
feiiing between money and nonmoney deposits, fiom a change in expectations, oi fiom a change in
piefeiences.
[2]
Panel (a) shows that the money demand cuive shifts to the left to
2
. We can see that
the inteiest iate will fall to G
2
. To see why the inteiest iate falls, we iecall that if people want to hold less
money, then they will want to hold moie bonds. Thus, Panel (b) shows that the demand foi bonds in-
cieases. The highei piice of bonds means lowei inteiest iates; lowei inteiest iates iestoie equilibiium in
the money maiket.
II GUR 25. 11 A Decrease in the Demand for Money
A Jec.e.se |r t|e Jen.rJ |c. ncrey Jue tc . c|.rge |r t..rs.ct|crs ccsts, p.e|e.erces, c. expect.t|crs, .s s|c.r
|r |.re| (.), .||| be .cccnp.r|eJ by .r |rc.e.se |r t|e Jen.rJ |c. bcrJs .s s|c.r |r |.re| (b), .rJ . |.|| |r t|e
|rte.est ..te. |e |.|| |r t|e |rte.est ..te .||| c.use . .|g|t...J s|||t |r t|e .gg.eg.te Jen.rJ cu.ve |.cn ^|
1
tc
^|
2
, .s s|c.r |r |.re| (c). As . .esu|t, .e.| C|| .rJ t|e p.|ce |eve| .|se.
CHAP7R 25 IINANCIAL MARk7S AND 7H CONOMY 633
Lowei inteiest iates in tuin inciease the quantity of investment. They also stimulate net expoits, as
lowei inteiest iates lead to a lowei exchange iate. The aggiegate demand cuive shifts to the iight as
shown in Panel (c) fiom
1
to
2
. Given the shoit-iun aggiegate supply cuive +*+, the economy
moves to a highei ieal GDP and a highei piice level.
An inciease in money demand due to a change in expectations, piefeiences, oi tiansactions costs
that make people want to hold moie money at each inteiest iate will have the opposite efect. The
money demand cuive will shift to the iight and the demand foi bonds will shift to the left. The iesult-
ing highei inteiest iate will lead to a lowei quantity of investment. Also, highei inteiest iates will lead
to a highei exchange iate and depiess net expoits. Thus, the aggiegate demand cuive will shift to the
left. All othei things unchanged, ieal GDP and the piice level will fall.
Changes in the Money SuppIy
Now suppose the maiket foi money is in equilibiium and the Fed changes the money supply. All othei
things unchanged, how will this change in the money supply afect the equilibiium inteiest iate and ag-
giegate demand, ieal GDP, and the piice level:
Suppose the Fed conducts open-maiket opeiations in which it buys bonds. This is an example of
expansionaiy monetaiy policy. The impact of Fed bond puichases is illustiated in Panel (a) of Figuie
23.12. The Fed's puichase of bonds shifts the demand cuive foi bonds to the iight, iaising bond piices
to (
b
2
. As we leained, when the Fed buys bonds, the supply of money incieases. Panel (b) of Figuie
23.12 shows an economy with a money supply of %, which is in equilibiium at an inteiest iate of G
1
.
Now suppose the bond puichases by the Fed as shown in Panel (a) iesult in an inciease in the money
supply to %; that policy change shifts the supply cuive foi money to the iight to +
2
. At the oiiginal in-
teiest iate G
1
, people do not wish to hold the newly supplied money; they would piefei to hold non-
money assets. To ieestablish equilibiium in the money maiket, the inteiest iate must fall to inciease the
quantity of money demanded. In the economy shown, the inteiest iate must fall to G
2
to inciease the
quantity of money demanded to %.
II GUR 25. 12 An Increase in the Money SuppIy
|e |eJ |rc.e.ses t|e ncrey supp|y by buy|rg bcrJs, |rc.e.s|rg t|e Jen.rJ |c. bcrJs |r |.re| (.) |.cn |
1
tc |
2
.rJ t|e p.|ce c| bcrJs tc |
b
2
. ||s cc..espcrJs tc .r |rc.e.se |r t|e ncrey supp|y tc / |r |.re| (b). |e |rte.est
..te nust |.|| tc
2
tc .c||eve e,u|||b.|un. |e |c.e. |rte.est ..te |e.Js tc .r |rc.e.se |r |rvestnert .rJ ret
expc.ts, .||c| s|||ts t|e .gg.eg.te Jen.rJ cu.ve |.cn ^|
1
tc ^|
2
|r |.re| (c). |e.| C|| .rJ t|e p.|ce |eve| .|se.
The ieduction in inteiest iates iequiied to iestoie equilibiium to the maiket foi money aftei an in-
ciease in the money supply is achieved in the bond maiket. The inciease in bond piices loweis inteiest
iates, which will inciease the quantity of money people demand. Lowei inteiest iates will stimulate in-
vestment and net expoits, via changes in the foieign exchange maiket, and cause the aggiegate demand
cuive to shift to the iight, as shown in Panel (c), fiom
1
to
2
. Given the shoit-iun aggiegate sup-
ply cuive +*+, the economy moves to a highei ieal GDP and a highei piice level.
Open-maiket opeiations in which the Fed sells bonds-that is, a contiactionaiy monetaiy
policy-will have the opposite efect. When the Fed sells bonds, the supply cuive of bonds shifts to the
iight and the piice of bonds falls. The bond sales lead to a ieduction in the money supply, causing the
money supply cuive to shift to the left and iaising the equilibiium inteiest iate. Highei inteiest iates
lead to a shift in the aggiegate demand cuive to the left.
As we have seen in looking at both changes in demand foi and in supply of money, the piocess of
achieving equilibiium in the money maiket woiks in tandem with the achievement of equilibiium in
the bond maiket. The inteiest iate deteimined by money maiket equilibiium is consistent with the in-
teiest iate achieved in the bond maiket.
634 PRINCIPLS OI CONOMICS
k Y 7 A k A W A Y S
< |ecp|e |c|J ncrey |r c.Je. tc buy gccJs .rJ se.v|ces (t..rs.ct|crs Jen.rJ), tc |.ve |t .v.||.b|e |c.
ccrt|rgerc|es (p.ec.ut|cr..y Jen.rJ), .rJ |r c.Je. tc .vc|J pcss|b|e J.cps |r t|e v.|ue c| ct|e. .ssets
suc| .s bcrJs (specu|.t|ve Jen.rJ).
< |e ||g|e. t|e |rte.est ..te, t|e |c.e. t|e ,u.rt|t|es c| ncrey Jen.rJeJ |c. t..rs.ct|crs, |c.
p.ec.ut|cr..y, .rJ |c. specu|.t|ve pu.pcses. |e |c.e. t|e |rte.est ..te, t|e ||g|e. t|e ,u.rt|t|es c|
ncrey Jen.rJeJ |c. t|ese pu.pcses.
< |e Jen.rJ |c. ncrey .||| c|.rge .s . .esu|t c| . c|.rge |r .e.| C||, t|e p.|ce |eve|, t..rs|e. ccsts,
expect.t|crs, c. p.e|e.erces.
< \e .ssune t|.t t|e supp|y c| ncrey |s Jete.n|reJ by t|e |eJ. |e supp|y cu.ve |c. ncrey |s t|us .
ve.t|c.| ||re. Vcrey n..ket e,u|||b.|un cccu.s .t t|e |rte.est ..te .t .||c| t|e ,u.rt|ty c| ncrey
Jen.rJeJ e,u.|s t|e ,u.rt|ty c| ncrey supp||eJ.
< A|| ct|e. t||rgs urc|.rgeJ, . s|||t |r ncrey Jen.rJ c. supp|y .||| |e.J tc . c|.rge |r t|e e,u|||b.|un
|rte.est ..te .rJ t|e.e|c.e tc c|.rges |r t|e |eve| c| .e.| C|| .rJ t|e p.|ce |eve|.
7 R Y I 7 !
|r 2005 t|e |eJ ..s ccrce.reJ .bcut t|e pcss|b|||ty t|.t t|e |r|teJ St.tes ..s ncv|rg |rtc .r |r|.t|cr..y
g.p, .rJ |t .JcpteJ . ccrt..ct|cr..y ncret..y pc||cy .s . .esu|t. |... . |cu.p.re| g..p| s|c.|rg t||s pc||cy
.rJ |ts expecteJ .esu|ts. |r |.re| (.), use t|e ncJe| c| .gg.eg.te Jen.rJ .rJ .gg.eg.te supp|y tc |||ust..te
.r eccrcny .|t| .r |r|.t|cr..y g.p. |r |.re| (b), s|c. |c. t|e |eJs pc||cy .||| .|ect t|e n..ket |c. bcrJs.
|r |.re| (c), s|c. |c. |t .||| .|ect t|e Jen.rJ |c. .rJ supp|y c| ncrey. |r |.re| (J), s|c. |c. |t .||| .|ect
t|e exc|.rge ..te. ||r.||y, .etu.r tc |.re| (.) .rJ |rcc.pc..te t|ese Jeve|cpnerts |rtc ycu. .r.|ys|s c| .g
g.eg.te Jen.rJ .rJ .gg.eg.te supp|y, .rJ s|c. |c. t|e |eJs pc||cy .||| .|ect .e.| C|| .rJ t|e p.|ce |eve|
|r t|e s|c.t .ur.
Case in Point: Money in 7oday's WorId
|e ncJe|s c| t|e ncrey .rJ bcrJ n..kets p.eserteJ |r t||s c|.pte. suggest t|.t t|e |eJ c.r ccrt.c| t|e |r
te.est ..te by Jec|J|rg cr . ncrey supp|y t|.t .cu|J |e.J tc t|e Jes|.eJ e,u|||b.|un |rte.est ..te |r t|e
ncrey n..ket. +et, |eJ pc||cy .rrcurcenerts typ|c.||y |ccus cr .|.t |t ..rts t|e |eJe..| |urJs ..te tc be
.|t| sc.rt .ttert|cr tc t|e ncrey supp|y. \|e.e.s t|.cug|cut t|e 1990s, t|e |eJ .cu|J .rrcurce . t..get
|eJe..| |urJs ..te .rJ .|sc |rJ|c.te .r expecteJ c|.rge |r t|e ncrey supp|y, |r 2000, .|er |eg|s|.t|cr .e,u|.
|rg |t tc Jc sc exp|.eJ, |t .b.rJcreJ t|e p..ct|ce c| sett|rg ncrey supp|y t..gets.
CHAP7R 25 IINANCIAL MARk7S AND 7H CONOMY 635
\|y t|e s|||t` |e |.ctc.s t|.t |.ve n.Je |ccus|rg cr t|e ncrey supp|y .s . pc||cy t..get J||cu|t |c. t|e
p.st 25 ye..s ..e |.st b.rk|rg Je.egu|.t|cr |r t|e 1980s |c||c.eJ by |r.rc|.| |rrcv.t|crs .sscc|.teJ .|t|
tec|rc|cg|c.| c|.rges|r p..t|cu|.. t|e n.tu..t|cr c| e|ect.cr|c p.ynert .rJ t..rs|e. nec|.r
|snst|e.e.|te..
be|c.e t|e 1980s, V1 ..s . |.|.|y .e||.b|e ne.su.e c| t|e ncrey pecp|e |e|J, p.|n..||y |c. t..rs.ct|crs. c buy
t||rgs, cre useJ c.s|, c|ecks ..|tter cr Jen.rJ Jepcs|ts, c. t..ve|e.s c|ecks. |e |eJ ccu|J t|us use .e||.b|e
est|n.tes c| t|e ncrey Jen.rJ cu.ve tc p.eJ|ct .|.t t|e ncrey supp|y .cu|J reeJ tc be |r c.Je. tc b.|rg
.bcut . ce.t.|r |rte.est ..te |r t|e ncrey n..ket.
|eg|s|.t|cr |r t|e e..|y 1980s .||c.eJ |c. ncrey n..ket Jepcs|t .cccurts (VV|As), .||c| ..e essert|.||y
|rte.estbe..|rg s.v|rgs .cccurts cr .||c| c|ecks c.r be ..|tter. VV|As ..e p..t c| V2. S|c.t|y .|te., ct|e.
|c.ns c| p.ynerts |c. t..rs.ct|crs Jeve|cpeJ c. bec.ne nc.e ccnncr. |c. ex.np|e, c.eJ|t .rJ Jeb|t c..J
use |.s nus|.ccneJ (|.cn 10.8 b||||cr |r 1990 tc 30 b||||cr |r 2000), .rJ pecp|e c.r p.y t|e|. c.eJ|t c..J
b|||s, e|ect.cr|c.||y c. .|t| p.pe. c|ecks, |.cn .cccurts t|.t ..e p..t c| e|t|e. V1 c. V2. Arct|e. |rrcv.t|cr c|
t|e |.st 20 ye..s |s t|e .utcn.t|c t..rs|e. se.v|ce (AS) t|.t .||c.s ccrsune.s tc ncve ncrey bet.eer c|eck
|rg .rJ s.v|rgs .cccurts .t .r AV n.c||re, c. cr||re, c. t|.cug| p.e....rgeJ .g.eenerts .|t| t|e|. |r.r
c|.| |rst|tut|crs. \|||e .e t.ke t|ese net|cJs c| p.ynert |c. g..rteJ tcJ.y, t|ey J|J rct ex|st be|c.e 1980
bec.use c| .est.|ct|ve b.rk|rg |eg|s|.t|cr .rJ t|e |.ck c| tec|rc|cg|c.| krc.|c.. |rJeeJ, be|c.e 1980, be|rg
.b|e tc p.y b|||s |.cn .cccurts t|.t e..reJ |rte.est ..s ur|e..J c|.
|u.t|e. b|u..|rg t|e ||res bet.eer V1 .rJ V2 |.s beer t|e Jeve|cpnert .rJ g.c.|rg pcpu|..|ty c| .|.t ..e
c.||eJ .et.|| s.eep p.cg..ns. S|rce 1994, b.rks |.ve beer us|rg .et.||s.eep|rg sc|t...e tc Jyr.n|c.||y .e
c|.ss||y b.|.rces .s e|t|e. c|eck|rg .cccurt b.|.rces (p..t c| V1) c. VV|As (p..t c| V2). |ey Jc t||s tc
.vc|J .ese.ve .e,u|.enerts cr c|eck|rg .cccurts. |e sc|t...e rct cr|y ncves t|e |urJs but .|sc ersu.es
t|.t t|e b.rk Jces rct exceeJ t|e |eg.| ||n|t c| s|x .ec|.ss||c.t|crs |r .ry ncrt|. |r t|e |.st 10 ye..s t|ese .e
t.|| s.eeps .cse |.cn .e.c tc re..|y t|e s|.e c| V1 |tse||!
Suc| c|.rges |r t|e ..ys pecp|e p.y |c. t..rs.ct|crs .rJ b.rks Jc t|e|. bus|ress |.ve |eJ eccrcn|sts tc
t||rk .bcut re. Je|r|t|crs c| ncrey t|.t .cu|J bette. t..ck .|.t |s .ctu.||y useJ |c. t|e pu.pcses be||rJ
t|e ncrey Jen.rJ cu.ve. Ore rct|cr |s c.||eJ VV, .||c| st.rJs |c. ncrey .e.c n.tu.|ty. |e |Je. be||rJ
VV |s t|.t pecp|e c.r e.s||y use .ry Jepcs|ts t|.t Jc rct |.ve spec||eJ n.tu.|ty te.ns tc p.y |c. t..rs.c
t|crs, .s t|ese .cccurts ..e ,u|te ||,u|J, .eg..J|ess c| .|.t c|.ss||c.t|cr c| ncrey t|ey |.|| |rtc. Scne .e
se..c| s|c.s t|.t us|rg VV .||c.s |c. . st.b|e p|ctu.e c| t|e ncrey n..ket. |rt|| nc.e .g.eenert |.s beer
.e.c|eJ, t|cug|, .e s|cu|J expect t|e |eJ tc ccrt|rue tc Jc.rp|.y t|e .c|e c| t|e ncrey supp|y |r |ts pc||cy
Je||be..t|crs .rJ tc ccrt|rue tc .rrcurce |ts |rtert|crs |r te.ns c| t|e |eJe..| |urJs ..te.
!385(* |eJe e|e oJ |o||| |oo. ^ .|o|e /oe, |eoJ |oo|| |o ||e |||| /oe|o, ^eo|e.' |eJeo| |ee.e |o| o| ||coo
|coo|c |eec||.e 29 (||| (oo|e. 2005: 5059
636 PRINCIPLS OI CONOMICS
A N S W R 7 O 7 R Y I 7 ! P R O 8 L M
|r |.re| (.), .|t| t|e .gg.eg.te Jen.rJ cu.ve ^|
1
, s|c.t.ur .gg.eg.te supp|y cu.ve .|^., .rJ |crg.ur .g
g.eg.te supp|y cu.ve ||^., t|e eccrcny |.s .r |r|.t|cr..y g.p c| `
1
`
|
. |e ccrt..ct|cr..y ncret..y
pc||cy ne.rs t|.t t|e |eJ se||s bcrJs. .|g|t...J s|||t c| t|e bcrJ supp|y cu.ve |r |.re| (b), .||c| Je
c.e.ses t|e ncrey supp|y.s s|c.r by . |e|t...J s|||t |r t|e ncrey supp|y cu.ve |r |.re| (c). |r |.re| (b),
.e see t|.t t|e p.|ce c| bcrJs |.||s, .rJ |r |.re| (c) t|.t t|e |rte.est ..te .|ses. A ||g|e. |rte.est ..te .||| .e
Juce t|e ,u.rt|ty c| |rvestnert Jen.rJeJ. |e ||g|e. |rte.est ..te .|sc |e.Js tc . ||g|e. exc|.rge ..te, .s
s|c.r |r |.re| (J), .s t|e Jen.rJ |c. Jc||..s |rc.e.ses .rJ t|e supp|y Jec.e.ses. |e ||g|e. exc|.rge ..te
.||| |e.J tc . Jec.e.se |r ret expc.ts. As . .esu|t c| t|ese c|.rges |r |r.rc|.| n..kets, t|e .gg.eg.te Jen.rJ
cu.ve s|||ts tc t|e |e|t tc ^|
2
|r |.re| (.). || .|| gces .ccc.J|rg tc p|.r (.rJ .e .||| |e..r |r t|e rext c|.pte.
t|.t |t n.y rct!), t|e re. .gg.eg.te Jen.rJ cu.ve .||| |rte.sect .|^. .rJ ||^. .t `
|
.
CHAP7R 25 IINANCIAL MARk7S AND 7H CONOMY 637
3. REVIEW AND PRACTICE
Summary
\e beg.r t||s c|.pte. by |cck|rg .t bcrJ .rJ |c.e|gr exc|.rge n..kets .rJ s|c.|rg |c. e.c| |s .e|.teJ tc
t|e |eve| c| .e.| C|| .rJ t|e p.|ce |eve|. bcrJs .ep.esert t|e cb||g.t|cr c| t|e se||e. tc .ep.y t|e buye. t|e
|.ce v.|ue by t|e n.tu.|ty J.te, t|e|. |rte.est ..te |s Jete.n|reJ by t|e Jen.rJ .rJ supp|y |c. bcrJs. Ar |r
c.e.se |r bcrJ p.|ces ne.rs . J.cp |r |rte.est ..tes. A .eJuct|cr |r bcrJ p.|ces ne.rs |rte.est ..tes |.ve .|s
er. |e p.|ce c| t|e Jc||.. |s Jete.n|reJ |r |c.e|gr exc|.rge n..kets by t|e Jen.rJ .rJ supp|y |c. Jc||..s.
\e t|er s.. |c. t|e ncrey n..ket .c.ks. |e ,u.rt|ty c| ncrey Jen.rJeJ v..|es reg.t|ve|y .|t| t|e |r
te.est ..te. |.ctc.s t|.t c.use t|e Jen.rJ cu.ve |c. ncrey tc s|||t |rc|uJe c|.rges |r .e.| C||, t|e p.|ce
|eve|, expect.t|crs, t|e ccst c| t..rs|e..|rg |urJs bet.eer ncrey .rJ rcrncrey .cccurts, .rJ p.e|e.erces,
espec|.||y p.e|e.erces ccrce.r|rg .|sk. |,u|||b.|un |r t|e n..ket |c. ncrey |s .c||eveJ .t t|e |rte.est ..te .t
.||c| t|e ,u.rt|ty c| ncrey Jen.rJeJ e,u.|s t|e ,u.rt|ty c| ncrey supp||eJ. \e .ssuneJ t|.t t|e supp|y
c| ncrey |s Jete.n|reJ by t|e |eJ. Ar |rc.e.se |r ncrey Jen.rJ ..|ses t|e e,u|||b.|un |rte.est ..te, .rJ .
Jec.e.se |r ncrey Jen.rJ |c.e.s t|e e,u|||b.|un |rte.est ..te. Ar |rc.e.se |r t|e ncrey supp|y |c.e.s t|e
e,u|||b.|un |rte.est ..te, . .eJuct|cr |r t|e ncrey supp|y ..|ses t|e e,u|||b.|un |rte.est ..te.
C O N C P 7 P R O 8 L M S
1. \|.t |.ctc.s n|g|t |rc.e.se t|e Jen.rJ |c. bcrJs` |e supp|y`
2. \|.t .cu|J |.pper tc t|e n..ket |c. bcrJs || . |.. .e.e p.sseJ t|.t set . n|r|nun p.|ce cr bcrJs t|.t
..s .bcve t|e e,u|||b.|un p.|ce`
3. \|er t|e p.|ce c| bcrJs Jec.e.ses, t|e |rte.est ..te .|ses. |xp|.|r.
4. Ore cu.r.||st ..|t|rg .bcut t|e ccnp|ex |rte..ct|crs bet.eer v..|cus n..kets |r t|e eccrcny st.teJ.
\|er t|e gcve.rnert sperJs nc.e t|.r |t t.kes |r t.xes |t nust se|| bcrJs tc |r.rce |ts excess
experJ|tu.es. but se|||rg bcrJs J.|ves |rte.est ..tes Jc.r .rJ t|us st|nu|.tes t|e eccrcny by
erccu..g|rg nc.e |rvestnert .rJ Jec.e.s|rg t|e |c.e|gr exc|.rge ..te, .||c| |e|ps cu. expc.t
|rJust.|es. ..e|u||y .r.|y.e t|e st.tenert. |c ycu .g.ee` \|y c. .|y rct`
5. \|.t Jc ycu p.eJ|ct .||| |.pper tc t|e |c.e|gr exc|.rge ..te || |rte.est ..tes |r t|e |r|teJ St.tes |rc.e.se
J..n.t|c.||y cve. t|e rext ye..` |xp|.|r, us|rg . g..p| c| t|e |c.e|gr exc|.rge n..ket. |c. .cu|J suc| .
c|.rge .|ect .e.| C|| .rJ t|e p.|ce |eve|`
6. Suppcse t|e gcve.rnert .e.e tc |rc.e.se |ts pu.c|.ses, |ssu|rg bcrJs tc |r.rce t|ese pu.c|.ses. |se
ycu. krc.|eJge c| t|e bcrJ .rJ |c.e|gr exc|.rge n..kets tc exp|.|r |c. t||s .cu|J .|ect |rvestnert
.rJ ret expc.ts.
. |c. .cu|J e.c| c| t|e |c||c.|rg .|ect t|e Jen.rJ |c. ncrey`
.. A t.x cr bcrJs |e|J by |rJ|v|Ju.|s
b. A |c.ec.st by t|e |eJ t|.t |rte.est ..tes .||| .|se s|..p|y |r t|e rext ,u..te.
c. A ..ve c| nugg|rgs
J. Ar .rrcurcenert c| .r .g.eenert bet.eer crg.ess .rJ t|e p.es|Jert t|.t, beg|rr|rg |r t|e
rext |sc.| ye.., gcve.rnert sperJ|rg .||| be .eJuceJ by .r .ncurt su|c|ert tc e||n|r.te .||
|utu.e bc..c.|rg
8. Scne |c.|rccne ccurt.|es Jc rct |.ve . bcrJ n..ket. |r suc| ccurt.|es, .|.t subst|tutes |c. ncrey Jc
ycu t||rk pecp|e .cu|J |c|J`
9. |xp|.|r .|.t |s ne.rt by t|e st.tenert t|.t pecp|e ..e |c|J|rg nc.e ncrey t|.r t|ey ..rt tc |c|J.
10. |xp|.|r |c. t|e |eJs s.|e c| gcve.rnert bcrJs s|||ts t|e supp|y cu.ve |c. ncrey.
11. ..ce t|e |np.ct c| . s.|e c| gcve.rnert bcrJs by t|e |eJ cr bcrJ p.|ces, |rte.est ..tes, |rvestnert, ret
expc.ts, .gg.eg.te Jen.rJ, .e.| C||, .rJ t|e p.|ce |eve|.
638 PRINCIPLS OI CONOMICS
N U M R I C A L P R O 8 L M S
1. cnpute t|e ..te c| |rte.est .sscc|.teJ .|t| e.c| c| t|ese bcrJs t|.t n.tu.es |r cre ye...
Iace VaIue SeIIing Price
.. 100 80
b. 100 90
c. 100 95
J. 200 180
e. 200 190
|. 200 195
g. |esc.|be t|e .e|.t|crs||p bet.eer t|e se|||rg p.|ce c| . bcrJ .rJ t|e |rte.est ..te.
2. Suppcse t|.t t|e Jen.rJ .rJ supp|y sc|eJu|es |c. bcrJs t|.t |.ve . |.ce v.|ue c| 100 .rJ . n.tu.|ty
J.te cre ye.. |erce ..e .s |c||c.s.
Price Quantity Demanded Quantity SuppIied
100 0 600
95 100 500
90 200 400
85 300 300
80 400 200
5 500 100
0 600 0
.. |... t|e Jen.rJ .rJ supp|y cu.ves |c. t|ese bcrJs, |rJ t|e e,u|||b.|un p.|ce, .rJ Jete.n|re
t|e |rte.est ..te.
b. |c. suppcse t|e ,u.rt|ty Jen.rJeJ |rc.e.ses by 200 bcrJs .t e.c| p.|ce. |... t|e re.
Jen.rJ cu.ve .rJ |rJ t|e re. e,u|||b.|un p.|ce. \|.t |.s |.ppereJ tc t|e |rte.est ..te`
3. cnpute t|e Jc||.. p.|ce c| . Ce.n.r c.. t|.t se||s |c. 40,000 eu.cs .t e.c| c| t|e |c||c.|rg exc|.rge
..tes.
.. 1 1 eu.c
b. 1 0.8 eu.c
c. 1 0.5 eu.c
4. crs|Je. t|e eu.c.cre c| t|e |u.cpe.r |r|cr .rJ '.p.r. |e Jen.rJ .rJ supp|y cu.ves |c. eu.cs ..e
g|ver by t|e |c||c.|rg t.b|e (p.|ces |c. t|e eu.c ..e g|ver |r '.p.rese yer, ,u.rt|t|es c| eu.cs ..e |r
n||||crs).
Price (in euros) uros Demanded uros SuppIied
+5 0 600
0 100 500
65 200 400
60 300 300
55 400 200
50 500 100
45 600 0
.. |... t|e Jen.rJ .rJ supp|y cu.ves |c. eu.cs .rJ st.te t|e e,u|||b.|un exc|.rge ..te (|r yer)
|c. t|e eu.c. |c. n.ry eu.cs ..e .e,u|.eJ tc pu.c|.se cre yer`
b. Suppcse .r |rc.e.se |r |rte.est ..tes |r t|e |u.cpe.r |r|cr |rc.e.ses t|e Jen.rJ |c. eu.cs by
100 n||||cr .t e.c| p.|ce. At t|e s.ne t|ne, |t .eJuces t|e supp|y by 100 n||||cr .t e.c| p.|ce.
|... t|e re. Jen.rJ .rJ supp|y cu.ves .rJ st.te t|e re. e,u|||b.|un exc|.rge ..te |c. t|e
eu.c. |c. n.ry eu.cs ..e rc. .e,u|.eJ tc pu.c|.se cre yer`
c. |c. .||| t|e evert |r (b) .|ect ret expc.ts |r t|e |u.cpe.r |r|cr`
J. |c. .||| t|e evert |r (b) .|ect .gg.eg.te Jen.rJ |r t|e |u.cpe.r |r|cr`
CHAP7R 25 IINANCIAL MARk7S AND 7H CONOMY 639
e. |c. .||| t|e evert |r (b) .|ect ret expc.ts |r '.p.r`
|. |c. .||| t|e evert |r (b) .|ect .gg.eg.te Jen.rJ |r '.p.r`
5. Suppcse ycu e..r 6,000 pe. ncrt| .rJ sperJ 200 |r e.c| c| t|e ncrt|s 30 J.ys. cnpute ycu.
.ve..ge ,u.rt|ty c| ncrey Jen.rJeJ ||.
.. +cu Jepcs|t ycu. ert|.e e..r|rgs |r ycu. c|eck|rg .cccurt .t t|e beg|rr|rg c| t|e ncrt|.
b. +cu Jepcs|t 2,000 |rtc ycu. c|eck|rg .cccurt cr t|e 1st, 11t|, .rJ 21st J.ys c| t|e ncrt|.
c. +cu Jepcs|t 1,000 |rtc ycu. c|eck|rg .cccurt cr t|e 1st, 6t|, 11t|, 16t|, 21st, .rJ 26t| J.ys c|
t|e ncrt|.
J. |c. .cu|J ycu expect t|e |rte.est ..te tc .|ect ycu. Jec|s|cr tc cpt |c. st..tegy (.), (b), c. (c)`
6. Suppcse t|e ,u.rt|ty Jen.rJeJ c| ncrey .t .r |rte.est ..te c| 5 |s 2 b||||cr pe. J.y, .t .r |rte.est ..te
c| 3 |s 3 b||||cr pe. J.y, .rJ .t .r |rte.est ..te c| 1 |s 4 b||||cr pe. J.y. Suppcse t|e ncrey supp|y |s
3 b||||cr pe. J.y.
.. |... . g..p| c| t|e ncrey n..ket .rJ |rJ t|e e,u|||b.|un |rte.est ..te.
b. Suppcse t|e ,u.rt|ty c| ncrey Jen.rJeJ Jec.e.ses by 1 b||||cr pe. J.y .t e.c| |rte.est ..te.
C..p| t||s s|tu.t|cr .rJ |rJ t|e re. e,u|||b.|un |rte.est ..te. |xp|.|r t|e p.ccess c| .c||ev|rg
t|e re. e,u|||b.|un |r t|e ncrey n..ket.
c. Suppcse |rste.J t|.t t|e ncrey supp|y Jec.e.ses by 1 b||||cr pe. J.y. |xp|.|r t|e p.ccess c|
.c||ev|rg t|e re. e,u|||b.|un |r t|e ncrey n..ket.
. \e krc. t|.t t|e |.S. eccrcny |.ceJ . .ecess|cr..y g.p |r 2008 .rJ t|.t t|e |eJ .espcrJeJ .|t| .r
exp.rs|cr..y ncret..y pc||cy. |.esert t|e .esu|ts c| t|e |eJs .ct|cr |r . |cu.p.re| g..p|. |r |.re| (.),
s|c. t|e |r|t|.| s|tu.t|cr, us|rg t|e ncJe| c| .gg.eg.te Jen.rJ .rJ .gg.eg.te supp|y. |r |.re| (b), s|c.
|c. t|e |eJs pc||cy .|ects t|e bcrJ n..ket .rJ bcrJ p.|ces. |r |.re| (c), s|c. |c. t|e n..ket |c. |.S.
Jc||..s .rJ t|e exc|.rge ..te .||| be .|ecteJ. |r |.re| (J), |rcc.pc..te t|ese Jeve|cpnerts |rtc ycu.
.r.|ys|s c| .gg.eg.te Jen.rJ .rJ .gg.eg.te supp|y, .rJ s|c. |c. t|e |eJs pc||cy .||| .|ect .e.| C||
.rJ t|e p.|ce |eve| |r t|e s|c.t .ur.
640 PRINCIPLS OI CONOMICS
1.
2.
ENDNOTES
crsunpt|cr n.y .|sc be .|ecteJ by c|.rges |r |rte.est ..tes. |c. ex.np|e, || |r
te.est ..tes |.||, ccrsune.s c.r nc.e e.s||y cbt.|r c.eJ|t .rJ t|us ..e nc.e ||ke|y tc
pu.c|.se c..s .rJ ct|e. Ju..b|e gccJs. c s|np|||y, .e |grc.e t||s e|ect.
|r t||s c|.pte. .e ..e |cck|rg cr|y .t c|.rges t|.t c.|g|r.te |r |r.rc|.| n..kets tc
see t|e|. |np.ct cr .gg.eg.te Jen.rJ .rJ .gg.eg.te supp|y. |.rges |r t|e p.|ce
|eve| .rJ |r .e.| C|| .|sc s|||t t|e ncrey Jen.rJ cu.ve, but t|ese c|.rges ..e t|e
.esu|t c| c|.rges |r .gg.eg.te Jen.rJ c. .gg.eg.te supp|y .rJ ..e ccrs|Je.eJ |r
nc.e .Jv.rceJ ccu.ses |r n.c.ceccrcn|cs.
CHAP7R 25 IINANCIAL MARk7S AND 7H CONOMY 641
642 PRINCIPLS OI CONOMICS

You might also like